Skip to main content

Full text of "Mechanics of fluids for practical men [microform] : comprising hydrostatics, descriptive and constructive : the whole illustrated by numerous examples and appropriate diagrams"

See other formats


MECHANICS     OF     FLUIDS 


FOR    PRACTICAL    MEN. 


It  is  observed  of  Archimedes,  by  his  philosophical  biographer  Plutarch,  in  the  Life  of  Marcellus, 
that  "  although  we  might  labour  long  without  success  in  endeavouring  to  demonstrate  from  our  own 
invention,  the  truth  of  his  propositions ;  yet  so  smooth  and  so  direct  is  the  way  by  which  he  leads  us, 
that  when  we  have  once  travelled  it,  we  fancy  that  we  could  readily  have  found  it  without  assistance, 
since  either  his  natural  genius,  or  his  indefatigable  application,  has  given  to  every  thing  that  he 
attempted  the  appearance  of  having  been  performed  with  ease." 


W^fyrf/ 

[.MECHANICS    OF    FLUIDS 

FOR  PRACTICAL  MEN, 


COMPHISING 


HYDROSTATICS, 


DESCRIPTIVE    AND    CONSTRUCTIVE: 


THE  WHOLE  ILLUSTRATED  BY 


NUMEROUS  EXAMPLES  AND  APPROPRIATE  DIAGRAMS. 


BY 


ALEXANDER    JAMIESON,    LL.D.  . 

Ml 
Author  of"  Elements  of  Algebra,"  fyc.  fyc.  $fe. 


LONDON : 
WILLIAM      S.     ORR     AND      CO., 

AMEN    CORNER,  PATERNOSTER  ROW. 

1837. 


or  THE 

UNIVERSITY 

or 


PREFACE. 


MECHANICS  is  the  science  which  inquires  into  the  laws  of 
equilibrium  and  the  motion  of  bodies,  whether  solid  or  fluid. 
The  term  originally  applied  only  to  the  doctrine  of  EQUILIBRIUM, 
and  in  this  volume  it  is  used  in  its  primitive  signification.  The 
adjunct  by  which  this  work  has  been  designated,  is  meant  to 
convey  the  idea  of  a  book  that  is  self-instructing,  and  which, 
in  its  details,  may  furnish  those  who  have  not  had  the  benefit  of 
a  regular  academic  education,  with  expeditious  and  practical 
methods  of  operation,  in  applying  the  principles  of  hydrostatic 
science  to  the  general  and  every-day  business  of  mechanics. 

The  volume  is  therefore  a  manual  of  principles  combining  the 
twofold  properties  of  precept  and  example,  and  exhibiting  in 
a  comprehensive  view  whatever  is  generally  and  particularly 
applicable  to  the  mechanics  of  practical  men.  But  the  same 
construction  will  render  it  available  in  any  course  of  public  or 
private  tuition,  in  which  it  may  be  desired  to  illustrate  by 
examples  those  operations  which,  in  practical  science,  are  go- 
verned by  the  laws  of  fluid  equilibrium,  pressure,  and  support : 
for  it  is  hoped  that  these  laws  have  been  demonstrated  and 
illustrated  with  sufficient  expansion  to  suit  the  progress  of 
modern  discoveries,  and  to  remove  some  part  of  that  uncertainty 
which  has  hitherto  prevailed  in  the  opinions  of  scientific  men. 


VI  PREFACE. 

Should  the  work  achieve  this,  and  contribute  also  to  greater 
precision  than  has  hitherto  been  attained  in  the  arrangements, 
structures,  and  estimates  required  for  works  of  public  or  private 
utility,  one  of  the  objects  for  which  it  was  originally  undertaken 
will  be  crowned  with  success. 

In  the  spring  of  1838,  Gcov  fleXovroc,  another  volume  of 
Mechanics  of  Fluids  will  appear,  comprising  Hydraulics,  which, 
as  the  term  implies,  will  exhibit  the  principles  of  Dynamics  in 
the  joint  operation  of  air  and  water,  hydraulic  architecture,  and 
the  principles  of  construction  of  various  machines  and  engines 
which  belong  to  the  mechanics  of  fluids. 

In  conclusion,  we  vir  tvK\tiq  0avw,  I  beg  leave  to  observe, 
that  as  my  own  avocations  did  not  allow  me  sufficient  leisure 
to  complete  such  an  undertaking  as  these  volumes  are,  in  any 
reasonable  time,  I  have  availed  myself  of  the  services  of 
WM.  TURNBULL,  the  author  of  a  treatise  on  Cast-iron  Beams 
and  Columns ;  and  it  affords  me  unfeigned  satisfaction  to  ac- 
knowledge the  extent  of  his  abilities  and  the  accuracy  of  his 
calculations,  in  subjects  connected  with  the  mechanics  of  fluids. 

Quam,  sit  uterque,  libens  (censebo)  exerceat  artem. 

But  having  elsewhere  alluded  to  the  calculations  and  exam- 
ples which  abound  in  this  volume,  I  shall  only  here  remark, 
that,  were  it  necessary  to  plead  authority  for  such  exercises, 
I  might  quote  NEWTON  himself,  who  has  thus  recorded  his 
opinion : — "  In  scientiis  ediscendis,  prosunt  exempla  magis  quam 
prtecepta."  '' .r  ' 

The  wood-engravings,  so  well  executed  by  Mr.  G.  VASEY, 
are  sufficiently  intelligible,  and  possess  besides  the  lasting 
property  of  being  destroyed  only  with  the  page  in  which  they 
appear. 

A.  J. 

Wyke  House ,  Sion  Hill, 
Isleworth,  Oct.  19,  1837. 


•I* 


CONTENTS. 


CHAPTER  I. 

DEFINITIONS  AND  OBVIOUS  PROPERTIES  OF  WATERY  FLUIDS, 
WITH  THE  PRELIMINARY  ELEMENTARY  PRINCIPLES  OF 
HYDRODYNAMICS,  FOR  ESTIMATING  THE  PRESSURE  OF 
INCOMPRESSIBLE  FLUIDS. 

THE  subject  introduced,  art.  1,  page  1.— Fluid  what,  art.  2,  page  2. — Fluid  presses 
equally  in  all  directions,  art.  3,  page  2.— Lateral  pressure  equal  to  the  perpendicular 
pressure,  art.  3,  page  2. — Fluid  equally  pressed  in  all  directions,  art.  4,  page  2. — 
Fluid  pressure  against  containing  surface,  perpendicular  to  that  surface,  art.  5, 
page  2. — Fluid,  surface  of  horizontal,  or  perpendicular  to  the  direction  of  gravity, 
art.  6,  page  2. — The  common  surface  of  two  fluids  which  do  not  mix,  parallel  to  the 
horizon,  art.  7,  page  2. — The  surfaces  of  fluids  continue  horizontal  when  subjected 
to  the  pressure  of  the  atmosphere,  art.  7,  page  2. — Fluid  particles  at  the  same  depth 
equally  pressed,  art.  8,  page  2. — Fluid  pressure  varies  as  the  perpendicular  depth, 
art.  9,  page  2. — Fluid  pressure  measured  by  the  weight  of  a  column  whose  base  is 
the  surface  pressed,  and  altitude  the  perpendicular  depth  of  the  centre  of  gravity, 
art.  10,  page  3. — Fluid  pressure  identified  with  a  property  of  the  centre  of  gravity, 
art.  11, 12,  and  13,  pages  3  to  7. — Fluid  pressure  assignable,  art.  13,  page  8,  corol. — 
Pressure  of  different  fluids  on  different  plane  surfaces  immersed  in  them,  as  the 
areas  of  the  planes,  the  perpendicular  depths  of  the  centres  of  gravity,  and  the 
specific  gravities  of  the  fluids,  art.  14,  page  8. — Pressure  of  the  same  fluid  on  dif- 
ferent plane  surfaces  immersed  in  it,  as  the  areas  of  the  planes  and  the  perpendicular 
depths  of  the  centres  of  gravity,  art.  15,  page  8. — Plane  parallel  to  fluid's  surface, 
pressure  on  ditto  varies  as  the  perpendicular  depth,  art.  16,  page  8. — Plane  surface 
inclined  to  the  surface  of  the  fluid,  pressure  on  ditto  varies  as  the  perpendicular 
depth  of  its  centre  of  gravity,  art.  17,  page  8. — Planes  of  equal  areas,  whose  centres 
of  gravity  are  at  the  same  perpendicular  depth,  sustain  equal  pressures,  whatever 
may  be  their  form  and  position,  art.  18,  page  8. — The  centre  of  gravity  remaining 
fixed,  the  pressure  upon  a  revolving  plane  is  the  same  at  all  points  of  the  revolu- 
tion, art.  19,  pages  8  and  9. — A  plane  surface  being  immersed  in  two  fluids  of 
different  densities,  if  the  pressures  are  equal,  the  depths  of  the  centres  of  gravity 
vary  inversely  as  the  densities  of  the  fluids,  art.  20,  page  9.— The  same  principle 
demonstrated,  art.  21,  page  9. 


VIII  CONTENTS. 

CHAPTER  II. 

OF  THE  PRESSURE  OF  NON-ELASTIC  FLUIDS  UPON  PHYSICAL 
LINES,  RECTANGULAR  PARALLELOGRAMS  CONSIDERED  AS 
INDEPENDENT  PLANES  IMMERSED  IN  THE  FLUIDS,  AND 
UPON  THE  SIDES  AND  BOTTOMS  OF  CUBICAL  VESSELS, 
WITH  THE  LIMIT  TO  THE  REQUISITE  THICKNESS  OF  FLOOD- 
GATES. 

Physical  line  when  obliquely  immersed,  pressure  on  determined,  art.  22,  pages 
10  and  11.— Practical  rule  for  calculating  ditto,  art.  23,  page  11. — Example  to 
illustrate  ditto,  art.  24,  pages  11  and  12. — The  same  determined  for  a  line  when 
perpendicularly  immersed,  art.  25,  page  12. — The  same  determined  for  a  line 
obliquely  immersed,  when  the  upper  extremity  is  not  in  contact  with  the  surface 
of  the  fluid,  art.  26,  pages  12  and  13. — Pressure  determined  when  the  perpendicular 
depth  of  the  upper  extremity  is  given,  art.  26,  equation  (5),  page  14. — Practical 
rule  for  ditto,  art.  28,  No.  1,  page  14. — The  same  determined  v  hen  the  perpendi- 
cular depth  of  the  lower  extremity  is  given,  art.  27,  equation  (6),  page  14. — Prac- 
tical rule  for  ditto,  art.  28,  No.  2,  page  14. — Example  to  illustrate  ditto,  art.  29, 
pages  14  and  15. — The  pressure  determined  when  the  line  is  perpendicularly  im- 
mersed, art.  30,  page  15. — Two  physical  lines  obliquely  immersed,  with  their  upper 
extremities  in  contact  with  the  surface  of  the  fluid,  to  compare  the  pressures  upon 
them,  under  any  angle  of  inclination,  art.  31,  pages  15  and  16. — The  lines  being 
unequally  inclined,  the  pressures  directly  as  the  squares  of  the  lengths  and  sines  of 
inclination,  art.  31,  inf.  1,  page  16. — When  the  lines  are  equally  inclined,  the  pres- 
sures are  directly  as  the  squares  of  the  lengths,  art.  31,  inf.  2,  page  16. — The  lines 
being  similarly  situated  the  pressures  can  be  compared,  art.  31,  inf.  3,  page  16. — 
The  lines  being  differently  situated  in  the  fluid,  the  pressure  is  determined,  art.  32, 
page  17. — Right  angled  parallelogram  obliquely  immersed,  pressure  determined, 
art.  33,  equation  (7),  page  1 9.— Practical  rule  for  ditto,  art.  34,  No.  1,  page  19. — 
Plane  perpendicularly  immersed,  the  pressure  determined,  art.  33,  equation  (8), 
page  19. — Practical  rule  for  ditto,  art.  34,  No.  2,  page  19. — Example  for  illustration 
of  the  inclined  case,  art.  35,  pages  19  and  20. — Example  for  illustrating  the  per- 
pendicular case,  art.  36,  page  20.  —  The  parallelogram  obliquely  immersed  and 
the  longer  side  coincident  with  the  surface  of  the  fluid,  pressure  determined,  art.  37, 
equation  (9),  page  20. — Perpendicularly  immersed,  pressure  determined,  art.  37, 
equation  (10),  page  20. — Pressures  compared,  art.  37,corols.  1, 2,  and  3,  page  21. — 
Right  angled  parallelogram  immersed  as  before  and  bisected  by  its  diagonal,  the 
pressures  on  the  triangles  determined  and  compared,  art.  38  and  39,  pages  21,  22, 
23,  and  24. — Practical  rules  for  the  pressures,  art.  40,  page  25. — Example  for 
illustration,  art.  41,  page  25. — Centre  of  gravity  of  a  right  angled  triangle  deter- 
mined, art.  42,  pages  26  and  27. — Rectangular  parallelogram  bisected  by  a  line 
parallel  to  the  horizon,  the  pressure  on  the  two  parts  determined  and  compared,  art. 
43,  pages  27, 28,  and  29.— Practical  rules,  art.  44,  Nos.  land  2,  page  29.— Examples 
for  illustration,  art.  45,  pages  29  and  30. — Rectangular  parallelogram  so  divided  by 
a  line  parallel  to  the  horizon,  that  the  pressures  on  the  two  parts  are  equal,  art.  46, 
pages  30,  31,  and  32.— The  same  effected  geometrically,  art.  47,  page  32. — Practical 
rule  for  the  calculation  of  ditto,  art.  48,  page  33. — Example  to  illustrate  ditto,  art. 


CONTENTS.  IX 

49,  page  33. — The  same  determined  when  the  point  of  division  is  estimated  upwards, 
art.  50,  page  33. — Practical  rule  for  calculating  ditto,  art.  51,  page 34.— The  solution 
independent  of  the  breadth  of  the  parallelogram,  art.  .51,  page  34,  corol. — Rectangu- 
lar parallelogram  divided  into  two  parts  sustaining  equal  pressures,  by  a  line  drawn 
parallel  to  the  diagonal,  art.  52,  pages  34,  35,  and  36.— Example  for  illustrating 
ditto,  art.  53,  pages  36  and  37. — Centre  of  gravity  of  the  upper  portion  determined, 
art.  54,  pages  38,  39,  and  40. — Rules  for  the  co-ordinates,  art.  55,  pages  40  and  41. — 
Example  for  illustrating  ditto,  art.  56,  pages  41  and  42.  — Construction  of  the 
figure,  art.  57,  page  42. — The  problem  simplified  when  the  line  of  division  is 
parallel  to  the  diagonal,  art.  57,  page  42. — Example  for  illustrating  ditto,  art.  57, 
pages  42  and  43. — Remarks  on  ditto,  art.  57,  page  43. — The  pressure  not  a  neces- 
sary datum,  compare  the  concluding  remark,  art.  57,  page  43,  with  equations  (20 
and  21),  art.  54,  page  40. — Rectangular  parallelogram  divided  into  two  parts  sustain- 
ing equal  pressures,  by  a  line  drawn  from  one  of  the  upper  angles  to  a  point  in  the 
lower  side,  art.  58,  pages  43  and  44. — Practical  rule  for  ditto,  art.  59,  page  45. — 
Example  for  illustrating  ditto,  art.  60,  page  45. — The  same  determined  when  the 
line  of  division  is  drawn  from  one  of  the  lower  angles  to  a  point  in  the  immersed 
length,  art.  61,  page  46. — Practical  rule  for  ditto,  art.  62,  page  47. — Example  for 
illustrating  ditto,  art.  63,  page  47. — Centre  of  gravity  determined,  art.  64,  pages 
48,49,  and  50. — Remark  on  ditto,  art.  66,  page  51.— Rectangular  parallelogram 
so  divided,  that  the  pressures  011  the  parts  shall  be  in  any  ratio,  art.  67,  68,  and  69, 
pages  51  and  52. — Remarks  on  ditto,  art.  69,  page  53. — Rectangular  parallelogram 
divided  by  lines  drawn  parallel  to  the  horizon,  into  any  number  of  parts  sustaining 
equal  pressures,  art.  70,  pages  53,  54,  55,  56,  and  57.— Practical  rule  for  ditto, 
art.  71,  pages  57  and  58. — Example  for  illustrating  ditto,  art.  72,  page  58. — Con- 
struction of  the  figure,  art.  72,  page  59.  — Requisite  thickness  of  floodgates  deter- 
mined, art.  73,  page  59. — Example  for  illustrating  ditto,  art.  73,  pages  59  and  60.— 
Construction  of  the  figure,  art.  74,  page  60.— Remarks  on  ditto,  art.  74,  page  61. — 
Pressure  on  the  sides  and  bottom  of  a  rectangular  vessel  compared,  art.  75,  pages 
61,  62,  and  63. — Examples  for  illustrating  ditto,  art.  76  and  77,  page  64. 

CHAPTER  III. 

ON  THE  PRESSURE  EXERTED  BY  NON-ELASTIC  FLUIDS  UPON 
PARABOLIC  PLANES  IMMERSED  TN  THOSE  FLUIDS,  WITH 
THE  METHOD  OF  FINDING  THE  CENTRE  OF  GRAVITY  OF 
THE  SPACE  INCLUDED  BETWEEN  ANY  RECTANGULAR  PARAL- 
LELOGRAM AND  ITS  INSCRIBED  PARABOLIC  PLANE. 

Pressure  on  a  parabolic  plane,  compared  with  that  on  its  circumscribing  rectan- 
gular parallelogram,  art.  77,  pages  65  and  66.— Practical  rules  for  ditto,  art.  78, 
page  67. — Example  for  illustrating  ditto,  art.  79,  page  67. —  Centre  of  gravity  of 
the  included  space  determined,  art.  80,  pages  67  and  68. — Remarks  on  ditto,  art.  80, 
page  69. — The  same  things  determined  when  the  base  of  the  parabola  is  coincident 
with  the  surface  of  the  fluid,  art.  81,  pages  69  and  70, — Practical  rules  for  ditto, 
art.  82,  page  71. — Example  for  illustrating  ditto,  art.  83,  page  71.— Remarks  on 
ditto,  art.  83,  page  72.— The  same  things  determined  when  the  base  of  the  parabola 
is  vertical,  and  just  in  contact  with  the  surface  of  the  fluid,  art.  84,  pages  72  and 
VOL.  I.  b 


X  CONTENTS. 

73. — Corollary  to  ditto,  art.  84,  page  74. — Practical  rules  for  ditto,  art.  85,  page 
73. — Example  for  illustrating  ditto,  art.  86,  page  74. — Corollaries  on  ditto,  art.  87, 
page  75. 

Pressure  on  a  semi-parabolic  plane,  compared  with  that  upon  its  circumscribing 
parallelogram,  art.  88,  pages  75,  76,  and  77. — Practical  rule  for  ditto,  art.  89, 
page  77.— Example  for  illustrating  ditto,  art.  90,  page  77.— The  same  determined 
when  the  axis  of  the  semi-parabola  is  horizontal,  art.  91,  pages  78  and  79. — Practi- 
cal rules  for  ditto,  art.  92,  page  79. — Example  for  illustrating  ditto,  art.  93,  page  79. 
The  centre  of  gravity  of  the  included  space  determined,  art.  94,  pages  80,  81,  and 
82.— Construction  of  the  figure,  art.  94,  pages  82  and  83. — Remark  on  ditto,  art. 
94,  page  83. 

Parabolic  plane  with  axis  vertical,  and  vertex  in  contact  with  the  surface  of  the 
fluid,  divided  by  a  horizontal  line  into  two  parts  sustaining  equal  pressures,  art.  95, 
pages  83,  84,  and  85. — Practical  rule  for  ditto,  art.  96,  page  85. — Example  for 
illustrating  ditto,  art.  97,  pages  85  and  86. — The  same  things  determined  generally, 
art.  97,  pages  86  and  87. — Example  for  illustrating  ditto,  art.  97,  page  87. 


CHAPTER  IV. 

OF  THE  PRESSURE  OF  INCOMPRESSIBLE  FLUIDS  ON  CIRCULAR 

PLANES  AND  ON  SPHERES  IMMERSED  IN  THOSE  FLUIDS, 

THE  EXTREMITY  OF  THE  DIAMETER  OF  THE  FIGURE  BEING 
IN  EACH  CASE  IN  EXACT  CONTACT  WITH  THE  SURFACE  OF 

;      THE    FLUID. 

Chord  of  maximum  pressure  in  a  circular  plane  determined,  art.  98,  pages  88 
and  89. — Construction  effected,  corollaries  1  and  2,  page  89. — Practical  rules  for 
calculating  ditto,  art.  99,  page  90. — Example  for  illustrating  ditto,  art.  100,  page 
90. — Pressures  on  two  immersed  spheres  determined  and  compared,  art.  101, 
pages  90  and  91. — A  globe  being  filled  with  fluid,  the  pressure  on  the  interior 
surface  is  three  times  the  weight  of  the  contained  fluid,  art.  101,  corol.  page  92. — 
Example  for  illustrating  ditto,  art.  102,  page  92. — The  exterior  surface  of  a  sphere 
divided  by  a  horizontal  circle  into  two  parts  sustaining  equal  pressures,  art.  103, 
pages  92,  93,  and  94.— Practical  rule  for  ditto,  art.  104,  page  94. — Example  for 
illustrating  ditto,  art.  105,  page  94.— The  solution  generalized,  art.  106,  page  95< — 
Practical  rule  for  the  general  solution,  art.  107,  page  95. 

CHAPTER  V. 

OF  THE  PRESSURE  OF   NON-ELASTIC   OR  INCOMPRESSIBLE    FLUIDS 
AGAINST   THE    INTERIOR    SURFACES    OF    VESSELS     HAVING    THE 
,  FORMS    OF    TETRAHEDRONS,    CYLINDERS,     TRUNCATED    CONES, 
&C. 

Pressure  on  the  base  of  a  tetrahedron  compared  with  the  pressure  on  its  sides, 
art.  108,  pages  96,  97,  and  98. — Comparison  effected,  corol.  page  98. — Pressure  on 
the  base  of  a  cylinder  compared  with  the  pressure  on  its  upright  surface,  art.  109, 


CONTENTS-.  Xi 

pages  99  and  100.  — When  in  any  vessel  whatever,  the  sides  are  vertical  and  the 
base  parallel  to  the  horizon,  the  pressure  on  the  base  is  equal  to  the  weight  of  the 
fluid,  art.  110,  pages  100  and  101. — The  concave  surface  of  a  cylindrical  vessel 
divided  into  annuli,  on  which  the  pressures  are  respectively  equal  to  the  pressure 
on  the  base,  art.  Ill,  pages  101,  102,  103,  and  104. — The  limits  of  possibility 
assigned,  art.  112,  page  104. — The  equations  expressed  in  terms  of  the  radius, 
art.  112,  page  104. — The  equation  generalized,  art.  112,  page  104. — The  practical 
rule  for  any  annulus,  art.  113,  page  105. — Example  for  illustrating  ditto,  art.  114, 
page  105. 

The  pressure  on  the  base  of  a  truncated  cone  compared  with  that  on  its  curved 
surface,  and  also  with  the  weight  of  the  contained  fluid,  art.  115,  pages  106, 107, 
108,  and  109. — The  same  principle  extended  to  the  complete  cone,  base  downwards, 
art.  115,  page  108,  equations  (77  and  80). — Comparison  completed,  art.  115,  corol. 
page  109. 

The  same  things  determined  for  a  truncated  vessel  with  the  sides  diverging 
upwards,  art.  116,  pages  109, 110,  and  111. — For  the  case  of  the  complete  cone  with 
the  base  upwards,  see  equation  (84).— Pressure  on  the  base  compared  with  the 
weight  of  the  contained  fluid,  art.  117,  page  112. — The  pressure  on  the  base  may 
be  greater  or  less  than  the  weight  of  the  contained  fluid  in  any  proportion,  art.  117, 
corol.  1,  page  112. — The  pressure  on  the  bottom  of  a  vessel  not  dependent  on  the 
quantity  of  the  contained  fluid,  art.117,  corol.  2,  page  113.  Any  quantity  of  fluid, 
however  small,  balances  any  other  quantity,  however  great,  art.  117,  corol.  3,  page 
113. — Pressure  on  the  bottom  of  a  cylindrical  vessel  equal  to  any  number  of  times 
the  fluid's  weight,  art.  118,  pages  114  and  115.— Practical  rule  for  ditto,  art.  119, 
page  115. — Example  for  illustrating  ditto,  art.  120,  page  115. — Remarks  on  ditto, 
art.  120,  corollaries  1  and  2,  page  116. — Concluding  remarks  on  the  Hydrostatic 
Press,  page  116. 

CHAPTER  VI. 

THE  THEORY  OF  CONSTRUCTION  AND  SCIENTIFIC*  DESCRIPTION 
OF  SOME  HYDROSTATIC  ENGINES,  VIZ.  THE  HYDROSTATIC 
PRESS,  HYDROSTATIC  BELLOWS,  THE  HYDROSTATIC  WEIGHING 
MACHINE,  AND  EXPERIMENTS  PROVING  THE  QUA  QUA  VERSUS 
PRESSURE  OF  FLUIDS. 

Principle  of  the  Hydrostatic  Press  developed,  art.  121,  pages  117  and  118. — 
First  brought  into  notice  by  Joseph  Bramah,  Esq.,  of  Pimlico,  art.  122,  page  118. — 
Not  a  new  mechanical  power,  ib. — Known  under  the  name  of  Hydrostatic  Paradox 
ib. — Principal  element  by  which  the  power  is  calculated,  art.  123,  page  119. — 
Example  to  illustrate  ditto,  art.  124,  page  119. — General  equation  for  the  pressure 
on  the  piston  of  the  cylinder,  art.  124,  equation  (89),  page  119. — Practical  rule  for 
reducing  ditto,  art.  124,  page  120. — Example  for  illustrating  ditto,  art.  125,  page 
120. — General  equation  for  the  pressure  on  the  piston  of  the  forcing  pump,  art.  125, 
equation  (90),  page  120.— Practical  rule  for  ditto,  art.  125,  page  120. — Example  for 
illustrating  ditto,  art.  126,  page  120. — General  expression  for  the  diameter  of  the 
piston  of  the  cylinder,  equation  (91),  page  121. — Practical  rule  for  ditto,  art.  126,  page 
121. — Example  to  illustrate  ditto,  art.  127,  page  121. — General  expression  for  the 


Xll  CONTENTS. 

diameter  of  piston  in  the  forcing  pump,  art.  127,  equation  (92),  page  121.— Practical 
rule  for  ditto,  art.  127,  page  121. — Safety  valve  introduced,  art.  128,  pages  121  and 
122. — General  expressions  established,  art.  128,  equations  (93  and  94),  page  122. — 
Example  for  illustrating  the  weight  upon  the  safety  valve,  art.  129,  page  122.— 
General  expression  for  the  weight  upon  the  safety  valve,  art.  129,  equation  (95), 
page  122. — Practical  rule  for  ditto,  art.  129,  page  123. — Example  for  illustrating 
ditto,  art.  130,  page  123. — General  expression  for  the  pressure  on  the  piston  of  the 
cylinder,  art.  130,  equation  (96),  page  123.— Practical  rule  for  ditto,  art.  130, 
page  123. — Example  for  illustrating  ditto,  art.  131,  page  123. — General  expression 
for  the  diameter  of  the  safety  valve,  art.  131,  equation  (97),  page  124. — Practical 
rule  for  ditto,  art.  131,  page  124. — Example  for  illustrating  ditto,  art.  132,  page  124. 
— General  expression  for  the  diameter  of  the  cylinder,  art.  132,  equation  (98),  page 
124. — Practical  rule  for  ditto,  ib. — Example  for  illustrating  ditto,  art.  133,  page  124. 
— General  expression  for  the  weight  upon  the  safety  valve,  art.  133,  equation  (99), 
page  125. — Practical  rule  for  ditto,  ib. — Example  for  illustrating  ditto,  art.  134, 
page  125. — General  expression  for  the  pressure  on  the  piston  of  the  forcing  pump, 
art.  134,  equation  (100),  page  125. — Practical  rule  for  ditto,  ib. — Example  for 
illustrating  ditto,  art.  135,  pages  125  and  126. — General  expression  for  the  diameter 
of  the  safety  valve,  art.  135,  equation  (101),  page  126. — Practical  rule  fpr  ditto,  ib. 
— Example  for  illustrating  ditto,  art.  136,  page  126. — General  expression  for  the 
diameter  of  the  forcing  pump,  art.  136,  equation  (102),  page  126. — Practical  rule 
for  ditto,  art.  136,  page  127. — Concluding  remarks,  ib. — The  thickness  of  the  metal 
in  the  cylinder  determined,  art.  137,  pages  127, 128,  129, 130,  and  131.— Example 
for  illustrating  ditto,  art.  138,  page  131. — Appropriate  remarks,  art.  139,  pages  131 
and  132. — A  determinate  and  uniform  value  assigned  to  the  pressure  on  a  square 
inch  of  surface,  art.  140,  page  132. —  Rules  for  the  pressure  in  tons,  and  for  the 
diameter  of  the  cylinder  in  inches,  art.  141,  page  133. — Remarks  on  the  theory, 
art.  142,  page  133. — Examples  for  illustrating  ditto,  ib.  and  page  134.— Remarks, 
art.  143,  page  134. — Observations  on  presses  previously  constructed,  with  rules 
and  examples  for  examining  them,  art.  144  and  145,  pages  134  and  135. — The 
description  of  the  Hydrostatic  Press,  with  its  several  parts  and  appendages,  both 
in  its  complete  and  disjointed  state,  arts.  146, 147,  and  148,  pages  136 — 142. 

The  Hydrostatic  Bellows  and  its  uses  introduced,  art.  149,  page  142. —  Descrip- 
tion and  principles  of  ditto,  art.  150,  pages  142  and  143.— Practical  rule  for 
calculating  the  weight  of  fluid  in  the  tube,  art.  151,  page  144. — Example  for 
illustrating  ditto,  ib. — The  solidity  or  capacity  of  the  fluid  in  the  tube  determined, 
art.  152,  page  144.— Altitude  of  the  tube  determined,  art.  152,  page  145.— The 
solution  given  generally,  art.  153,  page  145. — Practical  rule  for  determining  the 
height  of  the  tube,  art.  154,  page  146. — Illustrative  example  and  remark,  i&.— 
Weight  on  the  moveable  board  determined  in  the  case  of  an  equilibrium,  art.  155, 
page  146. —  Practical  rule  and  illustrative  example  for  ditto,  ib. — Concluding- 
remark,  art.  155,  page  147. — The  diameter  of  the  equilibrating  tube  determined, 
art.  156,  page  147. — Practical  rule  for  ditto,  ib.-—  Example  for  illustrating  ditto, 
art.  156,  page  148. —The  diameter  of  the  Bellows  Board  determined,  art.  157,  page 
148. — Practical  rule  and  illustrative  example  for  ditto,  ib. — When  the  equilibrium 
obtains,  if  more  fluid  be  poured  into  the  tube,  it  will  rise  equally  in  the  tube  and  in 
the  bellows,  art.  158,  page  149. 

The  ascent  of  the  weight  on  the  moving  board  determined,  art.  159,  page  150. — 
Practical  rule  and  example  for  ditto,  art.  159,  page  151. 


CONTENTS.  Xlll 

The  Hydrostatic  Weighing  Machine  introduced,  described,  and  investigated, 
art.  160,  pages  151, 152,  and  153.— Practical  rule  and  example  for  ditto,  art.  161, 
page  154. — Quantity  of  ascent  above  the  first  level  determined,  practical  rule  and 
example  for  ditto,  art.  162,  pages  154  and  155. — Concluding  remarks,  art.  162, 
page  155. 

Quaquaversus  pressure  of  incompressible  fluids  illustrated  by  experiments,  pages 
155-160. 

CHAPTER  VII. 

OF  PRESSURE  AS  IT  UNFOLDS  ITSELF  IN  THE  ACTION  OF  FLUIDS 
OF  VARIABLE  DENSITY,  OR  SUCH  AS  HAVE  THEIR  DENSITIES 
REGULATED  BY  CERTAIN  CONDITIONS  DEPENDENT  UPON  PAR- 
TICULAR LAWS,  WHETHER  EXCITED  BY  MOTION,  BY  MIXTURE, 
OR  BY  CHANGE  OF  TEMPERATURE. 

Preliminary  remarks  on  density,  pages  161,  162,  and  163. — The  alteration  of 
pressure  in  consequence  of  a  change  of  temperature  determined,  art.  163,  pages 
163,  164,  and  165. — The  chord  in  a  semicircular  plane  on  which  the  pressure  is 
a  maximum  determined,  on  the  supposition  that  the  diameter  is  in  contact  with  the 
surface  of  the  fluid,  and  the  density  increasing  as  the  depth,  art.  164,  pages  165, 
166,  and  167. — Practical  rule  for  ditto,  art.  165,  page  167. — Example  for  illus- 
trating ditto,  art.  166,  page  167. — Construction  of  ditto,  art.  167,  pages  167  and  168. 
— The  section  of  a  conical  vessel  parallel  to  the  base,  on  which  the  pressure  is  a 
maximum,  determined,  the  axis  of  the  cone  being  inclined  to  the  horizon  in  a  given 
, angle,  art.  168,  pages  168, 169,  and  170.— Practical  rule  for  ditto,  art.  169,  page  170. 
— Example  for  illustrating  ditto,  art.  170,  page  170. —  Concluding  remarks  on 
compressibility,  art.  171,  page  171.  —  The  diameter  of  a  globe  determined,  in 
ascending  from  the  bottom  to  the  surface  of  the  sea,  on  the  supposition  that  the 
globe  is  condensible  and  elastic,  art.  172,  pages  171  and  172. — Practical  rule  for 
ditto,  art.  173,  page  173. — Example  for  illustrating  ditto,  art.  174,  page  173. — 
Remark  on  ditto,  art.  175,  page  173. — The  depth  of  the  sea  determined,  art.  176, 
pages  173  and  174. — Practical  rule  and  example  for  ditto,  art.  177,  page  174.— 
Pressure  on  the  bottoms  of  vessels  containing  fluids  of  different  densities  deter- 
mined, art.  178,  pages  174  and  175. — Law  of  induction  expounded,  art.  179,  pages 
175  and  176. — Example  for  illustration,  art.  180,  page  176. — Another  example 
under  different  conditions,  art.  181,  pages  176  and  177. — Pressure  on  the  inner 
surface  determined  and  compared  with  that  upon  the  bottom,  art.  182,  pages  177 
and  178. — Altitudes  of  fluids  of  different  densities,  inversely  as  the  specific  gravities, 
art.  183,  pages  178  and  179.— Practical  rule  for  the  altitudes,  art.  184,  page  179. — 
Example  for  illustrating  ditto,  art.  185,  page  180. — A  column  of  mercury  of  2£  feet 
and  a  column  of  water  33.995  equal  to  the  pressure  of  the  atmosphere,  art.  186,  page 
180. — The  principle  of  the  common  or  sucking  pump  dependent  on  this  property, 
ib. — Altitudes  in  the  tubes  equal  when  the  specific  gravities  are  equal,  art.  187, 
page  180. — Surfaces  of  small  pools  near  rivers  on  the  same  level  as  the  rivers, 
art.  188,  page  180. 

Water  may  be  conveyed  from  one  place  to  another,  of  the  same  or  a  less  eleva- 
tion, art.  189,  page  180. — When  the  source  and  point  of  discharge  are  on  the  same 
level  the  water  is  quiescent,  but  when  the  point  of  discharge  is  lower  than  the 


XIV  CONTENTS. 

source,  the  water  is  in  motion,  art.  190,  page  180. — On  this  principle  large  towns 
and  cities  are  supplied  with  water,  art.  190,  page  181. — Edinburgh  thus  supplied, 
ib. — The  principle  of  the  proposition  art.  183  generalized,  art.  191,  page  181. — 
Remark  on  ditto,  ib. — Two  fluids  of  different  densities,  hut  equal  quantities,  being 
poured  into  a  circular  tube  of  uniform  diameter,  their  positions  determined  when 
in  a  state  of  equilibrium,  art.  192,  pages  181, 182, 183,  and  184. — The  principle 
assumed  to  be  similar  to  that  of  Mr.  Barclay's  Hydrostatic  Quadrant,  art.193,  page 
184. — Practical  rule  for  ditto,  ib. — Example  illustrative  of  ditto,  art.  194,  pages 
184  and  185.— The  actual  position  of  the  fluids  exhibited  by  a  construction,  art.  195, 
pages  185  and  186. — Mercury  preferable  to  water  for  the  tubes  of  philosophical 
instruments,  art.  196,  page  186. — Other  fluids  convenient  for  the  purpose,  art.  197, 
page  186. — The  result  of  the  investigation  only  applicable  to  mercury  and  water, 
arts.  196  and  198,  page  186. — The  same  rendered  general,  art.  199,  pages  186  and 
187. — The  general  equation  deducible  from  the  particular  one,  art.  199,  page  187. — 
Practical  rule  for  the  general  case,  art.  200,  pages  187  and  188. — Example  for 
illustrating  ditto,  art.  201,  page  188.— The  positions  of  the  fluid  different  in  the 
two  cases,  art.  202,  page  188. — Inquiry  into  the  changes  produced  on  the  general 
formulae,  under  certain  conditions,  art.  202,  page  189.— Practical  rule  for  reducing 
the  resulting  equation,  art.  203,  page  189.— Concluding  remark,  ib. 


CHAPTER  VIII. 

OF  THE  PRESSURE  OF  NON-ELASTIC  FLUIDS  UPON  DYKES,  EM- 
BANKMENTS, OR  OTHER  OBSTACLES  WHICH  CONFINE  THE 
FLUIDS,  WHETHER  THE  OPPOSING  MASS  SLOPE,  BE  PERPENDI- 
CULAR, OR  CURVED,  AND  THE  STRUCTURE  ITSELF  BE  MASONRY, 
OR  OF  LOOSE  MATERIALS,  HAVING  THE  SIDES  ONLY  FACED 
WITH  STONE. 

Introductory  remarks,  art.  204,  page  190. — The  manner  explained  in  which  a 
dyke,  mound  of  earth,  or  any  other  obstacle,  may  yield  to  fluid  pressure,  art.  205, 
page  191. — Remarks  on  ditto,  ib. — General  investigation,  art.  206—210,  pages 
191 — 197. — Reasons  adduced  for  not  giving  practical  rules  on  this  subject,  art.  210, 
page  197. — Example  for  illustrating  the  reduction  of  the  final  equations,  art.  211, 
pages  197  and  198. — The  same  illustrated  when  the  effect  produced  by  the  vertical 
pressure  of  the  fluid  is  omitted,  art.  212,  pages  198  and  199. — It  is  safer  to  calculate 
by  omitting  the  vertical  pressure,  art.  212,  corol.  page  199. — The  breadth  at  the 
foundation  determined  when  the  slopes  are  equal,  art.  213,  page  199. — Example 
for  illustrating  ditto,  art.  214,  pages  199  and  200. — The  same  determined  when  the 
side  on  which  the  fluid  presses  is  vertical,  art.  215,  page  200. — More  expensive  to 
erect  a  dyke  of  this  form,  than  if  both  sides  slope,  art.  215,  corol.  page  201. — The 
same  determined  when  the  side  of  the  dyke  opposite  to  that  on  which  the  fluid 
presses  is  vertical,  art.  216,  page  201. — The  stability  in  this  case  less  than  when 
the  vertical  side  is  towards  the  water,  art.  216,  pages  201  and  202. 

Thickness  of  the  dyke  determined  when  both  its  sides  are  perpendicular  to  the 
plane  of  the  horizon,  art.  217,  page  202. — Practical  rule  and  example  for  this  case, 
art.  218,  page  202. — Thickness  of  the  wall  determined  when  its  section  is  in  the 


CONTENTS.  XV 

form  of  a  right  angled  triangle,  and  first  when  the  fluid  presses  on  the  perpendi- 
cular, art.  219,  page  203. — Practical  rule  and  example  for  this  case,  art.  220,  page 
203. — The  same  determined  when  the  fluid  presses  on  the  hypothenuse,  art.  221, 
pages  203  and  204. — Practical  rule  and  example  for  this  case,  art.  222,  page  204. — 
The  construction  exhibited,  art.  223,  pages  204  and  205. — Concluding  remarks, 
art.  223,  page  205. — The  thickness  of  the  dyke  determined  when  it  yields  to  the 
pressure  by  sliding  on  its  base,  art.  224,  pages  205  and  206. — The  resistances  of 
adhesion  and  friction  compared  with  the  weight  of  the  dyke,  art.  225,  page  206. — 
Example  for  illustrating  the  resulting  formula,  art.  226,  page  207. — The  breadth  of 
the  dyke  at  the  top  determined,  art.  226,  page  207. — The  breadth  of  the  dyke  at 
the  bottom  determined,  when  the  side  on  which  the  water  presses  is  perpendicular 
to  the  horizon,  art.  227,  page  207. — The  same  determined  when  the  opposite  side  is 
perpendicular,  ib. — The  same  when  both  sides  are  perpendicular,  ib. — The  same 
determined  when  the  section  of  the  wall  is  triangular,  having  the  side  next  the 
fluid  perpendicular,  and  the  remote  slope  equal  to  the  breadth,  art.  228,  page  207. 
— The  same  takes  place  when  the  remote  side  is  perpendicular,  and  the  slope  on 
which  the  water  presses  is  equal  to  the  breadth,  art.  228,  page  208. — Concluding 
remarks,  ib. 

The  dimensions  of  the  dyke  determined  when  it  is  constructed  of  loose  materials, 
art.  229,  pages  208,  209,  and  210. — Example  for  illustration  of  ditto,  art.  230,  page 
210. — The  conditions  necessary  for  preventing  the  dyke  from  sliding  on  its  base 
determined,  art.  231,  page  210.  When  the  water  presses  against  the  vertical  side 
of  a  wall,  the  curve  bounding  the  other  side  so  that  the  strength  may  be  every 
where  proportional  to  the  pressure,  is  a  cubic  parabola,  ib.  Introductory  remarks 
to  Chapter  IX,  art.  232,  page  211. 

CHAPTER  IX. 

OF    FLOATATION,     AND    THE     DETERMINATION     OF    THE     SPECIFIC 
GRAVITIES    OF    BODIES    IMMERSED    IN    FLUIDS. 

The  buoyant  force  equivalent  to  the  weight  of  the  displaced  fluid,  art.  233,  pages 
212,  213,  and  214. — The  pressure  downwards  equal  to  the  buoyant  force,  art.  233, 
corol.  page  214.— The  height  determined  to  which  a  fluid  rises  in  a  cylindrical 
vessel,  in  consequence  of  the  immersion  of  a  given  sphere  of  less  specific  gravity, 
art.  234,  pages  214  and  215. — Practical  rule  for  ditto,  art.  235,  page  216. — Example 
for  illustrating  ditto,  art.  236,  page  216. — The  same  determined  when  the  sphere 
and  the  fluid  are  of  equal  specific  gravity,  art.  237,  page  216; — Practical  rule  for 
ditto,  ib.  Example  for  illustrating  ditto,  art.  238,  pages  216  and  217.  The  height 
determined  to  which  the  fluid  rises  in  a  paraboloidal  vessel,  in  consequence  of  the 
immersion  of  a  sphere  of  less  specific  gravity,  art.  239,  pages  217,  218,  and  219. 

Practical  rule  for  ditto,  art.  240,  page  219. — Example  for  illustrating  ditto,  art. 
241,  pages  219  and  220. — The  same  determined  when  the  sphere  and  the  fluid  are 
of  equal  specific  gravity,  art.  242,  page  220.— Practical  rule  for  ditto,  art.  243, 
page  220.— Example  for  illustrating  ditto,  art.  244,  pages  220  and  221. — Remarks 
on  ditto,  art.  244,  corol.  page  221. — A  homogeneous  body  placed  in  a  fluid  of  the 
same  density  as  itself,  remains  at  rest  in  all  places  and  in  all  positions,  art.  245, 
pages  221  and  222. — The  upward  pressure  against  the  base  of  a  body  immersed  in 


XVI  CONTENTS. 

a  fluid,  is  equal  to  the  weight  of  the  displaced  and  superincumbent  fluid,  art.  246, 
page  222. — The  difference  between  the  downward  and  upward  pressures,  is  equal 
to  the  difference  between  the  weight  of  the  solid  and  an  equal  bulk  of  the  fluid, 
art.  247,  page  222.— Absolute  and  relative  gravity,  what,  art.  248,  pages  222  and 
223. — By  absolute  gravity,  fluids  gravitate  in  their  proper  places,  by  relative 
gravity  they  do  not,  ib. — A  heavy  heterogeneous  body  descending  in  a  fluid,  has  the 
centre  of  gravity  preceding  the  centre  of  magnitude,  art.  249,  page  223. — The 
reason  of  this,  ib. — Concluding  remarks,  art.  250,  page  223. — The  force  with  which 
a  body  ascends  or  descends  in  a  fluid  of  greater  or  less  specific  gravity  than  itself, 
is  equal  to  the  difference  between  its  own  weight  and  that  of  the  fluid,  art.  251, 
pages  223  and  224. — The  force  of  ascent  and  descent  is  nothing  when  the  specific 
gravities  are  equal,  art.  251,  page  224. — When  a  body  is  suspended  or  immersed  in 
a  fluid,  it  loses  the  weight  of  an  equal  bulk  of  the  fluid  in  which  it  is  placed,  art. 
252,  page  224. — When  a  body  is  suspended  or  immersed  in  a  fluid  of  equal  or 
different  density,  it  loses  the  whole  or  a  part  of  its  weight,  and  the  fluid  gains  the 
weight  which  the  body  loses,  art.  253,  page  225. — Bodies  of  equal  magnitudes 
placed  in  the  same  fluid  lose  equal  weights,  and  unequal  bodies  lose  weights  pro- 
portional to  their  magnitudes,  art.  254,  page  225. — The  same  body  placed  in 
different  fluids,  loses  weights  proportional  to  the  specific  gravities  of  the  fluids, 
art.  255,  page  225. — When  bodies  of  unequal  magnitudes  are  in  equilibrio  in  any 
fluid,  they  lose  their  equilibrium  when  transferred  to  any  other  fluid,  art.  256,  page 
225. — When  a  body  rises  or  falls  in  a  fluid  of  different  density,  the  accelerating 
force,  what,  art.  257,  page  225. — When  the  solid  is  heavier  than  the  fluid  it 
descends,  when  lighter  it  ascends,  art.  258,  pages  225  and  226;  hence  relative 
gravity  and  relative  levity,  ib. — Practical  rule  for  the  general  formula,  art.  259, 
page  226.— Example  for  illustrating  ditto,  art.  260,  page  226. — The  distance  of 
descent  determined,  when  the  pressive  and  buoyant  forces  are  equal,  art.  261,  pages 
226,  227,  and  228.— Practical  rule  for  calculating  ditto,  art.  262,  page  228. — 
Example  for  illustrating  ditto,  art  263,  page  2'J8. 


CHAPTER  X. 

OF    THE    SPECIFIC    GRAVITIES    OF    FLUIDS,    AND    THE     THEORY     OF 
WEIGHING  SOLID  BODIES  BY  MEANS  OF  NON-ELASTIC  FLUIDS. 

Introductory  remarks  on  specific  gravity,  end  the  principles  or  criteria  of  com- 
parison, page  229.— The  weight  lost  by  a  body,  is  to  the  whole  weight,  as  the 
specific  gravity  of  the  fluid  is  to  that  of  the  solid,  art.  264,  pages  229  and  230. — 
The  weight  which  the  body  loses  in  the  fluid  is  not  annihilated,  but  only  sustained, 
art.  264,  page  230. — The  weight  of  a  body  after  immersion  determinable,  art.  265, 
page  231. — Practical  rule  for  ditto,  ib. — Example  for  illustration,  art.  266,  page  231. 
- — The  weights  which  a  body  loses  in  different  fluids,  are  as  the  specific  gravities  of 
the  fluids,  art.  266,  corol.  page  231. — The  real  weight  of  a  body  determinable  by 
having  its  weight  in  water  and  in  air,  art.  267,  pages  231  and  232. — Practical  rule 

for  ditto,  art.  268,  page  232. — Example  for  illustration,  art.  269,  page  232. The 

specific  gravity  of  a  body  determinable  from  its  weight,  as  indicated  in  water  and 
in  air,  art,  270,  pages  233  and  234. — Practical  rule  for  ditto,  art.  271,  page  234.— 


CONTENTS.  XVII 

Example  for  illustration,  art.  272,  page  234. — The  magnitude  of  a  globular  body 
determinate  from  its  real  weight  and  density,  art.  273,  pages  234  and  235. — The 
same  determinable  from  its  weight  in  air  and  in  water,  art.  274,  pages  235  and  236. 
— Practical  rules  for  ditto,  art.  275,  page  236. — Examples  for  illustration,  arts.  276 
and  277,  pages  236  and  237. — Different  bodies  of  equal  weights  immersed  in  the 
same  fluid,  lose  weights  that  are  inversely  as  their  densities,  or  directly  as  their 
magnitudes,  art.  278,  page  237. — The  difference  between  the  absolute  weight  of  a 
body  and  its  weight  in  any  fluid,  is  equal  to  the  weight  of  an  equal  bulk  of  the 
fluid,  art.  279,  page  237. — If  two  solid  bodies  of  different  magnitudes  indicate  equal 
weights  in  the  same  fluid,  the  larger  body  preponderates  in  a  rarer  medium,  art.  280, 
page  237. — Under  the  same  circumstances  the  lesser  preponderates  in  a  denser 
medium,  art.  281,  page  238. — If  solid  bodies  when  placed  in  the  same  fluid  sustain 
equal  diminutions  of  weight,  their  magnitudes  are  equal,  art.  282,  page  238.  To 
determine  the  equipoising  weight,  when  two  bodies  equally  heavy  in  air,  are  placed 
in  a  fluid  of  greater  density,  the  densities  of  the  bodies  being  different,  art.  283, 
pages  238  and  239. — Practical  rule  for  ditto,  art.  284,  page  239. — Necessary  remark, 
ib. — Example  for  illustration,  art.  285,  page  239. — The  ratio  of  the  quantities  of 
matter  determinable,  when  two  bodies  of  different  specific  gravities  equiponderate  in 
a  fluid,  art.  286,  page  240. — Example  for  illustration,  art.  287,  pages  240  and  241. — 
Problem  respecting  the  equiponderating  of  the  cone  and  its  circumscribing  cylinder, 
art.  288,  pages  241,  242,  and  243. — Practical  rule  for  ditto,  art.  289,  page  243. — 
Example  for  illustration,  art.  290,  page  244. — To  compare  the  specific  gravities  of 
a  solid  body,  with  that  of  the  fluid  in  which  it  is  immersed,  art.  291,  page  245. — 
Example  for  illustration,  art.  292,  page  245. — To  compare  the  specific  gravities  of 
two  solid  bodies,  when  weighed  in  vacuo  and  in  a  fluid  of  given  density,  art.  293, 
pages  245  and  246. — Example  for  illustration,  art.  294,  page  247. — The  specific 
gravities  of  different  fluids  compared,  by  weighing  a  body  of  a  given  density,  art. 
295,  pages  247  and  248. — Example  for  illustration,  art.  296,  pages  248  and  249. — 
Concluding  remarks,  ib. — The  specific  gravity  of  a  solid  body  determined  by  weigh- 
ing it  in  air  and  in  water,  art.  297,  page  249. — The  principle  of  solution  explained, 
ib. — The  practical  rule  for  ditto,  art.  298,  page  250. — Example  for  illustration,  art. 
299,  page  250.— Concluding  remarks,  art.  300,  page  250. — The  specific  gravity  of 
a  solid  body  determined  from  that  of  the  fluid  in  which  it  is  weighed,  art.  301, 
page  251. — Practical  rule  for  ditto,  art.  302,  page  252. — Example  for  illustration, 
art.  303,  page  252. — The  specific  gravity  of  a  solid  body  determined,  by  immersing 
it  in  a  vessel  of  water  of  which  the  weight  is  known,  art.  304,  pages  252,  253,  and 
254. — Practical  rule  for  ditto,  art.  305,  page  254. — Example  for  illustration,  art.  306, 
page  254. — Concluding  remarks  on  the  value  of  the  opal,  ib. 

CHAPTER  XL 

OF    THE  EQUILIBRIUM  OF  FLOATATION. 

Opening  remarks  on  floatation,  page  255. — The  centre  of  gravity  of  the  whole 
body  and  that  of  the  immersed  part  occur  in  the  same  vertical  line,  art.  307,  pages 
255  and  256. — The  weight  of  the  floating  body  and  that  of  the  displaced  fluid  are 
equal  to  one  another,  art.  307,  page  256. — Corresponding  remarks,  ib.  — Homogeneous 
plane  figures  divided  symmetrically  remain  in  equilibrio  with  their  axes  vertical, 
art.  308,  page  257. — Homogeneous  solid  bodies  generated  by  the  revolution  of  a 


XV1I1  CONTENTS. 

curve,  when  placed  upon  a  fluid  of  greater  specific  gravity,  maintain  their  equili- 
brium with  the  axis  vertical,  art.  309,  page  257. — Showing  under  what  conditions 
a  prismatic  hody  remains  in  equilibrio,  art.  310,  page  257. — The  magnitude  of  a 
floating  body  to  that  of  the  immersed  part,  as  the  specific  gravity  of  the  fluid  is  to 
that  of  the  solid,  art.  311,  pages  257  and  258. — Examples  for  illustration,  arts.  312 
and  313,  pages  258  and  259. — General  determination,  art.  314,  page  259. — Practical 
rule  for  ditto,  ib. — Example  for  illustration,  art.  315,  page  260. — General  investiga- 
tion, ib. — Practical  rule  and  example  for  ditto,  pages  260  and  261. — Inferences 
arising  therefrom,  arts.  316,  317,  318,  319,  and  320,  page  261. — Demonstration  of  a 
general  principle,  art.  321,  page  262. — To  determine  how  far  a  paraboloidal  solid 
will  sink  in  a  fluid,  art.  322,  pages  262  and  263. — Practical  rule  for  ditto,  art.  323, 
page  264. — Example  for  illustration,  art.  324,  page  264. — The  same  determined 
when  the  vertex  of  the  figure  is  downwards,  ib. — Practical  rule  for  ditto,  art.  325, 
page  265. — The  elevation  or  depression  of  a  body  determined,  when  the  equilibrium 
is  disturbed  by  the  subtraction  or  addition  of  a  certain  given  weight,  art.  326,  pages 
265  and  266. — The  same  thing  determined  independently  of  fluxions,  art.  327,  pages 
267  and  268. — Practical  rule  for  ditto,  art.  328,  page  268. — Example  for  illustration, 
art.  329,  page  268. — The  same  determined  for  a  body  in  the  shape  of  a  paraboloid, 
art.  329,  pages  26&  and  269. — Remark  on  the  resulting  equations,  art.  330,  page  269. 
— The  descent  occasioned  by  adding  a  weight  determined,  art.  330,  pages  269  and 
270. — Practical  rule  for  ditto,  art.  331,  page  270. — Example  for  illustration,  art. 
332,  page  270. — The  ascent  determined  when  a  given  weight  is  subtracted,  art.  333, 
page  270. — Practical  rule  for  ditto,  art.  334,  page  271. — Concluding  remark,  art. 
335,  page  271. — The  weight  determined  which  is  necessary  to  sink  a  body  to  the 
same  level  with  the  fluid,  art.  336,  pages  271  and  272.— Practical  rule  for  ditto, 
art.  337,  page  272.— Example  for  illustration,  art.  338,  page  272.— A  solid  body 
being  immersed  in  two  fluids  which  do  not  mix,  floats  in  equilibrio  between  them', 
when  the  weights  of  the  displaced  fluids  are  together  equal  to  the  weight  of  the 
body,  art.  339,  pages  272,  273,  and  274.— The  quantity  of  each  fluid  displaced  by  a 
cubical  body  determined,  art.  340,  pages  274  and  275. — Practical  rule  for  ditto, 
art.  341,  page  275. — Example  for  illustration,  art.  342,  page  275. — Another  example 
under  different  conditions,  art.  376,  page  343. — The  specific  gravity  of  a  solid  body 
determined,  so  that  any  part  of  it  may  be  immersed  in  the  lighter  of  two  unmixable 
fluids,  art.  344,  page  276. — Practical  rule  for  ditto,  art.  345,  page  277.— Example  for 
illustration,  art.  346,  page  277.— The  same  determined  when  equal  parts  of  the  body 
are  immersed  in  the  lighter  and  heavier  fluids,  art.  347,  page  277. — Practical  rule 
for  ditto,  art.  347,  page  278. — Example  for  illustration,  art.  348,  page  278.— The 
same  determined  when  the  lighter  fluid  vanishes,  art.  349,  page  278. — A  very 
curious  property  unfolded,  art.  349,'  page  279. — The  ratio  of  the  immersed  parts 
determined,  when  the  body  floats  on  water,  in  air,  and  in  a  vacuum,  art.  350, 
pages  279  and  280. —  Practical  rule  for  ditto,  art.  351,  page  280. — Example 
for  illustration,  art.  352,  page  280. — Remark  and  rule  for  determining  the  same 
otherwise,  art.  353,  page  281. — The  Hydrometer  or  Aerometer  introduced,  art. 
354,  page  281. — Improvements  on  ditto  by  various  writers,  art.  354,  page  282. 
— Description  of  the  instrument  according  to  Deparcieux,  art.  355,  pages  282 
and  283. — The  specific  gravity  of  a  fluid  determined  by  the  aerometer,  art.  356, 
pages  283  and  284. — Practical  rule  for  ditto,  art.  357,  page  284. —Example  for 
illustration,  art.  358,  page  284. — The  immersed  quantity  of  the  stem  determined, 
art.  359,  page  285. — Practical  rule  for  ditto,  360,  page  285. — Example  for  illustra- 


CONTENTS.  XIX 

tion,  art.  361,  page  285. — Transformation  of  the  equations,  art.  362,  pages  285  and 
286. — Concluding  remarks,  art.  362,  page  286. — The  change  in  the  aerometer  cor- 
responding to  any  small  variation  in  the  density  of  the  fluid,  art.  363,  page  286. — 
Practical  rule  for  ditto,  art.  364,  pages  286  and  287. — Example  for  illustration,  art. 
365,  page  287. — Remarks,  ib. — The  sensibility  of  the  instrument  increased  by 
decreasing  the  diameter  of  the  stem,  and  otherwise,  art.  366,  pages  287  and  288. — • 
Practical  rule  for  ditto,  art.  367,  page  288. — Example  for  illustration  and  remarks 
on  ditto,  art.  368,  pages  288  and  289. — Hydrostatic  Balance  introduced  and  de- 
scribed, art.  369,  pages  289  and  290.— The  specific  gravity  of  a  solid  body  determined 
by  the  balance,  that  of  distilled  water  being  given,  art.  370,  page  291. — Practical 
rule  for  ditto,  art.  371,  page  291. — Example  for  illustration,  art.  372,  page  291 . 


CHAPTER  XII. 

OF    THE    POSITIONS    OF    EQUILIBRIUM. 

The  positions  of  equilibrium  of  a  triangular  prism  determined,  art.  373,  pages 
292,  293,  and  294. — Remarks  on  the  form  of  the  resulting  equation,  art.  374,  pages 
294  and  295. — Geometrical  construction  effected  by  means  of  two  hyperbolas,  art. 
374,  page  295. — Example  for  illustrating  the  reduction  of  the  formula,  art.  375, 
pages  295  and  296. — Construction  and  calculation  of  the  figure  and  its  parts,  arts. 
376,  377,  and  378,  pages  296,  297,  and  298. — More  positions  of  equilibrium  exhi- 
bited by  the  equation,  and  remarks  on  ditto,  art.  379,  page  299. — The  positions 
determined  when  the  section  of  the  prism  is  isosceles,  art.  380,  pages  299  and  300. 
— Practical  rule  for  ditto,  art.  381,  page  300. — Example  for  illustration,  art.  382, 
pages  300  and  301.— Other  positions  determined,  art.  383,  page  301. — Remarks  on 
the  resulting  equation,  art.  383,  page  302. — Practical  rules  for  reducing  ditto,  art. 
384,  pages  302  and  303. — Limits  to  the  positions  of  equilibrium,  art.  385,  page  303. 
— Example  for  illustration,  art.  386,  pages  303  and  304. — Remarks  on  ditto,  art. 
386,  page  304. — The  positions  delineated,  art.  387,  page  304. — The  same  demon- 
strated, pages  305  and  306.— Remarks,  art.  388,  page  306. — The  positions  of  equi- 
librium dependent  upon  the  specific  gravity,  art.  389,  pages  307  and  308. — The 
true  positions  delineated  according  to  the  conditions  of  the  problem,  art.  390,  page 
308. — The  positions  of  equilibrium  determined  when  the  section  of  the  body  is  in 
the  form  of  an  equilateral  triangle,  art.  391,  pages  308  and  309. — The  positions 
calculated,  art.  392,  pages  309  and  310. — The  positions  delineated  according  to  this 
determination,  page  310. — Other  positions  determined,  art.  393,  pages  310  and  311. 
—The  same  positions  delineated  by  construction,  art.  393,  page  31 1. — The  positions 
of  equilibrium  determined  for  a  triangular  prism,  when  two  of  its  edges  fall  below 
the  plane  of  floatation,  art.  394,  pages  311,  312,  313,  314,  and  315.-— The  method  of 
reducing  the  general  equation  described,  art.  395,  page  315. — Example  to  show  the 
method  of  reduction,  art.  396,  pages  315,  316,  and  317. — The  position  delineated 
and  verified,  art.  397,  page  317. — Another  condition  of  equilibrium,  what,  art.  398, 
page  317.— The  same  verified,  page  318. — Other  positions  of  equilibrium  determin- 
able,  the  determination  left  for  exercise  to  the  reader,  art.  399,  page  318. — The 
same  determined  when  the  section  of  the  figure  is  in  the  form  of  an  isosceles 
triangle,  art.  400,  page  318. — The  equation  reduced,  art.  401,  page  319. — Practical 
rule  for  reducing  the  equation,  art.  402,  page  319.— Example  for  illustration,  art. 


or  THE       A 
UNIVERSITY  3 


XX  CONTENTS. 

403,  pages  319  and  320.  The  position  delineated  by  construction,  art.  404,  page 
320. — Other  two  positions  of  equilibrium  determinable,  art.  405,  pages  320,  321, 
and  322. 

Limits  to  the  specific  gravity  defined  when  the  body  floats  with  two  of  its  angles 
immersed,  art.  405,  page  321. — Expression  for  the  arithmetical  mean  between  the 
limits,  equation  (247),  page  321. — Expressions  for  the  extant  sides  of  the  triangular 
section,  (equations  248  and  249),  pages  321  and  322. — Remarks  on  ditto,  art.  405, 
page  322. — Example  for  illustration,  art.  406,  page  322. — Construction  indicating 
the  positions,  art.  407,  page  322. — The  construction  verified  by  calculation,  art.  407, 
page  323,  and  art.  408,  page  324. — The  positions  of  equilibrium  determined  when 
the  triangular  section  is  equilateral,  art.  409,  pages  324  and  325. — Remarks  on 
ditto,  art.  409,  page  325. —  Example  for  illustration,  art.  410,  page  325. — Construc- 
tion indicating  the  positions,  art.  410,  page  325. — The  same  verified  by  calculation, 
art.  410,  page  326. — Two  other  positions  assignable,  ib. — The  limits  of  the  specific 
gravity  defined,  art.  411,  pages  326  and  327. — Positions  of  equilibrium  determined, 
art.  411,  page  327. — Construction  indicating  the  positions,  art.  412,  page  327. — 
The  same  verified  by  calculation,  arts.  412  and  413,  pages  327  and  328. — Con- 
cluding remarks  on  ditto,  art.  414,  pages  328  and  329. — The  positions  of  equilibrium 
determined  for  a  rectangular  prism  with  one  of  its  edges  immersed,  art.  415,  pages 
329  to  331. — General  remarks  on  ditto,  art.  416,  page  332.— The  positions  deter- 
mined when  the  ends  of  the  prism  are  squares,  art.  417,  page  332. — General 
inference  from  ditto,  art.  417,  page  333. — Practical  rule  for  ditto,  art.  418,  page 
333. — Example  for  illustration,  art.  419,  page  333. — Construction  indicating  the 
position,  arts.  419  and  420,  page  333. — Condition  establishing  the  equilibrium, 
art.  421,  page  334.— The  maximum  limit  of  the  specific  gravity  defined,  art.  422, 
page  334. — Two  other  positions  determined,  as  they  depend  upon  the  limit  of  the 
specific  gravity,  art.  423,  page  334. — Conditions  that  limit  the  specific  gravity, 
art.  424,  page  334. — Positions  determined  according  to  the  limits,  and  repre- 
sented by  diagrams,  art.  425,  pages  334  and  335. — Positions  determined  from 
the  arithmetical  mean  of  the  limits,  and  represented  by  diagrams,  pages  335  and 
336. — The  positions  determined  when  three  edges  of  the  prism  are  immersed, 
art.  426,  page  336. — The  positions  determined  when  two  edges  are  immersed,  art. 
427,  page  336. — Remarks  on  the  superior  difficulty  of  this  case,  page  337.— Pre- 
paratory construction,  ib. — The  conditions  necessary  for  a  state  of  equilibrium, 
page  338. — The  first  condition  established,  equation  (265),  page  339. — The  second 
condition  determinable  by  a  separate  construction,  page  339. — The  investigation 
pursued,  pages  340,  341,  and  342. — The  positions  determined,  art.  429,  pages  342, 
343,  and  344. — The  same  determined  when  the  section  is  a  square,  art.  430,  page 
344.— When  the  specific  gravity  of  the  prism  is  half  the  specific  gravity  of  the 
fluid,  the  body  sinks  to  half  its  depth,  art.  431,  page  344. — This  position  repre- 
sented by  a  diagram, ib. — The  positions  which  the  body  would  assume  in  the  course 
of  revolution  assigned,  page  345. — Two  other  positions  assignable,  dependent  on 
the  limits  of  the  specific  gravity,  page  345,  equations  (273  and  274). — Construction 
indicating  the  position  according  to  the  limits,  art.  432,  page  346. — The  maximum 
limit  of  the  specific  gravity  determined,  art.  433,  page  346. — Remarks,  accompanied 
by  diagrams  to  indicate  the  positions,  art.  434,  pages  346  and  347. — The  arith- 
metical mean  between  the  limits  assigned,  art.  435,  page  347. — The  positions 
assigned  numerically,  art.  436,  pages  347  and  348. — The  same  indicated  by  con- 
struction, art.  436,  page  348. — The  reverse  positions  exhibited,  art.  437,  page  348. 


CONTENTS.  XXI 

— The  construction  verified  by  calculation,  art.  438,  page  349. — The  positions  of 
equilibrium  determined  for  a  solid,  of  which  the  transverse  section  is  in  the  form 
of  the  common  parabola,  art.  439,  pages  349-352. — Practical  rule  for  ditto,  art.  440, 
page  352.—  Example  to  illustrate  ditto,  art.  441,  page  352.— Construction  indicating 
the  position  of  equilibrium,  with  its  verification,  art.  442,  page  353. — The  same 
determinable  for  the  figure  in  the  inclined  position,  art.  443,  pages  353  to  356. — 
Example  to  illustrate  ditto,  art.  444,  pages  356  and  357. — The  positions  indicated 
by  construction,  page  357. — The  construction  verified,  page  358. 


CHAPTER  XIII. 

OF   THE    STABILITY    OF    FLOATING   BODIES    AND    OF    SHIPS. 

Stability  of  floating  bodies,  the  subject  introduced,  art.  446,  page  359. — General 
remarks  concerning  ditto,  arts.  447,  448,  and  449,  pages  359  to  362. — Definitions, 
art.  450,  pages  362  and  363. — A  floating  body  displaces  a  quantity  of  fluid  equal 
to  its  own  weight,  and  in  consequence,  the  specific  gravity  of  the  fluid  is  to  that  of 
the  solid,  as  the  whole  magnitude  is  to  the  part  immersed,  art.  451,  pages  363  and 
364. — A  floating  body  is  impelled  downwards  by  its  own  weight,  and  upwards  bj 
the  pressure  of  the  fluid,  and  these  forces  act  in  vertical  lines  passing  through  the 
centre  of  effort  and  the  centre  of  buoyancy,  art.  452,  page  364. — When  these  lines 
do  not  coincide,  the  body  revolves  upon  an  axis  of  motion,  ib. — If  a  floating  body  be 
deflected  from  the  upright  position,  the  stability  is  proportional  to  the  length  of  the 
equilibrating  lever,  or  to  the  horizontal  distance  between  the  vertical  lines,  passing 
through  the  centre  of  effort  and  the  centre  of  buoyancy,  art.  453,  page  364. — When 
this  distance  vanishes,  the  equilibrium  is  that  of  indifference,  ib. — When  it  falls  on 
the  same  side  of  the  centre  of  effort  as  the  depressed  parts  of  the  solid,  the  equili- 
brium is  that  of  stability,  art.  453,  pages  364  and  365.— When  it  falls  on  the  same 
side  as  the  elevated  parts,  the  equilibrium  is  that  of  instability,  art.  453,  page  365. 
— Concluding  remarks,  ib. — A  general  proposition  belonging  to  the  centre  of  gra- 
vity, art.  454,  page  365. — General  and  subsidiary  remarks,  ib. 

The  stability  of  floating  bodies  determined,  art.  455,  pages  366  to  370. — The 
manner  of  generalizing  the  result  explained,  together  with  the  constituent  elements 
of  the  equation,  art.  456,  page  370. — Example  for  illustration,  art.  457,  pages  370 
and  371. — The  steps  of  calculation  illustrated  by  reference  to  a  diagram,  arts.  458, 
459,  460,  461, 462,  463,  and  464,  pages  371  to  375. — Concluding  observations,  art. 
464,  page  375. 

The  principle  of  stability  applied  to  ships,  art.  465,  page  376. — The  conditions 
of  the  data  explained,  art.  466,  pages  376  and  377. — The  longer  and  shorter  axes, 
what,  art.  467,  page  377. — In  what  respects  a  ship  may  be  considered  as  a  regular 
body,  art.  468,  page  377. — The  principal  section  of  the  water,  what,  art.  468,  page 
377. — The  circumstances  and  conditions  of  calculation  explained  ,by  reference  to  a 
diagram,  arts.  469,  470,  471,  and  472,  pages  378  to  382.— A  numerical  example  for 
illustration,  art.  473,  page  382. — Table  of  the  measured  ordinates,  page  383. — 
Construction  of  the  example  explained,  pages  384  to  387.— Construction  continued, 
art.  474,  pages  387  to  389. — The  contents  of  the  displaced  volumes,  how  obtained, 
art.  475,  page  389. — Approximating  rules  for  calculating  the  areas  and  solidities, 
art.  476,  page  390. — The  construction  completed,  art. 477,  page 390. — The  practical 
delineation  of  the  vertical  and  horizontal  planes,  arts.  478  and  479,  pages  391  and 


XX11  CONTENTS. 

392. — The  manner  of  calculation  described,  art.  480,  page  393. — The  manner  of 
calculation  exemplified,  art.  481, pages  39S to  396. — Calculation  continued,  arts.  482 
and  483,  pages  396  and  397. — Concluding  remarks,  art.  483,  pages  397  and  398. — 
Reflections  suggested  by  the  importance  of  the  subject,  art.  484,  page  398. 

The  principles  of  stability  as  referred  to  steam  ships  considered,  art.  485,  pages 
398  and  399. — Reference  to  Tredgold's  work  on  the  Steam  Engine,  art.  485,  page 
399. — Tredgold's  method  of  simplifying  the  investigation,  art.  486,  page  399. — His 
subdivision  of  the  inquiry,  ib. — The  steps  of  investigation  not  necessary  to  be 
retraced,  art.  487,  page  399. — The  expression  for  stability  when  the  ordinates  are 
parallel  to  the  depth,  equation  (290),  art.  488,  page  399. — Remarks  deduced  from 
the  form  of  the  equation,  art.  489,  page  400. — The  expression  for  stability  in  the 
ease  of  a  triangular  section,  equation  (291),  art.  489,  page  400. — The  practical  rule 
for  reducing  the  equation,  art.  490,  page  400. — Example  for  illustrating  ditto,  art. 
491,  pages  400  and  401.— The  expression  for  stability  in  the  case  when  the  trans- 
verse section  is  in  the  form  of  a  common  parabola,  equation  (292),  art.  492,  page 
401. — Remarks  on  its  fitness  for  the  purpose  of  steam  navigation,  as  contrasted  with 
the  triangular  section  preceding,  art.  492,  page  401. — Comparison  of  the  results, 
art.  493,  pages  401  and  402.— Method  of  identifying  the  rule  in  the  two  cases,  art. 
494,  page  402. — When  the  transverse  section  is  in  the  form  of  a  cubic  parabola,  the 
stability  is  determined  by  equation  (293),  art.  495,  page  402. — Remarks  on  the 
superior  form  in  this  case,  art.  496,  page  402. — Practical  rule  for  ditto,  ib. — 
Example  for  illustration,  art.  497,  pages  402  and  403. — The  stability  determined  for 
a  parabolic  section  of  the  5th  order,  equation  (294),  art.  498,  page  403. — Remarks 
on  ditto,  ib.— General  remarks  in  reference  to  the  limiting  forms  of  steam  ships, 
art.  499,  page  403. — When  the  ordinates  of  the  transverse  section  are  parallel  to  the 
breadth,  the  stability  is  expressed  by  equation  (295),  art.  500,  page  404. — When 
the  transverse  section  is  in  the  form  of  a  triangle,  the  stability  is  expressed  by 
equation  (296),  art.  501,  page  404. — When  the  transverse  section  is  in  the  form 
of  the  common  parabola,  the  stability  is  expressed  by  equation  (297),  art.  502, 
page  404.— When  the  transverse  section  is  in  the  form  of  a  cubic  parabola,  the 
stability  is  expressed  by  equation  (298),  art.  503,  page  404. — This  form  superior 
for  stability,  ib.— When  the  transverse  section  is  formed  by  a  parabola  of  the  5th 
order,  the  stability  is  expressed  by  equation  (299),  art.  504,  page  405. — Concluding 
and  general  remarks,  i&.— The  stability  the  same  at  every  section  throughout  the 
length,  under  what  conditions  this  will  obtain,  art.  505,  page  405. 


CHAPTER  XIV. 

OF    THE    CENTRE    OF    PRESSURE. 

The  centre  of  pressure,  subject  introduced,  definition  and  preliminary  remarks, 
art.  506,  page  406. — The  centre  of  pressure  determined  for  a  plane  surface,  art.  507, 
pages  406,  407,  408,  and  409. — Formulae  of  condition,  equation  (302),  page  409. — 
The  centre  of  pressure  determined  for  a  physical  line,  art.  508,  pages  409  and  410. 
— Practical  rule  for  ditto,  art.  509,  page  410. — Example  for  illustration,  art.  510, 
pages  410  and  411. — The  same  determined  when  the  upper  extremity  of  the  line  is 
in  contact  with  the  surface  of  the  fluid,  art.  511,  page  411. — The  same  principle 
applicable  to  a  rectangle,  art.  511,  page  411.— The  centre  of  pressure  determined 


CONTENTS.  XXlll 

for  a  rectangle  when  its  upper  side  is  in  contact  with  the  surface  of  the  fluid,  art. 
512,  pages  411  and  412.— The  centre  of  pressure  determined  when  the  plane  is  a 
square,  art.  513,  page  412. — The  centre  of  pressure  determined  for  a  semi-paraholic 
plane,  art.  514,  pages  412  and  413. — Example  to  illustrate  ditto,  art.  515,  pages  413 
and  414. — The  same  determined  for  one  side  of  a  vessel  in  the  form  of  a  parallelo- 
pipedon,  art.  517,  pages  414,  415,  and  416. — Practical  rule  and  example  for  ditto, 
page  416. — The  balancing  force,  the  centre  of  pressure,  and  the  direction  of  its 
motion  determined  for  the  side  of  a  tetrahedron,  arts.  518,  519,  520,  and  521,  pages 
417  to  419.— Example  to  illustrate  ditto,  art.  522,  page  420. 


CHAPTER  XV. 

OF    CAPILLARY    ATTRACTION    AND    THE    COHESION    OF    FLUIDS. 

Capillary  attraction  and  the  cohesion  of  fluids,  the  subject  introduced,  with  its 
object,  and  remarks  thereon,  art.  525,  pages  421  and  422. — Definition  of  capillary 
attraction,  art.  526,  page  422. — Attraction  of  cohesion  between  glass  and  water, 
art.  527,  pages  422  and  423. — The  particles  of  a  fluid  attract  each  other,  art.  523, 
page  423. — The  particles  of  mercury  have  an  intense  attraction  for  each  other,  art. 
529,  page  423. — The  attraction  between  glass  and  water  only  sensible  at  insensible 
distances,  art.  530,  pages  423  and  424. — The  manner  described  in  which  the 
attractive  influence  is  exerted,  art.  531,  page  424. — The  forms  assumed  by  the 
summit  of  the  elevated  columns  described,  art.  532,  pages  424  and  425. — The  force 
of  attraction  proposed  to  be  determined,  art.  533,  page  425. — The  parts  by  which 
the  fluid  in  the  tube  is  attracted,  described,  art.  534,  page  426. — The  same  as 
respects  the  lower  portion  of  the  tube  described,  art.  535,  page  426. — The  fluid 
attracted  by  the  glass  only,  art.  535,  pages  426  and  427, — A  negative  force  in 
the  opposite  direction,  art.  536,  page  427. — The  force  of  attraction  determined, 
and  expressed  by  equation  (310),  art.  536,  page  427. — The  conditions  of  the 
rising  and  falling  column  explained,  ib. — The  expression  for  the  force  of  at- 
traction generalized,  equation  (311),  page  428. — The  height  to  which  the  fluid 
ascends  in  the  tube  determined,  art.  538,  pages  428  and  429. — The  mean  altitude 
expressed,  equation  (312),  page  429. — The  general  expression  modified,  arts.  539 
and  540,  pages  429  and  430. — The  constant  determined,  equation  (315),  page  430. 
Practical  rule  for  the  mean  height,  art.  541,  page  430. — Example  for  illustration, 
art.  542,  page  430. — The  radius  of  the  tube  determinable,  art.  543,  pages  430  and 
431. — The  method  illustrated  by  a  numerical  example,  art.  544,  page  431. — The 
height  to  which  the  fluid  rises  between  two  parallel  plates  determined,  arts.  545, 
546,  and  547,  pages  431,  432,  and  433. — The  practical  rule  for  ditto,  art.  547,  page 
433. — Example  for  illustration,  art.  548,  page  433. — The  distance  between  the 
plates  determinable,  art.  549,  page  434. — Concluding  remarks,  art.  550,  page  434. 
— When  two  smooth  plates  of  glass  meet  in  an  angle,  to  determine  the  nature  of  the 
curve  which  the  fluid  forms  between  them,  art.  551  and  552,  pages  435  and  436. — 
The  curve  determined  to  be  the  common  or  Apollonian  hyperbola,  art.  553,  page 
436. — Concluding  remarks,  ib. — Two  bodies  that  can  be  wetted  with  water,  when 
placed  an  inch  asunder  do  not  approximate  or  recede ;  but  if  placed  a  few  lines 
apart,  they  approximate  with  an  accelerated  velocity,  art.  554,  page  436. — Two 
bodies  that  cannot  be  wetted,  when  placed  a  few  lines  apart,  approximate  with  an 
accelerated  velocity,  art.  555,  page  437. — When  one  body  can  be  wetted  and  the  other 


XXIV  CONTENTS. 


not,  they  recede  from  each  other,  art.  556,  page  437. — Remarks  on  the  above,  ib. — 
General  laws  deducible  from  ditto,  arts.  557,  558,  and  559,  pages  437  and  438. — 
Hydrostatic  pressure  exemplified  in  springs  and  Artesian  wells,  arts.  560  to  569, 
pages  439  to  442. 


CHAPTER  XVI. 

MISCELLANEOUS     HYDROSTATIC     QUESTIONS,     WITH     THEIR 
SOLUTIONS. 


Miscellaneous  hydrostatic  questions,  with  their  solutions,  arts.  570  to  581,  pages 
443  to  448. 


On  a  careful  revision  of  the  sheets,  the  following  are  the  principal  errors  that 
have  been  discovered. 

/  Art.  9,  page  2,  for  varies  in  its  perpendicular  depth,  read  varies  as  its  perpendi- 
cular depth. 

J     Page  97,  line  17  from  top,  for  p=  the  pressure  upon  one  of  the  sides,  readp—  the 
pressure  upon  the  three  containing  sides. 

1     Page  183,  line  10  from  bottom,/or  cos.x  +  sin.20  sin.0,  read  cos.x  -J-  sin.20  sin.x. 
_  ^    Page  215,  line  16  from  top,  dele  as. 

Page  230,  line  6  from  top,/or  the  specific  of  the  solid,  read  the  specific  gravity  of 
the  solid ;  and  in  line  10,/or  respective  gravities,  read  respective  specific  gravities. 
Page  237,  line  6  from  top,/ar 

d=\3/  Ill21l£l=3.93l3,  or  nearly  4  inches,    read 
V     .5236X7 

d=  4s/       W     —\t/  -      14X16 =0.3939  feet,  or  4.7267  inches. 

V     .5236S      V      .5236X7X1000 

line  18  from  bottom,  far 


d=.3/  (13.9975-12)16 


.=    /(13.9975-12)X  ™=4^67  incheg. 
.5      V       .52361-001262.5 


. 
.o2o6(s-s)X62.5      V       .5236(1-0012)62. 

Art.  354,  page  282,/or  Levi  read  Lovi. 

Page  432,  line  13  from  bottom,/or  %bdzTr,  read$d*ir. 

-  line  11  from  bottom,/or  ±bd*—%bd*ir=$bd*(2—  TT),  read  ±bdz(l—  |TT). 

-  line  8  from  bottom,y!w  m=&<#i-f£fed2(2—  ?r),  read  m=bdh+±bdz(\—\ir). 

-  line  3  from  bottom,/or  {bdh-\-ibd\2—  ir)}8g,  read  {bdh+ibd*(l-$7r)}3g. 

-  line  1  from  bottom,/or  d{fc+id(l—  ^TT)},  read  d{h+ld(l—  ITT)}- 


INTRODUCTION. 


THE  analytical  table  of  contents,  which  the  reader  must  have 
perused,  will  have  shown  him  that  this  volume  is  not  a  selection 
of  shreds  and  patches,  garbled  from  contemporary  authorities : 
but  a  systematic  treatise  on  Hydrostatic  Science,  containing  a 
vast  mass  of  valuable  and  interesting  facts,  combining  indeed 
almost  all  that  needs  to  be  known  on  the  equilibrium  of  fluids. 
But  for  the  convenience  of  reference,  these  Mechanics  of  Fluids 
are  distributed  into  a  series  of  chapters,  whose  titles  indicate 
the  several  topics  that  receive  mathematical  demonstration. 
The  first  of  these  contains,  besides  a  few  brief  but  necessary 
definitions,  the  fundamental  proposition  upon  which  all  the  pro- 
blems that  are  drawn  up  in  Elementary  Hydrostatics  are  in 
reality  founded. 

The  principle  established  in  the  general  proposition,  enables 
the  reader  to  proceed  in  the  second  chapter  with  the  pressure  of 
incompressible  fluids  upon  physical  lines,  rectangular  parallelo- 
grams considered  as  independent  planes  immersed  in  the  fluid, 
and  to  determine  the  position  of  the  centre  of  gravity  of  the 
various  rectangular  figures  which  the  successive  problems  em- 
brace, together  with  the  pressures  of  fluids  upon  the  sides  and 
bottoms  of  cubical  vessels,  with  the  limits  which  theory  assigns 
to  the  requisite  thickness  of  flood-gates. 

One  distinguishing  characteristic  in  this  inquiry  is,  that  every 
problem  is  accompanied  by  a  practical  example  ;  and  in  order  that 
nothing  be  omitted  which  could  render  the  subject  intelligible  to 

VOL,  i.  c 


XXVI  INTRODUCTION. 

the  general  reader,  the  most  important  formulae  of  a  practical  and 
general  nature  have  been  thrown  into  rules,  in  words  at  length, 
whereby  all  the  arithmetical  operations  required  in  the  solution 
of  the  examples,  can  be  performed  without  any  reference  to  the 
algebraical  investigation,  which  is  the  surest  way  of  uniting 
precept  with  example. 

After  the  same  method,  the  third  chapter  treats  of  the  pres- 
sure exerted  by  non-elastic  fluids  upon  parabolic  planes  immersed 
in  these  fluids,  with  the  method  of  finding  the  centre  of  gravity 
of  the  space  included  between  any  rectangular  parallelogram 
and  its  inscribed  parabolic  plane.  This  is  a  valuable  proposi- 
tion in  the  practice  of  bridge-building,  and  it  is  very  satisfactory 
to  find  in  prosecuting  one  branch  of  science,  the  means  of  ac- 
complishing another ;  to  discover  in  a  subject  purely  hydrostatic, 
a  method  by  which  to  find  the  position  of  the  centre  of  gravity 
of  the  arch,  with  all  its  balancing  materials,  and  consequently 
many  important  particulars  respecting  the  weight  and  mechani- 
cal thrust,  with  the  thickness  of  the  piers  necessary  to  resist  the 
drift  or  shoot  of  a  given  arch,  independently  of  the  aid  afforded 
by  the  other  arches.  The  method  laid  down  in  Problem 
XII.  for  this  purpose  is  presumed  to  be  new;  at  any  rate  we 
have  not  seen  it  noticed  by  any  previous  writer  on  Mechanics. 
But  its  development  belongs  to  Hydraulic  Architecture  ;  the 
principle  here  established  being  all  that  is  required  in  Hy- 
drostatics. 

Chapter  IV.  introduces  the  reader  to  the  pressure  of  non- 
elastic  fluids  on  circular  planes,  and  spheres  immersed  in  those 
fluids  as  independent  bodies,  the  extremity  of  the  diameter  of 
the  figure  being  in  each  case  coincident  with  the  surface  of  the 
fluid.  These  problems  could  easily  have  been  extended  to 
examples  of  elliptical  planes  and  solids,  but  the  investigation 
would  not  embrace  any  practical  result :  and  where  that  is  un- 
attainable, this  work  presumes  not  to  advance. 

The  Fifth  Chapter,  in  which  are  classed  the  tetrahedron, 
cylinder,  conical  frustum,  and  indeed  the  frustum  of  any  other 
regular  pyramid,  completes  this  branch  of  fluid  pressure ;  but 
the  investigation  is  directed  altogether  to  the  pressure  of  the 
fluid  upon  the  internal  surfaces  of  the  vessels  under  considera- 
tion. Indeed  this  was  part  of  the  inquiry  when  the  sphere  was 


INTRODUCTION. 

treated  of  in  the  fourth  chapter;  but  in  the  fifth,  the  subject  is 
purely  practical,  and  involves  some  of  the  most  important  prin- 
ciples in  the  whole  range  of  Hydrodynamics.  The  reader  now 
enters  upon  that  remarkable  and  important  principle, 

That  any  quantity  of  fluid,  however  small,  may  be 
made  to  balance  or  hold  in  equilibrio  any  other  quantity, 
however  great ; 

and  is  enabled  thence  to  investigate  the  theory  and  expound 
the  construction  of  those  mechanical  contrivances  known  as 
Bramah's  hydrostatic  press,  the  hydrostatic  bellows,  and  weigh- 
ing machine,  which  are  all  methods  of  balancing  different 
intensities  of  force,  by  applying  the  simple  power  of  non- elastic 
fluids  to  parts  of  an  apparatus  moving  with  different  velocities  : 
and  this  is  all  the  mechanical  powers  can  effect. 

The  Sixth  Chapter,  which  treats  of  these  hydrostatic  engines, 
their  theory  of  construction  and  scientific  description,  com- 
mences with  a  distinct  proposition  ;  the  first  having  proved  suf- 
ficient to  resolve  every  problem  connected  with  fluid  pressure 
upon  rectilinear  and  curvilinear  figures  considered  as  independent 
planes  immersed  in  the  fluids,  together  with  the  pressure  of 
fluids  upon  the  interior  surfaces  of  vessels  containing  the  fluids 
and  belonging  to  the  class  of  regular  bodies, — the  second  pro- 
position, which  the  reader  now  enters  upon,  involves  the  prin- 
ciple whereon  depend  the  construction  and  appliancy  of  the 
hydrostatic  press,  an  engine  very  generally  employed  in  practical 
mechanics,  and  which  should  therefore  be  scientifically  as  it  is 
practically  known.  But  the  same  proposition  extends  to  the 
investigation  of  the  hydrostatic  bellows,  and  furnishes  the  prin- 
ciple of  a  particular  machine  by  which  goods  may  be  weighed 
as  by  the  common  balance.  It  may  thence  be  inferred,  that  as 
yet,  science  has  but  stepped  on  the  threshold  of  fluids  that  are 
heavy  and  liquid.  How  far  this  distinguishing  property,  the 
power  of  transmitting  pressure  equally  in  all  directions,  may  yet 
carry  mankind,  it  would  be  idle  to  conjecture.  Enough,  how- 
ever, is  here  shown  to  satisfy  the  reader,  that  in  expounding  the 
laws  of  the  pressure  and  equilibrium  of  fluids,  as  well  as  those 
of  their  motion  and  resistance,  he  will  encounter  principles  of 
great  practical  utility  in  the  construction  and  use  of  machines, 

c2 


XXVlll  INTRODUCTION. 

engines,  apparatus,  and  instruments  employed,  not  only  in  the 
higher  departments  of  natural  philosophy,  but  in  the  every-day 
concerns  of  society,  in  the  arts,  manufactures,  and  domestic 
operations  of  civilized  men.  The  occurrence  of  such  principles 
seems  to  present  the  legitimate  time  and  place  for  classifying  the 
inventions  to  which  they  gave  existence,  and  for  directing  genius 
in  its  attempts  to  elicit  new  applications  of  collateral  principles : 
for  though  fortuitous  circumstances  and  accidental  hints  may 
have  led  to  some  discoveries  in  Hydrodynamics,  the  greater  part 
of  modern  improvements  must  be  traced  to  patient  induction, 
which  arrives  at  those  coincidences  whereby  scientific  men  are 
enabled  to  expound  the  theory  of  particular  machines,  whose 
construction  and  principles  of  action  depend  upon  the  equi- 
librium or  motion  of  fluids.  By  this  method,  nothing  is  taken 
for  granted  which  can  be  investigated  from  a  series  of  mathe- 
matical truths  ;  for,  as  Mr.  Whitehurst  observes,  "  it  is  one  thing 
to  assent  to  truths,  and  another  to  prove  them  to  be  true :  the 
former  leaves  the  mind  in  a  state  of  suspense,  the  latter  in  the 
possession  of  truth."  * 

This  chapter  concludes  with  some  experiments  upon  the 
quaqua  versus  property  of  non-elastic  fluids  ;  these  experiments 
have  the  lowly  merit  of  placing  that  property,  the  power  of 
transmitting  pressure  equally  in  all  directions,  in  a  popular 
point  of  view,  "  level  to  the  capacity  of  ordinary  minds." 

Our  labours  hitherto  refer  exclusively  to  what  may  be  termed 
elementary  principles  in  the  mechanics  of  fluids ;  we  now  com- 
mence with  PRESSURE,  as  it  unfolds  itself  in  the  action  of  fluids 
of  variable  density,^  or  such  as  have  their  densities  regulated  by 
certain  conditions,  dependent  upon  particular  laws,  whether  ex- 
cited by  motion,  by  mixture,  or  by  change  of  temperature.  This 
is  the  subject  of  Chapter  Seventh,  in  which  it  will  be  found 
that  the  investigation  of  the  pressure  of  fluids  of  variable  den- 
sity is  fruitful  of  some  remarkably  curious  results :  among  these 
we  may  notice  the  circumstance  of  a  globe  of  condensible 


*  "  Inquiry  into  the  Original  State  and  Formation  of  the  Earth." — London,  1792. 

t  The  word  variable  is  perhaps  taken  in  a  too  general  sense :  the  densities  are  not 
variable  in  all  cases,  they  are  only  different  — yet  they  are  sometimes  variable  also ; 
but  there  can  be  no  more  correct  mode  of  writing  upon  this  subject. 


INTRODUCTION.  XXIX 


matter  immersed  in  the  sea  to  a  given  depth,  as  being  likely  to 
suggest  some  easy  and  accurate  methods  of  determining  the  depth 
of  the  ocean,  when  it  is  so  profound  as  to  preclude  the  appli- 
cation of  the  methods  npw  in  use.  The  next  fact  claiming  our 
attention  here,  is  the  result  we  obtain  by  putting  fluids  of  dif- 
ferent densities  into  bended  tubes,  when  the  perpendicular  alti- 
tudes of  these  fluids  above  their  common  surface  will  vary 
inversely  as  their  specific  gravity ;  for  we  herein  settle  at  once 
the  grand  problem  in  our  domestic  policy — what  is  the  best 
method  by  which  large  towns  and  cities,  or  in  fact  any  place, 
can  be  supplied  with  water  from  a  distance.  But  this  is  not  all 
— another  result  is,  the  construction  of  the  hydrostatic  quadrant, 
for  finding  the  altitude  of  the  heavenly  bodies,  when  from  haze 
or  atmospheric  obscurity,  the  horizon  is  rendered  indistinct  or 
invisible.  We  trust  our  investigation  of  this  beautiful  principle 
of  the  pressure  of  fluids  of  variable  density,  will  in  some  mea- 
sure facilitate  the  construction  of  the  hydrostatic  quadrant — an 
instrument  but  as  yet  in  its  infancy. 

The  Eighth  Chapter  is  one  of  vast  utility  in  constructive  me- 
chanics, when  it  is  necessary  to  investigate  the  pressure  of  fluids 
on  dykes  and  embankments,  a  subject  interesting  and  im- 
portant in  the  doctrine  of  Hydraulic  Architecture,  and  peculiarly 
applicable  to  the  inland  navigation  and  the  maritime  accommoda- 
tion of  a  country  situated  like  Great  Britain,  every  where  inter- 
sected by  canals,  and  seamed  in  all  the  sinuosities  of  her  coast 
by  the  tides  and  waves  of  the  restless  and  turbulent  commercial 
ocean.  Moreover,  this  subject  is  particularly  applicable  to  the 
great  works  now  in  progress,  as  rail-roads,  docks,  harbours,  and 
basins.  The  brevity  of  this  chapter  is  compensated  by  the  unity 
it  confers  on  separate  and  distinct  portions  of  fluid  pressure 
and  support :  and  the  exact  formulae  it  affords  to  practical  men 
in  estimating  expense,  while  their  undertakings  are  executed  with 
systematic  regard  to  permanent  durability. 

The  Ninth  Chapter  treats  of  floatation,  and  the  determina- 
tion of  the  specific  gravities  of  bodies  immersed  in  fluids,  com- 
prehending therein  some  of  the  most  interesting  and  important 
principles  of  Hydrodynamic  Science.  There  are  two  general 
propositions  embraced  by  this  department  of  the  philosophy  of 
fluids  : — viz. 


XXX  INTRODUCTION. 

1st.  That  when  a  body  floats,  or  when  it  is  in  a  state  of 
buoyancy  on  the  surface  of  a  fluid  of  greater  specific  gravity 
than  itself, 

It  is  pressed  uptvards  by  a  force,  whose  intensity  is 
equivalent  to  the  absolute  weight  of  a  quantity  of  the 
fluid,  of  which  the  magnitude  is  the  same  as  that  portion 
of  the  body  below  the  plane  of  floatation,  or  the  horizontal 
surface  of  the  fluid. 

2dly.  That  if  a  solid  homogeneous  body  be  placed  in  a  fluid 
of  a  greater  or  less  specific  gravity  than  itself, 

It  will  ascend  or  descend  with  a  force  which  is  equiva- 
lent to  the  difference  between  its  oivn  weight  and  that  of 
an  equal  bulk  of  the  fluid; 

a  proposition  which  is  almost  self-evident,  but  which  leads  to  a 
series  of  inferences,  practically  of  vast  importance  in  the  me- 
chanics of  fluids. 

Archimedes,  the  Sicilian  philosopher,  first  established  the  fun- 
damental laws  of  fluid  equilibrium,  and  the  specific  gravity  of 
bodies  immersed  in  fluids.  Having  determined  the  conditions 
which  are  requisite  to  produce  and  measure  the  equilibrium  of  a 
solid  floating  on  a  fluid,  the  philosopher  readily  perceived  that 

Two  bodies  equal  in  bulk,  and  immersed  in  a  fluid 
lighter  than  either  of  them,  lose  equal  quantities  of  their 
weight ; 

or  inversely,  that  when 

Two  bodies  lose  equal  quantities  of  their  weight  in  a 
fluid,  they  are  of  equal  volume  ; — 

this  is  the  7th  Prob.  of  his  first  book — De  Humido  Insidentibus, 
or  Of  bodies  floating  on  a  fluid.  Mathematicians  generally 
suppose  Archimedes  employed  this  proposition  to  solve  the  well- 
known  problem  proposed  to  him  by  Hiero,  king  of  Syracuse, 
who  having  employed  a  goldsmith  to  make  a  crown  of  pure 
gold,  and  suspecting  that  the  artist  had  not  kept  faith  with  him, 
applied  to  Archimedes  to  discover  the  truth  without  injuring 
the  crown.  The  philosopher,  it  is  said,  laboured  in  vain  at  the 


INTRODUCTION.  XXXI 

problem,  till,  going  one  day  into  the  bath,  he  perceived  that  the 
water  rose  in  the  bath  in  proportion  to  the  bulk  of  his  immersed 
body ;  it  occurred  to  him  at  that  moment  that  any  other  sub- 
stance of  equal 'size  would  have  raised  the  water  just  as  much, 
though  one  of  equal  weight  and  of  less  bulk  could  not  have 
produced  the  same  effect.  He  immediately  felt  that  the  solu- 
tion of  the  king's  question  was  within  his  reach,  for  taking  two 
masses,  one  of  gold  and  one  of  silver,  each  equal  in  weight  to 
the  crown,  and,  having  filled  a  vessel  very  accurately  with 
water,  he  first  plunged  the  silver  mass  into  it,  and  observed  the 
quantity  of  water  that  flowed  over ;  he  then  did  the  same  with 
the  gold,  and  found  that  a  less  quantity  had  passed  over  than 
before.  Hence  he  inferred  that,  though  of  equal  weight,  the 
bulk  of  the  silver  was  greater  than  that  of  the  gold,  and  that 
the  quantity  of  water  displaced  was,  in  each  experiment,  equal 
to  the  bulk  of  the  metal.  He  next  made  a  like  trial  with  the 
crown,  and  found  it  displaced  more  water  than  the  gold,  and 
less  than  the  silver,  which  led  him  to  conclude  that  it  was 
neither  pure  gold  nor  pure  silver. 

This  discovery  by  Archimedes,  which  after  all  is  but  the 
application  of  the  well-known  axiom,  that  two  bodies  cannot 
occupy  the  same  space  at  the  same  time,  has  been  considered 
one  of  the  most  fortunate  in  the  annals  of  science,  for  it  has 
led  to  great  advances  in  the  arts,  and  become  the  foundation  of 
chemical  analysis ;  just  in  the  same  way  that  his  development 
of  the  properties  of  floating  bodies  has  formed  the  rudiments 
of  naval  architecture,  how  much  soever  this  branch  of  con- 
structive mechanics  may  boast  of  its  modern  improvements. 

In  the  Tenth  Chapter,  specific  gravities  and  the  methods  of 
weighing  solid  bodies  in  fluids  are  treated  of;  and  the  principle 
here  to  be  demonstrated  is, 

That  when  a  solid  body  is  immersed  in  a  fluid  of  dif- 
ferent specific  gravity  from  itself,  the  weight  which  the 
body   loses   will  be  to    its  whole  weight,  as  the  specific 
gravity  of  the  fluid  is  to  the  specific  gravity  of  the  solid. 

In  this  chapter  we  have  a  full  developement  of  that  fine  thought, 
which  rendered  the  truth  of  experiment  an  overmatch  for  the 
craft  of  Hiero's  goldsmith ;  and  the  examples  we  have  pro- 


XXX11  INTRODUCTION. 

duced,  though  not  voluminous,  fully  show  the  different  ways  of 
solving  the  ancient  problem  of  Archimedes — 

To  find  the  respective  weights  of  two  known  ingredients 
in  a  given  compound. 

The  principle  enunciated  above,  may  be  popularly  expounded 
in  the  following  manner.  Every  body  placed  on  a  surface  of 
water,  has  a  tendency  to  sink  by  its  own  weight :  it  is,  however, 
resisted  by  a  force  equivalent  to  an  equal  bulk  of  the  fluid,  or 
of  as  much  fluid  as  will  fill  the  space  occupied  by  the  body. 
Should  the  body  be  heavier  than  the  fluid,  bulk  for  bulk,  its 
greater  weight  will  cause  it  to  descend,  for  the  upward  pressure 
of  the  fluid  will  not  prevent  the  descent.  When,  on  the  other 
hand,  the  body  is  specifically,  that  is  to  say  bulk  for  bulk, 
lighter  than  the  fluid,  its  pressure  downwards  will  be  less  than 
the  upward  pressure  of  the  fluid  at  the  same  depth;  conse- 
quently, as  the  greater  force  necessarily  overcomes  the  less, 
and  the  upward  pressure  is  the  greater,  the  body  will  rise. 
When  the  body  and  the  fluid  have  the  same  specific  gravity, 
then  equal  masses  of  each  are  of  the  same  weight,  and  the  de- 
scending force  being  equally  balanced  by  the  ascending  force, 
the  body  will  float  with  its  upper  surface  coincident  with  the 
surface  of  the  fluid,  or  in  any  other  position  whatever  in  which 
it  may  be  placed. 

It  is  very  obvious  from  these  laws,  that  if,  by  any  contrivance 
or  change,  the  specific  gravity  of  a  body  can  be  so  altered  and 
varied,  as  to  be  at  one  time  greater,  at  another  time  less,  and 
then  equal  to  the  specific  gravity  of  the  fluid  in  which  it  is 
placed,  the  said  body  will  sink,  or  rise,  or  remain  at  rest,  accord- 
ing to  the  variations  produced  in  its  specific  gravity.  Lecturers 
amuse  their  audiences  with  glass  images,  which,  upon  the 
principle  here  adverted  to,  ascend  or  descend,  or  remain  in  mid- 
water,  at  the  pleasure  of  these  philosophers. 

The  doctrine  of  the  Equilibrium  of  Floatation,  which  appears 
in  Chapter  XL,  is  as  old  as  the  days  of  Archimedes,  who  ex- 
amines the  conditions  which  are  requisite  to  produce  and  pre- 
serve the  equilibrium  of  a  solid  floating  in  a  fluid.  He  shows 
that  when  a  body  floats  in  a  state  of  equilibrium  on  the  surface 
of  an  incompressible  fluid, 


INTRODUCTION.  XXX111 


The  centre  of  gravity  of  the  whole  body,  and  that  of 
the  part  immersed,  must  occur  in  the  same  vertical  line,  or 
the  line  of  pressure  and  the  line  of  support  must  coincide; 
and,  secondly,  that  the  magnitude  of  the  body  is  to  that  of 
the  part  immersed  below  the  plane  of  floatation,  as  the 
specific  gravity  of  the  fluid  is  to  that  of  the  floating  body. 

Of  the  truth  of  the  doctrine  which  is  here  propounded,  and, 
let  us  hope,  satisfactorily  demonstrated  in  the  sequel  of  our 
work,  we  have  a  curious  illustration  afforded  by  an  Arab  ship- 
builder in  Java,  whose  task  is  thus  described  in  GEORGE 
EARL'S  Eastern  Seas: — "The  largest  merchant  vessel  in  Java, 
a  ship  about  1,000  tons  burden,  was  built  by  an  Arab  merchant, 
in  a  long  but  shallow  river,  which  runs  into  the  sea  near  Soura- 
baya.  As  great  expense  is  incurred  by  floating  the  timber  in 
rafts  down  the  river,  he  determined  to  commence  the  work  in 
the  forest  itself,  as  he  would  thereby  be  enabled  to  select  the 
best  trees  for  the  purpose.  He  accordingly  ascended  the  river, 
accompanied  by  a  sufficient  number  of  workmen,  conveying  the 
necessary  materials,  and  commenced  the  undertaking  about  80 
miles  from  the  sea.  When  the  keel  and  the  floor  timbers  were 
laid,  and  a  few  of  the  bottom  planks  nailed  on,  he  launched  the 
embryo  vessel,  and  floated  her  gently  down  the  river  to  a  place 
in  which  the  water  was  deeper.  Here  the  building  was  con- 
tinued, until  it  became  necessary  to  seek  a  deeper  channel,  and 
in  this  manner  the  work  proceeded,  the  vessel  being  floated 
further  down  the  river,  whenever  the  water  was  found  to  be  too 
shallow  for  her  to  float,  until  at  length,  she  was  fairly  launched, 
half  finished,  into  the  sea,  and  completed  in  the  harbour." 

The  operations  of  this  ingenious  orientalist  proceeded  upon 
the  truth  stated  in  Inference  5,  page  261,  that  if  a  body  float 
in  equilibrio  on  the  surface  of  a  given  fluid,  and  if  the  part 
below  the  plane  of  floatation  be  increased  or  diminished  by  a 
given  quantity,  the  absolute  weight  of  the  body,  (in  order  that 
the  equilibrium  might  still  obtain,)  must  be  increased  or  dimi- 
nished by  a  weight  which  is  equal  to  the  weight  of  the  portion 
of  the  fluid  that  is  more  or  less  displaced,  in  consequence  of 
increasing  or  diminishing  the  immersed  part  of  the  body,  or 
that  which  falls  below  the  plane  of  floatation. 


XXXIV  INTRODUCTION. 


As  his  work  proceeded,  the  Arab  could  calmly  and  skilfully 
contemplate  the  effect  of  the  antagonist  forces  directed  to  the 
centre  of  gravity  and  the  centre  of  buoyancy  of  his  ship,  and 
survey  her  equilibrium  as  it  might  be  permanent  or  instable ; 
even  though  he  knew  nothing  of  the  fine  theory  of  M.  Bouguer, 
or  the  laborious  calculations  of  the  Swedish  Admiral  Chapman 
or  of  Mr.  Atwood,  on  the  hull  of  the  Cuffnells. 

But  we  have  other  topics  of  equal  practical  importance  with 
the  floatation  of  vessels  in  this  chapter,  as  for  example :  1st. 
The  consideration  of  a  body  floating  in  equilibrio  between  two 
fluids  which  do  not  mix  when  the  weights  of  the  fluids  respec- 
tively displaced,  are  together  equal  to  the  weight  of  the  solid 
body  which  causes  the  displacement:  2dly.  The  construction 
and  application  of  the  hydrometer,  an  instrument  generally  em- 
ployed for  detecting  and  measuring  the  properties  and  effects  of 
water  and  other  fluids,  such  as  their  density,  gravity,  force  and 
velocity,  which  depends  upon  the  principles  explained  and  illus- 
trated in  the  eighth  proposition  :  3dly.  The  hydrostatic  balance, 
an  instrument  by  which  we  are  enabled  to  measure  the  specific 
gravities  of  bodies  with  great  accuracy  and  expedition,  whether 
the  bodies  be  in  a  fluid  or  a  solid  state. 

A  great  many  curious  facts  relating  to  the  equilibrium  of 
floatation  could  have  been  here  brought  under  the  reader's  con- 
sideration; but  these,  as  well  as  all  popular  illustrations  of 
natural  philosophy,  belong  essentially  to  Somatology,  or  the 
properties  of  matter,  a  subject  which  we  could  not  amalgamate 
with  the  calculations  that  illustrate  the  Mechanics  of  Fluids. 

The  Twelfth  Chapter  treats  of  the  positions  of  equilibrium  of 
floating  bodies,  to  determine  which,  from  strict  theory,  is  one  of 
the  finest  speculations  in  the  whole  range  of  natural  philosophy : 
to  ascertain  them,  as  we  have  done,  by  computation,  involves 
nothing  intricate  or  repulsive,  though  the  process  is  both  laborious 
and  irksome.  To  construct  them  geometrically,  demands  a  know- 
ledge of  principles  higher  than  elementary.  And  although  the 
geometrical  construction  may  truly  represent  the  position  which 
the  body  assumes  when  floating  in  a  state  of  equilibrium,  it  is  the 
application  of  numbers  after  all,  which  must  determine  the  true 
positions.  The  reason  is  this ;  the  specific  gravities  of  the 
solid  and  fluid  bodies,  which  are  always  elements  of  the  in- 


INTRODUCTION.  XXXV 

quiry,  cannot  be  represented  by  lines ;  but  having  once  obtained 
by  computation,  the  dimensions  of  the  extant  and  immersed 
portions  of  the  body,  the  sides  of  which  are  always  given  in  the 
question,  we  can  easily  exhibit  the  geometrical  construction. 
The  method  of  proof,  by  calculation,  which  we  have  applied  to 
this  part  of  our  work,  seems  to  leave  nothing  to  be  added  to  an 
elegant  branch  of  the  Mechanics  of  Fluids,  so  highly  important 
in  the  practice  of  naval  architecture. 

In  the  Thirteenth  Chapter,  we  have  considered  the  stability 
of  floating  bodies  and  of  ships.  The  subject  of  stability  is  the 
same  to  whatever  form  of  floating  body  it  may  be  referred, 
whether  the  body  be  a  ship  driven  by  wind  or  steam,  logs  of 
wood,  or  masses  of  ice,  and  it  consists  entirely  in  resolving  the 

equation  x  = S  sin.  d>.     The  determination  of  the  se- 
ra 

veral  quantities  of  which  this  equation  consists,  depends  entirely 
upon  calculations  drawn  from  the  particular  circumstances  of 
the  individual  case  under  consideration ;  and  these  circum- 
stances as  referred  to  a  ship,  it  is  impossible  to  assign  by  esti- 
mation ;  they  must  be  obtained  by  actual  measurement,  and  when 
they  have  been  obtained  in  this  manner,  they  are  to  be  inserted 
in  the  above  equation,  to  obtain  the  measure  of  stability.  The 
investigation  of  this  subject  is  both  laborious  and  intricate,  but 
from  what  we  have  done  in  Problems  LXI.  and  LXIL,  with 
their  subordinate  examples,  it  may  become  intelligible  to  the 
general  reader.  The  mathematician  who  has  consulted  the 
writings  of  the  Swedish  Admiral  CHAPMAN,  and  the  scientific 
investigations  of  ATT  WOOD,  knows  well  that  in  considering  the 
properties  of  a  vessel,  the  orderly  arrangement  requires  that  we 
should  treat,  First  of  stability,  or  the  power  a  vessel  has  of 
resisting  any  change  of  position  when  afloat.  Secondly,  the 
forms  having  stability  which  have  the  least  resistance,  and  are 
therefore  best  adapted  for  speed.  Thirdly,  the  different  methods 
of  propelling  ships  ;  and  Fourthly,  the  construction  for  strength. 
But  our  inquiries  are  much  more  limited  in  this  Treatise,  and 
might  conveniently  end  with  the  exposition  of  the  equation  of 
stability.  We  have,  however,  carried  the  subject  a  little  farther, 
and  considered  it  in  reference  to  steam  navigation,  in  order  to 
point  out  that  the  stability  of  a  ship  is  greatly  increased,  byaug- 


XXXVI  INTRODUCTION. 

meriting  the  lateral  dimension  of  the  water  line  ;  for  the  easiest 
and  most  advantageous  way  of  obtaining  stability  is  by  a  large 
area  of  floatation,  and  great  fulness  between  wind  and  water ; 
or,  which  is  the  same  thing,  by  keeping  the  centre  of  gravity  of 
the  displacement  at  the  least  possible  distance  below  the  water's 
surface,  in  order  to  obtain  the  maximum  of  stability  and  the 
fastest  rate  of  sailing  :  and  it  will  not  differ  much  from  the  truth 
to  assume  the  cross  section  of  the  vessel,  as  of  the  form  of  a 
parabola.  In  this  species  of  figure,  the  stability  and  capacity 
both  increase  as  the  ordinate  becomes  of  a  higher  power ;  but  a 
greater  breadth  is  necessary  in  proportion  to  the  vertical  height 
of  the  hull  to  give  stability.  The  breadth,  however,  should  be 
every  where  in  the  same  ratio  to  the  depth,  to  render  the  sta- 
bility equal  throughout  the  length,  or  so  that  the  vessel  will 
undergo  no  strain  from  change  of  position  by  pitching  or  rolling 
in  a  boisterous  sea. 

The  distinguishing  characteristic  of  Chapman's  works  on 
ship-building,  is  the  application  of  the  inductive  method  of 
philosophy  to  the  different  parts  of  this  subject,  to  found  a 
theory  on  experimental  results,  and  where  data  failed,  to  con- 
duct his  investigations  on  the  acknowledged  principles  of  me- 
chanics, and  subject  his  conclusions  to  the  test  of  observation 
and  experiment.  His  wrorks  have  never  been  surpassed ;  and 
in  the  treatise  on  ships  of  war,  he  collected  and  gave  in  detail 
all  the  data  which  affected  the  qualities  of  ships,  calculated 
their  effects  under  different  circumstances,  and  determined  on 
theoretical  principles,  deduced  from  his  experience,  the  dimen- 
sions and  forms  of  all  ships  of  war,  from  a  first-rate  to  the 
smallest  armed  vessel.  Their  calculated  elements  are  collected 
in  tables,  and  drawings  of  all  the  ships  constructed  agreeably 
to  these  elements  complete  the  work,  which  the  reader  will 
find  translated  by  MM.  Morgan  and  Creuze,  Naval  Architects, 
in  the  Papers  on  Naval  Architecture,  published  about  1830. 

Next  to  Chapman's,  must  be  ranked  the  Treatise  of  Leonard 
Euler,  on  the  Construction  and  Properties  of  Vessels.  The 
Calculations  relative  to  the  Equipment  and  Displacement  of 
Ships  and  Vessels  of  War,  by  John  Edi/e,  show  by  tables  and 
plates,  every  element  and  material  belonging  to  the  British 
navy. 


INTRODUCTION.  XXXVH 

The  Fifteenth  Chapter  embraces  cohesion  and  capillary 
attraction, — subjects  replete  with  many  curious  speculations, 
especially  in  our  investigations  of  the  phenomena  of  fluids. 
Whatever  may  be  the  cause  of  fluidity,  we  know  that  ice 
becomes  water  if  a  certain  degree  of  he#t  be  applied  to  it,  and 
steam  if  more  heat  be  used.  Whether  therefore,  caloric  or  motion 
be  the  cause  of  fluidity,  we  know  that  in  the  first  instance  of 
the  case  we  have  cited,  the  atoms  are  fixed  in  crystals — in  the 
second  they  are  thrown  into  intestine  motion — and  in  the  third 
state  they  are  forced  asunder  with  an  amazing  expansive  force. 

Philosophers  have  usually  assumed,  that  the  particles  of 
fluids,  since  they  are  so  easily  moved  among  one  another,  are 
round  and  smooth.  This  supposition  will  account  for  some 
circumstances  belonging  to  fluids,  as,  if  the  particles  are  round, 
there  must  be  vacant  spaces  between  them,  in  the  same  manner 
as  there  are  vacuities  between  cannon  balls  when  piled  toge- 
ther ;  between  these  balls  smaller  shot  may  be  placed,  and  be- 
tween these,  others  still  smaller,  or  gravel,  or  sand,  may  be 
diffused.  In  a  similar  manner,  a  certain  quantity  of  particles 
of  sugar  can  be  taken  up  in  a  quantity  of  water  without  in- 
creasing the  bulk  ;  and  when  the  water  has  dissolved  the  sugar, 
salt  may  be  dissolved  in  it,  and  yet  the  bulk  will  not  be  sensibly 
augmented ;  and  admitting  that  the  particles  of  water  are 
round,  this  is  easily  accounted  for.  Indeed  the  universal  law  of 
gravitation,  by  which  the  constituent  parts  of  all  bodies  mutually 
attract  each  other,  will  cause  all  such  as  are  fluid,  and  do  not 
revolve  on  their  own  axis,  to  assume  spherical  forms.  Others 
have  supposed,  that  the  cause  of  fluidity  is  the  mere  want  of 
cohesion  of  the  particles  of  fluids,  which  in  small  quantities, 
and  under  peculiar  circumstances,  arrange  themselves  in  a 
spherical  manner,  and  form  drops. 

Fluids  are  subject  to  the  same  laws  with  solids.  The  parts 
of  a  solid  are  so  connected  as  to  form  a  whole,  their  weight  is 
concentrated  in  a  single  point,  called  the  centre  of  gravity  :  but 
the  atoms  of  a  fluid  gravitate  independently  of  each  other,  and 
press  not  only  like  solids  perpendicularly  downwards,  but  also 
upwards,  sideways,  and  in  every  direction.  To  the  flexibility 
and  cohesion  of  their  particles,  is  owing  the  singular  property 
which  fluids  possess  of  forming  themselves  into  globules,  and  of 


XXXVlll  INTRODUCTION. 

remaining  heaped  up  above  the  brims  of  vessels ;  and  to  their 
attraction  of  cohesion,  may  be  referred  many  phenomena  in 
evaporation  and  solution,  their  spontaneous  ascent  in  capil- 
lary tubes,  whether  natural  or  artificial,  the  motion  of  the 
various  juices  through  .animal  bodies  and  vegetables,  of  water 
through  layers  of  ashes  and  sand  or  the  rocky  strata  of  the 
earth  and  its  ascent  between  plates  of  glass;  to  this  attraction 
may  be  referred  solid  bodies  dissolving  in  fluids,  whose  first 
colour  or  appearance  is  not  changed,  or  changed  without  sen- 
sible augmentation  of  the  volume ;  the  mutual  action  of  bodies 
in  contact  with  each  other  exhibiting  this  attraction,  as  when 
dry  salt  of  tartar  is  exposed  to  the  air,  it  becomes  fluid ;  the 
attraction  of  cohesion  evinced  in  the  process  of  evaporation,  as 
when  the  warm  air  of  a  room  is  crystallized  on  the  panes  of 
glass  during  a  cold  night.  We,  however,  are  employed  in  con- 
sidering the  cohesion  of  water,  which  is  known  to  be  a  com- 
pound of  hydrogen  and  oxygen,  in  the  proportion  of  15  parts 
of  the  former,  and  85  of  the  latter.  Now  this  oxygen,  which 
exists  in  so  large  a  proportion  in  water,  makes  exactly  one-fourth 
part  of  the  atmospheric  air  which  all  animals  breathe.  It  is 
the  pure  part  of  the  air,  for  the  nitrogen  or  azotic  gas  which 
exists  in  air,  in  the  proportion  of  three-fourths,  is  incapable  of 
sustaining  animal  life  or  combustion  for  a  single  instant.  The 
atmosphere  contains  besides  various  supplementary  matters,  but 
water  is  the  most  abundant,  being  there  found  in  its  different 
states  of  cloud,  mist,  rain,  dew,  snow — answering  a  thousand 
useful  purposes  in  the  great  laboratory  of  nature,  so  that  upon 
the  whole  there  is  a  perfect  balancing  of  actions,  preserving 
the  atmospheric  mass  in  a  uniform  state,  constantly  fit  for  its 
admirable  purposes  of  animal  and  vegetable  existence.  The 
sea-water,  however,  contains  besides  hydrogen  and  oxygen,  a 
solution  of  muriate  of  soda,  or  table  salt,  which  probably 
adapts  this  fluid  for  the  purposes  of  animal  life ;  at  all  events 
preserves  the  ocean  from  putrefaction.  That  the  oxygen  of  the 
water  does  not  by  cohesion  or  absorption  swallow  up  the 
oxygen  of  the  atmosphere,  and  leave  the  earth  to  be  surrounded 
with  a  covering  of  deadly  azotic  gas,  is  perhaps  to  be  accounted 
for  by  the  general  laws  of  electrical  attraction  and  repulsion, 
which  as  they  respect  the  physical  constitution  of  these  two 


INTRODUCTION.  XXXIX 

fluids,  preserve  a  perfect  equilibrium  between  both  and  to  each 
its  own  due  proportion  of  the  life-giving  gas,  either  as  an 
elastic  or  a  non-elastic  fluid.  And  it  is,  perhaps,  owing  to  this 
circumstance  operating  through  the  principle  of  specific  gra- 
vities, that  the  barometer — the  prophet  of  the  weather,  indicates 
the  changes  which  diversify  the  climate  of  our  earth.  When 
the  atmosphere  becomes  surcharged  with  water  it  falls  as  rain, 
and  the  weight  and  bulk  of  the  mass  being  diminished,  the 
rising  column  of  mercury  presages  serene  and  dry  weather,  as 
previously  the  falling  barometer  had  prognosticated  wind  and 
rain.  Our  inquiries  cease  the  moment  we  approach  the  limit, 
which  separates  chemical  analysis  from  the  mechanics  of  fluids. 

From  the  time  of  Archimedes  till  the  age  of  Pascal,*  the 
annals  of  scientific  discovery  present  no  improvement  in  hydro- 
statics. Pascal  has  the  merit  of  discovering  the  pressure  of 
the  atmosphere,  and  his  treatise  on  the  Equilibrium  of  Liquids 
raised  hydrostatics  to  the  dignity  of  a  science.  The  midnight 
of  barbarism,  that  for  a  thousand  years  had  brooded  over  the 
discoveries  of  the  Sicilian  philosopher,  and  had  concealed  the 
Commentary  of  Sextus  Julius  Frontinus  on  the  Aqueducts  of 
Rome,  fled  before  the  genius  of  Pascal  and  the  powers  of 
Newton's  mind ;  the  former,  in  the  most  perspicuous  and  simple 
manner,  demonstrating  and  proving  by  experiments  the  laws  of 
fluid  equilibrium ;  and  the  latter  expounding  the  oscillation  of 
waves,  a  subject  the  most  refined  in  Hydrodynamic  science, 
which,  from  that  time,  counts  among  its  votaries  the  engineers  and 
philosophers  of  Italy,  France,  Sweden,  Germany,  and  Britain. 

It  is  proper  here  to  state,  that  we  believe  in  the  compres- 
sibility of  water ;  but  we  hold  it  true  that  for  all  general  opera- 
tions in  the  mechanics  of  fluids  this  compressibility  is  so 
small  as  not  to  occasion  any  error  in  the  numerous  and  varied 
formulae,  from  which  we  have  drawn  practical  rules  for  the 

*  Pascal  gave  proof  of  his  skill  in  hydrostatics,  by  the  celebrated  well  which  he 
dug  at  Port  Royal  des  Champs,  about  six  miles  from  Versailles.  The  well  still  exists, 
in  the  midst  of  the  farm  yard  of  Les  Granges  ;  but  its  machinery,  by  which  a  child 
of  ten  years  old  could  with  ease  and  safety  draw  up  water,  is  now  no  more.  On 
one  side  of  the  farm  yard  is  a  hovel,  in  which  that  good  man  studied  during  his 
visits  to  Port  Royal-  a  place  rendered  famous  also  by  the  name  of  the  devout 
Arnauld  D'Andilli. 


Xl  INTRODUCTION. 

solution  of  such  questions  as  may  engage  the  attention  of  our 
readers. 

LESLIE  computes  that  air  would  become  as  dense  as  water 
at  the  depth  of  33 j  miles ;  it  would  even  acquire  the  density 
of  quicksilver  at  a  further  depth  of  163|  miles  ;  and  he  hence 
concludes  with  the  probability  that  the  ocean  may  rest  on  a 
bed  of  compressed  air.  Water  at  the  depth  of  93  miles  would 
be  compressed  into  half  its  bulk ;  at  the  depth  of  362.5  miles 
it  wrould  acquire  the  ordinary  density  of  quicksilver.  Even 
marble  itself,  subjected  to  its  own  pressure,  would  become  twice 
as  dense  as  before,  at  the  enormous  depth  of  283.6  miles.  But 
air,  from  its  rapid  compressibility,  would  sooner  acquire  the 
same  density  with  water,  than  this  fluid  would  reach  the  con- 
densation of  marble. 

For  the  coincidence  of  air  and  water  the  depth  is  35.5  miles ; 
for  equal  densities  of  water  and  marble  172.9  miles.  At  the 
depth  of  395.6  miles,  or  one-tenth  the  radius  of  the  earth,  air 
would  attain  the  density  of  101960  billions  ;  while  at  the  same 
depth  water  would  acquire  but  the  density  of  4.3492,  and  marble 
only  3.8095.  At  the  centre  of  the  earth,  the  density  of  air  would 
be  expressed  by  764  with  166  ciphers  annexed ;  while  water 
would  be  condensed  three  millions  nine  thousand  times  its  bulk 
at  the  surface  of  the  ocean ;  and  marble  would  acquire  the 
density  of  119.  The  inference  is,  that  if  the  structure  of  our 
globe  were  uniform,  and  its  mass  consisted  of  such  materials  as 
we  are  acquainted  with,  its  mean  density  would  far  surpass  the 
limits  assigned  by  Astronomy.  Now  both  Dr.  Maskelyne  and 
Cavendish  nearly  concur  in  representing  the  mean  density  at 
only  about  five  times  greater  than  that  of  water.  Leslie  is 
thence  of  opinion,  that  our  planet  must  have  a  vast  cavernous 
structure,  the  crust  of  which,  for  aught  we  know  to  the 
contrary,  may  cover  some  very  diffusive  medium  of  astonishing 
elasticity,  as  light,  which  when  embodied  constitutes  elemental 
heat  or  fire.* 

*  Elements  of  Natural  Philosophy,  vol.  i.  pp.  447—457,  second  edition. 


MECHANICS    OF    FLUIDS 


CHAPTER  I. 

DEFINITIONS  AND  OBVIOUS  PROPERTIES  OF  WATERY  FLUIDS,  WITH 
THE  PRELIMINARY  ELEMENTARY  PRINCIPLES  OF  HYDRODYNA- 
MICS, FOR  ESTIMATING  THE  PRESSURE  OF  INCOMPRESSIBLE 
FLUIDS. 

1 .  THE  phenomena  of  Hydrodynamics  are  those  truths  which  explain 
the  peculiarities  of  equilibrium  and  motion  among-  fluid  bodies,  espe- 
cially those  that  are  heavy  and  liquid.  As  that  branch  of  natural  phi- 
losophy which  points  out  and  explains  the  properties  and  affections  of 
fluids  at  rest,  it  comprehends  the  doctrine  of  pressure,  specific  gravity, 
equilibrium,  together  with  the  circumstances  attending  the  positions, 
equilibrium,  and  stability  of  floating  bodies,  the  phenomena  of  cohe- 
sion and  capillary  attraction.  And  as  that  other  branch  of  natural 
philosophy  which  points  out  and  explains  the  motions  of  such  fluids  as 
have  weight  and  are  liquids,  it  investigates  the  means  by  which  such 
motions  are  produced,  the  laws  by  which  they  are  regulated,  the  dis- 
charge of  fluids  through  orifices  of  various  dimensions,  forms,  and 
positions, — the  motion  of  fluids  in  pipes,  rivers,  and  canals,  and  the 
force  or  effect  they  exert  against  themselves,  or  against  solid  bodies 
which  may  oppose  them.  Hydrodynamics,  therefore,  from  Greek 
words  signifying  water  and  force,  comprehend  the  entire  science  of 
watery  fluids,  whether  in  a  state  of  rest  or  of  motion ;  and  this 
science,  practically  considered,  enables  us  to  investigate  and  apply 
a  fruitful  source  of  maxims  and  principles,  upon  which  depend  the 
construction  and  efficiency  of  engines  and  machines  employed  in  the 
arts,  manufactures,  and  domestic  concerns  of  society,  together  with 
that  extensive  class  of  mechanical  combinations  displayed  in  the  more 
delicate  and  important  operations  of  HYDRAULIC  ARCHITECTURE. 

VOL.  i.  B       * 


2  ELEMENTARY     PRINCIPLES    OF    FLUID    PRESSURE. 

2.  A  Fluid  is  a  body  so  constituted,  that  its  parts  are  all  ready  to  yield 
to  the  action  of  the  smallest  force  or  pressure,  in  whatsoever  direction 
it  may  be  exerted.     The  following  are  some  of  the  simplest  and  most 
obvious  properties  of  fluids.* 

3.  Every  particle  of  a  fluid  presses  equally  in  all  directions,  whether 
it  be  upwards  or  downwards,  laterally  or  obliquely;  consequently,  the 
lateral  pressure  of  a  fluid  is  equal  to  its  perpendicular  pressure.    The 
converse  of  this  is  equally  obvious,  and  is  thus  expressed. 

4.  Every  particle  of  a  fluid  in  a  state  of  quiescence,  is  pressed  equally 
in  all  directions. 

5.  When  a  fluid  is  m  a  state  of  rest,  the  pressure  exerted  against  the 
surface  of  the  vessel  which  contains  it,  is  perpendicular  to  that  surface. 

6.  When  a  mass  of  fluid  is  in  a  state  of  rest,  its  surface  is  horizontal, 
or  perpendicular  to  the  direction  of  gravity. 

7.  If  two  fluids  which  do  not  mix,  are  poured  into  the  same  vessel, 
and  suffered  to  subside,  their  common  surface  is  parallel  to  the  horizon ; 
consequently  the  surfaces  of  fluids  continue  horizontal,  when  sub- 
jected to  the  pressure  of  the  atmosphere. 

8.  The  particles  of  a  fluid,  situated  at  the  same  perpendicular  depth 
below  the  surface,  are  equally  pressed. 

9.  When  a  fluid  is  in  a  state  of  rest,  the  pressure  upon  any  of  its 
constituent  elements,  wheresoever  situated,  is  equal  to  the  weight  of 
a  column  of  fluid  particles,  whose  length  is  equal  to  the  perpendicular 
depth  of  the  particle  or  element  pressed  ;  consequently,  the  pressure 
on  any  particle  varies  ^s  its  perpendicular  depth,  and  in  any  vessel 
containing  a  fluid  in  a  state  of  rest,  the  parts  that  are  deepest  sustain 
the  greatest  pressure. 

These  principles,  which  flow  immediately  from  the  conditions  of 
fluidity,  are  too  simple  and  obvious  to  require  demonstration,  yet 
nevertheless,  the  writers  on  hydrostatical  science  generally  accompany 
them  with  a  sort  of  popular  proof,  which  may  be  found  in  almost 
every  treatise  that  has  appeared  on  the  subject.  But  our  immediate 
object  being  to  unfold  the  more  important  elementary  principles,  by 
the  resolution  of  a  series  of  examples  dependent  upon  one  general 
proposition,  we  have  thought  it  unnecessary  to  exhibit  the  demonstra- 
tions here  (Note  A).  The  general  proposition  is  as  follows  : — 

*  Fluids  are  generally  divided  into  two  sorts,  compressible  and  incompressible,  or 
elastic  and  non-elastic ;  the  latter  of  which,  or  incompressible  and  non-elastic  fluids, 
such  as  water,  mercury,  wine,  &c.,  form  the  subject  of  the  present  article ;  the  dis- 
cussion of  the  compressible  and  elastic  fluids,  is  reserved  for  another  place.  The 
compressibility  of  water  is  so  small,  that  in  all  practical  operations  in  mechanics 
its  bulk  or  mass  may  generally  be  considered  unalterable  :  for  at  a  thousand  fathoms 
depth  it  can  only  be  compressed  one-twentieth  of  its  bulk  at  the  surface. 


ELEMENTARY    PRINCIPLES    OF    FLUID    PRESSURE. 

PROPOSITION  I. 

10.  When  an  incompressible  and  non-elastic  fluid  is  in  a  state 
of  equilibrium,  and  subjected  only  to  the  action  of  gravity: — 

The  magnitude,  or  the  intensity  of  pressure  exerted  by  the 
fluid,  perpendicularly  to  any  surface  immersed  in  it,  or  other- 
wise exposed  to  its  influence,  is  measured  by  the  weight  of  a 
column  of  the  fluid,  whose  base  is  eqval  to  the  area  pressed, 
and  whose  altitude  is  the  same  as  the  depth  of  the  centre  of 
gravity  of  that  area  beneath  the  upper  surface  of  the  fluid. 

This  is  an  elegant  and  most  important  proposition  in  the  doctrine  of 
fluid  pressure,  and  in  order  that  the  principle  may  be  the  more  readily 
perceived,  and  the  demonstration  the  more  easily  comprehended,  it 
will  be  proper,  in  the  first  place,  to  exhibit  and  demonstrate  an  analo- 
gous property,  in  reference  to  the  common  centre  of  gravity  of  a 
system  of  bodies,  or  of  the  particles  of  matter  of  which  the  system  is 
composed. 

The  property  which  we  have  alluded  to  above,  is  noticed  by  almost 
every  writer  on  the  principles  of  mechanical  science,  and  it  has  at 
various  times  received  most  beautiful  and  rigorous  demonstrations  ;  it 
may  therefore,  at  first  sight,  appear  superfluous  to  introduce  it  here; 
but  in  order  to  bring  the  subject  more  immediately  before  the  atten- 
tion of  our  readers,  we  do  not  hesitate  to  repeat  the  process. 

PROPOSITION  (A). 

11.  If  there  be  any  system  of  bodies  and  a  plane  given  in  position 
with  respect  to  them  : — 

The  distance  of  that  plane  from  the  common  centre  of 
gravity  of  the  system,  is  equal  to  the  aggregate  of  the  pro- 
ducts, arising  from  multiplying  each  body  into  its  distance 
from  the  given  plane,  divided  by  the  sum  of  the  bodies. 

The  proposition  just  enunciated,  being  of  the  greatest  use  in  many 
departments  of  philosophical  inquiry,  and  of  essential  importance  in 
establishing  the  truth  of  the  hydrodynamic  principle  above  specified, 
we  shall  therefore  bestow  some  attention  on  its  illustration  for  the 
purpose  of  rendering  it  as  clear  as  possible,  by  connecting  the  steps 
with  separate  diagrams,  -and  pursuing  the  reasoning,  until  we  shall 
have  proceeded  so  far  that  the  law  of  induction  becomes  manifest,  and 
from  thence,  the  truth  of  the  principle  announced  in  the  proposition. 

To  accomplish  this  purpose,  let  a  and  b,  be  two  very  small  bodies 

E  2  V 


4  ELEMENTARY    PRINCIPLES   OF    FLUID    PRESSURE. 

or  particles  of  matter,  supposed  to  be  col- 
lected into  their  respective  centres  of  gravity, 
and  let  A  B  c  D  be  a  smooth  rectangular 
plane  or  surface,  placed  in  any  position 
with  respect  to  the  bodies  a  and  b. 

Connect  a  and  b  by  the  straight  line 
ab,   and   let    m   be   the   place    of   their 

common  centre  of  gravity;  draw  the  straight  lines  a  p,  m  q  and 
br  respectively  perpendicular  to  the  plane  A  B  c  D,  and  consequently 
parallel  to  one  another  ;  join  pr,  then  because  the  points  a,  m,  b  are 
situated  in  a  straight  line,  the  points  p,  q,  r  are  also  in  a  straight  line, 
and  therefore  p  r  will  pass  through  the  point  q. 

Through  mt  the  common  centre  of  gravity  of  the  two  bodies  a  and 
b,  draw  st  parallel  to  pr,  meeting  br  in  s,  and  pa  produced  in  t  ; 
then  the  triangles  ami  and  bms,  are  similar  to  one  another;  but 
by  the  property  of  the  lever,  we  have 

a  :  b  ::  bm  :  am, 
and  by  similar  triangles,  it  is 

bm  :  am  :  :  bs  :  at; 
therefore,  by  the  equality  of  ratios,  we  obtain 

a  :  b  :  :  bs  :  at  ; 

from  which,  by  equating  the  products  of  the  extreme  and  mean  terms, 
we  get 

a  X  at  —  bx  bs. 

Now,  it  is  manifest  by  the  construction,  that  at  —  pt—pa,  and 
b  s  —  r  b—  r  s;  therefore,  by  substitution,  we  obtain 

a  (p  t—pa)=b  (rb  —  rs)', 

but  by  reason  of  the  parallels  p  r  and  t  s,  the  lines  p  t  and  r  s  are 
respectively  equal  to  m  q  ;  hence  we  have 

a  (m  q  —p  a)  m:  b  (r  b  —  m  q), 

and  from  this,  by  collecting  the  terms  and  transposing,  we  get 
(a  +  b)  mq  —  a  X  p  a  '  -f-  b  X  r  b, 
and  finally,  by  division,  we  obtain 
a  xa      b  X  rb 


-  - 

a  -f-  b 

COROL.  Here  then,  the  truth  of  the  proposition  is  manifest  with 
respect  to  a  system  composed  of  only  two  bodies  ;  that  is, 

The  distance  of  the  common  centre  of  gravity  from  the 
plane  to  which  the  bodies  are  referred,  is  equal  to  the  sum  of 
the  products,  arising  by  multiplying  each  body  into  its  dis- 
tance from  the  given  plane,  divided  by  the  sum  of  the  bodies. 


ELEMENTARY    PRINCIPLES    OF    FLUID    PRESSURE.  5 

12.  Again,  let  a,  b  and  c,  be  a  system  of  three  very  small  bodies  or 
particles  of  matter,  any  how  situated 
with  respect  to  the  plane  A  B  c  D,  and 
connected  together  by  the  straight 
lines  a  b,  be  and  a  c ;  and  suppose 
the  two  bodies  a  and  b  to  be  collected 
into  their  common  centre  of  gravity 
at  the  point  m. 

Join  the  points  m  and  c  by  the 
straight  line  me,  and  let  n  be  the  place  of  the  common  centre  of 
gravity  of  the  three  bodies  a,  b  and  c ;  draw  the  lines  mq,nu  and 
cv  parallel  to  each  other,  and  respectively  perpendicular  to  the  plane 
A  B  c  D;  join  qv,  and  because  the  points  m,  n  and  c  are  situated  in 
the  straight  line  m  c  ;  it  follows,  that  the  points  q,  u  and  v  must  also 
occur  in  a  straight  line ;  consequently,  q  v  will  pass  through  the 
point  u. 

Through  n,  the  common  centre  of  gravity  of  the  three  bodies  o,  b 
and  c,  draw  st  parallel  to  qv,  meeting  mq  in  t  and  vc  produced  in 
s  ;  then  are  the  triangles  m  n  t  and  ens  similar  to  one  another ;  but 
by  the  property  of  the  lever,  and  because  the  body  at  m  is  equal  to 
a  4-  6,  we  obtain 

a  ~\-  b  :  c  :  :  c  n  :  m  w, 
and  by  similar  triangles,  we  have 

c  n  :  m  n  : :  c  s  :  m  t ; 
therefore,  by  the  equality  of  ratios,  we  get 

a  -|-  b  :  c  : :  c  s  :  m  t ; 

consequently,  by  equating  the  products  of  the  extreme  and  mean 
terms,  we  shall  obtain 

(a  -f  b)  X  m  t  zz  c  X  c  s ; 
now  mt  —  mq  —  t  q,  and  c  s—v  s— vc;  hence  we  get 

(a  -f-  b)  (mq  —  tq)  —  c(ys—vc). 

But  it  is  manifest  by  the  construction,  that  t  q  and  v  s  are  each  of 
them  equal  to  nu  ;  therefore,  by  substitution  we  have 

(a  -|-  b  (mq  —  n  «)  zr  c  (nu— v  c)  ; 
therefore,  by  collecting  the  terms  and  transposing,  we  get 

(a  +  b  +  c)  Xnu=(a+b)  Xmq  +  cXvc; 
now,  it  has  already  been  shown  in  the  case  of  two  bodies,  that 
a  X^a-f  b  X  rb 

n<*= ^+T~ 

therefore,  by  substituting  this  value  of  m  q  in  the  step  immediately 
preceding,  we  shall  obtain 


ELEMENTARY     PRINCIPLES    OF    FLUID    PRESSURE. 


(a  4-6-f-  c)  Xnu  —  aXp 

and  finally  by  division,  we  have 
_aXpa-\-bXrb-\-cXvc 
(a  +  b  +  c) 

Now,  n  u  is  the  distance  of  the  common  centre  of  gravity  of  the 
three  bodies  a,  b  and  c,  from  A  B  c  D  the  plane  to  which  they  are 
referred  ;  hence  again,  the  truth  of  the  proposition  is  manifest,  and 
if  another  body  were  added  to  the  system,  a  similar  investigation 
would  exhibit  the  same  law,  and  thus  we  might  proceed  to  any  extent 
at  pleasure,  the  nature  of  the  induction  being  fully  disclosed. 

COROL.  If  therefore,  we  suppose  the  system  to  be  constituted  of  an 
indefinite  number  of  small  bodies  or  particles  of  matter,  it  will  become 
assimilated  to  a  fluid  mass,  and  consequently,  the  proposition  which 
we  have  just  demonstrated  in  reference  to  the  centre  o'f  gravity,  is 
identified  with  the  well-known  theorem  for  estimating  the  pressure  of 
fluids  ;  to  which  subject  we  must  now  return. 

13.  Resuming  therefore,  the  conditions  specified  in  Proposition  I 
preceding,  let  us  suppose  that  ABCD,  de- 
notes  a  vertical  section  of  a  reservoir  full  of 
water,  E  and  F  representing  the  corresponding 
sections  of  the  walls  or  embankments  by 
which  it  is  contained  ;  then,  since  the  fluid 
is  supposed  to  be  quiescent  or  in  a  state  of 
equilibrium,  it  follows,  that  the  surface  AB  is  parallel  to  the  horizon. 

Let  b  df  hkm\>e  the  portion  of  the  containing  section  or  boundary, 
on  which  the  pressure  exerted  by  the  water  is  required  to  be  investi- 
gated, and  conceive  it  to  be  constituted  of  an  indefinite  number  of 
minute  bodies  or  particles  of  matter,  placed  at  infinitely  small  dis- 
tances from  one  another,  or  so  near,  that  their  aggregate  or  sum  shall 
make  up  the  entire  area  which  forms  the  subject  of  our  investigation. 

Suppose  the  points  b,  d,  f,  A,  k  and  m,  to  be  so  many  individual 
particles  of  the  surface  pressed,  and  through  the  points  thus  assumed 
draw  the  vertical  lines  b  a,  dc,fe,  hg,  ki  and  m  /,  which  lines  are 
severally  in  the  direction  of  gravity,  and  consequently  perpendicular 
to  the  surface  of  the  fluid,  indicating  by  their  lengths,  the  respective 
depths  of  the  several  bodies  of  which  our  immediate  system  is  corn- 


But  according  to  Proposition  I,  the  pressures  exerted  by  the  fluid 
on  the  particles  6,  d,f,  h,  k  and  m,  are  respectively  represented  by 
the  products 

b  X  ba,  dXe?c,/X  fe,  h  X  h  g,  k  X  k  i  and  m  X  m  I, 


ELEMENTARY    PRINCIPLES   OF   FLUID    PRESSURE.  7 

and  the  aggregate  or  sum  of  these  products  becomes 
p  —  b  X  ba  +  d  X  dc+fxfe  +  h  X  hg  -\- k  X  ki  +  mX  ml, 

where  p  denotes  the  sum  of  the  computed  pressures. 
Now,  it  is  manifest,  from  what  we  have  demonstrated  in  Proposition 
(A),  respecting  the  centre  of  gravity  of  a  system  of  bodies,  that 

The  sum  of  the  products,  arising  from  multiplying  each 
body  into  its  distance  from  a  certain  plane  given  in  position, 
is  equal  to  the  sum  of  the  bodies,  drawn  into  the  distance  of 
their  common  centre  of  gravity  from  that  plane. 

Let  therefore,  the  particles  b,  d,f,  h,  k  and  m  be  considered  as  a 
system  of  very  minute  bodies,  and  let  the  surface  of  the  fluid  denote 
the  plane  given  in  position,  to  which  the  system  is  referred  ;  then,  if 
G  be  the  place  of  the  common  centre  of  gravity  of  that  system,  put 
71  G  ==  3,  and  we  shall  obtain 

8  (b+d+f+h+k+m)  =  b.ba+d.dc+f.fe+h .  hg+k  .  ki+m.ml. 

But  we  have  seen  above,  that  the  sum  of  the  products  on  the  right 
hand  side  of  the  equation,  expresses  the  aggregate  pressure  on  the 
several  points  of  the  containing  surface,  to  which  the  present  step  of 
the  inquiry  refers,  and  that  pressure  we  have  briefly  represented  by  the 
symbol  p  ;  therefore  we  have 


and  this  expression  implies,  that  the  pressure  exerted  by  a  fluid,  on 
any  number  of  points  of  the  surface  that  contains  it, 

Is  equal  to  the  sum  of  the  points,  drawn  into,  the  perpendi- 
cular distance  of  their  common  centre  of  gravity  below  the 
upper  surface  of  the  fluid. 

Now,  it  is  evident,  that  the  same  law  would  obtain  if  another  point 
were  added  to  the  system,  and  even  if  the  number  of  points  were  to 
become  indefinite,  or  such  that  their  aggregate  or  sum  shall  be  essentially 
equal  to  the  area  pressed,  the  law  of  induction  would  remain  the 
same ;  consequently,  if  a  denote  the  sum  of  the  material  points,  or 
particles  of  space  in  the  surface  on  which  the  fluid  presses ;  then  we 
shall  have 

p  =  Za.  (1). 

This  equation  supposes,  that  the  specific  gravity  of  the  fluid  by 
which  the  pressure  is  propagated,  is  represented  by  unity,  which  cir- 
cumstance only  holds  in  the  case  of  water;  therefore,  let  s  denote  the 
specific  gravity  of  any  incompressible  fluid  whatever,  and  the  general 
form  of  the  equation  becomes 

p=$as.  (2). 


8  ELEMENTARY  PRINCIPLES  OF  FLUID  PRESSURE. 

Now,  it  is  obvious,  that  the  expression  5  a  s  indicates  the  weight  of 
a  column  of  the  fluid,  the  area  of  whose  base  is  a,  perpendicular  alti- 
tude S,  and  the  specific  gravity  s  ;  hence  the  truth  of  the  proposition 
is  manifest. 

COROL.  From  what  has  been  demonstrated  above,  it  appears,  that 
whatever  may  be  the  form  of  the  surface  on  which  the  fluid 
presses,  if  its  area,  and  the  position  of  its  centre  of  gravity  can  be 
ascertained,  the  intensity  of  pressure  which  it  sustains,  is  from  thence 
assignable. 

The  truth  of  the  proposition  being  thus  established,  we  shall  proceed 
to  deduce  from  it  a  few  of  the  most  useful  and  obvious  inferences. 

14.  INF.  1.  If  different  planes  be  immersed  perpendicularly,  hori- 
zontally, or  obliquely,  in  fluids  of  different  specific  gravities  : — 

The  pressures  upon  those  planes  perpendicularly  to  their 
surfaces,  are  as  their  areas,  the  perpendicular  depths  of  their 
centres  of  gravity,  and  the  specific  gravities  of  the  fluids 
jointly. 

15.  INF.  2.  If  different  planes  be  immersed  perpendicularly,  hori- 
zontally, or  obliquely  in  the  same  fluid  : — 

The  pressures  upon  those  planes  perpendicularly  to  their 
surfaces,  are  as  their  areas,  and  the  perpendicular  depths  of 
their  centres  of  gravity. 

16.  INF.  3.  If  a  plane  surface  of  given  dimensions  be  parallel  to 
the  surface  of  the  fluid  in  which  it  is  immersed  : — 

The  pressure  sustained  by  the  plane,  in  a  direction  perpen- 
dicular to  its  surface,  varies  directly  as  its  vertical  depth 
below  the  upper  surface  of  the  fluid. 

17.  INF.  4.  If  a  plane  surface  of  given  dimensions  be  any  how 
inclined  to  the  surface  of  the  fluid  in  which  it  is  immersed  : — 

The  pressure  sustained  by  the  plane,  in  a  direction  perpen- 
dicular to  its  surface,  varies  directly  as  the  vertical  depth  of 
its  centre  of  gravity,  below  the  upper  surface  of  the  fluid. 

18.  INF.  5.  If  any  number  of  planes  of  equal  areas  be  immersed 
in  the  same  fluid,  and  have  their  centres  of  gravity  at  the  same  vertical 
depth  below  the  surface  : — 

The  pressures  which  they  sustain  are  equal  to  one  another, 
whatever  be  their  form,  and  whatever  be  their  position  with 
respect  to  the  surface  of  the  fluid. 

19.  INF.  6.  If  any  plane  surface  revolve  about  its  centre  of  gravity, 
which  remains  fixed  in  position  : — 


ELEMENTARY  PRINCIPLES  OF  FLUID  PRESSURE.  9 

The  pressure  which  it  sustains  in  a  direction  perpendicular 
to  its  surface,  will  be  the  same  at  every  point  of  the  revolution 
.-,         as  if  it  remained  constantly  horizontal. 

20.  INF.  7.  If  the  perpendicular  pressures  upon  a  given  surface  be 
equal,  when  it  is  immersed  in  two  fluids  of  different  densities : — 

The  perpendicular  depths  of  the  centres  of  gravity  below 
the  surface,  will  vary  inversely  *  as  the  densities  or  specific 
gravities  of  the  fluids. 

21.  The  above   inferences   are  immediately  deducible  from  the 
general  proposition,  but  it  is  probable  that  the  last  may  require  a 
little  illustration ;  for  which  purpose — 

Put  p  •=.  the  pressure  sustained  by  the  plane  in  both  the  fluids, 
a~  the  area  of  the  plane  or  the  surface  pressed, 
s  —  the  density  or  specific  gravity  of  one  of  the  fluids, 
d  =  the  depth  at  which  the  given  surface  is  immersed  in  it, 
/  —  the  density  or  specific  gravity  of  the  other  fluid, 
and  5  =  the  depth  of  immersion. 

Then,  according  to  the  principle  indicated  by  the  general  equation  (2), 
we  have,  in  the  case  of  the  first  fluid, 

p—das, 
and  in  the  case  of  the  second  fluid,  it  is 

p  :zr  5  a  sf ; 

but  according  to  the  conditions  of  the  question,  these  expressions  are 
equal  to  one  another,  for  the  pressure  is  the  same  in  both  cases ;  con- 
sequently by  comparison,  we  have 

d  a  s  ~  S  a  /, 
and  this,  by  suppressing  the  common  factor,  becomes 

ds  =  W; 

therefore,  by  converting  this  equation  into  an  analogy  or  proportion, 
we  shall  exhibit  the  precise  conditions  of  the  inference ;  hence,  we  have 

d  :  a  :  :  /  :  s. 


*  One  quantity  is  said  to  vary  inversely  as  another,  when  of  two  quantities  the 
one  increases  as  the  other  decreases. 


CHAPTER  II. 

OF  THE  PRESSURE  OF  NON-ELASTIC  FLUIDS  UPON  PHYSICAL 
LINES,  RECTANGULAR  PARALLELOGRAMS  CONSIDERED  AS  INDE- 
PENDENT PLANES  IMMERSED  IN  THE  FLUIDS,  AND  UPON  THE 
SIDES  AND  BOTTOMS  OF  CUBICAL  VESSELS,  WITH  THE  LIMIT  TO 
THE  REQUISITE  THICKNESS  OF  FLOODGATES. 

1.    OF  THE  PRESSURE  OF  FLUIDS  ON  PHYSICAL  LINES. 

THE  principle  established  in  the  general  proposition  enables  us  now 
to  proceed  with  the  resolution  of  a  numerous  class  of  curious  and 
important  problems,  which  will  be  found  of  the  greatest  practical 
utility,  in  all  cases  in  which  the  pressure  of  watery  fluids  is  concerned. 
These  problems  we  shall  accompany  by  examples,  which  will  unfold 
their  geometrical  and  analytical  character,  and  leave  no  truth  in  the 
phenomena  of  this  branch  of  hydrodynamics  unrevealed. 

PROBLEM  I. 

22.  A  physical  line,*  of  a  given  length,  is  obliquely  immersed 
in  an  incompressible  fluid  in  a  state  of  equilibrium,  in  such  a 
manner  that  its  upper  extremity  is  just  in  contact  with  the 
surface ; — 

It  is  required  to  determine  what  pressure  it  sustains,  the 
angle  of  obliquity  being  a  given  quantity. 

Let  ABC,  represent  a  vertical  or  upright  section  of  a  lake  or 
pool  of  stagnant  water,  confined  by  the  walls  or  embankments  of 
which  E  E  is  a  vertical  section,  and  let  A  B  be 
the  |  surface  of  the  water,  supposed  by  the 
problem  to  be  in  a  state  of  equilibrium. 

In  A  B  take  any  point  a,  and  at  the  point 
a  thus  assumed,  immerse  the  line  a  b  of  the 
given  length,  and  tending  downwards  at  the 
given  inclination  or  angle  b  a  A. 

*  A  physical  line  is  that  which  belongs  to,  or  exists  in  nature,  and  is  so  called  to 
distinguish  it  from  a  mathematical  line,  which  exists  only  in  the  imagination. 


OF  THE  PRESSURE  OF  FLUIDS  ON  PHYSICAL  LINES.  11 

Bisect  a 6  in  m,  and  through  the  point  m  draw  mn  perpendicular 
to  AB,  the  surface  of  the  fluid;  then,  because  the  centre  of  gravity 
of,  a  straight  line  is  at  the  middle  of  its  length,  m  is  the  place  of  the 
centre  of  gravity,  and  nm  its  perpendicular  depth  below  the  surface 
AE;  through  b  draw  the  straight  line  be  parallel  to  mn,  and  cb  is 
the  perpendicular  depth  of  the  lower  extremity  at  b. 

Put  Z  zr  a  6,  the  length  of  the  line  whose  upper  extremity  is  at  a, 
d~=.nm,  the  perpendicular  depth  of  the  centre  of  gravity, 
$  ~  b  a  c,  the  angle  of  inclination,  or  the  given  obliquity. 
Then,  because  m  is  the  centre  of  gravity  of  the  straight  line  a  b,  we 
have  a  m  zz  \  I,  and  by  the  principles  of  Plane  Trigonometry ,we  obtain 

rad.  :  sin.  (j>  :  :  \  I  :  d, 
and  since  the  tabular  radius  is  expressed  by  unity,  we  get 

d—  \  I  sin.  <j>. 

Now,  the  whole  pressure  which  the  line  sustains  in  a  direction 
perpendicular  to  its  length,  according  to  the  second  inference  pre- 
ceding, 

Is  proportional  to  its  area,  drawn  into  the  perpendicular 
depth  of  its  centre  of  gravity  below  the  upper  surface  of  the 
fluid. 

But  the  area  of  a  physical  line  is  simply  equal  to  its  length ; 
therefore,  if  the  symbol  p  denote  the  pressure,  and  s  the  specific 
gravity  of  the  fluid  by  which  it  is  propagated,  we  shall  have 

p  =  isl*sm.<t>.  (3). 

and  this,  in  the  case  of  water,  where  the  specific  gravity  is  expressed 
by  unity,  becomes 

p~  JZ*sin.  0. 

23.  This  equation,  as  well  as  the  more  general  one  from  which  it  is 
derived,  is  sufficiently  simple  in  its  form  for  practical  application; 
but  in  order  that  nothing  may  be  omitted,  which  tends  to  render  the 
subject  intelligible  to  our  readers,  we  shall  in  this,  and  in  all  the 
succeeding  formulae  of  a  practical  or  general  nature,  draw  up  a  rule, 
describing  the  manner  in  which  the  several  steps  of  the  process  are  to 
be  performed  ;  pursuant  to  this  plan,  therefore,  the  rule  for  the  present 
case  will  be  as  follows  : — 

RULE.  Multiply  the  square  of  the  length  by  half  the  specific 
gravity  of  the  fluid,  and  again  by  the  natural  sine  of  the 
angle  of  inclination,  and  the  product  will  express  the  required 
pressure  on  the  line  in  the  oblique  position. 

24.  EXAMPLE  1.  A  physical  line  whose  length  is  36  feet,  is  im- 
mersed in  a  cistern  of  water,  in  such  a  manner  that  the  upper  extremity 


12  OF  THE  PRESSURE  OF  FLUIDS  ON  PHYSICAL  LINES. 

is  just  in  contact  with  the  surface,  and  the  other  inclining  downwards 
in  an  angle  of  67°  35';  what  pressure  does  the  line  sustain,  supposing 
the  fluid  in  which  it  is  placed  to  be  in  a  state  of  equilibrium  ? 

Here  by  the  question,  the  fluid  in  which  the  line  is  supposed  to  be 
immersed  is  water,  of  which  the  specific  is  unity;  consequently, 
according  to  the  rule,  we  have 

p  zz  36  X  36  X  \  X  sin.  67°  35' ; 

but  by  the  Trigonometrical  Tables,  the  natural  sine  of  67°  35'  is 
.92444  ;  hence  we  get 

p=\296  X  J  X  .92444  =  599.03712. 

In  this  case,  however,  the  resulting  pressure  is  only  relative,  the 
absolute  pressure  being  indeterminable,  upon  a  line  where  length 
merely  is  indicated  and  no  breadth  assigned ;  the  existence  of  surface 
being  indispensable  for  the  expression  of  a  determinate  measure. 

25.  If  the  line  were  immersed  perpendicularly  in  the  fluid,  or  so 
as  to  make  a  right  angle  with  its   surface,  the  equation  (3)  would 
become  transformed  into 

p=%sl*sin.  90°; 
but  by  the  principles  of  Trigonometry,  we  have 

sin.  90°  =  1 ; 
hence,  by  substitution,  we  obtain 

j.=  J«P;  (4). 

and  this,  in  the  case  of  water,  where  the  specific  gravity  is  unity, 
becomes 

P=ii*. 

Therefore,  the  relative  pressure  for  a  perpendicular  immersion,  on 
the  line,  as  given  in  the  above  example,  is 

p  =  36*36*  1=648. 

26.  If  the  upper  extremity  of  the  line  be  not  in  contact  with  the 
surface  of  the   fluid,  but  placed  as  in  the 

annexed  diagram,  then  the  method  of  solu- 
tion, and  consequently  the  form  of  the  re- 
sulting equation,  will  be  somewhat  different. 

Let  A  B  be  the  surface   of  the  water  or 
fluid  in   which    the   line  is   immersed,  and 

A  B  c  D  a  vertical  section,  in  whose  plane  the  line  a  b  is  situated,  E  E 
being  the  corresponding  section  of  the  walls  or  embankments  by 
which  the  fluid  is  contained. 

Bisect  the  given  line  ab  in  m,  and  through  the  point  m  thus  deter- 
mined, draw  m  n  perpendicular  to  A  B,  the  surface  of  the  fluid ;  and 
through  a  and  b  the  extremities  of  the  given  line,  and  parallel  to  mn, 
draw  ad  and  be,  and  produce  ba  to  meet  AB  in  A,  or  in  any  other 


OF  THE  PRESSURE  OF  FLUIDS  ON   PHYSICAL  LINES.  13 

point,  according  to  circumstances  ;  then  is  mn  the  depth  of  the  centre 
of  gravity  of  the  line  a  b,  below  the  surface  of  the  quiescent  fluid,  and 
ad,  be  are  respectively  the  depths  of  its  extremities,  b  AC  being  the 
angle  which  the  direction  of  the  given  submerged  line  makes  with  the 
horizontal  line  A  B. 

Put  d=ad,  the  depth  of  the  upper  extremity  of  the  given  line, 
^zrmTi,  the  depth  of  the  centre  of  gravity, 
D  in  b  c,  the  depth  of  the  lower  extremity, 
I  zz  a  by  the  length  of  the  proposed  line, 
p  zz  the  relative  pressure  upon  it  as  propagated  by  the  fluid, 
and  ^  zz  b  A  c,  the  angle  which  the  given  line  makes  with  the  horizon. 
Through  a  the  upper  extremity  of  the  given  line,  draw  ae  parallel 
to  AB  the  surface  of  the  fluid;  then   is  the  angle   bae  equal  to  the 
angle  b  A  c,  and  by  the  principles  of  Plane  Trigonometry,  we  have 

a  b  :  b  e  :  :  rad.  :  sin.  <f>; 

but  be  is  manifestly  equal  to  be  —  ad;  that  is,  6ezzD  —  c?,  and 
according  to  our  notation,  ab  —  l;  hence,  the  above  analogy  becomes 

I  :  (D  —  c?)  :  :  rad.  :  sin.  <p, 
or  by  putting  radius  equal  to  unity,  we  get 


This  equation  enables  us  to  determine  the  obliquity  of  the  line, 
when  the  perpendicular  depths  of  its  two  extremities  are  given  ;  but 
when  it  is  required  to  determine  the  relative  pressure  from  the  same 
data,  we  have  only  to  observe,  that  mn  the  perpendicular  depth  of 
the  centre  of  gravity,  is  equal  to  half  the  sum  of  the  depths  of  the  two 
extremities  ;  that  is, 


consequently,  we  obtain 


Again,  if  the  angle  of  inclination  and  the  perpendicular  depth  of 
one  extremity  of  the  line  are  given,  together  with  its  length,  the  per- 
pendicular depth  of  the  other  extremity  can  easily  be  found  ;  thus, 
suppose  that  da  is  the  given  depth,  then,  by  the  principles  of  Plane 
Trigonometry,  we  have 

b  e  zz  I  sin.  0  ; 

but  by  addition,  we  obtain 

b  c~b  e  -\~  e  c  ;  that  is,  D  zz  I  sin.  0  -f"  d  ; 

consequently,  the  perpendicular  depth  of  the  centre  of  gravity,  is 

£z=  |  I  sin.  <f>  -\-  d; 
and  the  relative  pressure  becomes 
/>zz  J  I*  sin.  p  Id. 


14  OF  THE   PRESSURE  OF  FLUIDS  ON  PHYSICAL  LINES. 

But  the  equation,  in  its  present  form,  supposes  the  specific  gravity 
of  the  fluid  to  be  expressed  by  unity,  which  only  takes  place  in  the 
case  of  water ;  in  order,  therefore,  to  generalize  the  formula,  we  must 
introduce  the  symbol  which  denotes  the  specific  gravity ;  hence, 
we  obtain 

pi=  |/2  s  sin.  0  -\-  Isd; 
or  by  collecting  the  terms,  we  get 

p  —  ls(\  Zsin.^>-|-c?).  (5). 

27.  This  is  the  general  form  of  the  equation,  on  the  supposition 
that  the  perpendicular  depth  of  the  upper  extremity  of  the  line  is 
given;  it  however  assumes  a  different  form,  when  the  depth  of  the 
lower   extremity  is  known ;    for   by  Plane  Trigonometry,  we  have 

as  above 

b  erz:£sin.0, 

and  by  subtraction,  we  obtain 

ec  —  bc — be;  that  is,  dznp — /  sin.  0  ; 

therefore,  the  perpendicular  depth  of  the  centre  of  gravity  is 

3=iD  —  \  I  sin.  cf), 

and  consequently,  the  general  expression  for  the  pressure  becomes 
p=ls(o — \  I  sin.  0).  (6). 

28.  Therefore,  the  practical  rule  for  each  of  these  cases,  when 
expressed  in  words  at  length,  is  as  follows : — 

1.  When  the  perpendicular  depth  of  the  upper  end  is  given  (5). 

RULE.  To  half  the  length  of  the  given  line  drawn  into  the 
natural  sine  of  the  angle  of  inclination,  add  the  depth  of  the 
upper  extremity  ;  then.,  multiply  the  sum  by  the  length  of  the 
line,  drawn  into  the  specific  gravity  of  the  fluid,  and  the  pro- 
duct  will  give  the  pressure  sought. 

2.  When  the  perpendicular  depth  of  the  lower  end  is  given  (6). 

RULE.  From  the  perpendicular  depth  of  the  lower  extre- 
mity, subtract  half  the  length  of  the  given  line  drawn  into  the 
natural  sine  of  the  angle  of  inclination ;  then,  multiply  the 
remainder  by  the  length  of  the  line,  drawn  into  the  specific 
gravity  of  the  fluid,  for  the  pressure  sought. 

29.  EXAMPLE  2.  A  physical  line,  whose  length  is  56  feet,  is  immersed 
in  a  cistern  of  water,  in  such  a  manner  that  its  upper  extremity  is  at 
the  distance  of  9  feet  below  the  surface,  and  its  direction  making  with 
the  horizon  an  angle  of  58  degrees ;  required  the  relative  pressure  on 
the  line,  the  water  being  in  a  state  of  quiescence  ? 

The  natural  sine  of  58  degrees,  according  to  the  Trigonometrical 
Tables,  is  .84805  ;  therefore  by  the  rule,  we  have 

28  X  .84805  +  9  =  32.7454,  the  perpendicular 


OF  THE  PRESSURE  OF  FLUIDS  ON  PHYSICAL  LINES.  15 

depth  of  the  centre  of  gravity ;  then,  finally,  because  the  specific 
gravity  of  water  is  unity,  we  have 

p  —  32.7454  X  56  =  1833.7424. 

Let  the  length  of  the  line  and  its  inclination  to  the  horizon  remain 
as  above,  and  suppose  the  depth  of  the  lower  extremity  to  be  56.4908 
feet ;  then,  by  the  rule  for  the  second  case,  we  have 

56.4908  -  28  x  .84805=  32.7454,  the  depth  of  the 
centre  of  gravity,  the  same  as  above,  from  which  the  relative  pressure 
is  found  to  be  1833.7424,  as  it  ought  to  be. 

30.  If  the  line  were  immersed  perpendicularly,  or  at  right  angles 
to  the  horizon,  then  sin.  0  is  equal  to  unity,  and  the  formulse  for  the 
pressure  become 

p=l8($l  +  d),  and^  =  Z5(D  —  J  I), 

where  it  is  manifest,  that  the  parenthetical  expressions  are  equal  to 
one  another,  each  of  them  expressing  the  perpendicular  depth  of  the 
centre  of  gravity,  or  the  middle  point  of  the  given  line. 

PROBLEM  II. 

31.  Two  physical  lines  of  different  given  lengths,  have  their 
upper  extremities  in  contact  with  the  surface  of  an  incompressible 
and  non-elastic  fluid  in  a  state  of  equilibrium  : — 

It  is  required  to  compare  the  pressures  which  they  sustain 
at  right  angles  to  their  lengths,  supposing  them  to  be  immersed 
at  given  inclinations  to  the  horizon. 

Let  A  B  c  D,  represent  a  vertical  section  of  a  vessel  filled  with 
water,  or  some  other  incompressible  and  non- 
elastic  fluid,  and  suppose  the  lines  a  b  and  cdto 
be  situated  in  the  plane  of  the  section,  in  such  a 
manner  that  the  upper  extremities  a  and  c  are 
respectively  in  contact  with  A  B  the  surface  of 
the  fluid,  while  their  directions  make  with  the 
horizon  the  angles  ba  A  and  dc  B  respectively. 

Through  the  points  b  and  d,  the  lower  extre- 
mities of  the  lines  ab  and  cd,  draw  be  and  df  respectively  perpen- 
dicular to  A  B  the  surface  of  the  fluid ;  and  through  m  and  r,  the 
middle  points  of  a  b  and  cd,  draw  the  lines  mn  and  rs  respectively 
parallel  to  the  perpendiculars  b  e  and  df;  then  are  mn  and  rs  the 
perpendicular  depths  of  the  centres  of  gravity. 

Put  I  zz  a  b,  the  length  of  the  line  whose  upper  extremity  is  a, 
I'  —  cd,  the  length  of  that  whose  upper  extremity  is  c; 
d  —  nm,  the  perpendicular  depth  of  the  centre  of  gravity  of  the 
line  a  b  ; 


16  OF  THE  PRESSURE  OF  FLUIDS  OK  PHYSICAL  LINES. 

3rz  s  r,  the  perpendicular  depth  of  the  centre  of  gravity  of  the 

line  e  d ; 

^>:n  b  a  A,  the  inclination  of  the  line  a b  to  the  horizon, 
0'=  d  c  B,  the  inclination  of  c  d  to  the  horizon,  or  to  the  line  A  B  ; 
j^nrthe  relative  pressure  upon  a  b, 
j/zzthe  relative  pressure  upon  cd, 
and  5  —  the  specific  gravity  of  the  fluid. 

Now,  because  the  centre  of  gravity  of  a  physical  straight  line  is  at 
the  middle  of  its  length,  we  have 

a  m  —  \  I,  and  c  r  rz  J  /'; 

therefore,  by  the  principles  of  Plane  Trigonometry,  we  obtain  from 
the  right-angled  triangle  a  m  n 

d=%  lsiu.(j>, 
and  from  the  right-angled  triangle  c  r  s  we  get 

£  =  |  V  sin.0. 

consequently,  the  general  expressions  for  the  relative  pressures  on  the 
lines  a  b  and  c  d,  according  to  equation  (5)  are 

p~  \  r  s  sin.0,  and;/—  J  l'*s  sin.  0', 
from  which,  by  comparison,  we  get 

p  :p'  :  :  I*  sin.  0  :  Z'a  sin.  ^'. 

INF.  1.  Hence  it  appears,  that  the  pressures  on  the  lines,  when 
their  directions  make  different  angles  of  inclination  with  the  horizon, 
Are  directly  as  the  squares  of  the  lengths,  and  the  sines  of 
the  inclinations  jointly. 

2.  Where  $=<£',  that  is,  when  the  lines  are  equally  inclined  to  the 
horizon,  whatever  may  be  the  magnitude  of  the  inclination,  then 

p:p'::l'-.l"; 

therefore,  when  the  lines  are  perpendicularly  immersed,  or  when  they 
are  equally  inclined  to  the  surface  of  the  fluid,  with  which  their 
upper  extremities  are  supposed  to  be  in  contact, 

The  pressures  which  they  sustain  perpendicular  to  their 
lengths,  are  directly  proportional  to  the  squares  of  those 
lengths. 

3.  Consequently,  if  two  or  more  lines  are  similarly  situated  in  the 
same  fluid,  the  relative  pressures  can  easily  be  compared ;  thus,  for 
example : — 

Suppose  two  physical  lines,  whose  lengths  are  respectively  36  and 
56  feet,  to  be  perpendicularly  immersed  in  the  same  fluid,  and  having 
their  upper  extremities  in  contact  with  the  surface,  or  equally  depressed 
below  it;  then,  the  pressures  sustained  by  these  lines,  are  to  one  ano- 
ther as  the  numbers  1296  and  3136 ;  that  is, 

pip'::  362:  56'  ::  1296  :  3136. 


OF  THE  PRESSURE  OF  FLUIDS  ON   PHYSICAL  LINES.  17 

But  when  the  lines  are  differently  situated  in  the  fluid,  the  compa- 
rison of  their  relative  pressures  requires  a  more  particular  exemplifi- 
cation; for  which  purpose  take  the  following  example. 

32.  EXAMPLE  3.  Two  physical  straight  lines,  whose  lengths  are 
respectively  18  and  27  feet,  are  immersed  in  the  same  fluid,  in  such 
a  manner  that  their  upper  extremities  are  just  in  contact  with  its 
surface,  and  the  angles  which  they  make  with  the  horizon  are  respec- 
tively equal  to  42  and  29  degrees  ;  what  is  the  pressure  on  the  longer 
line,  supposing  that  on  the  shorter  to  be  expressed  by  the  number 
78.54?. 

If  we  convert  the  preceding  analogy  for  the  oblique  lines  of  different 
inclinations  into  an  equation,  by  making  the  product  of  the  mean 
terms  equal  to  the  product  of  the  extremes,  we  shall  obtain 


Now,  by  assimilating  the  several  quantities  in  this  equation  to  the 
lines  in  the  foregoing  diagram,  and  according  to  the  conditions  of  the 
question,  it  appears  that  p'  is  the  required  quantity,  all  the  rest  being 
given  ;  therefore,  let  both  sides  of  the  equation  be  divided  by  /2  sin.  0, 
and  we  shall  obtain  t 

,  _  p  /*  sin.  <f 
/2sin.</>  ' 

But  it  is  a  well-known  principle  in  the  arithmetic  of  sines,  that  to 
divide  by  the  sine  of  any  arc,  is  equivalent  to  multiplying  by  the 
cosecant  of  that  arc  ;  hence  we  have 


, 

p'=—(p  sin.  0'  cosec.</>). 


Let  therefore  the  numerical  values,  as  proposed  in  the  example,  be 
substituted  for  the  respective  symbols  in  the  above  equation,  and  we 

shall  obtain 

.972 

»'  =  —  (78.54  sin.  29°  cosec.42°)  ; 
182 

now,  the  natural  sine  of  29°,  according  to  the  Trigonometrical  Tables, 
is  .48481,  and  the  natural  cosecant  of  42°  is  1.49447  ;  therefore,  by 
substitution,  we  get 

if  —  ?Z!x  78.54  X  .48481  X  1.49447  —  128  nearly  ; 

182 

consequently  the  pressures  on  the  inclined  lines,  are  to  one  another  as 
the  numbers  78.54  and  128;  but  had  the  inclinations  been  equal, 
the  comparative  pressures  would  have  been  as  78.54  to  176.72  very 
nearly. 

VOL.  i.  c 


18 


2.  OF  THE  PRESSURE  OF  FLUIDS  THAT  ARE  NON  -  ELASTIC  UPON 
RIGHT  ANGLED  PARALLELOGRAMS  CONSIDERED  AS  INDEPENDENT 
PLANES  IMMERSED  IN  FLUIDS. 

PROBLEM  III. 

33.  A  right  angled  parallelogram  is  immersed  in  a  quiescent 
fluid,  in  such  a  manner,  that  one  of  its  sides  is  coincident  with 
the  surface,  and  its  plane  inclined  to  the  horizon  in  a  given 
angle : — 

It  is  required  to  determine  the  pressure  perpendicular  to  the 

plane,  both  when  it  is  inclined  to  the  surface  of  the  fluid,  and 

when  it  is  perpendicular  to  it,  the  nature  of  the  fluid,  and 

consequently  its  specific  gravity,  being  known* 

Let  A  B  c  D  represent  a  vertical  section  of  a  volume  of  incompressible 

fluid  in  a  state  of  equilibrium,  of  which  A  B  E  F  is  the  surface,  and 

consequently  parallel  to  the  horizon ;    let  a  b  c  d  be  a  rectangular 

plane  immersed  in  the  fluid,  in  such  a  manner  that  the  upper  side  a  b 

coincides  with  the  surface,  and  the  plane  abed  is  inclined  to  the 

horizon  in  a  given  angle. 

Draw  the  diagonal  a  c,  which  bisect 
in  m,  and  through  m  the  centre  of  gra- 
vity of  the  parallelogram,  draw  mn 
parallel  to  ad  or  be,  meeting  ab  the 
line  of  common  section  perpendicularly 
in  the  point  n. 

In  the  horizontal  plane  A  BE  F,  and 
through  the  point  n,  draw  nr  also  at  right  angles  to  ab,  and  from 
m  the  centre  of  gravity  of  the  immersed  plane  abed,  let  fall  the 
perpendicular  m  r  ;  then  is  the  angle  m  n  r  the  inclination  of  the  plane 
to  the  horizon,  and  rm  the  perpendicular  depth  of  its  centre  of  gravity 
below  the  upper  surface  of  the  quiescent  fluid. 
Put  b  =  ab,  the  horizontal  breadth  of  the  immersed  parallelogram, 
/  zn  a  d  or  b  c,  the  immersed  length, 
d-=rm,  the  perpendicular  depth  of  the  centre  of  gravity, 
0  —  mn  r,  the  inclination  of  the  plane  to  the  horizon, 
p  =  the  pressure  on  the  plane  perpendicularly  to  its  surface, 
and  5  —  the  specific  gravity  of  the  fluid. 

*  By  the  pressure  upon  any  plane  or  curvilineal  surface,  is  always  understood 
the  aggregate  of  all  the  pressures  upon  every  point  of  those  surfaces,  estimated  in 
directions  perpendicular  to  them  at  each  point,  no  part  heing  lost  by  obliquity  of 
direction. 


OF  THE  PRESSURE  OF  FLUIDS  ON  RIGHT  ANGLED  PARALLELOGRAMS.    19 

Then,  because  the  point  m  is  at  the  middle  of  «e,  and  mn  parallel 
to  ad,  it  follows,  that  m  win  \  I;  and  by  reason  of  the  right-angled 
triangle  m  r  n,  we  have,  from  the  principles  of  Plane  Trigonometry, 

rm~d—^l  sin.^  ; 

consequently,  the  entire  pressure  upon  the  plane  perpendicularly  to 
its  surface,  is  expressed  by 

p  ~\  b  I*  s  sin.  (f>. 

This  is  manifest  from  Problem  I.  (art.  22),  for  \l  sin.  §  expresses 
the  perpendicular  depth  of  the  centre  of  gravity,  and  b  I  the  area  of 
the  surface  pressed  ;  therefore,  the  solidity  of  the  fluid  column  is 

\  I  sin.  0  X  bl—%bl*  sin.  0, 

and  since  s  denotes  the  specific  gravity  of  the  fluid,  the  weight  of  the 
column  is 

|  I  sin.  <j>XblXs  —  %bl*s  sin.  <f> ; 

but  the  perpendicular  pressure  upon  the  plane,  is  equal  to  the  weight 
of  the  fluid  column  ;  therefore,  we  obtain 

p~  J6Zsssin.  0.  (7). 

When  the  plane  of  the  immersed  rectangle  is  perpendicular  to  the 
surface  of  the  fluid,  we  have  0  in  90°,  and  sin.^>  zz:  1  ;  consequently, 
by  substitution,  the  above  equation  becomes 

P  —  iblzs.  (8). 

These  equations  are  sufficiently  simple  in  their  form  for  practical 
application,  and  we  shall  show  hereafter,  that  they  are  extremely 
useful  in  many  important  cases  of  hydrostatical  construction. 

34.  The  practical  rules  derived  from  these  equations,  for  determining 
the  pressure  in  the  particular  cases,  may  be  expressed  as  follows. 

1.  When  the  plane  is  oblique  to  the  horizon.  (Eq.  7). 

RULE.  Multiply  the  square  of  the  immersed  length  of  the 
plane,  by  the  horizontal  breadth  drawn  into  the  specific 
gravity  of  the  fluid,  and  again  by  the  natural  sine  of  the 
angle  of  inclination,,  and  half  the  product  will  give  the 
pressure  sought. 

2.  When  the  plane  is  perpendicular  to  the  horizon.          (Eq.  8). 

RULE.  Multiply  the  square  of  the  immersed  length  of  the 
plane,  by  the  horizontal  breadth  drawn  into  the  specific 
gravity  of  the  fluid,  and  half  the  product  will  give  the 
pressure  sought. 

35.  EXAMPLE  4.  A  rectangular  parallelogram,  whose  sides  are  re- 
spectively 1 8  and  3  feet,  is  immersed  in  a  quiescent  body  of  water,  in 
such  a  manner,  that  its  shorter  side  is  in  contact  with  the  surface, 

c  2 


20  OF  THE  PRESSURE  OF  FLUIDS 

and  its  plane  inclined  to  the  horizon  in  an  angle  of  68  degrees; 
required  the  pressure  which  it  sustains,  both  in  the  inclined  and  the 
perpendicular  position  ? 

In  this  example  the  area  of  the  parallelogram  is  1 8  X  3  ==  54  square 
feet,  and  the  longer  side  is  that  which  is  immersed  downwards  in  the 
fluid;  therefore,  according  to  the  rule  for  the  oblique  position,  the 
solidity  of  the  column  by  which  the  pressure  is  propagated,  becomes 
3  X  182  X  s  X  1  sin.  68°  =  486  X  s  sin.  68°. 

Now,  in  the  case  of  water,  the  specific  gravity  is  represented  by 
unity,  and  by  the  Trigonometrical  Tables,  the  natural  sine  of  68 
degrees,  is  0.92718 ;  consequently,  by  substitution,  the  pressure 
becomes 

^  =  486x  .92718  =  450.60948; 

the  pressure  here  obtained,  however,  is  estimated  in  cubic  feet  of 
water ;  but  in  order  to  have  it  expressed  in  a  more  appropriate  and 
definite  measure,  it  becomes  necessary  to  compare  it  with  some 
weight ;  now,  it  has  been  found  by  experiment,  that  the  weight  of  a 
cubic  foot  of  water  is  very  nearly  equal  to  62 1  Ibs.  avoirdupois; 
therefore,  the  absolute  pressure  upon  the  plane,  is 

p  —  450.60948  X  62.5  =  28163.0925  Ibs. 

36.  Let  the  dimensions  of  the  plane  remain  as  in  the  preceding 
case,  which  condition  is  supposed  in  the  example ;  then,  the  pressure 
on  its  surface,  when  perpendicular  to  the  horizon,  is 

p  —  3  x  18  X  18  X  1  X  J  =486  cubic  feet 

of  water ;  but  we  have  stated  above,  that  the  weight  of  one  cubic  foot 
is  equal  to  62  \  Ibs. ;  therefore,  we  have 

p  —  486  X  62^  zz  30375  Ibs. ; 

consequently,  the  pressures  on  the  plane  in  the  two  positions,  are  to  one 
another  as  the  numbers  450.60948  and  486,  when  expressed  in  cubic 
feet  of  water ;  but  when  expressed  in  pounds  avoirdupois,  they  are  as 
the  numbers  28163.0925  and  30375. 

37.  If  the  longer  side  of  the  rectangular  parallelogram  were  coin- 
cident with  the  surface  of  the  fluid,  while  its  plane  is  obliquely  inclined 
to  the  horizon  ;  then,  the  formula  for  the  pressure  perpendicular  to  its 
surface  becomes 

p=.  ±b*ls  sin.  <f>.  (9). 

But  if  the  plane  of  the  parallelogram,  instead  of  being  inclined  to 
the  horizon,  or  which  is  the  same  thing,  to  the  surface  of  the  fluid, 
were  immersed  perpendicularly  to  it ;  then,  0  zz  90°,  and  sin.  0  zz  1  ; 
hence,  the  formula  for  the  pressure  becomes 

p  =*#**.  (10). 


ON  RECTANGULAR  PARALLELOGRAMS.  2l 

Therefore,  by  retaining  the  data  of  the  preceding  example,  the 
absolute  pressure  on  the  plane  in  the  oblique  position,  is 

p=3*  X  18  x  62.5  X  \  X  .92718  =  4693.84875  Ibs. 
But  when  the  plane  is   perpendicularly  immersed,  the  absolute 
pressure  on  its  surface  is 

p=3*  X  18  X  62.5  X  J  =  5062Jlbs. 

COROL.  1 .  Hence,  the  pressures  on  the  plane  in  the  oblique  and  per- 
pendicular positions,  are  to  one  another  as  the  numbers  4693.84875 
and  5062 \ ;  but  in  order  to  compare  the  pressures  under  the  same 
conditions,  when  the  shorter  and  longer  sides  of  the  parallelogram 
are  respectively  in  contact  with  the  surface  of  the  fluid,  we  have  as 
follows,  viz. 

2.  When  the  shorter  side  of  the  parallelogram  is  horizontal,  the 
absolute  pressure  in  the  inclined  position  is  28163.0925  Ibs. ;  but 
when  the  longer  side  is  horizontal,  the  absolute  pressure  is  4693. 84875 
Ibs. ;  consequently,  the  absolute  pressures  in  the  two  cases  are  to 
one  another  as  6  to  1 . 

3.  Again,  when  the  shorter  side  of  the  parallelogram  is  horizontal, 
the  pressure  in  the  perpendicular  position  is  30375  Ibs. ;  and  when 
the  longer  side  is  horizontal,  the  pressure  is  5062 1  Ibs. ;  therefore,  the 
pressures  in  these  two  cases  are  to  one  another  as  6  to  1 ,  the  same  as 
before ;  from  which  we  infer,  that  the  quantity  of  inclination  affects 
only  the  magnitude  of  the  pressures,  and  that  in  so  far  as  it  changes 
the  position  of  the  centre  of  gravity,  but  it  has  no  effect  upon  the 
ratio  ;  therefore,  if  the  plane  were  to  vibrate  round  its  shorter  and 
longer  sides  respectively  as  axes,  the  pressures  on  its  surface,  in  the 
two  cases,  would  be  to  one  another  in  a  constant  ratio. 

3.  OF  THE  AGGREGATE  PRESSURE  EXERTED  BY  THE  FLUID  ON  THE 
IMMERSED  PARALLELOGRAM,  AND  ON  EACH  0¥  THE  CONSTITUENT 
TRIANGLES  FORMED  BY  ITS  DIAGONAL. 

PROBLEM  IV. 

38.  Suppose  the  parallelogram  to  be  placed  under  the  same 
circumstances  as  in  the  preceding  problem,  and  let  it  be  bisected 
by  one  of  its  diagonals : — 

It  is  required  to  determine  the  aggregate  pressure  exerted 
by  the  fluid,  in  a  direction  perpendicular  to  the  surface  of 
each  triangle  into  which  the  diagonal  divides  the  parallelo- 
gram, and  to  compare  the  pressures  on  the  two  triangles. 


22  OF  THE  AGGREGATE  PRESSURE  OF  FLUIDS 

Let  A  B  c  D  represent  a  vertical  section  of  a  mass  or  collection  of 
quiescent  fluid,  contained  by  the  walls  or  embankments  indicated  by 
the  shaded  boundary  ;  and  let  A  B  E  F  be  the  horizontal  surface  of  the 
fluid,  with  which  one  side  of  the  immersed  rectangle  is  supposed  to  be 
coincident. 

Now,  suppose  abdc,  to  be  the  immersed  rectangle,  and  draw 
the  diagonal  b  d;  then  are  a  b  d  and  bdc 
the  triangles,  into  which  the  parallelogram 
abed  is  divided  by  the  diagonal  b  d,  and 
for  which  the  pressures  are  required  to  be 
investigated. 

Draw  the  diagonal  a  c,  which  divide  into 
three  equal  portions  in  the  points  m  and  n  > 
then  are  m  and  n  respectively  the  centres  of  gravity  of  the  constituent 
triangles  a  b  d  and  bdc. 

Through  the  points  m  and  n,  and  parallel  to  a  d  or  b  c,  the  immersed 
sides  of  the  figure,  draw  me  and  nf  meeting  a  b  perpendicularly  in 
the  points  e  and/;  then,  through  the  points  e  and  /thus  determined, 
and  in  the  plane  of  the  fluid  surface,  draw  er  and/s  respectively 
perpendicular  to  ab;  then  are  the  angles  mer  and  nfs  equal  to 
one  another,  and  each  of  them  is  equal  to  the  angle  which  the  plane 
of  the  immersed  parallelogram  makes  with  the  surface  of  the  fluid. 

From  m  and  n,  the  centres  of  gravity  of  the  triangles  a  b  d  and 
b  d  c,  demit  the  lines  m  r  and  n  s  respectively  perpendicular  to  e  r  and 
fs;  then  are  rm  and  sn  the  perpendicular  depths  of  the  centres  of 
gravity. 

Put  b  —  ab,  the  horizontal  breadth  of  the  immersed  parallelogram, 
/  =  ad  or  be,  the  immersed  or  downward  length, 
d  =  rm,  the  perpendicular  depth  of  the  centre  of  gravity  of  the 

triangle  abd, 
S  ~  sn,  the  perpendicular  depth  of  the  centre  of  gravity  of  the 

triangle  bdc, 

D—  ac  or  ba,  the  diagonal  of  the  parallelogram, 
ty~mer,  or  nfs,  the  inclination  of  the  plane  to  the  surface  of 

the  fluid, 

Pzz  the  whole  pressure  on  the  parallelogram  abed, 
p  m the  pressure  on  the  triangle  abd, 
y—the  pressure  on  the  triangle  bdc, 
and  s  —  the  specific  gravity  of  the  fluid. 

Then,  because  the  parallelogram  abed  is  rectangular,  the  triangle 


ON  IMMERSED  RIGHT  ANGLED  PARALLELOGRAMS.  23 

udc  is  right  angled  at  d;   therefore,  by  the  property  of  the  right- 
angled  triangle,  we  have 


aczz  -v/  ad* 
or  by  employing  the  appropriate  symbols,  we  have 

Dizrv/r-M2. 

But,  according  to  the  construction,  and  by  the  nature  of  the  centre 
of  gravity,  we  have 

am"=z  -i-ac,  and  an='%ac, 

or  symbolically,  we  obtain 

am  —  -ly  Z2-f  62,  and  an  zr  f^/^-j-fe2. 

Now,  by  reason  of  the  parallel  lines  em,  fn,  and  be,  the  triangles 
a  em,  afn,  and  a  be,  are  similar  among  themselves  ;  consequently,  by 
the  property  of  similar  triangles,  we  have 

ac  :  be  :  :  an  :fn  :  :  am  :  em; 

therefore,  by  separating  the  analogies,  and  employing  the  symbols, 
it  is 

D:Z::|V~F+&2:/w, 
and  again,  we  have 

D  :  I  ::  i\/^  +  &  :  em'> 

from  these  analogies,  therefore,  we  obtain  fn  n:  |7,  and  em~\l\ 
which  is  otherwise  manifest  by  drawing  the  dotted  lines  mt  and  nu. 

Now,  in  the  right  angled  triangles  erm  and  fsn,  there  are  given 
the  hypothenuses  em  and  fn,  and  the  equal  angles  mer  and  nfs,  to 
find  rm  and  sn,  the  perpendicular  depths  of  the  centres  of  gravity; 
consequently,  by  Plane  Trigonometry,  we  have,  from  the  triangle 
mer, 

rad.  :  sin.  $  :  :  $1  :  d, 
and  from  the  triangle  nfs,  it  is 

rad.  :  sin.  <j>  :  :  %l  :  £, 
and  since  radius  is  equal  to  unity,  these  analogies  become 

d-=.\l  sin.  0,  and  £  zz  f  I  sin,  0. 

But  according  to  Inf.  2,  Proposition  (A),  the  pressure  sustained  by 
each  triangle,  in  a  direction  perpendicular  to  its  surface, 

Is  expressed  by  the  product  of  its  area,  drawn  into  the 
perpendicular  depth  of  the  centre  of  gravity. 

Now,  the  area  of  each  triangle  is  manifestly  equal  to  half  the  area 
of  the  given  parallelogram,  and  by  the  principles  of  mensuration,  the 
area  of  the  rectangular  parallelogram  is  equal  to  the  product  of  its 
two  dimensions  ;  that  is,  of  the  length  drawn  into  the  breadth  ;  there- 
fore, we  have  for  the  pressure  on  the  triangle  «  bd, 


24  OF  THE  AGGREGATE  PRESSURE  OF  FLUIDS 


sn.  0, 
and  in  like  manner,  the  pressure  on  the  triangle  bdc  is 

p'  ~\bl*  sin.  <f>. 

These  equations,  however,  express  the  pressures  simply  by  the 
magnitude  of  a  fluid  column,  whose  base  is  the  area  pressed,  and 
whose  altitude  is  equal  to  the  depth  of  the  centre  of  gravity  below  the 
upper  surface  of  the  fluid.  In  order,  therefore,  to  have  the  pressures 
expressed  in  general  terms,  the  specific  gravity  of  the  fluid  must  be 
taken  into  the  account;  in  which  case,  the  pressure  on  the  triangle 
abd  becomes 

p=±bl*ssm.<t>,  (11). 

and  the  pressure  on  the  triangle  bdc  is 

p'i=i&Z2ssin.;>.  (12). 

COROL.  Hence  it  appears,  that  the  pressure  perpendicular  to  the 
plane  of  a  triangle,  when  its  vertex  is  upwards  and  coincident  with 
the  surface  of  the  fluid,  is  double  the  pressure  on  the  same  triangle, 
when  its  base  is  upwards,  and  placed  under  the  same  circumstances. 

39.  If  the  immersed  plane  be  perpendicular  to  the  surface  of  the 
fluid,  then  <b  nz  90°,  and  sin.  ty  ml;  therefore,  by  substitution,  the 
preceding  equations  become 

p  =  ±bl*s,  andp'=ibr~s; 

here  again,  the  pressure  in  the  one  case  is  double  the  pressure  in  the 
other,  and  the  same  thing  will  obtain,  whatever  may  be  the  inclination 
of  the  plane,  provided  only  that  a  b  coincides  with  the  surface  of  the 
fluid  ;  for  then,  the  perpendicular  depths  of  the  centres  of  gravity  will 
vary  in  a  given  ratio. 

When  the  immersed  plane  is  a  square,  that  is,  when  b  and  I  are 
equal  to  one  another,  the  equations  for  the  pressures  in  the  oblique 
position  become 

p  :z:  %bss  sin.  0,  and  p'  —  ±b3s  sin.  0, 

and  when  the  plane  is  perpendicular  to  the  surface  of  the  fluid,  we 
have 

p  =  £  bs  s,  and  p'  —  |  b3s* 

Since  the  aggregate  pressure  upon  the  plane  is  equal  to  the  sum 
of  the  pressures  on  the  constituent  triangles,  the  expression  for  the 
aggregate  pressure  in  the  oblique  position,  becomes  in  the  case  of  a 
rectangle 

P  =p  +pf  ;  that  is,  P  z=  ±bl*s  sin.  0  -f  ±bl*s  sin.  0  =  £bl*s  sin.  0. 
COROL.  Hence  it  appears,  that  the  pressures  on  the  constituent 
triangles  and  that  on  the  entire  plane,  are  to  one  another  as  the 
numbers  1  ,  2  and  3  ;  and  the  same  thing  obtains  in  the  case  of  a 
square,  whatever  may  be  the  inclination  of  the  plane. 


ON   IMMERSED   RIGHT  ANGLED  PARALLELOGRAMS.  25 

40.  The  practical  rules  for  calculating  the  pressures  on  the  triangles, 
as  deduced  from  the  equations  (11)  and  (12)  are  as  follows. 

1.  When  the  base  of  the  triangle  is  coincident  with  the  surface. 

RULE.  Multiply  the  square  of  the  immersed  length,  or  the 
perpendicular  of  the  triangle,  by  the  base  drawn  into  the 
specific  gravity  of  the  fluid,  and  again  by  the  natural  sine  of 
the  plane's  inclination,  and  one  sixth  part  of  the  product  will 
express  the  whole  pressure  upon  the  triangle.  (Eq.  11). 

2.  When  the  vertex  of  the  triangle  is  coincident  with  the  surface. 

RULE.  Multiply  the  square  of  the  perpendicular  of  the 
triangle,  by  the  base  drawn  into  the  specific  gravity  of  the 
fluid,  and  again  by  the  natural  sine  of  the  plane's  inclination, 
and  one  third  of  the  product  will  express  the  whole  pressure 
on  the  triangle.  (Eq.  12). 

41.  EXAMPLE  5.    A  rectangular  parallelogram,  whose   sides  are 
respectively  26  and  14  feet,  is  immersed  in  a  cistern  of  water,  in  such 
a  manner,  that  its  shorter  side  is  coincident  with  the  horizontal  sur- 
face ;  what  will  be  the  pressure  on  each  of  the  triangles,  into  which 
the  parallelogram  is  divided  by  its  diagonal,  supposing  its  plane  to  be 
inclined  to  the  surface  of  the  fluid  in  an  angle  of  56°  35'  ? 

Here,  by  the  rule,  we  have 

p=  262  X  14  X  .83469  X  i=  1316.58436  cubic  feet 
of  water;  but  one  cubic  foot  of  water  weighs  62  J  Ibs. ;  therefore,  to 
express  the  pressure  in  Ibs.,  we  have 

p=  1316.58436  X  62J  =  82286.5225  Ibs. 

The  pressure  which  we  have  just  obtained,  refers  to  that  portion  of 
the  parallelogram  which  has  its  base  coincident  with  the  surface  of 
the  fluid;  that  is,  to  the  triangle  abd,  and  the  pressure  on  the  other 
portion,  or  the  triangle  bdc,  is  determined  as  follows. 

/z=:262  X  14  X  .83469  X  i  =  2633. 16872  cubic  feet  of  water; 

or  to  express  the  pressure  in  Ibs.  we  have 

p'=  2633. 1687  X  62.5=  164573.045  Ibs. 

If  the  plane  of  the  immersed  parallelogram  were  perpendicular  to 
the  surface  of  the  fluid,  the  pressures  on  the  triangles  abd  and  bdc 
would  be  respectively  as  follows. 
p=:262X  14  X  62 J  X  i  =  98583ilbs.,andy  —  26*  X  14  X  62|X 

i=  197 166|  Ibs. 

COROL.  The  circumstance  of  the  aggregate  pressure  on  the  paral- 
lelogram, being  equal  to  the  sum  of  the  pressures  on  the  constituent 
triangles,  furnishes  a  very  simple  and  elegant  method  of  determining 


26  OF  THE  AGGREGATE  PRESSURE  OF  FLUIDS 

the  centre  of  gravity;  which  method,  in  so  far  as  respects  plane 
figures  of  particular  forms,  may,  in  many  instances,  be  very  advan- 
tageously applied. 

It  would  be  foreign  to  our  present  purpose  to  enter  into  a  detail  of 
the  method  alluded  to  in  this  place  ;  nevertheless,  for  the  satisfaction 
of  our  readers,  we  shall  briefly  introduce  it,  not  being  aware  that  it 
has  been  suggested  by  any  other  writer  in  ancient  or  modern  times. 

PROBLEM  B. 

42.  The  base  and  perpendicular  of  a  right  angled  triangle 
being  supposed  known  :  — 

It  is  required  to  determine  the  position  of  its  centre  of 
gravity,  or  that  point  on  which,  if  the  surface  were  supported, 
it  would  remain  at  rest  in  any  position. 

Let  ABC  be  the  triangle  given,  of  which  it  is  required  to  determine 
the  centre  of  gravity. 

Complete  the  parallelogram  A  B  c  D,  by  drawing  the 
dotted  lines  AD  and  DC;  then,  because  the  entire 
pressure  on  the  parallelogram  ABCD,  is  equal  to  the 
sum  of  the  pressures  on  the  triangles  ABC  and  ACD; 
it  follows,  that  the  pressure  on  the  triangle  ABC,  is 
equal  to  the  difference  between  the  entire  pressure  on 
the  parallelogram,  and  that  on  the  triangle  ACD;  consequently,  we 
have,  by  retaining  the  foregoing  notation, 

p  =  P  —  pr  ;  that  is, 


But  it  has  been  elsewhere  demonstrated,  that  the  pressure  on  any 
surface,  is  expressed  by  the  area  of  that  surface,  drawn  into  the 
perpendicular  depth  of  its  centre  of  gravity  ;  consequently,  the  per- 
pendicular depth  of  the  centre  of  gravity,  must  be  equal  to  the 
pressure  divided  by  the  area  of  the  surface. 

Now,  in  the  present  instance  the  pressure  is  known,  and  since  by 
the  problem,  the  base  and  perpendicular  of  the  triangle  are  given,  its 
area  can  easily  be  found. 

Thus,  the  writers  on  mensuration  have  shown,  that  the  area  of  a 
triangle  is  equal  to  half  the  product  of  the  base  drawn  into  the  per- 
pendicular altitude  ;  consequently,  if  a  be  put  to  denote  the  area  of 
the  triangle  ABC,  we  shall  have 


therefore,  by  division,  the  perpendicular  depth  of  the  centre  of  gra- 
vity, is 


ON  DIFFERENT  SECTIONS  OF  PARALLELOGRAMS.  27 


And  in  like  manner  it  may  be  shown,  that  if  the  side  BC  were 
horizontal,  the  perpendicular  depth  of  the  centre  of  gravity  would  be 

d=±b; 

therefore,  take  T.m  and  BW  respectively  equal  to  one  third  of  BC  and 
BA,  and  through  the  points  m  and  n,  draw  mG  and  no  parallel  to  BA 
and  BC,  meeting  each  other  in  the  point  G  ;  then  is  G  the  position  of 
the  centre  of  gravity. 

The  intelligent  and  attentive  reader  will  readily  perceive  that  the 
above  determination  is  not  legitimate,  since  it  supposes  the  pressure 
upon  the  triangle  ABC  to  be  given  ;  now,  this  pressure  depends 
entirely  upon  the  position  of  the  centre  of  gravity,  and  consequently, 
the  problem  supposes  the  position  of  the  centre  of  gravity  of  the 
triangle  ADC  to  be  known;  the  principle,  however,  will  be  more 
distinctly  indicated  when  applied  to  other  figures,  where  the  above 
determination  may  be  admitted,  without  infringing  on  the  precepts  of 
scientific  propriety. 

4.     OF     THE    PRESSURE    OF     INCOMPRESSIBLE    FLUIDS    ON     DIFFERENT 
SECTIONS  OF    PARALLELOGRAMS    PARALLEL   TO    THE    HORIZON. 

PROBLEM  V. 

43.  A  rectangular  parallelogram  is  obliquely  immersed  in  an 
incompressible  and  non-elastic  fluid,  in  such  a  manner,  that  one 
side  is  just  coincident  with  the  surface  :  — 

It  is  required  to  compare  the  pressure  on  the  upper  and  the 
lower  portions,  supposing  the  parallelogram  to  be  bisected  by 
a  line  drawn  parallel  to  the  surface  of  the  fluid. 

Let  A  ED  represent  a  vessel  full  of  water,  or  some  other  non-elastic 
and  incompressible  fluid,  of  which  ABEF 
is  the  surface,  and  suppose  one  side  of  the 
vessel  to  be  removed,  exhibiting  the  fluid 
and  the  immersed  rectangle  as  represented 
by  A  B  c  D  and  abed. 

Bisect  the  parallelogram  abed  by  the 
straight  lines  ef  and  gh  respectively  parallel 
to  a  b  and  ad\  then  are  abfe  and  efcd, 
the  portions  on  which  the  pressures  are  to  be  compared,  and  gh  is  the 
line  in  which  the  centres  of  gravity  occur. 


28  OF  THE  PRESSURE  OF  INCOMPRESSIBLE  FLUIDS 

Draw  the  diagonals  d/and/a  meeting  the  straight  linear  A  in  the 
points  m  and  n  ;  then  are  m  and  n  the  centres  of  gravity  of  the  respec- 
tive portions  into  which  the  parallelogram  is  divided  by  the  line  ef. 

Through  the  point  g  and  in  the  plane  of  the  fluid  surface  ABEF, 
draw  gsr  at  right  angles  to  a  b,  and  the  angle  mgr  will  be  the  incli- 
nation or  obliquity  of  the  plane  ;  then,  through  the  points  m  and  n, 
draw  the  straight  lines  mr  and  ns  respectively  perpendicular  to  gsr, 
and  rm  and  sn  will  be  the  vertical  depths  of  the  centres  of  gravity 
below  the  upper  surface  of  the  fluid. 

Put  b=iab,  the  horizontal  breadth  of  the  given  parallelogram, 
/  zr  ad  or  be,  the  immersed  length  tending  downwards, 
d~rm,  the  vertical  depth  of  the  centre  of  gravity  of  the  lower 

portion  efcd, 
$  ~  sn,  the  vertical  depth  of  the  centre  of  gravity  of  the  upper 

portion  abfe, 

(j>=:mgr,  the  inclination  of  the  plane  to  the  surface  of  the  fluid, 
Pn=  the  pressure  on  the  whole  parallelogram  abed, 
p  —  the  pressure  on  the  lower  portion  efcd, 
and  p  —  the  pressure  on  the  upper  portion  abfe. 

Then,  because  the  straight  line  gh  is  bisected  in  0,  and  each  of  the 
portions  g  ©  and  h  ©  respectively  bisected  in  the  points  n  and  m  ; 
it  follows  that  g  n  zz  |,  and  gm  —  %  of  gh;  that  is 

g  n  nz  J  I,  and  g  m  —  j  /  ; 
consequently,  by  the  principles  of  Plane  Trigonometry,  we  have 

swrzrSzz  \l  sin.  <j>,  and  rm  —  d=%l  sin.  0; 

therefore,  since  the  area  of  each  portion  of  the  parallelogram  is 
expressed  by  \  bl,  the  pressure  on  each  portion  is  as  below,  viz. 
The  pressure  perpendicular  to  the  surface  a  bfe,  is 

p'zz:  ±blz  sin.  0, 
and  the  pressure  perpendicular  to  the  surface  efcd,  is 

p  m  %bl*  sin.0  ; 

consequently,  by  comparison,  the  pressures  on  the  upper  and  lower 
portions  of  the  parallelogram,  are  to  each  other  as  the  numbers  1  and 
3  ;  that  is 

pT:p::l:3. 

But  according  to  the  third  problem,  the  aggregate  pressure  sustained 
by  the  plane,  in  a  direction  perpendicular  to  its  surface,  is 


consequently,  the  pressures  on  the  two  portions  and  on  the  whole 
plane,  are  to  one  another  as  the  numbers  1,  3  and  4. 


ON  DIFFERENT  SECTIONS  OF  PARALLELOGRAMS.  29 

In  the  preceding  values  of  the  pressure,  it  is  supposed,  that  the 
specific  gravity  of  the  fluid  in  which  the  plane  is  immersed,  is  repre- 
sented by  unity,  which  is  true  only  in  the  case  of  water  ;  therefore,  in 
order  to  render  the  formulae  general,  we  must  introduce  the  symbol 
for  the  specific  gravity,  and  then  the  above  equations  become, 

1.  For  the  upper  half  of  the  parallelogram, 

p'=z±bl2  s  sin.  0.  (13). 

2.  For  the  lower  half  of  the  parallelogram, 

pz=f  bl*s  sin.  0.  (14). 

When  the  plane  is  perpendicularly  immersed  in  the  fluid,  or  when 
0  —  90°,  then  sin.  0  —  1,  and  the  equations  (13)  and  (14)  become 
p'  —  \bl^s,  andprzi&/25. 

In  which  equations  the  co-efficients  or  constant  quantities  remain ; 
therefore,  the  ratio  of  the  pressure  is  not  varied  in  consequence  of  a 
change  in  the  angle  of  inclination,  the  variation  takes  place  in  the 
magnitude  of  the  pressures  only,  and  not  in  the  ratio,  the  magnitude 
increasing  from  zero,  where  the  plane  is  horizontal,  to  its  maximum 
where  the  plane  is  perpendicular. 

44.  The  practical  rules  for  calculating  the  pressures,  as  derived  by 
the  equations  (13)  and  (14)  are  as  follows. 

1.  For  the  pressure  on  the  first,  or  upper  half  of  the  paral- 

lelogram. 

RULE.  Multiply  the  square  of  the  immersed  length,  by  the 
breadth  drawn  into  the  specific  gravity  of  the  fluid,  and  again 
by  the  natural  sine  of  the  angle  of  elevation ;  then,  one  eighth 
part  of  the  product  will  be  the  pressure  sought.  (Eq.  13). 

2.  For  the  pressure  on  the  second,  or  lower  half  of  the  paral- 

lelogram. 

RULE.  Multiply  the  square  of  the  immersed  length,  by  the 
breadth  drawn  into  the  specific  gravity  of  the  fluid,  and  again 
by  the  natural  sine  of  the  angle  of  elevation;  then,  three 
eighths  of  the  product  will  be  the  pressure  sought.  (Eq.  14). 

45.  EXAMPLE  6.    A  rectangular  parallelogram,  whose  sides   are 
respectively  20  and  30  feet,  is  immersed  in  a  cistern  of  water,  in  such 
a  manner,  that  its  breadth  or  shorter  side  is  just  coincident  with  the 
surface ;  required  the  pressures  on  the  upper  and  lower  portions  of 
the  plane,  supposing  it  to  be  bisected  by  a  line  drawn  parallel  to  the 
horizon,  the  inclination  of  the  plane  being  59°  38'  ? 

Here,  by  operating  according  to  the  rule,  we  have 
p  —  30*  X  20  X  sin.  59°  38'  X  i  ; 


30  OF  RECTANGULAR  PARALLELOGRAMS  DIVIDED  INTO 

but  by  the  Trigonometrical  Tables,  the  natural  sine  of  59°  38'  is 
.86281 ;  therefore,  we  have 

p'=i  302  X  20  x  .86281  X  i=  1941.3225, 

and  again,  by  a  similar  process  we  have 
p=  302  X  20  X  .86281  x  1  =  5823.9675. 

Now,  these  results  are  obviously  expressed  in  cubic  feet  of  water, 
for  they  are  respectively  equal  to  the  solidity  of  a  fluid  column,  whose 
base  is  equal  to  one  half  the  given  parallelogram,  and  whose  altitude, 
in  the  one  case,  is  expressed  by  \l  sin.  $  rz  7.5  x  .86281,  and  in  the 
other  by  |J  sin.  <j>=:  22.5  X  .86281 ;  but  the  weight  of  one  cubic  foot 
of  water  is  .equal  to  62J  Ibs. ;  consequently,  the  pressures  expressed 
in  Ibs.  avoirdupois,  are 

pf  =  1941.3225  X  62.5  =  121332.65625  Ibs. 
and;?  =5823.9675  X  62.5  =  363997.96875  Ibs. 
When  the  plane  is  perpendicular  to  the  surface  of  the  fluid,  the 
pressure  is  a  maximum,  and  in  that  case,  the  respective  pressures  on 
the  two  portions  of  the  parallelogram,  are 

p'  =  302  x  20  X  62.5  x  I  =  140625  Ibs. 
and  jo  rz302  X  20  x  62.5  x  |zr  421875  Ibs. 

and  the  sum  of  these,  is  obviously  equal  to  the  whole  pressure  on  the 
plane ;  hence  we  get 

P  =  140625  +  421875  ==  562500  Ibs. 

COROL.  If  the  plane,  instead  of  being  immersed  in  the  fluid,  as  we 
have  hitherto  supposed  it  to  be,  should  only  be  in  contact  with  it,  as 
we  may  conceive  the  surface  of  a  vessel  to  be  in  contact  with  the  fluid 
which  it  contains ;  then,  the  pressure  will  be  the  same  ;  for  the  quan- 
tity of  pressure  at  any  given  depth  upon  a  given  surface,  is  always  the 
same,  whether  the  surface  pressed  be  immersed  in  the  fluid  or  just  in 
contact  with  it,  and  whether  it  be  parallel  to  the  horizon,  or  placed  in 
a  position  perpendicular  or  oblique  to  it. 

5.  OF  RECTANGULAR  PARALLELOGRAMS  IMMERSED  IN  NON -ELASTIC 
FLUIDS,  AND  DIVIDED  INTO  TWO  PARTS  SUCH  THAT  THE  PRESSURES 
OF  THE  FLUID  UPON  THEM  SHALL  BE  EQUAL  BETWEEN  THEMSELVES. 

PROBLEM  VI. 

46.  A  rectangular  parallelogram  is  obliquely  immersed  in  an 
incompressible  and  non-elastic  fluid,  in  such  a  manner,  that  one 
side  is  just  coincident  with  the  surface : — 

It  is  required  to  divide  the  parallelogram  into  two  parts  by 
a  line  drawn  parallel  to  the  horizon,  so  that  the  pressures  on 
the  two  parts  shall  be  equal  to  one  another. 


TWO  PARTS  SUSTAINING  EQUAL  PRESSURES.  31 

Let  A  E  D  represent  a  rectangular  vessel  filled  with  water,  or  some 
other  incompressible  and  non-elastic  fluid, 
of  which  ABEF  is  the  surface,  and  ABCD 
the  fluid  as  exhibited  in  the  vessel,  on  the 
supposition  that  one  of  its  upright  sides  is 
removed. 

Let  abed  be  the  immersed  parallelo- 
gram, having  its  upper  side  a  b  coincident 
with  the  surface  of  the  fluid,  and  its  plane 
tending  obliquely  downwards  in  the  given  angle  of  inclination. 
Bisect  a  b  in  g,  and  through  g  draw  the  v  straight  line  gh  parallel 
to  ad  or  be,  the  side  of  the  given  immersed  rectangle,  and  let  ef 
parallel  to  ab  or  cd,  denote  the  line  of  division;  then,  by  the 
problem,  the  pressure  on  the  rectangle  abfe,  is  equal  to  the  pressure 
on  the  rectangle  efcd. 

Draw  the  diagonals  dfo.ud.fa,  cutting  the  bisecting  line  gh  in  the 
points  m  and  n ;  then  are  m  and  n  respectively,  the  places  of  the 
centres  of  gravity  of  the  spaces  efcd  and  abfe.  Through  the  point 
g  and  in  the  plane  of  the  fluid  surface,  draw  g  r  at  right  angles  to  a  b, 
and  from  m  and  n  demit  the  straight  lines  mr  and  ns,  respectively 
perpendicular  to  the  horizontal  line  gr ;  then  are  sn  and  rm  the 
perpendicular  depths  of  the  centres  of  gravity  of  the  rectangles  a  bfe 
and  efcd  on  which  the  pressures  are  equal. 
Put  b  rz  ab,  the  horizontal  breadth  of  the  proposed  rectangular  plane, 

Z=n  ad  or  be,  the  immersed  length  of  ditto,  or  that  which  tends 
downwards, 

d  =  rm,  the  vertical  depth  of  the  centre  of  gravity  of  the  lower 
portion  efcd, 

$i=sn,  the  vertical  depth  of  the  centre  of  gravity  of  the  upper 
portion  abfe, 

0z=  mgr,  the  inclination  of  the  plane  to  the  surface  of  the  fluid, 

P=r  the  pressure  on  the  entire  parallelogram, 

p  =z  the  pressure  on  each  of  the  portions  into  which  the  paral- 
lelogram is  divided, 

s  rr  the  specific  gravity  of  the  fluid, 
and  x  —  ae,  the  immersed  length  of  the  upper  portion  abfe. 

Then  is  ed—l  —  x;  gn  —  \x,  and  gm  —  #  -|-  \  (I  —  x)  ;  con- 
sequently, by  the  principles  of  Plane  Trigonometry,  we  have 

57i^rS=  |  a  sin.  0,  and  rm  —  d—  [x  -\-  %(l —  #)J  sin.0, 
and  moreover,  by  the  principles  of  mensuration,  the  area  of  the  upper 
portion  is  expressed  by  b  x,  and  that  of  the  lower  portion  by  b  (I — x) ; 


32  OF  RECTANGULAR  PARALLELOGRAMS  DIVIDED  INTO 

consequently,  the  absolute  pressures  as  referred  to  the  respective 
portions,  are 

but  according  to  the  conditions  of  the  problem,  these  pressures  are 
equal  to  one  another  ;  hence  by  comparison,  we  have 

and  this  by  a  little  farther  reduction,  becomes 

2*2  =  Z2.  (15). 

47.  The  equation  in  its  present  form,  suggests  a  very  simple  geometri- 
cal construction  ;  for  since  Z2  is  equal  to  twice  z2,  it  is  manifest,  that  Z 
is  the  diagonal  of  a  square  of  svhich  the  side  is  x  ;  hence 
the  following  process. 

Draw  the  straight  line  AB  of  the  same  length  as  the 
side  of  the  given  parallelogram,  and  bisect  AB  per- 
pendicularly in  c  by  the  straight  line  CD;  on  AB  as  a 
diameter,  and  about  the  centre  c  describe  the  semi- 
circle ADB,  cutting  the  straight  line  CD  in  the  point 
D;  join  AD,  and  about  the  point  A  as  a  centre,  with 
the  distance  A  D,  describe  the  arc  DE  meeting  AB  in  E  ;  then  is  F  the 
point  of  division  sought. 

Upon  A  B  and  with  the  given  horizontal  breadth,  describe  the  paral- 
lelogram ABHG,  and  through  the  point  E,  draw  the  straight  line  EF 
parallel  to  AG  or  BH  ;  then  will  EF  divide  the  parallelogram,  exactly 
after  the  manner  required  in  the  problem.  The  truth  of  the  above 
construction  is  manifest ;  for  by  the  property  of  the  right  angled 
triangle,  we  have 

A  D2  —  A  c2  -J-  c  D?  ; 

but  AC  is  equal  toco,  these  being  radii  of  the  same  circle,  hence 
we  get 

AD2  =2  AC2; 

but  by  the  construction,  we  have 

AE  m  AD  ; 
consequently,  by  substitution,  it  is 

AE2Z=  2  AC2, 

and  doubling  both  sides  of  the  equation,  we  get 

now  AC  is  equal  to  one  half  of  AB,  and  it  is  demonstrated  by  the 
writers  on  geometry,  that  the  square  of  any  quantity  is  equal  to  four 
times  the  square  of  its  half;  consequently,  we  have 

4AC2=IAB2; 

therefore,  by  substitution,  we  obtain 

2AE2ZZZ  AB2, 


TWO   PARTS  SUSTAINING  EQUAL  PRESSURES.  33 

being  the  very  same  expression  as  that  which  we  obtained  by  the 
foregoing  analytical  process,  a  coincidence  which  verifies  the  pre- 
ceding construction. 

Returning  to  the  equation  numbered  (15),  and  extracting  the  square 
root  of  both  sides,  we  obtain 

xy2=li 
and  by  division,  we  have 

x  =  kW2.  (16), 

48.  The  practical  rule   for  determining  the  point  of  division,  as 
supplied  by  the  above  equation,  is  extremely  simple ;  it  may  be  thus 
expressed  : 

RULE.  Multiply  half  the  length  of  the  immersed  side  of 
the  parallelogram  by  the  square  root  of  2,  or  by  the  constant 
number  1.4142,  and  the  product  will  express  the  distance 
downward  from  the  surface  of  the  Jluid. 

49.  EXAMPLE  7.    A   rectangular  parallelogram,   whose   sides  are 
respectively  14  and  28  feet,  is  immersed  in  a  cistern  of  water,  in  such 
a  manner,  that  its  shorter  side  is  just  coincident  with  the  surface; 
through  what  point  in  the  longer  side  must  a  line  be  drawn  parallel 
to  the  horizon,  so  that  the  pressures  on  the  two  parts,  into  which  the 
parallelogram  is  divided,  may  be  equal  to  one  another? 

Here,  by  operating  according  to  the  rule,  we  have 
x—  i  (28  X  1.4142)=  19.7988  feet. 

50.  If  the  point  through  which  the  line  of  division  passes,  were 
estimated  in  the  contrary  direction ;  that  is,  upwards  from  the  lower 
extremity  of  the  immersed  side  of  the  parallelogram;  then,  the  ex- 
pression for  the  place  of  the  point  will  be  very  different  from  that 
which  we  have  given  above,  as  will  become  manifest  from  the  follow- 
ing process. 

Recurring  to  the  original  diagram  of  Problem  5,  and  putting 
x  —  ed,  the  rest  of  the  notation  remaining,  we  shall  have  by  sub- 
traction, 

ae=il  —  x; 

consequently,  sn  the  depth  of  the  centre  of  gravity  of  the  rectangle 
abfe,  is 

3—  J  (/  —  x)  sin.  0, 

and  in  like  manner,  it  may  be  shown,  that  rm,  the  depth  of  the  centre 
of  gravity  of  the  rectangle  efcd,  is 

c?=  (/ —  !#)  sin.  (f>. 

Now,  according  to  the  writers  on  mensuration,  the  area  of  the 
rectangle  abfe  is  expressed  by  b  (i — x},  and  that  of  the  rectangle 
efcd  by  bx;  consequently,  the  respective  pressures  are 
VOL,  i.  i> 


34  SECTIONS  OF  RECTANGULAR  PARALLELOGRAMS 

p—  \  b(l  —  a;)8  s  sin.0,  andprrr  bx(l  —  \  x}  s  sin.0, 
but  by  the  conditions  of  the  problem,  these  pressures  are  equal  ; 
hence  we  get 

J(Z  —  *)2  =  a?  (/  —  I  *), 
and  this,  by  reduction,  becomes 

**  —  2/3?  =  —  i/2; 
consequently,  the  root  of  this  equation  is 


and  more  elegantly,  by  collecting  the  terms,  it  becomes 
x  =  l(l  —  \  V~)' 

51.  This  is  manifestly  the  same  result  as  would  arise,  by  subtract- 
ing the  value  of  x  in  equation  (16),  from  the  whole  length  of  the 
parallelogram  ;  and  the  rule  for  performing  the  operation  is  simply 
as  follows  : 

RULE.  From  unity  subtract  one  half  the  square  root  o/2  ; 
then  multiply  the  remainder  by  the  length  of  the  parallelo- 
gram, and  the  product  will  be  the  distance  of  the  point 
required  from  the  lower  extremity  of  the  immersed  dimension. 

Therefore,  by  taking  the  length  of  the  parallelogram,  as  proposed 
in  the  preceding  example,  we  shall  have  for  the  distance  from  its 
lower  extremity,  through  which  the  line  of  division  passes, 
x  =  28  (1  —  i  ^/2)  =  8.2012  feet. 

COROL.  It  is  manifest  from  the  equations  (16)  and  (17),  that  the 
solution  is  wholly  independent  of  the  breadth  of  the  parallelogram,  its 
inclination  to  the  horizon,  and  the  specific  gravity  of  the  fluid  ;  these 
elements,  therefore,  might  have  been  omitted  in  the  investigation  ; 
but  since  it  became  necessary  to  express  the  pressure  either  absolutely 
or  relatively,  we  thought  it  better  to  exhibit  the  several  quantities,  of 
which  the  measure  of  the  pressure  is  constituted. 

PROBLEM  VII. 

52.  A  given  rectangular  parallelogram  is  immersed  in  a  fluid, 
in  such  a  manner,  that  one  side  is  coincident  with  the  surface, 
and  its  plane  tending  obliquely  downwards  at  a  given  inclination 
to  the  horizon  :  — 

It  is  required  to  draw  a  straight  line  parallel  to  one  of  the 
diagonals,  so  that  the  pressures  on  the  parts  into  which  the 
parallelogram  is  divided,  may  be  equal  to  one  another. 


SUSTAINING   EQUAL  PRESSURES 

Let  A  ED  represent  a  cistern  filled  with 
fluid,  of  which  ABEF  is  the  surface,  sup- 
posed to  be  perfectly  quiescent,  and  con- 
sequently, parallel  to  the  horizon ;  and  let 
ABCD  be  a  vertical  section  of  the  cistern, 
exhibiting  the  fluid  with  the  immersed 
rectangle  abed. 

Draw  the  diagonal  ac,  and  in  a d  take 
any  point  e ;  through  the  point  e  thus 
assumed,  draw  the  straight  line  ef  parallel  to  a  c  the  diagonal  of  the 
parallelogram;  then  is  edf  the  triangle,  on  which  the  pressure  is 
equal  to  that  upon  the  polygonal  figure  e  a  b  cf. 

Take  dn  and  dt  respectively  equal  to  one  third  of  de  and  df,  and 
through  the  points  n  and  t,  draw  nm  and  tm  respectively  parallel  to 
ab  and  ad,  the  sides  of  the  parallelogram,  and  meeting  one  another 
in  the  point  m ;  then,  according  to  problem  B,  m  is  the  place  of  the 
centre  of  gravity  of  the  triangle  e  df. 

Produce  tm  directly  forward,  meeting  ab  the  upper  side  of  the 
parallelogram  perpendicularly  in  s ;  then,  through  the  point  s,  and  in 
the  plane  of  the  fluid  surface,  draw  the  straight  line  sr  also  at  right 
angles  to  a  b,  and  from  m,  the  centre  of  gravity  of  the  triangle  edf, 
demit  the  line  mr  perpendicularly  on  sr  ;  then  is  rm  the  perpendicular 
depth  of  the  centre  of  gravity  of  the  triangle  edf,  and  msr  is  the 
angle  of  inclination  of  the  plane  to  the  horizon. 

Put  b=^ab,  the  horizontal  breadth  of  the  given  parallelogram, 
I  =  ad,  the  length  of  the  immersed  plane  tending  downwards, 
d  —  rm,  the  perpendicular  depth  of  the  centre  of  gravity  of  the 

triangle  efd, 

p  ~  the  whole  pressure  perpendicular  to  its  surface, 
<t>  —  msr,  the  angle  which  the  immersed  plane  makes  with  the 

horizon, 

s  =the  specific  gravity  of  the  fluid, 
and  x  —  ed,  the  perpendicular  of  the  triangle  edf,  of  which  the  base 

isrf/. 

Then,  by  reason  of  the  parallel  lines  ac  and  ef,  the  triangles  adc 
and  ec?/are  similar  to  one  another,  and  consequently,  by  the  property 
of  similar  triangles,  we  have 

ad  :  dc  : :  ed  :  df, 
which,  by  restoring  the  symbols,  becomes 

I  :  b  :  :  x  :  df, 
and  from  this  analogy  we  have 

D2 


36  SECTIONS  OF  RECTANGULAR  PARALLELOGRAMS 


therefore,  by  the  principles  of  mensuration,  the  area  of  the  triangle 
efd  is 

bx        bx* 

.     4*XT=2T 

Now,  according  to  the  construction,  dn  is  equal  to  one  third  of  ed, 
and  an  is  equal  to  ad  minus  dn;  but  sn  is  obviously  equal  to  an\ 

hence  we  have 

sninl  —  \x, 

and  by  the  principles  of  Plane  Trigonometry,  it  is 

rm  zz  c?~  (I  —  ^  x)  sin.  <f>  ; 
consequently,  the  pressure  on  the  triangle  edf  becomes 

_  bx*s  (31  —  x}  sin.  <p 
P~  ~6T 

and  this,  by  the  conditions  of  the  problem,  is  equal  to  half  the  pres- 
sure on  the  entire  parallelogram  ;  therefore,  and  by  equation  (7),  we 
have 

bx*  s  (3  1  —  x")  sin.  <£  _  b  l*s  sin.  p  e 

61  ~T~ 

hence,  by  expunging  the  common  quantities,  we  get 


—  , 

and  furthermore,  by  separating  and  transposing  the  terms,  it  'IB 

2x3  —  6lx*=  —  3l3, 
and  dividing  all  the  terms  by  2,  we  obtain 

x8  —  3lx*=i  —  1.51s.  (18). 

It  is  somewhat  remarkable,  that  the  solution  of  a  problem  appa- 
rently so  simple,  should  require  the  reduction  of  a  cubic  equation  ; 
but  so  it  happens,  and  it  may  be  proper  to  observe,  that  in  the  present 
instance,  it  cannot  be  resolved  by  means  of  an  equation  of  a  lower 
degree. 

Now,  in  order  to  determine  the  value  of  x  from  the  above  equation, 
we  have  only  to  substitute  the  numerical  value  of  /  as  given  in  the 
question,  and  then  to  resolve  the  equation  by  the  rules  given  for  that 
purpose. 

53.  EXAMPLE  8.  Suppose  the  immersed  length  of  the  rectangle,  or 
that  tending  downwards,  to  be  20  feet  ;  how  far  below  the  surface  of 
the  fluid  must  the  point  be  situated,  through  which  a  line  drawn 
parallel  to  the  diagonal,  will  divide  the  parallelogram  into  two  parts 
sustaining  equal  pressures  ? 


SUSTAINING  EQUAL  PRESSURES.  37 

Here  the  given  length  is  20  ;  therefore,  by  substituting  20  and  208, 
respectively  for  /  and  P  in  the  above  equation,  we  shall  obtain 

a3  —  60  x*  =  —  12000. 

In  order,  therefore,  to  take  away  the  term  —  60  #8  and  prepare  the 
equation  for  solution,  we  must  put  x  =  z  -\-  20,  and  then  by  involu- 
tion, we  have 

x*  =  z3  -f-  60  z2  -f  1200  z  +  8000 
—  60  xz  =     —  60  z*  —  2400  z  —  24000, 

from  which,  by  summing  the  terms,  we  get 

xs  —  60  *8  =  z*  —  1200  z  —  16000  =  —  12000  ; 

therefore,  by  transposition,  we  obtain 

z3  —  1200  z  =  4000. 

Now,  since  the  equation  falls  under  the  irreducible  case  of  cubics, 
it  is  manifest,  that  its  solution  cannot  be  effected  by  Cardan's  formula  ; 
we  must  therefore  have  recourse  to  some  other  method,  and  in  the 
present  instance,  it  will  be  convenient  to  adopt  the  concise  and 
elegant  theorem  of  the  Chevalier  de  Borda. 

For  which  purpose,  put  a  zz  any  arc  such,  that  cosec.3a  =  -  \/  \m  ; 
then,  we  shall  have 

z  =  —  2  v/T™  sin.a,  (19). 

where  m  is  the  co-efficient  of  the  second  term,  and  n  the  absolute 
number  ;  consequently,  by  substitution,  we  obtain 


therefore,  by  the  Trigonometrical  Tables,  we  have 

3«  =14°  28'  39", 
and  by  division,  we  get 

a  =  4°  49'  33". 

But  the  natural  sine  of  4°  49'  33"  to  the  radius  unity,  is  0.08412, 
and  }m  =400  ;  consequently,  by  equation  (19),  we  have 
z——  40  x  0.08412  =  —  3.3648; 
now,  we  have  seen  above,  that 

arzrs+20; 
therefore,  by  substitution,  we  obtain 

x  =  —  3.3648  4-  20  =  16.6352. 

COROL.  Hence  it  appears,  that  if  we  take  3.3648  feet  downwards 
from  the  surface  of  the  fluid,  or  16.6352  feet  upwards  from  the  lower 
side  of  the  plane,  and  through  the  point  thus  determined  in  either 
case,  let  a  straight  line  be  drawn  parallel  to  the  diagonal  ;  then  shall 
the  rectangle  be  divided  as  required  in  the  problem. 


38  CENTRE  OF  GRAVITY  OF  MIXED  SPACE 

6.  METHOD  OF  FINDING  THE  POSITION  OF  THE  CENTRE  OF  GRAVITY 
OF  ANY  MIXED  SPACE  OF  RECTILINEAR  FIGURES  IMMERSED  IN 
NON-ELASTIC  FLUIDS. 

54.  Since  the  pressure  on  the  entire  parallelogram,  is  equivalent  to 
the  sum  of  the  pressures  on  the  two  parts  into  which  it  is  divided  by 
the  line  ef;  it  follows  from  thence,  that  the  position  of  the  centre  of 
gravity  of  the  figure  abcfe  can  be  determined,  as  is  shown  in  what 
follows. 

Let  ABCFE  be  the  figure,  of  which  the  centre  of  gravity  is  required 
to  be  found,  the  angles  at  A,  B  and  c  being  right 
angles ;  join  the  points  A  and  F  by  the  straight  line 
AF,  dividing  the  figure  into  two  parts,  one  of  which  is 
the  triangle  AFE  and  the  other  the  trapezoidal  space 
ABCF. 

Now,  almost  every  writer  on  mechanical  science 
has  given  the  method  of  finding  the  centre  of  gravity 
of  those  figures  separately,  from  which  that  of  the  compound  space 
may  easily  be  determined ;  but  we  are  not  aware  of  any  method  that 
has  been  proposed,  for  the  direct  discovery  of  the  centre  of  gravity 
of  the  mixed  space  ABCFE,  and  that  is  what  we  are  now  about  to 
investigate. 

Produce  the  sides  AE  and  CF  till  they  meet  in  D;  then,  because 
the  angles  at  A,  B  and  c  are  right  angles,  the  angle  at  D  is  also  a  right 
angle;  from  the  point  D,  set  off  Da  and  D&,  respectively  equal  to 
one  third  of  DE  and  DF,  and  through  the  points  a  and  b,  draw  am 
and  bm  parallel  to  DA  and  DC,  which  produce  directly  forward  to  d 
and  c ;  then  are  cm  and  dm  respectively,  the  perpendicular  depths  of 
the  centre  of  gravity  of  the  triangle  EDF,  according  as  the  side  AB  or 
BC  is  supposed  to  be  coincident  with  the  surface  of  the  fluid. 

Put  b  m  AB,  the  breadth  of  the  rectangular  parallelogram  A  BCD, 
I  —  AD,  the  length  of  ditto, 

/3=  DF,  the  base  of  the  right  angled  triangle  EDF, 
Z'nz  DE,  the  corresponding  perpendicular, 
d  —  cm,  the  perpendicular  depth  of  the  centre  of  gravity  of  the 

triangle  EDF,  when  the  side  AB  is  horizontal, 
3  —  dm,  the  perpendicular  depth  of  the  centre  of  gravity  m,  when 

the  side  BC  is  horizontal, 

jazr  the  pressure  perpendicular  to  the  surface  of  the  triangle  EDF, 
7>'z=  the  pressure  on  the  irregular  figure  ABCFE, 


OF  IMMERSED  RECTILINEAR  FIGURES.  39 

P=  the  pressure  on  the  entire  parallelogram  A  B  c  D, 

x—rn,  the  perpendicular  depth  of  the  centre  of  gravity  of  the 

figure  ABCFE,  when  the  side  AB  is  horizontal, 

and  y  —  sn,  the  perpendicular  depth,  when  the  side  BC  is  horizontal. 
Then,  because  the  sides  AE  and  CF  are  given  quantities,  it  follows, 
that  DE  and  DF  are  also  given,  and  consequently,  AC  or  cm,  and  cb 
or  dm  are  given  ;  therefore,  the  perpendicular  pressure  on  the  triangle 
EDF  can  easily  be  ascertained. 

Now,  A  a  is  manifestly  equal  to  the  difference  between  A  D  and  a  D, 
and  by  the  construction  an  is  equal  to  one  third  of  D  E  ;  therefore,  by 
restoring  the  analytical  representatives,  we  have 

cm  =  d  =  l  —  il'. 

Again  c  b  is  equal  to  the  difference  between  c  D  and  D  b  ;  but  D  b  by 
the  construction,  is  equal  to  one  third  of  DF  ;  hence,  by  restoring  the 
analytical  symbols,  we  shall  obtain 

dm  —  $=:b  —  ±  ft. 

But,  according  to  the  writers  on  mensuration,  the  area  of  the 
triangle  EDF  is  equal  to  half  the  product  of  the  base  DF  by  the  per- 
pendicular DE  ;  that  is 

JZ'X/3  =  jr|3; 

consequently,  if  we  suppose  the  plane  to  be  perpendicularly  immersed 
in  the  fluid,  while  the  side  AB  is  coincident  with  its  surface  ;  then,  the 
pressure  on  the  triangle  EDF  becomes 

p  =  %pl's(3l-l'). 

Now,  the  pressure  on  the  irregular  figure  ABCFE,  is  obviously 
equal  to  the  difference  between  the  pressures  on  the  entire  paral- 
lelogram A  B  c  D,  and  the  triangle  EDF;  but  the  pressure  on  the  entire 
parallelogram,  according  to  equation  (8),  is 


consequently,  by  subtraction,  the  pressure   on  the  figure   ABCFE, 
becomes 


but  its  area  is  also  equal  to  the  difference  between  that  of  the 
parallelogram  and  triangle;  therefore,  we  obtain  J(2£Z  —  (31')  for 
the  area  of  the  irregular  figure  ABCFE;  consequently,  by  division, 
the  perpendicular  depth  of  the  centre  of  gravity  below  the  line  AB, 
becomes 


and  if  we  suppose  the  fluid  in  which  the  plane  is  immersed  to  be 
water,  the  specific  gravity  of  which  i*  unity,  we  finally  obtain 


40  CENTRE  OF  GRAVITY  OF  MIXED  SPACE 

zbi*  —  fii'(3i—  r) 

3(2ft/-00 

Again,  if  we  suppose  the  side  BC  to  be  horizontal,  the  area  of  the 
triangle  remains  the  same,  and  the  pressure  which  it  sustains  in  a 
direction  perpendicular  to  its  surface,  becomes 
p  =  tfll's(3b—  0). 

But  the  pressure  on  the  whole  parallelogram  A  BCD,  on  the  suppo- 
sition that  the  side  BC  is  horizontal,  according  to  what  has  been 
proved  in  Problem  3,  is 


consequently,  the  pressure  on  the  irregular  figure  ABCFE,  becomes 


Now,  the  area  of  the  figure  corresponding  to  the  above  pressure,  is 
obviously  the  same  as  we  have  previously  determined  it  to  be  ;  that  is, 
the  difference  between  the  areas  of  the  triangle  and  the  entire  paral- 
lelogram ;  consequently,  by  division,  we  shall  obtain 
_ 

The  equations  (20)  and  (21)  are  manifestly  symmetrical  ;  if  there- 
fore, we  carefully  attend  to  the  conditions  of  the  problem,  from  which 
they  are  respectively  derived,  the  position  of  the  centre  of  gravity  of 
the  figure  ABCFE  can  easily  be  ascertained  byresolvng  the  equations. 

55.  The  practical  rules  for  determining  the  co-ordinates  which  fix 
the  position  of  the  centre  of  gravity,  may  be  expressed  in  the  follow- 
ing manner  : 

1.  When  the  side  AE  is  horizontal,  as  indicated  by  equation  (20). 

RULE.  From  three  times  the  vertical  length  of  the  given 
rectangular  parallelogram,  subtract  the  perpendicular  of  the 
triangle,  and  multiply  the  remainder  by  twice  its  area  ;  then^ 
subtract  the  product  from  three  times  the  square  of  the  length 
of  the  parallelogram  drawn  into  its  breadth,  and  the  remain- 
der will  be  the  dividend. 

Divide  the  dividend  above  determined,  by  three  times  the 
difference  between  twice  the  area  of  the  parallelogram^  and 
twice  that  of  the  triangle,  and  the  quotient  will  give  the 
co-ordinate  of  the  line  A  B. 

2.  When  the  side  BC  is  horizontal,  as  indicated  by  equation  (21). 

RULE.  From  three  times  the  vertical  breadth  of  the  paral- 
lelogram, subtract  the  base  of  the  triangle,  and  multiply  the 


OF  IMMERSED  RECTILINEAR  FIGURES,  41 

remainder  by  twice  its  area  ;  then,  subtract  the  product  from 
three  times  the  square  of  the  breadth  of  the  parallelogram 
drawn  into  its  length,  and  the  remainder  will  be  the  dividend. 
Divide  the  dividend  above  determined,  by  three  times  the 
difference  between  twice  the  area  of  the  parallelogram,  and 
twice  that  of  the  triangle,  and  the  quotient  will  give  the 
co-ordinate  of  the  line  BC. 

56.  EXAMPLE  9.  The  sides  of  a  rectangular  parallelogram  are 
respectively  28  and  50  feet,  and  from  one  of  the  lower  corners,  is 
separated  a  right  angled  triangle,  by  means  of  a  straight  line  ter- 
minating in  the  adjacent  sides;  it  is  required  to  determine  the 
position  of  the  centre  of  gravity  of  the  remaining  part,  the  base  and 
perpendicular  of  the  separated  triangle,  being  respectively  equal  to 
20  and  42  feet  ? 

Here  then,  by  operating  as  directed  in  the  first  rule,  we  have 

3  x  50—  42  =  150  —  42  =  108, 
and  by  the  principles  of  mensuration,  twice  the  area  of  the  triangle,  is 

42  X  20  =  840  square  feet ; 
therefore,  by  multiplication,  we  obtain 

108  x  840  =  90720. 
Again,  three  times  the  square  of  the  length  of  the  parallelogram,  is 

3x  502  =  7500, 
which  being  multiplied  by  its  breadth,  gives 

7500  x  28  =  210000; 
consequently,  by  subtraction,  the  dividend  is 

210000  —  90720=119280. 

Now,  twice  the  area  of  the  parallelogram,  is  2  x  50  x  28  =  2800 
square  feet,  and  twice  the  area  of  the  triangle,  is  42  x  20  =  840 
square  feet ;  therefore,  by  the  second  clause  of  the  rule,  we  obtain 

a  = H928° =  20.286  feet  nearly. 

3(2800  —  840) 

Hence  it  appears,  that  the  co-ordinate  of  the  line  AB,  according  to 
the  proposed  data,  is  very  nearly  20.286  feet;  and  by  operating  as 
directed  in  the  second  rule,  we  shall  have 

3  x  28  —  20  =  84  —20  =  64, 
and  by  the  principles  of  mensuration,  twice  the  area  of  the  triangle,  is 

42  X  20  =  840  square  feet ; 
therefore,  by  multiplication,  we  obtain 

64  x  840  =  53760. 

Again,  three  times  the  square  of  the  breadth  of  the  parallelogram,  is 
3  x  28*  =  2352, 


42  CENTRE  OF  GRAVITY  OF  MIXED  SPACE 

which  being  multiplied  by  its  length,  gives 

2352  X  50=117600; 
consequently,  by  subtraction,  the  dividend  becomes 

117600  —  53760  =  63840. 

Now,  the  second  clause  of  the  second  rule,  being  the  same  as  the 
second  clause  of  the  first  rule,  it  follows,  that  the  divisor  must  here 
be  the  same,  as  we  have  found  it  to  be  in  the  preceding  case  ;  conse- 
quently, by  division,  we  obtain 


therefore,  from  the  numerical  values  of  the  co-ordinates  as  we  have 
just  determined  them,  the  position  of  the  centre  of  gravity  of  the 
proposed  figure  can  easily  be  found,  in  the  following  manner. 

57.  Let  ABCD  represent  the  rectangular  parallelo- 
gram, of  which  the  side  AB  is  28  feet,  and  the  side  EC 
50  feet;  and  let  EDC  be  the  right  angled  triangle, 
whose  perpendicular  E  D  is  42  feet,  and  its  base  D  F  20 
feet,  all  taken  from  the  same  scale  of  equal  parts. 

From  the  angle  B,  and  on  the  sides  BC  and  BA,  set 
off  BS  and  Br  respectively  equal  to  20.286  and  10.857 
feet;  then,  through  the  points  s  and  r,  draw  the  lines  sn  and  rn, 
respectively  parallel  to  AB  and  BC,  and  the  point  n  is  the  Centre  of 
gravity  of  the  figure  ABCFE,  which  remains  after  the  right  angled 
triangle  EDF  is  separated  from  the  parallelogram  ABCD. 

If  the  line  of  division,  or  hypothenuse  of  the  triangle  EF,  were 
parallel  to  AC  the  diagonal  of  the  parallelogram,  as  is  distinctly  speci- 
fied in  the  foregoing  problem,  the  solution  would  become  much  more 
simple  ;  for  then,  in  order  to  determine  the  position  of  the  centre  of 
gravity,  it  is  only  necessary  to  reduce  one  of  the  equations,  and  it  is 
altogether  a  matter  of  indifference  which  of  them  it  is,  provided  that 
the  conditions  of  the  equation  be  strictly  attended  to. 

Supposing  E  D  the  perpendicular  of  the  triangle,  to  remain  as  above  ; 
then  the  base,  when  the  hypothenuse  is  parallel  to  the  diagonal  of  the 
rectangle,  will  be  found  by  the  following  analogy,  viz. 
50  :  28  :  :  42  :  23.52. 

Then,  by  calculating  according  to  rule  first,  or  equation  (20),  our 
dividend  and  divisor  are  103313.28  and  5436.48  respectively;  con- 
sequently, we  get 

103313.28 

*=  5436.48  =19fe 


OF  IMMERSED  RECTILINEAR  FIGURES.  43 

therefore,  by  analogy,  we  obtain 
50  :  19  ::  28  :  y  =  10.64  feet. 

Here,  the  whole  process  of  calculating  the  second  co-ordinate,  is 
replaced  by  the  simple  analogy  above  exhibited. 

The  example  now  before  us,  affords  a  striking  instance  of  the 
advantages  to  be  derived  from  this  mode  of  considering  the  centre  of 
gravity;  in  the  case  of  the  triangle  illustrated  under  Problem  (B),  its 
immediate  utility  was  not  so  conspicuously  displayed ;  but  we  are 
convinced,  that  in  figures  of  more  difficult  and  complicated  forms,  its 
usefulness  will  become  still  more  evident. 

In  the  investigation  of  the  formulae,  we  have  thought  it  necessary 
to  consider  the  pressure  on  the  surface  whose  centre  of  gravity  is 
sought ;  but  in  the  actual  application  of  the  resulting  equations,  the 
consideration  of  pressure  does  not  enter;  for  it  is  manifest,  that 
besides  the  dimensions  of  the  figure  and  constant  numbers,  no  other 
element  is  found  in  the  equations,  and  consequently,  the  reduction 
depends  upon  them  alone. 

7.  OF  EQUAL  FLUID  PRESSURES  ON  THE  SECTIONS  OF  A  RECTANGULAR 
PARALLELOGRAM  AND  THE  PERPENDICULAR  DEPTHS  OF  THE  CENTRE 
OF  GRAVITY. 

PROBLEM  VIII. 

58.  A  given  rectangular  parallelogram  is  immersed  in  an 
incompressible  and  non-elastic  fluid,  in  such  a  manner,  that  one 
of  its  sides  is  coincident  with  the  surface,  and  its  plane  tending 
downwards  at  a  given  inclination  to  the  horizon : — 

It  is  required  to  draw  a  straight  line  from  one  of  the  upper 
angles  to  the  lower  side,  so  that  the  pressures  on  the  two 
parts  into  which  the  parallelogram  is  divided,  may  be 
equal  to  one  another. 

Let  AED  represent  a  rectangular  cistern  filled  with  water,  or  some 
other  incompressible  and  non-elastic  fluid, 
of  which  ABEF  is  the  horizontal  surface, 
and  suppose  one  of  the  upright  sides,  as 
ABCD  to  be  removed,  exhibiting  the  fluid 
together  with  the  immersed  rectangle  abed. 

In  dc  the  lower  side  of  the  immersed 
parallelogram,  take  any  point/,  and  draw 
af  to  represent  the  line  of  division ;  then 
the  triangle  adf,  and  the  trapezoid  abcf,  T> 

are  the  figures  into  which  the  parallelogram  is  divided,  and  on  which 
the  pressures  are  equal. 


44  PARALLELOGRAM   DIVIDED  TO  SUSTAIN 

From  the  angle  d,  set  off  dn  and  dt  respectively  equal  to  one  third 
of  da  and  df,  and  through  the  points  n  and  t,  draw  nm  and  tm 
parallel  to  df  and  da  the  sides  of  the  triangle,  and  meeting  each 
other  in  the  point  m  ;  then,  according  to  what  has  been  demonstrated 
in  Problem  (B),  m  is  the  centre  of  gravity  of  the  triangle  adf. 

Produce  tm  directly  forward,  meeting  a  b  at  right  angles  in  the 
point  r,  and  through  the  point  r  and  in  the  plane  of  the  fluid  surface, 
draw  rs  also  at  right  angles  to  a  b,  and  demit  ms  meeting  rs  perpen- 
dicularly in  s;  then  is  mrs  the  inclination  of  the  plane  to  the  horizon, 
and  sm  the  perpendicular  depth  of  the  centre  of  gravity  of  the  triangle 
ad/*  below  the  upper  surface  of  the  fluid. 

Put  b  —  ab,  the  horizontal  breadth  of  the  parallelogram  abed, 
I  —  ad,  the  immersed  length  tending  downwards, 
p  zn  the  pressure  perpendicular  to  the  surface  of  the  triangle  a  df. 
P=  the  pressure  on  the  entire  parallelogram  abed, 
0  =  mrs,  the  angle  which  the  immersed  plane  makes  with  the 

horizon  , 
d^ism,  the  perpendicular  depth  of  the  centre  of  gravity  of  the 

triangle  adf, 

s  =z  the  specific  gravity  of  the  fluid, 
and  x  zz  df,  the  distance  between  d  and  the  point  through  which  the 

line  of  division  passes. 

Then,  according  to  the  principles  of  Plane  Trigonometry,  the  per- 
pendicular depth  of  the  centre  of  gravity  of  the  triangle  adf,  becomes 

6?rr-|Zsin.0; 
consequently,  the  pressure  on  its  surface,  is 

p  —  %l*xs  sin.0  ;   J  see  equation  (10)  J  . 

But  according  to  equation  (7)  under  the  3rd  problem,  (art.  33),  the 
pressure  on  the  entire  rectangle,  is 


and  by  the  conditions  of  the  present  problem,  the  pressure  on  the 
triangle,  is  equal  to  one  half  the  pressure  on  the  entire  parallelogram  ; 
therefore,  we  have 

/?==  !P;  that  is 


from  which,  by  expunging  the  common  terms,  we  get 

4x  =  3b; 

consequently,  by  division,  we  obtain 
_3b 

~  4  ' 


EQUAL  FLUID  PRESSURES.  45 

59.  This  equation  is  too  simple  in  its  arrangement  to  require  any 
formal  directions  for  its  resolution ;  nevertheless,  the  following-  rule 
may  be  useful  to  many  of  our  readers. 

RULE.  {Take  three  fourths  of  that  side  of  the  given  rec- 
tangular parallelogram, in  which  the  line  of  division  terminates , 
and  the  point  thus  discovered,  is  that  through  which  the  line 
of  division  passes. 

60.  EXAMPLE  10.    A  rectangular  parallelogram,  whose  sides  are 
respectively  equal  to  24  and  42  feet,  is  immersed  in  a  cistern  full  of 
water,  in  such  a  manner,  that  its  shorter  side  is  coincident  with  the 
surface  of  the  fluid,  and  its  plane  inclined  to  the  horizon  in  an  angle 
of  52  degrees ;  it  is  required  to  determine  a  point  in  its  lower  side, 
to  which,  if  a  straight  line  be  drawn  from  one  of  the  upper  angles,  the 
parallelogram  shall  be  divided  into  two  parts  sustaining  equal  pres- 
sures ;  and  moreover,  if  a  straight  line  be  drawn  from  the  same  point 
in  the  lower  side,  to  the  other  upper  angle,  it  is  required  to  assign  the 
pressure  on  the  triangle  thus  cut  off? 

Here,  by  operating  according  to  the  rule,  the  point  of  division  is 

x  =  %  X  24=  18  feet. 

In  the  next  place,  to  determine  the  pressure  sustained  by  the 
triangle  bcft  cut  off  from  the  parallelogram  abed,  by  means  of  the 
line/6  drawn  from  the  point  f  to  the  angle  at  b,  we  have  according 
to  equation  (12),  (Problem  4), 

p  z=  i  (b  —  x)  I9  s  sin.0, 

where    (b  —  #)    in  this   equation,  takes  place  of  b  in   the  one   re- 
ferred to. 

The  natural  sine  of  52  degrees  according  to  the  Trigonometrical 
Tables,  is  .78801 ;  hence,  by  substituting  the  respective  data  in  the 
above  equation,  we  shall  have 

p—l  (24—18)  X  422  X  .78801  —  2746.09928  cub.  ft.  of 
water ;  consequently,  the  pressure  expressed  in  Ibs.  avoirdupois,  is 

p1  =  2746.09928  X  62.5  =  181631.205  Ibs. 

This  seems  to  be  an  immense  pressure,  on  a  triangle  whose  surface 
is  only  126  square  feet ;  it  is  however  but  one  sixth  part  of  the  pres- 
sure on  the  entire  parallelogram ;  this  is  manifest,  for  the  pressure  on 
the  triangle  adf,  is  three  times  the  pressure  on  the  triangle  bcf,  since 
the  base  df  is  equal  to  three  times  the  base  cf,  and  the  altitudes  of 
the  triangles,  as  well  as  the  perpendicular  depths  of  the  centres  of 
gravity,  are  the  same;  but  the  pressure  on  the  parallelogram  abed, 


46  PARALLELOGRAM  DIVIDED  TO  SUSTAIN 

according  to  the  problem,  is  double  the  pressure  on  the  triangle  adf; 

hence  we  have 

P  —  2p  i=  6p'  =  1089787.23  Ibs. 

61.  If  the  line  of  division  were  drawn  from  one  of  the  lower  angles 
to  a  point  in  the  immersed  length,  after  the  manner  represented  in  the 
annexed  diagram ;  then,  the  equation  (22),  would  assume  a  different 
form,  as  will  become  manifest  from  the  following  investigation. 

From  the  angle  d  on  da  and  dc  the  sides  of  the 
parallelogram,  set  off  dn  and  dt,  respectively  equal  to 
one  third  of  df  and  dc,  and  through  the  points  n  and 
t  thus  found,  draw  the  straight  lines  nm  and  tm  \ 
parallel  to  dc  and  df,  the  base  and  perpendicular  of 
the  triangle  fdc,  and  meeting  one  another  in  m,  the 
place  of  its  centre  of  gravity. 

Produce  tm  directly  forward,  meeting  a b,  the  hori- 
zontal side  of  the  given  parallelogram  perpendicularly  in  the  point  r ; 
at  the  point  r  in  the  straight  line  mr,  make  the  angle  mrs  equal  to 
the  angle  of  the  plane's  inclination,  and  draw  ms  perpendicularly  to 
rs ;  then  is  sm  the  perpendicular  depth  of  the  centre  of  gravity  of  the 
triangle /We. 

Let  therefore,  the  notation  of  the  preceding  case  be  retained,  and 
put  x  ==  df;  then  we  have 

am^irmm.  I' —  ^x,  and  consequently sm  —  di=:  (I  —  ^x)  sin .0  ; 
but  the  area  of  the  triangle  fdc  is  expressed  by  \b x;  therefore,  the 
pressure  perpendicular  to  its  surface,  is 


now,  according  to  the  conditions  of  the  problem,  the  pressure  on  the 
separated  triangle  is  equal  to  half  the  pressure  on  the  entire  paral- 
lelogram ;  consequently,  we  obtain 


and  this,  by  expunging  the  common  quantities,  becomes 

2  x  (3  /  —  x)  =  3  l\ 
or  dividing  by  2  we  get 

x(3l  —  x}  =  l.5l\ 
and  from  this,  by  separating  and  transposing  the  terms,  we  have 

x*  —  3lx  —  —  1.5  Z2.  (23). 

If  the  equations  (18)  and  (23)  be  compared  with  one  another,  it 
will  readily  appear,  that  they  are  precisely  similar  in  form,  but  dif- 
ferent in  degree  ;  the  former  being  an  incomplete  cubic,  wanting  the 
first  power  of  the  unknown  quantity,  and  the  latter  an  adfected 
quadratic,  having  all  its  terms.  Indeed,  the  diagrams  from  which  the 


EQUAL  FLUID  PRESSURES.  47 

two  equations  are  derived,  as  well  as  the  specified  conditions  of  the 
problems,  are  nearly  similar,  the  difference  consisting  simply  in  the 
position  of  the  dividing  line,  it  being  parallel  to  the  diagonal  of  the 
parallelogram  in  the  one  case,  and  oblique  to  it  in  the  other. 

62.  Let  the  quantity  2JZ2be  added  to  both  sides  of  the  preceding 
equation,  and  we  shall  obtain 


from  which,  by  extracting  the  square  root,  we  get 

x—  i$i  =  +  \i  V3"; 

therefore,  by  transposition,  we  have 


(24). 

The  practical  rule  by  which  the  point  of  division  is  to  be  determined, 
may  be  expressed  as  follows  : 

RULE.  Multiply  the  difference  between  3  and  the  square 
root  of  3,  by  half  the  length  of  that  side  of  the  parallelogram 
in  which  the  line  of  division  terminates,  and  the  product  will 
'be  the  distance  of  the  required  point  from  the  lower  extremity 
of  the  given  length. 

63.  EXAMPLE  11.  Let  the  numerical  data  remain  precisely  as  in 
the  preceding  case;  from  what  point  in  the  length  of  the  paral- 
lelogram, must  a  straight  line  be  drawn  to  the  opposite  lower  angle, 
so  that  the  parallelogram  may  be  divided  into  two  parts  sustaining 
equal  pressures  ;  and  moreover,  if  a  straight  line  be  drawn  from  the 
same  point,  to  the  opposite  upper  angle,  what  will  be  the  pressure  on 
the  triangle  thus  cut  off? 

Here,  by  proceeding  as  directed  in  the  above  rule,  we  have 

x  —  21  (3  —  V  3)  =  26.628  feet. 

In  order  to  find  the  pressure  on  the  triangle  a  bf  cut  off  by  the 
line  bf,  we  have  af—l  —  x,  and  ae  —  rp  —  \(l  —  x)\  conse- 
quently, vp^n^(l  —  x)  sin.0,  where  it  must  be  observed,  that  ep 
and  vp  are  respectively  parallel  to  a  b  and  sm. 

Now,  the  pressure  perpendicular  to  the  surface  of  the  triangle  a  bf, 
is  found  by  multiplying  its  area  into  vp,  the  perpendicular  depth  of 
its  centre  of  gravity  ;  hence,  we  have 

p'  —  ^bs  (I  —  #)2  sin.^.  ; 
but  the  value  of  x,  according  to  equation  (24),  is 

x  =  .634  I  ; 

consequently,  by  substitution,  we  have 

p'  =  %b  Z2  s  (1  —  .634)*  sin.0,  from  which,  by  substituting  the 
several  numerical  values,  we  obtain 

>'  =  4  X  42s  X  62.5  X  .366*  X  .78801  =  46551.  35  Ibs. 


48 


S.    PERPENDICULAR  DEPTH  OF  THE    CENTRE   OF    GRAVITY  Of  A  PARAL- 
LELOGRAM DIVIDED  INTO  TWO  PARTS  SUSTAINING  EQUAL  PRESSURES. 

64.  With  respect  to  the  centre  of  gravity  of  the  figure  a  b  cf,  which 
remains  after  the  triangle  adf,  or  fdc  has  been  separated  from  the 
parallelogram,  it  is  in  this  particular  instance  very  easily  determined  ; 
for,  since  the  area  of  trapezoids,  whose  parallel  sides  and  perpendi- 
cular breadths  are  equal  each  to  each,  are  also  equal ;  it  follows,  that 
the  centre  of  gravity  must  occur  in  the  straight  line  which  bisects  the 
parallel  sides ;  it  is  therefore,  only  necessary  to  investigate  the  theorem 
for  calculating  one  of  the  co-ordinates,  the  other  being  determinable 
from  the  circumstance  just  stated. 

Let  ABCF  be  the  trapezoid,  having  the  angles  at  B  and  c  respec- 
tively right  angles,  and  of  which  the  position  of  the  centre  of  gravity 
is  required. 

Produce  the  side  CF  directly  forward  to  any  convenient  length  at 
pleasure,  and  through  the  point  A,  draw  the  straight  line  AD  parallel 
to  BC,  the  longer  side  of  the  trapezoidal  figure,  and 
meeting  c  F  produced  perpendicularly  in  the  point  D. 

Then,  the  pressure  upon  the  trapezoid  ABCF,  is 
manifestly  equal  to  the  difference  between  the  pressure 
on  the  parallelogram  A  BCD,  and  that  upon  the  triangle 
ADF,  and  its  area,  is  also  equal  to  the  difference  be- 
tween their  areas.  Bisect  the  parallel  sides  AB  and  CF 
in  the  points  a  and  b,  and  join  a  b  ;  then,  according  to 
what  lias  been  demonstrated  by  the  writers  on  mechanics,  the  centre 
of  gravity  of  the  trapezoid  ABCF  occurs  in  the  straight  line  ai. 

Suppose  it  to  occur  at  m,  and  through  the  point  m  draw  mr  and 
ms  respectively  parallel  to  BC  and  BA,  meeting  AB  and  BC  perpen- 
dicularly in  the  points  a  and  b  ;  then  are  rm  and  sm  the  co-ordinates, 
whose  intersection  determines  the  position  of  the  point  m. 

Put  b  =z  AB,  the  breadth  of  the  parallelogram  A  BCD, 
/  —  AD,  or  BC,  its  corresponding  length, 

£  =  m  r  the  depth  of  the  point  m  as  referred  to  the  line  A  B  con- 
sidered to  be  horizontal, 
cTzu  ms,  the  depth  of  the  point  m  as  referred  to  the  line  BC  under 

similar  circumstances, 
and  /3~  DF,  the  base  of  the  triangle  ADF. 

Then,  by  conceiving  the  plane  to  be  immersed  perpendicularly  in  a 
fluid  whose  specific  gravity  is  expressed  by  unity,  the  pressure  upon 


CENTRE  OF  GRAVITY  OF  PARALLELOGRAMS,  &C.  49 

the  entire  surface  ABCD,  according  to  equation  (8)  under  the  third 
problem,  becomes 

P  =  J&/2; 

and  moreover,  by  equation  (12)  under  the  fourth  problem,  the  general 
expression  for  the  pressure  on  the  triangle  ADF,  is 

p  —  4/3  P  s  sin.0  ; 

but  according  to  the  particular  case  now  under  consideration,  the 
above  expression  becomes 


the  terms  s  and  sin.0,  being  each  equal  to  unity,  they  disappear  in 
the  equation. 

Now,  according  to  what  we  have  stated  above,  the  pressure  on  the 
trapezoid  ABCF,  is  equal  to  the  difference  between  the  pressure  on  the 
entire  parallelogram  ABCD,  and  that  on  the  triangle  ADF  ;  that  is 


or,  by  reducing  the  fractions  to  a  common  denominator  and  collecting 
the  terms,  we  obtain 

^    j/  =  ^(36  —  2/3). 

By  the  principles  of  mensuration,  the  area  of  the  trapezoid  ABCF, 
is  equal  to  the  product  that  arises,  when  half  the  sum  of  the  parallel 
sides  AB  andcF,  is  multiplied  by  BC  the  perpendicular  distance 
between  them  ;  that  is, 

BC  X  |(AB  +  CF)Z=  ¥(^—  /?), 

and  the  perpendicular  depth  of  the  centre  of  gravity,  is  equal  to  the 
pressure  on  the  surface,  divided  by  the  area  of  the  figure  ;  conse- 
quently, we  obtain 


—  jS)   ' 

The  form  of  this  equation  is  extremely  simple,  but  it  may  be  arrived 
at  independently  of  the  preceding  investigation,  by  having  recourse 
to  equation  (20)  under  Problem  6  ;  for  according  to  the  conditions  of 
the  question,  the  line  of  division  AF  originates  at  the  angle  A,  and 
consequently,  the  perpendicular  of  the  triangle  and  the  length  of  the 
parallelogram  are  equal  ;  therefore,  by  putting  /  instead  of  /'  in  equa- 
tion (20),  the  above  expression  immediately  obtains. 

Now,  by  taking  the  length  and  breadth  of  the  parallelogram,  as 
given  in  the  preceding  example,  and  the  base  of  the  triangle  as  com- 
puted by  equation  (22),  we  shall  obtain, 


3(2  x24—  18) 

65.  Having  thus  determined  the  magnitude  of  the  co-ordinate  BS 
or  rm  from  the  equation  (25),  the  magnitude  of  the  corresponding 

VOL.  I.  E 


50  CENTRE  OF  GRAVITY  OF  PARALLELOGRAMS 

co-ordinate  sr  or  sm,  can  very  easily  be  found;  for  through  the 
point  b  the  bisection  of  FC,  draw  bn  parallel  to  BC  and  meeting  AB 
perpendicularly  in  n  ;  then,  the  triangles  ban  and  mar  are  similar  to 
one  another,  and  the  sides  bn,  mr  and  an,  are  given  to  find  ar,  ajid 
from  thence  the  rectangular  co-ordinate  sr  or  sm\  consequently, 
we  have 

b  a  :  na  :  :  mr  :  r  a; 

therefore,  by  subtraction,  we  get 

BrorsrazzaB  —  r  a. 

Now,  £CZZBW.,  is  obviously  equal  to  half  the  difference  between 
D  c,  the  breadth  of  the  parallelogram,  and  DF,  the  base  of  the  triangle 
ADF  ;  therefore,  we  have 

B»  =  j(6.-0); 
but  an  zr  a  B  —  BW  ;  that  is,  a  n  zz  J/3,  and 


•••   3(24-0)- 

therefore,  by  reducing  the  analogy,  we  get 
_0(36  —  20). 
'   6(26-0)' 
hence,  by  subtraction,  we  obtain 

36(6-0)4-0* 

3(26  —  0)  (26). 

After  the  same  manner  that  equation  (25)  is  deducible  from  equa- 
tion (20),  by  putting  Z'zzZ;  so  also,  is  equation  (26)  deducible  from 
equation  (21),  by  means  of  the  same  equality  ;  we  might  therefore  have 
dispensed  with  the  preceding  investigation,  and  derived  the  expression 
from  principles  already  established  ;  we  however  preferred  obtaining 
it  as  above,  for  the  purpose  of  exhibiting  that  agreeable  variety  which 
gives  additional  embellishment  to  scientific  investigations.  The  method 
of  establishing  the  formulee,  on  the  supposition  that  the  side  BC  is 
horizontal,  is  sufficiently  obvious  from  what  has  been  done  in  the 
sixth  problem  preceding,  and  therefore,  it  need  not  be  repeated  here. 

COROL.  By  substituting  the  numerical  values  of  b  and  0,  as  given 
in  the  preceding  example,  we  shall  have  from  equation  (26) 


Therefore,  from  the  point  B,  set  off  BS  and  sr  respectively  equal  to 
16.8  and  8.4  feet;  and  through  the  points  s  and  r,  draw  sm  and  rm 
parallel  to  AB  and  BC,  the  perpendicular  sides  of  the  given  trapezoid, 


SUSTAINING  EQUAL  PRESSURES.  51 

and  meeting  one  another  in  the  point  m  ;  then  is  m  the  required  place 
of  the  centre  of  gravity. 

66.  In  computing  numerically  the  values  of  the  rectangular  co- 
ordinates mr  and  ms,  we  have  supposed,  that  DF  the  base  of  the 
applied  triangle,  is  determiriable  by  the  application  of  equation  (20)  ; 
this  supposition  however  is  perfectly  unnecessary,  for  the  base  of  the 
triangle  is  always  equal  to  the  difference  between  the  parallel  sides  of 
the  given  trapezoid ;  and  moreover,  the  equation  (20),  applies  only  to 
the  particular  case  for  which  it  has  been  deduced,  viz.  when  the 
pressure  on  the  applied  triangle  and  that  on  the  trapezoid  to  which  it 
is  applied  are  equal  to  one  another. 

9.    WHEN  THE  PARALLELOGRAM    IS    SO  DIVIDED,  THAT  THE  PRESSURES 
ON  THE  TWO  PARTS  ARE  TO  ONE  ANOTHER  IN  ANY  RATIO  WHATEVER. 

67.  In  the  sixth,  seventh  and  eighth  problems  preceding,  we  have 
supposed  the  given  rectangular  parallelogram  to  be  divided  into  two 
parts,  such,  that  the  pressures  upon  them  shall  be  equal  between 
themselves,  and  the  investigation  has  accordingly  been  limited  to  that 
particular  case  ;  but  in  order  to  render  the  solution  general,  we  shall 
consider  the  division  to  be  so  effected,  that  the  pressures  on  the  two 
parts  may  be  to  one  another  in  any  ratio  whatever,  such  as  that  of 
m  to  n, 

For  which  purpose  then,  by  referring  to  the  fifth  problem,  where  the 
given  parallelogram  is  divided  horizontally,  we  find,  that  the  pressure 
on  the  upper  portion  is  expressed  by  J6a;2ssin.^,  and  that  on  the 
lower  portion,  by  {  J  (I  —  x)*-\-x(l —  ,z)  }  6ssin.0 ;  but  these  ex- 
pressions in  their  present  state  are  equal  to  one  another,  and  they 
are  now  required  to  be  reduced  in  the  ratio  of  m  to  n ;  consequently, 
we  have 

1*':  \  k(l  —  x)*  +  x(l  —  x}}  ::m:n, 
and  this,  by  expanding  the  second  term,  becomes 

x*  :  I* —  a8  :  :  m  :  n; 
or  by  equating  the  products  of  the  extremes  and  means,  we  obtain 

n  a;2  nz  m  /2  —  ma?  \ 
therefore,  by  transposition,  we  get 

(m  -\-  n}  o^zr  mZ% 
and  finally,  by  division  and  evolution,  we  have 


E2 


52  SECTIONS  SUSTAINING  PRESSURES 

68.  Again,  in  the  case  of  the  sixth  problem,  where  the  given  paral- 
lelogram is  divided  by  a  line  drawn  parallel  to  the  diagonal  ;  we  find, 
that  the  pressure  on  the  triangle  cut  off  by  the  line  of  division,  is 
expressed  by  bx*s(3l  —  x)  sin.0  -H  6  /,  and  consequently,  by  sub- 
traction, that   on   the  remaining  portion  is   expressed   by   bssm  .<f> 
{  3  la  —  x*  (3  I  —  #)  }  x  ~  6  /  ;  now,  these  expressions,  by  the  condi- 
tions of  the  problem,  are  equal  to  one  another  ;  but  in  the  present 
case,  they  are  to  be  reduced  in  the  ratio  of  m  to  n  ;  for  which  purpose 

we  have 

a*  (3  /  —  x)  :  3  /8  —  x*(3  I  —  x)  :  :  m  :  n  ; 

therefore,  by  equating  the  products  of  the  extreme  and  mean  terms, 
we  get 

nx*(3l  —  x)  —  3ml3  —  mx*(3l  —  a?)  ; 
and  from  this,  by  transposition,  we  shall  obtain 

(m  +  »)  (3  lx*  —  xs)  =  3  m  Is  ; 

therefore,  by  dividing  and  transposing  the  terms,  we  have 

3ml9 

(28). 

Now,  in  order  to  reduce  the  above  equation,  there  must  be  substi- 
tuted the  numbers  which  express  the  given  ratio,  together  with  the 
length  of  the  parallelogram,  and  then,  the  value  of  a;  will  be  obtained 
by  any  of  the  rules  for  resolving  cubic  equations. 

69.  In  like  manner  as  above,  by  referring  to  the  eighth  problem, 
where  the  given  parallelogram  is  divided  by  a  line  drawn  from  one  of 
the  upper  angles,  and  terminating  in  the  lower  side  ;  we  find,  that  the 
pressure  on  the  triangle  cut  off  by  the  line  of  division,  is  expressed  by 
^/8#ssin.0,  and  consequently,  by  subtraction,  the  pressure  on  the 
remaining  portion  is  expressed  by  ±Fssm.(f>(3b  —  2#);  and  these 
expressions,  according  to  the  conditions  of  the  problem,  are  equal  to 
one  another  ;  but  in  the  present  instance,  they  are  to  be  reduced  in 
the  ratio  of  m  to  n  ;  hence,  we  have 


consequently,  by  equating  the  products  of  the  extreme  and  mean 
terms,  we  get 

2nx  nr  3  bm  —  2  war, 

from  which,  by  transposition,  we  obtain 

2(m  +  n)x  =  3bm, 
and  finally,  by  division,  we  have 

36m 
•r-2(m-fn)'  (29). 


ANY  RATIO  TO  ONE  ANOTHER. 


53 


Hence  then,  the  equations  (27,)  (28,)  and  (29,)  express  generally 
the  relation  between  the  parts  of  division,  which  in  the  several  pro- 
blems is  restricted  to  a  ratio  of  equality ;  and  it  is  presumed,  that  by 
paying  a  due  attention  to  the  examples  that  have  been  proposed  and 
illustrated,  the  diligent  reader  will  find  no  difficulty  in  resolving  any 
example  that  may  present  itself  under  one  or  other  of  the  general 
forms  above  investigated. 

In  all  the  above  cases,  we  have  supposed  the  breadth,  or  that  side 
of  the  parallelogram  which  is  denoted  by  b  to  be  horizontal,  and 
coincident  with  the  surface  of  the  fluid ;  but  it  is  manifest,  that 
equations  of  the  same  form  would  be  obtained  from  the  other  side, 
having  b  in  place  of  I,  and  /  in  place  of  b. 


10.  OF  RECTANGULAR  PARALLELOGRAMS  DIVIDED  INTO  SECTIONS 
SUSTAINING  EQUAL  PRESSURES;  WITH  THE  METHOD  OF  DETER- 
MINING A  LIMIT  TO  THE  NECESSARY  THICKNESS  OF  FLOOD-GATES, 
AND  OTHER  CONSTRUCTIONS  OF  A  SIMILAR  NATURE. 

PROBLEM  IX. 

70.  A  given  rectangular  parallelogram,  is  immersed  in  an 
incompressible  and  non-elastic  fluid,  in  such  a  manner,  that  one 
of  its  sides  is  coincident  with  the  surface,  and  its  plane  inclined 
at  a  given  angle  to  the  horizon : — 

It  is  required  to  divide  the  rectangle  by  lines  drawn 
parallel  to  the  horizon,  into  any  number  of  parts,  such, 
that  the  pressures  on  the  several  parts  of  division  shall  be 
equal  to  one  another. 

Let  A  ED  represent  a  rectangular  cistern  filled  with  water,  or  some 
other  transparent  and  incompressible  fluid 
in  a  state  of  rest ;  one  side  of  the  vessel 
being  removed,  for  the  purpose  of  exhibit- 
ing the  fluid  and  the  immersed  parallelo- 
gram, together  with  the  several  subordinate 
lines  on  which  the  investigation  depends. 

Suppose  e,  I,  g  and  i  to  be  the  several 
points  of  division,  and  through  these  points 
draw  the  lines  em,  If,  gk  and  ih,  respec- 
tively parallel  to  a  b  or  dc,  the  horizontal  sides  of  the  figure.  Bisect 
the  sides  a  b  and  dc  in  the  points  G  and  H;  join  GH,  and  draw  the 
zigzag  diagonals  am,  ml,  Ik,  ki  and  ic,  cutting  the  bisecting  line 


54      HORIZONTAL  SECTIONS  CONTAINING  EQUAL  FLUID  PRESSURES. 

GH  in  the  points  z,  q,  p,  o  and  n,  which  points  are  the  respective 
centres  of  gravity  of  the  several  parts  into  which  the  given  parallelo- 
gram is  divided,  and  on  which  the  pressures  are  supposed  to  be  equal 
among  themselves. 

Through  the  point  G,  and  in  the  plane  of  the  horizon,  draw  the 
straight  line  or  at  right  angles  to  ab,  making  the  angle  ron  equal  to 
the  given  angle  of  the  plane's  inclination,  and  from  the  points  n,  o,p, 
q  and  z,  let  fall  the  perpendiculars  zv,  qu,  pt,  os  and  nr,  meeting 
the  line  or  respectively  in  the  points  v,  u,  t,  s  and  r;  then  are  the 
lines  zv,  qu,  pt,  os  and  nr,  the  perpendicular  depths  of  the  centres 
of  gravity  of  the  several  portions  into  which  the  proposed  rectangle  is 
divided,  the  points  of  division  being  estimated  from  the  surface  down- 
wards. 

Put£  =iab  or  dc,  the  horizontal  breadth  of  the  given  parallelogram 

abed, 
I  —  ad  or  be,  the  entire  immersed  length,  or  that  tending  down- 

wards, 

^  zzrGH,  the  given  angle  of  inclination, 

d  zn  vz,  the  vertical  depth  of  the  centre  of  gravity  of  the  part  abme, 
d  '  zz  w  £,  the  vertical  depth  of  -  e  mfl, 
d"  —  tp,  the  vertical  depth  of— 


S   —so,  the  vertical  depth  of  --  ykhi, 
I'  —  rn,  the  vertical  depth  of  --  iked', 
n  zzthe  number  of  parts  into  which  the  parallelogram  is  divided, 
P  zzrthe  entire  pressure  on  the  parallelogram  abed, 
p  nz  the  pressure,  common  to  each  of  the  parts  into  which  the 

given  parallelogram  is  divided, 

v  =n  ae,  the  required  length  of  the  upper  portion  abme, 
w  ~el,  the  length  of  the  second  portion  emfl, 
x  m  Ig,  the  length  of  the  third  portion  If  kg, 
y  —gi,  the  length  of  the  fourth  portion  gkhi, 
z  zn  id,  the  length  of  the  fifth  portion  i  hcd. 

Then,  according  to  equation  (7)  under  the  third  problem,  the  entire 
pressure  on  the  parallelogram  abed,  is 


therefore,  the  pressure  on  each  part  of  the  divided  figure,  is 
bl2 


But  because  the  value  of  $,  the  angle  of  inclination,  and  s  the 
specific  gravity  of  the  fluid,  are  the   same  for  all  the  parts  ;  those 


HORIZONTAL  SECTIONS  SUSTAINING  EQUAL  FLUID  PRESSURES.      55 

quantities  may  be  omitted  in  the  equation,  and  then  the  element  of 
comparison,  or  the  nthpart  of  the  total  pressure  becomes 

_bl* 
p-^~n  (30). 

Now,  according  to  the  principles  of  Plane  Trigonometry,  the  lines 
vz,  uq,  tp,  so  and  rn,  are  respectively  as  below,  viz. 


$  nz  G  o  sin.^>,  and  3'  zz  G  n  sin.0  ; 

but  the  lines  GZ,  G^,  Gp,  GO  and  GW,  when  expressed  in  terms  of  the 
respective  lengths,  are  as  follows,  viz. 

G2  =  |v;  Gq=:v-\-%w;  Gp  —  v  -f-  w  -f-  J#; 
Gozzv-j-w-|-a;4"  iy>  and  ftwss:*  +'IIP  +  *  +  y  +  J«; 

therefore,  by  substitution,  the  above  values  of  the  vertical  depths  of 
the  respective  centres  of  gravity,  become 

d  —  \v  sin.<£  ;  d'  zz:  (v  4-  Jw;)  sin.^  ;  dw  zz  (w  •+•  w  4-  Jar)  sin.0  ; 
5  zz  (v  4-  w  4-  a?  +  Jy)  sin.^,  and  ^  zz:  (v  4-  w  -f  x  4-  2/  +  |z)  sin.0. 

Consequently,  by  throwing  out  the  common  factor  sin.0  and 
neglecting  the  specific  gravity  of  the  fluid,  the  value  of  jo,  or  the 
pressure  sustained  by  each  of  the  parts,  may  be  expressed  as  follows, 
viz. 

The  pressure  on  the  part  abme,  is  p~%bv*,  (1). 

-  em  fly  isp~bw  (v  -\-  Jw),  (2). 

-  If  kg,    isp=ibx(v-{-w+^,  (3). 

-  gkhi,   isp  =  by(v-\-w  +  x+  jy),      (4). 

-  ihcd,    isp  =  bz  (v  •+•  w  -{-  x+y+£z).(5). 

Now,  according  to  the  conditions  of  the  problem,  all  these  expres- 
sions for  the  value  of  p,  are  equal  to  one  another,  and  each  of  them  is 
equal  to  the  element  of  comparison,  as  given  in  the  equation  (30)  ; 
hence,  from  the  first  of  the  above  equations,  or  values  of  p,  we  have 


or  by  expunging  the  common  factor,  |6,  we  obtain 

*<  =  -?!; 
n 

therefore,  by  extracting  the  square  root,  we  have 


56       HORIZONTAL  SECTIONS  SUSTAINING  EQUAL  FLUID  PRESSURES. 

Again,  by  comparing  equation  (30),  with  the  second  of  the  fore- 
going expressions  for  the  value  of  p,  we  shall  have 

bl* 
bw(v  +  \w}=  —  - 

and  substituting  the  value  of  v  in  terms  of  /  and  n,  we  obtain 

n  w*  -f-  2/-V//T7  w  —  P, 
from  which,  by  dividing  by  n,  we  get 


w    --  —  .  w  •=.  —  > 
n  n 

therefore,  if  this  be  reduced  by  the  rule  which  applies  to  the  resolution 

of  adfected  quadratic  equations,  we  shall  obtain 

I       _          ___ 

w  :rr  —  -(  ^/2w  —  Vn)' 
n 

Proceed  as  above,  by  comparing  the  equation  (30),  with  the  third 
of  the  preceding  expressions  for  the  value  of  p,  and  we  shall  have 

=  — 

and  if  the  above  values  of  v  and  w,  as  expressed  in  terms  of  I  and  w, 
be  substituted  instead  of  them  in  this  equation,  it  will  become 


and  dividing  by  n,  we  get 


n  n 

therefore,  by  completing  the  square,  evolving  and  transposing,  we 
obtain 


By  pursuing  a  similar  mode  of  comparison,  and  reasoning  in  the 
same  manner,  with  respect  to  the  fourth  value  of  p  foregoing,  we 
shall  have 


let  the  values  of  v,  w  and  x,  as  determined  above,  be  respectively 
substituted  in  this  equation,  and  it  becomes 


complete  the  square,  and  we  obtain 


PARALLELOGRAM  DIVIDED  TO  SUSTAIN  EQUAL  FLUID  PRESSURES.  57 

and  from  this,  by  evolution  and  transposition,  we  get 


Pursuing  still  the  same  mode  of  induction  for  the  fifth  value  of  p, 
and  substituting  the  respective  values  of  v,  w,  x  and  y,  as  we  have 
determined  them  above  in  terms  of  I  and  n  ;  then  we  shall  have 

z  —  —  {  |/5»  —  \/4n~}. 

And  in  like  manner  we  may  proceed  for  any  number  of  divisions  at 
pleasure  ;  but  what  we  have  now  done  is  sufficient  to  exhibit  the  law 
of  induction. 

The  formulae  which  we  have  investigated,  for  determining  the 
several  sections  of  the  given  parallelogram,  may  now  be  advantage- 
ously collected  into  one  place  ;  for  it  is  manifest,  that  by  exhibiting 
them  in  juxta-position,  the  law  of  their  formation  is  more  easily 
detected,  and  the  difference  which  obtains  between  the  co-efficients  of 
the  successive  terms  becomes  at  once  assignable. 

The  several  equations  therefore,  when  arranged  according  to  the 
order  of  the  corresponding  sections,  will  stand  as  under,  viz. 


3-  x  =  —  (V^- 


&C.&C.ZZ  &C.  &C. 

71.  The  practical  rule  for  determining  the  points  of  section,  in 
reference  to  their  respective  distances  from  the  upper  extremity  of  the 
parallelogram,  may  be  expressed  in  words,  as  follows,  viz. 

RULE.  Multiply  the  number  of  parts  into  which  the  paral- 
lelogram is  proposed  to  be  divided,  by  the  number  that  indicates 
the  place  of  any  particular  section;  then,  multiply  the  square 


58  PARALLELOGRAM  DIVIDED  TO  SUSTAIN  EQUAL  FLUID  PRESSURES. 

root  of  the  product  by  the  length  of  the  parallelogram,  and 
divide  by  the  whole  number  of  sections,  and  the  quotient  will 
express  the  distance  of  any  particular  point  from  the  upper 
extremity  of  the  divided  length. 
If  the  length  of  any  particular  section,  or  the  distance  between  any 

two  contiguous  points  should  be  required,  which  is  the  condition 

expressed  by  each  of  the  above  equations  ;  then, 

Calculate  for  each  of  the  points  according  to  the  preceding 
rule,  and  the  difference  of  the  results  will  give  the  length  of 
the  required  section. 

72.  EXAMPLE  12.  A  rectangular  parallelogram  whose  length  is  25 
feet,  is  perpendicularly  immersed  in  a  fluid,  in  such  a  manner,  that 
its  breadth  or  upper  side  is  just  in  contact  with  the  surface ;  now,  if 
it  be  proposed  to  divide  the  parallelogram  by  lines  drawn  parallel  to 
the  horizon,  into  five  parts  sustaining  equal  pressures  ;  it  is  required 
to  determine  the  distance  of  each  point  of  section  from  the  surface  of 
the  fluid,  and  the  respective  distances  between  the  several  points  ? 

Here  then,  we  have  given  I  —  25  feet,  and  n  =r  5  an  abstract 
number ;  therefore,  by  proceeding  according  to  the  rule,  we  shall  have, 
for  the  distance  of  the  first  point, 

25V5---  5  =  11. 18034  feet. 
For  the  distance  of  the  second  point,  we  get 

25V2  x  5  +  5  =  15.81139  feet. 
For  the  distance  of  the  third  point,  we  obtain 

25  /3~x^5  H-  5  =  19.36492  feet, 
and  for  the  distance  of  the  fourth  point,  it  is 

25  V  4~X~5  ~  5  =  22.36068  feet. 

The  preceding  is  all  that  is  necessary  to  be  calculated,  for  the 
distance  of  the  fifth  point  is  manifestly  equal  to  the  whole  length  of 
the  parallelogram,  and  consequently,  by  the  question,  it  is  a  given 
quantity. 

Having  thus  determined  the  distances  of  the  several  points  of 
section  below  the  upper  surface  of  the  fluid,  the  respective  distances 
between  them,  or  the  breadths  of  the  several  sections  can  easily  be 
ascertained,  since  they  are  merely  the  consecutive  differences  of  the 
quantities  above  calculated ;  hence,  we  have 

Distances  11.18034,  15.81139,  19.36492,  22.36068,  25. 

Differences          4.63105,  3.55353,  2.99576,  2.63932 ; 


LIMIT  TO  THE  REQUISITE  THICKNESS  OF  FLOOD-GATES.  59 

therefore,  the  breadths  of  the  respective  sections,  estimated  in  order 
from  the  surface  of  the  fluid,  are 

11.18034,  4.63155,  3.55352,  2.99576,  2.63932  feet. 
Let  A  BCD  be  the  rectangular  parallelogram,  whose  length  AD  or 
B c  is  equal  to  25  feet,  and  the  breadth  A B  or  DC 
of  any  convenient  magnitude  at  pleasure.    Upon 
the  length  A  D,  set  off  the  distances  AE,  A  a,  A  b 
and  AC,  respectively  equal  to  the  preceding  num- 
bers, taken  in  the  order  of  their  arrangement, 
and  through  the  points  E,  a,  b  and  c,  draw  the 
straight  lines  EF,  af,   be  and  cd,  respectively 
parallel  to  AB  or  DC  the  horizontal  sides  of  the  parallelogram ;  then 
are  the  rectangular  spaces  AF,  ae,  bd  and  cc,  the  respective  portions 
into  which  the  given  parallelogram  ABCD  is  divided,  and  on  which, 
according  to  the  conditions  of  the  problem,  the  perpendicular  pres- 
sures are  equal  among  themselves. 

OF  THE  REQUISITE  THICKNESS  OF  FLOOD-GATES,  &C. 

73.  The  problem  which  we  have  just  resolved  is  a  veiy  important 
one;  by  it  we  can  determine  a  limit  to  the  requisite  thickness  of 
flood-gates  and  other  constructions  of  a  similar  nature,  and  also  the 
form  which  the  section  ought  to  assume,  in  order  that  the  strength  in 
every  part  may  be  proportional  to  the  pressure  sustained. 

For  according  to  the  preceding  notation,  and  by  equation  (7)  under 
the  third  problem,  the  pressure  on  the  rectangle  ABFE,  is 

p  —  J6v2ssin.^, 
and  the  pressure  on  the  whole  rectangle  ABCD,  is 

Pzz:  ^bFssin.ty; 
consequently,  by  analogy  and  comparison,  we  get 

P:p:i?i  v\ 

And  in  like  manner  it  may  be  shown,  that  the  same  relation 
obtains  in  respect  of  any  other  rectangle  AB/a,  when  compared 
with  the  entire  figure  ABCD;  consequently,  the  pressures  are  uni- 
versally as  the  squares  of  the  depths,  the  breadth  being  constant ; 
therefore,  the  thickness  should  be  as  the  square  of  the  depth,  being 
greatest  at  the  bottom  and  decreasing  upwards  to  the  surface  of  the 
fluid. 

Thus  for  example,  let  the  flood-gate  be  of  the  same  depth  as  the 
rectangle  in  the  foregoing  question,  and  let  the  thickness  at  the 
bottom  be  equal  to  one  foot  or  twelve  inches ;  then,  the  corresponding 
thicknesses  for  the  several  feet  of  ascent  estimated  upwards,  will  be  as 
follows. 


60 


LIMIT  TO  THE  REQUISITE  THICKNESS  OF  FLOODGATES. 


At  25  feet,  we 

have  25s  :  252 

OC2    .    0x14 

:  12  :  12  inches. 
•  ln  •  11  060  • 

058    .    032 

•  12  :  10.156     •    - 

r»o 

,   OC2   .   00« 

•  1°  •    9  °9° 

O1 

OAJ2   .    012 

•  1°  •     8  4fi8 

on 

052   .    OQ2 

•  1°  •    7  680  

in 

.  °52  •  192 

•  12  •    6  93° 

1  9 

,   052   .    1  g2 

.    JO    .        Q   OOQ 

V7 

..    OA2    .    172 

•  1°  •     5  'i/IR 

Ifi 

OC2    .    Ifi2 

.  10  .      xi  qcf? 

1  ^ 

052    .    ^52 

•  1°  •    1  3°0         • 

\*\ 

OC2    .     1/12 

•  1°  •    S  7fi^l 

11 

0  «   .    1  02 

.      10    .        0    0^^        ,     .,  .. 

ln 

,   0«   .    102 

.  10  .     o  7«4 

0  £2   .    112 

.  10  .     o  qo^  , 

10 

052    .    1Q2 

•  1°  •    1  9°0 

o 

052  .     os 

•  1°  •    1  556  • 

Q 

.    0  C2  .       Q« 

.    10    .        1    000 

7 

o«  .     72 

.    10    .       0  C]A()   , 

fi  . 

0«   .       f!2 

.  10  .     o  fiQ2 

* 

052   .       y, 

•  1°  •     0  160 

/I 

OC2   .       /|2 

•  1°  •    0  308  • 

'}  j 

052   .        02 

•  10  .     017° 

o 

OC2   .       02 

.  10  •     n  07fi  - 

1 

9/5*  :      I8 

•  19  •     0.090  ..      - 

74.  Having  calculated  the  numerical  values  of  the  thicknesses  or 
ordinates,  corresponding  to  each  foot  in  length,  estimated  from  the 
bottom  where  the  pressure  is  a  maximum,  upwards  to  the  summit 
where  it  vanishes ;  we  shall  now  proceed  to  construct  the  section,  in 
order  to  exhibit  the  particular  form  which  the  preceding  theory  assigns. 

Let  the  straight  line  A  B  represent  the  perpen- 
dicular depth  of  the  flood-gate  supporting  the 
fluid  F,  and  whose  vertical  section  is  denoted  by 
the  figure  ABC,  the  exterior  boundary  of  which  is 
the  curve  line  ABC,  and  the  greatest  thickness, 
or  that  at  the  bottom,  equal  to  BC. 

Divide  the  depth  AB  into  twenty  five  equal 
parts,  having  an  interval  of  one  foot  for  each ; 
then,  through  the  several  points  of  division,  and  parallel  to  the  horizon, 
draw  straight  lines,  beginning  at  the  bottom  and  proceeding  upwards, 
making  these  lines  respectively  equal  to  12,  11.06,  10.156,  9.292,  &c. 


OF  FLUID  PRESSURE  ON  THE  SIDES  AND  BASE  OF  CUBICAL  VESSELS.    61 

inches,  according  to  the  numbers  in  the  foregoing  tablet,  and  through 
the  remote  extremities  of  the  several  ordinates,  trace  the  curve  line 
ADC,  which  will  mark  the  exterior  boundary  of  the  section. 

The  intelligent  reader  will  readily  perceive,  that  in  the  actual  con- 
struction of  the  above  figure,  it  has  been  found  impossible  to  preserve 
the  proper  proportion  between  the  several  abscissse  and  their  corre- 
sponding ordinates ;  if  this  had  been  attempted,  the  figure  must  either 
have  been  enlarged  to  an  inconvenient  size,  or  the  ordinates  would 
have  been  so  small  as  to  render  the  general  appearance  of  the  section 
very  indistinct. 

We  have  therefore  thought  it  preferable  to  preserve  the  line  of  the 
abscissee  within  moderate  bounds,  and  to  enlarge  the  ordinates  in  a 
given  constant  ratio ;  by  this  means  the  form  of  the  curve  is  correct, 
and  the  whole  diagram  is  sufficiently  distinct  for  practical  illustration. 

In  all  that  has  hitherto  been  done  respecting  the  rectangular 
parallelogram,  we  have  constantly  considered  it  as  being  an  inde* 
pendent  plane  immersed  in  the  fluid,  and  having  its  upper  side 
coincident  with  the  surface ;  but  we  must  now  observe,  that  whatever 
relations  have  been  shown  to  exist  on  such  a  supposition,  the  same 
will  hold,  if  the  plane  be  considered  as  the  side  of  a  vessel  filled  with 
the  fluid  by  which  the  pressure  is  propagated. 

We  have  already  alluded  to  this  principle,  at  the  conclusion  of  our 
illustration  of  the  fourth  problem ;  it  therefore  only  remains  to  deter- 
mine by  it,  the  pressure  on  the  bottom  and  sides  of  a  vessel  filled  with 
a  fluid  of  uniform  density,  on  the  supposition  that  the  bottom  and  the 
sides  are  respectively  rectangular  planes. 


11.  METHOD  OF  COMPARING  THE  PRESSURE  ON  THE  PERPENDICULAR 
SIDES  AND  ON  THE  BOTTOM  OF  ANY  RECTANGULAR  CISTERN,  BASIN, 
OR  CANAL  LOCK. 

PROBLEM  X. 

75.  Suppose  that  a  vessel  in  form  of  a  rectangular  paral- 
lelopipedon,  is  filled  with  fluid  of  uniform  density,  and  placed 
with  its  sides  perpendicular  to  the  horizon : — 

It  is  required  to  compare  the  pressure  on  the  upright  sides 
with  that  upon  the  bottom,  both  when  the  sides  are  all  equal, 
and  when  the  opposite  sides  only  are  equal. 


62  OF  FLUID  PRESSURE  ON  THE  SIDES  AND  BASE  OF  CUBICAL  VESSELS. 

In  the  solution  of  the  present  problem,  it  will  be  unnecessary  to 
exhibit  the  construction  of  a  separate  diagram  ;  because,  the  bound- 
aries when  considered  individually,  being  rectangular  parallelograms, 
the  investigation  for  each  would  be  similar  to  that  required  in  Pro- 
blem 4,  and  the  resulting  formulae  would  coincide  in  form  with  that 
exhibited  in  equation  (8). 

Therefore,  put  b  =  the  horizontal   breadth  of  the   greater  opposite 

sides, 

/3  n:  the  horizontal  breadth  of  the  lesser  ditto  ditto, 
I  z=  the  perpendicular  depth  of  the  fluid,  whose  den- 

sity is  uniform, 
P=  the  aggregate,  or  total  pressure  on  the  upright 

surface,  and 
p  zz  the  pressure  on  the  bottom. 

Then,  according  to  equation  (8)  under  Problem  3,  the  pressure  on 
each  of  the  narrower  sides  is  expressed  by  |/3/2s,  and  that  on  each  of 
the  broader  sides  by  \bl*s\  consequently,  the  entire  pressure  on  the 
upright  surface,  is 

P±=l'«04..&)< 

and  by  the  third  inference  to  Proposition  (A),  the  pressure  sustained 
by  the  bottom  of  the  vessel,  is 


consequently,  by  analogy,  we  obtain 
P:p::l(P  +  b):pb. 

Therefore,  when  the  opposite  sides  of  the  rectangular  vessel  only, 
are  equal  to  one  another, 

The  total  pressure  on  the  upright  surface,  is  to  the  pressure 
on  the  bottom,  as  half  the  area  of  the  former  is  to  the  area  of 
the  latter. 

If  b  in  /3,  or  if  all  the  sides  of  the  vessel  are  equal  to  one  another  ; 
then,  the  entire  pressure  on  the  upright  sides,  is 


and  that  on  the  bottom,  is 

p=b*ls; 

therefore,  by  analogy,  we  obtain 
P  :p  ::  2/  :  6. 


OF  FLUID  PRESSURE  ON  THE  SIDES  AND  BASE  OF  CUBICAL  VESSELS.    63 

Consequently,  when  all  the  four  sides  of  the  rectangular  vessel  are 
equal  to  one  another, 

The  total  pressure  on  the  upright  surface,  is  to  the 
pressure  on  the  bottom,  as  twice  the  length  of  the  side  is  to 
its  breadth. 

Again,  when  all  the  sides  of  the  vessel  have  the  same  breadth,  and 
the  length  I  equal  to  the  breadth  b  ;  then,  the  vessel  becomes  a  cube, 
and  the  total  pressure  on  the  upright  surface,  is 

P=263s, 

and  that  on  the  bottom,  is 
p  =  b9  s  ; 

therefore,  by  analogy,  we  obtain 

P  :p  ::  2  :  1. 

Hence  it  appears,  that  when  the  vessel  is  a  cube,  that  is,  when  the 
bottom  and  the  four  upright  sides  are  equal  to  one  another, 

The  total  pressure  upon  the  four  sides,  is  to  the  pressure  on 
the  bottom,  in  the  ratio  of  2  to  1 . 

Since  the  pressure  on  the  upright  surface  of  a  cubical  vessel,  is 
double  the  pressure  on  the  base ;  it  follows,  that  the  entire  pressure 
which  the  vessel  sustains,  is  equal  to  three  times  the  pressure  upon  its 
bottom ;  that  is, 

P+p=:'3b*s.  (32). 

But  the  expression  b*s  is  manifestly  equal  to  the  weight  of  the 
fluid ;  consequently,  the  total  pressure  upon  the  sides  and  base  of 
the  vessel, 

Is  equal  to  three  times  the  weight  of  the  fluid  which  it 
contains. 

Now,  in  the  case  of  water,  where  the  specific  gravity  is  represented 
by  unity,  the  equation  marked  (32)  becomes 

P+p  =  3b*; 

but  when  the  dimensions  of  the  vessel  are  estimated  in  feet,  and 
the  pressure  expressed  in  pounds  avoirdupois,  of  which  62|  are  equal 
to  the  weight  of  one  cubic  foot  of  water ;  then,  the  above  equation  is 
transformed  into 

p'  =  187.56s.  (33). 

COROL.  This  equation  in  its  present  form  implies,  that  if  the  solid 
content  of  the  vessel  in  cubic  feet,  be  multiplied  by  the  constant 


64    OF  FLUID  PRESSURE  ON  THE  SIDES  AND  BASE  OF  CUBICAL  VESSELS. 

co-efficient  187.5,  the  product  will  express  the  number  of  Ibs.  to 
which  the  pressure  on  the  bottom  and  the  four  upright  sides  is 
equivalent. 

76.  EXAMPLE  13.  Suppose  the  length  of  the  side  of  a  cubical 
cistern  to  be  35  feet ;  what  is  the  pressure  sustained  by  it,  when  it  is 
completely  filled  with  water  ? 

Here  we  have  given,  b  zz  35  feet ;  therefore,  by  proceeding  accord- 
ing to  the  composition  of  the  foregoing  equation,  we  shall  obtain 

p'  =  35  X  35  x  35  X  187.5=  8039062.5  Ibs. 
Hence  it  appears,  that  the  aggregate  pressure  upon  the  bottom 
and  the  upright  surface  of  a  cubical  vessel  whose  side  is  35  feet, 
is  8039062.5  Ibs.  or 

8039062.5  -f-  2240  =  3588.867  tons, 

while  the  absolute  weight  of  the  contained  fluid,  is  only  one  third 
of  that  quantity,  or 

35  X  35  x  35  X62.5  -;   2240  =  1196.289  tons. 

77.  EXAMPLE  14.  Let  the  dimensions  of  the  vessel  remain  as  in  the 
preceding  example,  and  suppose  it  to  be  filled  with  wine,  of  which  the 
specific  gravity  is  .96,  instead  of  water,  whose  specific  gravity  is  unity, 
what  pressure  does  it  then  sustain  ? 

For  the  weight  of  a  cubic  foot  of  wine,  we  have 

1  :  62.5  :  :  .96  :  60  Ibs. 

and  the  pressure  on  the  bottom  and  sides  of  a  vessel  containing  a 
cubic  foot  is  60  X  3  zz:  180  Ibs. ;  consequently,  the  pressure  on  the 
bottom  and  sides  of  a  vessel  whose  side  is  35  feet,  is 

p'  =  35  X  35  X  35  X  180  =  7717500  Ibs., 
and  this,  by  reducing  it  to  tons,  is 

= 3445-3125 tons'  ;  :" 

while  the  absolute  weight  of  the  contained  fluid,  is  only  one-third  of 
that  quantity,  or 

35  X  35  X  35  x  60  4-  2240  =  1 148.4375  tons. 


CHAPTER  III. 


t)N     THE     PRESSURE     EXERTED     BY     NON-ELASTIC     FLUIDS 

PARABOLIC  PLANES  IMMERSED  IN  THOSE  FLUIDS,  WITH  THE 
METHOD  OF  FINDING  THE  CENTRE  OF  GRAVITY  OF  THE  SPACE 
INCLUDED  BETWEEN  ANY  RECTANGULAR  PARALLELOGRAM  AND 
ITS  INSCRIBED  PARABOLIC  PLANE. 


1.  WHEN  THE  AXIS  OF  THE  PARABOLIC  PLANE  IS  PERPENDICULAR  TO 
THE  HORIZON,  AND  ITS  VERTEX  COINCIDENT  WITH  THE  SURFACE 
OF  THE  FLUID. 

PROBLEM  XL 

77.    If  a  parabolic  plane  be  just  perpendicularly  immersed 
beneath  the  surface  of  an  incompressible  fluid  :  — 

It  is  required  to  compare  the  pressure  upon  it,  with  that 
upon  its  circumscribing  rectangular  parallelogram,  and  to 
determine  the  intensity  of  pressure,  according  as  the  vertex 
or  the  base  of  the  parabola  is  in  contact  with  the  surface 
of  the  fluid. 

First,  when  the  vertex  of  the  parabola  is  uppermost,  and  just  in 
contact  with  the  surface  of  the  fluid  ;  let  AC  B 
be  the  parabolic  plane,  of  which  AB  is  the 
base  or  double  ordiriate  parallel  to  the  hori- 
zon, and  CD  the  vertical  axis  just  covered  by 
the  fluid  whose  surface  is  EF,  and  let  ABFE 
be  a  rectangular  parallelogram  circumscribing 
the  parabola. 

Now,  it  is  demonstrated  by  the  writers  on 
mechanics,  that  the  centre  of  gravity  of  a  parabolic  plane  is  situated 
in  the  vertical  axis,  and  the  point  where  it  occurs,  is  at  the  distance 
of  three  fifths  of  that  axis  from  the  summit  of  the  figure. 

Therefore,  if  the  axis  CD  be  divided  at  m,  into  two  parts  such,  that 
cm  is  to  Dm  as  3  is  to  2,*  then  is  m  the  centre  of  gravity  of  the 

VOL.  I.  F 


66  OF  THE  PARALLELOGRAM   AND  ITS  INSCRIBED  PARABOLA. 

parabolic  space  ACB  ;  and  if  the  axis  CD  be  bisected  in  the  point  n  ; 
n  is  the  centre  of  gravity  of  the  circumscribing  rectangular  parallelo- 
gram ABEF. 

Put  b  zz  A  B,  the  base  of  the  parabola,  or  the  horizontal  breadth  of  its 
circumscribing  rectangular  parallelogram, 

/—CD,  the  vertical  axis  of  the  parabola,  or  the  depth  of  its  sur- 
rounding rectangle, 

dm  en,  the  depth  of  the  centre  of  gravity  of  the  parallelogram 
ABFE  below  the  upper  surface  of  the  fluid, 

S  zz  cm,  the,  depth  of  the  centre  of  gravity  of  the  parabola  ACB, 

Pzz  the  pressure  on  the  circumscribing  rectangle, 

p  zz  the  pressure  on  the  parabolic  plane, 

A  zz  the  area  of  the  parallelogram, 

a  zz  the  area  of  the  parabola,  and 

s  —  the  specific  gravity  of  the  fluid. 

Then,  according  to  the  writers  on  mensuration,  the  area  of  the  cir- 
cumscribing rectangular  parallelogram,  is 

A  =  bl', 

but  the  area  of  a  parabola,  is  equal  to  two  thirds  of  the  area  of  its 
circumscribing  rectangle  ;  therefore,  we  have 

a  zz  %bl. 

Now,  5  zz  %l  by  the  construction,  and  we  have  shown  in  Proposition 
(A),  that  the  pressure  upon  any  surface, 

Is  expressed  by  the  area  of  that  surface,  drawn  into  the 
perpendicular  depth  of  its  centre  of  gravity,  and  also  into  the 
specific  gravity  of  the  fluid. 

Consequently,  the  pressure  perpendicular  to  the  surface  of  the 
parabolic  plane,  is 

/>==#/  X#X  *  =  lbP*.  (34). 

But  in  order  to  compare  the  pressure  on  the  parabola,  as  repre- 
sented by,  or  implied  in  the  above  equation,  with  that  upon  its  cir- 
cumscribing parallelogram,  we  have  only  to  recollect,  thatc?zz|/, 
and  consequently,  the  pressure  on  the  rectangle,  is 


consequently,  by  omitting  the  common  factors,  and  rendering  the 
fractions  similar,  we  have 

p  :  P  :  :  4  :  5. 

78.  Now,  the  practical  rule  for  determining  the  pressure  on  the 
parabolic  plane,  as  deduced  from  the  equation  (34),  or  from  the  pre- 
ceding analogy,  may  be  expressed  in  words,  as  follows. 


OF  THE  PARALLELOGRAM  AND  ITS  INSCRIBED   PARABOLA.  67 

RULE.  Multiply  two  fifths  of  the  base  of  the  parabola,  by 
the  square  of  the  length  of  its  axis  drawn  into  the  specific 
gravity  of  the  fluid,  and  the  product  will  express  the  pressure 
sustained  by  the  parabolic  plane,  in  a  direction  perpendicular 
to  its  surface.  Or  thus  : 

Find  the  pressure  on  the  circumscribing  rectangular  paral- 
lelogram, according  to  the  second  case  of  the  rule  under  the 
third  problem,  and  four  fifths  of  the  pressure  so  determined, 
will  express  the  pressure  perpendicular  to  the  parabolic 
surface. 

79.  EXAMPLE  14.  A  parabolic  plane,  whose  base  and  vertical  axis 
are  respectively  equal  to  28  and  42  feet,  is  perpendicularly  immersed 
in  a  reservoir  of  water,  so  that  its  vertex  is  just  in  contact  with  the 
surface ;  what  weight  is  equivalent  to  the  pressure  on  the  plane,  the 
weight  of  a  cubic  foot  of  water  being  equal  to  62  J  Ibs.  ? 

Here,  according  to  the  rule,  we  have 
p  =  f  x  28  X  422  X  62J  =  1234800  Ibs., 

or  by  the  second  clause  of  the  rule,  it  is 
p  —  \  X  28  x  42*  X  62J  X  *  =  1234800  Ibs. 
Either  of  these  methods  is  sufficiently  simple  for  every  practical 
purpose  ;  but  it  will  be  found  of  essential  advantage,  to  bear  in  mind 
the  relation  between  the  pressure  on  the  parabola  and  that  on  its 
circumscribing  rectangle ;  for  which  reason,  the   latter  method  may 
probably  claim  the  preference. 

2.  METHOD  OF  FINDING  THE  CENTRE  OF  GRAVITY  OF  THE  SPACE 
INCLUDED  BETWEEN  ANY  RECTANGULAR  PARALLELOGRAM  AND 
ITS  INSCRIBED  PARABOLA. 

80.  It  is  a  principle  almost  self-evident,  that  the  centre  of  gravity, 
and  the  centre  of  magnitude  of  a  rectangular  parallelogram,  exist  in 
one  and  the  same  point ;  consequently,  admitting  the  position  of  the 
centre  of  gravity  of  the  rectangle  to  be  known  or  determinable  a  priori, 
the  position  of  the  centre  of  gravity  of  the  inscribed  parabola  can 
very  readily  be  found. 

For  by  knowing  the  position  of  the  centre  of  gravity  of  a  rectangu- 
lar surface,  the  pressure  upon  it  can  easily  be  ascertained,  and  we 
have  shown  above,  that  the  pressure  upon  a  parabolic  plane,  and  that 
upon  the  surface  of  its  circumscribing  parallelogram,  are  to  one 
another  in  the  ratio  of  4  to  5  ;  hence,  when  the  pressure  on  the 

F2 


68  OF  THE  PARALLELOGRAM  AND  ITS  INSCRIBED  PARABOLA. 

rectangular  parallelogram  is  known,  the  pressure  on  the  inscribed 
parabola  is  also  known,  being  equal  to  four  fifths  of  that  upon  the 
parallelogram. 

Now,  it  has  already  been  demonstrated,  that  the  pressure  upon  any 
surface,  whatever  may  be  its  form,  is  always  equal  to  its  area,  drawn 
into  the  perpendicular  depth  of  the  centre  of  gravity,  below  the  upper 
surface  of  the  fluid  ;  therefore,  conversely,  the  perpendicular  depth 
of  the  centre  of  gravity  of  any  surface,  is  equal  to  the  pressure  which 
it  sustains,  divided  by  the  area. 

But  from  what,  has  been  demonstrated  above,  it  is  manifest  that 
the  area  of  the  parabola  and  the  pressure  upon  it,  are  respectively 
expressed  by 

f&Z,  and  f&/2s; 

consequently,  by  division,  we  obtain 


and  when  s  is  expressed  by  unity,  as  in  the  case  of  water,  we  get 

a=# 

Now,  because  the  parabola  ACB  is  symmetrically  divided  by  the 
axis  CD,  it  follows,  that  the  centre  of  gravity  occurs  in  that  line,  and 
we  have  just  shown,  that  it  occurs  at  the  distance  of  three  fifths  of  its 
length  from  the  vertex  ;  hence,  the  position  is  determined,  and  that 
independently  of  computing  the  corresponding  horizontal  rectangular 
co-ordinate,  whose  intersection  with  the  axis  fixes  the  place  of  the 
centre  sought. 

The  aggregate  pressure  upon  the  two  equal  and  similar  spaces  A  E  c 
and  BFC,  is  obviously  equal  to  the  difference  between  the  pressures 
on  the  rectangular  parallelogram  ABFE,  and  that  on  the  inscribed 
parabola  A  c  D  ;  that  is, 


where  p'  denotes  the  pressure  on  the  spaces  A  EC  and  BFC. 

Again,  the  area  of  the  spaces  A  EC  and  BFC,  is  equal  to  the  dif- 
ference between  the  area  of  the  parallelogram  ABFE,  and  that  of  the 
inscribed  parabola  ACB;  therefore,  if  a1  denote  the  area  of  the  trian- 
gular spaces,  we  have 

a'  —  A  —  a=zbl  —  %bl  =  %bl. 

But  the  depth  of  the  centre  of  gravity  of  any  surface,  is  equal  to 
the  pressure  upon  that  surface  divided  by  its  area  ;  consequently,  the 
depth  of  the  centre  of  gravity  of  the  figure  A  EC  FB,  which  is  composed 
of  the  two  triangular  spaces  AEC  and  BFC,  is 


OF  THE  PARALLELOGRAM  AND  ITS  INSCRIBED  PARABOLA. 


hence,  if  the  specific  gravity  of  the  fluid  be  expressed  by  unity,  we  get 

V  =  Thl.  (35). 

COROL.  It  therefore  appears,  that  the  centre  of  gravity  of  the  space, 
included  between  any  rectangular  parallelogram  and  its  inscribed 
parabola,  is  situated  in  the  axis,  at  the  distance  of  three  tenths  of  its 
length  from  the  vertex. 

The  preceding  investigation  determines  the  place  of  the  centre  of 
gravity  to  be  at  three  tenths  of  the  axis  below  a  tangent  line  passing 
through  the  vertex  of  the  parabola,  and  that  it  is  situated  in  the  axis 
is  manifest;  for  the  spaces  A  EC  and  BFC  are  equal,  and  they  are 
similarly  and  symmetrically  placed  with  respect  to  the  axis  c  D  ;  there 
can  therefore  be  no  reason,  why  the  centre  of  gravity  should  occur  at 
a  point,  which  is  nearer  to  the  one  than  it  is  to  the  other ;  it  conse- 
quently occurs  at  a  point  which  is  equally  distant  from  both,  and  that 
point  must  obviously  be  found  in  the  axis  of  the  figure. 

The  above  is  a  valuable  proposition  in  the  practice  of  bridge  build- 
ing ;  for  by  it,  we  can  readily  assign  the  position  of  the  centre  of  gravity 
of  the  arch  with  all  its  balancing  materials,  and  consequently,  many  im- 
portant particulars  respecting  the  weight  and  mechanical  thrust,  may 
be  determined  and  examined  with  the  greatest  facility :  all  of  which  will 
be  investigated  and  applied  in  our  treatise  on  Hydraulic  Architecture. 


3.    WHEN    THE    PARABOLIC    PLANE    PERPENDICULARLY    IMMERSED  HAS 
ITS  BASE  COINCIDENT  WITH  THE  SURFACE  OF  THE  FLUID. 

81 .  What  has  hitherto  been  done  under  the  present  problem,  applies 
only  to  the  case,  in  which  the  parabola  is  perpendicularly  immersed  in 
the  fluid,  and  having  its  vertex  coincident  with  the  surface ;  but  when 
the  parabolic  plane  is  perpendicularly  immersed,  and  having  its  base 
coincident  with  the  fluid  surface,  the  circumstances  will  be  something 
different,  as  will  become  manifest  from  the  following  investigation. 

Let  ABFE  be  a  rectangular  parallelogram  immersed  in  a  fluid,  with 
its  plane  perpendicular  to  the  plane  of  the 
horizon,  and  having  its  upper  side  coincident 
with  the  surface  of  the  fluid  ;  and  let  A  c  B  be 
a  parabola  described  upon  the  rectangular 
plane,  in  such  a  manner,  that  its  vertex  may 
be  downwards,  its  axis  vertical,  and  its  base 
in  contact  with  the  surface  of  the  fluid  in  which 
it  is  placed. 


JV  C 


70  OF  THE  PARALLELOGRAM  AND  ITS  INSCRIBED  PARABOLA. 

Draw  the  diagonal  AF  intersecting  CD,  the  vertical  axis  of  the 
parabola  in  the  point  n ;  then  is  n  the  centre  of  gravity  of  the  rectan- 
gular parallelogram  ABFE  ;  and  because,  as  we  have  stated  in  the 
construction  of  the  preceding  case,  the  centre  of  gravity  of  a  parabolic 
plane  is  situated  in  the  axis,  at  the  distance  of  three  fifths  of  its  length 
from  the  vertex  ;  it  follows,  that  if  the  axis  DC  be  divided  in  the  point 
m,  into  two  parts  such,  that  Dm  is  to  me  in  the  ratio  of  2  to  3  ;  then 
shall  m  be  the  centre  of  gravity  of  the  parabola  ACB. 

Put  6  =  AB,  the  horizontal  breadth  of  the  rectangular  parallelogram 

ABF,E,  or  the  base  of  its  inscribed  parabola  ACB, 
/  —  DC  or  AE,  the  vertical  axis  of  the  parabola,  or  the  depth  of 

its  circumscribing  rectangle, 

d  —  D  n ,  the  depth  of  the  point  n,  below  A  B  the  surface  of  the  fluid , 
£  nr  DW,  the  depth  of  the  point  m  as  referred  to  AB, 
P —  the  pressure  on  the  surface  of  the  rectangle  ABFE, 
j9=:the  pressure  on  the  parabolic  surface  ACB, 
Az=the  area  of  the  rectangular  parallelogram, 
a  —  the  area  of  its  inscribed  parabola,  and 
s  zn  the  specific  gravity  of  the  fluid. 

Now,  it  is  manifest  from  the  principles  of  mensuration,  that  the 
area  of  a  rectangular  parallelogram,  is  equal  to  the  product  that 
arises  when  the  two  dimensions  of  length  and  breadth  are  multiplied 
into  one  another ;  that  is, 

A=bl, 

and  according  to  the  writers  on  conic  sections,  the  area  of  a  parabola 
is  equal  to  two  thirds  of  the  area  of  its  circumscribing  rectangle ; 

therefore,  we  have 

oz=f6Z. 

Referring  to  the  construction  of  the  figure,  we  find  that  the  axis 
DC  is  divided  at  m,  into  the  two  parts  Dm  and  me,  having  to  one 
another  the  ratio  of  2  to  3  ;  it  therefore  follows,  that 

Dmzi:  £  —  -§:/; 
consequently,  the  pressure  on  the  parabolic  surface  ACB,  is 

p  —  ^blX^lXs  —  ^b  l*s.  (36). 

Again,  since  AF  the  diagonal  of  the  parallelogram,  bisects  DC  the 
axis  of  the  parabola  in  the  point  n ;  it  follows,  that 

Dw  =  d—|/; 

therefore,  the  pressure  perpendicular  to  the  surface  of  the  rectangular 
parallelogram  ABFE,  becomes 

P  —  blX  i/X  s= 


OF  THE  PARALLELOGRAM  AND  ITS  INSCRIBED  PARABOLA.  71 

hence,  by  analogy,  we  obtain 

p  :  P  ::T\bl*s  :  \bl*s, 
and  this,  by  omitting  the  common  quantities,  b,  I1  and  s,  becomes 

P  :  P  : :  rV  :  1, 

and  finally,  by  reducing  the  fractions  in  this  analogy  to  a  common 
denominator,  we  shall  obtain 

p  :  P  : :  8  :  15. 

82.  Having  thus  established  the  formula  for  determining  the  pres- 
sure on  the  parabolic  plane,  and  also  the  ratio  which  compares  the 
said  pressure  with  that  upon  the  circumscribing  rectangular  paral- 
lelogram ;  we  shall  in  the  next  place  deduce  the  rules,  by  which  the 
numerical  process  is  to  be  performed,  when  the  principle  is  applied  to 
the  actual  determination  of  the  pressure  in  reference  to  cases  of  prac- 
tice.    The  rule,  as  derived  from  the  equation  numbered  (36),  may  be 
expressed  in  the  following  manner. 

RULE.  Multiply  the  base  of  the  parabola  by  the  square  of 
its  vertical  axis,  and  again  by  the  specific  gravity  of  the 
fluid ;  then,  four  fifteenths  of  the  product  will  express  the 
pressure  perpendicular  to  the  surface  of  the  parabolic  plane. 

The  rule  for  determining  the  pressure  on  the  surface  of  a  parabola, 
as  deduced  from  the  analogy  of  comparison  investigated  above,  may 
be  expressed  in  words  in  the  following  manner. 

Find  the  pressure  perpendicular  to  the  surface  of  the 
circumscribed  rectangular  parallelogram,  after  the  manner 
described  in  the  second  case  of  the  rule  under  the  third 
problem;  then,  eight  fifteenths  of  the  pressure  so  determined, 
will  express  the  pressure  on  the  parabolic  plane. 

83.  EXAMPLE  15.  The  data  remaining  as  in  the  preceding  case  ; 
what  will  be  the  pressure  on  the  parabolic  plane,  its  axis  being  verti- 
cal, and  its  base  in  contact  with  the  surface  of  the  fluid  ? 

28  X  42  X  42  X  62J  =  3087000, 

and  four  fifteenths  of  this,  is 
3087000  X  4  -r  15  zz  823200  Ibs. 

This  result  is  derived  from  the  first  of  the  above  rules,  or  that  which 
corresponds  to  the  equation  (36) ;  and  the  process  as  performed  by  the 
second  rule,  or  that  obtained  from  the  ratio  of  comparison,  will  stand 
as  below. 

}  X  28  X  42  X  42  X  62£  X  TV  =  823200  Ibs. 


72  OF  THE  PARALLELOGRAM   AND  ITS  INSCRIBED  PARABOLA. 

Hence,  the  pressure  on  the  plane  in  this  case,  is  only  two  thirds  of 
what  we  found  it  to  be  in  the  foregoing  case,  where  the  vertex  is  in 
contact  with  the  surface  of  the  fluid. 

With  respect  to  the  position  of  the  centre  of  gravity  in  this  case,  it 
is  manifest,  that  the  mode  of  discovering  it,  is  similar  to  that  which 
we  employed  in  the  case  immediately  preceding,  where  the  axis  of  the 
parabola  was  supposed  lo  be  vertical,  and  its  s-ummit  in  contact  with 
the  surface  of  the  fluid  ;  it  is  therefore  unnecessary  to  repeat  the 
investigation,  but  there  is  another  condition  of  the  figure  remaining 
to  be  considered,  in  which  a  knowledge  of  the  position  of  the  centre 
of  gravity  becomes  of  more  importance,  as  will  readily  appear  from 
the  circumstances  which  present  themselves  in  the  solution  of  the 
following  problem. 

4.  WHEN  THE  BASE  OF  THE  PARABOLIC  PLANE  IS  PERPENDICULAR  TO 
THE  HORIZON,  ITS  AXIS  HORIZONTAL,  AND  THE  PRESSURE  UPON 
IT  IS  TO  BE  DETERMINED  AS  COMPARED  WITH  THAT  UPON  ITS 
CIRCUMSCRIBING  RECTANGULAR  PARALLELOGRAM. 

PROBLEM  XII. 

84.  If  a  parabolic  plane  be  perpendicularly  immersed  in  an 
incompressible  fluid,  in  such  a  manner,  that  its  base  may  be 
vertical,  and  just  in  contact  with  the  surface : — 

It  is  required  to  determine  the  pressure  upon  it,  and  to 
compare  it  with  that  upon  its  circumscribing  rectangular 
parallelogram. 

Let  ABEF  be  a  rectangular  parallelogram  immersed  in  a  fluid,  with 
its  plane  perpendicular  to  the  plane  of  the 
horizon,  and  its  upper   side   AB   coincident 
with  the  surface  of  the  fluid  in  which  it  is 
immersed. 

Bisect  AF  and  BE  in  the  points  D  and  c; 
join  DC,  and  upon  AF  as  a  base,  with  the 
corresponding  axis  DC,  describe  the  parabola 
ACF,  touching  AB  the  surface  of  the  fluid  in 
the  point  A  ;  then  is  AC  F  the  surface  for  which  the  pressure  is  required 
to  be  investigated. 

Join  BF,  intersecting  DC  the  axis  of  the  parabola  in  the  point  n-r 
then  is  n  the  centre  of  gravity  of  the  rectangular  parallelogram  ABEF. 


OF  THE  PARALLELOGRAM  AND  ITS  INSCRIBED  PARABOLA.  73 

Divide  the  axis  DC  into  two  parts  Dm  and  cm  such,  that  Dm  is  to 
cm,  in  the  ratio  of  2  to  3;  and  the  point  m  thus  determined,  is  the 
place  of  the  centre  of  gravity  of  the  parabola  ACF.  Through  the 
points  m  and  n,  draw  the  straight  lines  mr  and  ns  respectively  per- 
pendicular to  the  axis  DC;  then  are  mr  and  ns7  the  perpendicular 
depths  of  the  points  m  and  n  below  AB,  the  surface  of  the  fluid,  and 
which  in  the  present  case  are  equal  to  one  another. 

Put  &ZZIAB,  the  horizontal  breadth  of  the  rectangular  parallelogram 
ABEF,  or  the  axis  of  its  inscribed  parabola  ACF, 

I  —  AF,  the  length  of  the  circumscribing  rectangle,  or  the  base 
of  the  inscribed  parabola, 

d  —  rm  or  sn,  the  vertical  depths  of  the  centres  of  gravity,  below 
AB  the  surface  of  the  fluid, 

Pmthe  pressure  on  the  rectangle  ABEF, 

/>  =  that  on  the  inscribed  parabola  ACF, 

A  =  the  area  of  the  circumscribing  rectangular  parallelogram, 

a  —  the  area  of  the  parabola,  and 

s  —the  specific  gravity  of  the  fluid  in  which  they  are  immersed. 

Then,  according  to  the  principles  of  mensuration,  the  area  of  the 
rectangular  parallelogram  ABEF,  is  equal  to  the  product  of  the 
breadth  AB  drawn  into  the  depth  AF  ;  that  is, 

A  =  bl; 
and  by  the  property  of  the  parabola,  its  area  is 

a  =  \b  I. 

But  the  pressure  perpendicular  to  the  surface  of  the  rectangular 
parallelogram,  is,  as  we  have  already  frequently  stated,  expressed  by 
the  area  drawn  into  the  perpendicular  depth  of  the  centre  of  gravity  ; 
and  this  being  the  case,  whatever  may  be  the  form  of  the  surface 
pressed,  it  follows,  that  the  pressure  on  the  rectangle  ABEF,  is 

P  —  bl  X  d  X  s  —  bdls'y 
and  that  on  the  parabola  ACF  is 


Now,  it  is  manifest  from  the  nature  of  the  figure,  and  from  the 
principles  upon  which  it  is  constructed,  that  rm  and  sn  are  each  of 
them  equal  to  JAF;  that  is,  d=L\l\  therefore,  let  \l  be  substituted 
for  d  in  each  of  the  above  equations,  and  we  shall  obtain 
For  the  rectangle  ABEF, 


and  for  the  parabola  ACF,  it  is 

(37). 


74  OF  THE  PARALLELOGRAM  AND  ITS  INSCRIBED  PARABOLA. 

Consequently,  by  analogy,  the  comparative  pressures  on  the  para- 
bola and  its  circumscribing  rectangle,  are  as  follows  : 

p:  P  ::ibl*s:%bl*s; 
and  from  this,  by  casting  out  the  common  quantities  and  assimilating 

the  fractions,  we  get 

p  :  P  :  :  2  :  3. 

COROL.  Hence  it  appears,  that  when  the  axis  of  the  parabola  is 
horizontal,  and  its  base  perpendicular  to  the  horizon ;  the  pressure 
perpendicular  to  its  surface,  when  compared  with  that  on  its  circum- 
scribing parallelogram,  bears  precisely  the  same  relation,  that  its  area 
bears  to  the  area  of  the  rectangle  by  which  it  is  circumscribed. 

85.  The  practical  rule  for  determining  the  pressure  on  the  parabolic 
plane,  when  placed  in  the  position  specified  in  the  problem,  may  be 
expressed  in  words  at  length  in  the  following  manner. 

RULE.  Multiply  the  horizontal  axis,  by  the  square  of  the 
vertical  base  or  double  ordinate,  and  again  by  the  specific 
gravity  of  the  fluid ;  then,  take  one  third  of  the  product  for 
the  pressure  perpendicular  to  the  surface  of  the  parabolic 
plane. 

Or  thus,  Calculate  the  pressure  on  the  circumscribing 
rectangle,  and  take  two  thirds  of  the  result  for  the  pressure 
on  the  parabola. 

86.  EXAMPLE  16.  The  data  remaining  as  in  the  example  to  the 
foregoing  problem,  it  is  required  to  determine  the  pressure  on  the 
parabolic  plane,  supposing  its  axis  to  be  horizontal,  its  base  or  double 
ordinate  vertical,  and  the  upper  extremity  of  the  base  in  contact  with 
the  surface  of  the  fluid,  which,  according  to  the  conditions  of  the 
previous  question,  is  water,  whose  specific  gravity  is  expressed  by 
unity,  and  the  weight  of  one  cubic  foot  of  which  is  equal  to  62£  Ibs. 
avoirdupois  ? 

Referring  the  numerical  data  to  the  same  parts  of  the  figure,  as  in 

the  preceding  cases,  we  have  given  6  =  42  feet;    /iz:28  feet,  and 

s  zz:  62  j  Ibs. ;  therefore,  by  proceeding  according  to  the  rule,  we  have 

42  X  28  X  28  X  62  J  =  2058000, 

which  being  divided  by  3,  gives 

p  =  2058000  -r  3  =  686000  Ibs. 

Hence  it  appears,  that  the  total  pressures  perpendicular  to  the 
parabolic  surface,  according  to  the  several  positions  in  which  we  have 
placed  it,  are  to  one  another  respectively  as  the  numbers 
3087,  2058  and  1715; 


OF  THE  PARALLELOGRAM  AND   ITS  INSCRIBED   PARABOLA.  75 

the  first  two  of  which,  by  reason  of  the  parts  of  the  figure  being  the 
same  in  each,  are  obviously  dependent  upon  one  another;  but  the 
third,  in  which  the  parts  of  the  figure  are  reversed,  is  wholly  inde- 
pendent and  distinct  from  the  other  two. 

87.  COROL.  1.    Admitting  therefore,  that  the  pressure   upon   the 
parabolic  surface,  under  the  three  circumstances  of  position  in  which 
we  have  considered  it,  is  represented  by  the  equations  (35),  36)  and 
(37) ;  it  follows,  that  the  situation  of  the  centre  of  gravity  can  easily 
be  ascertained ;  for  the  pressure  in  each  case,  as  we  have  elsewhere 
shown,  is  represented  by  the  area  of  the  figure,  drawn  into  the  per- 
pendicular depth  of  its  centre  of  gravity ;  consequently  by  reversing 
the  process,  the  depth  of  the  centre  of  gravity  will  become  known,  if 
the  pressure  be  divided  by  the  area  of  the  surface  on  which  the  fluid 
presses. 

COROL.  2.  Since  the  parabola  is  a  figure  symmetrically  situated  with 
respect  to  its  axis,  it  is  obvious,  that  the  centre  of  gravity  of  its  surface 
must  occur  at  the  same  point,  in  whatsoever  position  it  may  be  placed; 
but  when  the  place  of  its  centre  is  referred  to  the  surface  of  the  fluid 
in  which  it  is  immersed,  the  distance  varies  for  each  particular  case: 
thus, 

In  the  first  instance,  the  perpendicular  distance,  is  in  f  ths  of  the  axis, 

second, izrfths  

third,    m  ^  the  base. 

But  as  we  have  just  stated,  the  centre  of  gravity  of  the  parabolic  sur- 
face as  referred  to  its  vertex,  or  any  other  fixed  point,  in  all  these  cases, 
remains  unaltered,  in  whatever  position  the  figure  itself  may  be  placed. 

5.  THE  METHOD  OF  DETERMINING  THE  PRESSURE  OF  THE  FLUID  UPON 
A  SEMI-PARABOLIC  PLANE  AS  COMPARED  WITH  THAT  ON  THE 
CIRCUMSCRIBING  RECTANGULAR  PARALLELOGRAM. 

88.  When  the  semi-parabola   only  is  considered,  the  determina- 
tion of  its  centre  of  gravity,  and  consequently,  of  the  pressure  on 
its  surface  becomes  more  difficult ;  for,  since  the  figure  is  not  sym- 
metrical with  respect  to  its  axis,  we  are  under  the  necessity  of  com- 
puting the  two  rectangular  co-ordinates,  whose  in- 
tersection determines  the  place  of  the  required  centre. 

Let  CBD  be  a  semi-parabola,  perpendicularly 
immersed  in  a  fluid,  so  that  its  axis  CD  is  vertical, 
and  the  vertex  in  contact  with  CF  the  surface  of  the 
fluid,  and  let  CFBD  be  the  circumscribing  rectan- 
gular parallelogram. 


76  OF  THE  PARALLELOGRAM  AND  ITS  INSCRIBED  PARABOLA. 

Now,  since  by  Problem  XI,  it  has  been  proved  that  the  pressure  on 
the  entire  parabola  with  its  axis  vertical,  is  equal  to  four  fifths  of  that 
upon  its  circumscribing  parallelogram  ;  it  follows,  that  the  pressure 
on  the  semi-parabola  in  the  same  position,  is  also  equal  to  four  fifths 
of  that  upon  its  circumscribing  parallelogram,  it  being  manifestly 
equal  to  half  the  pressure  on  the  whole  parabola. 

Divide  the  axis  CD  into  two  parts,  such,  that  Dm  and  cm  shall  be 
to  one  another  in  the  ratio  of  2  to  3  ;  and  in  like  manner,  let  the 
ordinate  or  base  BD  be  divided  into  two  parts,  such,  that  DW  and  vn 
shall  be  to  one  another  in  the  ratio  of  3  to  5*  ;  then,  through  the 
points  m  and  n,  and  respectively  parallel  to  DB  and  DC,  draw  the 
straight  lines  WIG  and  no,  meeting  each  other  in  G,  the  centre  of 
gravity  of  the  semi-parabola  DC  B. 

Put  &IZICF  or  DB,  the  horizontal  breadth  of  the  rectangle  CFED,  or 

the  base  of  the  semi-parabola  c  B  D, 
I  =:  CD  or  FB,  the  vertical  depth  of  the  rectangle,  or  axis  of  its 

inscribed  semi-parabola, 

cZzz  cm  or  EG,  the  perpendicular  depth  of  the  centre  of  gravity, 
Pzzthe  pressure  on  the  rectangular  parallelogram  CFBD, 
p  zz  the  pressure  on  its  inscribed  semi-parabola, 
A  zz  the  area  of  the  parallelogram, 
a  zz  the  area  of  the  semi-parabola,  and 

s  zz  the  specific  gravity  of  the  fluid  in  which  they  are  immersed. 
Then,  according  to  the  principles  of  mensuration,  the  area  of  a 
rectangle  is  expressed  by  the  product  of  its  two  dimensions  ;  that  is, 
by  its  length  drawn  into  its  breadth  ;  therefore,  we  have 

A  =  bl, 

and  by  Proposition  (A),  the  pressure  exerted  by  a  fluid,  perpendicu- 
larly to  any  surface  immersed  in  it,  or  otherwise  exposed  to  its 
influence, 

Is  equal  to  the  area  of  the  surface  pressed,  drawn  into  the 

perpendicular  depth  of  its  centre  of  gravity,  and  again  into 

the  specific  gravity  of  the  fluid. 

Consequently,  the  pressure  on  the  circumscribing  rectangular  paral- 
lelogram CFBD,  becomes 


*  It  is  demonstrated  by  the  writers  on  mechanics,  that  the  centre  of  gravity  of  a 
semi-parabola  is  situated  in  its  plane,  at  the  distance  of  three  eighths  of  the  ordinate 
from  the  axis,  and  two  fifths  of  the  axis  from  the  ordinate,  or  three  fifths  of  the  axis 
from  its  vertex. 


OF  THE  PARALLELOGRAM  AND  ITS  INSCRIBED  PARABOLA.  77 

Now,  since  the  pressure  on  the  semi-parabola  is  equal  to  four  fifths 
of  the  pressure  on  its  circumscribing  parallelogram,  we  shall  obtain 

f  P  =  $  X  Ibl^s  —  ^b^s—p.  (38). 

The  expression  which  we  have  here  determined  for  the  pressure  on 
the  surface  of  the  semi-parabola  D  c  B,  is  precisely  the  same  as  that  which 
we  have  given  in  equation  (35)  for  the  entire  figure ;  only  in  the  pre* 
sent  instance, the  value  of  6,  the  horizontal  breadth  of  the  parallelogram, 
is  but  one  half  the  value  as  applied  to  the  parabola,  when  placed 
under  the  conditions  specified  in  the  eleventh  problem  foregoing. 

89.  If  the  symbol  b  retain  its  former  value,  that  is,  if  it  be  referred 
to  the  base  of  the  entire  parabola,  or  to  the  breadth  of  the  parallelo- 
gram circumscribing  the  entire  parabola,  then,  the  pressure  on  the 
semi-parabola,  becomes  ' 

p  —  \bl^s.  (39). 

Consequently,  the  practical  rule  for  determining  the  pressure  on  the 
semi-parabola  as  deduced  from  this  equation,  may  be  expressed  as 
follows. 

RULE.  Multiply  one  fifth  of  the  base,  or  double  ordinate  of 
the  whole  parabola,  by  the  square  of  the  length  of  its  axis, 
and  again  by  the  specific  gravity  of  the  fluid,  and  the  product 
will  express  the  pressure  on  the  semi-parabola  in  a  direction 
perpendicular  to  its  surface. 

But  if  the  symbol  b  refer  to  the  ordinate,  or  base  of  the  semi- 
parabola,  then,  the  rule  as  deduced  from  the  equation  (38),  will  be 
precisely  the  same  as  that  which  we  have  given  under  the  equation 
numbered  (35)  in  Problem  XI,  to  which  place  the  reader  is  referred 
for  the  purpose  of  avoiding  a  direct  repetition. 

90.  EXAMPLE  17.  A  plane  in  the  form  of  a  semi-parabola  whose 
base  or  ordinate  is  16  feet,  and  its  axis  40  feet,  is  perpendicularly 
immersed  in  a  cistern  of  water,  in  such  a  manner,  that  its  axis  is 
vertical,  and  its  vertex  in  contact  with  the  surface  of  the  fluid ;  what 
pressure  does  it  sustain,  the  weight  of  a  cubic  foot  of  water  being 
equal  to  62 Jibs.? 

The  equation  in  its  present  state,  supposes  the  ordinate,  or  base  of 
the  semi-parabola  to  be  given,  and  therefore,  the  pressure  is  deter- 
mined by  the  rule  to  the  equation  (35)  or  (38),  in  the  following 
manner : 

p—l  x  16  X402  X  621=  640000 Ibs. 

But  in  order  to  determine  the  pressure  by  the  rule  immediately 
preceding,  we  must  suppose  the  breadth  or  base  of  the  figure  to  be 


78  OF  THE  PARALLELOGRAM  AND  ITS  INSCRIBED  PARABOLA. 

doubled,  in  which  case  it  will  have  reference  to  the  whole  parabola, 
and  the  pressure  will  be  reduced  to  that  upon  its  half,  by  employing 
the  constant  \  instead  of  f-  according  to  the  rule,  thus, 
p  =  i  x  32  x  402  X  62J  =  640000  Ibs. 

91.  If  the  axis  of  the  semi-parabola  were  horizontal  and  its  ordinate 
vertical,  as  in  the  annexed  diagram  ;  then, 
the  area  of  the  semi-parabolic  figure,  as  well 
as  that  of  its  circumscribing  parallelogram, 
will  remain  the  same,  but  the  pressures 
perpendicular  to  their  respective  surfaces 
will  be  very  different. 

Divide  BD  in  n,  in  such  a  manner,  that  En  and  DW  may  be  to  one 
another  in  the  ratio  of  5  to  3 ;  and  in  like  manner,  divide  CD  in  m, 
so  that  cm  and  Dm  shall  be  to  each  other  in  the  ratio  of  3  to  2. 

Through  the  points  n  and  m,  and  parallel  respectively  to  B  F  and 
BD,  draw  the  straight  lines  no,  and  ma,  intersecting  one  another  in 
the  point  G,  arid  produce  m  G  to  E  ;  then ,  by  the  note  to  the  pre- 
ceding case,  the  point  G  is  the  centre  of  gravity  of  the  semi-parabola 
DCB,  and  EG  is  its  perpendicular  depth  below  BF,  the  horizontal 
surface  of  the  fluid. 

Therefore,  let  the  preceding  notation  remain,  and  let  the  several 
symbols  refer  to  the  same  parts  of  the  figure  as  in  the  preceding  case, 
disregarding  the  change  of  position  which  has  taken  place ;  then,  as 
before,  the  area  of  the  parallelogram  BFCD,  is 

A  =  6/, 

and  the  pressure  perpendicular  to  its  surface,  is 


For  draw  the  diagonals  B  c  and  F  D  intersecting  each  other  in  the 
point  r,  and  through  r  draw  rs  parallel  to  BD  or  Em;  then,  r  is  the 
centre  of  gravity  of  the  rectangle  BFCD,  and  sr  its  perpendicular 
depth  below  BF  the  surface  of  the  fluid;  but  according  to  our  notation 
srzr  \b,  and  we  have  seen  above,  that  A  zz:  bl;  now,  the  pressure  on 
any  surface,  whatever  may  be  its  form, 

Is  equal  to  the  product  that  arises,  when  the  area  of  the 
surface  pressed,  is  drawn  into  the  perpendicular  depth  of  its 
centre  of  gravity,  and  again,  into  the  specific  gravity  of  the 
fluid. 

Consequently,  the  pressure  perpendicular  to  the  surface  of  the 
rectangular  parallelogram  BFCD,  is 


OF  THE  PARALLELOGRAM  AND  ITS  INSCRIBED  PARABOLA.        79 

Again,  the  area  of  the  semi-parabola  BCD,  is  equal  to  two  thirds  of 
its  circumscribing  rectangular  parallelogram  BFCD  ;  therefore  we  have 

a  =  l  X  bl=$bl, 
and  the  pressure  perpendicular  to  its  surface,  is 

p  =  &Vl8. 

This  is  manifest,  for  according  to  the  construction  and  the  nature 
of  the  figure  of  the  parabola,  BW  or  EG  is  equal  to  five  eighths  of  BD  ; 
therefore,  we  have 

p  —  ^blXibX  s  =  -frb*ls',  (40). 

consequently,  by  analogy,  we  obtain 

p  :  P  :  :  &&ls  :  ±b*ls. 

Therefore,  by  suppressing  the  common  factors,  and  rendering  the 
fractions  T\  and  |  similar,  we  shall  get 

p:P::5:  6; 

hence  it  appears,  that  when  the  ordinate  of  the  semi-parabola  is 
vertical,  and  its  upper  extremity  in  contact  with  the  surface  of  the 
fluid  :— 

The  pressure  upon  the  semi-parabola,  is  to  that  upon  its 
circumscribing  rectangular  parallelogram,  as  5  is  to  6,  or  as 
1  is  to  1.2. 

92.  Consequently,  the  practical  rule  for  determining  the  pressure 
in  the  present  instance,  as  deduced  from  the  equation  marked  (40), 
or  from  the  above  analogy,  may  be  expressed  as  fdllows. 

RULE.  Multiply  the  square  of  the  given  ordinate  by  the 
axis  of  the  semi-parabola,  and  again  by  the  specific  gravity 
of  the  fluid ;  then,  Jive  twelfths  of  the  result  will  give  the 
pressure  sought.  Or  thus, 

Find  the  pressure  on  the  circumscribing  parallelogram,  and 
take  five  sixths  of  the  pressure  thus  found,  for  the  pressure  on 
the  semi-parabola. 

93.  EXAMPLE  18.    Let  the  numerical  values  of  the  axis  and  ordi- 
nate remain  as  in  the  preceding  example  ;  what  will  be  the  pressure 
on  the  surface  of  the  semi-parabola,  supposing  the  axis  to  be  hori- 
zontal, the  ordinate  vertical,  and  its  remote  extremity  in  contact  with 
the  surface  of  the  fluid  ? 

If  the  operation  be  performed  according  to  the  rule  deduced  from 
equation  (40),  we  shall  obtain 

p=  162  X  40  X  62£  X  TV  =  2666661-  Ibs. 


80  OF  THE  PARALLELOGRAM  AND  ITS  INSCRIBED  PARABOLA. 

but  if  the  operation  be  performed  according  to  the  rule  derived  from 
the  analogy  of  comparison,  we  shall  have 

p  =  f P  ;  that  is,  p  —  \  x  16*  X  40  X  62 1  X  £  =  266666f  Ibs. 


6.  THE  METHOD  OF  DETERMINING  THE  POSITION  OF  THE  CENTRE 
OF  GRAVITY  OF  THE  SPACE  COMPREHENDED  BETWEEN  THE 
PARABOLIC  CURVE  AND  ITS  CIRCUMSCRIBING  PARALLELOGRAM. 

94.  By  means  of  the  pressure  on  the  semi-parabola,  as  we  have 
investigated  it  in  the  two  foregoing  cases,  we  are  enabled  to  determine 
the  pressure  on  the  space  CFB,  and  from  thence,  the  position  of  its 
centre  of  gravity. 

This  is  an  important  inquiry  in  the  practice  of  bridge  building,  for, 
in  determining  the  thickness  of  piers  necessary  to  resist  the  drift  or 
shoot  of  a  given  arch,  independently  of  the  aid  afforded  by  the  other 
arches,  it  becomes  requisite  to  find  the  centre  of  gravity  of  the  span- 
drel or  space  BFC,  which  is  used  for  the  purpose  of  balancing  the  arch 
and  filling  up  the  haunches  or  flanks. 

Now,  the  method  which  has  generally  been  employed  for  the  deter- 
mination of  this  centre  is  extremely  operose,  and  in  many  cases  it 
involves  considerable  difficulty,  requiring  the  calculations  of  solids 
and  planes,  which  are  by  no  means  easy ;  but  the  method  which  we 
are  about  to  employ,  requires  no  such  tedious  and  prolix  operations, 
as  will  become  manifest  from  the  following  investigation,  which  refers 
to  the  space  comprehended  between  a  semi-parabola  and  its  circum- 
scribing rectangle. 

Let  BCD  be  a  semi-parabola,  having  the  axis  DC  vertical  while  the 
corresponding  ordinate  DB  is  horizontal,  and 
let  B DC F  be  the  circumscribing  rectangular 
parallelogram. 

Suppose  the  point  D  to  remain  fixed,  and 
conceive  the  semi-parabola  BCD  to  revolve 
about  the  point  D  until  it  comes  into  the 
position  A  ED,  where  the  axis  DE  is  horizon- 
tal, and  the  corresponding  ordinate  DA  vertical;  then  it  is  manifest, 
that  the  circumscribing  rectangular  parallelogram  ADEH  in  this 
latter  position,  is  equal  to  BDCF  in  the  former,  and  consequently, 
the  space  AHE  comprehended  between  the  sides  of  the  rectangle 
AH,  HE  and  the  curve  AE,  is  equal  to  the  space  BFC  similarly  con- 
stituted. 


OF  THE  PARALLELOGRAM   AND  ITS  INSCRIBED  PARABOLA.  81 

It  is  further  manifest,  that  while  the  semi-parabola  revolves  about 
the  point  D,  from  the  position  BCD  to  that  of  A  ED,  the  points  B  and  c, 
the  extremities  of  the  ordinate  and  axis,  describe  respectively,  the 
circular  quadrants  BA  and  CE,  while  the  point  F  describes  another 
quadrant,  whose  containing  radii  are  the  diagonals  DF  and  DH. 

Put  6:zr  BD  or  AD,  the  ordinate  of  the  semi-parabola  in  either  posi- 

tion, 

I  n=  CD  or  ED,  the  corresponding  axis, 
d  —  nG,  the  depth  of  the  centre  of  gravity  of  the  space  BFC, 

when  the  axis  is  vertical, 
3  rz  m  G,  or  A  ft,  the  depth  of  the  centre  of  gravity  of  the  space 

AHE  or  BFC,  when  the  axis  is  horizontal, 
Pzz:  the  pressure  on  the  circumscribing  rectangular  parallelogram 

BDCF,  Or  AH  ED, 

p  —  the  pressure  on  the  inscribed  semi-parabola,  and 

p  —  the  pressure  on  the  space  comprehended  between  the  semi- 

parabola  and  its  circumscribing  rectangular  parallelo- 

gram. 

Then,  according  to  equation  (8)  under  the  third  problem,  the 
pressure  on  the  circumscribing  rectangular  parallelogram  when  the 
length  is  vertical,  is 


and    agreeably  to  equation  (38)    under  the  eleventh  problem,  the 
pressure  on  the  inscribed  semi-parabola  with  the  axis  vertical,  is 


consequently,  by  subtraction,  the  pressure  upon  the  space  BFC,  com- 
prehended between  the  sides  of  the  parallelogram  BF,  FC  and  the 
curve  of  the  parabola  BC,  is 


hence,  by  suppressing  the  symbol  for  the  specific  gravity,  we  get 
p'  —  bl^(l—  f)rrTV^2.  (41). 

Now,  according  to  the  writers  on  mensuration,  the  area  of  the 
semi-parabola  is  equal  to  two  thirds  of  the  area  of  the  circumscribing 
parallelogram  ;  it  therefore  follows,  that  the  area  of  the  space  BFC,  is 
equal  to  one  third  of  the  rectangle  BDCF;  that  is, 
bi  —  $bl=;$bl. 

But  it  has  been  demonstrated,  that  the  pressure  upon  any  surface, 
is  equal  to  the  area  of  that  surface,  drawn  into  the  perpendicular 
depth  of  the  centre  of  gravity  ;  consequently,  we  have 

VOL.  i.  G 


82  OF  THE  PARALLELOGRAM  AND  ITS  INSCRIBED  PARABOLA. 


and  we  have  shown  above  in  equation  (41),  that  when  the  axis  of  the 
semi-parabola  is  vertical,  the  pressure  on  the  space  BFC  is 


consequently,  by  comparison,  we  obtain 
#*!=&*+ 

and  finally,  dividing  by  ^bl,  we  shall  have 


Again,  when  the  axis  of  the  semi-parabola  is  horizontal,  as  indicated 
by  A  ED,  the  pressure  on  the  circumscribing  rectangle,  according  to 
equation  (10)  under  the  third  problem,  is 


and  the  pressure  upon  the  inscribed  parabola,  according  to  equation 
(40)  under  the  eleventh  problem,  is 


therefore,  by  subtraction,  the  pressure  upon  the  space  comprehended 
between  the  rectangular  parallelogram  and  its  inscribed  semi-parabola, 

p'  —  P  —  p  =  J6*  /*  —  TM>2  '  «, 
and  by  suppressing  the  symbol  for  the  specific  gravity,  we  have 


Now,  the  area  of  the  inscribed  semi-parabola  is,  as  we  have  seen 
above,  equal  to  two  thirds  of  its  bounding  rectangle,  and  the  area  of 
the  space  comprehended  between  the  rectangle  and  the  semi-parabola, 
is  therefore,  equal  to  one  third  of  the  same  quantity  ;  that  is, 

bl—lbl  =  *bl; 
consequently,  the  pressure  on  the  irregular  space  A  HE,  is 

p'=ibtl; 
hence,  by  comparison,  we  shall  have 

^=TV^; 

therefore,  by  division,  we  obtain 


Having  thus  determined  the  values  of  the  rectangular  co-ordinates, 
as  represented  by  the  equations  (42)  and  (43),  the  position  of  the 
centre  of  gravity  can  easily  be  found  ;  for,  from  the  point  F,  set  off 
rm  equal  to  three  tenths  of  the  axis  CD,  and  jfn  equal  to  one  fourth 
of  the  ordinate  BD,  or  its  equal  FC  ;  then,  through  the  points  m  and  n, 
and  parallel  respectively  to  the  ordinate  BD  and  axis  CD,  draw  the 


OF  THE  PARALLELOGRAM  AND   ITS  INSCRIBED   PARABOLA.  813 

straight  lines  ma  and  HG,  intersecting  each  other  in  the  point  G  ;  and 
the  point  G  thus  determined,  is  the  position  of  the  centre  of  gravity 
of  the  space  comprehended  between  the  semi-parabola  and  its  circum- 
scribing rectangular  parallelogram. 

This  method  of  determining  the  position  of  the  centre  of  gravity  of 
the  space  comprehended  between  the  curve  and  its  circumscribing 
parallelogram,  will  be  illustrated  and  applied  in  all  its  generality, 
when  we  come  to  treat  on  the  subject  of  Hydraulic  Architecture,  to 
which  it  more  properly  belongs ;  and  for  this  reason,  we  shall  take 
no  further  notice  of  it  in  this  place,  but  proceed  with  our  inquiry 
respecting  pressure,  which  is  more  immediately  the  object  of  our 
research. 

7.  METHOD  OF  DIVIDING  A  PARABOLIC  PLANE  PARALLEL  TO  ITS  BASE, 
SO  THAT  THE  FLUID  PRESSURES  ON  EACH  PART  MAY  BE  EQUAL 
TO  ONE  ANOTHER. 

PROBLEM  XIII. 

95.  If  a  parabolic  plane  be  immersed  perpendicularly  in  an 
incompressible  fluid,  in  such  a  manner,  that  its  vertex  is  j  ust  in 
contact  with  the  surface  : — 

It  is  required  to  determine  at  what  distance  from  the 
vertex,  a  straight  line  must  be  drawn  parallel  to  the  base,  so 
that  the  figure  may  be  divided  into  two  parts,  on  which  the 
pressures  are  equal  to  one  another. 

Let  ACB  be  the  given  parabola,  immersed  in  the  fluid  after  the 
manner  specified  in  the  problem,  and  let  aAEb  be  the  circumscribing 
rectangular  parallelogram. 

Take  cm  for  the  distance  from  the  vertex 
through  which  the  line  of  division  passes,  and 
draw  EF  parallel  to  the  base  AB;  then  are 
the  spaces  ABFE  and  ECF,  the  parts  into 
which  the  parabola  is  divided,  and  on  which, 
by  the  conditions  of  the  problem,  the  pres- 
sures are  equal. 

Through  the  points  E  and  F,  the  extremities  of  the  line  of  division, 
draw  EC  and  FG?  respectively  parallel  to  CD  the  axis  of  the  figure; 
then  is  CEFC?,  the  rectangular  parallelogram  circumscribing  the  para- 
bola ECF. 

G2 


84  OF  THE  PARALLELOGRAM  AND  ITS  INSCRIBED  PARABOLA. 

Put  26  =  AB  or  ab,  the  base  of  the  parabola  ACB,  or  the  horizontal 

breadth  of  the  circumscribing  parallelogram  OAB&, 
I    zz  CD  or  a  A,  the  vertical  axis  of  the  parabola  ACB,  or  the 

depth  of  the  rectangle  by  which  it  is  encompassed, 
2y  zz  EF  or  cd,  the  base  of  the  parabola  ECF,  or  the  breadth  of 

the  rectangle  c  E  F  d, 
x    zz  cm  or  CE,  the  axis  of  the  parabola  ECF,  or  the  depth  of  its 

circumscribing  rectangle  CEFC?, 
P  zzthe  pressure  on  the  rectangular  parallelogram  CEAB&,  cir- 

cumscribing the  parabola  ACB, 
p   zz  the  pressure  on  the  inscribed  parabola, 
P'  zz  the  pressure  on  the  rectangular  parallelogram  CEF^,  cir- 

cumscribing the  parabola  ECF, 
p'  zz  the  pressure  on  the  inscribed  parabola,  and 
*    zz  the  specific  gravity  of  the  fluid. 

Then  since  ABzz26  and  EFZz2y,  it  follows,  that  AD  —  6  and 
EWI  zz  y  ;  therefore,  by  the  property  of  the  parabola,  we  have 


and  consequently,  by  equating  the  products  of  the  extreme  and  mean 
terms,  we  shall  obtain 

y=p*, 

and  by  division,  it  is 

b*x 

y=T> 
and  this,  by  extracting  the  square  root,  becomes 


and  finally,  multiplying  by  2,  we  obtain 
EFZZ  ^    —  ^ 


Now,  the  pressure  perpendicular  to  the  surface  of  the  rectangular 
parallelogram  aAB#,  according  to  equation  (8)  under  the  third 
problem,  is 


but  we  have  seen  elsewhere,  that  the  pressure  on  the  surface  of  a 
parabola,  is  equal  to  four  fifths  of  that  upon  its  circumscribing  paral- 
lelogram ;  consequently,  the  pressure  on  the  parabola  ACB,  is 

(44). 


OF  THE  PARALLELOGRAM  AND  ITS  INSCRIBED  PARABOLA.  85 

Again,  the  pressure  perpendicular  to  the  surface  of  the  rectangular 
parallelogram  CEF<?,  is 


and  the  pressure  upon  the  inscribed  parabola  ECF,  is  four  fifths  of 
the  pressure  on  the  circumscribing  rectangle ;  that  is, 


I  '  (45). 

but  according  to  the  conditions  of  the  problem, 
p'  •=.  \p ;  hence  we  have 


and  by  suppressing  the  common  factors,  we  get 


from  which,  by  squaring  both  sides,  we  obtain 

o:5-|*5; 
consequently  by  extracting  the  fifth  root  of  both  sides,  we  get 

x  =  l#1i 
but  according  to  the  arithmetic  of  surd  quantities 

^7=1^87 

therefore,  by  substitution,  we  shall  have 


now,  the  sursolid,  or  fifth  root  of  8,  is  1.51571  ;  hence  we  get 

a:  =  .75785*.  (46). 

96.  The  practical   rule  supplied  by  this   equation   is  extremely 
simple;    it  may  be  expressed  in  words  at  length  in  the  following 
manner. 

RULE.  Multiply  the  axis  of  the  given  parabola  by  the 
constant  number  .75785,  and  the  product  will  give  the  distance 
from  the  vertex  through  which  the  line  of  division  passes. 

97.  EXAMPLE  19.  The  axis  of  a  parabola  is  29  feet,  and  its  plane 
is  perpendicularly  immersed  in  a  cistern  of  water,  in  such  a  manner, 
that  its  vertex  is  just  in  contact  with  the  surface  ;  through  what  point 


86  OF  THE  PARALLELOGRAM  AND  ITS  INSCRIBED  PARABOLA. 

in  the  axis  must  a  line  be  drawn  parallel  to  the  base,  so  that  the  pres- 
sures on  the  two  parts  into  which  the  parabola  is  divided,  may  be 
equal  to  one  another  ? 

By  operating  according  to  the  preceding  rule  derived  from  equation 
(46),  we  shall  have  for  the  distance  from  the  vertex 

a,—  .75785  X  29  =  21.97765  feet. 

In  the  case  which  we  have  investigated  above,  the  parabola  is 
divided  into  two  parts  on  which  the  pressures  are  equal  ;  but  in  order 
to  render  the  solution  general,  we  must  so  arrange  it,  that  the  parts 
of  division  may  bear  any  ratio  to  one  another,  as  denoted  by  the 
symbols  m  and  n  ;  that  is, 

P  '•  P  —  P'  '•  '  m  :  n. 

Now,  we  have  seen  in  equation  (44),  that  the  pressure  on  the  entire 
parabola  ACB,  is 


and    according    to    equation    (45),    the    pressure    on    the   parabola 
ECF,  is 


but  the  pressure  upon  the  space  AEFB,  is  manifestly  equal  to  the 
difference  of  these  ;  that  is, 


consequently,  we  obtain 


:  :  m  :  n; 


and  from  this,  by  equating  the  products  of  the  extreme  and  mean 
terms,  we  shall  obtain 


which,  by  transposing  and  collecting  the  terms,  becomes 
(m  -f  w)  x*^/  -^-  =  ml\ 

from  which,  by  division,  we  shall  get 
/~~x  _      ml* 

XV  T—^r~n; 


OF  THE  PARALLELOGRAM  AND  ITS  INSCRIBED  PARABOLA.  87 

therefore,  by  involving  or  squaring  both  sides  of  this  equation,  we 
shall  obtain 


and  by  extracting  the  sursolid  root,  we  get 


=  '1/0 


»)*'  (47). 

If  m  and  n  be  equal  to  one  another  as  in  the  preceding  case,  then 
it  is  obvious  that  the  equation  becomes 


and  if  any  other  numerical  ratio  be  proposed,  such  as  4  to  5;  then 
we  shall  have 


Thus  for  example;  let  the  axis  of  the  parabola  remain  as  in  the 
foregoing  question,  and  let  it  be  required  to  find  a  point,  through 
which  a  line  must  be  drawn  parallel  to  the  base,  so  that  the  pressure 
on  the  part  above  the  dividing  line,  may  be  to  that  below  it  in  the 
ratio  of  4  to  5  ? 

Here  we  have 


=  20.967  feet. 

Hence  it  appears,  that  if  a  point  be  taken  in  the  axis  of  the  given 
parabola  at  the  distance  of  20.967  feet  from  the  vertex,  and  if  through 
that  point,  a  line  be  drawn  parallel  to  the  base,  the  parabola  will  be 
divided  into  two  parts,  on  which  the  pressures  are  to  one  another 
as  4  to  5. 


CHAPTER  IV. 

OF    THE     PRESSURE     OF    INCOMPRESSIBLE    FLUIDS    ON    CIRCULAR 

PLANES  AND    ON    SPHERES    IMMERSED    IN    THOSE  FLUIDS, THE 

EXTREMITY  OF  THE  DIAMETER  OF  THE  FIGURE  BEING  IN  EACH 
CASE  IN  EXACT  CONTACT  WITH  THE  SURFACE  OF  THE  FLUID. 


PROBLEM  XIV. 

98.  Suppose  a  circular  plane  to  be  immersed  perpendicularly 
in  an  incompressible  fluid,  in  such  a  manner,  that  the  extremity 
of  the  diameter  is  just  in  contact  with  the  surface : — 

It  is  required  to  draw  from  the  lowest  point  of  the  circular 
plane,  that  chord  on  which  the  pressure  shall  be  a  maximum. 

Let  ABC  be  the  circular  plane  immersed  in  the  fluid  according  to 
the  conditions  of  the  problem ;  draw  the  ver- 
tical diameter  BC  touching  the  surface  of  the 
fluid  in  the  point  B,  and  let  CA  be  the  chord 
required. 

Bisect  the  chord  CA  in  the  point  m,  and 
through  the  point  m  thus  determined,  draw 
mn  parallel  to  BC  the  vertical  diameter,  meeting  the  surface  of  the 
fluid  in  n  j  then  is  n  m  the  perpendicular  depth  of  the  centre  of  gravity 
of  the  chord  AC,  below  the  surface  of  the  fluid  in  which  it  is  immersed  ; 
draw  also  AE  and  mD  respectively  perpendicular  to  the  diameter  BC. 

Now,  we  have  already  demonstrated  in  the  first  Problem,  that  the 
pressure  upon  a  physical  line,  is  equal  to  the  product  of  its  length  by 
the  perpendicular  depth  of  its  centre  of  gravity,  and  again  by  the 
specific  gravity  of  the  fluid  ;  consequently,  we  have 

p  =  AC  x  nm  X  s. 
Put  d~  BC,  the  diameter  of  the  immersed  circular  plane, 

I  •=.  mn,  the  perpendicular  depth  of  the  centre  of  gravity  of  the 
chord  AC, 


Of  FLUID  PRESSURE  ON  CIRCULAR  PLANES  AND  ON  SPHERES.      89 

/  =  AC,  the  length  of  the  chord  on  which  the  pressure  is  a 

maximum, 

p  in  the  pressure  on  the  chord  AC, 
x  =.  c  D,  the  perpendicular  height  of  the  centre  of  gravity  of  the 

chord  A  c  above  the  lower  extremity  of  the  diameter, 
and  s  ~  the  specific  gravity  of  the  fluid. 

Then  we  have  o  =  c?  —  x;  CE=z:2a:,  and  by  the  property  of  the 
circle,  the  length  of  the  chord  becomes 


consequently,  the  pressure  upon  it  is 


but  this,  according  to  the  conditions  of  the  problem,  is  to  be  a  maxi- 
mum ;  therefore,  by  putting  the  fluxion  of  the  expression  equal  to 
nothing,  we  obtain 

MX  (d*  —  4dx  +  3z2)  =  0  ; 

therefore,  by  omitting  the  common  factors  2dx  and  transposing,  we 
shall  have 


and  this  quadratic  equation  being  reduced,  we  get 

x  —  ^d.  (48). 

COROL.  1.  Consequently,  to  determine  the  chord  by  construction, 
make  BE  equal  to  one  third  of  the  vertical  diameter  BC,  and  through 
the  point  E,  draw  the  straight  line  EA  at  right  angles  to  BC,  and 
meeting  the  circumference  in  the  point  A  ;  then  from  c,  the  lower 
extremity  of  the  diameter,  inflect  the  straight  line  CA,  and  the  thing 
is  done.  Or  thus  : 

2.  Find  0  an  angle  such,  that  tan.  0  —  J^/2"—.  707  11,  which 
happens  when  0zn  35°  15'  51"  ;  therefore,  at  c  the  lower  extremity 
of  the  diameter,  make  the  angle  BCA  equal  to  35°  15'  51",  and  the 
straight  line  CA  will  be  the  chord  required. 

It  has  been  shown  above,  that  according  to  the  property  of  the 
circle,  the  length  of  the  chord  is  £==  ^2dx;  if,  therefore,  the  value 
of  x  as  determined  in  equation  (48),  be  substituted  instead  of  it,  in 
the  foregoing  value  of  /,  we  shall  have 

l=ldi/6.  (49). 

99.  This  is  the  proper  form  of  the  expression  when  adapted  for 
numerical  operation,  and  the  practical  rule  which  it  supplies,  may  be 
expressed  as  follows. 


90       OF  FLUID  PRESSURE  ON  CIRCULAR  PLANES  AND  ON  SPHERES. 

RULE.  Multiply  one  third  of  the  diameter  of  the  given 
plane  by  the  square  root  of  6,  and  the  product  will  be  the 
length  of  the  chord  required.  Or  thus  : 

Multiply  the  given  diameter  by  the  constant  number 
.81647,  and  the  product  will  be  the  length  of  the  chord 
required. 

100.  EXAMPLE  20.  A  circular  plane  whose  diameter  is  equal  to 
36  feet,  is  perpendicularly  immersed  in  a  fluid,  so  that  the  upper 
extremity  of  the  vertical  diameter  is  in  contact  with  the  horizontal 
surface ;  what  is  the  length  of  a  chord,  which  being  inflected  from 
the  lower  extremity  of  the  diameter,  sustains  a  greater  pressure  than 
any  other  chord  which  can  be  drawn  from  the  same  point  ? 

Here  by  the  rule,  we  have 
f  =  i  X  36  x  VQ  =  29.3929  feet, 
and  by  the  second  part  of  the  rule,  we  have 
1=36  X  .81647 —29.3929  feet. 


PROBLEM  XV. 

101.  If  two  spheres  or  globes  of  different  diameters,  be 
immersed  in  a  fluid,  in  such  a  manner,  that  the  uppermost  point 
on  their  surface  is  just  in  contact  with  the  horizontal  surface 
of  the  fluid  :— 

It  is  required  to  determine  the  pressure  on  each  of  the 
spheres,  and  to  compare  the  pressures  with  one  another. 

Let  ABD  and  abd  be  the  two  spheres,  whose  diameters  AB  and  ab, 
have  their  upper  extremities  A  and  a  in 
contact  with  EF,  the  horizontal  surface  of 
the  fluid. 

Bisect  the  diameters  AB  and  a b,  respec- 
tively in  the  points  c  and  c  ;  then  are  the 
points  c  and  c,  the  centres  of  magnitude  of 

the  respective  spheres  ;  but  in  a  sphere,  the  centre  of  magnitude  and 
the  centre  of  gravity  occur  in  the  same  point ;  therefore,  c  and  c  are 
the  centres  of  gravity  of  the  spheres  ABD  and  abd,  and  AC  and 
ac  are  the  perpendicular  depths  below  the  surface  of  the  fluid. 


OF  FLUID  PRESSURE  ON  CIRCULAR  PLANES  AND  ON  SPHERES.       91 

Put  D  zz  AB,  the  diameter  of  the  greater  sphere  ABD, 

d  n=  ab,  the  diameter  of  the  lesser  sphere  abd, 
'|D  =  AC,  the  radius  of  the  greater  sphere,  or  the  perpendicular 

depth  of  its  centre  of  gravity, 
\d  zz  a  c,  the  radius  of  the  lesser  sphere,  or  the  perpendicular 

depth  of  its  centre  of  gravity, 
•S  zz  the  surface  of  the  greater  sphere  ABD, 
P  z=  the  pressure  perpendicular  to  its  surface, 
>S  =  the  surface  of  the  lesser  sphere  a  b  d, 
p  —  the  pressure  perpendicular  to  its  surface, 

and  TT  nr  3.1416,  the  circumference  of  a  circle  whose  diameter  is 
expressed  by  unity. 

Then,  according  to  the  principles  of  mensuration,  the  surface  of  a 
sphere  or  globe  — 

Is  equal  to  four  times  the  area  of  one  of  its  great  circles, 
or  that  whose  plane  passes  through  the  centre  of  the  sphere. 

Consequently,  the  convex  surface  of  the  greater  sphere  ABD,  is 
expressed  as  follows. 


and  that  of  the  lesser  sphere  abd,  is 

S  =:  3.1416  eP. 

But  the  pressure  perpendicular  to  any  surface,  is  equal  to  the  area 
of  that  surface  multiplied  by  the  perpendicular  depth  of  the  centre  of 
gravity,  and  again  by  the  specific  gravity  of  the  fluid  ;  consequently, 
when  the  specific  gravity  of  the  fluid  is  denoted  by  unity,  we  have  for 
the  pressure  on  the  surface  of  the  greater  sphere, 

P=r3.1416Da  X  iD=1.5708D8.  (50). 

and  for  the  pressure  on  the  lesser  sphere,  it  is 


hence,  by  comparison,  we  shall  have 

P  :  p  :  :  D3  :  d3. 

Consequently,  if  two  spheres  of  different  diameters  be  placed  in  a 
fluid  under  similar  circumstances,  the  pressures  perpendicular  to  their 
surfaces,  are  to  one  another  as  the  cubes  of  their  diameters. 

By  the  principles  of  mensuration,  the  solid  content  of  a  sphere  or 
globe,  is  equal  to  the  cube  of  the  diameter  multiplied  by  the  constant 
number  .5236  ;  therefore,  if  c  denote  the  solid  content,  we  have 

cin.5236D3, 


92       OF  FLUID  PRESSURE  ON  CIRCULAR  PLANES  AND  ON  SPHERES. 

or  multiplying  both  sides  of  the  equation  by  3,  we  get 

3czzl.5708D3; 
consequently,  by  equation  (50),  we  have 

or  if  s  denote  the  specific  gravity  of  the  fluid,  we  shall  obtain 

COROL.  Hence  we  infer,  that  if  a  hollow  sphere  or  globe  be  filled 
with  an  incompressible  and  non-elastic  fluid : — 

The  whole  pressure  sustained  by  the  internal  surface  of  the 
sphere  is  equal  to  three  times  the  weight  of  the  fluid  which  it 
contains. 

102.  EXAMPLE  21.  A  hollow  spherical  shell  or  vessel,  whose  inte- 
rior diameter  is  equal  to  30  feet,  is  completely  filled  with  water ;  what 
weight  is  equivalent  to  the  pressure  sustained  by  its  internal  surface  ? 

Here,  by  operating  according  to  the  process  indicated  in  equation 
(50),  we  have 

P=:  1.5708  X  303zr42411.6cub.ft. 

Now,  since  the  fluid  with  which  the  vessel  is  filled,  is  water,  giving  a 
weight  of  62 \  Ibs.  to  a  cubic  foot,  we  have 

P  •=.  4241 1.6  X  62  J  =:  2650725  Ibs. ; 

but  2240  Ibs.  are  equal  to  one  ton;  therefore,  the  pressure  on  the 
internal  surface  of  a  hollow  spherical  vessel  whose  diameter  is  30  feet, 
when  completely  filled  with  water,  is 

P  =  2650725  H-  2240  —  1 183  Hi  tons. 

PROBLEM  XVI. 

103.  Suppose  a  sphere  or  globe  to  be  immersed  in  an  incom- 
pressible and  non-elastic  fluid,  in  such  a  manner,  that  the  upper 
extremity  of  the  vertical  diameter  is  just  in  contact  with  its 
surface : — 

It  is  required  to  determine  through  what  point  of  the  axis 
a  horizontal  plane  must  pass,  so  to  divide  the  sphere,  that  the 
pressure  on  the  convex  surface  of  the  lower  segment,  may  be 
equal  to  the  pressure  on  the  convex  surface  of  the  upper. 

Let  AD  BE  represent  the  sphere  in  question,  so  placed,  that  A  the 
upper  extremity  of  the  vertical  diameter,  is  just  in  contact  with  FG 
the  surface  of  the  fluid. 


OF  FLUID  PRESSURE  ON  CIRCULAR  PLANES  AND  ON  SPHERES.       93 

Suppose  that  P  is  the  point  in  the  vertical 
diameter  through  which  the  plane  of  division 
passes,  separating  the  sphere  into  the  seg- 
ments DAE  and  DBE,  sustaining  equal  pres- 
sures on  their  convex  surfaces. 

Bisect  A  p  and  B  P  in  the  points  m  and  n  ; 
then  are  m  and  ny  the  points  thus  determined, 

respectively  the  centres  of  gravity  of  the  surfaces  of  the  spheric  seg- 
ments DAE  and  DBE,*  and  Am,  A.n  are  their  perpendicular  depths 
below  FG  the  horizontal  surface  of  the  fluid. 

Put  D  —  AB,  the  vertical  diameter  of  the  sphere  or  globe  A  DBE, 

d=:Am,  the  depth  of  the  centre  of  gravity  of  the  surface  of  the 

upper  segment  DAE, 
§  zz  A  n,  the  depth  of  the  centre  of  gravity  of  the  surface  of  the 

lower  segment  DBE, 
S  zz  the  surface  of  the  upper  segment, 
P  zn  the  pressure  upon  it, 
&'zz  the  surface  of  the  lower  segment, 
p  —  the  pressure  upon  it, 
s  zz  the  specific  gravity  of  the  fluid, 
x  zz  AP,  the  perpendicular  depth  of  the  point  through  which  the 

plane  of  division  passes,  and 
7TZZ3.1416,  the  circumference  of  the  circle  whose  diameter  is 

unity. 

Then,  according  to  the  principles  of  mensuration,  the  convex  sur- 
face of  a  spheric  segment  :  — 

Is  equal  to  the  circumference  of  the  sphere,  drawn  into  the 
versed  sine  or  height  of  the  segment  ivhose  surface  is  sought. 

And  moreover,  the  circumference  of  a  sphere,  or  the  circumference 
of  any  of  its  great  circles  :  — 

Is  equal  to  the  diameter  multiplied  by  the  constant  quantity 
TT,  or  the  number  3.1416. 

consequently,  the  convex  surface  of  the  upper  segment  DAE,  is 


and  that  of  the  lower  segment  DBE,  is 
S  zz  D  TT  (D  —  x)  zz  3.1416  D  (D  —  x}. 


*  It  is  demonstrated  by  the  writers  on  mechanics,  that  the  centre  of  gravity  of 
the  surface  of  a  spheric  segment,  is  at  the  middle  of  its  versed  sine  or  height. 


94       OF  FLUID  PRESSURE  ON  CIRCULAR  PLANES  AND  ON  SPHERES. 

But  the  pressure  perpendicular  to  any  surface,  whatever  may  be  its 
form,  as  we  have  already  sufficiently  demonstrated  :  — 

Is  equal  to  the  area  of  the  surface  multiplied  by  the 
perpendicular  depth  of  the  centre  of  gravity,  and  again  by 
the  specific  gravity  of  the  fluid. 

Therefore,  the  pressure  perpendicular  to  the  convex  surface  of  the 
upper  segment  DAE,  is 

P  =  3.1416Da:  X  i*  X  *  =  1.5708  DSX\  (51). 

and  the  pressure  perpendicular  to  the  convex  surface  of  the  lower 
segment  DBE,  is 


(52). 

Now  these  two  expressions,  according  to  the  conditions  of  the 
problem,  are  equal  to  one  another;  consequently,  by  comparison, 
we  get 

1  .5708  D  s  x*  —  3.1416  D  s  {(D  —  x)  x  -f  g  (D  —  a;)8}, 
and  from  this,  by  suppressing  the  common  quantities,  we  have 

*a:=:2{(D-aO*+i(D-.af}; 
therefore,  by  expanding  the  terms,  we  obtain 

2x*  —  DS  ; 
consequently,  by  division  and  evolution,  we  get 

ar=zJDV2.  (53). 

104.  The  ultimate  form  of  this  equation  is  extremely  simple,  and 
the  practical  rule  which  it  supplies,  may  be  expressed  as  follows. 

RULE.  Multiply  the  radius,  or  half  the  diameter  of  the 
sphere  by  the  square  root  of  2,  and  the  product  will  give 
the  point  in  the  vertical  diameter  through  which  the  plane  of 
division  passes,  estimated  downwards  from  the  surface  of  the 
fluid. 

105.  EXAMPLE  22.  A  sphere  or  globe,  whose  diameter  is  18  inches, 
is  immersed  in  a  fluid,  in  such  a  manner,  that  the  upper  extremity  is 
just  in  contact  with  the  surface  ;  through  what  point  of  the  diameter 
must  a  horizontal  plane  be  made  to  pass,  so  to  divide  the  sphere,  that 
the  pressures  on  the  curve  surface  of  the  upper  and  lower  segments 
may  be  equal  to  one  another  ? 

The  square  root  of  2,  is  1.4142,  and  half  the  given  diameter  is  9 
inches  ;  consequently,  by  the  rule  we  have 

x  =1.4142  X  9  —  12.  7278  inches, 


OF  FLUID  PRESSURE  ON   CIRCULAR  PLANES  AND  ON   SPHERES.       95 

106.  The  preceding  investigation  applies  to  the  particular  case,  in 
which  the  pressures  on  the  curve  surfaces  of  the  segments  are  equal 
to  one  another ;  but  in  order  to  render  the  solution  general,  we  must 
investigate   a  formula  to  indicate   the  point  of  division,  when  the 
pressures  are  to  one  another  in  any  ratio  whatever ;  for  instance,  that 
of  m  to  n. 

By  expunging  the  common  factors  from  the  equations  (51)  and 
(52),  we  obtain 

a» :  2  {  (D  —  a)  x  +  }  (D  —  *)2 }  :  :  m  :  n  ; 

therefore,  by  equating  the  products  of  the  extreme  and  mean  terms, 
we  get 

2m{(D—x)x  +  J(D  —  xY}  =  nx*, 
which,  by  expanding  the  bracketted  expression,  becomes 

nxt  —  m^  —  x*}, 
or  by  transposition,  we  obtain 

(m  -\-  n)  x*  nz  m  D2, 
and  finally,  by  dividing  and  extracting  the  square  root,  we  have 

rm 
m+n  (54). 

107.  The  general  equation  just  investigated,  is  sufficiently  simple 
in  its  form  for  every  practical  purpose  that  is  likely  to  occur ;  it  may 
therefore  appear  superfluous  to  reduce  it  to  a  rule,  yet  nevertheless, 
that  nothing  may  be  wanting  for  the  general  accommodation  of  our 
readers,  we  think  proper  to  draw  up  the  following  enunciation. 

RULE.  Divide  the  first  term  of  the  ratio  by  the  sum  of  the 
termSj  and  multiply  the  square  root  of  the  quotient  by  the 
diameter  of  the  sphere ;  then,  the  product  thus  arising,  will 
express  the  distance  below  the  surface  of  the  fluid,  of  that 
point  through  which  the  plane  of  division  passes. 

It  is  unnecessary  to  propose  an  example  for  the  purpose  of  illus- 
trating the  above  rule ;  that  which  we  have  already  given,  where  the 
values  of  m  and  n  are  equal  to  one  another,  being  quite  sufficient. 


CHAPTER  V. 

OF  THE  PRESSURE  OF  NON-ELASTIC  OR  INCOMPRESSIBLE  FLUIDS 
AGAINST  THE  INTERIOR  SURFACES  OF  VESSELS  HAVING  THE 

;  FORMS  OF  TETRAHEDRONS,  CYLINDERS,  TRUNCATED  CONES, 
&C. 

1.    WHEN  THE  VESSEL  IS  IN  THE  FORM  OF  A  TETRAHEDRON. 

PROBLEM  XVII. 

108.  Suppose  a  vessel  in  the  form  of  a  tetrahedron,  or  equi- 
lateral triangular  pyramid,  to  be  filled  with  an  incompressible 
and  non-elastic  fluid : — 

It  is  required  to  compare  the  pressure  on  the  base  with  that 
upon  the  sides,  and  also  with  the  weight  of  the  fluid;  the  base 
of  the  vessel  being  parallel  to  the  horizon. 

Let  ABCD  be  the  tetrahedron  filled  with  fluid,  of  which  ABC  is  the 
base  parallel  to  the  horizon,  and  ABD, 
c  B  D  and  ADC  the  sides  or  equal  contain- 
ing planes. 

From  D  the  vertex  of  the  figure,  let  fall 
the  perpendicular  DP,  upon  the  base  or 
opposite  side  ABC;  then  will  DP  be  the 
vertical  depth  of  the  centre  of  gravity  of 
the  base  ABC,  below  the  horizontal  plane  passing  through  D,  the 
summit  of  the  figure,  or  highest  particle  of  the  fluid. 

Bisect  AD  and  AB,  two  of  the  adjacent  edges  of  the  figure,  in  the 
points  m  and  n\  draw  the  straight  lines  BWI  and  DW,  intersecting 
each  other  in  the  point  r ;  then  is  r  the  centre  of  gravity  of  the 
triangular  plane  ABD. 

Through  the  point  r  draw  rs  perpendicular  to  DP,  the  altitude  of 
the  vessel  or  pyramid;  then  is  DS,  the  perpendicular  depth  of  the 
centre  of  gravity  of  the  triangular  plane  ADB,  below  the  vertex  D,  or 
the  uppermost  particle  of  the  fluid. 


OF  FLUID  PRESSURE  UPON  THE  INTERIOR  OF  A  TETRAHEDRON.        97 

By  the  nature  of  the  figure,  the  three  containing  planes  A  D  E,  ADC 
and  BDC,  are  equal  to  one  another,  and  they  are  also  equally  inclined 
to,  or  similarly  situated  with  respect  to  the  base  ABC;  consequently 
DS,  the  perpendicular  depth  of  the  centre  of  gravity,  is  common  to 
them  all. 

Now,  by  the  property  of  the  centre  of  gravity,  we  know,  that  or  is 
equal  to  two  thirds  of  D  n  ;  therefore,  by  reason  of  the  parallel  lines 
np  and  rs,  DS  is  also  equal  to  two  thirds  of  DP. 

Put  a  zn  the  area  of  the  base  and  each  of  the  other  containing  planes, 
I  ~  the  length  of  the  side  of  each  triangular  plane,  or  the  edges 

of  the  figure, 
d  zz:  DP,  the  perpendicular  depth  of  the  centre  of  gravity  of  the 

base  ABC, 
3  zr  DS,  the  perpendicular  depth  of  the  centre  of  gravity  of  the 

side  ADB  ; 

P  zz  the  pressure  upon  the  b*ase, 
i  ' 

p  zz  the  pressure  upon 

wzzthe  weight  of  the  fluid  contained  in  the  vessel,  and 
s  zz  the  specific  gravity. 

Then,  by  the  principles  of  Plane  Trigonometry  and  the  property  of 
the  right  angled  triangle,  we  have 


DP  —  d~  —  \/4  —  sec.*  30°, 

but  by  the  arithmetic  of  sines,  we  know,  that 

secJ30°zzli-; 
consequently,  by  substitution,  we  have 

.    i?  •         d=-LVe.  '  $  v;  :•:§":•:•"; 

Now,  according  to  the  construction  of  the  figure  and  the  property 
of  the  centre  of  gravity,  it  follows,  that  DS  is  equal  to  two  thirds  of 
DP  ;  hence  we  get 

*       2J     _ 
DszzrSz::  —  ^6. 

By  the  nature  of  the  figure,  it  is  manifest,  that  the  area  of  each  of 
the  triangular  sides  is  equal  to  the  area  of  the  base ;  and  by  the  prin- 
ciples of  mensuration  : — 

The  area  of  an  equilateral  triangle,  is  equal  to  one  fourth 
the  square  of  the  side,  multiplied  by  the  square  root.  *f 
three. 

X-oTST-^ 

/          or  THE 


98      OF  FLUID  PRESSURE  UPON  THE  INTERIOR  OF  A  TETRAHEDRON. 

Consequently,  the  area  of  the  base,  and  each  of  the  containing 
sides  of  the  vessel,  is  expressed  by 


therefore,  the   area  of  the  three   containing  equilateral  triangular 
planes,  becomes 


hence,  for  the  pressure  upon  the  base,  we  have 

P  =  4JV3"X  iV6  X  s-  iP5V%  (55). 

and  the  pressure  upon  the  three  containing  planes,  is 

j>  =  JJV3  X  JV6  X  s=il*s^2;  (56). 

consequently,  by  analogy,  we  shall  have 

P  :p  ::±Psi/iTi  JJ»«t/2; 

and  this,  by  suppressing  the  common  factors,  becomes 
P  :  p  :  :  1  :  2. 

If  the  two  equations  marked  (55)  and  (56)  be  added  together,  the 
sum  will  express  the  aggregate  pressure  upon  the  vessel  ;  therefore 
we  have 

P  +  P=J>'=(i  +  1)  l*sV2  =  $l*s^.  (57). 

According  to  the  principles  of  mensuration,  the  solid  content  of  a 
tetrahedronal  vessel,  is  equal  to  the  area  of  its  base,  multiplied  by  one 
third  of  its  perpendicular  altitude  ;  therefore,  we  have 

IJViFx  $1^/6  x  i=ZTW2; 

now,  the  weight  of  the  contained  fluid,  is  manifestly  equal  to  its 
magnitude  multiplied  by  the  specific  gravity  ;  consequently,  we 
obtain 

w  =  -frPsV2l  (58). 

hence,  by  analogy,  we  get 

P  :  w  :  :  %lss  j$  :  ^s  ^2, 
and  this,  by  suppressing  the  common  factors,  gives 

P  :  w  :  :  3  :  1. 

COROL.  It  therefore  appears,  that  the  pressure  upon  the  base,  is  to 
the  pressure  on  the  three  sides,  in  the  ratio  of  1  to  2,  and  to  the 
weight  of  the  contained  fluid  in  the  ratio  of  3  to  1  ;  consequently,  the 
weight  of  the  fluid,  the  pressure  on  the  base,  and  the  pressure  on  the 
sides,  are  to  one  another  as  the  numbers  1,  3  and  6. 


99 

2.    WHEN  THE  VESSEL  IS  IN  THE  FORM  OF  A  CYLINDER. 

PROBLEM  XVIII. 

109.  If  a  cylindrical  vessel  be  completely  filled  with  an 
incompressible  and  non-elastic  fluid,  and  so  placed,  that  its 
bottom  may  be  parallel  to  the  horizon: — 

It  is  required  to  compare  the  pressure  against  its  bottom, 
with  that  against  its  upright  surface,  and  also  with  the 
weight  of  the  fluid  which  it  contains. 

Let  ABCD  be  a  vertical  section  of  a  cylindrical  vessel,  filled  with 
an  incompressible  and  non-elastic  fluid,  whose 
surface  AB  is  horizontal,  and  let  it  be  required 
to  compare  the  pressure  exerted  by  the  fluid  on 
the  bottom  DC,  with  that  upon  the  whole  upright 
surface. 

Draw  the  diagonals  AC  and  BD  intersecting 
one  another  in  the  point  p,  and  through  the  point 
p,  draw  the  vertical  line  mn,  meeting  DC  the 
bottom  of  the  vessel  in  the  point  m,  and  A  B  the  surface  of  the  fluid  in 
n;  then  is  m  the  position  of  the  centre  of  gravity  of  the  bottom,  and 
nm  its  perpendicular  depth  below  the  surface  AB. 

Bisect  AD  in  E,  and  through  E  draw  EP  parallel  to  A  B  or  DC,  and 
meeting  the  vertical  line  mn  in  p  ;  then  is  p  the  position  of  the  centre 
of  gravity  of  the  upright  surface,  and  n  p  its  perpendicular  depth  below 
AB  the  surface  of  the  fluid. 

Put  D  nz  AB  or  DC,  the  diameter  of  the  cylindrical  vessel  proposed, 
d  ~  nm,  the  perpendicular  depth  of  the  centre  of  gravity  of  the 

bottom  DC,  below  AB  the  surface  of  the  fluid, 
3  ~  nv,  the  perpendicular  depth  of  the  centre  of  gravity  of  the 

upright  surface, 

Anr  the  area  of  the  base  or  bottom  of  the  cylinder, 
P  —  the  pressure  upon  it, 
«  nz  the  area  of  the  curved  or  upright  surface, 
p  —  the  pressure  upon  it, 
w  zzz  the  weight, 
and  s  z=  the  specific  gravity  of  the  fluid. 

Then,  by  the  principles  of  mensuration,  the  area  of  the  base  or 
bottom  of  the  cylindric  vessel,  is 

A=r.7854D2, 
H  2 


100      OF  FLUID  PRESSURE  UPON  TTTE  INTERIOR  OF  A  CYLINDER. 

and  that  of  the  upright  surface,  is 

a  —  3.1416DG?; 
consequently,  the  pressure  on  the  bottom  becomes 

P  =  .r7S54v*ds,  (59). 

and  for  the  pressure  upon  the  upright  surface,  we  have 

p=:3.Ul6vdSs;  (60). 

therefore,  by  analogy,  we  obtain 
P  :p  :  :  .7854  D*ds  :  3.1416D^s; 
from  which,  by  omitting  the  common  factors,  we  get 

P  :p  ::  D  :  43; 

now,  by  the  construction  of  the  figure,  we  have  3  z=  \d  ;  therefore 
4£  zz  2d,  and  by  substitution,  we  obtain 

P  :  p  :  :  D  :  2d  :  :  |D  :  d. 

Hence  it  appears,  that  the  pressure  upon  the  bottom  of  a  cylindrical 
vessel,  is  to  the  pressure  upon  its  upright  surface,  as  the  radius  of  the 
base  is  to  the  perpendicular  altitude. 

Since  the  entire  pressure  sustained  by  a  cylindrical  vessel,  is  equal 
to  the  sum  of  the  pressures  on  the  bottom  and  the  upright  sides,  it 
follows,  that 

P  -f  p  =  p'=  .7854  (D  +  43)  vds, 
or  substituting  \d  for  3,  we  shall  obtain 

p'  =  .7854  (D  +  2d)  vds.  (61). 

It  is  demonstrated  by  the  writers  on  mensuration,  that  the  solid 
content  of  a  cylinder,  is  equal  to  the  area  of  its  base,  drawn  into  its 
perpendicular  altitude  ;  therefore,  we  have 


where  C  denotes  the  solid  content  of  the  cylinder. 

Now,  it  is  manifest,  that  the  weight  of  an  incompressible  and  non- 
elastic  fluid,  is  equal  to  its  magnitude  drawn  into  its  specific  gravity; 
hence  we  have 


but  this  is  precisely  the  expression  which  we  have  given  in  equation 
(59),  for  the  pressure  perpendicular  to  the  bottom  of  the  vessel  ;  con- 
sequently, the  weight  of  the  fluid,  and  the  pressure  on  the  bottom  of 
the  vessel,  are  equal  to  one  another  ;  hence,  the  following  inference. 

110.  When  the  sides  of  a  vessel  of  any  form  whatever,  are  perpen- 
dicular, and  its  base  parallel  to  the  horizon  :  — 


OF  FLUID  PRESSURE   UPON  THE  ANNULI  OF  A  CYLINDER.          101 

The  pressure  perpendicular  to  the  base  of  the  vessel,  is 
equal  to  the  whole  weight  of  the  fluid  which  it  contains. 

This  is  manifest,  for  the  whole  pressure  of  the  fluid  is  sustained  by 
the  base  and  the  sides  together,  and  the  sides  being  in  the  direction 
of  gravity,  sustain  no  part  of  the  pressure  which  is  exerted  perpendi- 
cularly downwards ;  consequently,  the  whole  weight  of  the  fluid  is 
sustained  by  the  base. 

3.    WHEN  THE  PRESSURE  UPON  THE  ANNULI  OF  A  CYLINDER    IS    TO    BE 

DETERMINED. 

PROBLEM  XIX. 

111.  If  a  cylindrical  vessel  whose  bottom  is  parallel,  and 
sides  perpendicular  to  the  horizon,  be  filled  with  an  incom- 
pressible and  non-elastic  fluid: — 

It  is  required  to  divide  the  concave  surface,  into  any 
number  n  of  horizontal  annuli,  in  such  a  manner,  that  the 
pressure  on  each  annulus  shall  be  equal  to  the  pressure  on 
the  bottom  of  the  vessel. 

Let  ABCD  be  a  vertical  section,  passing  along  the  axis  of  the 
cylinder,  or  vessel  containing  the  fluid,  whose  surface 
is  AB  ;    draw  the  diagonals  AC  and  BD  intersecting 
one  another  in  the  point  p  ;  then  is  p  the  centre  of 
gravity  of  the  cylindrical  surface. 

Through  p  the  point  of  intersection,  draw  the  ver- 
tical line  mn  parallel  to  AD  or  BC,  and  let  a,  b  and  c 
be  the  points,  which  with  the  extremities  A  and  D  of 
the  side  AD,  terminate  the  several  annuli :  then,  through  the  points 
a,  b  and  c,  and  parallel  to  AB  or  DC,  draw  the  straight  lines  af, 
be  and  cd,  cutting  BC  the  opposite  side  of  the  section  in  the  points 
ft  e  and  d. 

Draw  the  zigzag  diagonals  Af,  fb,  bd  and  dv,  intersecting  the 
vertical  line  mn  in  the  points  k,  i,  h  and  g ;  then  are  the  points  thus 
determined,  respectively  the  centres  of  gravity  of  the  several  annuli, 
into  which  the  concave  surface  of  the  vessel  is  supposed  to  be  divided ; 
and  n  k,  ni,  nh  and  ng,  are  the  respective  depths  below  the  surface 
of  the  fluid,  WP  being  the  depth  of  the  centre  of  gravity  of  the  whole 
upright  surface  of  the  cylindrical  vessel,  and  nm  the  vertical  depth 
of  the  bottom. 


'    -     i  ^  o  *»y 

• ' ' '     :  •     ' 

aa 

«   &pO  e     *' 

102         OF  FLUID  PRESSURE  UPON  THE  ANNULT  OF  A  CYLINDER. 

Put  D  =  AB  or  DC,  the  diameter  of  the  proposed  cylindrical  vessel, 
A   •=.  the  area  of  its  bottom, 
d  =nm,  the  whole   perpendicular  depth,  or   altitude   of  the 

cylinder, 

x  ~Aa,  the  breadth  of  the  first  annulus, 
x'  z=ab,  the  breadth  of  the  second, 
x"  —  £c,  the  breadth  of  the  third, 
x'"—  CD,  the  breadth  of  the  fourth,  and  so  on,  to  any  number 

of  annuli  n, 
P  —  the  pressure  on  the  concave  surface  of  the  cylinder,  or  the 

sum  of  the  pressures  on  the  several  annuli  into  which 

it  is  divided, 
p   —  the  pressure  on  the  bottom  of  the  vessel,  and  each  of  the 

several  annuli, 
TT  =z  3.1416  the  circumference  of  a  circle  whose  diameter  is 

unity,  and 
s    zr  the  specific  gravity  of  the  fluid. 

Then  we  have,  nk  —  \x ;  ni  in  x  -\-  \x  ;  n h  —  x  -f-  x'  -|-  \x", 
and  ng  —  x  -\-  x1  -|-  x"  +  kx>"  »  an(*  by  the  principles  of  mensuration, 
the  area  of  the  bottom  of  the  vessel,  is 

AZrjTTD*, 

and  by  Proposition  (1),  the  pressure  upon  it,  is 

p  —  l7n>*ds=  .7854  i>M$,  (62). 

Again,  by  the  principles  of  mensuration,  the  concave  surfaces  of  the 
respective  annuli  are  as  follows,  viz. 

For  the  first  annulus,  we  have  TT  D  x    —  3.1416  D  x,  the  surface, 

second TTDO;'  ~  3.1416  D  x',    

third TTDX"  —  3.1416  DX",  

fourth TT DB"'  =  3.1416 Da?1", 

&c. &c.      —  &c. 

And  by  Proposition  (1),  the  pressures  perpendicular  to  these  sur- 
faces, are  respectively  as  below,  viz. 
For  the  first  annulus,  the  pressure  isp=zl.5708Da:a  s, 

second j0=3.1416D*'  s(x+ J#'), 

third p— 3.1416Dx"s(aj+x/+Ja:1'), 

fourth  -  -P=i3.U\6»x'"s(x+x'+x"+%xf"), 

&c. &c.  &c. 

Now  each  of  these  pressures,  according  to  the  conditions  of  the 
problem,  is  equal  to  the  pressure  upon  the  bottom,  exhibited  in  the 
equation  (62) ;  consequently,  by  comparison,  we  have 


OF  FLUID  RRESSURE  UPON   THE  ANNULI  OF  A  CYLINDER.         103 


1.  5708  vx*s  :=. 
and  casting  out  the  common  factors,  we  get 

2a??=  vd; 
therefore,  by  division  and  evolution,  we  have 

x=%^2nd'.  (63). 

By  proceeding  in  a  similar  manner  for  the  breadth  of  the  second 
annulus,  we  shall  obtain 


and  this,  by  expunging  the  common  terms,  becomes 

4*'(*  +  J*')  =  Drf; 

therefore,  by  substituting  for  x,  its  value  as  expressed  in  equation  (63), 
we  shall  get 


complete  the  square,  and  we  have 

a'2  -\-^2iTdx  +  %vd=:  vd; 
and  finally,  extracting  the  square  root  and  transposing,  we  obtain 

*'  =  4(2—^2)  J~d.  (64). 

Again,  by  performing  a  similar  process  for  the  breadth  of  the  third 
annulus,  we  shall  have 

3.1416D3"s  (x  -f-  x'  +  i«")  =  .7854  D«d«, 
from  which,  by  casting  out  the  common  quantities,  we  get 

4x"(x  +  x'+%x")=:i>d; 

therefore,  by  substituting  for  x  and  x',  their  values  as  expressed  in  the 
equations  marked  (63)  and  (64),  and  we  shall  obtain 

4x"  {J  Jtod+  J(2—  V2)  V~^~d+  ix"}  =  Dd, 
and  this,  by  a  little  reduction  and  proper  arrangement,  gives 


complete  the  square,  and  we  obtain 


consequently,  by  extracting  the  square  root  and  transposing,  we  get 

a/;-=JV6—  2)^25-  (65> 

Pursuing  a  similar  train  of  reasoning  for  the  breadth  of  the  fourth 
annulus,  we  shall  obtain 

3.1416D*"'s  (x  +  x'  +  x"  -f  iO  =  .7854D«rf«, 
and  by  suppressing  the  common  factors,  we  have 

4*"'  (x  +  x'  +  x"  4-  1*'")  =  »d  ; 


104         OF  FLUID  PRESSURE  UPON   THE  ANNULI  OF  A  CYLINDER. 

sustitute  in  this  equation,  the  values  of  x,  x  and  x"  as  represented  in 
the  equations  marked  (63),  (64),  and  (65),  and  we  get 


(2  —  -t/2)  VD^-KV6  —  2)  V 
and  this,  by  a  little  further  reduction,  becomes 


therefore,  by  completing  the  square,  we  obtain 

a;"'*  4-  i/fiDd.x"1  -f  i  Drf—  2od; 

and  finally,  by  extracting  the  square  root  and  transposing,  we  have 
x1"  =  J  (2  V~2  —  V  6)  V^  (66). 

112.  And  thus  we  may  proceed  to  any  extent  at  pleasure  ;  that  is, 
to  any  number  of  annuli  within  the  limit  of  possibility  ;  for  it  is  mani- 
fest, from  the  nature  of  the  problem,  that  impossible  cases  may  be 
proposed,  but  the  limit  can  easily  be  ascertained  in  the  following 
manner. 

It  is  obvious,  that  the  sum  of  the  breadths  of  the  several  annuli,  is 
equal  to  the  whole  depth  of  the  vessel  ;  and  that  the  sum  of  the 
pressures  is  equal  to  the  pressure  on  the  concave  surface  ;  but  in  the 
problem  immediately  preceding,  we  have  demonstrated  that  the  pres- 
sure on  the  bottom  of  a  cylindrical  vessel,  is  to  that  upon  its  upright 
surface,  as  the  radius  of  the  base  is  to  the  perpendicular  altitude. 

Now,  according  to  the  conditions  of  the  question,  the  pressure  on 
each  annulus  is  equal  to  that  upon  the  base  ;  consequently,  in  order 
that  the  problem  may  be  possible,  the  depth  of  the  vessel  must  be 
equal  to  the  radius  of  the  base,  drawn  into  the  number  of  annuli. 

If  instead  of  D  the  diameter  of  the  cylindrical  vessel,  we  substitute 
2R  its  equivalent  in  terms  of  the  radius,  the  preceding  equations  (63), 
(64),  (65),  and  (66),  will  become  transformed  into 

x  =  (/f  —  v'O)  y^,  (67). 

*'  —  (V2—  /f)  Va3,  (68). 

~  "  (69). 

(70). 


From  these  equations  the  law  of  induction  becomes  manifest,  and 
the  general  expression  for  the  breadth  of  the  n^  annulus,  is 


(71). 


OF  FLUID  PRESSURE  UPON  THE  ANNULI  OF  A  CYLINDER.         105 

113.  And  from  the  above  general  form  of  the  equation,  the  follow- 
ing practical  rule  may  be  derived,  for  calculating  the  breadth  of  any 
proposed  annulus,  independently  of  the  breadths  of  those  which  pre- 
cede it. 

RULE.  From  the  square  root  of  the  number  which  expresses 
the  place  of  the  required  annulus,  subtract  the  square  root  of 
that  number  minus  unity ;  then,  multiply  the  remainder  by 
the  geometric  mean  between  the  altitude  of  the  vessel  and  the 
radius  of  its  base,  and  the  product  will  give  the  breadth  of  the 
required  annulus. 

114.  EXAMPLE.  A  cylindrical  vessel  has  the  radius  of  its  base,  and 
its  perpendicular  depth,  respectively  equal  to  4  and  24  feet;  now, 
supposing  the  concave  surface  to  be  divided  into  6  horizontal  annuli, 
such,  that  the  pressure  upon  each  shall  be  equal  to  the  pressure  upon 
the  base  ;  required  the  breadth  of  the  fourth  annulus  ? 

By  performing  the  operation  as  directed  in  the  preceding  rule,  we 
shall  obtain 

x'"  —(^—  V3)X  v/4  X  24 z=2.625  feet  nearly. 

The  annulus  which  we  have  just  determined,  corresponds  to  the 
fourth  of  the  preceding  class  of  equations,  or  that  marked  (69),  and 
the  distance  of  its  centre  of  gravity  below  the  surface  of  the  fluid,  or 
its  position  with  respect  to  the  bottom  or  top  of  the  vessel,  can  easily 
be  ascertained. 

The  area  of  the  cylinder's  base,  is 

A  — 3.1416B9; 

the  pressure  which  it  sustains,  is 

p  =  3. 1416  R2d=  1206.3744, 

and  this  is  equal  to  the  pressure  on  the  annulus. 

Now,  according  to  the  writers  on  mensuration,  the  area  of  the 

annulus,  or  the  curved  surface  of  a  cylinder,  whose  radius  is  4  feet 

and  its  perpendicular  altitude  2.625  feet,  is  expressed  as  follows,  viz. 

6.2832  X  4  X  2.625  =  65.9736. 

If  therefore,  we  divide  the  pressure  on  the  base  of  the  vessel,  by  the 
area  of  the  annulus,  the  depth  of  its  centre  of  gravity  will  become 
known ;  thus,  we  have 


106 


4.  WHEN  THE  VESSEL  ASSUMES  THE  FORM  OF  A  TRUNCATED  CONE, 
THE  BASE  OF  WHICH  IS  ALSO  THE  BOTTOM  OF  THE  VESSEL,  AND 
ITS  AXIS  PERPENDICULAR  TO  THE  HORIZON. 

PROBLEM  XX. 

115.  If  a  vessel  in  the  form  of  the  frustum  of  a  cone,  be 
filled  with  an  incompressible  and  non-elastic  fluid,  and  have  its 
axis  perpendicular  to  the  horizon : — 

It  is  required  to  compare  the  pressure  on  the  bottom  of  the 
vessel  with  that  upon  its  curved  surface,  and  also  with  the 
weight  of  the  fluid  which  it  contains,  both  when  the  sides  of 
the  vessel  converge,  and  when  they  diverge  from  the  extremities 
of  the  bottom. 

Let  ABCD  represent  a  vertical  section  of  a  vessel  in  the  form  of  the 
frustum  of  a  cone,  and  filled  with  an  incom- 
pressible and  non-elastic  fluid  whose  horizontal 
surface  is  AB  ;  produce  AB  both  ways,  to  any 
convenient  distance,  and  through  D  and  c  the 
extremities  of  the  bottom  diameter,  draw  Da 
and  c  b  respectively  perpendicular  to  D  c,  and 
meeting  AB  produced  in  the  points  a  and  b; 
then  is  abcv  the  vertical  section,  passing 
along  the  axis  of  the  cylinder  which  circumscribes  the  conic  frustum. 

Bisect  AB  and  DC  respectively  in  the  points  m  and  n,  and  draw 
the  straight  line  mn\  then,  because  the  figure  ABCD  is  symmetrical 
with  respect  to  the  axis  mn,  it  follows,  that  mn  bisects  the  figure 
or  trapezoid  ABCD,  and  consequently  passes  through  its  centre  of 
gravity. 

Draw  the  diagonal  AC,  dividing  the  figure  ABCD  into  the  two 
triangles  ABC  and  ADC  ;  then  it  is  manifest,  that  the  common  centre 
of  gravity  of  the  two  triangles,  and  that  of  the  trapezoid  constituted 
by  their  sum,  must  occur  in  one  and  the  same  point ;  therefore,  bisect 
the  diagonal  A  c  in  the  point  t,  and  draw  A  n  and  D  t  intersecting  each 
other  in  the  point  r,  and  c  m,  B  t  intersecting  in  s ;  then  are  r  and  s 
the  centres  of  gravity  of  the  triangles  ADC  and  ABC;  draw  rs  inter- 
secting mn  in  G,  and  G  will  be  the  centre  of  gravity  of  the  trapezoid 
ABCD. 

Now,  it  is  demonstrated  by  the  writers  on  mechanics,  that  the 
centre  of  gravity  of  the  surface  of  a  conic  frustum  : 


OF  FLUID  PRESSURE  UPON  THE  INTERIOR  OF  CONICAL  VESSELS.     107 

Is  situated  in  the  axis,  and  at  the  same  distance  from  its 
extremities,  as  is  the  centre  of  gravity  of  the  trapezoid,  which 
is  a  vertical  section  passing  along  the  axis  of  the  solid. 

Therefore,  since  by  the  construction,  the  point  G  has  been  shown  to 
be  the  centre  of  gravity  of  the  trapezoid,  it  is  also  the  centre  of  gravity 
of  the  surface  of  the  conic  frustum,  and  TWO  is  its  perpendicular  depth 
below  the  surface  of  the  fluid. 

Put  AZZ:  the  area  of  the  base  or  bottom  of  the  vessel,  whose  diameter 

is  DC, 
v=mn,  the  perpendicular  depth  of  its  centre  of  gravity,  or  the 

length  of  the  axis  of  the  vessel, 
a  —  the  curve  surface  of  the  conic  frustum, 
d^nmo,  the  perpendicular  depth  of  the  centre  of  gravity, 
/3  zz:  DC,  the  diameter  of  the  base  or  bottom  of  the  vessel, 
S  zz:  AB,  the  diameter  of  the  top, 
Pzz:  the  pressure  on  the  bottom, 
p  HZ  the  pressure  on  the  curve  surface, 
wzz:  the  weight,  and  s  the  specific  gravity  of  the  fluid. 

Then,  according  to  the  principles  of  mensuration,  the  area  of  the 
lower  base  of  the  conic  frustum,  or  the  bottom  of  the  vessel  on  which 
the  fluid  presses,  becomes 

A  =  .  7854/3% 
and  consequently,  the  pressure  which  it  sustains,  is 

Pzz:. 7854 /32DS.  (72). 

In  the  next  place,  the  area  of  the  curved  surface  of  the  conic 
frustum,  or  the  sides  of  the  vessel  containing  the  fluid,  is 


a=  1.5708  08  +  3)X  V**  +  1  (ft  —  W  I 
and  therefore,  the  pressure  which  it  sustains,  is 

p  =  1.5708  ()8  4-  5)  ds  VD*  4-  403—  S)2.  (73). 

Now,  according  to  the  writers  on  mechanics,  the  depth  of  the  centre 
of  gravity  of  the  trapezoid  ABCD,  below  the  horizontal  line  AB,  is 
obtained  in  the  following  manner  : 

3  (/3  4  3)  :  D  :  :  2/3  4  3  :  d, 
and  by  equating  the  products  of  the  extremes  and  means,  we  get 


therefore,  dividing  by  3  (/3  4-  3),  we  obtain 
_ 
= 


108    OF  FLUID  PRESSURE  UPON  THE  INTERIOR  OF  CONICAL  VESSELS. 

Let  this  value  of  d  be  substituted  instead  of  it,  in  the  equation 
marked  (74),  and  we  shall  have  for  the  pressure  on  the  curved  surface 
of  the  vessel 

p  —  .5236  D  s  (2/3  4-  3)  V^-4-  J03  —  3)*  ;  (75). 

consequently,  by  comparing  the  equations  (72)  and  (74),  we  get 
Pip::  .7854/3*DS  :  .5236Ds(2/3  +  3)  J  D2 


and  this,  by  suppressing  the  common  factors,  becomes 

P  :  p  :  :  3/3*  :  2  (2/3  +  3)  V»*  +  i  (P  -  *)«-  (76). 

If  J,  the  upper  diameter  of  the  frustum  vanishes,  the  figure  becomes 
a  complete  cone,  and  consequently,  the  pressure  upon  the  base,  is  to 
that  upon  the  curve  surface,  as  three  times  the  diameter  of  the  cone, 
is  to  four  times  its  slant  height  ;  that  is 

P:p::  3/3:  4^+1^.  (77). 

According  to  the  principles  of  solid  geometry,  the  capacity  of  the 
conic  frustum,  or  the  quantity  of  fluid  which  the  vessel  contains,  is 


where  c  denotes  the  solid  content  of  the  vessel. 
But  the  weight  of  any  quantity  or  mass  of  fluid,  varies  directly  as 
the  magnitude   and  specific  gravity  conjointly;    consequently,  the 
weight  of  fluid  in  the  vessel,  is  expressed  by 

w  —  .2618  D  s  (/3s  +  /3  J  -f  a2).  (78). 

Hence,  if  the  equations  marked  (72)  and  (78),  be  compared  with 
each  other,  we  shall  obtain 

p^i-.s/^os'-h/sa+a2),  (79). 

and  when  3  vanishes,  the  vessel  becomes  a  complete  cone,  and  conse- 
quently, we  get 

P  :  w  :  :  3  :  1,  (80). 

It  therefore  appears,  that  the  pressure  against  the  bottom  of  a 
conical  vessel,  when  filled  with  an  incompressible  and  non-elastic 
fluid,  (the  bottom  being  downwards)  : 

Is  equivalent  to  three  times  the  weight  of  the  fluid  which  it 
contains. 

The  solidity  of  the  cylinder  circumscribing  the  conic  frustum,  of 
which  abci>  is  a  vertical  section,  is 

c'  =r.7854  f?  D, 
where  c'  denotes  the  capacity  of  the  cylinder  circumscribing  the  vessel; 


OF  FLUID  PRESSURE  UPON  THE  INTERIOR  OF  CONICAL  VESSELS.    109 

and  because  the  weight  of  any  quantity  or  mass  of  fluid,  is  propor- 
tional to  the  magnitude  and  specific  gravity  jointly ;  it  follows,  that  if 
w'  denote  the  weight  of  the  circumscribing  column  of  fluid,  we  obtain 
w/z=.7854/32Ds.  (81). 

COROL.  Now  this  expression  is  precisely  the  same,  as  that  which  we 
obtained  for  the  pressure  on  the  bottom,  indicated  by  the  equation 
marked  (72) ;  hence  it  appears,  that  when  the  sides  of  the  vessel 
converge  from  the  extremities  of  the  diameter  of  its  base  towards  each 
other  : — 

The  pressure  on  the  base  or  bottom  of  the  vessel,  is  equal  to 
the  weight  of  a  column  of  the  fluid,  of  the  same  magnitude 
as  the  cylinder  circumscribing  the  conic  frustum,  or  the  vessel 
by  which  the  fluid  is  contained. 

But  the  circumscribing  cylinder  is  manifestly  greater  than  the  conic 
frustum ;  consequently,  the  pressure  upon  the  base  or  bottom  of  the 
vessel,  is  greater  than  the  weight  of  the  fluid  which  it  contains  ;  and 
it  is  obvious,  that  the  additional  pressure  arises  from  the  re-action  of 
the  converging  sides. 

5.    WHEN  THE    VESSEL    REPRESENTS    AN    INVERTED    TRUNCATED    CONE, 
WITH    ITS    AXIS    PERPENDICULAR    TO    THE    HORIZON. 

116.  If  the  sides  of  the  vessel  diverge  from  the  extremities  of  the 
base,  as  represented  in  the  subjoined  diagram ;  then,  it  may  be 
shown,  that  the  weight  of  the  fluid  which  the  vessel  contains,  exceeds 
the  pressure  upon  its"  base. 

Let  ABCD  be  a  vertical  section,  passing 
along  the  axis  of  a  vessel  in  the  form  of  a 
conic  frustum,  and  which  is  filled  with  an 
incompressible  fluid  whose  horizontal  surface 
is  AB  ;  the  greater  base  of  the  frustum  being 
uppermost,  or  which  is  the  same  thing,  the 
sides  diverging  from  the  extremities  of  the  lower  diameter. 

Bisect  the  diameters  A  B  and  c  D  respectively  in  the  points  m  and  n ; 
draw  mn,  and  through  the  points  D  and  c,  the  extremities  of  DC,  draw 
the  straight  lines  Da  and  cb  respectively  parallel  to  mn,  and  meeting 
AB  in  the  points  a  and  b;  then  is  abcn  a  vertical  section  passing 
along  the  axis  of  the  inscribed  cylinder. 

Draw  the  diagonal  AC,  dividing  the  trapezoid  ABCD,  into  the  two 
triangles  ABC  and  ADC;  bisect  the  diagonal  AC  in  the  point  t,  and 


110    OF  FLUID  PRESSURE  UPON  THE  INTERIOR  OF  CONICAL  VESSELS. 

draw  B  t  and  D  t,  which  will  be  intersected  by  the  straight  lines,  c  m 
and  \n  iu  the  points  r  and  s;  then  are  r  and  s  respectively  the 
centres  of  gravity  of  the  triangles  ABC  and  ADC. 

Join  the  points  r  and  s,  by  the  straight  line  rs,  intersecting  mn  in 
the  point  G  ;  then  it  is  obvious,  that  the  common  centre  of  gravity  of 
the  triangles  ABC  and  ADC,  (which  coincides  with  that  of  the  trapezoid 
ABCD),  must  occur  in  the  line  rs,  which  joins  their  respective  centres. 

Now,  because  the  trapezoid  A  B  c  D  is  symmetrically  situated  with 
respect  to  the  axis  mn,  it  follows,  that  its  centre  of  gravity  must  occur 
in  that  line ;  but  we  have  shown  above,  that  it  also  occurs  in  the  line 
rs,  it  consequently  must  be  situated  in  the  point  G,  where  these  lines 
intersect  one  another  ;  hence,  the  centre  of  gravity  of  the  surface  of 
the  conic  frustum  occurs  at  the  point  G,  and  m  G  is  its  perpendicular 
depth  below  AB  the  upper  surface  of  the  fluid. 

Put  A  •=.  the  area  of  the  lower  base  or  bottom  of  the  vessel,  whose 

diameter  is  DC, 
P  —  the  pressure  perpendicular  to  its  surface,  or  the  weight  of  a 

quantity  of  fluid  equal  to  the  inscribed  cylinder, 
Dzzmw,  the  axis  of  the  frustum,  or  the  perpendicular  depth  of 

the  centre  of  gravity  of  the  bottom, 

a  zz:  the  area  of  the  curve  surface  of  the  vessel  or  conic  frustum, 
p  zz:  the  pressure  perpendicular  to  the  curve  surface, 
d  zz:  m&,  the  perpendicular  depth  of  its  centre  of  gravity, 
3  zz:  DC,  the  diameter  of  the  lower  base  or  bottom  of  the  vessel, 
ft  zz:  AB,  the  diameter  of  the  top  or  upper  base, 
w—  the  weight,  and  s  the  specific  gravity  of  the  contained  fluid. 

Then,  by  the  principles  of  mensuration,  the  area  of  the  lower  base 
of  the  conic  frustum,  or  the  bottom  of  the  vessel  on  which  the  fluid 

presses,  is 

Azzr.785422, 

and  consequently,  the  pressure  upon  it,  is 

Pzz:.7854j2Ds.  (82). 

This  equation,  having  52  instead  of  /32,  is  the  same  as  that  which  we 
obtained  for  the  pressure  on  the  bottom  in  the  preceding  case,  when 
the  greater  base  of  the  vessel  was  downwards ;  it  therefore  follows, 
since  our  notation  is  adapted  to  the  same  parts  of  the  vessel,  that  not- 
withstanding the  inversion,  the  pressure  on  the  curved  surface  of  the 
conic  frustum,  will  still  be  expressed  as  in  the  equation  marked  (73) ; 
consequently,  we  have 

P  :p:  :.7854$2ns::  1.5708  (/3-f  2)ds  VD' 


OF  FLUID  PRESSURE  UPON  THE  INTERIOR  OF  CONICAL  VESSELS.    Ill 

or  by  expunging  the  common  quantities,  we  get 


P  :p  :  :  3*D  :  2(/3  +  3)  dji>*  +  J(/3  —  3)'. 

But  the  writers  on  mechanics  have  demonstrated,  that  the  depth  of 
the  centre  of  gravity  of  the  trapezoid  A  BCD,  below  the  horizontal  line 
AB,  is  expressed  as  follows,  viz. 


'  (83). 


let  therefore,  this  value  of  d  be  substituted  instead  of  it  in  the  above 
analogy,  and  we  shall  obtain 


P  :  p  :  :  33*  :  2  (ft  +  23)  V  »*  +  I  (ft  —  V- 

If  £,  the  diameter  of  the  bottom  or  lower  base  should  vanish,  the 
vessel  becomes  a  complete  cone  with  its  vertex  downwards,  in  which 
case,  the  value  of  d  as  expressed  in  the  equation  marked  (83),  is 


Let  this  value  of  d  be  substituted  instead  of  it,  in  the  equation 
marked  (73),  and  suppose  3  to  vanish  ;  then,  the  pressure  on  the  con- 
cave surface  of  a  conical  vessel  with  its  vertex  downwards,  becomes 


p  =  .5236  ft  D  s  V  D9  +  J r/P.  (84). 

The  solid  content  of  the  inscribed  cylinder,  of  which  the  vertical 
section  passing  along  the  axis  is  a6co,  becomes 

c'=r.785432D, 

and  as  we  have  already  stated,  its  weight  is  proportioned  to  its  mag- 
nitude drawn  into  the  specific  gravity ;  hence  we  have 

«/  =  .  7854  3*Ds; 

but  this  is  the  same  expression  which  indicates  the  pressure  on  the 
bottom,  as  exhibited  in  the  equation  marked  (82) ;  hence  it  follows, 
that  the  pressure  on  the  bottom  or  lower  base  of  the  conic  frustum, 
when  the  sides  diverge  from  the  extremities  of  its  diameter, 

Is  equal  to  the  weight  of  a  column  of  the  fluid,  of  the  same 
magnitude  as  the  cylinder  inscribed  in  the  conic  frustum. 

But  the  solid  content  of  the  inscribed  cylinder,  and  consequently 
its  weight,  is  manifestly  less  than  the  content  of  the  vessel ;  hence  we 
infer,  that  when  the  sides  of  the  vessel  diverge  from  the  extremities  of 
the  diameter  of  its  bottom,  the  pressure  on  the  bottom  is  less  than 
the  weight  of  the  fluid  which  it  contains,  the  remaining  weight  being 
supported  by  the  resistance  of  the  diverging  sides. 


112    OF  FLUID  PRESSURE  UPON  THE  INTERIOR  OF  CONICAL  VESSELS. 

117.  In  order  to  compare  the  pressure  on  the  bottom  of  the  vessel, 
with  the  weight  of  the  fluid  which  it  contains,  we  must  again  have 
recourse  to  the  principles  of  solid  geometry  ;  from  which  we  learn, 
that  the  solid  content  of  a  conic  frustum,  whose  diameters  are  denoted 
by  (3  and  5,  and  its  perpendicular  altitude  by  D,  is 


and  consequently,  its  weight  becomes 

to  =  .2618  DS  (0*  +  ft  3  +  S2)  ;  (85). 

therefore,  by  comparing  this  equation  with  that  marked  (82),  we  get 

.  P  :  w  :  :  33*  :  (/32  +  /3  3  +  a2).  (86). 

Again,  if  we  compare  the  equations  marked  (73  and  (85)  with  one 
another,  we  shall  have 


p  :  w  ::  1.5708  (0  +  3)  dsv'D*  -*•%(&—  3)  :  .2618  Ds(/32  +  /33  +  J')  ; 
if  therefore,  we  expunge  the  common  quantities,  from  the  third  and 
fourth  terms  of  the  above  analogy,  and  in  the  third  term,  substitute  the 
value  of  d  as  it  is  expressed  in  the  equation  (83),  then  we  shall  obtain 

p  :  w  :  :  2(0  +  2  3)  <J  D*  -f  i  (ft  —  *>  :  (/32  +  ft  3  +  *). 
When  5  vanishes,  or  when  the  vessel  becomes  a  complete  cone  with 
its  vertex  downwards,  the  preceding  analogy  gives 

p  :  w  :  :  2  ^  D*  +  |/32  :  /3. 

In  complying  with  the  conditions  of  the  20th  problem,  the  fore- 
going investigation  has  been  conducted  on  the  supposition,  that  the 
vessel  in  question  is  in  the  form  of  the  frustum  of  a  cone  ;  but  the 
attentive  reader  will  readily  perceive,  that  the  same  mode  of  procedure 
will  apply  to  the  frustum  of  any  other  regular  pyramid,  and  the  result- 
ing formulae  will  partake  of  similar  forms  and  combinations,  differing 
only  in  so  far  as  depends  upon  the  constant  numbers  which  express 
their  respective  areas  and  solidities  ;  it  is  therefore  unnecessary  to 
pursue  the  inquiry  further,  taking  it  for  granted,  that  by  a  careful 
perusal  of  what  has  been  done  above,  no  difficulty  will  be  met  with 
in  applying  the  same  principles  to  any  other  case  of  form  or  condition 
that  is  likely  to  occur. 

COUOL.  1.  By  the  preceding  investigation,  then,  and  the  formulae 

arising  from  it,  we  learn,  that  by  causing  the  sides  of  a  vessel,  which 

is  filled  with  an  incompressible  and  non-elastic  fluid,  to  converge  or 

diverge  from  the  extremities  of  the  base,  supposed  to  be  horizontal  :  — 

The  pressure  on  the  base,  may  be  greater  or  less  than  the 

weight  of  the  fluid  which  the  vessel  contains,  in  any  propor- 

tion whatever. 


OF  FLUID  PRESSURE   USED  AS  A  MECHANICAL   POWER.  113 

2.  Upon  these  principles  therefore,  and  others  of  a  similar  nature, 
which  we  have  mentioned  at  the  outset,  is  explained  the  paradoxical 
property  of  non-elastic  fluids  : — 

That  the  pressure  on  the  bottom  of  a  vessel  Jilted  with 
fluid,  does  not  depend  upon  its  quantity,  but  solely  upon  the 
perpendicular  altitude  of  its  highest  particles  above  the 
bottom  of  the  vessel  or  the  surface  by  which  the  pressure  is 
sustained. 

3.  And  from  the  property  here  propounded,  is  deduced  the  remark- 
able and  important  principle  : — 

That  any  quantity  of  fluid  however  small,  may  be  made  to 
balance,  or  hold  in  equilibrio,  any  other  quantity,  however 
great. 

Let  the  upright  or  vertical  section  of  a  vessel  containing  an  incom- 
pressible and  non-elastic  fluid,  be  such  as  is  repre- 
sented by  A  BCD  in  the  annexed  diagram,  and  let 
cdbe  the  corresponding  section  of  a  small  pipe  or 
tube  inserted  into  its  upper  surface  at  the  point  d. 

Then,  supposing  the  vessel  and  the  tube  to  be 
filled  with  fluid  as  far  as  the  point  c  ;  it  is  manifest 
from  the  first  case  of  the  preceding  problem,  that 
the  pressure  upon  DC  the  bottom  of  the  vessel,  is  precisely  the  same 
as  if  it  were  entirely  filled  to  the  height  acb\  for  the  pressure  upon 
the  bottom  DC,  is  equal  to  the  weight  of  a  fluid  column,  the  diameter 
of  whose  base  is  DC  and  its  perpendicular  altitude  an  or  be;  but  this 
is  evidently  greater  than  the  weight  of  the  fluid  in  the  vessel,  and  by 
increasing  the  height  of  fluid  in  the  tube,  the  pressure  on  the  bottom 
will  be  increased  in  the  same  proportion,  while  the  actual  increase 
of  weight  is  very  small,  being  only  in  proportion  to  the  increase  of 
pressure,  as  the  area  of  a  section  of  the  tube  is  to  the  area  of  the 
bo  torn. 

PROBLEM  XXI. 

118.  Having  given  the  diameter  and  perpendicular  height  of 
a  cylindrical  vessel,  together  with  the  diameter  of  a  tube  fixed 
vertically  into  the  top  of  it  :— 

VOL.  I.  I 


114  OF  FLUID  PRESSURE  USED  AS  A  MECHANICAL  POWER. 

It  is  required  to  find  the  length  of  the  tube,  such,  that 
when  it  and  the  vessel  are  filled  with  an  incompressible  fluid, 
the  pressure  on  the  bottom  of  the  vessel  may  be  equal  to  any 
number  of  times  the  fluid's  weight. 

In  resolving  this  problem,  it  will  be  sufficient  to  refer  to  the  pre- 
ceding diagram,  because  a  separate  construction  would  exhibit  no 
variety  ;  for  this  purpose  then, 

Put  D  zz  DC,  the  diameter  of  the  base  of  the  cylindrical  vessel, 
A  zz  the  area  of  its  base, 

h  zz  A  D,  the  altitude,  or  perpendicular  depth  of  the  vessel, 
C  —  the  capacity,  or  solid  content, 
P  —  the  pressure  on  the  bottom, 

d  —  ed,  the  diameter  of  the  tube  inserted  in  the  top  of  the  vessel, 
a  zz  the  area  of  its  horizontal  section, 
c  zz  the  capacity,  or  solid  content  of  the  tube, 
w  ~~  the  weight  of  the  fluid  in  the  vessel,  and  w'  the  weight  of 

that  in  the  tube  ; 

s  zz  the  specific  gravity  of  the  fluid, 

n  zz  the  number  of  times  the  pressure  exceeds  the  weight, 
and  x  zz  the  required  length  of  the  tube. 

Then,  according  to  the  principles  of  mensuration,  the  area  of  the 
bottom  of  the  vessel  becomes 

AZZ.7854D2, 
and  that  of  a  horizontal  section  of  a  tube,  is 

azz.7854d2; 
and  again,  by  the  geometry  of  solids,  the  capacity  of  the  vessel  is 

Czz.7854D2A, 
and  for  the  capacity  of  the  tube,  we  have 


the  respective  weights  being 

wzz.7854D2As,  and  w1  zz  .7854d2a-s; 

consequently,  the  whole  weight  of  the  fluid  in  the  vessel  and  tube,  is 

w  +  w'  =  .7854s  (D2A,  -f  d*x). 

Now,  we  have  shown  above,  that  the  pressure  on  the  bottom  of  the 
vessel  is  equal  to  the  weight  of  a  fluid  cylinder,  whose  diameter  is 
DC  and  altitude  <ZD;  consequently,  the  pressure  on  the  bottom  is 
expressed  by 


OF  FLUID  PRESSURE  USED   AS  A  MECHANICAL  POWER.  115 

and  this,  according  to  the  conditions  of  the  problem,  is  equal  to  n 
times  the  entire  weight  ;  hence  we  have 

.7854  D2s  (h  +  x)  =  .7854  ns  (tfh  +  d*x), 
therefore,  by  casting  out  the  common  factors,  we  get 


or  by  separating  the  terms  and  transposing,  we  get 

(  D2  —  n  (T  )  x  =  D2  h  (  n  —  1  )  , 
from  which,  by  division,  we  obtain 
_D8A(n--l) 
1    D'^-nd-    '  (87). 

119.  It  may  be  perhaps  proper  to  illustrate  this  case  by  an  example; 
but  in  the  first  place,  it  becomes  necessary  to  give  the  rule  by  which 
the  operation  is  to  be  performed. 

RULE.  From  the  number  of  times  which  the  pressure  on  the 
bottom  of  the  vessel,  is  proposed  to  exceed  the  weight  of  the 
fluid,  subtract  unity  ;  multiply  the  remainder  by  the  square 
of  the  vessel  's  diameter,  drawn  into  its  depth  or  perpendicular 
altitude,  and  the  result  will  be  the  dividend. 

Then,  from  the  square  of  the  vessel's  diameter,  subtract 
n  times  the  square  of  the  diameter  of  the  tube,  and  divide  the 
above  dividend  by  the  remainder  for  the  length  of  the  tube 
required. 

120.  EXAMPLE.  If  the  perpendicular  height  of  a  cylindric  vessel  be 
18  inches,  its  diameter  5  inches;  the  diameter  of  a  tube  fixed  to  the 
top  of  the  vessel  one  inch  ;  and  if  the  vessel  and  tube  be  filled  with  an 
incompressible  and  non-  elastic  fluid,  till  the  pressure  on  the  bottom 
of  the  vessel  is  equal  to  twelve  times  the  entire  weight  of  the  fluid; 
what  is  the  length  of  the  tube  into  which  the  fluid  is  poured  ? 

Here,  by  proceeding  according  to  the  rule,  it  is 

n—  1=12—  1~11, 

D«A=z5X5x  18=450; 

therefore,  by  multiplication,  we  obtain 

«2/*  (n  —  1)  =  450X11  ==  4950  dividend. 

Again,  to  determine  the  divisor,  we  have 

D2  —  Bd»=z5X5  —  12=13; 

consequently,  by  division,  we  obtain 

^  =  49504-  13  =  380|f  inches. 

i  2 


116  OF  FLUID  PRESSURE  USED  AS  A  MECHANICAL  POWER. 

COROL.  1.  If  the  tube,  instead  of  being  fixed  perpendicularly  to  the 
top  of  the  vessel,  were  inserted  obliquely  into  any  part  of  its  sides  and 
inclined  upwards,  the  principle  above  exemplified  would  still  obtain ; 
and  the  pressure  in  the  narrow  tube  may  be  produced,  not  merely  by 
the  addition  of  a  little  fluid,  but  by  the  application  of  any  kind  of 
force,  such  as  the  working  of  a  piston  and  the  like. 

2.  If  the  bottom,  or  the  cover  of  the  cylindric  vessel  be  made  move- 
able,  the  pressure  on  either  may  be  brought  to  bear  on  any  one  point 
of  an  external  body,  and  may  then  produce  an  inconceivable  com- 
pression, as  is  very  successfully  done  in  the  Hydrostatic  Press,  an 
instrument,  which,  on  account  of  the  simplicity  of  its  application,  its 
expeditious  performance,  and  the  almost  unlimited  extent  of  its  power, 
is  altogether  without  a  parallel  in  the  annals  of  mechanical  invention, 
and  the  numerous  purposes  to  which  it  is  applied,  entitle  it  to  no 
small  share  of  popular  approbation. 

This  machine  is  not  only  used  for  pressing  bodies  together,  with  a 
view  of  diminishing  their  bulk,  in  order  to  render  them  the  more  easily 
stowable  ;  but  it  is  equally  applicable  to  the  operation  of  drawing  and 
lifting  great  loads,  and  overcoming  immense  resistances,  however 
opposed  to  its  action ;  even  piles,  which  have  been  driven  to  a  great 
depth  for  the  purpose  of  forming  coffer-dams,  can  be  drawn  by  it  with 
the  greatest  facility,  and  moreover,  trees  of  the  greatest  size  and  most 
tenacious  growth,  offer  but  a  feeble  resistance  to  its  energy ;  and  in 
addition,  iron  bolts  and  cables,  capable  of  holding  the  largest  ships  in 
the  British  navy,  are  totally  incompetent  to  resist  its  influence. 

An  instrument  possessing  such  immense  power  in  combination  with 
so  many  other  advantages,  such  as  cheapness  of  construction,  porta- 
bility, and  simplicity  of  application,  certainly  merits  the  greatest 
attention,  and  too  many  attempts  cannot  be  made  to  simplify  the 
theory,  and  render  its  operations  easily  understood ;  we  shall  there- 
fore, in  the  following  pages,  endeavour  to  unfold  the  principles,  and 
to  describe  its  construction  and  mode  of  operation. 


CHAPTER  VI. 

THE    THEORY     OF    CONSTRUCTION     AND    SCIENTIFIC    DESCRIPTION 
OF    SOME    HYDROSTATIC    ENGINES. 

1.    OF    THE    HYDROSTATIC    OR    BRAMAH    PRESS. 

PROPOSITION  II. 

121.  IF  there  be  any  number  of  pistons  of  different  magni- 
tudes, any  how  applied  to  apertures  in  a  cylindrical  vessel  filled 
with  an  incompressible  and  non-elastic  fluid  : — 

The  forces  acting  on  the  piston  to  maintain  an  equilibrium, 
will  be  to  one  another  as  the  areas  of  the  respective  apertures, 
or  the  squares  of  the  diameters  of  the  pistons. 

Let  ABC  D  represent  a  section  passing  along  the  axis  of  a  cylindrical 
vessel  filled  with  an  incompressible  and  non-elastic 
fluid,  and  let  E,F  be  two  pistons  of  different  magni- 
tudes, connected  with  the  cylinder  and  closely  fitted 
to  their  respective  apertures  or  orifices ;  the  piston 
F  being  applied  to  the  aperture  in  the  side  of  the 
vessel,  and  the  piston  E  occupying  an  entire  section 
of  the  cylinder  or  vessel,  by  which  the  fluid  is  contained. 

Then,  because  by  the  nature  of  fluidity,  the  pressures  on  every  part 
of  the  pistons  E  and  F,  are  mutually  transmitted  to  each  other  through 
the  medium  of  the  intervening  fluid  ;  it  follows,  that  these  pressures 
will  be  in  a  state  of  equilibrium  when  they  are  equal  among  them- 
selves. 

Now,  it  is  manifest,  that  the  sum  of  the  pressures  propagated  by 
the  piston  E,  is  proportional  to  the  area  of  a  transverse  section  of  the 
cylinder ;  and  in  like  manner,  the  sum  of  the  pressures  propagated  by 
the  piston  F,  is  proportional  to  the  area  of  the  aperture  which  it 
occupies;  consequently,  an  equilibrium  must  obtain  between  these 
pressures  : — 


118         THEORY  OF  CONSTRUCTION  AND  SCIENTIFIC  DESCRIPTION 

When  the  forces  on  the  pistons,  are  to  one  another,  respec- 
tively as  the  areas  of  the  apertures  or  spaces  which  they 
occupy. 

And  it  is  obvious,  that  the  same  thing  will  take  place,  whatever 
may  be  the  number  of  the  pistons  pressed. 

Hence  it  appears,  that  by  taking  the  areas  of  the  pistons  E  and  F, 
in  a  proper  ratio  to  one  another,  we  can,  by  means  of  an  incompres- 
sible fluid,  produce  an  enormous  compression,  and  that  too  by  the 
application  of  a  very  small  force. 

Put  P  zz  the  force  or  pressure  on  the  piston  E, 
A=z  the  area  of  the  orifice  which  it  occupies, 
p  zz  the  pressure  on  the  piston  F,  and 
a  zz  the  area  of  the  orifice  or  space  to  which  it  is  fitted. 

Then,  according  to  the  principle  announced  in  the  foregoing  pro- 
position and  demonstrated  above,  we  shall  obtain 

a  :  A  ::p  :  P. 

But  because,  by  the  principles  of  mensuration,  the  areas  of  different 
circles  are  to  one  another  as  the  squares  of  their  diameters ;  if  there- 
fore, we  substitute  d?  and  D2  respectively  for  a  and  A  in  the  above 
analogy,  we  shall  have 

d* :  D9  :  :  p  :  P, 

and  from  this,  by  making  the  product  of  the  mean  terms  equal  to  the 
product  of  the  extremes,  we  get 

p^  —  ^d\  (88). 

122.  This  is  the  principle  upon  which  depends  the  construction  and 
use  of  that  very  powerful  instrument,  the  Hydrostatic  Press,  first 
brought  into  notice  about  the  year  1796,  by  Joseph  Bramah,  Esq., 
of  Pimlico,  London ;  who  announced  it  to  the  world  as  the  discovery 
of  a  new  mechanical  power. 

In  this  however,  he  was  mistaken,  for  although  the  principle  upon 
which  it  depends  may  be  said  to  constitute  a  seventh  mechanical 
power,  yet  the  principle  announced  in  Proposition  II.  was  not  new 
to  philosophers  at  the  time  when  Mr.  Bramah  applied  it  to  the 
construction  of  his  presses,  it  having  long  been  familiarly  known 
under  the  designation  of  the  Hydrostatic  Paradox ;  and  besides, 
the  celebrated  Pascal  obscurely  hinted  at  its  application  to  mecha- 
nical purposes,  but  did  not  pursue  the  idea  far  enough  to  produce 
any  thing  useful,  or  to  entitle  him  to  the  full  merit  of  the  discovery. 

The  improvement  introduced  by  Mr.  Bramah,  consisted  in  the 
application  of  the  common  forcing  pump  to  the  injection  of  water, 


OF  THE  HYDROSTATIC  PRESS.  1 19 

or  some  other  incompressible  and  non-elastic  fluid,  into  a  strong 
metallic  cylinder,  truly  bored  and  furnished  with  a  moveable  piston, 
made  perfectly  water-tight  by  means  of  leather  collars  or  packing, 
neatly  fitted  into  the  cylinder. 

123.  The  proportion  which  subsists  between  the  diameter  of  this 
piston,  and  that  of  the  plunger  in  the  forcing  pump,  constitutes  the 
principal  element  by  which  the  power  of  the  instrument  is  calculated  ; 
for,  by  reason  of  the  equal  distribution  of  pressure  in  the  fluid,  it  is 
evident,  that  whatever  force  is  applied,  that  force  must  operate  alike 
on  the   piston   in  the  cylinder,  and  on  the  plunger  in  the  forcing 
pump,  and  consequently, 

In  proportion  as  the  area  of  the  transverse  section  of  the 
one,  exceeds  the  area  of  a  similar  section  of  the  other,  so  must 
the  pressure  sustained  by  the  one,  exceed  that  sustained  by  the 
other. 

Therefore,  if  the  piston  F  in  the  preceding  diagram,  be  assimilated 
to  the  plunger  in  the  barrel  of  a  forcing  pump,  and  the  piston  E  to 
that  in  the  cylinder  of  the  hydrostatic  press ;  then,  the  equation 
marked  (88),  notwithstanding  the  very  simple  and  concise  form  in 
which  it  appears,  involves  every  particular  respecting  the  power  and 
effects  of  the  engine,  of  which  a  detailed  description  with  illustrative 
drawings  will  be  given  a  little  further  on. 

This  being  premised,  we  shall  now  proceed  to  exhibit  the  use  and 
application  of  the  formula,  by  the  resolution  of  the  following  practical 
examples. 

124.  EXAMPLE  1.  If  the  diameter  of  the  cylinder  is  5  inches,  and 
that  of  the  forcing  pump  one  inch  ;  what  is  the  pressure  on  the  piston 
in  the  cylinder,  supposing  the  force  applied  on  the  plunger  or  smaller 
piston,  to  be  equivalent  to  750  Ibs.  ? 

Here  we  have  given  D  =r  5  inches ;  d  —  1  inch,  and  p  —  750  Ibs. ; 
therefore,  by  substitution,  equation  (88)  becomes 

5?X750  =  PX12;  that  is,  Pz=  18750  Ibs. 

Or  the  equation  for  the  value  of  P,  may  be  expressed  in  general 
terms,  as  follows. 

P_/>D! 

P-^r-  (89). 

And  from  the  equation  in  its  present  form,  we  deduce  the  following 
practical  rule. 


120          THEORY  OF  CONSTRUCTION  AND  SCIENTIFIC  DESCRIPTION 

RULE.  Multiply  the  square  of  the  diameter  of  the  cylinder 
by  the  magnitude  of  the  power  applied,  and  divide  the  product 
by  the  square  of  the  diameter  of  the  forcing  pump,  and  the 
quotient  will  express  the  intensity  of  the  pressure  on  the  piston 
of  the  cylinder. 

125.  EXAMPLE  2.  If  the  diameter  of  the  cylinder  is  5  inches,  and 
that  of  the  forcing  pump  one  inch  ;  what  is  the  magnitude  of  the 
power  applied,  supposing  the  entire  pressure  on  the  piston  of  the 
cylinder  to  be  18750  Ibs.  ? 

Here  we  have  given  D  =  5  inches;  d=  1  inch,  and  P=  18750 
Ibs.  ;  therefore,  by  substitution,  equation  (88)  becomes 

5a  Xp—  18750  X  1*;  or  p  —  750  Ibs. 

If  both  sides  of  the  fundamental  equation  (88)  be  divided  by  D*,  the 
general  expression  for  the  value  of  p,  is 

_Pd2 

p~       D2  '  (90). 

And   the    practical    rule   which   this    equation   supplies,   may  be 
expressed  in  words  at  length  in  the  following  manner. 

RULE.  Multiply  the  given  pressure  on  the  piston  of  the 
cylinder,  by  the  square  of  the  diameter  of  the  forcing  pump, 
and  divide  the  product  by  the  square  of  the  diameter  of  the 
cylinder  for  the  power  required. 

126.  EXAMPLE  3.  The  diameter  of  the  forcing  pump  is  one  inch, 
and  the  power  with  which  the  plunger  descends  is  equivalent  to  750 
Ibs.  ;  what  must  be  the  diameter  of  the  cylinder,  to  admit  a  pressure 
of  18750  Ibs.  on  the  piston  ? 

Here  we  have  given  c?n=  1  inch;  ^  =  750  Ibs.,  and  P  n=  18750  Ibs.  ; 
consequently,  by  substitution,  the  equation  marked  (88)  becomes 

750  D2=l  8750  x  I2; 
hence,  by  division,  we  obtain 


consequently,  by  evolution,  we  have 
D  —  ^  25  —  5  inches. 

If  both  sides  of  the  equation  (88)  be  divided  by  p,  and  the  square 
root  of  the  quotient  extracted,  the  general  expression  for  the  diameter 
of  the  piston,  is 


OF  THE  HYDROSTATIC  PRESS.  121 


And  the  practical  rule  for  the  determination  of  D,  may  be  expressed 
in  words  as  follows. 

RULE.  Multiply  the  pressure  on  the  piston  of  the  cylinder, 
by  the  square  of  the  diameter  of  the  forcing  pump,  and  divide 
the  product  by  the  force  with  which  the  plunger  descends  ; 
then,  the  square  root  of  the  quotient  will  be  the  diameter  of 
the  cylinder  sought. 

127.  EXAMPLE  4.  The  diameter  of  the  cylinder  is  5  inches,  and 
the  force  with  which  the  plunger  descends,  is  equivalent  to  750  Ibs.  ; 
what  must  be  the  diameter  of  the  forcing  pump,  in  order  to  transmit 
a  pressure  of  18750  Ibs.  to  the  piston  of  the  cylinder  ? 

Here  we  have  given  D  r=  5  inches  ;  p  —  750  Ibs.,  and  P  zz  18750 
Ibs.  ;  consequently,  by  substitution,  equation  (88)  becomes 

1  8750  d*  =  750  X  52, 
and  by  division,  we  shall  have 
750  X  25 


therefore,  by  extracting  the  square  root,  we  get 

d—  /f  zzl  inch. 

If  both  sides  of  the  original  equation  marked  (88),  be  divided  by 
P,  and  the  square  root  extracted,  the  entire  pressure  on  the  piston, 
the  general  expression  for  the  value  of  d  becomes 


=  /l/   "P"'  (92). 

And    the   practical   rule   which   this   equation    supplies,    may   be 
expressed  in  words  in  the  following  manner. 

RULE.  Multiply  the  force  with  which  the  plunger  descends, 
by  the  square  of  the  diameter  of  the  cylinder,  and  divide  the 
product  by  the  entire  pressure  on  the  piston;  then,  extract 
the  square  root  of  the  quotient  for  the  diameter  of  the  forcing 
pump. 

128.  The  foregoing  is  the  theory  of  the  Hydrostatic  Press,  as 
restricted  to  the  consideration  of  the  diameters  of  the  cylinder  and 
forcing  pump,  and  the  respective  pressures  on  the  piston  and  plunger; 
but  since  the  instrument  is  generally  furnished  with  an  indicator  or 


122          THEORY  OF  CONSTRUCTION  AND  SCIENTIFIC   DESCRIPTION 

safety  valve  for  measuring  the  intensity  of  pressure,  the  theory  would 
be  incomplete  without  considering  it  in  connection  with  the  diameters 
of  the  pump  and  cylinder.  For  which  purpose 

Put  3  zz  the  diameter  of  the   safety  valve,  expressed  in  inches  or 

parts, 
and  w —  the  weight  thereon,  or  the  force  that  prevents  its  rising. 

Then,  according  to  the  principle  announced  in  Proposition  II.,  we 
obtain  the  following  analogies,  viz. 

D2  :  ^  :  :  P  :  w, 
d*  :  ^  :  :  p  :  w ; 

and  from  these  analogies,  by  making  the  products  of  the  extreme 
terms  equal  to  the  products  of  the  means,  we  get 

D'wzz^P,  (93). 

zudd*w  =  tfp.  (94). 

Now,  in  order  to  pursue  the  expansion  of  these  equations,  we  shall 
suppose  the  value  of  &  to  be  one  fourth  of  an  inch,  while  the  numerical 
values  of  the  other  letters  remain  the  same  as  supposed  for  the  several 
examples  under  equation  (88) ;  then,  to  determine  the  corresponding 
value  of  w,  or  the  power  which  prevents  the  safety  valve  from  rising, 
when  all  the  parts  of  the  instrument,  or  the  several  powers  and  pres- 
sures are  in  a  state  of  equilibrium,  we  have  the  following  examples  to 
resolve  according  to  the  proposed  conditions. 

129.  EXAMPLE  5.  The  diameter  of  the  cylinder  is  5  inches,  that  of 
the  indicator  or  safety  valve  J  of  an  inch,  and  the  entire  pressure 
upon  the  piston  of  the  cylinder  18750  Ibs. ;  what  is  the  corresponding 
force  preventing  the  ascent  of  the  safety  valve,  on  the  supposition  of  a 
perfect  equilibrium  ? 

Here  we  have  given  D  zz  5  inches ;  3  zz  J  of  an  inch,  and  P  zz  1 8750 
Ibs. ;  consequently,  by  substitution,  the  equation  (93)  becomes 

52wzz.252  X  18750; 
from  which,  by  division,  we  get 

.0625  X  18750 

wzz — zz46.875  Ibs. 

AQ 

But  the  general  expression  for  the  value  of  iv,  as  derived  from  the 
equation  (93),  becomes 

_J2P 
-  V'  (95). 

From  which  we  derive  the  following  rule. 


OF  THE  HYDROSTATIC  PRESS.  123 

RULE.  Multiply  the  entire  pressure  on  the  piston  of  the 
cylinder,  by  the  square  of  the  diameter  of  the  indicator  or 
safety  valve,  and  divide  the  product  by  the  square  of  the 
diameter  of  the  cylinder  for  the  weight  required. 

130.  EXAMPLE  6.  The  diameter  of  the  safety  valve  is  J  of  an  inch, 
that  of  the  cylinder  5  inches,  and  the  weight  on  the  safety  valve 
46.875  Ibs. ;  what  is  the  corresponding  pressure  on  the  piston  of  the 
cylinder  ? 

Here  we  have  given  3  =  £  of  an  inch ;  D  zz:  5  inches,  and 
w  zz:46.875  Ibs  ;  therefore,  by  substitution,  equation  (93)  becomes 

.252P  =  52x  46.875, 
and  by  division,  we  obtain 


The  general  expression  for  the  value  of  P,  as  derived  from  the 
equation  marked  (93),  becomes 

* 
- 


(96). 

And  the  practical  rule  supplied  by  this  equation,  may  be  expressed 
in  words  as  follows. 

RULE.  Multiply  the  weight  on  the  safety  valve,  by  the 
square  of  the  diameter  of  the  cylinder,  and  divide  the  product 
by  the  square  of  the  diameter  of  the  safety  valve,  and  the 
quotient  will  give  the  entire  pressure  on  the  piston  of  the 
cylinder. 

131.  EXAMPLE  7.  The  diameter  of  the  cylinder  is  5  inches,  the 
entire  pressure  of  the  piston  is  18750  Ibs.,  and  the  weight  on  the 
safety  valve  is  46.875  Ibs.  ;  what  is  its  diameter  ? 

Here  we  have  given  ozz:5  inches  ;  P  zz:  18750  Ibs.,  and  wzr46.875 
Ibs.  ;  therefore,  by  substitution,  equation  (93)  becomes 

18750a2z=52X  46.875, 
and  from  this,  by  division,  we  get 


and  by  extracting  the  square  root,  we  obtain 

3=  -V/  .0625  zz:  .25,  or  £  of  an  inch. 

The  general  expression  for  the  value  of  3,  as  derived  from  the 
equation  (93),  is  as  follows,  viz. 


124          THEORY  OF  CONSTRUCTION  AND  SCIENTIFIC  DESCRIPTION 

*=y  -p--  (97). 

And  the  practical  rule  which  this  equation  affords,  may  be  expressed 
in  words  in  the  following  manner. 

RULE.  Multiply  the  load  on  the  safety  valve  by  the  square 
of  the  diameter  of  the  cylinder ;  divide  the  product  by  the 
entire  pressure  on  the  piston,  and  the  square  root  of  the 
quotient  will  give  the  diameter  of  the  safety  valve  required. 

132.  EXAMPLE  8.  The  diameter  of  the  safety  valve  is  ^  of  an  inch, 
the  load  upon  it  46.875  Ibs.,  and  the  entire  pressure  on  the  piston 
of  the  cylinder  is  18750  Ibs. ;  what  is  its  diameter? 

Here  we  have  given  £=  J  of  an  inch,  w= 46.875  Ibs.,  and  Pzr  18750 
Ibs. ;  consequently,  by  substitution,  we  have 

46.875  D*=. 25*  X  18750, 
from  which,  by  division,  we  shall  obtain 
.25aX  18750 


46.875 
and  finally,  by  extracting  the  square  root,  we  get 

D  =  -v/  25  —  5  inches. 

If  both  sides  of  the  equation  marked  (93),  be  divided  by  w  the 
weight  on  the  safety  valve,  we  get 


and  by  extracting  the  square  root,  the  general  expression  for  the  value 
of  D  the  diameter  of  the  cylinder,  becomes 


-y  ^'  (98). 

And  from  this  equation  we  derive  the  following  rule. 

RULE.  Multiply  the  entire  pressure  on  the  piston  of  the 
cylinder  by  the  square  of  the  diameter  of  the  safety  valve, 
divide  the  product  by  the  weight  upon  the  safety  valve,  and 
extract  the  square  root  of  the  quotient  for  the  diameter  of  the 
cylinder  sought. 

133.  EXAMPLE  9.  The  diameter  of  the  forcing  pump  is  one  inch, 
that  of  the  safety  valve  is  one  fourth  of  an  inch,  and  the  power  or 
force  with  which  the  plunger  descends,  is  equivalent  to  750  Ibs. ;  what 
is  the  corresponding  weight  on  the  safety  valve  ? 


OF  THE  HYDROSTATIC  PRESS.  125 

Here  we  have  given  d—  1  inch;  Ji=^  of  an  inch,  and  p  —  750 
Ibs.  ;  consequently,  by  substitution,  the  equation  (94)  becomes 

P  X  w  =  .25*  X  750  ;  that  is,  w  =  46.875  Ibs,,  the 
very  same  value  as  we  derived  from  the  fifth  example. 

If  both  sides  of  the  equation  marked  (94)  be  divided  by  d*,  the 
general  expression  for  the  value  of  w  becomes 

_3> 

~d2*  (99). 

And  the  practical  rule  supplied  by  this  equation,  may  be  expressed 
in  words  at  length  in  the  following  manner. 

RULE.  Multiply  the  force  with  which  the  plunger  descends 
by  the  square  of  the  diameter  of  the  safety  valve,  and  divide 
the  product  by  the  square  of  the  diameter  of  the  plunger  ; 
then  the  quotient  will  express  the  load  upon  the  safety  valve. 

134.  EXAMPLE  10.  The  diameter  of  the  safety  valve  is  J  of  an 
inch,  that  of  the  forcing  pump  is  one  inch,  and  the  load  upon  the 
safety  valve  is  46.875  Ibs.  ;  what  is  the  power  applied,  or  the  force 
with  which  the  plunger  in  the  forcing  pump  descends  ? 

Here  we  have  given  3=J  of  an  inch,  d=:l  inch,  and  w=r46.875 
Ibs.  ;  consequently,  by  substitution,  equation  (94)  becomes 

.252j9=46.875  X  I2, 
and  from  this,  by  division,  we  obtain 

'  ' 


The  general  expression  for  the  value  of  p,  as  obtained  from  the 
equation  marked  (94),  becomes 

_d*w 

:~F*  (100). 

from  which  we  derive  the  following  rule. 

RULE.  Multiply  the  load  on  the  safety  valve  by  the  square 
of  the  diameter  of  the  forcing  pump  ;  then,  divide  the  product 
by  the  square  of  the  diameter  of  the  safety  valve,  and  the 
quotient  will  give  the  force  with  which  the  piston  descends. 

135.  EXAMPLE  11.  The  diameter  of  the  plunger  or  the  piston  of 
the  forcing  pump  is  one  inch,  the  force  with  which  it  descends  is 
equivalent  to  750  Ibs.,  and  the  load  on  the  safety  valve  is  46.875  Ibs.; 
what  is  its  diameter  ? 


126  THEORY  OF  CONSTRUCTION  AND  SCIENTIFIC   DESCRIPTION 

Here  we  have  given  dnz  1  inch,  p  •=.  750  Ibs.,  and  w  zz  46.875  Ibs.  ; 
consequently,  by  substitution,  we  have 

750  3"=  1s  X  46.875, 
and  from  this,  by  division,  we  obtain 


and  finally,  by  evolution,  we  have 

a  =  V.0625  =  .25  of  an  inch.          , 

Let  both  sides  of  the  equation  marked  (94)  be  divided  by  p,  the 
power  or  force  with  which  the  piston  of  the  forcing  pump  descends, 
and  we  shall  have 


and  by  extracting  the  square  root,  we  get 

(101). 
Hence,  the  following  practical  rule. 

RULE.  Multiply  the  weight  or  load  upon  the  safety  valve, 
by  the  square  of  the  diameter  of  the  forcing  pump,  and  divide 
the  product  by  the  force  with  which  the  plunger  or  piston 
of  the  forcing  pump  descends ;  then,  the  square  root  of  the 
quotient  will  be  the  diameter  of  the  safety  valve. 

136.  EXAMPLE  12.  The  diameter  of  the  safety  valve  is  one  fourth 
of  an  inch,  the  weight  upon  it  is  46.875  Ibs.,  and  the  power  applied, 
or  the  force  with  which  the  plunger  descends,  is  750  Ibs ;  what  is  the 
diameter  of  the  forcing  pump  ? 

Here  we  have  given  £  —  J  of  an  inch,  w>  — 46.875  Ibs.,  and  /?=z750 
Ibs. ;  consequently,  by  substitution,  the  equation  marked  (94)  becomes 

46.875d2=.25*x750; 

therefore,  by  division,  we  obtain 

.25^X750 

:    46.875    Z 

and  finally,  by  extracting  the  square  root,  we  get 

<frz  1  inch. 

The  general  expression  for  the  value  of  the  diameter  of  the  forcing 
pump,  as  derived  from  the  equation  (94),  is 


d=^  (102). 


OF  THE  HYDROSTATIC  PRESS.  127 

And  from  this,  we  obtain  the  following  practical  rule. 

RULE.  Multiply  the  force  with  which  the  piston  of  the 
forcing  pump  descends,  by  the  square  of  the  diameter  of  the 
safety  valve ;  divide  the  product  by  the  load  on  the  safety 
valve,  and  extract  the  square  root  of  the  quotient  for  the 
diameter  of  the  forcing  pump. 

The  foregoing  twelve  examples  exhibit  all  the  varieties  of  cases  that 
can  arise,  from  the  combination  of  the  six  data  which  we  have  em- 
ployed in  our  theory,  viz.  the  diameters  of  the  cylinder,  the  forcing 
pump  and  the  safety  valve ;  together  with  the  entire  pressure  on  the 
piston  of  the  cylinder,  the  power  applied  to  the  plunger  of  the  forcing 
pump,  and  the  weight  upon  the  safety  valve. 

We  have  determined  each  of  the  quantities,  composing  the  several 
fundamental  equations,  in  terms  of  the  others,  and  have  drawn  up 
rules  from  the  general  expressions,  merely  for  the  assistance  of  those 
who  are  not  accustomed  to  algebraic  reductions;  those  who  are, 
will  prefer  finding  each  quantity  directly  from  the  general  equation 
expressing  its  value. 

137.  It  is  manifest  from  the  principles  of  mensuration,  that  the  area 
of  a  transverse  section  of  the  cylinder,  or  the  base  of  the  piston,  is 
expressed  by  .7854  D2 ;  and  we  have  shown,  equations  (89)  and  (96), 
that  the  entire  pressure  upon  the  base  of  the  piston  in  the  case  of 
equilibrium,  is 


consequently,  if  n  denotes  the  pressure  in  pounds  avoirdupois  on  one 
square  inch  of  the  piston,  then  we  have 

P  p  w 

n=7imtf'r'    ~-7JS54d*al         -T7854F-          (104). 

Now,  from  principles  investigated  by  Professor  Barlow,  of  the  Royal 
Military  Academy  at  Woolwich,  it  appears,  that  if  c  denote  the  cohe- 
sive force  of  the  material  employed  in  the  construction  of  the  cylinder, 
t  its  thickness,  and  r  the  interior  radius  ;  then,  in  order  that  the  strain 
produced  by  the  pressure,  shall  not  exceed  the  elastic  power  of  the 
material ;  it  is  necessary  that 

ct 

In  order  to  demonstrate  this,  let  ABD  be  a  transverse  section  of  the 
cylinder,  perpendicular  to  the  axis  passing  through  c ;  then,  sup- 


128         THEORY  OF  CONSTRUCTION  AND  SCIENTIFIC  DESCRIPTION 

posing  a  certain  uniform  pressure  to  be  exerted  all  round  the  interior 
boundary;  it  will  readily  appear,  from  the 
theory  of  resistance,  that  each  successive 
circular  lamina,  estimated  from  the  interior 
towards  the  exterior  circumference,  offers  a 
less  and  less  resistance  to  the  straining  force. 
But  it  is  obvious  from  the  very  nature  of 
the  subject,  that  by  reason  of  the  internal 
pressure  or  strain,  the  metal  must  undergo  a 
certain  degree  of  extension,  and  since  the  resistance  of  the  outer 
boundary  is  less  than  that  of  the  inner  one,  it  follows,  that  the  exten- 
sion must  also  be  less ;  this  is  manifest,  for  the  resistance  which  any 
body  offers  to  the  force  by  which  it  is  strained,  is  proportional  to  the 
extension  which  it  undergoes,  divided  by  its  length ;  now,  since  the 
resistances  of  the  several  laminee,  decrease  as  they  recede  from  the 
interior  boundary  towards  the  exterior,  while  at  the  same  time,  the 
corresponding  circumferences  increase;  it  is  manifest,  that  the  exten- 
sion for  the  several  laminse  decreases  to  the  last  or  exterior  boundary, 
where  it  is  the  least  of  all : — It  is  therefore  the  law  of  the  decreasing 
resistance,  that  the  present  enquiry  is  instituted  to  determine. 

Put  d  ~  ab,  the  interior  diameter  of  the  cylinder  before  the  pressure 

is  applied, 

e  m  the  increase  of  d  occasioned  by  the  pressure, 
d'—  A B,  the  exterior  diameter  in  its  original  state, 
e  —  the  increase  induced  by  pressure. 

Then  (d  -\-  e)  and  d'  -f  e'),  are  respectively  the  interior  and  exterior 
diameters  of  the  cylinder  as  affected  by  extension. 

By  the  principles  of  mensuration,  the  area  of  the  annulus,  or  cir- 
cular ring  contained  between  the  interior  and  exterior  boundaries  : — 

Is  equal  to  the  difference  of  the  squares  of  the  diameters, 
drawn  into  the  constant  fraction  0.7854  ;  or  it  is  proportional 
to  the  sum  of  the  diameters,  drawn  into  their  difference. 

But  according  to  the  nature  of  the  present  enquiry,  the  area  of  the 
ring  is  the  same,  both  before  and  after  the  extension  takes  place  ; 
consequently,  we  have 

(d'  +  c')f  — (<*  +  *)*  =  <**  —  d*; 

therefore,  by  expanding  the  terms  on  the  left  hand  side,  we  get 
e'*  —  d"'  —  2de  —  e?  —  d3  —  d* ; 


OF  THE  HYDROSTATIC  PRESS.  129 

or  by  transposing  and  expunging  the  common  terms,  it  is 


and  this  equation  being  converted  into  an  analogy,  gives 
2^4-e'  :  2d-\-e  :  :  e  :  e'. 

Now,  the  quantity  of  extension  that  the  material  will  allow  before 
rupture  being  very  small,  especially  as  compared  with  the  quantities 
2d'  and  2d  ;  it  therefore  follows,  that  the  quantities  e'  and  e,  in  the 
first  and  second  terms,  may  be  conceived  to  vanish,  and  the  above 
analogy  becomes 

d'  :  d  :  :  e  :  e'. 

From  this  it  appears,  that  the  extensions  of  the  respective  circum- 
ferences, are  inversely  as  the  corresponding  diameters  ;  but  we  have 
stated  above,  that  the  resistance  is  as  the  extension  divided  by  the 
length;  therefore,  we  have 

d     d' 

d'''d' 

or  which  amounts  to  the  same  thing, 

d2  :  d'1  ; 

hence  this  law,  that  the  magnitude  of  the  resistance  offered  by  each 
successive  circular  lamina  :  — 

Is  inversely  as  the  square  of  its  diameter,  or,  which  is  the 
same  thing,  inversely  as  the  square  of  its  distance  from  the 
common  centre  to  which  they  are  referred. 

From  the  general  law  thus  established,  the  actual  resistance  due  to 
any  point  in  the  annulus,  or  to  any  thickness  of  metal,  can  very  easily 
be  ascertained. 

Put  r  iz:  C«,  the  interior  radius  of  the  cylinder,  of  which  the  annexed 

diagram  is  a  section, 
£r±«A,    the    entire    thickness    of  the 

metal, 
#:zi  an,    any    variable    thickness    esti-      A 

mated    from    a,    the    interior 

surface, 
n  —  the  pressure   on   a  square  inch   of 

the  inner  surface  in  pounds  avoirdupois, 
/m  the  measure  of  the  straining  force,  or  the  resistance  sustained 

by  the  first  or  interior  lamina,  and 
c  =i  the  cohesive  force  of  the  material. 
VOL.  i.  K 


130         THEORY  OF  CONSTRUCTION  AND  SCIENTIFIC  DESCRIPTION 

Then,  agreeably  to   the   law   of  the   resistances  which  we  have 
established  above,  we  have 

::^:  (r  +  x)2' 

this  result  expresses  the  strain  at  the  point  x,  or  the  resistance  of  the 
material  whose  thickness  is  an ;  and  the  fluxion  of  this  quantity  as 
referred  to  the  variable  thickness  x,  is 


consequently,  the  fluent,  or  the  sum  of  all  the  strains,  is 

/fr*x 
—  -  -  4-  C,  and  this  when  x  —  t  becomes 
(r  +  xf 


Therefore,  if  the  strain  or  resistance  /,  were  to  act  uniformly  on  the 
thickness  expressed  by  ^—  ,  it  would  produce  the  same  effect,  as  if 

all  the  variable  strains  were  to  act  on  the  whole  thickness  t. 

The  above  law  being  admitted,  let  us  suppose  that  the  interior 
radius  of  the  cylinder,  and  the  pressure  per  square  inch  on  the  surface 
are  given,  and  let  it  be  required  to  determine  the  thickness  such,  that 
the  strain  and  resistance  may  be  in  equilibrio. 

Here  it  is  manifest,  that  the  greatest  strain  the  thickness   —  — 

CT  t 

can  resist,  is  —  -  ,  and  the  strain  to  which  it  is  actually  exposed,  is 
nr;  consequently,  when  these  are  equal,  we  have 

crt 

***%? 

from  which,  by  expunging  the  common  factor  r,  we  get 

ct 

-7+?  (105). 

If  this  value  of  n  be  compared  with  its  respective  values,  as  indi- 
cated in  the  equations  (104)  preceding,  we  shall  have  the  following 
expressions,  for  the  thickness  of  metal  in  the  cylinder  to  resist  any 
pressure,  while  the  elastic  power  of  the  material  remains  perfect,  viz. 

Pr  _  pr  .    _  wr 

'* 


—  .7854CD9  —  P  .7854cd8  —    '  .78540?  —  w 


OF  THE  HYDROSTATIC  PRESS.  131 

Therefore,  if  for  c  in  each  of  the  preceding  expressions,  we  substitute 
its  value  as  determined  by  experiment,  and  which  for  cast  iron,  accord- 
ing to  Dr.  Robison,  is  16648  pounds  avoirdupois  upon  a  square  inch  ; 
then  we  shall  have 

,_  Pr  * 

- 13076  Da  —  P'  (106). 


A107)- 


-130763s  —  w  (108). 

The  following  example  will  illustrate  the  use  of  these  equations,  the 
value  of  t  the  thickness  of  the  metal  coming  out  the  same  by  each. 

138.  EXAMPLE  13.  What  must  be  the  thickness  of  metal  in  the 
cylinder  of  a  Hydrostatic  Press,  to  resist  a  pressure  of  30000  Ibs. ; 
the  diameter  of  the  cylinder  being  5  inches,  that  of  the  forcing  pump 
one  inch,  and  of  the  safety  valve  one  fourth  of  an  inch ;  being  the 
same  dimensions  which  we  have  employed  in  the  preceding  examples? 

Here  we  have  given  P  ±=  30000  Ibs. ;  D  =  5  inches  ;  and  conse- 
quently, r— 2J  inches;  therefore,  by  substitution,  equation  (106) 
gives 

30000 


n  .    , 

'=  13076  X  5*-  30000  =  '^  "^  bem* 
thing  more  than  one  fourth  of  an  inch. 

In  order  that  the  entire  pressure  on  the  piston  of  the  cylinder,  may 
be  equal  to  30000  Ibs.  according  to  the  conditions  of  the  question  ; 
the  force  with  which  the  plunger  of  the  forcing  pump  descends,  must 
be  equal  to  1200  Ibs.  ;  therefore,  by  equation  (107),  we  have 


before. 

Again,  in  order  that  the  entire  pressure  may  be  equal  to  30000  Ibs. 
the  weight  upon  the  safety  valve  must  be  75  Ibs.  ;  hence,  from  equa- 
tion (108),  we  obtain 


the  two  cases  foregoing. 

139.  It  may  not  be  improper  here  to  remark,  that  although  the 

requisite  thickness  of  metal  is  alike  assignable  from   either  of  the 

above  equations,  when  the  respective   pressure  and  diameters  are 

*  Where  the  constant  number  13076  =  16648  X  .7854. 
K2 


132        THEORY  OF  CONSTRUCTION  AND  SCIENTIFIC   DESCRIPTION 

known;  yet  it  is  the  first  of  the  class  only,  or  that  marked  (106), 
which  becomes  available  in  practice,  and  for  this  reason,  that  the 
power  of  the  press,  or  the  aggregate  pressure  which  it  is  capable  of 
exciting,  is  known  d  priori,  or  immediately  assignable  from  the  con- 
ditions of  construction,  while  the  load  upon  the  safety  valve,  and  the 
force  with  which  the  plunger  descends,  have  each  to  be  determined 
by  calculations  founded  on  circumstances  connected  with  the  aggre- 
gate or  ultimate  pressure. 

140.  Referring  to  equation  (105),  which  has  been  purposely  inves- 
tigated, for  expressing  the  intensity  of  pressure  on  a  square  inch  of 
surface,  and  multiplying  both  sides  by  r  -\-  t  the  denominator  of  the 
fraction,  we  shall  have 

nr  -f-  nt~ct, 

from  which,  by  transposing  and  collecting  the  terms,  we  get 

(c  —  n)  t  —  n  r ; 

then  by  division,  the  value  of  #,  or  the  thickness  of  metal  in  the 
cylinder  to  withstand  the  pressure,  becomes 

.,nr 
-^T*  (109). 

From  which  it  appears,  that  if  a  constant  value  adapted  to  practical 
purposes,  can  be  assigned  to  n,  the  rule  for"  calculating  the  thickness 
of  metal  in  the  cylinder  will  become  exceedingly  simple. 

Now,  it  has  been  remarked  by  several  eminent  practical  engineers, 
as  well  as  by  the  most  approved  and  intelligent  manufacturers,  that 
the  extreme  pressure  on  a  square  inch  of  the  piston,*  should  never 
exceed  half  the  cohesive  power  of  the  material ;  but  according  to  Dr. 
Robison,  the  cohesive  power  of  cast  iron  of  a  medium  quality  is  equal 
to  16648  Ibs. ;  hence  we  have 


therefore,  if  8324  Ibs.  be  adopted  as  the  limit  of  pressure  upon  a 
square  inch  of  surface,  the  foregoing  value  of  t  becomes 
-         8324 r 
"16648  —  8324" 

consequently,  in  order  that  the  strain  produced  by  the  pressure  may 
not  exceed  the  elastic  power  of  the  material ; — 

*  There  is  no  occasion  to  limit  the  pressure  to  the  piston  only,  since  every  square 
inch  of  surface  in  contact  with  the  fluid  sustains  the  same  pressure.  This  limitation 
has  frequently  caused  a  misapprehension  respecting  the  mode  of  ascertaining  the 
pressure  on  an  inch  of  surface. 


OF  THE  HYDROSTATIC  PRESS.  133 

The   thickness  of  metal  ought  never  to  be  less  than  the 
interior  radius  of  the  cylinder. 

By  the  first  equation  of  class  (104),  it  has  been  shown,  that  the 
pressure  on  a  square  inch  of  the  piston  in  Ibs.  avoirdupois,  is 

P 

~.7854D8' 
or  by  substituting  the  foregoing  value  of  n,  it  is 


from  which,  by  multiplication,  we  obtain 

8324  X  .7854D2rz:P; 

but  in  order  to  express  the  pressure  in  tons,  it  is 
6537.6696D* 

2240  (110). 

141.  Therefore,  when  the  diameter  of  the  cylinder  is  given,  the 
entire  pressure  in  tons  is  determined  by  the  following  very  simple 

rule. 

RULE.  Multiply  the  square  of  the  diameter  in  inches,  by 

the  constant  number  2.9186,   and  the  product  will  be    the 
pressure  in  tons. 

And  again,  when  the  pressure  in  tons  is  given,  the  diameter  of  the 
cylinder  may  be  determined  by  reversing  the  process,  or  by  the  fol- 
lowing rule. 

RULE.  Divide  the  given  pressure  in  tons  by  the  constant 
number  2.9186,  and  extract  the  square  root  of  the  quotient, 
for  the  diameter  of  the  cylinder  in  inches. 

142.  The  preceding  theory,  as  we  have  developed  it,  unfolds  every 
particular  connected  with   the  Hydrostatic  Press,  and   by  paying 
proper  attention  to  the  equations,  rules,  and  examples,  as  we  have 
delivered  them,  every  difficulty  attending  the   construction   of  the 
instrument  will  be  removed  ;  to  practical  persons,  however,  that  part 
of  the   theory  exhibited  in   the   equations  marked  (109)  and  (110) 
will  be  found  the  most  valuable,  as  they  do  the  more  immediately 
contain  the  particulars  which  direct  their  operations.     The  following 
examples  will  prove  the  truth  of  these  remarks. 

EXAMPLE  14.  The  diameter  of  the  cylinder  in  a  Hydrostatic  Press, 
is  10  inches  ;  what  is  its  power,  or  what  pressure  does  it  transmit? 
Here  by  the  first  rule  above,  we  have 
PzzlO2  X  2.9186zz291.  86  tons. 


134         THEORY  OF  CONSTRUCTION   AND  SCIENTIFIC  DESCRIPTION 

EXAMPLE  15.  What  is  the  diameter,  and  what  the  thickness  of 
metal,  in  a  press  of  300  tons  power  ? 

By  the  second  rule  above,  we  have 
D2  =  300  ~  2.9186  —  102.81  nearly ; 
therefore,  by  extracting  the  square  root,  we  obtain 


D=  V  102.81  =  10.13  inches; 

consequently,  according  to  the  remark  under  the  equation  (109),  the 
thickness  of  metal  is 

t  =  10.13  4-  2  =  5.065  inches. 

143.  The  rules  by  which  the  preceding  examples  have  been  resolved, 
are  very  nearly,  but  not  precisely  the  same  as  those  employed  by 
Messrs.  Bramah  in  the  construction  of  their  excellent  presses ;  the  only 
difference,  however,  consists  in  their  assuming  a  higher  number  as 
the  limit  of  pressure,  the  standard  which  they  employ  being  8556  Ibs. 
upon  a  square  inch  of  the  piston,  thereby  indicating,  that  they  reckon 
on  a  higher  cohesive  power  in  the  material,  than  that  which  we  have 
adopted  as  the  basis  of  our  theory. 

Now,  8556  Ibs.  on  a  square  inch,  is  equivalent  to  6619.8824  Ibs. 
upon  a  circular  inch ;  whereas  the  constant  which  we  have  chosen  is 
only  6537.6696  Ibs.,  being  a  difference  of  82.2128  Ibs.  upon  the 
circular  inch,  a  difference  that  need  not  be  regarded  in  practice,  as 
the  error  will  always  fall  on  the  side  of  safety,  giving  a  smaller  power 
to  the  press  than  what  it  really  possesses. 

144.  It  sometimes,  indeed  it  very  frequently  happens,  that  presses 
are  constructed,  without  any  attention  being  paid,  to  the  relation  which 
subsists  between  the  strength  of  the  parts,  and  the  strain  which  they 
have  to  resist ;  in  all  such  cases,  therefore,  it  may  be  interesting  to 
possess  a  rule,  by  which  the  merits  or  demerits  of  a  press  so  con- 
structed can  be  ascertained,  for  in  this  way  a  failure  in  the  instrument 
may  be  prevented,  and  a  remedy  applied  to  any  defect  that  may 
exist. 

Now,  according  to  the  first  equation  of  class  (104),  the  pressure 
upon  a  square  inch  is 

P 

-.~7854D~" 

and  according  to  equation  (105),  it  is. 
ct 


OF  THE  HYDROSTATIC  PRESS.  135 

therefore,  by  comparison,  we  have 
P  ct 


consequently,  by  multiplying  and  substituting  the  cohesive  power  of 
cast  iron,  we  have 

(*4-r)P:=13076D2f.  (111). 

Let  4r2,  be  substituted  in  this  equation,  instead  of  D2  its  equivalent, 
and  we  shall  obtain 


consequently,  the  pressure  in  tons,  is 

52304  r*t  _  23.35  r*t 
~2240(*+r)~-   (f+r)    '  (112). 

From  which  it  appears,  that  by  knowing  the  interior  radius  of  the 
cylinder  and  the  thickness  of  the  metal,  the  power  of  the  press  can 
easily  be  ascertained  ;  the  following  is  the  rule  for  that  purpose. 

RULE.  Multiply  23.35  times  the  thickness  of  metal  by  the 
square  of  the  radius  of  the  cylinder,  and  divide  the  product 
by  the  radius  plus  the  thickness  of  metal,  and  the  quotient 
will  give  the  power  of  the  press  in  tons. 

145.  EXAMPLE  16.  A  Hydrostatic  Press  is  so  constructed,  as  to 
have  the  interior  radius  of  its  cylinder  equal  to  3  inches,  and  the 
thickness  of  metal  4  inches  ;  now  this  press  is  designed  for  packing 
flax,  and  is  estimated  to  stand  a  pressure  of  180  tons;  query  if  its 
power  is  not  overrated  ? 

According  to  the  above  rule,  it  is 


consequently,  the  power  of  the  press  is  overrated  by  about  60  tons, 
being  one  third  less  than  the  estimated  pressure  according  to  the 
question. 

The  thickness  of  metal  necessary  to  resist  a  pressure  of  180  tons  or 
403200  Ibs.  is  equal  to  17.9  inches  nearly,  and  the  proposed  thickness 
is  only  4  inches,  being  less  than  one  fourth  of  the  thickness  which  is 
really  necessary  to  resist  the  strain  ;  hence  we  infer  that  the  press  in 
its  present  state,  is  entirely  unfitted  for  its  intended  purpose,  and 
altogether  inconsistent  with  safety  and  precision  of  operation.  Here 
follows  the  description  of  a  press  when  completely  furnished  in  all  its 
parts  and  fit  for  immediate  action. 


136         THEORY  OF  CONSTRUCTION   AND  SCIENTIFIC  DESCRIPTION 

146.  The  Hydrostatic  Press,,  in  its  present  high  and  refined  state  of 
improvement,  is  a  machine  that  is  capable  of  generating  and  trans- 
mitting a  'greater  degree  of  force,  for  the  purpose  of  overcoming 
immense  resistances,  and  raising  enormous  loads  to  a  small  height, 
than  any  other  instrument  or  engine  with  which  we  are  acquainted ; 
it  is  therefore  of  the  highest  importance  th,at  the  principles  of  its 
construction  and  the  mode  of  operation  should  be  rightly  understood, 
and  in  order  to  render  the  subject  as  clear  and  intelligible  as  possible, 
we  think  proper  to  lay  before  our  readers  the  following  detailed 

description. 

Fig.  1. 


The  wood  cut  before  us,  fig.  1,  exhibits  an  elevation  of  the  press 
in  its  complete  state,  accompanied  by  the  forcing  pump  and  all  its 
appurtenances  as  fitted  up  for  immediate  action  :  F  is  a  strong  metal- 
lic cylinder  of  cast  iron,  or  some  other  material  of  sufficient  density 
to  prevent  the  fluid  from  issuing  through  its  pores,  and  of  sufficient 
strength  to  preclude  the  possibility  of  rupture,  by  reason  of  the  im- 
mense pressure  which  it  is  destined  to  withstand. 

The  cylinder  F  is  bored  and  polished  with  the  most  scrupulous 
precision,  and  fitted  with  the  moveable  piston  D,  which  is  rendered 
perfectly  water-tight,  by  means  of  leather  collars  constructed  for  the 
purpose,  and  fixed  in  the  cylinder  by  a  simple  but  ingenious  con- 
trivance to  be  described  hereafter. 


OF  THE  HYDROSTATIC  PRESS.  137 

Into  the  side  or  base  of  the  cylinder  F,  the  end  of  a  small  tube  bbb 
is  inserted,  and  by  this  tube  the  water  is  conveyed  or  forced  into  the 
cylinder;  the  other  end  of  the  tube  is  attached  to  the  forcing  pump, 
as  represented  in  the  diagram ;  but  this  will  be  more  particularly 
explained  in  another  place. 

A  A  are  two  very  strong  upright  bars,  generally  made  of  wrought  iron, 
and  of  any  form  whatever,  corresponding  to  the  notches  in  the  sides  of 
the  flat  table  E,  which  is  fixed  upon  the  end  of  the  piston  r>,  and  by 
workmen,  is  usually  denominated  the  '  Follower  or  '  Pressing  Table.' 

B  is  the  top  of  the  frame  into  which  the  upright  bars  A  A  are  fixed, 
and  cc  is  the  bottom  thereof,  both  of  which  are  made  of  cast,  in 
preference  to  wrought  iron,  being  both  cheaper  and  more  easily 
moulded  into  the  intended  form. 

The  bottom  of  the  frame  cc,  is  furnished  with  four  projections  or 
lobes,  with  circular  perforations,  for  the  purpose  of  fastening  it  by  iron 
bolts  to  the  massive  blocks  of  wood,  whose  transverse  sections  are 
indicated  by  the  lighter  shades  at  GG.  The  top  B  has  two  similar 
perforations,  through  which  are  passed  the  upper  extremities  of  the 
vertical  bars  A  A,  and  there  made  fast,  by  screwing  down  the  cup-nuts 
represented  at  a  and  a. 

Fiy.  2  represents  the  plan  of  the  top,  or  as  it  is  more  frequently 
termed,  the  head  of  the  frame ;  the  lower  side  Fig.  2. 

or  surface  of  which  is  made  perfectly  smooth, 
in  order  to  correspond  with,  and  apply  to  the 
upper  surface  of  the  pressing  table  E  in^.  1 ; 
this  correspondence  of  surfaces  becomes  ne- 
cessary on  certain  occasions,  such  as  the  copy- 
ing of  prints,  taking  fac-similes  of  letters  and 

the  like ;  in  all  such  cases,  it  is  manifest,  that  smooth  and  coincident 
surfaces  are  indispensable  for  the  purpose  of  obtaining  true  impres- 
sions. 

The  figure  before  us  represents  the  upper  side  of  the  block,  where 
it  is  evident,  that  the  middle  part  B,  (through  whose  rounded  extremi- 
ties a  and  a,  the  circular  perforations  are  made  for  receiving  the 
upright  bars  or  rods  AA,^.  1),  is  considerably  thicker  than  the  parts 
on  each  side  of  it ;  this  augmentation  of  thickness,  is  necessary  to 
resist  the  immense  strain  that  comes  upon  it  in  that  part ;  for  although 
the  pressure  may  be  equally  distributed  throughout  the  entire  surface, 
yet  it  is  obvious,  that  the  mechanical  resistance  to  fracture,  must  prin- 
cipally arise  from  that  part,  which  is  subjected  to  the  re-action  of  the 
upright  bars. 


138 


THEORY  OF  CONSTRUCTION  AND  SCIENTIFIC  DESCRIPTION 


Fig.  3  represents  the  plan  of  the  base  or  bottom  of  the  frame  ;  it  is 
generally  made  of  uniform  thickness,  Fig.  3. 

and  of  sufficient  strength  to  withstand 
the  pressure,  for  be  it  understood,  that 
all  the  parts  of  the  machine  are  sub- 
jected to  the  same  quantity  of  strain, 
although  it  is  exerted  in  different  ways.* 

The  circular  perforations  cc  correspond  to  a  a  in  the  top  of  the 
frame,  and  receive  the  upright  bars  in  the  same  manner  ;  the  perfora- 
tions dddd,  receive  the  screw  bolts  which  fix  the  frame  to  the  beams 
of  timber  represented  at  GG,  fig.  \  ;  the  large  perforation  r  receives 
the  cylinder,  the  upper  extremity  of  which  is  furnished  with  a  flanch, 
for  the  purpose  of  fitting  the  circular  swell  around  the  perforation, 
and  preventing  it  from  moving  backwards  during  the  operation  of 
the  instrument. 

When  the  several  parts  which  we  have  now  described  are  fitted 
together,  they  will  present  us  with  that  portion  of  the  drawing  in 
fig.  1  denominated  "  Elevation  of  the  Press." 

A  side  view  of  the  engine  as  thus  com- 
pleted, is  represented  in  fig.  4,  where,  as  is 
usual  in  all  such  descriptions,  the  same  letters 
of  the  alphabet  refer  to  the  same  parts  of  the 
structure. 

F  is  the  cylinder  into  which  the  fluid  is 
injected ;  D  the  piston,  on  whose  summit  is 
the  pressing  table  E  ;  A  one  of  the  upright 
rods  or  bars  of  malleable  iron  ;  B  the  head  of 
the  press,  fixed  to  the  upright  bar  A  by  means 
of  the  cup-nut  a;  c  the  bottom,  in  which  the 
upright  bar  is  similarly  fixed  ;  and  G  a  beam 
of  timber  supporting  the  frame  with  all  its 
appendages. 

147.   But  the  Hydrostatic  Press  as  here 
.described  and  constructed,  must  not  be  con- 
sidered as  fit  for  immediate  action ;  for  it  is 
manifestly  impossible  to  bore  the  interior  of 
the  cylinder  so  truly,  and  to  turn  the  piston 


Fig.  4. 


*  The  upright  bars,  cylinders,  and  connecting  tubes,  resist  by  tension,  the  pistons 
by  compression,  and  the  pressing  table,  together  with  the  top  and  bottom  of  the 
frame,  resist  transversely. 


OF  THE  HYDROSTATIC  PRESS.  139 

with  so  much  precision,  as  to  prevent  the  escape  of  water  between 
their  surfaces,  without  increasing  the  friction  to  such  a  degree,  that  it 
would  require  a  very  great  force  to  counterbalance  it. 

In  order,  therefore,  to  render  the  piston  water-tight,  and  to  prevent 
as  much  as  possible  the  increase  of  friction,  recourse  must  be  had  to 
other  principles,  which  we  now  proceed  to  explain. 

The  piston  D  is  surrounded  by  a  collar  of  pump  leather  oo,  repre- 
sented in  Jig.  5,  which  collar  being  doubled  up,  so  Fig.  5. 
as  in  some  measure  to  resemble  a  lesser  cup  placed 
within  a  greater,  it  is  fitted  into  a  cell  made  for  its 
reception  in  the  interior  of  the  cylinder ;  and  when 


there,  the  two  parts  are  prevented  from  coming  toge-          ^^m^ 
ther,  by  means  of  the  copper  ring  pp,  represented  in  FtS-  6- 

Jig.  6,  being  inserted  between  the  folds,  and  retained  in  its  place,  by  a 
lodgement  made  for  that  purpose  on  the  interior  of  the  cylinder. 

The  leather  collar  is  kept  down  by  means  of  a  brass  or  bell-metal 
ring  mm,  Jig.  7,  which  ring  is  received  into  a  Fig.  7. 

recess  formed  round  the  interior  of  the  cylinder, 
and  the  circular  aperture  is  fitted  to  admit  the 
piston  D  to  pass  through  it,  without  materially 
increasing  the  effects  of  friction,  which  ought  to  be 
avoided  as  much  as  possible. 

The  leather  is  thus  confined  in  a  cell,  with  the  edge  of  the  inner 
fold  applied  to  the  piston  D,  while  the  edge  of  the  outer  fold  is  in 
contact  with  the  cylinder  all  around  its  interior  circumference ;  in 
this  situation,  the  pressure  of  the  water  acting  between  the  folds  of 
the  leather,  forces  the  edges  into  close  contact  with  both  the  cylinder 
and  piston,  and  renders  the  whole  water-tight ;  for  if  the  leather  be 
properly  constructed  and  rightly  fitted  into  its  place,  it  is  almost 
impossible  that  any  of  the  fluid  can  escape;  for  the  greater  the 
pressure,  the  closer  will  the  leather  be  applied  to  both  the  piston  and 
the  cylinder. 

The  metal  ring  mm  is  truly  turned  in  a  lathe,  and  the  cavity  in 
which  it  is  placed  is  formed  with  the  same  geometrical  accuracy ;  but 
in  order  to  fix  it  in  its  cell,  it  is  cut  into  five  pieces  by  a  very  fine 
saw,  as  represented  by  the  lines  in  the  diagram,  which  are  drawn 
across  the  surface  of  the  ring.  The  four  segments  which  radiate  to 
the  centre  are  put  in  first,  then  the  segment  formed  by  the  parallel 
kerfs,  (the  copper  ring  pp  and  the  leather  collar  oo  being  previously 
introduced),  and  lastly,  the  piston  which  carries  the  pressing  table. 

That  part  of  the  cylinder  above  the  ring  mm,  where  the  inner 


140          THEORY  OF  CONSTRUCTION  AND  SCIENTIFIC  DESCRIPTION 

surface  is  not  in  contact  with  the  piston,  is  filled  with  tow,  or  some 
other  soft  material  of  a  similar  nature ;  the  material  thus  inserted  has 
a  twofold  use  ;  in  the  first  place,  when  saturated  with  sweet  oil,  it 
diminishes  the  friction  that  necessarily  arises,  when  the  piston  is 
forced  through  the  ring  mm;  and  in  the  second  place,  it  prevents 
the  admission  of  any  extraneous  substance,  which  might  increase  the 
friction  or  injure  the  surface  of  the  piston,  and  otherwise  lessen  the 
effects  of  the  machine. 

The  packing  here  alluded  to,  is  confined  by  a  thin  metallic  annulus, 
neatly  fitted  and  fixed  on  the  top  of  the  cylinder,  the  circular  orifice 
being-  of  sufficient  diameter,  to  admit  of  a  free  and  easy  motion  to  the 
piston . 

If  a  cylinder  thus  furnished  with  its  several  appendages  be  placed 
in  the  frame,  and  the  whole  firmly  screwed  together,  and  connected 
with  the  forcing  pump,  as  represented  in  fig.  1,  the  press  is  completed 
and  ready  for  immediate  use ;  but  in  order  to  render  the  construction 
still  more  explicit  and  intelligible,  and  to  show  the  method  of  con- 
necting the  press  to  the  forcing  pump,  let  jig.  8  represent  a  section 
of  the  cylinder  with  all  its  furniture,  jr^.  3. 

and  a  small  portion  of  the  tube  im- 
mediately adjoining,  by  which  the 
connexion  is  effected. 

Then  is  FF  the  cylinder;  D  the 
piston;  the  unshaded  parts  oo  the 
leather  collar,  in  the  folds  of  which 
is  placed  the  copper  ring  pp,  dis- 
tinctly seen  but  not  marked  in  the 
figure;  mm  is  the  metal  ring  by 
which  the  leather  collar  is  retained 

in  its  place ;  nn  the  thin  plate  of  copper  or  other  metal  fitted  to  the 
top  of  the  cylinder,  between  which  and  the  plate  m  m  is  seen  the  soft 
packing  of  tow,  which  we  have  described  above,  as  performing  the 
double  capacity  of  oiling  the  piston  and  preventing  its  derange- 
ment. 

The  combination  at  wx,  represents  the  method  of  connecting  the 
injecting  tube  to  the  cylinder :  it  may  be  readily  understood  by  in- 
specting the  figure;  but  in  order  to  remove  all  causes  of  obscurity,  it 
may  be  explained  in  the  following  manner. 

The  end  of  the  pipe  or  tube,  which  is  generally  made  of  copper^ 
has  a  projecting  piece  or  socket  flanch  soldered  or  screwed  upon  it^ 
which  fits  into  a  perforation  in  the  side  or  base  of  the  cylinder,  accord- 


OF  THE  HYDROSTATIC  PRESS.  141 

ing  to  the  fancy  of  the  projector,  but  in  the  figure  before  us  the  per- 
foration is  in  the  side. 

The  tube  thus  furnished,  is  forcibly  pressed  into  its  seat  by  a  hollow 
screw  w,  called  an  union  screw,  which  fits  into  another  screw  of  equal 
thread  made  in  the  cavity  of  the  cylinder ;  the  joint  is  made  water- 
tight, by  means  of  a  collar  of  leather,  interposed  between  the  end  of 
the  tube  and  the  bottom  of  the  cavity. 

A  similar  mode  of  connection  is  employed  in  fastening  the  tube  to 
the  forcing  pump,  the  description  of  which,  although  it  constitutes  an 
important  portion  of  the  apparatus,  does  not  properly  belong  to  this 
place ;  the  principles  of  its  construction  and  mode  of  action,  must 
therefore  be  supposed  as  known,  until  we  come  to  treat  of  the  con- 
struction and  operation  of  pumps  in  general. 

Admitting  therefore,  that  the  action  of  the  forcing  pump  is  under- 
stood, it  only  now  remains  to  explain  the  nature  of  its  operation  in 
connection  with  the  Hydrostatic  Press,  the  construction  of  which  we 
have  so  copiously  exemplified. 

148.  In  order  to  understand  the  operation  of  the  press,  we  must 
conceive  the  piston  D  Jig.  1,  as  being  at  its  lowest  possible  position  in 
the  cylinder,  and  the  body  or  substance  to  be  pressed,  placed  upon  the 
crown  or  pressing  table  E  ;  then  it  is  manifest,  that  if  water  be  forced 
along  the  tube  b  b  b  by  means  of  the  forcing  pump,  it  will  enter  the 
chamber  of  the  cylinder  F  immediately  beneath  the  piston  D,  and  cause 
it  to  rise  a  distance  proportioned  to  the  quantity  of  fluid  that  has  been 
injected,  and  with  a  force,  determinable  by  the  ratio  between  the  square 
of  the  diameter  of  the  cylinder  and  that  of  the  forcing  pump.  The 
piston  thus  ascending,  carries  its  crown,  and  consequently,  the  load 
along  with  it,  and  by  repeating  the  operation,  more  water  is  injected, 
and  the  piston  continues  to  ascend,  till  the  body  comes  into  contact 
with  the  head  of  the  frame  B,  when  the  pressure  begins ;  thus  it  is 
manifest,  that  by  continuing  the  process,  the  pressure  may  be  carried 
to  any  extent  at  pleasure ;  but  we  have  already  stated,  in  developing 
the  theory,  that  there  are  limits,  beyond  which,  with  a  given  bore 
and  a  given  thickness  of  metal,  it  would  be  unsafe  to  continue  the 
strain. 

When  the  press  has  performed  its  office,  and  it  becomes  necessary 
to  relieve  the  action,  the  discharging  valve,  placed  in  the  furniture 
of  the  forcing  pump,  must  be  opened,  which  will  admit  the  water  to 
escape  out  of  the  cylinder  and  return  to  the  cistern,  while  the  table 
and  piston,  by  means  of  their  own  weight,  return  to  their  original 
position. 


142         THEORY  OF  CONSTRUCTION  AND  SCIENTIFIC  DESCRIPTION 

The  method  of  calculating  the  power  of  the  press,  as  well  as  every 
other  particular  respecting  it,  has  been  fully  exemplified  in  the  fore- 
going theory ;  it  is  hence  unnecessary  to  dwell  longer  on  the  subject : 
we  shall  therefore  conclude  our  description  of  the  press,  and  proceed 
with  that  of  the  Hydrostatic  Bellows,  which  depends  upon  the  same 
principle,  viz.  the  quaqua  versum  pressure  of  non-elastic  fluids. 


2.    THEORY  OF   CONSTRUCTION    AND    SCIENTIFIC    DESCRIPTION    OF   THE 
HYDROSTATIC  BELLOWS. 

149.  In  the  preceding  pages  we  have  developed   the  theory  and 
exemplified  the  application  of  the  hydrostatic  press  ;  and  furthermore, 
in  order  to  render  the  subject  as  complete  as  possible,  we  have  given 
a  minute  and  comprehensive  description  of  its  several  parts,  and  for 
the  purpose  of  guiding  the  practical  mechanic  in  its  erection,  the 
instrument  is  exhibited  in  its  complete  and  finished  state,  accompanied 
by  the  forcing  pump  and  all  its  requisite  appendages. 

The  next  subject,  therefore,  that  claims  our  attention,  is  the 
Hydrostatic  Bellows,  an  instrument  of  very  frequent  occurrence  in 
philosophical  experiments ;  it  is  chiefly  employed  in  illustrating  the 
upward  pressure  of  non-elastic  fluids  and  the  hydrostatic  paradox, 
and  consequently,  it  depends  upon  the  same  principle  as  the  hydro- 
static press,  admitting  of  a  similar,  but  a  more  concise  mode  of  dis- 
cussion and  illustration. 

150.  The  Hydrostatic  Bellows  consists  of  a  tube  or  pipe  FEI,  of 
very  small  diameter,  and  of  any  convenient  length 

at  pleasure,  connected  by  means  of  the  elbow  at  ^ 
i,  with  a  cylindrical  vessel  whose  vertical  section 
is  CDGH,  and  whose  sides  are  made  of  leather 
like  a  common  bellows,  represented  by  the  waving 
lines  AmD  and  BWK;  the  upper  and  the  lower 
surfaces  AB  and  DC,  being  formed  of  circular 
boards  corresponding  to  the  cylindrical  form  of 
the  vessel. 

When  the  bellows  is  empty,  it  is  manifest  that  the  boards  A  B  and 
DC  are  very  nearly  in  contact,  and  would  be  completely  so,  but  for 
the  leather  sides  forming  into  folds  and  preventing  a  coincidence: 
in  this  state,  when  water  or  any  other  incompressible  and  non-elastic 
fluid,  is  poured  into  the  tube,  it  flows  into  the  bellows  and  separates 
the  boards ;  a  heavy  weight  as  w  is  then  placed  upon  the  upper 


OF  THE  HYDROSTATIC  BELLOWS.  143 

board,  and  by  pouring  more  fluid  into  the  tube,  the  moveable  plane 
A  B  and  its  incumbent  load  w,  will  be  raised  and  kept  in  equilibrio 
by  the  column  of  fluid  in  the  tube ;  and  when  the  equilibrium  obtains, 
we  infer,  that : — 

The  iveight  of  the  supporting  column  of  fluid  in  the  tube,  is 
to  the  weight  upon  the  moveable  plane,  as  the  area  of  a  section 
of  the  tube,  is  to  the  area  of  the  plane. 

This  is  manifest,  for  the  fluid  at  i,  the  lowest  point  of  the  vertical 
tube  FEI,  is  pressed  by  a  force  varying  as  the  altitude  LI,  and  by  the 
nature  of  fluidity,  this  pressure  is  communicated  horizontally  to  all 
the  particles  in  DC,  and  thence  transmitted  throughout  the  whole  mass 
of  fluid  in  the  bellows  ;  consequently,  the  pressure  upwards  on  the 
board  AB,  is  equal  to  the  weight  of  a  column  of  the  fluid,  the  diameter 
of  whose  base  is  DC,  and  altitude  LI  or  GD  ;  but  the  actual  weight  of 
the  fluid  supported,  is  that  of  a  column  whose  diameter  is  DC,  and 
altitude  EI  or  AD. 

Hence,  the  weight  which  maintains  the  equilibrium,  will  be  that  of 
a  cylinder  of  fluid,  whose  base  is  A  B  and  altitude  A  G  ;  consequently, 
the  weight  w,  placed  upon  the  moveable  plane  of  the  bellows,  since  it 
balances  the  column  of  fluid  L  E,  is  equivalent  to  the  weight  of  a  fluid 
cylinder,  whose  section  along  the  axis  is  ABHG. 

Put  D  rz  AB  or  DC,  the  diameter  of  the  cylindrical  vessel  or  bellows, 
d  zz  LM,  the  diameter  of  the  vertical  tube, 
w  ~  the  weight  upon  the  moveable  plane,  and 
w'-=.  the  weight  or  pressure  of  the  fluid  in  the  column  LE. 

Then,  because  by  the  principles  of  mensuration,  the  areas  of  circles 
are  to  one  another  as  the  squares  of  their  diameters ;  the  foregoing 
inference  gives 

w'  :  w  :  :  d*  :  D2, 

and  this,  by  equating  the  products  of  the  extreme  and  mean  terms, 
becomes 

tfw'=:d*w.  (113). 

Let  both  sides  of  this  equation  be  divided  by  the  quantity  D2,  which 
is  found  in  combination  with  the  weight  or  pressure  of  the  fluid  in  the 
tube,  and  we  shall  obtain 

,      d*w 
W=^'  (114). 

Here  again,  that  singular  property  of  non-elastic  and  incompressible 
fluids  becomes  manifest,  viz. 


144         THEORY  OF  CONSTRUCTION   AND  SCIENTIFIC  DESCRIPTION 

That  any  quantity  however  small,  may  be  made  to  balance 
any  other  quantity  however  great. 

151.  If  the  diameter  of  the  tube,  the  diameter  of  the  cylinder  or 
bellows,  and  the  weight  upon  the  moveable  board  AB  be  given,  the 
weight  of  the  fluid  in  the  tube,  or  its  perpendicular  altitude  to  main- 
tain the  equilibrium,  can  easily  be  determined  by  means  of  the  equa- 
tion (114),  which  affords  the  following  practical  rule. 

RULE.  Multiply  the  square  of  the  diameter  of  the  tube  by 
the  load  upon  the  moveable  board,  and  divide  the  product  by 
the  square  of  the  diameter  of  the  bellows  or  cylinder  ;  then, 
the  quotient  will  give  the  weight  of  the  fluid  by  which  the 
equilibrium  is  maintained. 

EXAMPLE.  The  diameter  of  the  bellows  or  cylindrical  vessel  is  18 
inches,  that  of  the  tube  or  pipe,  through  which  the  fluid  is  conveyed 
into  the  vessel,  is  one  fourth  of  an  inch,  and  the  weight  upon  the 
moveable  board  is  5760  Ibs.  ;  what  weight  of  water  must  be  poured 
into  the  vertical  tube,  so  that  the  whole  may  remain  at  rest  ? 

In  this  example  there  are  given,  Dm  18  inches;  c?=:|  of  an  inch, 
and  w  =  5760  Ibs.  ;  therefore,  by  performing  as  directed  in  the  rule, 
we  shall  have 

.25*  X  5760      360 


Here  it  appears,  that  a  quantity  of  water  weighing  l£lbs.,  disposed 
in  a  tube  of  £  of  an  inch  in  diameter,  is  capable  of  balancing  another 
quantity  of  5760  Ibs.,  disposed  in  a  cylinder  of  18  inches  diameter; 
it  is  therefore  manifest,  that  the  height  of  the  one  column  must  far 
exceed  the  height  of  the  other,  and  the  excess  of  altitude  may  be 
determined  in  the  following  manner. 

152.  It  has  been  abundantly  proved  by  experiment,  that  a  cubic 
foot  of  distilled  water,  at  the  temperature  of  about  39°  of  Fahrenheit's 
Thermometer,  weighs  very  nearly  1000  avoirdupois  ounces,  or  62  Jibs.  ; 
consequently,  the  number  of  cubic  inches  in  the  column  whose  weight 
is  l^lbs.,  is  found  by  the  following  analogy,  viz. 

62£  :  1728  ::  l£  :  30±f  inches; 

hence,  the  solidity  of  a  column  which  maintains  the  equilibrium  is 
30£f-  inches,  and  according  to  the  conditions  of  the  question,  the 
diameter  of  its  base  or  section,  is  one  fourth  of  an  inch,  and  con- 
sequently, the  area  of  the  base  or  section,  is 

.25*  X.  7854  =  .  0490875  square  inches. 


OF  THE  HYDROSTATIC  BELLOWS.  145 

Now,  according  to  the  principles  of  mensuration,  the  solidity  of  a 
cylinder  is  determined,  by  multiplying  the  area  of  its  base  into  its 
perpendicular  altitude  ;  consequently,  if  h  denote  the  perpendicular 
height  of  the  column,  we  have 

.0490875  h  =  30.72; 
therefore,  by  division,  we  shall  obtain 

OA   (TO 


153.  The  solution  which  we  have  here  given,  applies  to  the  parti- 
cular example  preceding,  in  which  the  data  are  assigned  ;  but  in  order 
to  accommodate  the  theory  to  every  case,  it  becomes  necessary  to 
draw  up  the  solution  in  general  terms  ;  for  which  purpose,  we  must 
recur  to  equation  (114),  where  the  weight  of  the  equilibrating  column 
has  already  been  found  ;  then,  according  to  the  above  analogy,  we 
have 

62i  :  1728  :  :  £f  ,  ., 

where  s  denotes  the  solidity  of  the  column. 

If  in  the  above  analogy,  we  make  the  product  of  the  jnean  terms 
equal  to  the  product  of  the  extremes,  we  shall  have 


and  from  this,  by  division,  we  get 


tf  (115). 

Therefore,  if  the  solidity  of  the  equilibrating  column  be  divided  by 
the  area  of  its  base,  viz.  the  quantity  .7854d2,  the  quotient  will  fur- 
nish the  perpendicular  altitude  ;  hence  we  have 

_  35.2024  w 

D*  (116). 

154.  From  this  it  appears,  that  in  order  to  determine  the  altitude 
of  the  equilibrating  column,  it  is  not  necessary  that  its  diameter 
should  be  known,  for  the  equation  is  wholly  independent  of  that 
element,  the  diameter  of  the  bellows,  and  the  weight  upon  tne 
moveable  board  only,  entering  into  its  composition.  The  following 
practical  rule  will  therefore  determine  the  altitude  of  the  column  by 
which  the  equilibrium  is  maintained. 

VOL.   I.  L 


146         THEORY  OF  CONSTRUCTION   AND  SCIENTIFIC  DESCRIPTION 

RULE.  Divide  35.2024  times  the  load  to  be  sustained  upon 
the  moveable  board,  by  the  square  of  the  diameter  of  the 
bellows,  and  the  quotient  will  be  the  altitude  of  the  equi- 
librating column. 

We  shall  determine  the  perpendicular  altitude  by  this  rule,  on  the 
supposition  that  the  diameter  of  the  bellows  and  the  weight  upon  the 
moving  plane,  are  the  same  as  in  the  foregoing  example ;  therefore 
we  have 

.       35.2024X5760       _,Q10.     , 
^—  — nz  625.8 19  inches. 

The  equation  (114)  for  the  weight  of  the  equilibrating  column,  was 
deduced  from  the  equation  (113),  by  simple  division  only,  without  the 
enunciation  of  any  problem ;  but  in  order  to  render  the  subject  a 
little  more  systematic,  we  shall  determine  the  other  elements  of  the 
general  equation,  severally  from  the  resolution  of  their  respective  and 
appropriate  problems. 

PROBLEM  XXII. 

155.  In  a  hydro-statical  bellows  of  a  cylindrical  form,  there 
are  given,  the  diameters  of  the  bellows  and  of  the  equilibrating 
tube,  together  with  the  weight  of  the  fluid  by  which  the  equili- 
brium is  maintained : — 

It  is  required  to  determine  the  weight  upon  the  moveable 
plane,  at  the  instant  when  the  equilibrium  obtains. 

Let  both  sides  of  the  general  equation  (113),  be  divided  by  d*  the 
square  of  the  diameter  of  the  balancing  tube,  and  we  shall  obtain 

_pV 

~-~d^'  (117). 

And  this  equation  affords  the  following  practical  rule. 

RULE.  Multiply  the  weight  of  the  equilibrating  fluid,  by 
the  square  of  the  diameter  of  the  bellows,  and  divide  the 
product  by  the  square  of  the  diameter  of  the  tube,  for  the 
weight  upon  the  moveable  plane. 

EXAMPLE.  The  diameter  of  a  cylindrical  bellows  is  24  inches,  the 
diameter  of  the  balancing  tube  is  one  fourth  of  an  inch,  and  the 
weight  of  the  fluid  in  the  tube  is  2  J  Ibs. ;  what  weight  will  this  coun- 
terpoise on  the  moving  board  of  the  bellows  ? 


OF  THE  HYDROSTATIC  BELLOWS.  147 

Here,  by  proceeding  as  directed  in  the  rule,  we  obtain 


This  is  something  more  than  10  tons  and  a  quarter,  which  is  mani- 
festly a  great  load  to  be  suspended  by  2  Jibs.;  but  the  altitude  of  the 
suspending  column  must  be  proportionably  great,  which  circumstance, 
without  the  aid  of  some  artificial  force,  would  render  the  instrument 
very  inconvenient  for  any  practical  purpose  ;  it  was,  no  doubt,  by 
viewing  the  matter  in'this  light,  that  Mr.  Bratnah,  senior,  was  led  to 
apply  the  forcing  pump,  and  thereby  to  produce  that  very  powerful 
engine,  which  formed  the  subject  of  our  last  article. 

PROBLEM  XXIII. 

156.  In  a  hydrostatical  bellows  of  a  circular  form,  there  are 
given,  the  diameter  of  the  bellows,  the  load  suspended,  and  the 
weight  of  the  suspending  fluid  :  — 

It  is  required  to  determine  the  diameter  of  the  equilibrating 
tube,  so  that  the  instrument  may  be  just  in  a  state  of  equi- 
librium. 

Let  both  sides  of  the  general  equation  (113),  be  divided  by  w  the 
weight  upon  the  bellows,  and  we  shall  obtain 

D2M/ 

w 
and  from  this,  by  extracting  the  square  root,  we  get 


D2^' 


w    '  (118). 

_ 
And    the    practical    rule    which   this   equation    supplies,    may    be 

expressed  in  words  at  length  in  the  following  manner. 

RULE.  Multiply  the  square  of  the  diameter  of  the  bellows, 
by  the  weight  of  the  fluid  which  maintains  the  equilibrium, 
and  divide  the  product  by  the  weight  upon  the  bellows,  then, 
the  square  root  of  the  quotient  will  be  the  diameter  of  the 
equilibrating  tube. 

*  This  equation  for  the  diameter  of  the  tube  may  be  otherwise  expressed  ;  thus  * 


i  2 


148         THEORY  OF  CONSTRUCTION   AND  SCIENTIFIC  DESCRIPTION 

EXAMPLE.  The  diameter  of  the  bellows  or  cylindrical  vessel,  is  24 
inches,  the  weight  of  the  suspending  fluid  is  2  Ibs.,  and  the  weight 
suspended  on  the  bellows  8000  Ibs.  ;  what  is  the  diameter  of  the 

tube? 

Performing  according  to  the  rule,  we  have 


and  from  this,  by  extracting  the  square  root,  we  obtain 
6?=V°-144  =  -38  of  an  i 


PROBLEM  XXIV. 

157.  In  a  hydrostatic  bellows  of  a  cylindrical  form,  the 
diameter  of  the  tube,  the  weight  suspended,  and  the  weight  of 
the  suspending  fluid,  are  given  :  — 

It  is  required  to  determine  the  diameter  of  the  bellows,  so 
that  the  whole  may  be  in  a  state  of  equilibrium. 
Let  both  sides  of  the  general  equation  (113),  be  divided  by  w'  the 
weight  of  the  suspending  fluid,  and  we  shall  have 


__ 
w' 


from  which,  by  extracting  the  square  root,  we  get 


w1  '  (119). 

And  from  this  equation,  we  obtain  the  following  practical  rule. 

RULE.  Multiply  the  square  of  the  diameter  of  the  suspend- 
ing tube,  by  the  weight  suspended,  and  divide  the  product  by 
the  weight  of  the  fluid  which  maintains  the  equilibrium;  then, 
the  square  root  of  the  quotient  will  be  the  diameter  of  the 
cylinder  sought. 

EXAMPLE.  The  diameter  of  the  suspending  tube  in  a  cylindrical 
hydrostatic  bellows,  is  half  an  inch,  the  weight  of  the  suspending  fluid 
is  2  Ibs.,  and  the  weight  suspended  on  the  bellows  board  is  12000  Ibs. ; 
what  is  the  diameter  of  the  bellows  ? 

Here,  by  proceeding  as  directed  in  the  foregoing  rule,  we  get 

.5X. 5X12000 
D*~ —  1500, 

*  This  equation  for  the  diameter  of  the  bellows  may  be  otherwise  expressed ;  thus  : 


OF  THE  HYDROSTATIC  BELLOWS.  149 

and  by  extracting  the  square  root,  we  have 


:rr  38.73  inches. 

158.  The  foregoing  problems  and  rules,  unfold  every  particular 
respecting  the  calculation  of  the  hydrostatic  bellows,  and  from  them 
we  may  infer,  that  in  the  case  of  an  equilibrium,  if  more  fluid  be 
added  : — 

It  will  ascend  equally  in  the  suspending  tube,  and  in  the 
cylindrical  vessel  composing  the  bellows,  whatever  may  be 
their  relative  magnitudes. 

The  demonstration  of  this  is  very  simple,  for  let  A  BCD  be  a  vertical 
section,  passing  along  the  axis  of  the  cylindrical 
vessel,  and  also  along  the  axis  of  the  suspending 
tube  KI  ;  and  suppose  that  F  and  c  are  the  points 
to  which  the  fluid  rises  in  the  vessel  and  the  tube, 


when  the  bellows  is  in  a  state  of  equilibrium. 

Take  ic  equal  to  Da,  and  through  the  points  a  D  C  I 
and  c  let  a  horizontal  plane  be  drawn,  intersecting 
the  vertical  plane  A  BCD  in  the  line  ab ;  then  it  is  manifest,  that  the 
weight  w  in  the  position  EF,  is  equivalent  to  the  weight  of  the  fluid 
column  a&FE.  Let  more  fluid  be  poured  into  the  tube  at  K;  the 
equilibrium  will  then  be  destroyed,  and  the  weight  w  will  ascend, 
until  by  discontinuing  the  supply,  the  equilibrium  is  restored,  and  the 
fluid  in  the  vessel  and  the  tube  becomes  again  quiescent  at  the  points 
n  and  K. 

Take  IK  equal  to  DA,  and  through  the  points  A  and  K,  let  a  hori- 
zontal plane  be  drawn,  cutting  the  vertical  plane  A  BCD  in  the  line 
AB;  then  as  before,  the  weight  w  in  the  position  mn,  is  equivalent 
to  the  weight  of  the  fluid  cylinder,  of  which  A&nm  is  a  vertical 
section. 

Now,  the  weight  w  is  not  altered  in  consequence  of  the  change  of 
position  from  EF  to  mn\  therefore,  because  EF  is  equal  to  mn,  it 
follows,  that  E«  is  equal  to  mA;  consequently,  by  taking  away  the 
common  space  ma,  the  remainders  Em  and  a  A  are  equal  to  one 
another;  but  by  reason  of  the  parallel  lines  ac  and  AK,  the  spaces 
a  A  and  CK  are  equal  to  one  another;  therefore  CK  is  equal  to 
Em. 

From  the  principle  here  demonstrated,  the  resolution  of  the  follow- 
ing problem  may  readily  be  derived. 


150         THEORY  OF  CONSTRUCTION  AND  SCIENTIFIC  DESCRIPTION 

PROBLEM  XXV. 

159.  If  a  hydrostatic  bellows  of  a  cylindrical  form,  have  a 
given  quantity  of  fluid  poured  into  the  equilibrating  or  suspend- 
ing tube  :  — 

It  is  required  to  determine  through  what  space  the  weight 
on  the  moving  board  will  ascend  in  consequence  of  the  supply. 

Before  we  proceed  to  the  resolution  of  this  problem,  it  may  be 
proper,  as  in  the  foregoing  cases,  to  exhibit  an  appropriate  notation  ; 
for  which  purpose, 

Put  D=  AB  or  DC,  the  diameter  of  the  cylindrical  vessel  or  bellows, 
c?zz  the  diameter  of  the  equilibrating  or  suspending  tube, 
q  HZ  the  quantity  of  fluid  poured  into  the  tube,  and 
a;  —  Em,  the  space  through  which  the  weight  ascends  by  reason 
of  the  supply. 

Then,  according  to  the  principles  of  mensuration,  the  area  of  a 
transverse  section  of  the  cylindrical  vessel  or  bellows,  is 


and  the  area  of  the  corresponding  section  of  the  tube,  is 


where  the  symbols  a  and  a'  denote  the  respective  areas. 
But  by  the  property  demonstrated  above,  the  fluid  rises  equally  in 
the  bellows  and  in  the  tube  ;  therefore,  the  quantity  of  fluid  which 
flows  into  the  bellows  in  consequence  of  the  supply,  is 


and  the  quantity  which  remains  in  the  tube,  is 


where  the  symbols  s  and  s'  denote  the  solidities  of  the  cylinders, 
whose  diameters  are  D  and  d,  and  their  common  altitude  x. 

Now,  the  sum  of  these  quantities,  is  manifestly  equal  to  the  quantity 
of  fluid  poured  into  the  tube  ;  hence  we  have 


and  by  division,  we  obtain 

-  9 

(120). 


OF  THE  HYDROSTATIC  WEIGHING  MACHINE.  151 

It  therefore  appears,  that  the  space  through  which  the  weight 
ascends  by  reason  of  the  supply  : — 

Is  equal  to  the  quantity  of  fluid  which  is  poured  into  the 
tube,  divided  by  the  sum  of  the  areas  of  a  cross  section  of  the 
tube  and  the  cylindrical  vessel  or  bellows. 

The  practical  rule,  or  method  of  applying  the  equation,  may  there- 
fore be  expressed  in  words  at  length  in  the  following  manner. 

RULE.  Divide  the  quantity  of  fluid  which  is  poured  into 
the  tube,  by  .7854  times  the  sum  of  the  squares  of  the  dia- 
meters, and  the  quotient  will  give  the  quantity  of  ascent,  or 
the  space  through  which  the  weight  is  raised  in  consequence 
of  the  supply. 

EXAMPLE.  The  diameter  of  a  cylindrical  vessel  is  20  inches,  and 
that  of  the  suspending  tube  is  one  inch  ;  now,  suppose  that  an  incom- 
pressible fluid  is  poured  into  the  tube,  until  its  weight  sustain  in 
equilibrio,  a  load  of  8760  Ibs.  upon  the  moveable  bellows  board ; 
then,  how  much  higher  will  the  load  be  raised,  when  150  cubic  inches 
of  the  fluid  are  superadded  ? 

Here  then,  we  have  given  D=r20  inches,  d—  1  inch,  and  <?—  150 
cubic  inches ;  consequently,  by  the  rule,  we  obtain 

*  Hi       *  =  .7854(450°0+1)  ='476  °f  a"  inch'  • r: '  I' 

From  this  it  appears,  that  if  a  machine  of  the  given  dimensions  be 
brought  into  a  state  of  equilibrium,  the  addition  of  150  cubic  inches 
will  raise  the  load  .476,  or  very  nearly  half  an  inch  higher ;  in  which 
case  the  equilibrium  will  still  obtain,  for  the  altitude  of  the  fluid  in 
the  suspending  tube,  is  increased  exactly  as  much  as  the  load  has 
been  raised,  while  the  magnitude  of  the  load,  and  consequently,  the 
height  of  the  equilibrating  column,  remain  the  same. 

3.    THEORY  OF  CONSTRUCTION    AND    SCIENTIFIC    DESCRIPTION    OF   THE 
HYDROSTATIC   WEIGHING  MACHINE. 

160.  The  preceding  principles  may  also  be  applied  to  the  con- 
struction of  a  very  simple  and  convenient  weighing  machine ;  for,  if 
into  the  side  of  an  open  cylindrical  or  other  vessel,  a  bent  tube  be 
inserted,  and  if  on  the  surface  of  the  fluid,  a  moveable  cover  exactly 


152          THEORY  OF  CONSTRUCTION  AND  SCIENTIFIC  DESCRIPTION 

fitting  the  vessel   be  placed  with   a  weight  upon   it,  and  the  tube 
graduated : — 

Then,  any  additional  weight  placed  upon  the  cover,  may  be 
determined  by  knowing  the  height  to  which  the  fluid  rises  in 
the  tube  ;  and  conversely  : — 

If  the  additional  weight  be  known,  the  height  to  which  the 
fluid  rises  in  the  tube  may  be  found. 

Let  ABCD  represent  a  vertical  section  of  a  cylindrical  vessel,  or  of 
any  other  vessel,  whose  sides  are  perpendicular 
to  the  horizon ;  and  let  K  i  c  be  the  corres- 
ponding section  of  the  equilibrating  tube. 

Let  both  the  vessel  and  the  communicating 
tube  be  open  at  the  upper  parts  AB  and  de, 
and  conceive  the  vessel  to  be  filled  with  fluid 
to  the  line  EF  or  altitude  DE;  then,  on  the 
surface  of  the  fluid  at  EF,  let  there  be  placed 
a  moveable  cover  exactly  fitting  the  vessel,  so 
that  the  whole  may  be  water-tight. 

Produce  EF  to  b,  then  is  the  point  b  at  the  same  level  in  the  tube 
IK,  as  the  surface  of  the  fluid  in  the  vessel  whose  level  is  EF  :  upon 
the  cover  EF  let  the  weight  w  be  placed,  and  suppose  a  to  be  the 
point  in  the  tube,  to  which  the  fluid  will  rise  by  the  action  of  the 
cover,  together  with  the  weight  w  which  is  placed  upon  it;  in  this 
case,  the  machine  is  in  a  state  of  equilibrium. 

If  some  additional  weight  w'  be  placed  upon  the  cover,  then  the 
original  equilibrium  will  be  destroyed,  and  can  only  be  restored,  by 
the  fluid  ascending  in  the  tube  to  a  sufficient  height  to  balance  the 
additional  weight. 

Put  DZZ  AB  or  DC,  the  diameter  of  the  cylindrical  vessel,  of  which 

ABCD  is  a  section, 

d  zz  de,  the  diameter  of  the  communicating  tube  KIC, 
h  zz  ba,  the  height  of  the  original  equilibrating  column, 
wzz  the  weight  supported  by  the  column  b  a, 
w/zzthe  additional  weight,  whose  quantity  is  required, 
A'zzaK,  the  increased  altitude  of  the  supporting  column, 
$  zz  Em,  the  descent  of  the  cover  occasioned  by  the  additional 

load  w',  and 
$  zz  the  specific  gravity  of  the  fluid, 


OF  THE  HYDROSTATIC  WEIGHING  MACHINE.  153 

Then  it  is  manifest,  that  when  the  equilibrium  originally  obtains  ; 
that  is,  when  the  surface  of  the  fluid  in  the  tube  is  at  a,  and  that  in 
the  vessel  at  EF,  the  pressure  of  the  fluid  in  the  tube  exerted  at  b,  is 


where  the  symbol  p  denotes  the  pressure  at  b  ; 

but  this  is  manifestly  in  equilibrio  with  the  pressure  of  the  column 
wEFxt  or  the  weight  w  ;  consequently,  we  have 

.7854rf2As  :  w  :  :  .7854tf  :  .7854D2; 
or  by  suppressing  the  common  factors,  we  obtain 

hs  :  w::  1  :  .7854  D8; 
and  by  equating  the  products  of  the  extremes  and  means,  it  is 

wn=.7854£sD2.  (121). 

Again,  by  means  of  the  additional  weight  w',  whose  magnitude  is 
required,  the  cover  EF  is  supposed  to  descend  to  the  position  mn; 
while,  in  order  to  regain  the  equilibrium,  the  fluid  rises  in  the  tube  as 
far  as  the  point  K,  in  which  case,  the  altitude  of  the  equilibrating 
column  CK  becomes  (h  -f-  ti  -f-  5)  and  consequently,  the  pressure  at 
c,  is 

;>'=.7854eZ2s  (&  +  &'  +  3), 

and  this  is  in  equilibrio  with  the  pressure  of  the  column  ymnz,  or 
the  weight  (w  +  w')  ;  consequently,  we  have 

.7854d2s  (h  +  h'  -f  3)  :  w  -f  w'  :  :  .7854d2  :  .7854D*  ; 
or  by  suppressing  the  common  factors,  we  have 

s(k-\-h'  -M)  :  w  +  w'  :  :  1  :  .7854o2; 

therefore,  by  equating  the  products  of  the  extremes  and  means,  we  get 
w  -\-  w/zz  .7854D2s(/z-r-A'-r-S).  (122). 

But  we  have  seen  above,  equation  (121),  that  wnr.7854/i5D2; 
consequently,  by  substituting  and  separating  the  terms,  we  obtain 

W/i=.7854D2s(A/  +  a).  (123). 

Now,  it  is  manifest,  that  the  descent  of  the  cover  in  the  vessel,  and 
the  rise  of  the  fluid  in  the  tube,  must  be  to  one  another,  inversely  as 
the  squares  of  the  respective  diameters  ;  therefore,  we  have 

Ztf^h'tf, 

*     or  by  division,  we  get 
h'd* 

S  =  l^' 

and  finally,  by  substitution,  we  obtain 

w'  =  .7854  h's  (D2  -f  d3).  (124). 


154 SCIENTIFIC  DESCRIPTION  OF  THE  HYDROSTATIC  WEIGHING  MACHINE. 

161.  If  the  fluid  be  water,  whose  specific  gravity  is  represented  by 
unity,  the  equation  becomes  somewhat  simpler  ;  for  in  that  case,  we 
have 

w'  =  .7854  &'  (D«  -f  d2).  (125). 

From  this  equation  the  magnitude  of  the  additional  weight,  or  the 
measure  by  which  it  is  expressed,  can  very  easily  be  ascertained  ; 
and  the  practical  rule  by  which  it  is  discovered,  is  as  follows. 

RULE.  Multiply  the  sum  of  the  squares  of  the  diameters, 
by  .7854  times  the  rise  of  the  fluid  in  the  tube,  or  the  eleva- 
tion above  the  first  level,  and  the  product  will  express  the 
magnitude  of  the  additional  weight. 

EXAMPLE.  The  diameter  of  a  cylindrical  vessel  is  16  inches,  and 
that  of  the  communicating  tube  one  inch ;  now,  supposing  the  machine 
in  the  first  instance,  to  be  in  a  state  of  equilibrium,  and  that  by  the 
addition  of  a  certain  weight  on  the  moveable  cover,  the  water  in  the 
tube  rises  6  inches  above  the  original  equilibrating  level ;  how  much 
weight  has  been  added  ? 

By  proceeding  according  to  the  rule,  we  have 
D2  -|-  d*=  162  +  P  i=256  +  1  =  257, 

and  by  multiplication,  we  obtain 
w'  =  .7854X6X257  zz  1211.0868  avoirdupois  Ibs.* 

162.  If  the  additional  weight,  by  which  the  water  is  made  to  rise 
in  the  tube  be  given,  the  distance  above  the  first  level  to  which  it  will 
rise,  can  easily  be  found ;  for  let  both  sides  of  the  equation  (125),  be 
divided  by  the  quantity  .7854  (D*  -\-  dz),  and  we  shall  obtain 

,,_ 

~.7854(D*4-d2)' 

And  from  this  equation,  we  deduce  the  following  rule. 

RULE.  Divide  the  additional  weight,  by  the  sum  of  the 
areas  of  the  moveable  cover  and  the  cross  section  of  the 
communicating  tube,  and  the  quotient  will  give  the  height  to 
which  the  fluid  will  rise  above  the  first  level. 

*  It  is  manifest  from  the  form  of  the  equation  which  supplies  the  rule,  that 
without  paying  particular  attention  to  the  nature  of  the  load  which  produces  the 
equilibrium  in  the  first  place,  the  value  of  vf  is  ambiguous,  and  may  be  read  in 
ounces,  Ibs.,  cwts.,  or  tons  ;  and  indeed,  in  any  denomination  of  weight  whatever ; 
but  it  must  always  be  read  in  the  same  name  as  that  by  which  the  equilibrium 
is  produced. 


EXPERIMENTS  ON  THE  QUAQUAVERSUS  PRESSURE  OF  FLUIDS.       155 

EXAMPLE.  The  diameter  of  the  moveable  cover  is  16  inches,  and 
that  of  the  communicating  tube  one  inch  ;  then,  supposing  that  the 
machine  in  the  first  instance  is  brought  to  a  state  of  equilibrium, 
and  that  a  load  of  1211  Ibs.  is  applied  on  the  cover,  in  addition  to 
that  which  produces  the  equipoise  ;  to  what  height  above  the  first  level 
will  the  water  ascend  in  the  communicating  tube? 

Proceeding  according  to  the  rule,  we  obtain 

.7854  (D8  +  <P)  =  .7854  (16*  +  I2)  =  201  .8478  divisor  ; 

consequently,  by  division  it  is 


And  exactly  after  the  manner  of  these  two  examples,  may  any 
other  case  be  calculated  ;  but  in  applying  the  principles  to  the 
determination  of  weights,  mercury  ought  to  be  employed  in  preference 
to  water,  as  it  exerts  an  equal  influence  in  less  space,  and  besides,  it 
is  not  subject  to  a  change  of  density  by  putrefaction  and  the  like. 

4.     EXPERIMENTS     ILLUSTRATING    THE    QUAQUAVERSUS    PRESSURE    OF 
INCOMPRESSIBLE    FLUIDS. 

Before  we  conclude  our  inquiries  on  fluid  pressure,  it  may  be  both 
interesting  and  instructive  to  the  readers  of  this  work,  to  describe  a 
few  select  experiments,  by  which  the  equal  distribution  of  pressure, 
among  the  particles  of  an  incompressible  fluid  is  beautifully  and 
rigorously  demonstrated,  and  its  equal  propagation  in  all  directions, 
placed  beyond  the  possibility  of  the  smallest  doubt. 

Allied  to  the  preceding  subject,  is  the  following,  by  which  is  exhi- 
bited a  very  surprising  effect  of  the  equilibrium  of  incompressible 
fluids,  but  which,  for  the  sake  of  convenience,  we  shall  suppose  to  be 
water,  since  that  is  more  easily  obtained  in  small  or  large  quantities, 
than  any  other  fluid  whatever. 

EXPERIMENT  1.  Let  A  BCD  represent  an  upright  section  of  a  square 
or  cylindrical  vessel,  closed  at  top  with  a  cover  of 
which  A  D  is  a  section  ;  make  a  hole  in  the  top  at  E, 
and  fix  a  tube  FE  therein  of  any  convenient  dia- 
meter at  pleasure,  but  small  in  comparison  of  the 
diameter  of  the  vessel.  Let  the  tube  be  closely 
fixed  in  the  cover  with  pitch,  or  some  other  glutin- 
ous matter,  so  as  to  be  rendered  air  and  water-tight 
all  round  the  orifice,  and  suppose  its  length  or 
height  to  be  twelve  or  fifteen  inches  according  to 
circumstances  ;  then,  fill  the  vessel  with  water  by  some  holes  made 


156  THE  QU  AQUA  VERSUS  PRESSURE  OF  FLUIDS 

in  the  top  and  afterwards  stopped  up,  or  it  may  be  filled  through 
the  tube  alone. 

Now,  if  a  load  of  about  seven  or  eight  hundred  Ibs.,  be  laid  upon 
the  cover  of  the  vessel,  it  will  be  depressed  into  a  concavity  repre- 
sented by  the  dotted  line  AMD,  the  displaced  water  ascending  in  the 
tube,  in  proportion  as  the  cover  is  bent  by  the  pressure  of  the  super- 
incumbent load ;  but  if  we  pour  water  into  the  tube  FE,  the  cover  of 
the  vessel,  together  with  its  incumbent  load,  will  not  only  be  raised 
to  the  original  situation,  but  will  even  assume  a  convex  form,  as 
represented  by  the  dotted  line  AND,  rising  in  the  middle  as  much 
above  the  point  E,  as  it  was  formerly  depressed  below  it,  the  quantity 
of  elevation  being  measured  by  the  index  or  ruler  IL,  which  is  fixed 
in  an  adjoining  support  in  such  a  manner,  as  to  remain  immoveable, 
the  point  H  which  is  marked  on  the  tube,  ascending  or  descending 
with  the  cover  of  the  vessel. 

If  the  tube  be  increased  in  length  and  more  water  added,  it  will  be 
found  that  the  cover  of  the  vessel,  together  with  its  load,  will  rise 
higher  and  higher  until  a  rupture  takes  place  by  overstretching  the 
fibres  of  the  material ;  this  however,  is  a  case  not  admitted  in  the 
experiment,  and  consequently,  we  may  conclude,  that  the  small 
column  of  water  in  the  tube  : — 

Exerts  the  same  force  in  raising  the  cover  of  the  vessel, 
together  with  the  load  upon  it,  as  if  the  tube  and  the  vessel 
were  of  equal  diameters,  and  the  incumbent  column  equal  to 
AGHD,  instead  of  that  contained  in  the  tube. 

Now,  this  is  precisely  the  property  of  the  weighing  machine  for- 
merly exemplified  ;  for  the  water  in  the  small  tube  F  E,  will  raise  the 
cover  of  the  vessel,  (supposing  it  to  be  moveable  and  water-tight), 
together  with  its  load,  even  although  it  were  a  thousand  times  greater: 
this  is  manifest,  because  the  velocity  with  which  the  water  descends 
in  the  tube,  is  to  the  velocity  with  which  it  ascends  in  the  vessel,  as 
the  area  of  a  section  of  the  vessel,  is  to  the  area  of  a  corresponding 
section  of  the  tube ;  for  instance,  if  the  vessel  is  30  inches  in  diameter, 
while  the  supplying  tube  is  only  one ;  then,  we  know  by  the  prin- 
ciples of  mensuration,  that  the  area  of  the  top  of  the  vessel,  is  to  that 
of  a  section  of  the  tube,  in  the  ratio  of  900  to  unity ;  consequently, 
when  the  water  in  the  tube  has  descended  one  inch,  the  top  of  the 
vessel,  and  the  load  upon  it,  has  ascended  by  a  one  nine-hundredth 
part  of  an  inch ;  therefore,  if  the  water  in  the  tube  weighs  one  lb.,  it 
will  be  in  equilibrio  with  900  Ibs.  in  the  vessel,  or  which  is  the  same 


ILLUSTRATED  BY  EXPERIMENTS. 


157 


thing,  one  Ib.  of  water  in  the  tube,  will  suspend  the  top  of  the  vessel, 
together  with  the  load  upon  it,  supposing  them  to  weigh  conjointly 
900  Ibs. 

EXPERIMENT  2.  Let  mno  represent  a  vertical  section  of  a  spherical 
vessel  filled  with  water,  or  some  other  incompressible  and  non-elastic 
fluid,  and  let  AB  be  a  common 
tumbler  glass,  held  vertically  with 
its  mouth  exactly  in  contact  with 
the  fluid's  surface ;  then  it  is  mani- 
fest, that  in  this  state  the  glass  is 
completely  rilled  with  air  of  the 
natural  density ;  that  is,  with  air  of 
the  same  density  as  atmospheric  air 
on  the  surface  of  the  earth. 

If  the  tumbler  be  still  held  in  a 
vertical  position,  but  a  little  depressed  below  the  surface  of  the  fluid, 
as  represented  by  c  D,  then  it  is  obvious,  that  a  small  quantity  of  the 
fluid  has  entered,  and  the  rest  of  the  glass  is  filled  with  air  in  a  state 
of  slight  condensation,  corresponding  to  the  pressure  of  the  super- 
incumbent column  of  water  represented  by  T>d.  And  moreover,  if  the 
glass  be  still  farther  depressed,  the  fluid  will  ascend  higher  and  higher, 
and  the  air  will  be  compressed  into  a  less  and  less  space. 

Again,  if  the  glass  be  inclined  in  any  degree  from  the  vertical 
position,  as  represented  by  EF  and  GH,  taking  care  to  have  its  mouth 
wholly  immersed  in  the  water,  then  it  is  evident,  that  the  greater  the 
degree  of  inclination,  the  greater  is  the  quantity  of  fluid  which  enters, 
and  the  greater  also  is  the  condensation  of  the  included  air;  but  when 
the  quantity  of  fluid  which  enters  the  glass  is  the  same,  both  in  the 
vertical  and  the  inclined  position,  the  density  of  the  air  is  also  the 
same,  being  compressed  by  the  same  force ;  consequently,  the  water 
or  fluid  in  which  the  glass  is  placed,  exerts  the  same  pressure  in 
whatever  direction  it  is  propagated.  One  sees  this  experiment  verified 
daily  by  empty  casks  having  only  one  end,  thrown  into  water. 

EXPERIMENTS.  If  the  several  tubes  A,  B,  c,  D  A;B 
and  E,  bent  at  various  angles,  be  .inserted  in  an 
empty  vessel,  or  if  they  be  held  in  the  hand,  and 
mercury  be  introduced  at  their  lower  extremities, 
in  such  a  manner,  as  to  come  close  to  the  ori- 
fices ;  then  let  water  be  poured  into  the  vessel, 
and  it  will  be  seen,  that  during  the  time  of  its 
filling,  the  mercury  is  pressed  gradually  from  the 


158  THE  QUAQUAVERSUS  PRESSURE  OF  FLUIDS 

lower  towards  the  higher  extremities  of  the  tubes,  which  are  supposed 
to  rise  to  a  height  considerably  above  the  surface  of  the  water. 

Now,  since  the  lower  extremities  of  the  tubes  may  be  conceived  to 
point  in  every  possible  direction,  it  follows,  that  the  pressure  of  the 
superincumbent  fluid  is  also  propagated  in  every  direction.  But  when 
it  is  required  that  the  lower  orifice  should  point  directly  downwards, 
in  order  to  show  the  upward  pressure  of  fluids,  a  straight  tube  must 
be  employed,  and  the  mercury  which  is  introduced  must  be  kept  in 
by  the  finger,  until  the  height  of  the  water  above  the  lower  surface,  is 
about  fourteen  times  the  height  of  the  mercurial  column ;  for  if  the 
finger  be  removed  before  the  water  has  attained  that  height,  the  mer- 
cury will  fall  out  of  the  tube,  since  its  weight  is  fourteen  times  greater 
than  the  weight  of  an  equal  bulk  of  water.  If  the  finger  be  continued 
upon  the  orifice,  until  the  height  of  the  water  be  equal  to  fourteen 
times  the  height  of  the  mercury,  then,  on  removing  the  finger,  and 
pouring  in  more  water,  the  mercury  will  be  seen  to  ascend  in  the 
tube,  and  will  continue  to  rise  higher  and  higher,  according  to  the 
quantity  of  water  poured  in,  thereby  showing  the  upward  pressure  of 
the  water. 

EXPERIMENT  4.  The  pressure  of  fluids  at  different  points  of  their 
depths,  may  be  very  simply  illustrated 
in  the  following  manner :  let  K  be  a  bag 
of  leather,  or  some  other  tough  and 
flexible  material,  filled  with  mercury, 
and  attached  to  the  extremity  of  a 
glass  tube  zi,  in  such  a  manner,  that 
the  mercury  may  just  enter  the  tube 
when  the  bag  is  held  in  air. 

Then,  if  the  bag  be  immersed  in 
water,  it  is  manifest  that  the  pressure 
of  the  fluid  will  cause  it  to  collapse,  and  the  mercury  will  ascend  in 
the  tube  to  a  certain  height,  corresponding  to  the  pressure  exerted  by 
the  water,  at  the  depth  where  the  bag  is  placed.  If  the  bag  continue 
to  be  lowered  in  the  water,  it  will  become  more  and  more  collapsed  in 
consequence  of  the  increased  pressure,  and  the  mercury  will  ascend 
higher  and  higher  in  the  tube,  and  the  heights  to  which  it  rises,  will 
indicate  the  magnitude  of  pressure  at  different  depths. 

EXPERIMENT  5.  There  is  a  very  simple  and  amusing  experiment, 
by  which  the  propagation  of  pressure  through  fluids  is  illustrated, 
called  the  "  Cartesian  Devil"  from  M.  Descartes,  the  celebrated 
French  philosopher,  by  whom  it  was  discovered  ;  it  is  as  follows. 


ILLUSTRATED    BY    EXPERIMENTS.  159 

Let  the  little  figure  in  the  inverted  jar  AB  represent  the  "  Cartesian 
Devil,"  surmounted  by  a  bag-like  crown  of  great  size  in  proportion  to 
his  body,  filled  with  some  very  light  substance,  such  as  air,  and  we 
shall  therefore  suppose  that  air  is  the  body  which  it  contains.  The  imp 
himself  must  be  constructed  of  glass  or  enamel,  so  as  to  possess  the 
same  specific  gravity  as  water,  and  therefore  to  remain  suspended  in 
the  fluid.  A 

At  the  bottom  of  the  vessel  or  jar,  is 
placed  a  diaphragm  or  bladder,  that  can  be 
pressed  upwards  by  applying  the  finger  to 
the  extremity  of  a  lever  eo,  moving  round  o 
as  its  fulcrum  or  centre  of  motion.  The  pres- 
sure applied  at  a  is  communicated  through  the 
water  to  the  bag  of  air  at  m,  which  is  thus 
compressed,  and  consequently,  the  specific 
gravity  of  the  figure  is  increased,  by  which 
it  sinks  to  the  bottom  of  the  jar. 

By  removing   the  pressure   on  the  dia- 
phragm at  a,  the  figure  will  again  ascend, 
so  that  it  may  be  made  to  oscillate,  or  rise 
upwards  and  sink  downwards  alternately,  and  to  dance  about  in  the 
jar,  without  any  visible  cause  for  its  movements. 

Other  figures,  such  as  fishes  made  of  glass,  are  sometimes  employed 
in  this  experiment,  but  the  principle  is  nevertheless  the  same,  and 
when  a  common  jar  is  used,  the  pressure  is  applied  to  the  upper 
surface  of  it,  as  at  A. 

EXPERIMENT  6.  The  pressure  of  fluids  at  very  great  depths,  is 
beautifully  illustrated  by  an  experiment  which  has  often  been  made 
at  sea,  where  the  water  is  sufficiently  deep  to  admit  of  the  principle 
being  accurately  put  to  the  trial. 

The  experiment  is  this :  an  empty  bottle  well  corked  is  made  to 
descend  to  a  great  depth,  on  which  the  pressure  of  the  fluid  becomes 
so  great  as  to  drive  in  the  cork,  and  the  bottle  when  brought  up  is 
always  filled  with  water.  Several  methods  have  been  employed  to 
prevent  the  cork  from  being  driven  inwards,  but  although  this  has 
been  effected,  yet  the  bottle  on  being  brought  to  the  surface,  is  con- 
stantly filled  with  the  fluid  in  which  it  has  been  sunk. 

The  following  experiments  of  this  sort,  are  detailed  by  Mr.  Campbell, 
the  author  of  "  Travels  in  the  South  of  Africa,"  published  at  London 
in  the  year  1815;  the  experiments  were  tried  on  his  voyage  home- 
wards from  the  Cape  of  Good  Hope.  He  drove  very  tightly  into  an 


160  THE    QUAQUAVERSUS    PRESSURE    OF    FLUIDS. 

empty  bottle,  a  cork  of  such  a  large  size,  that  one  half  of  it  remained 
above  the  neck  :  a  cord  was  then  tied  round  the  cork  and  fastened  to 
the  neck  of  the  bottle,  and  a  coating  of  pitch  was  put  over  the  whole. 

When  the  bottle  was  let  down  to  the  depth  of  about  fifty  fathoms, 
he  perceived  by  the  additional  weight,  that  it  had  instantly  filled ; 
and  on  drawing  it  up,  the  cork  was  found  in  the  inside  of  the  bottle, 
which  of  course  was  filled  with  water. 

Another  bottle  was  prepared  in  a  similar  manner ;  but  in  order  to 
secure  the  cork,  and  to  prevent  it  from  being  pressed  within  the  bottle, 
a  sail  needle  was  passed  through  it,  so  as  just  to  rest  on  the  margin  of 
the  glass,  and  the  whole  was  carefully  covered  with  a  coating  of  pitch. 

When  the  bottle  had  descended  to  the  depth  of  about  fifty  fathoms, 
as  in  the  former  case,  it  was  again  perceived  to  have  been  filled  with 
water;  and  on  bringing  it  to  the  surface,  the  cork  and  needle  were 
found  in  the  same  position,  and  no  part  of  the  pitch  appeared  to  be 
broken,  although  the  bottle  was  completely  filled  with  water.  Here 
the  water  must  have  insinuated  itself  through  the  pores  of  the  pitch 
and  the  cork,  and  not  as  the  experimentalist  supposes,  through  the 
pores  of  the  glass. 

The  equality  of  fluid  pressure  in  every  direction,  is  very  easily 
demonstrated  in  the  following  manner. 

EXPERIMENT  7.  If  a  piece  of  very  soft  wax,  as  GUI,  and  the  egg 
E,  be  placed  in  a  bladder,  or  some  other  flexible  vessel  filled  with 
water,  and  if  the  bladder  be  put  into  a  brass  box,  and  a  moveable 
cover  laid  upon  the  bladder  so  as  to  be  wholly  supported  by  it. 

Then,  if  one  hundred,  or  one  hun- 
dred and  fifty  Ibs.  be  laid  upon  this 
cover,  so  as  to  press  upon  the  bladder 
and  its  contained  fluid ;  this  enor- 
mous force,  although  propagated 
throughout  the  fluid,  and  acting 
upon  the  soft  wax  and  the  egg,  will 
produce  no  effect,  the  wax  will  not 
change  its  form,  and  the  egg  will  not  be  broken.  And  in  like  manner, 
if  a  living  fish  should  be  put  into  the  cylinder  of  a  hydrostatic  press,  when 
under  a  very  high  degree  of  pressure,  it  will  not  suffer  the  least  incon- 
venience ;  from  which  it  is  obvious  that  every  particle  of  the  fluid  is 
equally  pressed,  and  presses  equally  in  all  directions. 

Numerous  other  examples  might  be  adduced  for  proving  the  same 
thing,  but  since  the  principle  is  manifest,  it  is  needless  to  dwell  longer 
on  the  subject. 


CHAPTER  VII. 

OF  PRESSURE  AS  IT  UNFOLDS  ITSELF  IN  THE  ACTION  OF  FLUIDS 
OF  VARIABLE  DENSITY,  OR  SUCH  AS  HAVE  THEIR  DENSITIES 
REGULATED  BY  CERTAIN  CONDITIONS  DEPENDENT  UPON  PAR- 
TICULAR LAWS,  WHETHER  EXCITED  BY  MOTION,  BY  MIXTURE, 
OR  BY  CHANGE  OF  TEMPERATURE. 


IN  the  former  part  of  this  treatise,  we  have  displayed  the  nature  of 
pressure  as  it  occurs  in  the  action  of  non-elastic  fluids  of  uniform 
density,  and  in  addition,  we  have  investigated  the  theory  and  exem- 
plified the  application  of  the  Hydrostatic  Press,  the  Hydrostatic 
Bellows,  and  the  Hydrostatic  Balance  or  weighing  machine ;  instru- 
ments whose  operations  depend  upon  the  quaqua-versus  principle  of 
non-elastic  and  incompressible  fluids: — We  come  therefore  in  the 
next  place,  to  consider  pressure  as  it  unfolds  itself  in  the  action  of 
fluids  of  variable  density,  or  such  as  have  their  densities  regulated  by 
certain  conditions,  dependent  upon  particular  laws,  whether  excited 
by  motion,  by  mixture,  or  by  change  of  temperature. 

In  mechanical  science  density  is  used  as  a  term  of  comparison, 
expressing  the  proportion  of  the  number  of  equal  moleculee  in  the 
same  bulk  of  another  body;  density,  therefore,  is  directly  as  the 
quantity  of  matter  ;  and  inversely  as  the  magnitude  of  the  body* 

We  cannot  by  means  of  our  senses  discover  the  figure  and  magnitude  of  the  ele* 
mentary  particles  of  matter.  Mechanical  inventions  have  wonderfully  magnified 
objects  invisible  to  the  unassisted  eye;  but  no  microscopical  assistance  has  yet  en- 
abled us  to  assume  that  we  have  seen  an  elementary  particle  of  matter.  A  number  of 
elementary  particles  uniting  bv  the  power  of  cohesion  form  greater  particles,  and 
these  again  uniting,  by  the  same  power,  form  still  greater;  and  we  may  consider  the 
aggregate  of  many  such  formations  to  become  at  length  an  atom  of  a  sensible  bulk. 
All  bodies  seem  to  be  composed  of  these  derivative  corpuscles,  which,  formed  of 
more  or  fewer  of  these  repeated  unions,  compose  bodies  more  or  less  dense.  These 
derivative  corpuscles  are  sometimes  similar,  as  the  coloured  rays  of  a  beam  of  light, 

VOL.    I.  M 


162  OF  THE  PRESSURE  OF  FLUIDS  OF  VARIABLE  DENSITY 

separated  by  the  prism ;  mercury,  when  squeezed  through  the  pores  of  leather,  or 
raised  in  fume  and  received  upon  clean  glass,  which  exhibits  globules  similar  and 
undistinguishable.  In  short,  every  mass  of  matter  is  divisible  into  particles,  which 
we  designate  by  the  Greek  term  atom,  or  that  which  is  so  exceedingly  minute  that 
it  cannot  be  further  cut  or  divided,  and  which  therefore,  as  far  as  sense  is  concerned, 
is  the  ultimate  resisting  particle.  It  must  be  obvious,  that  the  density  or  quantity 
of  atoms  which  exist  in  a  given  space  is  very  different  in  different  substances. 
Hence,  if  it  be  asked  why  bodies  are  called  dense  ?  the  answer  is,  Because  they 
contain  more  atoms  than  others  of  the  same  size.  There  are  more  atoms  in  a  cubic 
inch  of  lead  than  in  a  cubic  inch  of  cork  :  the  former  is  forty  times  heavier  than  the 
latter.  A  cubic  foot  of  rain  water  weighs  62£  Ibs. ;  but  an  equal  volume  of  mercury, 
which  is  fourteen  times  heavier  than  water,  weighs  (62^x14)  =875  Ibs. 

Density  must  depend  on  three  circumstances,  to  which  we  should  carefully  attend 
in  all  our  disquisitions :  first,  the  size  or  weight  of  the  individual  atom  ;  secondly,  on 
porosity,  or  the  arrangement  of  the  atoms  by  cohesion,  or  mechanical  and  physical 
arrangement ;  thirdly,  the  proximity  of  the  atoms  determined  by  the  substance  of 
which  they  are  constituent  particles,  possessing  tenacity  and  incompressibility. 
Thus,  heat  dilates  some  bodies  and  contracts  others.  A  pound  of  tin  and  a  pound  of 
copper  melted  together  form  bronze ;  but  this  new  mass  occupies  less  space  by  one 
fifteenth  than  the  two  masses  did  when  separate  j  proving  that  the  atoms  of  the  one 
are  partially  received  into  what  were  empty  spaces  of  the  other.  In  other  words, 
the  affinity  of  cohesion  is  one  fifteenth  greater  in  the  bronze  than  in  the  tin  and 
copper  separately.  Two  pounds  of  brine  are  made  out  of  a  pound  of  salt  and  a 
pound  of  water ;  but  the  mass  is  of  less  bulk  than  the  aggregate  of  the  ingredients 
apart. 

Water,  we  have  seen,  resists  compression  very  powerfully,  but  at  the  depth 
of  1000  fathoms  yielding  a  very  small  part  of  its  bulk  at  the  surface,  shows  the 
particles  not  to  be  in  contact,  and  that  the  fluid  may  acquire  density  in  propor- 
tion to  its  depth.  Wood  swims  in  water,  because  the  water  has  more  atoms  in  the 
same  bulk  than  the  wood,  and  therefore  more  weight  or  central  force  than  the  wood  ; 
consequently,  the  water  falls  first  and  leaves  the  wood  behind;  in  other  words,  the 
wood  floats  upon  the  water — the  wood  is  borne  on  the  surface  of  the  water  with  a  force 
exactly  proportional  to  the  difference  between  its  weight  and  that  of  an  equal  bulk 
of  water.  The  pressures  which  the  fluid  exerts  in  supporting  the  wood  are  together 
equivalent  to  a  force  directed  upwards  through  the  centre  of  gravity  of  the  fluid 
displaced,  and  equal  to  the  weight  of  a  quantity  of  the  fluid  so  displaced  by  the 
immersed  part  of  the  body.  But  it  is  not  necessary  here  to  dwell  further  on  this 
topic,  the  density  of  water.  We  therefore  pass  on  to  another  character  it  possesses, 
viz.  gravity  or  weight :  and  it  is,  in  fact,  by  comparing  the  weight  of  a  body  with 
the  force  which  holds  it  up  in  the  fluid,  that  the  comparative  weights  or  specific 
gravities  are  found,  as  of  metals  compared  with  water,  and  of  admixtures  of  metals 
for  the  purpose  of  ascertaining  at  once  the  proportion  of  each  in  the  compound 
mass. 

Water  is  the  common  standard  with  which  all  other  substances  are  compared, 
whose  weight  we  would  fix  and  record  in  tables  of  specific  gravities.  When 
we  say,  therefore,  that  gold  is  of  the  specific  gravity  of  19,  and  copper  of  9,  and 
cork  of  one  seventh,  we  mean  that  these  substances  are  just  so  much  heavier  or 
lighter  than  their  bulk  of  pure  water  in  its  densest  form,  viz.  at  the  temperature  of 
40  degrees  of  Fahrenheit's  thermometer.  It  appears,  therefore,  that  the  terms 


0fTHe  ^\ 
(   UNIVERSITY  J 

VSi^WX 

UPON  THE  SIDES  AND  BOTTOM  OF  A  CYLINDRICAL  VESSEL.          163 

density  and  specific  gravity  express  the  same  thing"  under  different  aspects ;  the 
former  being'  more  accurately  restrained  to  the  greater  or  less  vicinity  of  particles, 
the  latter  to  a  greater  or  less  weight  in  a  given  volume ;  hence,  as  weight  depends 
upon  the  closeness  of  the  particles,  the  density  varies  as  the  specific  gravity,  and 
the  terms  may  in  most  cases  be  indiscriminately  used. 

The  specific  gravities  of  fluids  are  usually  considered  without  any  regard  to  the 
empty  spaces  between  the  particles,  though  if  the  particles  of  fluids  are  spherical, 
the  vacuities  make  at  least  one  fourth  of  the  whole  bulk.  But  it  is  sufficient  that 
we  know  precisely  in  what  sense  the  specific  gravities  of  fluids  are  understood. 

PROBLEM  XXV. 

163.  A  cylindrical  vessel  whose  sides  are  perpendicular  to 
the  horizon,  has  a  certain  quantity  of  fluid  in  it;  which  fluid, 
by  reason  of  a  sudden  change  of  temperature,  has  its  magnitude 
or  bulk  increased  by  a  certain  part  of  itself : — 

It  is  therefore   required   to   determine  what   will    be  the 
alteration  of  pressure  on  the  sides  and  bottom  of  the  vessel. 

Let  ABCD,  and  abed  respectively,  represent  vertical  sections  of 
the  cylindrical  vessel,  of  which  the  sides  are  perpendicular  and  the 
base  parallel  to  the  horizon ;  then  in 
the  first  instance,  let  E  F  be  the  height 
to  which  the  vessel  is  filled,  and  ef 
the  height  to  which  the  fluid  rises, 
by  reason  of  the  change  that  takes 
place  in  the  temperature. 

Draw  the  diagonals  EC,   FD  and 
ec,  fd  intersecting  respectively   in 

the  points  G  and  g,  and  through  the  points  G,  g,  draw  the  Vertical  lines 
MN  and  mn  ;  then  are  MG  and  mg,  the  respective  depths  of  the  centres 
of  gravity  of  the  cylindric  surfaces,  in  contact  with  the  fluid  before 
and  after  the  expansion,  and  MN,  mn,  are  the  depths  of  the  centres 
of  gravity  of  the  bases  or  bottoms  D  c  and  dc. 

Through  the  points  G  and  g,  draw  the  straight  lines  GT  and  gs, 
parallel  to  the  horizon  and  to  one  another;  then  is  GS  or  rg  the 
height  which  the  centre  of  gravity  of  the  cylindric  surface  is  elevated, 
by  reason  of  the  expansion  of  the  fluid. 

Put  d  =  DC  or  dc,  the  diameter  of  the  cylindric  vessel, 

h  =  MN,  the  height  to  which  the  vessel  is  originally  filled, 
h'—mn,  the  height  at  which  the  fluid  stands  in  the  vessel  after 
expansion, 

M  2 


164  OF  THE  PRESSURE  OF  FLUIDS   OF  VARIABLE  DENSITY 

Put  P  zz:  the  pressure  on  the  bottom  DC,  by  the  fluid  in  its  original 

state, 

p  zz:  the  corresponding  pressure  on  the  cylindric  surface, 
P'zz:  the  pressure  on  the  bottom  dc,  by  the  fluid  after  expansion, 
p'  zz:  the  corresponding  pressure  on  the  cylindric  surface, 
s  zz:  the  specific  gravity  of  the  fluid  before  expansion, 
s'  z=  the  specific  gravity  of  the  fluid  after  expansion, 
a  zz:  the  area  of  the  base  or  bottom  of  the  vessel  in  both  cases  ; 

<j>  and  $'  the  cylindric  surfaces,  and 
£zz:  the  part  of  its  bulk  by  which  the  fluid  is  increased. 

Then,  since  d  denotes  the  diameter  of  the  bottom,  the  area  accord- 
ing to  the  principles  of  mensuration,  becomes 


and  the  pressure  exerted  by  the  fluid  in  its  original  state,  is 

(126). 


Again,  according  to  the  principles  of  mensuration,  the  cylindric 
surface  in  contact  with  the  fluid  before  expansion,  is 


and  consequently,  the  pressure  upon  it,  is 
/;zz:3.1416d/iXjAXszz:1.5708dA2s.  (127). 

Now,  it  is  manifest,  that  since  the  diameter  of  the  vessel  is  the 
same  both  before  and  after  the  expansion  of  the  fluid,  the  capacity 
and  the  altitude  must  vary  directly  as  each  other;  consequently, 
because  the  capacity  or  bulk  is  increased  by  ^th  part  of  itself,  it 
follows,  that  the  altitude  is  increased  in  the  same  proportion  ;  there- 
fore we  have 


but  when  the  weight  of  the  fluid  remains  the  same,  the  density,  and 
consequently  the  specific  gravity,  varies  inversely  as  the  magnitude. 

The  specific  gravity  of  the  fluid,  after  it  has  expanded  by  reason 
of  an  increase  of  temperature,  is  therefore, 

*J>±  ')  .ft  ........-»' 

n  n-j-l  ' 

hence,  the  pressure  on  the  bottom  of  the  vessel,  after  the  fluid  has 
increased  by  expansion,  becomes 

P'  =  .7864d«AV;  that  is, 


'Vv 


ON  SEMICIRCULAR  PLANES  IMMERSED  VERTICALLY.  165 


P'  —  .7854ef  X  h  -     X  --7  =  .7854d«  h  s.     (128). 

\     n      '       n  -f-  1 

The  cylindric  surface  in  contact  with  the  fluid  after  expansion,  may 
be  expressed  as  follows,  viz. 


but  it  has  been  shown  above,  that 

0  —  3.1416eM; 
therefore,  by  substitution,  we  obtain 


n 
and  consequently,  the  pressure  becomes 


If  therefore,  the  equations  (126)  and  (128)  be  compared  with  one 
another,  it  will  be  found  that  the  pressure  is  the  same,  and  equal  to 
the  weight  of  the  fluid  in  both  cases;  but  if  the  equations  (127)  and 
(129)  be  compared,  the  pressure  in  the  one  case,  is  to  that  in  the  other, 
in  the  ratio  of  n  :  n  +  1  ;  that  is, 

p  :  p'  :  :  n  :  n  -\-  I  . 


PROBLEM  XXVI. 

164.  A  semi-circular  plane  is  vertically  immersed  in  a  fluid 
whose  density  increases  as  the  depth,  and  in  such  a  manner, 
that  the  horizontal  diameter  coincides  with  the  upper  surface  of 
the  fluid  :— 

It  is  required  to  determine,  on  which  chord  parallel  to  the 
horizon,  the  pressure  is  a  maximum,  or  greater  than  the 
pressure  on  any  other  chord. 

Let  A  BCD  represent  a  vertical  section  of  a  mass  of  fluid,  of  which 
AB  is  the  surface,  and  whose  density 
varies  directly  as    its  depth  ;    and   let         jmr 
aGFHB  be  the  semi-circular  plane  im- 
mersed in  it,  in  such  a  manner,  that 
the   horizontal  diameter  a  b,  coincides 
with  AB  the  upper  surface. 

Let  m  be  the  point  in  the   vertical 


166  OF  THE  PRESSURE  OF  FLUIDS  OF  VARIABLE  DENSITY 

radius  EF,  through  which  the  chord  of  maximum  pressure  is  supposed 
to  pass;  draw  the  chord  GH,  and  the  radius  EG;  then,  because  the 
chord  GII  is  parallel  to  the  diameter  aby  it  follows,  that  GH  is  bisected 
in  m  by  the  vertical  radius  EF;  consequently,  m  is  the  place  of  the 
centre  of  gravity  of  the  chord  GH,  and  Em  is  its  perpendicular  depth 
below  AB,  the  upper  surface  of  the  fluid. 

Put  r  IZTEG,  the  radius  of  the  semi-circular  plane, 

0  =  GF,  half  the  arc  subtended  by  the  required  chord  GH, 
and  x  —  Em,  the  distance  of  the  chord  below  the  surface  of  the  fluid. 

Then,  because  by  the  conditions  of  the  problem,  the  density  of  the 
fluid  varies  directly  as  its  depth  ;  it  follows,  that  the  pressure  on  the 
chord  GH  varies  directly  as  Gra  drawn  into  Em2;  that  is, 


where  p  denotes  the  pressure  upon  the  chord  ;  but  this,  by  the  condi- 
tions of  the  problem,  is  to  be  a  maximum  ;  therefore,  we  have 

x*  \/  r2  —  #2  n:  a  maximum, 
from  which,  by  equating  the  fluxion  with  zero,  we  get 


or  by  transposing  and  expunging  the  common  factors,  we  obtain 

3z2  =  2r2; 
therefore,  by  division,  we  have 

z'zn-fr2, 
and  finally,  by  evolution,  it  becomes 

*  =  rV?.  (130). 

The  same  result,  however,  may  be  otherwise  determined  ;  for  by 
the  arithmetic  of  sines,  we  have,  to  radius  unity 

Gmzzsin.0,  and  Emzncos.0  ; 

but  in  order  to  accommodate  these  quantities  to  the  radius  r,  it  is 

Gmmr  sin.0,  and  Em2zziiK2z=:r2cos.2^; 

consequently,  by  multiplication,  we  obtain 


and  this,  by  the  conditions  of  the  problem,  is  to  be  a  maximum  ; 
hence  we  get 

r*sin.0  cos.2^  =  a  maximum, 
which  being  thrown  into  fluxions,  becomes 
0  —  r3(0-  cos.3<£  —  20-  sin.2^  cos.^)  ; 


ON  SEMICIRCULAR  PLANES.     THE  CHORD  OF  MAXIMUM  PRESSURE.     167 

therefore,  by  transposing  and  casting  out  the  common  terms,  it  is  * 

cos.20nz2sin.20. 

But  according  to  the  principles  of  Plane  Trigonometry,  we  have 
2sin.20  =  2  —  2cos.2<£;  consequently,  by  substitution,  the  above 
expression  becomes 

cos.20  zz  2  —  2  cos.20  ; 

therefore,  by  transposition  and  division,  we  obtain 

cos.20=rf, 
and  by  extracting  the  square  root,  we  get 

cos.0rz:-v/-| ; 

finally,  let  both  sides  of  this  expression  be  multiplied  by  r,  for  the 
purpose  of  adapting  it  to  the  proper  radius,  and  we  shall  have 

#  =  rcos.0:zrr|/!-,  the  same  as  above. 

165.  The  practical  rule  for  reducing  this  equation,  may  be  expressed 
in  words  at  length  in  the  following  manner. 

RULE.  Multiply  one  third  of  the  radius  of  the  given  semi- 
circular plane  by  the  square  root  of  6,  or  by  the  constant 
number  2.44947,  and  the  product  will  give  the  distance  of 
the  point  on  the  vertical  axis  below  the  surface  of  the  fluid, 
through  which  the  chord  of  maximum  pressure  passes. 

166.  EXAMPLE.  The  radius  of  a  semi-circular  plane,  immersed  in  a 
fluid  agreeably  to  the  conditions  of  the  problem,  is  27  inches ;  at 
what  distance  below  the  surface  of  the  fluid  must  a  horizontal  chord 
be  drawn,  so  that  the  pressure  which  it  sustains  may  be  greater  than 
the  pressure  sustained  by  any  other  chord  drawn  parallel  to  it? 

By  operating  according  to  the  rule,  we  shall  obtain 

x  —  9 ^6*=  9 X2.44947  =  22.04523  inches. 

167.  The  same  example  admits  of  a  very  simple  and  elegant  geo- 
metrical construction,  which  may  be  effected  in  the  following  manner. 

Let  ACS  be  the  semi-circular  plane,  of 
which  the  diameter  AB  is  parallel,  and  the 
radius  D  c  perpendicular  to  the  horizon  ;  draw 
the  chord  BC,  and  from  c  as  a  centre,  with 
the  radius  CD,  describe  the  circular  arc  DKH 
cutting  BC  produced  in  the  point  H. 

Through  the  point  c  draw  the  tangent  c  K, 
and  let  fall  the  perpendicular  H  i  meeting  c  K 
in  i;  join  DI  to  intersect  the  arc  A  EC  in  E,  and  through  the  point  E 


168  OF  THE  PRESSURE  OF  FLUIDS  OF  VARIABLE  DENSITY. 

draw  the  chord  EGF  parallel  to  AB  the  diameter  of  the  semicircle; 
then  is  EGF  the  chord,  on  which  the  pressure  is  a  maximum. 

That  the  line  DG  corresponds  with  x  in  the  equation  marked  (168), 
may  be  thus  demonstrated. 

By  reason  of  the  parallel  lines  AB  and  KC,  the  angles  ABII  and 
KCH  are  equal  to  one  another  ;  but  the  angle  ABH  is  manifestly  equal 
to  half  a  right  angle  or  45  degrees,  therefore,  the  angles  KCH  and 
CHI,  are  each  of  them  equal  to  half  a  right  angle,  and  the  lines  ci 
and  HI  are  equal,  being  respectively  the  sine  and  cosine  of  45  degrees 
to  the  radius  CH  or  CD. 

Now,  according  to  the  principles  of  Plane  Trigonometry,  the  sine 
and  cosine  of  45  degrees  to  the  radius  unity,  are  respectively  expressed 
by  \  V%  >  hence  we  have 

ci^JvC 

and  by  the  property  of  the  rightangled  triangle,  it  is 

Dizr-v/ci*-4-DC2=:r-v/|-, 
and  by  similar  triangles,  we  have 

r  V  i  :  r  '•  :'r  '•  D  G  -=-  r  V  $•• 

The  length  of  the  chord  line  EF  is  very  easily  found,  for  by  reason 
of  the  right  angled  triangle  EDG,  of  which  the  two  sides  DE  and  DG 
are  known,  it  is 

E  G2  ~  D  E?  —  D  G2 ; 

but  by  the  elements  of  geometry,  the  square  of  a  line  is  equal  to  four 
times  the  square  of  its  half,  therefore,  we  have 

EF2H=4(DE2  —  DG2); 

hence,  by  extracting  the  square  root,  we  get 

E  F  Zr  2  ^  D  E2 D  G2  J 

now  D  L2 1=1  r*,  and  DG*I=:  f  r9 ;  therefore  it  is 

EF=z|rV~3.  (131). 

Wherefore,  if  we  take  the  radius  of  the  semi-circle  equal  to  27 
inches,  as  in  the  preceding  example,  the  whole  length  of  the  chord 
will  be  18X1. 7321=31. 176inches. 

PROBLEM  XXVII. 

168.  If  a  given  conical  vessel  be  filled  with  fluid,  and  sup- 
ported with  its  axis  inclined  to  the  horizon  at  a  given  angle : — 

It  is  required  to  determine,  on  vjhat  section  parallel  to  the 
base  the  pressure  is  a  maximum. 


MAXIMUM  PRESSURE  ON   A  SECTION   OF  A  CONICAL  VESSEL.  169 

Let  ABC  be  a  section  passing  along  the  axis  of  the  conical  vessel, 
of  which  c  is  the  vertex,  and  AB  the  diameter  of  its  base. 

Conceive  AI  to  be  horizontal,  and  produce 
the  axis  CD  to  meet  the  horizontal  line  AI  in 
the  point  i  ;  then  is  A  ic  the  angle  of  inclina- 
tion between  the  axis  and  the  horizontal  line 
AI. 

Let  G  be  the  point  in  the  axis  through  which 
the  plane  of  the  required  section  is  supposed 
to  pass,  arid  through  G  draw  the  straight  line 

EF  parallel  to  AB,  and  GH  perpendicular  to  AI;  then  is  EF  the  dia- 
meter of  the  section,  and  GH  the  perpendicular  depth  of  its  centre  of 
gravity  below  A,  the  highest  particle  of  the  fluid. 

Put  Rizr  AD,  the  radius  of  the  base  of  the  conical  vessel, 
H  —  c  i),  the  axis  or  height, 
r  ZZTEG,  the  radius  of  the  section  on  which  the  pressure  is  a 

maximum, 

a  ~  the  area  of  the  section, 

d  ~  GH,  the  perpendicular  depth  of  its  centre  of  gravity, 
p  —  the  pressure  perpendicular  to  its  surface, 
0  zr:  A  ic,  the  angle  of  inclination  between  the  axis  of  the  cone 

and  the  horizon, 
x  zr  CG,  the  distance  between  the  section  and  the  vertex  of  the 

cone, 
and  s  z=  the  specific  gravity  of  the  fluid. 

Then,  because  of  the  rightangled  triangle  ADI,  and  from  the  prin- 
ciples of  Plane  Trigonometry,  we  have 

R  :  DI  :  :  tan.0  :  1, 
and  from  this,  we  obtain 


consequently,  by  adding  the  axis,  we  get 

cizz  R  cot.<j>  -f-H, 
and  again  by  subtraction,  it  is 

G  i  ~  R  cot.p  4"  H  —  x. 

But  the  triangle  GHI,  is  by  construction  right  angled  at  H  ;  there- 
fore, by  Plane  Trigonometry,  we  have 

GHmc?zz:{Rcot.^  -j~  H  —  a?}sin.0. 

Again,  the  triangles  CD  A   and  CGE  are  similar  to  one   another; 
therefore,  by  the  property  of  similar  triangles,  we  have 


170      OF  THE  PRESSURE  OF  FLUIDS  OF  VARIABLE  DENSITY 

H2  :  R2  :  :  x*  :  r2  ; 
or  from  this,  by  equating  and  dividing,  we  get 

r*-^. 

~    H2     ' 

consequently,  by  the  principles  of  mensuration,  the  area  of  the  section, 
on  which  the  pressure  is  proposed  to  be  a  maximum,  becomes 

3.1416R2*2 
-rf  --  ; 

therefore,  the  pressure  upon  its  surface  is 
3.1416Ra*2s 


Now,  because  the  whole  of  the  quantities  which  enter  this  equa- 
tion are  constant,  excepting  x,  and  the  bracketted  expression 
{Rcot.^-f  H  —  x}  which  is  affected  by  it;  it  follows,  that  the  value 
of  p  varies  as  rc2{RCOt.0-|-  H  —  x},  and  consequently,  is  a  maximum, 
when  the  quantity  which  limits  its  variation  is  a  maximum  ;  hence 
we  have 

a?2  {R  cot.^  -|-  H  —  x}  zz  a  maximum. 

Let  the  above  expression  for  the  maximum  be  thrown  into  fluxions, 
and  we  shall  obtain 

2  (R  cot.0  -f-  H)  xx  —  3a?ai  •=.  0  ; 

therefore,  by  transposing  and  expunging  the  common  quantities,  we 
get 

3a?i=2(RCot.0+  H), 
and  finally,  by  division,  we  obtain 

a?  =  |-(RCOt.^  +  H).  (132). 

169.  The  practical  rule  for  reducing  this  equation,  may  be  expressed 
in  words  at  length  in  the  following  manner. 

RULE.  Multiply  the  natural  cotangent  of  the  angle  which 
the  axis  of  the  cone  makes  with  the  horizon,  by  the  radius  of 
the  vessel's  base,  and  to  the  product  add  the  altitude  or  axis 
of  the  cone  ;  then,  two  thirds  of  the  sum  will  give  the  distance 
of  the  section,  on  which  the  pressure  is  a  maximum,  from  the 
vertex  of  the  cone. 

170.  EXAMPLE.  A  conical  vessel  whose  altitude  is  20  inches,  and 
the  radius  of  its  base  8  inches,  is  filled  with  fluid  and  so  inclined,  that 
its  axis  makes  with  the  horizontal  line  passing  through  the  extremity 
of  the  diameter  of  its  base,  an  angle  of  48  degrees  ;  on  what  section, 
parallel  to  the  base  is  the  pressure  a  maximum  ? 


ON  A  GLOBULAR  BODY  OF  CONDENSIBLE  AND  ELASTIC  MATTER.    171 

Here  we  have  given,  RZH  8  inches,  H  m  20  inches,  and  0m  48 
degrees,  of  which  the  natural  cotangent  is  0.9004  very  nearly;  con- 
sequently, by  the  rule,  we  have 

x  =  $ {8X0.9004  +  20}  =18.1355  inches. 

If  therefore,  18.1355  inches  be  set  off  from  the  vertex,  a  straight 
line  drawn  through  that  point  parallel  to  the  base,  will  be  the  diameter 
of  the  section  on  which  the  pressure  is  a  maximum. 

171.  In  any  fluid  the  particles  towards  its  base  support  those  that 
are  immediately  above  ;  these  again  bear  the  load  above  them,  and 
so  on  to  the    surface,   where  the  whole   mass   supports  the  super- 
incumbent atmosphere.     There  is    therefore   a   pressure  among  the 
successive  strata  of  an  homogeneous  fluid  increasing  in  exact  propor- 
tion to  the  perpendicular  depth.     Hence  a  bubble  of  air  or  of  steam, 
set  at  liberty  far  below  the  surface  of  water,  is  small  at  first,  and 
gradually  enlarges  as   it  rises.     This  phenomenon  shows   that   the 
compressive  power  of  the  fluid  slackens  by  ascent.     Experiment  and 
calculation  most  readily  demonstrate  the  compressibility  of  water : 
and  the  next  problem  exhibits  the  striking  effects  from  the  increase  of 
pressure  at  great  depths  of  the  sea. 

PROBLEM  XXVIII. 

172.  A  globular  body  of  condensible  and  elastic  matter,  is 
suffered  to  ascend  vertically  from  the  bottom  to  the  surface 
of  the  sea  : — 

It  is  required  to  determine  its  diameter  at  the  surface,  the 
depth  of  the  sea,  and  the  diameter  at  the  bottom  being  given. 

Let  AB  be  the  surface  of  the  sea,  ab  its  bottom,  and  ecf,  ECF,  two 
positions  of  the  globular  body  in  its  ascent 
from  the  bottom  to  the  surface,  and  A  sea, 
B  vfb  the  curves  described  by  the  extremities 
of  the  diameter. 

Through  G  and  g,  the  centre  of  the  globe 
in  the  two  positions,  draw  the  vertical  line 
cc,  which  is  manifestly  the  abscisse  to  the 
curve,  the  radii  ge  and  GE,  as  well  as  ca  and 
CA,  being  ordinates. 

Produce  the  abscisse  cc  to  D,  and  make  CD  equal  to  the  height  of 
a  column  of  sea  water,  which  would  be  in  equilibrio  with  the  pressure 


172  OF  THE  PRESSURE  OF  FLUIDS  OF  VARIABLE  DENSITY 

of  the  atmosphere  ;  through  the  point  D  draw  the  straight  line  11  1 
parallel  to  the  surface  of  the  sea,  and  HI  will  manifestly  be  the 
asymptote  to  the  curve  described  by  the  extremities  of  the  diameter. 

Put  r=ac,  the  radius  of  the  globe  at  the  bottom  of  the  water, 
d  —  cc,  the  depth  of  the  water  at  the  place  of  immersion, 
7i  —  CD,  the  height  of  a  column  of  water  equal  to  the  weight  of 

the  atmosphere, 
x  rr  CG,  any  abscissa, 
y  nz  G  E,  the  corresponding  ordinate,  or  radius  of  the  globe. 

Then,  because  the  magnitude  of  the  globular  body,  is  inversely  as 
the  density,  (the  weight  and  the  quantity  of  matter  remaining  the 
same,)  and  the  density  is  directly  as  the  pressure  ;  it  follows,  that  the 
magnitude  of  the  body  at  different  points  of  its  ascent,  is  inversely  as 
the  pressure  at  those  points,  and  the  pressure  is  directly  as  the  depth  ; 
therefore,  we  have 

DC  :  DG  :  :  GES  :  ca8; 
but  according  to  the  foregoing  notation,  we  have 

d+h  :  h  +  x::  y*  :  r3  ; 

from  which,  by  equating  the  products  of  the  extremes  and  means, 
we  get 


hence,  by  division,  we  obtain 


= 


(*+*) 

and  by  extracting  the  cube  root,  it  is 
,/(d  +  h) 

ry   (T+T)'  (133). 

The  equation  in  its  present  form,  exhibits  the  nature  of  the  curve 
described  by  the  diameter  of  the  body  during  its  ascent;  or  it  ex- 
presses generally,  the  value  of  the  ordinate  or  radius  corresponding 
to  any  depth ;  but  in  order  to  determine  the  radius  at  the  surface, 
which  is  the  primary  demand  of  the  problem,  we  must  suppose  the 
quantity  x  to  vanish,  in  which  case,  the  above  equation  becomes 

d+h 


h  (134). 

173.  The  practical  rule  supplied  by,  or  derived  from  this  equation, 
may  be  expressed  in  words  at  length  in  the  following  manner. 


ON  A  GLOBULAR  BODY  OF  CONDENSIBLE  AND  ELASTIC  MATTER.       173 

RULE.  To  the  given  depth  of  the  sea,  add  the  height  of  a 
column  of  sea  water,  which  is  equal  to  the  weight  or  pressure 
of  the  atmosphere  ;  divide  the  sum  by  the  height  of  the  atmo- 
spheric column,  and  multiply  the  radius  of  the  body  at  the 
bottom  of  the  sea  by  the  cube  root  of  the  quotient,  and  the 
product  will  give  the  radius  at  the  surface. 

174.  EXAMPLE.  The  radius  of  a  globe  of  elastic  arid  condensible 
matter,  when  placed  at  the  depth  of  75  fathoms  in  sea  water,  is  equal 
to  4  inches ;  what  will  be  the  radius  on  ascending  to  the  surface,  the 
atmospheric  column  being  equal  to  33  feet  ? 

Here  we  have  given  d~  75  fathoms,  or  75  x  6  m  450  feet ;  h  zz  33 
feet;  fizz 4  inches;  consequently,  by  the  rule,  we  have 


=v 


-—  izz  9. 785  inches  nearly. 


33 

175.  From  this  it  appears,  that  if  a  globe  of  condensible  matter, 
whose  radius  is  9.785  inches,  be  immersed  in  the  sea  to  the  depth  of 
450  feet,  its  radius  will  be  decreased  to  4  inches ;  this  circumstance 
may  suggest  some  easy  and  accurate  methods  of  determining  the  depth 
of  the  ocean,  when  it  is  so  great  as  to  preclude  the  application  of 
other  methods. 

176.  In  order,  however,  to  adapt  our  equation  to  the  determination 
of  the  depth,  we  must  consider  the  radii  at  the  surface  and  at  the 
bottom,  together  with  the  height  of  the  atmospheric  column,  to  be 
accurately  known  at  the  time  of  trial ;  then,  by  a  very  obvious  trans- 
formation, the  depth  of  descent  may  be  ascertained  ;   for  let  R  be 
substituted  instead  of  y  in  the  foregoing  equation,  to  denote  the  radius 
at  the  surface,  and  we  shall  have 

R— rV  ~T~~' 

in  which  equation,  d  is  the  unknown  quantity. 
Let  both  sides  of  the  equation  be  divided  by  r,  the  radius  of  the 
globe  at  the  bottom  of  the  sea,  and  we  shall  obtain 

(d+h), 
~T~~' 
and  cubing  both  sides,  it  becomes 


multiply  by  h,  and  we  obtain 


174  OF  THE  PRESSURE  OF  UNMIXABLE  FLUIDS  OF  DIFFERENT  DENSITIES, 

**=-  +  *. 

and  finally,  by  transposition,  we  have 


r3  (135). 

177.  The  practical  rule  for  reducing  the  above  equation,  may  be 
expressed  in  words  at  length  in  the  following  manner. 

RULE.  Multiply  the  difference  of  the  cubes  of  the  radii) 
by  the  height  of  the  atmospheric  column,  and  divide  the 
product  by  the  cube  of  the  lesser  radius  for  the  depth 
required. 

EXAMPLE.  The  radius  of  a  globe  of  condensible  matter  is  10  inches 
before  immersion,  and  it  is  suffered  to  descend  so  far  as  to  have  its 
radius  diminished  to  3  inches  ;  required  the  depth  of  descent,  the 
atmospheric  column  at  the  time  of  the  experiment  being  equivalent 
to  33  feet. 

Here  we  have  given  R  —  10  inches,  r  —  3  inches,  and  h  —  33  feet  ; 
therefore,  by  proceeding  according  to  the  rule,  we  have 
Rs_  r8__  10QO  _27  —  973  ; 

consequently,  multiplying  by  33  feet,  we  obtain 

973x33  —  32109, 
therefore,  by  division,  it  is 


PROBLEM  XXIX. 

178.  Let  a  vessel  of  any  form  whatever,  whose  base  is  hori- 
zontal, be  filled  with  fluids  of  different  densities  which  do  not 

mix  : — 

It  is  required  to  determine  the  pressure  on  the  bottom  of  the 
vessel,  supposing  the  fluids  to  succeed  each  other  in  the  order 
of  their  densities. 

Let  ABGH  represent  a  vertical  section  of  the 
vessel,  containing  fluids  of  different  densities  or 
specific  gravities,  as  indicated  by  the  shading  of 
the  several  strata  AC,  DF  and  EG;  and  for  the 
sake  of  simplicity  of  investigation,  let  the  bottom 
HG  be  parallel,  and  the  sides  AH,  BG  perpendicular 


ON   THE  BOTTOM  OF  ANY  VESSEL.  175 

to  the  horizon.  Then  are  AB,  DC  and  EF,  the  respective  surfaces  of 
the  several  fluids,  as  mercury,  water,  and  olive  oil,  also  parallel  to 
the  horizon  ;  for,  as  we  have  elsewhere  stated  :  — 

The  common  surface  of  two  fluids  which  do  not  mix,  is 
parallel  to  the  horizon. 

Now,  it  is  manifest,  (since  the  sides  A  H  and  B  G  are  perpendicular 
to  the  base  HG),  that  the  pressure  upon  the  base  HG,  is  equal  to  the 
pressures  or  weights  of  the  several  fluids  contained  in  the  vessel  ; 
therefore 

Put  d  zz  EH,  the  perpendicular  depth  of  the  lowest  stratum  EG, 
d'  •=.  DE,  the  perpendicular  depth  of  the  middle  stratum  DF, 
d"=:  AD,  the  perpendicular  depth  of  the  upper  stratum  AC, 
p  zz  the  pressure  of  the  stratum  E  G  upon  the  line  H  G, 
jt/zz  the  pressure  of  the  stratum  DF  upon  the  line  EF, 
p"—  the  pressure  of  the  stratum  A  c  upon  the  line  D  c  ;  and  let  s, 

s'  and  s"  denote  the  specific  gravities  of  the  respective 

fluids. 

Then,  since  the  pressure  upon  any  surface,  is  equal  to  the  area  of 
that  surface,  drawn  into  the  perpendicular  depth  of  its  centre  of 
gravity;  it  follows,  that  the  pressure  upon  HG,  occasioned  by  the 
fluid  in  EG,  is 

^ZZTHGX^S, 
and  in  like  manner,  the  pressure  upon  EF,  is 

j/  =  EFXdV, 
and  lastly,  the  pressure  upon  D  c,  is 


But  the  total  pressure  upon  H  G,  is  manifestly  equal  to  the  sum  of 
these  pressures  ;  therefore,  if  P  denote  the  entire  pressure  on  the  line 
H  G,  we  have 
P— 


but  the  lines  HG,  EF  and  DC,  are  equal  among  themselves,  therefore 
we  get 

P  =  HG  (ds  +  d's'  -f-  d"8u).  (136). 

179.  In  the  preceding  investigation,  we  have  considered  three  fluids 
of  different  densities  to  be  contained  in  the  vessel;  but  the  same 
mode  of  procedure  will  extend  to  any  number  whatever,  and  what  we 
have  done  respecting  three  fluids  is  sufficient  to  discover  the  law  of 
induction  for  any  other  number.  It  is  this  :  — 


176  OF  THE  PRESSURE  OF  UNMIXABLE  FLUIDS  OF  DIFFERENT  DENSITIES. 

The  perpendicular  pressure  upon  the  horizontal  base  of  a  vessel 
containing  any  number  of  fluids  of  different  densities,  which  do  not 
mix  in  the  vessel  :  — 

Is  equal  to  the  area  of  the  base,  multiplied  by  the  sum  of 
the  products  of  the  specific  gravities  drawn  into  the  altitudes 
of  the  several  fluids  . 

But  the  pressure  upon  the  base,  will  manifestly  be  the  same,  if  we 
suppose  the  vessel  to  be  filled  with  a  fluid  of  uniform  density,  arising 
from  the  composition  of  the  densities  of  the  several  fluids  according  to 
their  magnitudes  ;  or  if  the  magnitudes  are  equal,  the  uniform  density 
will  be  a  medium  between  the  several  given  densities. 

180.  EXAMPLE.  A  cylindrical  vessel,  whose  diameter  is  6  and  alti- 
tude 24  inches,  is  filled  with  mercury,  water  and  olive  oil,  in  the 
following  proportions,  viz.  mercury  7,  water  8,  and  olive  oil  9  inches; 
what  is  the  pressure  on  the  bottom  of  the  vessel,  the  specific  gravities 
being  13598,  1000  and  915  respectively? 

Here,  by  the  principles  of  mensuration,  the  area  of  the  bottom  of 
the  vessel  containing  the  fluids,  is 

36  X  .7854  —  28.2744  square  inches  ; 
consequently,  the  pressure  produced  by  the  mercury,  is 

;?  =  28.2744x7X13598  =  2691327.0384, 
and  in  like  manner,  the  pressure  of  the  water,  is 

p1  =  28.2744  X8  X  1000  =  226195.2, 
and  lastly,  the  pressure  produced  by  the  oil,  is 

p"~  28.2744x9x915  =  232839.684  ; 

and  the  sum  of  these  is  manifestly  the  whole  pressure  ;  hence  we  get 
P  =.  2691327.0384  +  226195.2  +  232839.684  z=z  3150361  .9224. 
If  the  pressure  as  here  expressed  be  divided  by  1728,  the  number 
of  solid  inches  in  a  cubic  foot,  we  shall  have 


181.  Again,  suppose  the  dimensions  of  the  vessel  to  remain  as 
above,  and  let  it  be  filled  with  the  same  fluids  in  equal  quantities  ; 
that  is,  8  inches  of  mercury,  8  of  water,  and  8  of  olive  oil  ;  what  then 
is  the  pressure  upon  the  bottom  ? 

Here,  by  proceeding  as  above,  we  have  for  mercury, 
p  =  28.2744  X  8  X  1  3598  =  3075802,3296  ; 


ON  THE  CONCAVE  SURFACE  OF  A  VESSEL.  177 

for  water,  it  is 
p'  zz  28.2744X  8  X  1000  —  226195.2, 

and  for  olive  oil,  it  is 

p"  zz  28.2744  X  8  X915  zz  206968.  608  ; 

hence  by  summation,  the  entire  pressure  on  the  bottom,  is 

P  zz  3075802.3296  -f  226195.2  4.  206968.608  zz  3508966.1376, 

and  lastly,  dividing  by  1728,  we  obtain 


172o 

The  pressure  which  we  have  found  in  this  last  instance,  is  the  very 
same  as  that  which  would  arise,  if  the  vessel  were  filled  with  fluid  of 
a  medium  density  ;  for  we  have 

i  (13598  4-  1000  +  915)  zz  5171  medium  density  ; 

hence,  the  entire  pressure  on  the  bottom,  is 

P  —  28.2744  X  24X5171—  3508966.1376; 

which  by  division,  gives 

3508966.1376 
P—  -  r^rr  -  zz  2030.6517  ounces,  the  same  as  before. 


182.  Let  the  conditions  of  the  problem  remain  as  above,  and  let  it 
be  required  to  determine  the  pressure  on  the  concave  surface  of  the 
vessel,  and  to  compare  it  with  that  upon  the  bottom. 

Let  ABGH,  as  in  the  preceding  case,  represent  an  upright  section 
of  the  vessel,  of  which  the  base  HG  is  parallel,  and 
the  sides  AH,  BG  perpendicular  to  the  horizon; 
and  suppose  the  fluids  of  different  densities  to  be 
contained  in  the  strata  AC,  DF  and  EG. 

Bisect  the  surface  and  base  A  B  and  HG,  in  the  ™ 
points  m  and  n,  and  join  mn  ;  then  do  the  centres 
of  gravity  of  the  several  cylindric  surfaces  occur  in 
that  line.     Draw  the  diagonals  AC,   DF  and  EG, 
cutting  the  vertical  line  mn  in  the  points  c,  b  and  a,  which  mark 
the  places  of  the  respective  centres  of  gravity. 

Put  D  zz  AB  or  HG,  the  diameter  of  the  vessel  containing  the  fluids, 
\d  zz  ea,  the  depth  of  the  centre  of  gravity  of  the  lower  cylindric 

surface, 

\d'  zz  db,  the  depth  of  the  middle  cylindric  surface, 
£<i"zz  me,  the  depth  of  the  upper  cylindric  surface  ;  each  of  these 

being  referred  to  the  surface  of  the  respective  fluid. 
VOL.  i.  N 


178  OF  THE  PRESSURE  OF  UNMIXABLE  FLUIDS  OF  DIFFERENT  DENSITIES 


Then,  the  pressures  and  the  specific  gravities   being  denoted  as 

before,  by  the  letters  p,p',p",  and  s,  s',  s",  we  shall  obtain  as  follows. 

According  to  the  principles  of  mensuration,  the  circumference  of 

the  vessel  is  expressed  by  3.  1416  D;  consequently  the  several  surfaces, 

estimated  in  order  upwards,  are 

3.1416Drf;   3.1416Dd',  and  3.1416D<f  ; 

and  the  corresponding  pressures,  are 
j?=1.5708Dd2*;  p'  =  1.5708  D<T  s1,  and  p"  =1  1.5708  D  d"2  s"  ; 

and  the  total  pressure  is 

P'  z=  (p  4-p'  4-p")  =  1  .5708  D  (d*s  4-  d'*s'  4-  dns").      (137). 
But  the  area  of  the  base  is  expressed  by  .7854  D8;  consequently, 
the  equation  numbered  (136)  becomes 

P  =  .7854  D9  (d  s  4-  d's'  4-  d"s"), 

and  the  pressures  on  the  base,  and  on  the  upright  surface  of  the  vessel, 
are  to  one  another  as  D  (ds  4-  d's'  4-  d"  s")  :  2  (d*s  4-  d'*s'+  d"*s". 


PROPOSITION  III. 

183.  If  two  fluids  of  different  densities  or  specific  gravities, 
communicate  with  one  another  through  a  bent  tube  or  otherwise, 
and  remain  in  a  state  of  equilibrium : — 

The  perpendicular  altitudes  of  these  fluids  above  their  com- 
mon surface,  will  vary  inversely  as  their  specific  gravities. 

Let  ABCD  be  a  tube,  through  the  bent  arms  of  which,  two  fluids  of 
different  specific  gravities,  communicate 
with  one  another  in  the  common  surface 
a  b,  and  suppose  the  horizontal  plane  E  F 
to  pass  through  the  surface  of  communi- 
cation. 

Take  G  the  centre  of  gravity  of  the 
plane  EF,  and  through  o  draw  the  vertical 
line  GW,  meeting  Am  and  DTI  respectively 

in  the  points  m  and  n  ;  then,  because  the  lines  Am  and  DTI  are  parallel 
to  the  horizon,  TKG  and  no  are  the  perpendicular  depths  of  the  plane 
EF,  below  the  surfaces  of  the  fluids  at  A  and  D. 

Now  it  is  manifest,  that  since  the  part  of  the  plane  ab,  which  is 
contiguous  to  the  common  surface  of  the  fluids,  is  sustained  in  its 
place  by  the  downward  pressure  of  the  lighter  fluid  in  A  b,  and  by  the 


ON  THE  MERCURIAL  COLUMN    EQUILIBRATING  ONE  OF  WATER.        179 

upward  pressure  of  the  heavier  fluid  in  DCB;  it  follows,  that  these 
pressures  are  equal  to  one  another.  But  because  the  plane  EF  is 
parallel  to  the  horizon,  the  pressure  of  the  fluid  in  £B,  is  equal  and 
opposite  to  the  pressure  in  cc,  the  fluid  in  de  serving  no  other  pur- 
pose than  for  mutually  transmitting  the  opposing  pressures ;  conse- 
quently, the  pressure  of  the  lighter  fluid  in  A  by  is  counterpoised  by 
the  pressure  of  the  heavier  fluid  in  DC,  the  fluid  in  CCB&  serving  only 
as  a  medium  of  communication. 

Put  dinwG,  the  perpendicular  depth  of  the  plane  EF,  below  the 
surface  of  the  lighter  fluid  at  A, 

3  in  w G,  the  perpendicular  depth  of  the  plane  EF,  below  the 
surface  of  the  heavier  fluid  at  D, 

p  in  the  pressure  on  the  plane,  occasioned  by  the  lighter  fluid 
in  A  5, 

7/zz:  the  pressure  on  the  plane,  occasioned  by  the  heavier  fluid  in 
DC  ;  and  let  s  and  s'  represent  the  corresponding  spe- 
cific gravities  of  the  lighter  and  the  heavier  fluids. 

Then,  since  the  pressures  on  the  plane,  occasioned  by  the  actions 
of  the  two  fluids,  are  respectively  as  the  depths  of  the  centre  of 
gravity,  and  the  specific  gravity  of  the  fluids  jointly  ;  it  follows,  that 

p  :  p'  :  :  ds  :  Ss' ; 

but  according  to  the  conditions  of  the  proposition,  these  pressures  are 
equal  to  one  another ;  hence  we  have 

ds  =  Ss';  (138). 

and  by  converting  this  equation  into  an  analogy,  it  becomes 

d  :  $  :  :  sf  :  s  • 

hence,  the  truth  of  the  proposition  is  rendered  manifest. 
Let  both  sides  of  the  equation  numbered  (138),  be  divided  by  s,  the 
specific  gravity  of  the  lighter  fluid,  and  we  shall  obtain 

,7-!fl 

•    s  *  (139). 

184.  Hence,  in  order  to  determine  the  perpendicular  depth,  or 
altitude  of  a  column  of  the  lighter  fluid,  that  will  balance  or  keep  in 
equilibrio  a  given  column  of  the  heavier ;  we  must  observe  the  follow- 
ing practical  rule. 

RULE.  Multiply  the  altitude  of  the  heavier  fluid  by  its 
specific  gravity,  and  divide  the  product  by  the  specific  gravity 
of  the  lighter  fluid,  for  the  altitude  sought. 

N2 


180  OF  THE  PRESSURE  OF  UNMIX  ABLE  FLUIDS  OF  DIFFERENT  DENSITIES 

185.  EXAMPLE.  The  height  of  a  column  of  mercury  is  30  inches, 
or  2  1  feet,  and  its  specific  gravity  13598  ounces  per  cubic  foot;  what 
is  the  height  of  the  equilibrating  column  of  water,  its  specific  gravity 
being  1000  ounces  per  cubic  foot? 

The  operation  performed  according  to  the  rule,  is  as  below. 


186.  A  column  of  mercury  30  inches  or  2|  feet  in  perpendicular 
height,  is  found  to  equiponderate  with  the  atmosphere  in  a  medium 
state  of  temperature;  consequently,  a  column  of  water  33.995  or  34 
feet  nearly  in  perpendicular  height,  will  produce  the  same  effect  ;  it 
is  therefore  manifest,  that  water  will  ascend  in  a  vacuum  tube,  to  the 
height  of  about  34  feet,  by  means  of  the  pressure  of  the  atmosphere  ; 
and  on  this  principle  depends  the  operation  of  the  sucking  pump, 
to  which  we  shall  have  occasion  to  advert  in  another  place. 

187.  If  in  the  equation  numbered  (139),  the  specific  gravities  are 
equal  to  one  another,  that  is,  if  szr  s',  then  £/  —  o,  or  the  perpendi- 
cular altitudes  of  two  fluids  whose  specific  gravities  are  equal,  are  also 
equal,  the  fluids  being  supposed  to  communicate  with  one  another  in 
the  arms  of  a  bent  tube,  whatever  may  be  the  shape  or  position  of  the 
arms  through  which  the  communication  takes  place. 

188.  This  explains  the  reason  why   the  surfaces  of  small  pools 
or  collections  of  water  near  rivers,  are  always  on  a  level  with  the 
surfaces  of  the  rivers,  when  there  is  any  subterraneous  communication 
between  them. 

189.  It  is  on  this  principle  also,  that  water  may  be  conveyed  from 
any  one  place,  to  any  other  place  of  the  same  or  a  less  elevation  ;  for 
by  means  of  pipes,  a  communication  can  be  opened  between  the  places, 
arid  whatever  may  be  the  number  of  elevations  and  depressions,  or 
deviations  from  the  same  vertical  plane,  and  whatever  may  be  the 
distance  from  the  source  to   the  point  of  discharge,  the  water  will 
continue  to  flow  along  the  communicating  vessels,  provided  always, 
that  none  of  the  intervening  elevations  exceeds  the  level  of  the  stag- 
nant fluid,  or  the  source  from  which  the  water  flows. 

190.  When  the  point  to  which  the  water  is  conveyed,  is  of  the 
same  altitude  as  that  from  which  it  proceeds,  the  surface  will  be  in  a 
state  of  quiescence  ;  but  if  the  point  of  discharge  be  lower  than  the 
point  of  supply,  the  fluid,  by  endeavouring  to  rise  to  the  same  level, 
will  cause  a  stream  to  flow. 

It  is  by  this  property  of  fluids  endeavouring  to  rise  to  the  same 


ORIGINATING  THE  CONSTRUCTION   OF  A  HYDROSTATIC  QUADRANT.    181 

level,  that  large  towns  and  cities  are  supplied  with  water  from  a 
distance;  the  city  of  Edinburgh,  in  Scotland,  is  supplied  in  this  way; 
but  the  successful  execution  of  all  such  complicated  and  elaborate 
undertakings,  requires  an  immense  outlay  of  capital,  directed  by  the 
skill  and  judgment  of  the  most  eminent  engineers.* 

191.  The  principle  which  we  have  demonstrated  in  the  foregoing 
proposition,  with  respect,  to  two  fluids  of  different  specific  gravities, 
may  in  like  manner,  be  shown  to  obtain  with  any  number  of  fluids 
whatever ;  a  separate  demonstration,  however,  would  here  be  out  of 
place,  we  shall  therefore  content  ourselves  with  the  general  enuncia- 
tion ;  but  the  reader  may,  for  his  own  satisfaction  and  improvement, 
supply  the  demonstration  ;  the  enunciation  is  as  follows. 

If  any  number  of  fluids  of  different  specific  gravities,  communicate 
with  one  another  through  the  arms  of  a  bent  tube,  and  remain  in 
equilibrio  : — 

The  sums  of  the  products  of  their  perpendicular  heights  and 
specific  gravities,  in  each  branch  of  the  communicating  tube, 
shall  be  equal  to  one  another. 

Various  interesting  and  important  problems  might  be  proposed,  on 
the  principle  of  fluids  of  different  densities,  communicating  with  one 
another  in  the  opposite  branches  of  a  bent  tube ;  but  our  limits  will 
only  admit  of  the  following,  which  on  account  of  its  elegance,  is 
worthy  of  a  place  in  our  present  inquiry. 

PROBLEM  XXX. 

192.  The  ratio  of  the  specific  gravities  of  two  fluids  being 
given,  if  equal  quantities  of  the  fluids  be  poured  into  a  circular 
tube  of  uniform  diameter  : — 

It  is  required  to  determine  their  position  when  in  a  state  of 
equilibrium. 

*  Baths  and  aqueducts  contributed  largely,  in  the  Roman  empire,  to  the  wealth 
and  comfort  of  the  meanest  citizen,  whether  in  the  solitudes  of  Asia  and  Africa,  or 
the  well  watered  provinces  of  the  west.  The  splendour,  the  wealth,  even  the 
existence  of  those  numerous  and  populous  cities  which  are  now  no  more,  was 
derived  from  such  artificial  supplies  of  a  perennial  stream  of  fresh  water.  The 
boldness  of  the  enterprise  and  the  solidity  of  the  workmanship  may  be  judged  of  by 
those  of  Spoleto,  Metz,  Segovia,  &c.  The  aqueduct  of  Troas,  constructed  partly  at 
the  expense  of  the  generous  Atticus,  is  but  a  solitary  instance  of  the  spirit  of 
those  PATRICIANS  who  were  not  afraid  of  displaying  to  the  world  that  they  had  the 
wisdom  to  conceive  and  wealth  to  accomplish  the  noblest  undertakings. — See 
Gibbon's  Decline  and  Fall,  vol.  i.  chap.  II. 


182  OF  THE  PRESSURE  OF  UNMIXABLE  FLUIDS  OF  DIFFERENT  DENSITIES 

Let  AB  be  the  interior  diameter  of  the  circular  tube,  and  ci,  IE  the 
spaces  occupied  by  the  fluids  when  they  have 
attained  the  state  of  quiescence,  GI  being  the 
common    surface,   or   the   plane   in   which   the 
communication  occurs. 

Through  the  common  surface  IG,  the  surface 
of  the  heavier  fluid  at  c,  and  the  surface  of  the 
lighter  at  E,  draw  the  lines  GH,  CD  and  EF 
respectively  parallel  to  the  horizon,  and  meeting 
the  diameter  AB  at  right  angles  in  the  points  H,  D  and  F  ;  then  is  DH 
the  vertical  altitude  of  the  heavier  fluid,  above  the  common  surface  IG, 
and  FH  is  the  vertical  altitude  of  the  lighter  fluid,  as  referred  to  the 
same  plane. 

Put  d  z=:  FH,  the  perpendicular  altitude  of  the  lighter  fluid, 
S  nr  DH,  the  perpendicular  altitude  of  the  heavier  fluid, 
s  =i  the  specific  gravity  of  the  lighter  fluid, 
s'  •=:  the  specific  gravity  of  the  heavier  fluid, 
0  —  c  G  or  G  E,  the  portion  of  the  circular  tube  which  is  occupied 

by  each  of  the  fluids, 
x  zr  c  B,  the  number  of  degrees  between  the  highest  and  lowest 

points  of  the  heavier  fluid.  •* 

Then,  because  by  the  preceding  proposition,  the  perpendicular 
altitudes  of  two  fluids  of  different  densities,  which  communicate  with 
one  another  through  the  branches  of  a  bent  tube,  are  inversely  as  the 
densities  or  specific  gravities  ;  it  follows,  that 

d  :  5  :  :  s'  :  s, 

and  from  this  analogy,  by  making  the  product  of  the  mean  terms 
equal  to  the  product  of  the  extremes,  we  obtain 

c?snr£s'  ; 

the  very  same  result  as  equation  (138),  and  if  both  sides  be  divided 
by  5,  we  shall  obtain 


Now,  if  the  specific  gravity  of  the  lighter  fluid  be  expressed  by 
unity,  while  that  of  the  heavier  is  denoted  by  m  ;  then  the  above 
equation  becomes 

d  =  md.  (140). 

By  referring  to  the  diagram,  it  will  readily  appear,  that  the  space 
occupied  by  both  the  fluids,  when  in  a  state  of  equilibrium,  is  repre- 
sented by 


ORIGINATING  THE  CONSTRUCTION  OF  A  HYDROSTATIC  QUADRANT.    183 

CGEHI20, 

and  according  to  our  notation,  it  is  shown  that 

c  B  zr  x  ; 
consequently,  by  subtraction,  we  obtain 

B  G  ~  0  —  x,  and  B  G  E  rz:  20  —  x. 

But  by  the  nature  of  the  circle  and  the  principles  of  Plane  Trigo- 
nometry, it  is  manifest,  that  BD  is  the  versed  sine  of  the  arc  BC  ; 
B  H  the  versed  sine  of  the  arc  B  G,  and  B  F  the  versed  sine  of  the  arc  B  G  E  ; 
therefore,  by  re-establishing  the  respective  symbols,  we  shall  obtain 
F  H  =r  d  •=.  vers.  (20  —  x)  —  vers.  (0  —  a1), 
and  by  a  similar  subtraction,  we  get 

D  H  zz  I  =:  vers.z —  vers.  (0 —  x). 

Let  both  sides  of  this  equation  be  multiplied  by  m,  the  co-efficient 
of  8  in  equation  (140),  and  we  shall  have 

mS  —  m  {vers. a;  —  vers.  (0  —  x)}  ; 
consequently,  by  comparison,  we  obtain 
vers.  (20  —  x}  —  vers<(0  —  x)  ~  m{ vers.x  —  vers.  (0  — x)}. 
Now,  in  order  to  simplify  the  reduction  of  this  equation,  it  will  be 
proper  to  substitute  for  the  several  versed  sines  of  which  it  is  com- 
posed, their  corresponding  values  in  terms  of  the  radius  and  cosines ; 
for,  by  such  a  substitution  we  obtain 

cos.  (0  —  x)  —  cos.  (20  —  x)  =  m  {cos.  (0  —  x)  —  cos. a-}  ; 

but  by  the  arithmetic  of  sines,  we  have 
cos.  (0  —  #)i=:cos.0cos.a7  -f-  sin.  0  sin. a:,  and  cos.  (20  —  ^)zzcos.20 

cos.  x  -)-.  sin.  20  sinfjgj 

consequently,  by  substitution,  we  get 

cos.  0  cos.  x  -f-  sin.0sin.a: —  cos.  20  cos.  x  —  sin.  20  sin.  z:zzwi{cos.0 

cos. a?  -j-  sin.0  sin. a:  —  cos. a;}  ; 

and  from  this,  by  transposition,  we  have 

mcos.x — cos.20cos.a:-j-sin.20sm.,r-}-(7W — l)(cos.0cos.o;-4-sin.0sin.a:). 

Let  all  the  terms  of  this  equation  be  divided  by  cos.  x,  and  we  shall 
obtain 

w=icos.20  -f-  sin. 20  tan  .x  -}-  (m —  l)(cos.0-|-  sin.0  tan. a:) ;  * 
therefore,  by  separating  and  transposing  the  terms,  we  get 


*  It  is  demonstrated  by  the  writers  cm  Analytical  Trigonometry,  that  the  sine 
divided  by  the  cosine  to  the  same  radius,  is  equal  to  the  tangent;  hence  we  have 

sin.r 


tan. a  zr . 

cos..r 


184  OF  THE  PRESSURE  OF  UNMTXABLE  FLUIDS  OF  DIFFERENT  DENSITIES 

—  sin.  20  tan.  re  —  (m  —  1)  sin.  0  tan.  x  =  cos.20  -\-(m  —  1)  cos.0  —  m, 
and  finally,  by  division,  we  obtain 

cos.2$  -j-  (m  —  1)  cos.0  —  m 

~l)sin.  (141). 


193.  We  believe  that   Mr.  Barclay's  Hydrostatic  Quadrant,  for 
finding-   the   altitude  of  the  heavenly  bodies  when   the   horizon   is 
obscure,  is  founded  on  principles  similar  to  those  propounded  in  this 
problem,  and    expressed   in  the  above  equation  ;    but  it  would  be 
improper  in  this  place  to  attempt  a  delineation  of  this  instrument  ; 
it  will   therefore   suffice,  to  illustrate   the   reduction  of  the   above 
formula,  by  a  numerical  example  performed  according  to  the  direc- 
tions contained  in  the  following  practical  rule. 

RULE.  From  the  specific  gravity  of  the  heavier  fluid,  sub- 
tract unity  ;  multiply  the  remainder  by  the  natural  cosine  of 
the  circular  space  occupied  by  each  fluid;  to  the  product  add 
the  natural  cosine  of  the  circular  space  occupied  by  both 
fluids  ;  then,  from  the  sum  subtract  the  greater  specific  gravity, 
and  the  remainder  will  be  the  dividend. 

Again.  From  the  specific  gravity  of  the  heavier  fluid,  sub- 
tract unity  ;  multiply  the  remainder  by  the  natural  sine  of 
the  circular  space  occupied  by  each  fluid  ;  then,  to  the  pro- 
duct add  the  natural  sine  of  the  circular  space  occupied  by 
both  fluids,  and  the  sum  will  be  the  divisor. 

Lastly.  Divide  the  dividend  by  the  divisor,  and  the  quotient 
will  give  the  natural  tangent  of  a  circular  arc,  which  being 
found  in  the  tables,  enables  us  to  assign  the  position  of  the 
fluids  when  in  a  state  of  equilibrium. 

194.  EXAMPLE.  Suppose  that  100  degrees  of  the   inner  circum- 
ference of  a  circular  tube,  exhibits  equal  quantities  of  mercury  and 
water,  whose  specific  gravities  are  to  one  another,  very  nearly,  as 
14  to  1  ;  it  is  required  to  assign  the  position  of  the  fluids,  with  respect 
to  the  vertical  diameter  of  the  tube,  when  they  are  in  a  state  of  equili- 
brium with  each  other  ;  that  is,  when  they  excite  equal  pressures  on 
the  plane  passing  through  their  common  surface  ? 

Here  we  have  given  mm  14  :0z=:500,  its  natural  sine  and  cosine 
.76604  and  .64279  respectively;  20zr  100°,  its  natural  sine  .98481, 
and  its  cosine  —  .17365;  therefore,  by  the  rule,  we  have 

For   the  dividend, 
—  cos.2(H-(w-l)cos.0-?w——  .17365+13  X.  64279—  14r=—  5.81738, 


ORIGINATING  THE  CONSTRUCTION  OF  A  HYDROSTATIC  QUADRANT     185 

For  the  divisor, 

—  sin.20—  (w—  l)sin.20=  —  .98481—  13  X.76604z=  —  10.94333; 
consequently,  by  division,  we  obtain 

=  -53159  =  nat.tan.  27°  59'  41". 


195.  Having  discovered  the  value  of  x  by  the  preceding  operation, 
the  actual  position  of  the  fluids,  with  respect  to  the  vertical  diameter 
of  the  tube,  may  from  thence  be  very  easily  exhibited. 

Let  AC  BE  represent  a  circular  tube  of  glass,  or  some  other  trans- 
parent matter,  partly  filled  with  mercury  and 
water,  in  such  quantities,  that  when  the  tube 
is  retained  in  a  vertical  plane,  and  the  fluids 
in  equilibrio,  a  space  equivalent  to  fifty  de^ 
grees  of  the  inner  surface  comes  in  contact 
with  each  ;  it  is  therefore  required  to  assign 
the  actual~position  of  the  fluids. 

Draw  the  vertical  diameter  AB,  and  from  the  point  B  where  it  meets 
the  inner  circumference  of  the  tube,  set  offsc  from  a  scale  of  chords, 
equal  to  27°  59'  41"  ;  then,  take  50°  in  the  compasses,  and  setting  one 
foot  on  c  extend  the  other  to  G,  thereby  marking  off  the  space  occu- 
pied by  the  mercury,  including  the  lowest  portion  of  the  tube  ;  then, 
with  the  same  extent  of  the  compasses,  set  off  G  E  the  space  occupied 
by  the  water,  and  the  position  of  the  fluids  is  from  thence  determined. 

Through  the  points  c,  G  and  E  draw  the  straight  lines  CD,  GII  and 
EF  respectively  parallel  to  the  horizon,  and  meeting  AB  the  vertical 
diameter  perpendicularly  in  the  points  D,  H  and  F  ;  then  are  DH  and 
FH  the  perpendicular  altitudes  of  the  mercury  and  the  water,  as 
referred  to  the  plane  passing  through  their  common  surface  at  G  ;  and 
BD,  BF  are  the  respective  altitudes,  as  referred  to  the  vertical  dia- 
meter AB. 

Now,  according  to  the  question,  the  specific  gravity  of  the  mercury, 
is  fourteen  times  greater  than  that  of  the  water  ;  and  by  the  third 
proposition  preceding,  the  perpendicular  altitudes  are  inversely  as  the 
specific  gravities;  consequently,  FH  must  be  fourteen  times  greater 
than  DH;  when  the  positions  of  the  fluids  are  properly  determined  ; 
let  us  therefore  inquire  if  this  be  the  case. 

It  has  been  found  above,  that  BC  is  equal  to  27°  59'  41",  and  by 
construction  CG  and  GE  are  each  equal  to  50  degrees;  consequently, 
BG  —  50°  —  27°  59'  41"—  22°  0'  19",  and  B  E  =  50°  +  22°  0'  19"  — 
72°  0'  19"  ;  hence  we  have 


186  OF  THE  PRESSURE  OF  UNMIXABLE  FLUIDS  OF  DIFFERENT  DENSITIES 

BEz=72°0'  19"  -  -  -  nat.  vers.  —  .69106, 
BC  —  27°  59'  41"  -  -  -  nat.  vers.  —  .11700, 
B  G  =  22°  0'  19"  -  -  -  nat.  vers.  =  .07285  ; 

consequently,  by  subtraction,  we  obtain 
FH  =  BF  —  BH  ;  that  is,  FIX  =  .69106  —  .07285  —  .61821  , 

and  after  the  same  manner,  we  get 

DH  =  BD  —  BH;  that  is,  DH  —  .11700  —  .07285  =  .04415  ; 

therefore,  according  to  the  proposition,  we  have 

.04415  :  .61821  :  :  1  :  14  very  nearly. 

196.  The  above  result  has  been  obtained  on  the  supposition,  that 
the  fluids  enclosed  in  the  tube  are  mercury  and  water  ;  mercury,  on 
account  of  ,its  great  density  and  high  degree  of  purity,  is  very  fre- 
quently enclosed  in  tubes  and  applied  to  permanent  purposes  ;  but 
water,  by  reason  of  its  liability   to  become  putrid,  is  not  so  well 
adapted  for  the  occasion,  and  consequently,  is  seldom  or  never  em- 
ployed in  the  construction  of  philosophical  instruments. 

197.  There  are  however,  several  other  fluids  that  do  not  partake 
of  the  putrescent  nature  of  water,  and  whose  specific  gravities  may  be 
either  greater  or  less  according  to  the  required  circumstances  ;  some 
of  these,  on  account  of  the  colouring  matter  which  they  contain,  are 
very  convenient,  and  from  the  length  of  time  that  they  retain  their 
spirit  and  purity,  are  generally  employed  in  preference  to  others,  which 
do  not  possess  these  very  requisite  and  important  characteristics. 

198.  Now,  the  result  of  our  investigation,   as  we  have  already 
observed,  is  only  applicable  in  the  case  of  mercury  and  water  ;  at 
least,  equation  (141)  implies,  that  the  specific  gravity  of  one  of  the 
fluids  is   expressed  by   unity,   and   this,   according   to  our  present 
standard,    can    only   obtain    when   water   becomes    the    subject   of 
reference. 

199.  It  is  therefore  necessary,  in  order  that  our  formula  may  apply 
to  fluids  without  distinction,  to  bring  it  into  a  general  form,  and  this  is 
very  easily  done;  for  we  have  shown  in  the  preceding  investigation,  that 


but  it  has  also  been  shown,  that 
d  —  vers.  (20  —  x)  —  vers.  (0  —  a:)  ; 

consequently,  by  comparison,  we  obtain 

$s' 

—  —  vers.  (20  —  a:)  —  vers.(0  —  a-)  ; 


ORIGINATING  THE  CONSTRUCTION  OF  A  HYDROSTATIC  QUADRANT.    187 

and  moreover,  it  has  been  further  proved, 'that 
5  =r  vers.a;  —  vers.(<£  —  a;) ; 

therefore,  multiplying  both  sides  by—,  we  shall  get 
— m— {vers.a?  —  vers.(<£  —  x}}  ; 

S  5 

let  these  two  values  of  —  be  compared  with  one   another,  and  we 
shall  obtain 

vers.(20 —  a?) —  vers.(0  —  #)  m  —  {vers.z  —  vers.(</>  —  x)}  ; 

or  multiplying  by  s,  we  have 

s  vers.(2^>  —  ar)  —  s  vers.(^>  —  x}  zz:  s'  vers.a?  —  s'  vers.(0  —  x). 
Now,  by  substituting  for  the  several  versed  sines,  their  values  in 
terms  of  the  cosines  and  radius,  we  shall  obtain 

s{cos.(0  —  x}  —  cos. (2^  —  a:)}  — s'{cos.(0 —  x) —  cos. a-  } 

from  which,  according  to  the  arithmetic  of  sines,  we  get 
s  {cos.0  cos.a:  -f-  sin.0  sin.a;  —  cos.2^  cos. x  —  sin.20  sin. a;}  zz:  s'  {cos.<£ 

cos. a;  -j-  sin.0  sin. a?  — cos. a;}. 

Let  all  the  terms  of  this  equation  be  divided  by  cos. a;,  and  it 
becomes  transformed  into 
s  cos.0  -{-  s  sin.0  tan. a;  —  s  cos.20  —  s  sin.2^>  tan. a:  zz:  sr  cos.0  -f~  s/  X 

sin.  ^  tan.  x  —  5'; 

therefore,  by  bringing  to  one  side,  all  the  terms  that   involve  tan. a:, 
we  shall  have  * 

—  sin.^(s' — s)tan.a;  —  5  sin. 20  tan. x n:  scos.2^  -|-  (*' — s)cos.^> — s' ; 
hence,  by  division,  we  shall  obtain 

s  cos.2<6  4-  (s'  —  s)  cos.0  —  5' 

tan  x  "~~  — - 

s  sin.  20  +  (s>  —  s)  sin.0  (142). 

If  the  equation  which  we  have  just  obtained,  be  compared  with  that 
numbered  (141),  it  will  readily  appear,  that  the  one  might  have  been 
deduced  immediately  from  the  other,  by  simply  substituting  s'  for  m, 
and  s  for  unity,  in  the  several  terms  of  the  numerator  and  denominator; 
but  in  order  to  render  the  formation  of  the  formula  more  intelligible, 
we  have  thought  proper  to  trace  the  steps  throughout. 

200.  The  practical  rule  for  reducing  the  above  equation,  will  require 
a  different  mode  of  expression  from  that  which  we  have  given  in  the 
rule  to  equation  (141),  but  it  will  not  be  more  operose ;  the  rule  is 
as  follows. 


188  OF  THE  PRESSURE  OF  UNMIXABLE  FLUIDS  OF  DIFFERENT  DENSITIES 

RULE.  Multiply  the  difference  of  the  yiven  specific  gravities, 
by  the  natural  cosine  of  the  circular  space  in  contact  with  one 
of  the  fluids  ;  to  the  product,  add  the  natural  cosine  of  the 
whole  circular  space  drawn  into  the  less  specific  gravity  ,  and 
from  the  sum  subtract  the  greater  specific  gravity  for  a 
dividend. 

Again.  Multiply  the  difference  between  the  specific  gravi- 
ties, by  the  natural  sine  of  the  circular  space  in  contact^with 
one  of  the  fluids,  and  to  the  product,  add  the  natural  sine  of 
the  whole  circular  space  drawn  into  the  less  specific  gravity, 
and  the  sum  ivill  be  the  divisor. 

Lastly.  Divide  the  dividend  by  the  divisor,  and  the  quotient 
will  give  the  natural  tangent  of  a  circular  arc,  which  being 
found  in  the  tables,  enables  us  to  assign  the  actual  position  of 
the  fluids  when  in  a  state  of  equilibrium. 

201.  EXAMPLE.  On  the  inner  surface  of  a  circular  tube  containing 
mercury  and  rectified  alcohol,  it  is  observed,  that  when  the  tube  is 
held  in  a  vertical  plane,  and  the  fluids  in  a  state  of  equilibrium,  a 
space  of  75  degrees  of  the  circumference,  is  occupied  by,  or  in  contact 
with  each  fluid  ;  it  is  required  to  determine  the  position  of  the  fluids 
at  the  instant  of  observation,  their  specific  gravities  being  14000  and 
829  respectively  ? 

In  this  example  there  are  given  /=  14000;  s  —  829;  ^  —  7£°7 
its  natural  sine  and  cosine  equal  to  .96593  and  .25882;  2^>rr:  150°, 
its  natural  sine  being  .50000,  and  its  cosine  —  .86603  ;  consequently, 
by  proceeding  according  to  the  directions  contained  in  the  foregoing 
rule,  we  shall  obtain 

For  the  dividend  —  scos.2^  -f  (s'  —  s)cos.<£  —  s'=  —  829  X  .86603  -|- 

(14000  —  829)  x  .25882  —  14000  =  —  11  309  .02065  ; 

For  the  divisor  —  s  sin  .2<£  —  (V  —  s)  sm.^>  —  —  829  X  .50000  —  (14000 

—  829)  X  .96593  =  —  1  3136.76403  ; 

consequently,  by  division,  we  obtain 


ta"-*  =  =  -86086  =  °at-  tan-  40-  43'  25'. 


202.  The  positions  of  the  fluids  in  this  example,  are  manifestly 
very  different  from  what  they  are  in  the  preceding,  the  point  F  in  the 
vertical  diameter  falling  on  the  other  side  of  the  centre;  but  in  this 
case,  we  shall  leave  the  construction  for  the  reader's  amusement,  and 
proceed  to  inquire  what  changes  the  general  formula  will  undergo,  in 


ORIGINATING  THE  CONSTRUCTION  OF  A   HYDROSTATIC  QUADRANT.    189 

consequence  of  certain  assumed  spaces  of  the  inner  surface,  being  in 
contact  with  each  of  the  contained  fluids. 

If  0:zzhalf  a  right  angle,  that  is,  if  each  fluid  cover  a  space  of  45 
degrees;  then  20  zz  90°,  and  consequently,  sin.20  zr  1  and  cos.20:zrO, 
while  sin.  0:=!  ^/ 2,  and  cos.0:=r  |  ^2  ;  therefore,  by  substitution, 
equation  (142)  becomes 

l(s' 

-J(s' 

Now,  let  the  fluids  be  mercury  and  rectified  alcohol,  as  in  the  pre- 
ceding example,  then  we  shall  have 

829  V2- 


i  (14000—829)^2  -f  829 

which  answers  to  the  natural  tangent  of  28°  55'  nearly. 
Again,  if  0zza  right  angle,  that  is,  if  each  fluid  cover  a  space  of 
90  degrees  on  the  inner  surface  of  the  tube;  then,  20zzl80°,  of 
which  the  sine  and  cosine  are  respectively  0  and  —  1,  while  the  sine 
and  cosine  of  0,  are  respectively  1  and  0 ;  consequently,  by  substitu- 
tion, equation  (142)  becomes 

s'  -4-  s 

tan.zzz-7-- — .  /IAA\ 

s1 — s  (!44). 

203.  This  is  a  very  neat  and  obvious  expression,  and  the  practical 
rule  derived  from  it,  may  be  enunciated  in  the  following  manner. 

RULE.  Divide  the  sum  of  the  specific  gravities  of  the  two 
fluids  by  their  difference,  and  the  quotient  will  give  the 
natural  tangent  of  an  arc,  which  being  estimated  from  the 
lowest  point  of  the  tube,  will  indicate  the  highest  point  of  the 
heavier  fluid. 

If  therefore,  the  contained  fluids  be  mercury  and  rectified  alcohol, 
as  in  the  preceding  cases,  we  shall  have 
14000  -|-  829 

ta"-*=14000-829=U2587- 
which  answers  to  the  natural  tangent  of  48°  23'  19''. 
We  might  assume  other  particular  values  of  the  spaces  in  contact 
with  the  fluids,  and  thereby  deduce  corresponding  forms  of  the  equa- 
tion ;    but   what   we   have    already    done  on  this   subject   is    quite 
sufficient. 


CHAPTER  VIII. 

OF  THE  PRESSURE  OF  NON-ELASTIC  FLUIDS  UPON  DYKES,  EM- 
BANKMENTS, OR  OTHER  OBSTACLES  WHICH  CONFINE  THEM, 
WHETHER  THE  OPPOSING  MASS  BE  SLOPING,  PERPENDICULAR 
OR  CURVED,  AND  THE  STRUCTURE  ITSELF  BE  MASONRY 
OR  OF  LOOSE  MATERIALS,  HAVING  THE  SIDES  ONLY  FACED 
WITH  STONE. 

1.    OF  FLUID  PRESSURE  AGAINST  MASONIC  STRUCTURES. 

204.  BEFORE  we  proceed  to  develope  the  theory  of  Floatation,  and 
to  explain  the  method  of  weighing  solid  bodies  by  immersing  them  in, 
or  otherwise  comparing  them  with  liquids ;  it  is  presumed  that  it  will 
not  be  considered  out  of  place,  to  take  a  brief  survey  of  the  circum- 
stances attending  the  pressure  of  non-elastic  fluids,  when  exerted 
against  dykes  or  other  obstacles,  that  may  be  opposed  to  the  efforts 
which  they  make  to  spread  themselves. 

This  is  an  interesting  and  important  subject  in  the  doctrine  of 
Hydraulic  Architecture,  and  since  the  principles  upon  which  it  is 
founded,  depend  in  a  great  measure  on  Hydrostatic  pressure,  it  cannot 
properly  be  omitted  in  unfolding  the  elementary  departments  of  the 
Mechanics  of  Fluids,  which  come  so  directly  before  our  view  in  what 
is  called  level  cutting  in  the  practice  of  canal  making.  Every  one 
knows  that  in  cutting  a  canal,  no  further  excavation  is  required  than 
that  which  will  hold  the  water  at  a  given  depth  and  breadth  ;  when 
a  bank  is  made  on  both  sides  with  the  earth  excavated,  the 'level  sur- 
face of  the  canal  may  be  elevated  above  the  natural  surface  of  the 
adjacent  land,  and  in  this  case  great  part  of  the  cost  of  excavation 
will  be  saved.  But  when  the  canal  is  to  be  carried  along  wholly 
within  embankments,  too  much  attention  cannot  be  paid  to  the  prin- 
ciples of  fluid  pressure,  if  we  would  avoid  unnecessary  expense,  and 
at  the  same  time  complete  the  work  with  systematic  regard  to  its 
permanent  durability ;  this  therefore  is  the  object  of  the  present 


OF  THE  PRESSURE  OF  FLUIDS  ON   DYKES  AND  EMBANKMENTS.     191 

section,  intended  as  a  preliminary  article  to  our  Inland  Navigation, 
which  will  consequently  form  a  part  of  Hydraulic  Architecture. 

205.  When  an  incompressible  and  non-elastic  fluid  presses  against 
a  dyke,  mound  of  earth,  or  any  other  obstacle  that  it  endeavours  to 
displace,  there  are  two  ways  in  which  the  obstacle  thus  opposed  may 
yield  to  the  effort  of  the  fluid. 

1 .  It  may  yield  by  turning  upon  the  remote  extremity  of 
its  base. 

2.  It  may  yield  by  sliding  along  the  horizontal  plane  on 
which  it  stands. 

In  either  case,  the  effort  to  overcome  the  obstacle,  arises  from  the 
force  which  the  fluid  exerts  in  a  horizontal  direction ;  and  the  stability 
of  the  obstacle,  or  the  resistance  which  it  opposes  to  being  overcome 
or  displaced,  arises  from  its  own  weight,  combined  with  the  vertical 
pressure  of  the  fluid  upon  its  sloping  surface. 

206.  When  the  vertical  pressure  of  the  fluid  is  considered,  the 
investigation,  as  well  as  the  resulting  formulae,  a're  necessarily  tedious 
and  prolix  ;  but  when  the  effect  of  the  vertical  pressure  is  omitted,  the 
subject  becomes  more  easy,  and  the  computed  dimensions  are  better 
adapted  for  an  effectual  resistance ;  but  in  order  to  render  the  inves- 
tigation general,  it  becomes  necessary  to  include  its  effects. 

Now,  it  is  manifest  from  the  nature  of  the  inquiry,  that  when  an 
equilibrium  obtains  between  the  opposing  forces,  the  momentum  of 
the  horizontal  pressure  must  be  equal  to  the  momentum  of  the  vertical 
pressure,  together  with  the  weight  of  the  body  on  which  the  pressure 
is  exerted ;  and  for  the  purpose  of  showing  when  this  condition  takes 
place,  let  A  BCD  represent  a  vertical  section 
of  the  dyke,  whose  resistance  is  opposed  to 
the  pressure  of  the  stagnant  fluid,  of  which 
the  surface  is  ME  and  the  perpendicular 
depth  EF. 

Let  AB  and  DC  be  parallel  to  the  hori- 
zon, and  consequently  parallel  to  one  another;  and  from  the  points 
E,  A  and  B,  demit  the  straight  lines  EF,  AK,  and  BL,  respectively 
perpendicular  to  DC  the  base  of  the  section. 

Take  EG  any  small  portion  of  the  sloping  side  AD,  and  through 
the  point  G,  draw  the  lines  GH  and  01,  respectively  parallel  and 
perpendicular  to  the  horizon,  constituting  the  similar  triangles  EHG, 
EFD  and  GID. 

The  figure  being  thus  prepared,  it  only  remains  to  establish  the 
proper  symbols  of  reference,  before  proceeding  with  the  investigation. 


192       OF  THE  PRESSURE  OF  FLUIDS  ON    DYKES  AND  EMBANKM  EN  TS. 

Put  b  zz:  DC,  the  breadth  of  the  section's  base,  or  the  thickness  of  the 
dyke  at  the  foundation, 

D  zz  AK  or  BL,  the  perpendicular  altitude  or  height  of  the 
section, 

d  =z  EF,  the  perpendicular  depth  of  the  fluid  whose  surface  is 
at  EM, 

I  zz:  DF,  the  distance  between  the  near  extremity  of  the  base  at 
D,  and  the  perpendicular  E  F, 

c  zz:  DK,  the  measure  of  the  slope  AD,  or  the  distance  between 
the  near  extremity  of  the  base  at  D,  and  the  perpendi- 
cular from  the  extremity  of  the  opposite  side  at  A, 

c  zz:  CL,  the  distance  between  the  remote  extremity  of  the  base 
at  c,  and  the  perpendicular  from  the  extremity  of  the 
opposite  side  at  B,  or  the  measure  of  the  slope  BC, 

a  zz:  ABCD,  the  area  of  a  vertical  section  of  the  obstacle  to  be 
displaced, 

p  —  the  horizontal  pressure  of  the  fluid  on  the  increment  of  EG, 

/  zz:  the  force  with  which  the  horizontal  pressure  operates  to 
overcome  the  resistance  of  the  dyke, 

m  zz:  the  momentum  of  that  force, 

p'  =  the  vertical  pressure  of  the  fluid  on  the  increment  of  EG, 

/'  zr  the  force  with  which  the  vertical  pressure  resists  the  dis- 
placement of  the  obstacle, 

tfi'nr  the  momentum  of  that  force, 

w  zz:  the  symbol  which  denotes  the  weight  of  the  dyke  or  obstacle 
of  resistance, 

F  =z  the  force  with  which  it  opposes  the  horizontal  pressure  of 
the  fluid, 

M  zz  the  momentum  of  that  force, 

s    zz-  the  specific  gravity  of  the  fluid, 

s'  zz:  the  specific  gravity  of  the  dyke,  or  opposing  body, 

z  zz:  EG,  any  small  portion  of  the  sloping  side  AD  on  which  the 
fluid  presses, 

z  zz:  the  increment  or  fluxion  of  that  portion, 

y  zz:  EH,  the  perpendicular  depth  of  the  point  G, 

y  z=  the  increment  or  fluxion  of  y, 

x  zz:  G  H,  the  ordinate  or  horizontal  distance, 

and  x  z=  the  increment  or  fluxion  of  the  horizontal  ordinate  or  dis- 
tance G  H. 

Then,  since  the  pressure  upon  any  line  or  surface,  is  equal  to,  or 
expressed  by  the  magnitude  of  that  line  or  surface,  multiplied  by  the 


OF  THE  PRESSURE  OF  FLUIDS  ON  DYKES  AND  EMBANKMENTS.       193 

perpendicular  depth  of  its  centre  of  gravity,  and  again  by  the  specific 
gravity  of  the  fluid  ;  it  follows,  that  the  horizontal  pressure  on  the 
increment  of  EG,  is 


but  by  the  principles  of  mechanics,  the  aggregate  or  accumulated  force, 
with  which  the  horizontal  pressure  operates  to  overturn  or  remove  the 
dyke,  is 


and  by  taking  the  fluent  of  this,  it  is 

/=  %sy*. 

But  the  perpendicular  distance  from  E,  at  which  this  force  must  be 
applied,  is  manifestly  equal  to  %y ;  for  the  centre  of  gravity  of  the 
triangle  EHG,  occurs  in  the  horizontal  line  passing  through  that  point; 
therefore,  the  length  of  the  lever  on  which  the  force  operates  to  over- 
turn the  dyke  is 

consequently,  for  the  momentum  of  the  force,  we  have 


and  when  y  becomes  equal  to  d,  the  whole  height  of  the  fluid,  it  is 

m  —  ±sd\  (145). 

Again,  the  vertical  pressure  exerted  by  the  fluid  on  the  increment  of 
EG,  is  obviously  equal  to  the  weight  of  the  incumbent  column  ;  that  is 


and  this  pressure  expresses  the  force,  with  which  the  fluid  operates 
vertically  to  retain  the  obstacle  in  its  position,  or  to  prevent  it  from 
rising  to  turn  about  the  point  c  ;  consequently, 

p'=f'=syx. 

Now,  the  length  of  the  lever  on  which  this  force  acts,  is  evidently 
equal  to  ic,  the  distance  between  the  fulcrum  c,  and  the  point  i,  where 
the  perpendicular  passing  through  G  cuts  the  base  DC  ;  but  ic  accord- 
ing to  the  figure,  is  equal  to  DC  —  DF  -f  i  F  ;  that  is 

ic  =  b  —  £  +  *; 
consequently,  the  momentum  of  the  force  /',  is 

m'  =  syx(b  —  3  -far), 
or  taken  collectively,  the  momentum  on  EG,  is 


VOL.   I. 


194       OF  THE  PRESSURE  OF  FLUIDS  ON    DYKES  AND  EMBANKMENTS. 

but  by  reason  of  the  similar  triangles  EHG  and  EFD,  we  have  the 
following  proportion,  viz. 

d  :  3  :  :  y  :  x, 

from  which  we  obtain 


: 

and  because  the  fluxions  of  equal  quantities  are  equal,  it  is 


Let  these  values  of  x  and  x,  be  substituted  instead  of  them  in  the 
preceding  value  of  m',  and  we  shall  obtain 


consequently,  by  taking  the  fluent,  it  becomes 


there  being  no  correction,  since  the  whole  expression  becomes  equal  to 
nothing  when  y  is  equal  to  nothing. 

When  y  becomes  equal  to  d  the  whole  perpendicular  height  of  the 
fluid,  then  the  foregoing  value  of  m'  becomes 

wf  =  «8d(J6—  >8).  (146). 

The  foregoing  equations  (145)  and  (146),  exhibit  the  horizontal 
and  vertical  momenta  of  the  pressure  exerted  by  the  fluid  on  the 
sloping  side  of  the  obstacle  ;  and  it  is  manifest  from  the  nature  of 
their  action,  that  they  operate  in  opposition  to  one  another  ;  the 
horizontal  pressure,  endeavouring  to  turn  the  body  round  the  point  c 
as  a  fulcrum  or  centre  of  motion,  and  the  vertical  pressure  tending  to 
turn  it  the  contrary  way  round  the  same  point,  or  otherwise  to  render 
it  more  stable  and  firm  on  its  foundation. 

208.  But  the  stability  of  the  dyke  is  farther  augmented  by  means 
of  its  own  weight,  which  being  conceived  to  be  collected  into  its 
centre  of  gravity,  opposes  the  horizontal  pressure  of  the  fluid  with  a 
force,  which  is  equivalent  to  its  own  weight  drawn  into  a  lever,  whose 
length  is  equal  to  the  perpendicular  distance  between  the  centre  of 
motion,  and  a  vertical  line  passing  through  the  centre  of  gravity  of 
the  section  ABCD. 

Now,  it  is  manifest  from  the  principles  of  mensuration,  that  if  the 
transverse  section  of  the  dyke  be  uniform  throughout,  the  weight  is 
proportional  to  the  area  of  the  section,  multiplied  into  the  specific 


OF  THE  PRESSURE  OF  FLUIDS  ON  DYKES  AND  EMBANKMENTS.      195 

gravity  of  the  material  of  which  it  is  composed,  and  again  into  ijs 
length ;  but  the  length  of  the  dyke  is  the  same  as  the  length  of  the 
fluid  which  it  supports  ;  consequently,  the  weight  is  very  properly 
represented  by  the  area  of  the  section  and  the  specific  gravity  of  the 
material ;  thus  we  have 

w  —  as'.  (147). 

But  according  to  the  writers  on  mensuration,  the  area  of  the  trape- 
zoid  A  BCD,  is  equal  to  the  sum  of  the  parallel  sides  AB  and  DC,  drawn 
into  half  the  perpendicular  distance  AK  or  BL  ;  hence  we  have 

a=(AB  -f-  DC)XjAK, 

but  by  the  foregoing  notation,  it  is 

A  B  —  b  —  (c  -{-  e) ; 
consequently,  by  addition,  we  have 

A  B  -f  DC  =  26  —  (c  +  e) ; 
therefore,  the  area  of  the  section  is 

a  =  ID  (26  —  c  —  e); 

let  this  value  of  a  be  substituted  instead  of  it  in  the  equation  marked 
(147),  and  it  becomes 

w  •=.  |DS'  (26  —  c  —  e) ; 

but  the  weight  of  the  dyke  is  equivalent  to  the  force  whose  momentum, 
combined  with  that  of  the  vertical  pressure  of  the  fluid,  counterpoises 
the  momentum  of  the  horizontal  pressure,  which  force  we  have  repre- 
sented by  F  ;  hence  we  have 

F— IDS' (26  —  c  —  e\ 
and  the  momentum  of  this  force,  is 

IF  —  M  zz  JD  Is'  (26  —  c  —  e), 

where  /  denotes  the  lever  whose  length  is  equal  to  the  distance 
between  the  fulcrum,  or  centre  of  motion  at  c,  and  the  vertical  line 
passing  through  the  centre  of  gravity  of  the  section  A  BCD  ;  conse- 
quently, in  the  case  of  an  equilibrium,  we  have 

mi^.m'  -j-  M, 
and  this,  by  restoring  the  analytical  values,  becomes 

Lsd*  =  s3d(tib  —  £3) -{- ID/*' (26  —  c  —  e).  (148). 

209.  This  is  the  general  equation  which  includes  all  the  cases  of 
rectilinear  sloping  embankments,  but  it  has  not  yet  obtained  its 
ultimate  form ;  for  the  value  of  I  has  still  to  be  expressed  in  terms 
of  the  sectional  dimensions,  and  in  order  to  this,  a  separate  investi- 
gation becomes  necessary. 

o2 


196       OF  THE  PRESSURE  OF  FLUIDS  ON   DYKES  AND  EMBANKMENTS. 

Thus,  let  A  B  c  D  be  a  vertical  section  of 
the  dyke  as  before,  and  bisect  the  parallel  /Hn\ 

sides  AB  and  DC  in  the  points  m  and  w, 
and  join  mn\  then,  the  straight  line  mn 
will  pass  through  the  centre  of  gravity  of 
the  figure  ABCD. 

Take  g  a  point  such,  that  mg  is  to  ng,  as  2n  c  -j-  A  B  is  to  BC  -}-  2AD, 
and  g  will  be  the  centre  of  gravity  sought  ;  through  the  points  m  and 
g,  draw  the  straight  lines  mr  and  gs,  respectively  perpendicular  to 
DC  the  base  of  the  section,  then  is  sc  the  length  of  the  lever  by  which 
the  weight  of  the  dyke  or  embankment  opposes  the  horizontal  pressure 
of  the  fluid. 

From  the  points  A  and  B,  draw  the  straight  lines  AK  and  BL,  re- 
spectively perpendicular  to  D  c  ;  then  it  is  manifest  from  the  principles 
of  geometry,  that 

rw=  £(CL  —  DK), 

and  this,  by  restoring  the  symbols  for  CL  and  DK,  becomes 
rn~^(e  —  c). 

But  mrzrD;  consequently,  by  the  property  of  the  right  angled 
triangle,  we  have 

m  n*  m  m  i*  -f-  n  r3  ; 
or  by  restoring  the  analytical  values,  it  is 

raw2=iD94-  \(e  —  cf\ 
therefore,  by  extracting  the  square  root,  we  have 


m  n  —  J  -Y/  4o2  -|-  (e  -=—  c)". 

By  the  property  of  the  centre  of  gravity,  and  according  to  the 
foregoing  construction,  the  point  g  is  determined  in  the  following 
manner. 

2DC-{-    AB  =  36  —    c  —    e 
—  36  —  2c  —  2e 


66  —  3c  —  3e  :  i^/4D24-(e  —  c)8:  :  36  —  2c—  2e  :  gn, 
from  which,  by  reducing  the  analogy,  we  get 


(36  —  2c  —  2e)  v/4o2-f-  (e  —  c)s 
9n-~  6(26-c-e) 

and  by  the  property  of  similar  triangles,  it  is 


— 2c  — 


OF  THE   PRESSURE  OF  FLUIDS  ON  DYKES  AND  EMBANKMENTS.        197 

wherefore,  by  reducing  the  analogy,  we  obtain 
_(e  —  c)(3b  —  2c  —  2e) 
6(26  —  c  —  e)       ' 

But  by  referring  to  the  diagram,  it  will  readily  appear  that  scur 
sn  -\-  nc  ;  therefore,  by  addition,  we  obtain 

.36(26  —  c  —  e)  +  (e  —  c)(36  —  2c  —  2e) 

6(26  —  c  —  e) 

Let  this  value  of  I  be   substituted  instead  of  it  in  the  equation 
marked  (148),  and  we  shall  obtain 


and  this  being  reduced  to  its  simplest  general  form,  becomes 
sd8~das(36  —  S}  +  3bvJ(b  —  c)-hDs'(c8  —  e2).      (149). 

210.  The  general  equation  in  the  form  which  it  has  now  assumed, 
is  very  prolix  and  complicated  ;  but  its  complication  and  prolixity,  as 
we  have  before  observed,  are  much  increased  by  the  introduction  of  the 
vertical  pressure;  if  that  element  be  omitted,  the  equation  becomes 

sd3  =  3bDs'(b  —  c)  +  DS'(C*  —  e8).  (150). 

An  expression  sufficiently  simple  for  every  practical  purpose  ;  but 
it  must  be  observed,  that  if  e2  be  greater  than  c2,  the  term  in  which  it 
occurs  will  be  subtractive. 

We  shall  not  attempt  to  express  these  equations  in  words,  or  to 
give  practical  rules  for  their  reduction  ;  the  combinations  are  too 
complex,  to  admit  of  this  being  done  in  a  neat  and  intelligible 
manner;  it  is  necessary,  however,  to  illustrate  the  subject  by  proper 
numerical  examples,  for  which  purpose,  the  following  are  proposed  in 
this  place. 

211.  EXAMPLE  1.  The  water  in  a  reservoir  is  24  feet  deep,  and  the 
wall  which  supports  it  is  30  feet  in  perpendicular  height,  the  slope  of 
the  side  next  the  water  being  one  foot,  and  that  of  the  opposite  side 
one  foot  and  a  half;  it  is  required  to  determine  the  transverse  section 
of  the  wall  or  dyke,  supposing  it  to  be  built  of  materials  whose  mean 
specific  gravity  is  2  J,  that  of  water  being  unity  ? 

By  contemplating  the  conditions  of  the  question  as  here  proposed, 
it  will  readily  be  observed,  that  the  breadth  of  the  section  at  the 
base,  is  the  first  thing  to  be  determined  from  the  equation  ;  for  since 
the  quantity  of  the  slopes,  as  well  as  the  perpendicular  height  are 
given,  the  breadth  of  the  dyke  at  top  can  easily  be  found,  when  the 
breadth  at  the  foundation  is  known. 


198       OF  THE  PRESSURE  OF  FLUIDS  ON  DYKES  AND  EMBANKMENTS. 

In  the  first  place  then,  let  us  take  into  consideration  the  effect 
produced  by  means  of  the  vertical  pressure  of  the  fluid ;  this  will 
refer  us  to  equation  (149),  but  previously  to  the  substitution  of  the 
several  numerical  quantities,  it  becomes  necessary  to  assign  the  nu- 
merical value  of  S,  which  is  not  expressed  in  the  question,  but  is 
determinable  from  the  perpendicular  altitudes  of  the  wall  and  the 
fluid,  together  with  the  slope  of  that  side  on  which  the  fluid  presses : 
thus, 
£•'-'  30  :  24  :  :  1  :  8  =  ±  of  a  foot. 

Let  therefore,  the  several  given  numbers  replace  their  representa- 
tives in  equation  (149),  and  we  shall  have 

248=24X£X2K3&— i)4-3X30x2J(6  — 1)6  — 30X2|(1F— I2), 

in  which  expression,  b  is  the  unknown  quantity. 
If  the  several  terms  be  expanded,  collected,  and  arranged,  accord- 
ing to  the  dimensions  of  the  unknown  quantity,  we  shall  have 

2256s  — 816=13956.15; 

complete  the  square,  and  we  get 

2025006*  —  729006  X  8 12  =  12567096, 

and  extracting  the  square  root,  it  is 


4506—  81=  |/1 2567096  =  3545  nearly; 

therefore,  by  transposition  and  division,  we  get 

b  —  8.05  feet. 

Consequently,  if  from  the  breadth  of  the  foundation  as  above 
determined,  we  subtract  the  sum  of  the  slopes,  the  remainder  will  be 
the  breadth  of  the  dyke  at  the  top ;  hence,  the  section  can  be  deli- 
neated. 

212.  The  above  is  the  method  of  performing  the  operation,  when 
the  effect  produced  by  the  vertical  pressure  of  the  fluid  is  taken  into 
consideration ;  but  when  that  effect  is  omitted,  the  process  is  consi- 
derably shortened  ;  for  in  the  first  place,  there  is  no  occasion  to  cal- 
culate the  value  of  S,  that  term  not  occurring  in  equation  (150),  and 
in  the  next  place,  there  are  fewer  quantities  to  be  substituted  for ; 
this  greatly  abbreviates  the  labour  of  reduction ;  but  the  equation  is 
still  of  the  same  degree,  and  consequently,  it  must  be  resolved  in  the 
same  manner. 

Let  the  several  given  quantities  remain  as  in  the  preceding  case, 
and  let  them  be  respectively  substituted  in  the  equation  (150),  and 
we  shall  obtain 

2256*  —  2256=  13917.75; 


OF  THE  PRESSURE  OF  FLUIDS  ON  DYKES  AND  EMBANKMENTS.        199 

if  all  the  terms  of  this  equation  be  divided  by  225,  the  co-efficient  of 
b*,  we  shall  get 

b*  —  6  =  61.856, 
and  this,  by  completing  the  square,  becomes 


therefore,  by  extracting  the  square  root  and  transposing,  we  have 
b  rz  8  .  1  4  feet  nearly. 

COROL.  It  therefore  appears,  that  under  the  same  circumstances, 
the  computed  breadth  of  the  foundation  differs  very  little,  when  the 
vertical  pressure  of  the  fluid  is  considered,  from  what  it  is  when  the 
pressure  is  omitted  ;  and  what  is  very  remarkable,  the  difference, 
whatever  it  may  amount  to,  leans  to  the  side  of  safety  and  conve- 
nience in  the  case  of  the  omission  ;  it  will  therefore  be  sufficient  in 
all  cases  of  practice,  to  employ  equation  (150),  but  under  certain 
circumstances  of  the  data,  it  will  admit  of  particular  modifications. 

213.  When  the  slopes  c  and  e  are  equal  ;  that  is,  when  the  vertical 
transverse  section  of  the  dyke  or  embankment, 

is  in  the  form  of  the  frustum  of  an  isosceles 
triangle,  as  represented  by  ABC  D  in  the  annexed 
diagram  ;  then,  the  general  equation  (149),  be- 
comes transformed  into 

sd*=idSs(3b  —  S)  +  36D/(6  —  c).    (151). 

If  the  perpendicular  height  of  the  dyke,  and 

the  depth  of  the  fluid,  are  equal  to  one  another  ;  that  is,  if  the  water 
is  on  a  level  with  the  top  of  the  wall  ;  then,  rfzz  D  and  3  =  c,  and  the 
above  equation  becomes 

sd*  =  cs(3b  —  c)  +  3bs'(b  —  c).  (152). 

Again,  if  we  neglect  the  effect  of  vertical  pressure,  and  express  the 
specific  gravity  of  water  by  unity,  we  get 

3s'(tf  —  cb)  =  d\  (153). 

And  finally,  if  both  sides  of  the  equation  be  divided  by  the  quantity 
3/,  we  shall  obtain 

b*-cb  =  37-  (154). 

The  method  of  applying  this  equation  is  manifest,  for  we  have  only 
to  substitute  the  given  numerical  values  of  c,  d  and  /,  and  the  value 
of  b  will  become  known  by  reducing  the  equation. 

214.  EXAMPLE  2.  The  dyke  or  embankment  which  supports  the 
water  in  a  reservoir,  is  20  feet  in  perpendicular  height,  and  it  slopes 
equally  on  both  sides  to  the  distance  of  2  feet  ;  what  is  the  breadth  of 


200         OF  THE  PRESSURE  OF  FLUIDS  ON  DYKES  AND  EMBANKMENTS. 

the  base,  supposing  the  water  to  be  on  a  level  with  the  top  of  the 
wall,  the  specific  gravity  of  the  materials  of  which  it  is  built  being  If, 
that  of  water  being  unity  ? 

Let  these  numbers  be  substituted  for  the  respective  symbols  in  the 
above  equation,  and  we  get 

b*  —  26  —  61.619, 
complete  the  square,  and  it  becomes 

&*  —  26+  1—62.619, 
from  which,  by  evolution  and  transposition,  we  get 

b  —  8.9  13  feet  nearly. 

Here  then,  the  transverse  section  of  the  dyke  is  8.913  feet  across  at 
the  bottom,  and  consequently  it  is  8.913  —  4  —  4.9  13  feet  broad  at 
the  top  ;  hence  the  delineation  is  very  easily  effected. 

215.  If  the  slope  c  should  vanish;  that  is,  if 
the  side  of  the  dyke  on  which  the  fluid  presses  A.JB 

be  vertical,  as  represented  by  A  BCD  in  the  an- 
nexed diagram  ;  then  S  vanishes  also,  and  the 
equation  marked  (149)  becomes 


(155). 


where  it  is  manifest  there  is  no  vertical  pressure 
on  the  dyke,  the  whole  effect  of  the  fluid  being  exerted  in  the  hori- 
zontal direction,  tending  to  turn  the  wall  about  the  remote  extremity 
of  its  base. 

When  the  perpendicular  altitude  of  the  wall  or  dyke,  and  the  depth 
of  the  water  are  equal  ;  then  dizzo,  and  admitting  that  the  value  of 
5,  or  the  specific  gravity  of  water  is  represented  by  unity,  we  obtain 


and  this,  by  transposition  and  division,  becomes 

*-=^' 

and  lastly,  by  extracting  the  square  root,  we  get 


'—  y  — 


(156). 

Let  the  slope  of  the  dyke  be  two  feet,  its  perpendicular  altitude,  or 
the  depth  of  the  fluid  20  feet,  and  the  specific  gravity  of  the  material 
If,  as  in  the  preceding  example;  then,  by  substitution,  we  obtain 


6=i/400+"  =  8.804  feet. 
V    3X1.75 


OF  THE  PRESSURE  OF  FLUIDS  ON  DYKES  AND  EMBANKMENTS.       201 

COROL.  The  breadth  at  the  base,  as  determined  by  this  and  the 
preceding  equation,  exhibits  but  a  small  difference,  being  in  excess  in 
the  former  case,  by  a  quantity  equal  to  0.109  of  a  foot;  but  the 
breadth  at  the  top  in  the  latter  case,  exceeds  that  in  the  former,  by  a 
quantity  equal  to  1.891  feet;  and  the  difference  in  the  area  of  the 
section,  is  17.82  feet:  it  is  consequently  more  expensive  to  erect  a 
dyke  or  embankment,  with  the  side  next  the  fluid  perpendicular,  than 
it  is  to  erect  one  of  equal  stability  with  both  sides  inclined  or  sloping 
outwards. 

216.  If  the  slope  e  should  vanish;  that  is,  if  the  side  of  the  dyke, 
opposite  to  that  on  which  the  fluid  presses,  be 
perpendicular  to  the  horizon,  as  represented  by 
A  BCD  in  the  annexed  diagram,  then,  the  equa- 
tion (149)  becomes 
sd*=d$s(3b— £)-}-3Ds'(6?—  c£)4-DcY.  (157). 

But  when  the  effect  of  the  vertical  pressure  of 
the  fluid  is  omitted,  we  obtain 

sds=i  SDS'  (b*  —  c&)  -f-  DC9 s',  (158). 

and  by  supposing  the  altitude  of  the  dyke,  and  the  depth  of  the  fluid 
to  be  equal  (the  specific  gravity  of  the  fluid  being  expressed  by  unity) ; 
then  we  have  d  — D,  and  the  foregoing  equation  becomes 

d*  =  3s'(b*  —  c&)4-cV; 
consequently,  by  transposition  and  division,  we  get 

A*  I  —  *—  CV 

~~37~  (159). 

from  which  equation,  the  value  of  b  is  easily  determined. 
Let  the  slope  of  that  side  of  the  dyke  on  which  the  fluid  presses,  be 
equal  to  2  feet,  and  the  perpendicular  altitude  of  the  dyke,  or  the 
depth  of  the  fluid  20  feet,  the  specific  gravity  of  the  material  being 
If  as  before ;  then  by  substitution,  the  foregoing  equation  becomes 

,-„==£$!!=««,, 

by  completing  the  square,  we  obtain 
p  —  2b  +  1  =  74.8571  4-  1—75.8571  ; 

consequently,  by  extracting  the  square  root  and  transposing,  we  get 

b  =  9.709  feet. 

In  this  case,  the  dyke  has  less  stability  than  it  has  when  the  perpen- 
dicular side  is  towards  the  water,  as  is  manifest  from  its  requiring  a 
greater  section,  and  consequently,  a  greater  quantity  of  materials  to 
resist  the  effort  of  the  pressure  which  tends  to  overturn  it. 


202        OF  THE  PRESSURE  OF  FLUIDS  ON  DYKES  AND  EMBANKMENTS. 

The  sectional  area  in  the  one  case,  is  7.  804  X  20  =:  156.08  square 
feet,  and  in  the  other,  it  is  8.709x20=:  174.18  square  feet,  being  a 
difference  of  18.1  square  feet  in  favour  of  magnitude  in  the  latter 
form,  where  the  sloping  side  is  adjacent  to  the  fluid  ;  and  this  being 
multiplied  by  the  length  of  the  dyke,  will  give  the  extra  quantity  of 
materials  necessary  for  obtaining  the  same  degree  of  stability. 

217.  If  both  the  slopes  c  and  e  become  evanescent;  that  is,  if  the 
section  of  the  dyke  be  rectangular,  having  both  its 
sides  perpendicular  to  the  horizon,  as  represented 
by  ABC  D  in  the  annexed  diagram  ;  then,  the  general 
equation  (149),  becomes  transformed  into 

sda  =  3vs'b\  (160). 

Then,  by  supposing  the  depth  of  the  fluid,  and 
the  perpendicular  altitude  of  the  dyke  to  become 
equal,  (the  specific  gravity  of  water  being  expressed  by  unity,)  we 
have 

3s'&2zrd8; 

and  this  by  division  becomes 

-I- 

consequently,  if  the  square  root  of  both  sides  of  this  equation  be 
extracted,  we  shall  have 


'-  (161). 

218.  This  is  indeed  a  very  simple  form  of  the,  equation,  applicable 
to  the  very  important  case  of  rectangular  walls  ;  it  is  however  accu- 
rate, and  corresponds  in  form  with  that  investigated  by  other  writers 
for  the  same  purpose,  and  by  different  methods  ;  the  mode  of  its 
reduction  is  simply  as  follows. 

RULE.  Divide  the  specific  gravity  of  the  fluid  to  be  sup- 
ported, by  three  times  the  specific  gravity  of  the  dyke  or 
embankment,  and  multiply  the  square  root  of  the  quotient 
by  the  perpendicular  altitude  of  the  dyke,  for  the  required 
thickness. 

Let  the  perpendicular  depth  of  the  water,  or  the  altitude  of  the 
dyke  be  equal  to  20  feet,  and  the  specific  gravity  of  the  materials  of 
which  it  is  built  If,  as  in  the  foregoing  cases;  then,  by  proceeding 
according  to  the  rule,  we  have 


OF  THE  PRESSURE  OF  FLUIDS  ON  DYKES  AND  EMBANKMENTS.       203 

219.  There  is  still  another  case  of  very  frequent  occurrence  that 
remains  to  be  considered,  viz.  that  in  which  the  section  is  in  the  form 
of  a  right  angled  triangle,  having  its  vertex  on  the  same  level  with 
the  surface  of  the  fluid. 

This  case  will  also  admit  of  two  varieties,  according  as  the  perpen- 
dicular side  of  the  dyke  is,  or  is  not  in  contact  with  the  fluid  ;  when 
it  is  in  contact  with  it  c  vanishes,  and  since  the  section  is  in  the  form 
of  a  triangle,  the  breadth  of  the  base  b  is  equal  to  the  remote  slope  e, 
and  the  vertical  pressure  of  the  fluid  on  the  dyke  is  evanescent;  con- 
sequently, the  equation  marked  (149)  becomes 

sd*  =  3Ds'P  —  DsV;  (162). 

but  by  the  nature  of  the  problem  e2  is  equal  to  62,  and  by  the  hypo- 
thesis of  equal  altitudes  c?=z=  D  ;  therefore,  in  the  case  of  water,  whose 
specific  gravity  is  expressed  by  unity,  we  obtain 

Zs'tfi^d*; 
and  from  this,  by  division,  we  get 


and  finally,  by  extracting  the  square  root,  it  is 

b  —  d\/  _L 

V    2s''  (163). 

220.  This  is  also  a  very  simple  expression  for  the  base  of  the  section, 
and  the  rule  for  its  reduction  is  simply  as  follows. 

RULE.  Divide  the  specific  gravity  of  the  incumbent  fluid, 
by  twice  the  specific  gravity  of  the  dyke  or  embankment,  and 
multiply  the  perpendicular  depth  of  the  fluid  by  the  square 
root  of  the  quotient,  for  the  required  thickness  of  the  dyke. 

Let  the  perpendicular  altitude  and  the  specific  gravity  of  the  wall, 
be  20  feet  and  1  j  respectively,  as  in  the  foregoing  cases,  and  we  shall 
have 


6  =  2<V2-^5=  10.68  feet. 

221.  Lastly,  if  the  fluid  come  in  contact  with,  or  press  upon  the 
hypothenuse  of  the  triangle ;  then  the  slope  e  vanishes,  and  b  and  c 
are  equal ;  consequently,  equation  (149)  becomes 

and  if  the  vertical  pressure  of  the  fluid  be  omitted,  the  first  term  on 
the  right  hand  side  of  the  equation  vanishes,  and  consequently,  we  get 

sd*=vs'b*; 


204       OF  THE  PRESSURE  OF  FLUIDS  ON  DYKES  AND  EMBANKMENTS. 

but  C?:=D,  and  s=z»l  ;  therefore,  we  shall  have 
and  finally,  by  division  and  evolution,  we  obtain 

t=i^ 

222.  This  equation  is  of  a  still  simpler  form  than  that  which  arises 
when  the  perpendicular  side  is  towards  the  pressure,  and  the  rule  for 
its  reduction  is  as  follows. 

RULE.  Divide  the  specific  gravity  of  the  fluid,  by  that  of 
the  dyke  or  embankment,  and  multiply  the  perpendicular  alti- 
tude by  the  square  root  of  the  quotient,  for  the  breadth  of 
the  base. 

Therefore,  by  taking  the  altitude  and  specific  gravities  hitherto 
employed,  the  rule  will  give 

6  =  20A/  —  =15.11  feet. 
V     1.75 

223.  This  gives  a  thickness  for  the  base  of  the  section,  exceeding 
the  thickness  in  the  former  case  by  4.33  feet;  which  seems  to  be  a 
very  great  difference,  when  it  is  considered  that  both  the  form  and 
the  perpendicular  altitude  of  the  wall  are  the  same  in  both  cases ;  but 
the  reason  of  the  difference  will  become  manifest  from  the  following 
construction. 

Let  ABC  and  a  be  be  two  right  angled  triangles,  equal  to  one 
another  in  every  respect,  but  having 
their  perpendiculars  opposed  in  such  a 
manner,  that  the  water  pressing  in  the 
same  horizontal  direction,  is  resisted  by 
the  perpendicular  AB  in  the  one  case, 
and  by  the  hypothenuse  ac  in  the  other. 

Bisect  the  sides  AB,  AC  in  the  point 
M  and  N,  and  a  b,  ac  in  the  points  m 

and  n  respectively;  draw  the  lines  CM  and  BN  intersecting  in  G,  and 
cm  and  bn  intersecting  in  g;  then  are  G  and  g,  the  centres  of  gravity 
of  the  respective  triangles  ABC  and  a  be. 

Demit  the  straight  lines  GH  and  gh,  perpendicularly  to  BC  and  be; 
then  are  HC  and  h  b  the  levers,  by  which  the  weights  of  the  sections, 
supposed  to  be  concentrated  in  their  respective  centres  of  gravity, 
resist  the  horizontal  pressure  of  the  fluid  which  tends  to  turn  them 
round  the  points  c  and  b. 


OF  THE  PRESSURE  OF  FLUIDS  ON  DYKES  AND  EMBANKMENTS.       205 

Now,  according  to  the  property  of  the  centre  of  gravity,  HC  is 
equal  to  two  thirds  of  BC,  while  h  b  is  only  one  third  of  be  ;  but  the 
horizontal  pressure  of  the  water  is  the  same  in  both  cases  ;  it  will 
therefore  require  the  same  mechanical  energy  to  resist  it;  and  since, 
by  the  conditions  of  the  problem,  the  altitudes  A  B  and  a  b  are  equal, 
it  follows,  that  in  order  to  produce  an  equilibrium,  the  product  of  the 
base  of  the  triangle  a  be,  into  the  length  of  the  lever  h  b,  must  be 
increased  in  such  a  manner,  that 

BCXHC  =  6cX^&,  (166). 

and  by  converting  this  equation  into  an  analogy,  it  becomes 
BC  :  be  :  :  hb  :  HC. 

We  have  seen,  that  by  the  construction  and  the  property  of  the 
centre  of  gravity,  the  lever  HC  is  equal  to  two  thirds  of  BC,  and  h  b 
equal  to  one  third  of  be;  let  therefore,  fsc  and  %bc,  be  substituted 
for  HC  and  h  b,  in  the  equation  marked  (166),  and  we  shall  obtain 


or  dividing  both  terms  by  -^  ,  it  becomes 

2BC2  =  £C2, 

and  finally,  by  extracting  the  square  root,  it  is 


(167). 

Hence  the  reason  for,  and  the  nature  of  the  increased  breadth 
become  obvious,  the  one  being  the  side,  and  the  other  the  diagonal 
of  a  square. 

Now,  we  have  found  that  the  breadth  of  the  dyke  at  the  base,  is 
equal  to  10.68  feet,  when  the  perpendicular  side  is  in  contact  with 
the  fluid  ;  consequently,  when  the  pressure  is  exerted  on  the  hypo- 
thenuse,  we  have 

6=10.68X1.4142  =  15.11  feet, 

being  the  very  same  result  as  that  which  we  obtained  from  the  reduc- 
tion of  the  equation  (165). 

224.  What  we  have  hitherto  done,  has  reference  to  the  case  in 
which  the  obstacle  yields  to  the  pressure  of  the  fluid,  by  turning  upon 
the  remote  extremity  of  its  base  ;  we  have  therefore,  in  the  next  place, 
to  investigate  the  conditions  of  equilibrium,  when  the  obstacle  is  sup- 
posed to  yield,  by  sliding  along  the  horizontal  plane  on  which  it  is 
erected. 

Since  the  base  of  the  dyke  or  wall  is  horizontal,  it  is  manifest  that 
the  mass  which  it  sustains,  resists  the  horizontal  pressure  of  the  fluid, 
only  by  its  adhesion  to  the  base,  and  the  resistance  occasioned  by 
friction. 


206        OF  THE  PRESSURE  OF  FLUIDS  ON   DYKES  AND   EMBANKMENTS. 

Suppose  therefore,  that  the  resistances  of  adhesion  and  friction,  are 
equal  to  n  times  the  weight  of  the  dyke,  which  we  have  represented 
by  w  ;  then  we  have 

nw  —  \d*s\ 
but  we  have  shown,  in  the  investigation  of  the  preceding  case,  that 

w  =  jD/(26  —  c  —  e}\ 
consequently,  by  substitution,  we  obtain 

d2s  =  i>nJ(2b  —  c  —  e).  (168). 

This  is  the  equation  of  equilibrium,  or  that  in  which  the  resistance 
of  the  dyke  is  counterpoised  by  the  horizontal  pressure  of  the  fluid, 
the  effect  of  the  vertical  pressure  not  being  considered  ;  but  in  order 
to  express  the  breadth  of  the  base  in  terms  of  the  other  quantities,  let 
both  sides  of  the  equation  be  divided  by  ons',  and  it  becomes 


D7ZS 

consequently,  by  transposition  and  division,  we  obtain 

d*s 

6  =  2^7+i(c  +  e):  (169). 

and  finally,  if  the  perpendicular  depth  of  the  fluid  and  the  height  of 
the  dyke  are  equal,  we  shall  have 


-  -     .         (170). 

225.  In  order  therefore,  to  illustrate  the  reduction  of  the  above 
equation;1by  means  of  a  numerical  example,  we  must  assume  a  value 
to  the  letter  n,  having  some  relation  to  the  nature  of  the  materials  of 
which  the  resisting  obstacle  is  constructed  ;  now,  it  has  been  found 
by  numerous  experiments,  that  when  rough  and  uneven  bodies  rub 
upon  one  another,  or  when  a  heavy  body  composed  of  hard  and 
rough  materials,  is  urged  along  a  horizontal  plane,  the  effect  of  the 
friction  is  equivalent  to  about  one  third  of  the  weight  of  the  body 
moved  ;  or  in  other  words,  it  requires  about  one  third  part  of  the  force 
applied  to  overcome  the  effects  of  the  friction  ;  and  moreover,  in  the 
case  of  a  wall  built  of  masonry,  there  is,  in  addition  to  the  friction, 
the  adhesion  of  the  materials  to  the  plane  on  which  the  wall  is  built. 

If  therefore,  we  consider  the  effect  of  adhesion  to  be  equivalent  to 
the  effect  of  friction,  it  is  manifest,  that  their  conjoint  effects  will 
destroy  about  two  thirds  of  the  force  applied  ;  consequently,  in  the 
case  of  masonry,  we  may  suppose  that  the  value  of  w,  is  very  nearly 
equal  to  1J,  but  for  other  materials  it  will  vary  according  to  the 
specific  gravity  or  weight. 


OF  THE  PRESSURE  OF  FLUIDS  ON  DYKES  AND  EMBANKMENTS.       207 

Having  thus  assigned  a  particular  value  to  the  letter  n,  we  shall 
next  proceed  to  illustrate  the  reduction  of  the  equation  ;  for  which 
purpose,  take  the  following  example. 

226.  EXAMPLE  3.  The  vertical  transverse  section  of  the  wall  which 
supports  the  water  in  a  reservoir,  is  24  feet  in  perpendicular  height  ; 
what  is  the  thickness  at  the  base  of  the  wall,  supposing  the  section  to 
be  in  the  form  of  the  frustum  of  an  isosceles  triangle,  the  slope  or 
inclination  on  each  side,  being  equal  to  2  feet,  and  the  specific  gravity 
of  the  material  If,  that  of  water  being  expressed  by  unity  ? 

Let  the  several  numerical  values  here  specified,  be  substituted 
instead  of  the  respective  symbols  in  the  equation  (170),  and  we  shall 
obtain 

_24>O_         ___  nearly. 


The  breadth  of  the  section,  or  the  thickness  of  the  dyke  at  the 
bottom,  being  thus  determined,  the  breadth  or  thickness  at  the  top 
can  easily  be  found,  for  we  have 

6.571  —  4  =  2.571  feet. 

127.  If  the  slope  c  should  vanish;  that  is,  if  the  side  of  the  dyke 
on  which  the  water  presses  be  perpendicular  to  the  horizon  ;  then,  the 
equation  (170),  becomes 


•  (171). 

And  if  the  opposite  slope  e  becomes  evanescent,  while  the  slope  c 
remains  ;  then  we  have 

*  =  2^7+ic<  (172). 

But  if  both  slopes  vanish,   or  the  section   of  the  wall  becomes 
rectangular;  then,  the  equation  (170)  is 


(173). 

If  therefore,  the  perpendicular  altitude  of  the  section,  and  the 
specific  gravity  of  the  materials  of  which  the  dyke  is  composed,  remain 
as  in  the  preceding  example  ;  then  we  shall  have 

b=   24X1    =4.571  feet. 
2XfX£ 

228.  When  the  section  of  the  wall  assumes  the  form  of  a  right 
angled  triangle  ;  that  is,  when  the  slope  c  vanishes,  and  e  becomes 
equal  to  the  whole  breadth  b  ;  then  we  have 


208        OF  THE  PRESSURE  OF  FLUIDS  ON  DYKES   AND  EMBANKMENTS. 

ds 

-W'  (174). 

And  exactly  the  same  equation  would  arise,  if  the  slope  e  remote 
from  the  fluid  were  to  vanish,  and  the  slope  c  adjacent  to  the  fluid, 
become  equal  to  b  the  whole  breadth  of  the  section;  consequently, 
the  thickness  of  a  dyke  in  the  case  of  a  triangular  section,  whether 
the  water  presses  on  the  perpendicular  or  hypothenuse  of  the  tri- 
angle, is 


In  all  the  preceding  cases,  it  is  supposed  that  the  section  of  the 
dyke  or  embankment  is  of  such  dimensions,  as  to  oppose  an  equipois- 
ing resistance  to  the  pressure  of  the  fluid  which  it  supports  ;  but  in 
the  actual  construction  of  all  works  of  this  nature,  it  becomes  neces- 
sary, for  the  sake  of  safety,  to  enlarge  the  dimensions  considerably 
beyond  what  theory  assigns  to  them  ;  but  it  does  not  belong  to  this 
place  to  determine  the  limits  of  the  enlargement. 

2.    OF    THE    PRESSURE    OF    FLUIDS    AGAINST    EMBANKMENTS    OF    LOOSE 

MATERIALS. 

229.  The  theory  which  we  have  established  above,  supposes  that  a 
perfect  connection  obtains  between  all  the  parts  of  the  dyke  or 
embankment  which  is  opposed  to  the  pressure  of  the  fluid,  so  that  any 
one  portion  of  it  cannot  be  displaced  or  overthrown,  unless  the  whole 
be  overthrown  at  the  same  time;  the  formulge  thence  arising,  are 
therefore,  only  applicable  to  dykes  or  embankments  that  are  con- 
structed of  masonry  ;  in  those  which  are  constructed  of  earth  or  other 
loose  materials,  and  having  the  sides  faced  or  fortified  with  stone,  the 
same  connection  between  the  component  portions  of  the  wall  does  not 
exist,  and  consequently,  although  the  several  equations  apply  when 
the  whole  perpendicular  height  of  the  dyke  is  considered,  yet  the  dyke 
will  not  resist  equally  at  every  part  of  the  height,  but  is  liable  to 
be  separated  into  horizontal  sections. 

In  order  therefore,  to  adapt  our  prin- 
ciples to  this  case  also,  it  becomes  neces- 
sary to  trace  out  the  steps  of  another 
investigation  ;  for  which  purpose, 

Let  ACD  represent  a  vertical  section  of 
the  dyke  or  embankment,  whose  summit 
at  A  is  on  a  level  with  the  surface  of  the  fluid  AE. 


OF  THE  PRESSURE    OF  FLUIDS  ON  DYKES  AND  EMBANKMENTS.       209 

Take  any  point  G  in  the  line  AGD,  and  through  the  point  o  thus 
assumed,  draw  the  horizontal  ordinate  GB,  cutting  the  vertical  axis 
AC  in  the  point  B  :  now,  it  is  required  to  determine  the  nature  of  the 
curve  AGD  such,  that  each  portion  of  the  dyke,  or  of  its  section,  as 
AGE,  estimated  from  the  vertex,  may  be  equally  capable  of  resisting 
the  horizontal  pressure  of  the  fluid  exerted  against  AG  ;  or  which  is 
the  same  thing,  that  each  portion  may  retain  its  stability  and  remain 
in  equilibrio  on  its  base  GB;  not  separating  from  the  lower  portion 
GBCD,  either  by  turning  about  the  point  c  as  a  centre  of  motion,  or 
by  sliding  in  a  horizontal  direction  along  the  base  GB. 

Put  x  —  AE,  the  abscissa  of  the  curve  estimated  from  the  vertex 

at  A, 
y  =  EG,  the   horizontal   ordinate   corresponding  to  the  ab- 

scissa x, 
s  z=  the  specific  gravity  of  the  fluid,  which  endeavours  to 

displace  the  dyke  by  pushing  it  along  the  lin«  BG, 
/  zn  the  specific  gravity  of  the  materials  of  which  the  dyke 

is  constituted, 

m  m  the  momentum  of  the  horizontal  pressure, 
m'—  the  momentum  of  the  resistance  offered  by  the  dyke,  and 
n  :=z  the  number  of  times  that  the  adhesion  and  friction  of  the 

dyke  are  equal  to  its  weight. 

Then  we  have  already  seen,  equation  (145),  that  the  momentum  of 
the  horizontal  pressure  of  the  fluid  as  referred  to  the  point  c,  is 


from  which,  by  substituting  xs  instead  of  d3,  we  obtain  m  — 
which  equation  indicates  the  momentum  of  pressure  at  the  point  D. 

But  the  momentum  of  the  resistance  offered  by  the  wall,  that  is, 
the  momentum  of  the  portion  of  the  section  represented  by  ABG,  is 

m'  =  is'fy*x; 

and  these  momenta  in  the  case  of  an  equilibrium  must  be  equal  to 
one  another  ;  hence  we  have 


from  which,  by  taking  the  fluxion,  we  shall  obtain 

or  by  suppressing  the  common  factors,  it  becomes  sx^  —  s'y*; 
by  extracting  the  square  root  of  both  terms,  we  get  x\/s  —  y\/s'. 
Now,  when  x  becomes  equal  to  d,  the  whole  perpendicular  depth 
of  the  fluid,  or  the  altitude  of  the  section  ;  then  y  becomes  equal  to 
b,  the  thickness  of  the  dyke,  or  the  greatest  breadth  of  the  section  ; 
VOL.  i.  p 


210       OF  THE  PRESSURE  OF  FLUIDS  ON  DYKES  AND  EMBANKMENTS. 

consequently,  if  d  and  b  be  respectively  substituted  for  x  and  y  in 
the  preceding  equation,  we  shall  have 

d^/7—b^S  (175). 

This  equation  involves  the  conditions  necessary  for  preventing  the 
dyke  from  turning  about  the  point  B,  and  if  the  equation  be  resolved 
into  an  analogy,  we  shall  have  b  :  d  :  :  \/s  :  ^/sf. 

COROL.  From  which  we  infer,  that  the  section  is  in  the  form  of  a 
rectilinear  triangle,  whose  base  is  to  the  perpendicular  height,  as  the 
square  root  of  the  specific  gravity  of  the  fluid,  is  to  the  square  root  of 
the  specific  gravity  of  the  wall  or  dyke. 

230.  EXAMPLE.  The  perpendicular  altitude  of  an  embankment  of 
earth  is  20  feet ;  what  must  be  the  breadth  of  its  base,  so  that  each 
portion  of  it  estimated  from  the  vertex,  shall  resist  the  effort  of  the 
fluid,  to  turn  it  round  the  remote  extremity  of  the  base,  with  equal 
intensity ;  the  water  and  the  dyke  having  equal  altitudes,  and  their 
specific  gravities  being  1  and  1 .5  respectively  ? 

Here  we  have  given  rfrz  20  feet,  szr  1,  and  sr  zz  1 .5  ;  consequently, 
by  the  preceding  analogy,  we  have  ^  1.5  :  ^  1  :  :  20  :  16.33  feet. 

231.  The  conditions  necessary  for  preventing  the  portion  of  the 
section  ABG  from  sliding  on  its  base,  may  be  thus  determined. 

We  have  seen  (art.  220),  that  the  momentum  of  the  horizontal 
pressure,  to  urge  the  section  along  its  base,  is  m—  Jse?% 

consequently,  by  substituting  x9  for  d2,  we  have  m  —  Jsa:2, 

but  the  momentum  of  the  section  opposed  to  this,  is  m'  zz  ns'fyx; 

therefore  in  the  case  of  an  equilibrium,  we  have  isx*  =  ns'fyx, 

from  which,  by  taking  the  fluxion,  we  obtain  sxx~  ns'yx, 
and  by  casting  out  the  common  factor,  we  get  sx^nns'y.     (176). 
From  this  equation,  when  converted  into  an  analogy,  we  shall  obtain 
x  :  y  :  :  ntf  :  s. 

Which  also  indicates  a  rectilinear  triangle,  whose  altitude  is  to  the 
base,  as  n  times  the  specific  gravity  of  the  embankment,  is  to  the 
specific  gravity  of  the  fluid. 

If  the  water  presses  against  the  perpendicular  side  of  the  wall,  the 
curve  bounding  the  other  side,  so  that  the  strength  of  the  wall  may 
be  every  where  proportional  to  the  pressure  which  it  sustains,  must  be 
a  semi-cubical  parabola,  whose  vertex  is  at  the  surface  of  the  fluid, 
and  convex  towards  the  pressure. 


OF  THE  PRESSURE  OF  FLUIDS  ON  DYKES  AND  EMBANKMENTS.       211 

232.  We  are  now  arrived  at  that  particular  division  of  our  subject, 
which  comprehends  some  of  the  most  interesting  and  important 
departments  of  hydrodynamical  science ;  it  unfolds  the  principles  of 
floatation,  explains  the  method  of  weighing  solid  bodies  in  fluids, 
determines  the  relations  of  their  specific  gravities ;  and  moreover,  it 
investigates  the  laws  of  equilibrium,  and  assigns  the  conditions  neces- 
sary for  a  state  of  perfect  or  imperfect  stability.  Every  term  in  this 
enumeration  conveys  the  idea  of  mechanical  action. 

Floating  bodies,  those  which  swim  on  the  surface  of  a  fluid,  which  is  bulk  for 
bulk  heavier  than  the  body  afloat,  are  pressed  downward  by  their  own  weight  in  a 
vertical  line  passing  through  their  centre  of  gravity  :  and  they  are  supported  by 
the  upward  pressure  of  the  fluid,  which  acts  in  a  vertical  line  passing  through  the 
centre  of  gravity  of  the  part  which  is  under  the  water.  When  these  lines  are  coin- 
cident, the  equilibrium  of  floatation  will  be  permanent.  In  the  present  instance  we 
have  merely  to  consider  the  principles  of  floatation  as  fluids  exhibit  the  properties 
of  the  mechanical  powers,  as  the  lever  or  balance,  the  screw,  &c.  The  pulley,  in 
lowering  a  great  weight  or  in  lifting  it  up  again,  does  no  more  than  the  ocean  tide 
when  it  silently  recedes  and  leaves  dry,  or  majestically  advances  and  without  effort 
floats  a  stupendous  ship.  The  lever  or  balance  does  no  more  than  a  canal  lock 
effects,  when  it  transfers  from  one  level  to  another  a  heavy  barge  or  vessel  laden 
with  ponderous  commodities.  And  we  behold  too  the  ocean,  like  a  vast  screw  or 
press,  forcing  down  to  its  dark  recesses  vast  masses,  which  in  shipwrecks  are  sub- 
merged in  its  bosom,  and  which  yet  might  be  fashioned  to  be  bulk  for  bulk  much 
lighter  than  the  devouring  flood  which  has  swallowed  them  up  in  its  insatiable 
womb.  The  eternal  and  immutable  laws  of  Nature,  in  all  these  cases,  are  most 
satisfactorily  accounted  for  in  the  doctrine  of  fluid  pressure  and  support  j  but  this 
doctrine,  like  all  the  rudiments  of  human  skill  applied  to  natural  phenomena,  must 
depend  on  matters  of  fact,  which  can  only  be  learned  from  observation  and  experi- 
ment, and  which  can  generally  and  successfully  be  applied  by  the  help  of  mathema- 
tical and  philosophical  investigations.  This  is  the  only  scientific  view  we  ought 
to  take  of  all  those  truths  that  are  denominated  the  phenomena  of  fluids,  whose 
affections,  from  a  series  of  concurring  experiments,  we  undertake  to  expound  j  or 
assuming  these  as  established  pi-inciples  that  operate  generally  in  the  pressure  and 
elasticity  of  fluids,  we  demonstrate  them  to  be  adequate  to  the  production,  not  only 
of  the  particular  effects  adduced  to  prove  their  existence  and  power,  but  of  all 
similar  phenomena.  This  is  the  only  method  by  which  to  make  the  results  of 
practical  men  available  in  scientific  discussions,  and  on  the  other  hand  render  these 
discussions  the  handmaids  of  genius  in  constructive  mechanics.  This  is  the  province 
of  the  mathematician;  and  we  shall  in  the  sequel  follow  it  very  closely,  in  expound- 
ing the  doctrine  of  floatation  and  the  specific  gravities  of  bodies,  the  laws  of  equili- 
brium, and  the  conditions  necessary  for  a  state  of  perfect  or  imperfect  stability,  &c. 


P  2 


CHAPTER  IX. 

OF     FLOATATION,     AND    THE     DETERMINATION     OF    THE     SPECIFIC 
GRAVITIES    OF    BODIES    IMMERSED    IN    FLUIDS. 

OF  the  several  particulars  with  which  we  concluded  the  last 
chapter,  we  shall  speak  in  order,  beginning  with  the  theory  of  float- 
ation and  the  determination  of  the  specific  gravity  of  bodies,  the 
leading  principles  of  which  are  contained  in  the  following  proposition. 

PROPOSITION  III. 

233.  When  a  body  floats,  or  when  it  is  in  a  state  of  buoyancy 
on  the  surface  of  a  fluid  of  greater  specific  gravity  than  itself: — 

It  is  pressed  upwards  by  a  force,  whose  intensity  is  equi- 
valent to  the  absolute  weight  of  a  quantity  of  the  fluid,  of 
which  the  magnitude  is  the  same  as  that  portion  of  the  body 
below  the  plane  of  floatation.* 

Let  ABC  represent  a  vertical  section  of  a  solid  body  floating  on  a 
fluid,  whose  horizontal  surface  is  DE,  mn  being  the  plane  of  floata- 
tion, and  men  the  immersed  portion  of  the  floating  body. 

Take  any  two  points  G  and  H  on 
the  surface  of  the  solid,  indefinitely 
near  to  each  other,  and  through  the 
points  G  and  H  thus  arbitrarily  as- 
sumed, draw  the  straight  lines  G  F 
and  HI,  respectively  parallel  to  DE 

the  surface  of  the  fluid,  and  meeting  the  opposite  sides  of  the  solid  in 
the  points  r  and  i,  so  that  each  point  in  either  of  the  intercepted 
portions  GH  and  FI,  may  be  considered  as  being  at  the  same  perpen- 
dicular depth  h&  or  ZF  below  the  horizontal  surface  of  the  fluid. 

At  H  and  i  erect  the  perpendiculars  H  r  and  i  s,  which  produce  to 
t  and  w,  and  through  the  points  G  and  F,  draw  the  straight  lines  G  b 
and  F/,  respectively  perpendicular  to  the  surface  of  the  solid  in  the 

*  The  Plane  of  Floatation  is  the  imaginary  plane,  in  which  the  floating  solid  is 
supposed  to  be  intersected  by  the  horizontal  surface  of  the  fluid. 


OF   FLOATATION   AND  THE  SPECIFIC  GRAVITY  OF  BODIES.  213 

points  G  and  F  ;  make  ob  and  F/each  equal  to  /IG  or  ZF,  the  perpen- 
dicular depth  of  the  points  G  and  F  below  the  surface  DE;  then, 
according  to  the  principles  which  we  have  propounded  and  demon- 
strated in  the  first  proposition  and  its  subordinate  inferences,  the 
perpendicular  pressures  upon  the  indefinitely  small  portions  of  the 
body  GH  and  FI,  may  be  expressed  as  follows,  viz. 

/>  zr  s XG  H  X G  &,  and p' m s X F i X F/, 

where  s  denotes  the  specific  gravity  of  the  fluid,  and  p,  j/  the  respec- 
tive pressures  exerted  by  it  perpendicularly  to  GH  and  FI,  any 
indefinitely  small  portions  of  the  floating  body. 

But  it  is  manifest  from  the  resolution  of  forces,  that  the  pressures 
of  the  fluid  in  the  directions  bo  and  y*F,  may  each  be  decomposed 
into  two  other  pressures,  the  one  vertical  and  the  other  horizontal ; 
for  by  completing  the  rectangular  parallelograms  oabc  and  Tdfe,  it 
is  obvious  that  the  pressures  in  the  directions  «G,  CG  and  dr,  ev  are, 
when  taken  two  and  two,  respectively  equivalent  to  the  pressures  in 
the  directions  bo  andy*F. 

Now,  the  horizontal  pressures  CD  and  CF,  by  construction  are  equal 
to  one  another,  and  they  operate  in  contrary  directions;  consequently 
they  destroy'each  other's  effects,  and  the  upward  vertical  pressures  on 
the  solid  at  the  points  G  and  F,  are  respectively  indicated  by  the 
straight  lines  «G  and  C?F  drawn  into  the  specific  gravity  of  the  fluid; 
therefore,  the  whole  vertical  pressures  on  the  indefinitely  small  por- 
tions GH  and  FI,  are  as  follows,  viz. 

j9  =  5XGHX«G,  and  jy'zzrsXFiX^F, 

where  p  and  p',  instead  of  indicating  the  perpendicular  pressures  as 
formerly,  are  now  considered  in  reference  to  the  vertical  pressures. 

Since  the  parallel  straight  lines  GF  and  HI  are  indefinitely  near  to 
one  another,  the  lines  G  n  and  FI  may  be  assumed  as  nearly  straight, 
and  consequently,  the  elementary  triangles  Giir  and  FIS  are  respec- 
tively similar  to  the  triangles  GBO  and  F/W;  therefore,  by  the  pro- 
perty of  similar  triangles,  we  have 

G  6  :  G  a  :  :  G  H  :  G  r,  and  $f:  F  d  :  :  F  i  :  F  s ; 

and  from  these  analogies,  by  equating  the  products  of  the  extreme 
and  mean^terms,  we  obtain 

G&XGnziGaXGH,  an(j  Fy  XFSZHFC?  XFI. 

Let  therefore,  the  products  G&Xor  and  F/XFS  be  substituted 
instead  of  GiiXctG  and  FiXc?r  in  the  above  values  of  p  and  //,  and 
we  shall  have 

/;  —  s  X  G  b  X  G  r,  and  p  m  s  X  F/  X  F  s. 


214          OF  FLOATATION  AND  THE  SPECIFIC  GRAVITY  OF  BODIES. 

Now,  these  pressures  are  manifestly  equal  to  the  weights  of  the 
columns  Gt  and  vu  considered  as  fluid,  and  since  the  same  may  be 
demonstrated  with  respect  to  every  other  portion  of  the  immersed 
surface,  we  therefore  conclude,  that  the  whole  pressure  upwards,  is 
equal  to  the  sum  of  the  weights  of  all  the  columns  Gt,  FU,  &c. ;  that 
is,  to  the  weight  of  a  quantity  of  the  fluid  equal  in  magnitude  to  the 
immersed  part  of  the  body ;  hence  the  truth  of  the  proposition  is 
manifest. 

COROL.  From  the  principles  demonstrated  above,  it  follows,  that 
when  a  solid  body  floating  on  the  surface  of  a  fluid  is  in  a  state  of 
quiescence : — 

The  pressure  downwards  is  equal  to  the  buoyant  effort ; 

that  is,  the  weight  of  the  floating  body,  is  equal  to  the  weight 

of  a  quantity  of  the  fluid,  whose  magnitude  is  the  same  as 

that  portion  of  the   solid,  which  falls  below   the  plane  of 

floatation. 

PROBLEM  XXXI. 

234.  A  cylindrical  vessel  of  a  given  diameter,  is  filled  to  a 
certain  height  with  a  fluid  of  known  specific  gravity,  and  a 
spherical  body  of  a  given  magnitude  and  substance  is  placed 
in  it : — 

It  is  required  to  determine  how  high  the  fluid  will  rise  in 
consequence  of  the  immersion  of  the  spherical  segment  which 
falls  below  the  plane  of  floatation. 

Let  ABCD  represent  a  vertical  section  passing  along  the  axis  of  a 
cylindrical  vessel,  filled  with  an  incompressible  and 
non-elastic  fluid  to  the  height  ED,  EF  being  the  sur- 
face of  the  fluid  before  the  sphere  whose  diameter  is 
mn,  is  placed  in  it,  and  ab  the  surface  after  the 
immersion  of  the  segment  tnu,  the  liquid  rising  to  the 
height  A  D. 

Then  it  is  manifest  from  the  nature  of  the  problem, 
that  the  spherical  segment  tvnwu,  together  with  the  quantity  of  fluid 
in  the  vessel,  must  be  equal  to  the  capacity  of  the  cylinder  whose 
diameter  is  DC,  and  perpendicular  altitude  «D;  for  the  fluid  rises  in 
consequence  of  the  immersion  of  the  segment,  and  fills  the  spaces 
atvE  and  buw^s  all  around  the  vessel ;  we  have  therefore  to  calculate 


OF  FLOATATION  AND  THE  SPECIFIC  GRAVITY  OF  BODIES.          215 

the  spherical  segment  tvmuu,  and  the  cylinders  EFCD  and  a  ben, 
for  which  purpose, 

Put  d  zz  ED,  the  height  to  which  the  vessel  is  originally  filled  with 

the  fluid, 

$  ==  DC,  the  diameter  of  the  cylindrical  vessel, 
r  zz  ct,  or  cv,  the  radius  of  the  sphere, 
s  =  the  specific  grav;ty  of  the  fluid  in  the  vessel, 
s'  nr  the  specific  gravity  of  the  floating  body,  and 
x  —  «D,  the  height  to  which  the  fluid  rises  on  the  immersion  of 

the  spheric  segment. 

Then,  since  by  the  principles  of  mensuration,  the  solid  content  or 
capacity  of  a  sphere,  is  equal  to  two  thirds  of  that  of  its  circumscribing 
cylinder,  it  follows,  that  the  capacity  of  the  sphere  mvnw,  is  ex- 
pressed by 

3.1416r2x2rXf=:  4.1888^; 

but^we  have  elsewhere  demonstrated,  that  the  magnitudes  of  bodies 
are  inversely  as  their  specific  gravities  ;  consequently,  the  magnitude  of 
the  part  immersed,  is  determined  by  the  following  analogy,  viz. 

.:.':-.  4.1888^: 


Now,  as  we  have  already  observed,  the  quantity  of  fluid  in  the 

vessel  at  first,  is 

.7854X^X^—7854^, 

and  the  capacity  of  the  cylinder  formed  by  the  fluid  and  the  spherical 

segment,  is 


consequently,  by  addition,  we  shall  have 

.7854  tf  x  =  .7854^-f  H^^i  . 

s 

and  therefore,  if  all  the  terms  of  this  equation  be  divided  by  the 
quantity  .78543%  we  shall  obtain 

16rV 
' 


(177). 

Or  if  the  height  to  which  the  vessel  is  originally  filled,  be  subtracted 
from  both  sides  of  the  above  expression,  the  increase  of  height  in  con- 
sequence of  the  immersion  of  the  spheric  segment,  becomes 

,       16rV 

—  d=x—  3?7' 
where  &•'=:  a£  the  increase  of  height. 


216         OF  FLOATATION  AND  THE  SPECIFIC  GRAVITY  OF  BODIES. 

235.  Either  of  these  equations  will  resolve  the  problem,  but  the 
latter  form  is  the  most  convenient  for  a  verbal  enunciation,  and  the 
practical  rule  which  it  supplies  is  as  follows. 

RULE.  Multiply  sixteen  times  the  specific  gravity  of  the 
sphere,  by  the  cube  or  third  power  of  its  radius  ;  then,  divide 
the  product  by  three  times  the  specific  gravity  of  the  fluid, 
drawn  into  the  square  of  the  cylinder's  diameter,  and  the 
quotient  will  give  the  increase  of  height,  in  consequence  of  the 
immersion  of  the  spheric  segment. 

236.  EXAMPLE.  A  cylindrical  vessel  whose  diameter  is  8  inches,  is 
filled  with  water  to  the  height  of  10  inches;  how  much  higher  will 
the  water  rise,  and  what  will  be  its  whole  weight,  when  a  globe  of 
alder  of  6  inches  diameter  is  dropped  into  the  vessel  ;  the  specific 
gravity  of  alder  being  equal  to  .8,  when  that  of  water  is  expressed  by 
unity  ? 

Here,  by  operating  according  to  the  above  rule,  we  get 

16rV=16x3x3x3x.  8  z=  345.6, 

and  in  like  manner  we  have 

33^  =  3x8x8x1  —  192; 

consequently,  by  division,  we  obtain 

ar'zr—  -—  —  z=  z=  1.8  inches,  and  the  whole  height  is  11.8  inches. 

v    oo  s         ly.2 

237.  If  the  specific  gravity  of  the  globe,  and  that  of  the  fluid  in  which 
it  is  placed,  are  equal  to  one  another,  then  equation  (178)  becomes 


,     33*  '  (179). 

In  this  case  it  is  manifest,  that  the  sphere  is  wholly  immersed  in 
the  fluid  ;  consequently,  the  increase  of  height  wilt  be  equal  to  the 
altitude  of  a  cylinder,  whose  diameter  is  3,  and  whose  capacity  is 
equal  to  that  of  the  immersed  body  ;  hence,  the  method  of  computa- 
tion is  obvious  ;  but  the  practical  rule  deduced  from  the  equation  for 
this  purpose,  may  be  expressed  in  the  following  manner. 

RULE.  Divide  sixteen  times  the  cube  or  third  power  of  the 
radius  of  the  sphere,  by  three  times  the  square  of  the  cylin- 
der s  diameter,  and  the  quotient  will  give  the  increased  height 
of  the  fluid. 

238.  EXAMPLE.  A  cylindrical  vessel  whose  diameter  is  12  inches, 
is  filled  with  fluid  to  the  height  of  6  inches;  to  what  height  will 


OF  FLOATATION  AND  THE  SPECIFIC  GRAVITY  OF  BODIES.  217 

the  fluid  ascend  when  a  sphere  of  4  inches  diameter  is  placed  in  it, 
the  specific  gravities  of  the  fluid  and  the  sphere  being  equal  to  one 
another  ? 

lii  this  example  there  are  given  5  ~  6  inches  and  r  zz  2  inches ; 
therefore,  by  operating  as  directed  in  the  rule,  we  shall  have 

16r3=:16X23— 128,  the  dividend, 
and  in  like  manner  for  the  divisor,  we  get 

3S2  =  3  X 122  —  432,  the  divisor  ; 
consequently,  by  division,  we  obtain 

xf  —  Hi=  0.2962  of  an  inch. 

Hence  it  appears,  that  the  height  of  the  fluid  in  the  vessel,  is 
increased  by  a  quantity  equal  to  0.2962  of  an  inch,  in  consequence 
of  the  immersion,  and  the  whole  height  to  which  it  rises,  is  6.2962 
inches. 

PROBLEM  XXXII. 

239.  A  vessel  in  the  form  of  a  paraboloid,  is  placed  with  its 
vertex  downwards  and  its  base  parallel  to  the  horizon;  now, 
supposing  the  vessel  to  be  filled  to  the  wth  part  of  its  capacity 
with  a  fluid  of  known  specific  gravity,  and  let  a  spherical  body 
of  a  given  size  and  substance  be  placed  in  it : — 

It  is  required  to  ascertain  the  height  to  which  the  fluid 
will  rise,  in  consequence  of  the  immersion  of  the  spherical 
segment. 

Let  ABC  represent  a  vertical  section  passing  along  the  axis  of  the 
vessel,  whose  form  is  that  of  a  paraboloid, 
generated  by  the  revolution  of  the  common 
parabola ;  and  suppose  the  vessel  to  be  filled 
with  an  incompressible  and  non-elastic  fluid 
to  the  height  sc,  DE  being  its  horizontal 
surface  when  in  a  state  of  quiescence,  before 
the  sphere  whose  diameter  is  mn  is  placed  in 
it;  then  will  a b  be  the  surface  or  the  plane 
of  floatation  after  the  immersion  of  the  segment  rnw,  the  fluid  rising 
to  the  height  ts  all  around  the  spherical  body. 

Now,  it  is  obvious  from  the  nature  of  the  problem,  that,  the  solidity 
of  the  spherical  segment  rnw,  together  with  the  quantity  of  fluid  in 
the  vessel,  is  equal  to  the  magnitude  of  the  paraboloid  acb,  whose 


218          OF  FLOATATION  AND  THE  SPECIFIC  GRAVITY  OF  BODIES. 

base  is  ab  and  axis  cf  ;  therefore,  in  order  to  calculate  the  solidity 
of  the  segment,  and  that  of  the  paraboloids  DCE  and  acb, 

Put  d  n:  me,  the  whole  axis  or  height  of  the  paraboloid, 
p  zn  the  parameter  or  latus  rectum  of  the  axis, 
r  zz:  cr,  cm  or  cw,  the  radius  of  the  sphere, 
s  —  the  specific  gravity  of  the  fluid  in  the  vessel, 
/  zz  the  specific  gravity  of  the  floating  body,  and 
x  zz  tc,  the  whole  height  to  which  the  fluid  ascends,  n  being 
the  part  originally  filled. 

By  the  principles  of  solid  mensuration,  the  capacity  or  solidity  of  a 
sphere,  is  equivalent  to  two  thirds  of  that  of  its  circumscribing  cylin- 
der ;  consequently,  the  capacity  of  the  floating  sphere,  is 


now,  we  have  demonstrated  in  another  place,  that  the  magnitudes  of 
bodies,  are  inversely  as  their  respective  gravities  ;  hence  we  have  for 
that  portion  actually  immersed, 

*:^:  4.1888,*  : 


Again,  by  the  principles  of  mensuration,  the  solidity  of  a  paraboloid 
is  equal  to  one  half  the  solidity  of  its  circumscribing  cylinder,  and  by 
the  property  of  the  parabola,  we  have 

niA*~pd; 
therefore,  the  capacity  of  the  paraboloidal  vessel,  is 

3.1416X^^X^—1-5708^^, 
and  consequently,  the  quantity  of  fluid  in  it  is  expressed  by 


But  the  capacity,  or  the  solid  content  of  the  paraboloid  a  c  b,  whose 
axis  is  tc,  becomes 


consequently,  by  addition  and  comparison,  we  have 

4.1888rV      \.570Spd* 
1  .57(%  X*  zz  --  -f  -  £—  , 
s  n 

and  dividing  all  the  terms  by  1.5708,  we  get 

8rV      pd* 
**-  =  —  +  - 


OF  FLOATATION  AND  THE  SPECIFIC  GRAVITY  OF  BODIES.          219 

and  again,  if  all  the  terms  be  divided  by  p  the  parameter  of  the 
parabola,  and  the  square  root  be  extracted  from  both  sides  of  the 
equation,  we  shall  have 


But  because  the  parameter  of  a  parabola  is  a  third  proportional  to 
any  abscissa  and  its  ordinate  ;  it  follows,  that  if  b  denote  the  base  A  B 
of  the  paraboloid,  of  which  the  axis  is  d,  we  shall  have 


let  this  value  of  the  parameter  be  substituted  instead  of  it  in  the  above 
equation,  and  we  shall  obtain 


(180). 

240.  The  following  practical  rule  supplied  by  this  equation,  will 
serve  to  direct  the  reader  to  the  method  of  its  reduction. 

RULE.  Multiply  thirty -two  times  the  axis  of  the  paraboloid, 
by  the  cube  or  third  power  of  the  radius  of  the  sphere  drawn 
into  its  specific  gravity ;  then,  divide  the  product  by  three 
times  the  square  of  the  base  of  the  vessel  multiplied  by  the 
specific  gravity  of  the  Jluid,  and  to  the  quotient,  add  the 
square  of  the  axis  or  depth  of  the  vessel,  divided  by  the 
number,  which  expresses  what  part  of  it  is  occupied  by  the 
Jluid ;  then,  the  square  root  of  the  sum,  will  give  the  height 
to  which  the  Jluid  rises  after  the  immersion  of  the  spheric 
segment. 

241.  EXAMPLE.  The  axis  of  a  vessel  in  the  form  of  a  paraboloid  is 
27  inches,  and  the  diameter  of  its  mouth  is  18  inches ;  now,  supposing 
that  the  vessel  is  one  fifth  full  of  water,  into  which  is  dropped  a  sphere 
of  hazel  whose  diameter  is  8  inches ;  to  what  point  of  the  axis  will 
the  fluid  ascend,  the  specific  gravity  of  hazel  being  0.6,  when  that  of 
water  is  expressed  by  unity  ? 

By  proceeding  according  to  the  rule,  we  get 

32X27X4X4X4X.6  =  33177.6,  the  dividend, 

and  in  like  manner,  for  the  divisor,  we  have 

3X18X18X1  =  972,  the  divisor  ; 
consequently,  by  division,  we  obtain 


220         OF  FLOATATION  AND  THE  SPECIFIC  GRAVITY  OF  BODIES. 

This  is  the  value  of  the  first  term  under  the  radical  sign,  in  the 
expression  for  x  equation  (180),  and  the  value  of  the  second  term,  is 

278 

-  =  145.8 ; 

therefore,  by  addition  and  evolution,  we  obtain 

a;=^/179.93  =  13.414  inches  nearly. 

242.  If  the  specific  gravity  of  the  ball,  and  that  of  the  fluid  in 
which  it  is  placed,  be  equal  to  one  another,  then  equation  (180) 
becomes 

„_    /32^       ^ 
'V    -W~  +  n' 

and  by  reducing  the  fractions  under  the  radical  sign  or  vinculum  to  a 
common  denominator,  we  obtain 

a?  = 


243.  The  practical  method  of  reducing  the  above  equation,  is 
expressed  in  words  at  full  length  in  the  following  rule. 

RULE.  Multiply  the  cube  or  third  power  of  the-  radius  of 
the  sphere,  by  thirty  two  times  the  number  which  indicates 
what  part  of  the  vessel  is  occupied  by  the  fluid,  and  to  the 
product  add  three  times  the  axis  of  the  vessel  drawn  into  the 
square  of  its  diameter ;  then,  divide  the  sum  by  three  times 
the  square  of  the  vessel's  diameter,  drawn  into  the  number 
which  denotes  what  part  of  it  the  fluid  occupies ;  multiply 
the  quotient  by  the  axis  of  the  vessel,  and  extract  the  square 
root  of  the  product ,  for  the  height  to  which  the  fluid  rises. 

244.  EXAMPLE.  Let  the  dimensions  of  the  vessel  and  the  immersed 
body,  remain  as  in  the  preceding  example,  the  vessel  containing  also 
the  same  quantity  of  fluid  ;  to  what  height  on  the  axis  will  the  fluid 
ascend,  supposing  its  specific  gravity  to  be  the  same  as  that  of  the 
immersed  body  ? 

Here,  by  operating  as  directed  by  the  rule,  we  get 
32wr3=  32X5X4X4X4  =  320X32  =  10240,  and 

3b*d=  3X  18  X 18  X27  =  972  X27  =  26244 ; 
consequently,  the  sum  of  the  parenthetical  terms  is 

32»r3  4-  3b*d  =  10240  +  26244  =  36484, 

and  for  the  denominator  of  the  fraction,  we  have 

367w  =  3X18X18X5  =  324X15  =  4860; 


OF  FLOATATION  AND  THE  SPECIFIC  GRAVITY  OF  BODIES.          221 

consequently,  by  division,  we  obtain 
32«r»+3ffd_  36484 

36s n  4860  ~ 

therefore,  by  multiplication  we  shall  have 

7.507X27  =  202.689, 

and  finally,  by  evolution,  it  is 

x=  y  202.689=  14.23  inches. 

COROL.  Hence  it  appears,  that  when  the  specific  gravities  of  the 
fluid  and  the  immersed  body,  are  equal  to  one  another,  the  fluid  rises 
in  the  vessel  to  the  height  of  14.23  inches ;  but  when  the  specific 
gravities  are  to  each  other  as  1  :  0.6,  it  rises  only  to  13.414;  the 
reason  of  the  difference,  however,  is  manifest,  for  in  the  case  of  equal 
specific  gravities,  the  spherical  body  is  wholly  immersed ;  but  when 
the  specific  gravities  are  unequal,  only  a  part  of  the  body  falls  below 
the  plane  of  floatation.  From  the  above  we  deduce  the  following 
inferences. 

245.  INFERENCE  1.  If  a  homogeneous  body  be  immersed  in  a  fluid 
of  the  same  density  with  itself: — 

It  will  remain  at  rest>  or  in  a  state  of  quiescence,  in  all 
places  and  in  all  positions. 

Let  ABCD  represent   a  vessel,  filled  with  an    incompressible  and 
non-elastic  fluid  to  the  height  «D,  and  let  G  be  a 
homogeneous  body,  of  the  same  density  or  specific 
gravity  as  the  fluid. 

Now,  it  is  manifest,  that  when  the  body  G  is  put 
into  the  vessel  and  left  to  itself,  it  will  by  reason  of 
its  own  weight,  sink  below  ab  the  original  surface, 
and  raise  the  fluid  to  the  height  E  D,  where  the  body 
will  be  entirely  under  the  fluid,  and  the  whole  mass 
in  a  state  of  equilibrium  with  the  surface  at  EF. 

Then  it  is  evident,  that  the  body  being  of  the  same  density  as  the 
fluid  in  which  it  is  placed,  it  will  press  the  fluid  under  it,  just  as  much  as 
the  same  quantity  of  the  fluid  would  do  if  put  in  its  stead,  and  conse- 
quently, the  pressure  exerted  by  the  solid,  together  with  that  of  the 
superincumbent  fluid,  presses  downwards  with  the  same  energy,  as  if 
it  were  a  column  of  fluid  of  equal  depth. 

Therefore,  the  pressure  of  the  body  against  the  fluid  at  H,  is  equal 
to  the  pressure  of  the  fluid  against  the  body  there;  consequently, 


222  OF  FLOATATION  AND  THE  SPECIFIC  GRAVITY  OF  BODIES. 

these  two  pressures  are  equal  and  opposite  to  one  another,  and  must 
therefore  be  in  a  state  of  equilibrium,  in  which  case,  the  body  will 
remain  at  rest. 

Hence,  the  truth  of  the  inference  is  manifest  with  respect  to  a 
vertical  pressure ;  but  it  is  equally  true  in  reference  to  a  motion 
horizontally  and  obliquely;  for  the  horizontal  pressures  are  obviously 
equal  to  one  another,  and  they  are  in  opposite  directions;  therefore, 
they  are  in  equilibrio  with  one  another,  and  no  motion  can  take  place. 

And  again,  with  regard  to  the  oblique  pressure,  it  is  evidently 
compounded  of  a  vertical  and  horizontal  one  ;  but  we  have  just 
demonstrated  that  these  are  equal  and  opposite;  consequently,  the 
body  can  have  no  oblique  motion,  it  must  therefore  remain  at  rest  in 
any  place  and  in  any  position. 

If  the  specific  gravity  of  the  immersed  body  be  greater  than  that  of 
the  fluid,  the  pressure  downwards  will  exceed  the  pressure  upwards ; 
consequently,  the  weight  of  the  body  will  overcome  the  resistance  of 
the  fluid  under  it,  and  it  will  therefore  sink  to  the  bottom. 

But  if  the  specific  gravity  of  the  body  be  less  than  that  of  the  fluid, 
the  pressure  upwards  will  exceed  the  pressure  downwards  ;  therefore, 
the  buoyant  principle  will  overcome  the  weight  of  the  solid,  and  it 
will  rise  to  the  surface  of  the  fluid. 

246.  INF.  2.  If  a  solid  body  be  immersed  in  a  fluid,  and  the  whole 
mass  be  in  a  state  of  equilibrium : — 

The  pressure  upwards  against  the  base  of  the  body,  is 
equal  to  the  weight  of  a  quantity  of  fluid  of  equal  magnitude, 
together  with  the  weight  of  the  superincumbent  fluid. 

247.  INF.  3.  If  a  solid  body  be  placed  in  a  fluid  of  greater  or  less 
specific  gravity  than  itself : — 

The  difference  between  the  pressures  downwards  and  up- 
ivards  is  equal  to  the  difference  between  the  weight  of  the 
solid  and  that  of  an  equal  bulk  of  the  fluid. 

248.  INF.  4.  Heavy  bodies  when  placed  in  fluids  have  a  twofold 
gravity,  the  one  true  and  absolute,  the  other  apparent  or  relative. 

Absolute  gravity  is  the  force  with  which  bodies  tend 
downwards. 

By  reason  of  this  force,  all  sorts  of  fluid  bodies  gravitate  in  their 
proper  places,  and  their  several  weights,  when  taken  conjointly, 
compose  the  weight  of  the  whole ;  for  the  whole  is  possessed  of 
weight,  as  may  be  experienced  in  vessels  full  of  liquor. 


OF  FLOATATION  AND  THE  SPECIFIC  GRAVITY  OF  BODIES.          223 

Apparent  or  relative  gravity,  is  the  excess  of  the  gravity 
of  the  body  above  that  of  the  fluid  in  which  it  is  placed. 

By  this  sort  of  gravity,  fluids  do  not  gravitate  in  their  proper  places ; 
that  is,  they  do  not  preponderate ;  but  opposing  one  another's  descent, 
they  retain  their  positions  as  if  they  were  possessed  of  no  weight. 

249.  INF.  5.  If  a  heavy  irregular  heterogeneous  body  descends  in 
a  fluid,  or  if  it  moves  in  any  direction,  and  a  straight  line  be  drawn, 
connecting  the  centres  of  magnitude  and  gravity  of  the  body  : — 

It  will  so  dispose  itself  as  Jo  move  in  that  line,  the  centre 
of  gravity  preceding  the  centre  of  magnitude . 

This  is  a  manifest  and  a  beautiful  fact ;  for  the  centre  of  gravity 
being  surrounded  by  more  matter  and  less  surface  than  the  centre  of 
magnitude,  it  will  meet  with  less  resistance  from  the  fluid ;  conse- 
quently, the  body  will  so  arrange  itself,  as  to  move  in  the  line  of 
direction  with  its  centre  of  gravity  foremost. 

250.  What  has  been  here  adverted  to,  in  regard  to  bodies  of  greater 
density  or  specific  gravity  sinking  in  a  fluid,  must  only  be  understood 
to  apply  to  such  as  are  solid  ;  for  if  a  body  be  hollow,  it  may  swim  in 
a  fluid  of  less  specific  gravity  than  that  which  is  due  to  the  substance 
of  which  the  body  is  composed ;  but  if  the  hollows  or  cavities  are 
filled  with  the  fluid,  the  body  will  then  descend  to  the  bottom. 

Again,  if  bodies  of  greater  specific  gravity  than  the  fluid  in  which 
they  are  placed,  be  reduced  to  extremely  small  particles,  they  may 
also  be  suspended  in  the  fluid ;  but  the  principle  or  force  by  which 
this  is  effected,  does  not  belong  to  hydrodynamics. 

PROPOSITION  IV. 

251.  If  a  solid  homogeneous  body,  be  placed  in  a  fluid  of 
greater  or  less  specific  gravity  than  itself: — 

It  will  ascend  or  descend  with  a  force,  which  is  equivalent 
to  the  difference  between  its  own  weight,  and  that  of  an  equal 
bulk  of  the  fluid. 

The  principle  announced  in  this  proposition  is  almost  self-evident, 
yet  nevertheless,  it  may  be  demonstrated  in  the  following  manner. 

Put  m  —  the  common  magnitude  of  the  body  and  the  fluid, 
w'  —  the  weight  of  the  solid  body, 
s'  —  its  specific  gravity. 


224  OF  FLOATATION   AND  THE  SPECIFIC  GRAVITY  OF  BODIES. 

w  zz  the  weight  of  an  equal  quantity  of  the  fluid, 
s   zz:  its  specific  gravity,  and 

/  zz  the  force  with  which  the  body  ascends  or  descends  in 
the  fluid. 

Then,  because  as  we  have  elsewhere  demonstrated,  the  absolute 
weights  of  bodies,  are  as  their  magnitudes  and  specific  gravities ;  it 
follows,  that 

w  zz:  ms,  and  w'  zz  ms' ; 

but  according  to  the  third  inference  preceding,  the  difference  between 
the  pressures  downwards  and  upwards  : — 

Is  equal  to  the  difference  between  the  weight  of  the  solid 
body,  and  that  of  an  equal  bulk  of  the  fluid. 

But  the  difference  between  the  upward  and  downward  pressures,  is 
equivalent  to  the  force  of  ascent  and  descent ;  consequently,  we  have 

/zz:  w^w'-mms^ms', 
and  this,  by  collecting  the  terms,  becomes 

/z=m(sv-s').  (182). 

If,  therefore,  the  specific  gravity  of  the  solid  be  less  than  that  of 
the  fluid,  the  force  of  ascent  will  be 

/z=m(s-/); 

but  when  the  specific  gravity  of  the  solid  exceeds  that  of  the  fluid,  the 
force  of  descent  becomes 

/zz:m<X-s), 

and  when  the  specific  gravities  are  equal  to  one  another,  the  force  of 
ascent  and  descent  vanishes,  in  which  case,  the  body  will  remain  at 
rest,  in  whatsoever  position  it  may  be  placed ;  this  agrees  with  what 
we  have  already  stated  in  the  first  inference  to  Problem  32. 

From  the  above  proposition  and  its  subordinate  formulae,  the  fol- 
lowing inferences  may  be  deduced. 

252.  INF.  1.  When  a  solid  body  is  immersed,  or  suspended  in  a 
fluid  of  equal  or  of  different  specific  gravity  : — 

It  loses  the  weight  of  an  equal  magnitude  of  the  fluid  in 
which  it  is  placed. 

This  is  obvious,  for  when  the  specific  gravities  are  equal,  the  body 
loses  the  whole  of  its  weight ;  and  therefore,  it  neither  endeavours  to 
ascend  nor  descend ;  but  when  the  specific  gravities  are  unequal,  the 
body  only  endeavours  to  ascend  or  descend,  by  the  difference  between 


OF  FLOATATION    AND  THE  SPECIFIC  GRAVITY  OF  BODIES.          225 

its  own  weight  and  that  of  an  equal  bulk  of  the  fluid,  and  has  there- 
fore lost  the  weight  of  as  much  fluid. 

253.  INF.  2,  When  a  solid  body  is  immersed,  or  suspended  in  a 
fluid  of  the  same,  or  of  different  specific  gravity  : — 

It  loses  the  whole  or  a  part  of  its  weight,  according  as  it 
is  totally  or  partially  immersed,  and  the  fluid  gains  the  weight 
which  the  body  loses. 

This  is  manifest,  for  the  sum  of  the  weights  of  the  body  and  the 
fluid  must  be  the  same,  both  before  and  after  the  immersion. 

254.  INF.  3.  If  bodies  of  equal  magnitude  are  placed  in  the  same 
fluid,  whatever  may  be  their  specific  gravities : — 

They  lose  equal  weight,  and  unequal  bodies  lose  weights 
that  are  proportional  to  their  magnitudes. 

255.  INF.  4.  If  the  same  body  be  immersed,  or  suspended  in  fluids 
of  different  specific  gravities: — 

The  weights  lost  by  the  body  are  as  the  densities  or  specific 
gravities  of  the  fluids. 

256.  INF.  5.  When  two  bodies  of  unequal  magnitude  are  in  equilibrio 
with  one  and  the  same  fluid  : — 

They  will  lose  their  equilibrium,  if  they  be  transferred  to 
another  fluid  of  different  density. 

257.  INF.  6.  When  a  body  ascends  or  descends,  in  a  fluid  of  greater 
or  less  specific  gravity  than  itself: — 

The  force  which  accelerates  its  ascent  or  descent,  is  equal 
to  the  quotient  that  arises,  when  the  difference  between  the 
weight  of  the  body,  and  that  of  an  equal  bulk  of  the  fluid,  is 
divided  by  the  common  magnitude. 

It  consequently  follows,  that  when  the  solid  is  entirely  immersed  in 
the  fluid,  the  force  which  urges  its  ascent  or  descent  is  constant;  in 
which  case,  the  motion  upwards  or  downwards  must  be  uniformly 
accelerated,  if  it  be  not  disturbed  by  the  resistance  of  the  medium  in 
which  it  moves. 

258.  If  the  solid  is  specifically  heavier  than  the  fluid,  it  will  tend 
downwards,  and  press  the  bottom  of  the  vessel,  with  a  force  which  is 
equivalent  to  the  excess  of  its  weight  above  an  equal  bulk  of  the 
fluid  ;  and  this  is  what  we  understand  by  the  relative  giavity  of  the 
body  in  the  fluid. 

But  if  the  body  be  specifically  lighter  than  the  fluid  in  which  it  is 
placed,  it  seems  to  lose  a  greater  weight  than  it  actually  possesses, 

VOL.  I.  Q 


226         OF  FLOATATION   AND  THE  SPECIFIC  GRAVITY  OF  BODIES, 

and  consequently,  it  tends  upwards,  with  a  force  equal  to  the  dif- 
ference between  its  own  weight  and  that  of  an  equal  bulk  of  the 
fluid  ;  and  continuing  to  ascend,  it  will  attain  a  position  in  which  its 
weight  is  equal  to  that  of  a  quantity  of  the  fluid  of  the  same  magni- 
tude as  the  part  immersed  ;  and  this  is  what  we  understand  by  the 
relative  levity  of  the  body  in  the  fluid. 

259.  These  inferences  being  admitted,  we  shall  now  proceed  to 
exemplify  the  general  formula  resulting  from  our  proposition,  viz. 
that  in  which  we  have 


The  practical  rule  for  reducing  this  equation,  may  be  expressed  in 
general  terms  in  the  following  manner. 

RULE.  Multiply  the  common  magnitude  of  the  body  and 
the  fluid,  by  the  difference  of  their  specific  gravities,  and 
the  product  will  be  the  force  of  ascent  or  descent,  according 
as  the  specific  gravity  of  the  body  is  less  or  greater  than  that 
of  the  fluid  in  which  it  is  placed. 

260.  EXAMPLE.  A  mass  of  dry  oak,  whose  magnitude  is  equal  to 
7  cubic  feet,  and  specific  gravity  equal  to  0.8  (that  of  water  being 
unity),  is  plunged  into  a  vessel  of  fluid,  whose  specific  gravity  is 
0.932  ;  with  what  force  will  it  ascend  ? 

Here,  according  to  the  rule,  we  have 

/=:  m  (s  —  s'}  =:  7(.932  —  .8)  —  .924  of  a  cubic  foot  of  the 
body  whose  specific  gravity  is  0.932;  consequently,  for  the  force  in 
Ibs.  avoirdupois,  we  have  1  :  62.5  :  :  924  :  57.75  Ibs. 

PROBLEM  XXXIII. 

261.  In  a  vessel  filled  with  an  incompressible  and  non-elastic 
fluid,  is  placed  a  hollow  cylinder,  which  we  shall  consider  as 
being  perfectly  void  of  gravity  or  weight;    to  the  bottom  of 
which,  a  cylindrical  body  of  a   given  magnitude,  and  whose 
specific  gravity  is  greater  than  that  of  the  fluid,  is  so  closely 
fitted  that  no  fluid  can  enter  :  — 

It  is  required  to  determine,  how  far  below  the  surface  of 
the  fluid  the  body  will  descend,  before  the  tendency  down- 
wards, and  the  pressure  upwards,  are  in  equilibria  with  one 
another. 

Let  A  BCD  in  the  annexed  diagram,  be  a  vertical  section  of  the 


OF  FLOATATION  AND  THE  SPECIFIC  GRAVITY  OF  BODIES.         227 

vessel  containing  the  fluid,  abed  a  corresponding 
section  of  the  hollow  cylinder,  and  EFGH  that  of 
the  attached  or  cylindrical  body. 

Now,  it  is  manifest,  that  in  consequence  of  the 
connection  between  the  hollow  cylinder  and  the 
attached  body,  the  downward  pressure  of  the  fluid 
can  have  no  effect  upon  that  portion  of  the  upper 
surface  of  the  body  whose  diameter  is  dc ;  and  be- 
cause the  hollow  cylinder  abed,  is  supposed  to  be 
without  weight,  it  can  have  no  influence  on  the 
downward  tendency  of  the  body  EFGH  :  an  equilibrium  will  therefore 
obtain,  when  the  downward  pressure  on  the  surface  EeZ,  CF,  together 
with  the  weight  of  the  body,  is  equal  to  the  upward  pressure  on  the 
bottom  HG. 

Put  d  —  dc,  the  diameter  of  the  hollow  cylinder  destitute  of  weight, 
5  zz:  EF,  the  diameter  of  the  attached  body, 
I   zz:  EH,  its  perpendicular  length,  or  vertical  altitude, 
a  zz:  the  area  of  the  end  of  the  hollow  cylinder, 
A  z±  that  of  the  attached  cylindrical  body, 
s   zz:  its  specific  gravity,  greater  than  that  of  the  fluid, 
s'  zzz  the  specific  gravity  of  the  fluid » 
w  zz:  the  weight  of  the  body, 
p  zz:  the  pressure  on  its  upper  surface, 
p  zz:  the  pressure  on  its  base,  and 
x  zz:  e d,  the  distance  below  AB,  the  upper  surface  of  the  fluid. 

Then,  by  the  mensuration  of  surfaces,  the  area  of  the  lower  extre- 
mity of  the  hollow  cylinder  abed,  becomes  «:z±  .7854J2, 

and  that  of  the  base  of  the  attached  body,  is  A  zz:  .78543* ; 
and  the  difference  of  these,  or  the  quantity  of  the  upper  surface  of 
the  body,  which  is  exposed  to  the  downward  pressure  of  the  fluid,  is 

A  —  «z=.7854(32  —  <Z»); 

consequently,  the  downward  pressure  becomes  p  zz:  .7854  (32 —  c?2)  s'  x  \ 
but  the  absolute  weight  of  the  body  is  expressed  by  its  magnitude  or 
solidity,  drawn  into  its  specific  gravity ;  consequently  the  expression 
for  the  weight  of  the  attached  body  becomes  tuzz:  A/szzr.785432Js; 
therefore,  we  have  for  the  whole  tendency  downwards, 

p  4-  w  zz:  .7854  (32  —  d*}  s'x  +  .78543* /s, 
and  from  this,  by  collecting  the  terms,  we  obtain 
p  4-  w  =  .7854  {(32—  «P)  s'  x  +  tf  Is}, 
Q  2 


228         OF  FLOATATION  AND  THE  SPECIFIC  GRAVITY  OF  BODIES. 

Now,  the  pressure  upwards  on  the  bottom  of  the  attached  cylindri- 
cal body,  according  to  the  principle  of  the  first  proposition,  is 

//=.78543V(J  +  a?)  ; 

but  in  the  case  of  an  equilibrium,  or  when  the  body  has  attained  a 
state  of  quiescence,  the  pressure  upwards  is  exactly  equal  to  the 
downward  tendency  ;  consequently,  by  comparison,  we  have 

.78543V  (l  +  x)  =  .7854  {  (3*  —  d*)  s'  x  +  Vis}  ; 
therefore,  by  suppressing  the  common  factor  .7854,  and  transposing, 

we  get 

3V  (I  +  x}  —  (a2  —  d2)  s'  x  =  V  Is,  and  this, 

by  expanding  and  collecting  the  terms,  becomes  d?s'x=i&l(s  —  s'), 
and  finally,  by  division,  we  obtain  x  zz:  —  ~-f  —  .       (183) 

262.  The  following  practical  rule,  drawn  out  in  words  at  length, 
will  serve  for  the  reduction  of  the  equation. 

RULE.  Multiply  the  difference  between  the  greater  and 
less  specific  gravities,  by  the  square  of  the  diameter  of  the 
attached  body  drawn  into  its  length  ;  then  divide  the  product 
by  the  square  of  the  diameter  of  the  hollow  cylinder,  drawn 
into  the  specific  gravity  of  the  fluid,  and  the  quotient  will 
be  the  distance  below  the  surface  of  the  fluid  at  which  the 
body  rests. 

263.  EXAMPLE.    A  cylinder  of  lignum  vitse,  whose  diameter  is 
8  inches,  length  36  inches,  and  specific  gravity  1.327,  is  attached  to 
the  lower  end  of  a  hollow  tube,  whose  diameter  is  3  inches,  in  such  a 
manner  that  no  fluid  can  enter  ;  now,  supposing  the  body,  and  the 
hollow  cylinder  to  which  it  is  attached,  to  be  placed  in  a  vessel  full  of 
water,  it  is  required  to  determine,  at  what  distance  below  the  surface 
of  the  fluid  the  body  will  become  quiescent  ? 

Here,  by  operating  according  to  the  rule,  we  obtain 

82X36(1.327  —  1.000) 
,=  -  (  -  -'  =  83.707  inches, 


Hence  it  appears,  that  a  cylinder  of  lignum  vitae  of  the  proposed 
dimensions,  will  sink  to  the  depth  of  83.707  inches,  or  very  nearly 
7  feet  below  the  surface  of  the  water,  before  the  upward  pressure 
becomes  an  equipoise  for  its  downward  tendency  ;  and  this  being 
added  to  the  three  feet  which  it  is  in  length,  gives  10  feet  for  the 
depth  of  the  body  of  water,  necessary  for  admitting  an  equilibrium, 
under  the  specified  conditions  of  magnitude  and  attachment. 


CHAPTER  X. 

OF    THE    SPECIFIC    GRAVITIES    OF    FLUIDS,    AND    THE    METHOD    OF 
WEIGHING  SOLID  BODIES  BY    MEANS    OF  NON-ELASTIC  FLUIDS. 

THE  specific  gravity  of  a  body  is  its  weight  compared  with  that  of  another  body 
of  the  same  magnitude.  The  magnitude  may  be  expressed  by  a  number  denoting  its 
relation  to  some  standard  generally  used,  and  as  a  criterion  of  comparison,  similar 
to  itself,  as  a  cubical  inch,  a  foot,  &c. ;  and  if  the  criteria  be  different,  as  in  solids 
and  fluids,  the  magnitudes  of  bodies  are  to  each  other  as  the  criteria  multiplied 
into  the  numbers  expressing  these  magnitudes.  The  clear  and  scientific  expositions 
which  in  the  last  chapter  were  given  of  absolute  and  relative  weight,  must  have 
well  prepared  the  reader  for  entering  upon  the  doctrine  of  specific  gravities,  which 
distinguishes  different  species  of  matter  from  each  other,  in  one  of  their  most 
obvious  properties,  namely,  the  weight  of  matter  contained  in  a  given  space.  The 
weight  of  any  portion  of  matter  is  easily  ascertained ;  but  it  is  not  always  easy  to 
measure  the  space  occupied  by  a  body,  or  its  magnitude;  and  in  some  instances 
this  cannot  well  be  effected  without  artificial  means.  We  employ  for  this  purpose 
distilled  water,  the  specific  gravity  of  which,  or  weight  of  a  given  bulk,  is  nearly 
at  all  times  the  same.  Adopting,  therefore,  this  pure  homogeneous  substance  as 
our  criterion  or  unit  of  measure,  by  comparing  it  with  other  substances  the  ratio  of 
their  specific  gravities  may  be  easily  discovered ;  and  denoting  the  specific  gravity 
of  water  by  any  number  taken  at  pleasure,  the  numbers  expressing  the  specific 
gravities  of  other  bodies  are  hence  given,  or,  at  least,  assignable. 

PROPOSITION  V. 

264.  When  a  solid  body  is  immersed  in  a  fluid  of  different 
specific  gravity  from  itself: — 

The  weight  which  the  body  loses,  will  be  to  its  whole 
weight,  as  the  specific  gravity  of  the  fluid  is  to  the  specific 
gravity  of  the  solid. 

This  is  a  very  important  proposition  in  hydrostatics,  but  its  demon- 
stration does  not  require  the  assistance  of  a  diagram  ;  we  must  there- 
fore endeavour  to  establish  its  validity  by  the  application  of  symbolical 
arithmetic  ;  for  which  purpose — 


230         OF  FLOATATION   AND  THE  SPECIFIC  GRAVITY  OF  BODIES. 

Put  m  nr  the  magnitude  of  the  immersed  solid, 
w  ~  its  weight, 

w"~  the  weight  lost  by  the  pressure  of  the  fluid, 
w'  —  the  weight  of  a  quantity  of  the  fluid,  of  the  same  bulk  as 

the  solid  tody,  - 

s    —  the  specif  of*  the  solid,  and       „ 
s'   in  the  specific  gravity  of  the  fluid. 

Then,  because,  as  we  have  already  stated  in  a  former  part  of  this 
work,  the  wejghts  are  directly  as  their  magnitudes  drawn  into  their 
respective*  gravities ;  it  follows,  that 

A  j     /  / 

w=.ms,  and  w  ~ms  ; 

consequently,  by  analogy,  we  obtain 
w'  :  w  : :  ms'  :  ms. 

Now,  when  a  body  is  immersed  in  a  fluid  of  less  specific  gravity 
than  itself,  it  obviously  must  descend,  in  consequence  of  its  superior 
gravity,  and  the  force  of  descent  is  equal  to  the  difference  between 
the  weight  of  the  solid  body  and  the  weight  which  it  loses  by  the 
action  of  the  fluid ;  but  it  has  been  shown  in  the  preceding  propo- 
sition, that  the  force  of  descent  is  equal  to  the  difference  between  the 
weight  of  the  solid,  and  that  of  an  equal  bulk  of  the  fluid ;  conse- 
quently, if  f  denote  the  force  of  descent,  then  in  the  one  case,  we  have 

f—w  —  w'9 
and  in  the  other  case,  it  is 

f—w  —  w"; 
therefore,  by  comparison,  we  obtain 

w  —  w'  ~  w  — 10"  ; 
hence,  by  expunging  w,  it  becomes 

w'  —  w". 

If,  therefore,  we  substitute  w"  instead  of  w' ,  in  the  first  term  of  the 
above  analogy,  we  shall  get 

w"  :  w  : :  ms*  :  ms  : :  s'  :  s ; 
hence  the  truth  of  the  proposition  is  manifest. 

The  part  of  the  weight  which  the  body  loses  by  descending  in  the 
fluid,  is  not  annihilated,  it  is  only  sustained  by  the  upward  pressure 
of  the  fluid  opposing  the  descent  of  the  body ;  and  this  is  the  reason 
why  the  weight  of  a  vessel  full  of  water  is  not  perceptible  while  it  is 
beneath  the  surface. 


OF  SPECIFIC  GRAVITY  AND  THE  WEIGHING  OF  SOLID  BODIES.       231 

By  equating  the  products  of  the  extreme  and  mean  terms  in  the 
preceding  analogy,  we  obtain 

sw"  —  s'w, 

and  dividing  by  s,  we  have 
s'w 


but  according  to  our  notation,  w"  denotes  the  weight  which  the  body 
loses ;  consequently,  the  weight  which  it  retains  in  the  fluid,  becomes 

s'w      w  (s  —  s') 

W W'-^HW m - .  /1Q/|\ 

s       .       s  (184)- 

265.  If,  therefore,  the  weight  of  the  body,  together  with  its  specific 
gravity,  be  known,  before  it  is  immersed  in  a  fluid  of  a  given  specific 
gravity ;  its  weight  after  immersion  can  easily  be  ascertained  by  the 
following  practical  rule. 

RULE.  From  the  specific  gravity  of  the  body,  subtract  that 
of  the  fluid  in  which  it  is  immersed ;  multiply  the  remainder 
by  the  weight  of  the  body,  and  divide  the  product  by  its 
specific  gravity  for  the  weight  which  it  retains  after  im- 
mersion. 

266.  EXAMPLE.   A  piece   of  cast   iron  which  weighs   14  Ibs.  is 
plunged   into    a   cistern  of  water;    what  force  will  be  required  to 
sustain  the  iron  at  rest  in  any  point,  its  specific  gravity  being  to  that 
of  water  as  7  to  1  ? 

Here,  by  operating  according  to  the  rule,  we  have 

14(7—1) 

— - ~  12  Ibs.  avoirdupois. 

From  which  it  appears,  that  14  Ibs.  of  cast  iron  being  suspended 
in  water,  loses  2  Ibs.  of  its  weight ;  or  which  is  the  same  thing,  the 
upward  pressure  of  the  water  exceeds  its  downward  pressure,  by  a 
force  which  is  equivalent- to  2  Ibs. 

COROL.  We  may  also  infer  from  the  above,  that  the  weights  which 
the  same  body  loses  by  being  immersed  in  different  fluids : — 
Are  as  the  specific  gravities  of  the  fluids. 

PROBLEM  XXXIV. 

267.  If  a  body  be  weighed  in  air  and  in  water  respectively, 
and  the  weights  be  exactly  ascertained  : — 

It  is  required,  from  the  weights  thus  exhibited,  to  deter- 
mine the  real  weight. 


232       OF  SPECIFIC  GRAVITY  AND  THE  WEIGHING  OF  SOLID  BODIES. 

Here  again,  we  are  under  the  necessity  of  dispensing  with  the 
assistance  of  a  diagram,  the  investigation  being  wholly  analytical ;  in 
order  therefore  to  proceed, 

Put  w  =  the  weight  of  the  body  when  weighed  in  water, 
w'~  the  weight  when  weighed  in  air, 

s  —  the  specific  gravity  of  water,  generally  expressed  by  unity, 
s*  nz  the  specific  gravity  of  air,  and 
x  rzz  the  real  weight  of  the  body  required. 

Then  we  have  x  —  w' ',  and  x  —  w>  for  the  weights  which  the  body 
loses  in  air  and  in  water ;  but  we  have  deduced  it  as  an  inference 
from  Proposition  V.,  that  when  the  same  body  is  weighed  in  different 
fluids,  it  loses  weights  in  proportion  to  the  specific  gravities  of  the 
fluids  in  which  it  is  weighed ;  consequently,  we  have 

x  —  w'  :  x  —  w  : :  s'  is; 

therefore,  by  making  the  product  of  the  mean  terms  equal  to  the 
product  of  the  extremes,  we  have 

s  (x  —  w')  —  s'  (x  —  w), 
and  from  this,  by  separating  the  terms,  and  transposing,  we  get 

(s  —  s')  x  ~  s  w'  —  s'w; 

consequently,  by  division,  we  obtain 

__sw' — s'  w 

s  —  s'  (185). 

268.  The  equation  in  its  present  form,  supplies  us  with  the  follow- 
ing practical  rule  for  its  reduction. 

RULE.  Divide  the  difference  between  the  products  of  the 
alternate  weights  and  specific  gravities,  by  the  difference  of 
the  specific  gravities,  and  the  quotient  will  be  the  real  weight 
of  the  body. 

269.  EXAMPLE.  A  certain  body  when  weighed  in  water  and  in  air, 
is  found  to  equiponderate  12  and  13.9975  Ibs.  respectively;  what  is 
its  real  weight,  the  specific  gravities  of  air  and  water  being  as  1  to 
.0012? 

Here,  by  operating  as  directed  in  the  rule,  we  have 
1X13.9975  — .0012X12 


x  — 


— .0012 


—  14  Ibs. 


From  which  it  appears,  that  a  body  of  14  Ibs.  avoirdupois,  will 
completely  fulfil  the  conditions  of  the  question. 


OF  SPECIFIC  GRAVITY  AND  THE  WEIGHING  OF  SOLID  BODIES,       233 

PROBLEM  XXXV. 

270.  If  the  weights  which  a  body  indicates,  when  weighed 
in  air  and  in  water,  are  exactly  ascertained  :  — 

It  is  required  from  thence  to  determine  the  specific  gravity 
of  the  body,  the  specific  gravities  of  air  and  water  being 
known. 

Here  also,  as  in  the  case  of  the  preceding  Problem  XXXIV.,  and 
Proposition  V.,  the  aid  of  a  diagram  is  not  required  ;  for  it  would  be 
totally  inconsistent  with  scientific  precision,  to  denote  the  specific 
gravities  of  bodies  by  geometrical  magnitudes, 

Put  W  i=  the  real  weight  of  the  solid  body, 
w   =z  the  weight  when  weighed  in  water, 
w'  zz  the  weight  when  weighed  in  atmospheric  air, 
s     =.  the  specific  gravity  of  water,  expressed  by  unity, 
s'   •=.  the  specific  gravity  of  air,  and 
S   =  the  required  specific  gravity  of  the  solid  body. 

Then,  according  to  the  principle  announced  and  demonstrated  in 
the  5th  proposition,  we  have 

W  —  w  :  W  ::s  :  S  ; 

where  it  is  manifest,  that  W—  w;  expresses  the  weight  which  the  body 
loses  by  being  weighed  in  water  ;  therefore,  we  have 
W  —  (W—  -w)  :  W  :  :  S—s  :  S; 
or  by  equating  the  products  of  the  extremes  and  means,  we  get 

Sw  =  (S—s)Vf; 

and  by  proceeding  in  a  similar  manner  when  the  body  is  weighed  in 
air,  we  obtain 


Now,  from  the  first  of  these  equations,  we  have 


and  from  the  second,  it  is 

W  =  (S=7); 

hence  by  comparison,  we  obtain 
Sw  Sw' 


234      OF  SPECIFIC  GRAVITY  AND  THE  WEIGHING  OF  SOLID  BODIES, 

and  by  taking  away  the  denominators,  we  get 

Sw  —  s'w~Sw'  —  s  uf, 
from  which,  by  transposing  and  collecting  the  terms,  we  obtain 

(10*  —  w)S-=.sw'  —  s'w, 
and  finally,  by  division,  we  have 

sw' — s'  w 

''    w'  —  w  '  (186). 

271.  The  practical  rule  or  method  of  reducing  this  equation,  may 
be  expressed  in  words  in  the  following  manner. 

•  -  RULE.  Divide  the  difference  between  the  products  of  the 
alternate  weights  and  specific  gravities,  ly  the  difference  of 
the  weights  when  weighed  in  air  and  in  water,  and  the 
quotient  will  express  the  specific  gravity  of  the  body. 

272.  EXAMPLE.  A  certain  body  when  weighed  in  water  indicates 
exactly  12  Ibs.  avoirdupois ;  but  when  the  same  body  is  weighed  in 
air,  it  indicates  13.9975  Ibs.;    required  the  specific  gravity  of  the 
body,  the  specific  gravities  of  water  and  air  being  as  in  the  preceding 
problem,  or  as  1  to  .0012  ? 

The  process  performed  according  to  the  directions  given  in  the 
rule,  or  after  the  manner  indicated  in  equation  (186),  will  stand  as 
follows. 

1X13.9975—  .0012X12 

13.9975  —  12 

Therefore,  a  body  whose  specific  gravity  is  seven  times  the  specific 
gravity  of  water,  will  fulfil  the  conditions  of  the  question.  - 

PROBLEM  XXXVI. 

273.  If  the  weights  which  a  solid  body  indicates,  when  weighed 
in  air  and  in  water,  together  with  its  specific  gravity  and  real 
weight,  are  exactly  ascertained : — 

It  is  required  from  thence,  to  determine  the  magnitude  of 
the  body,  on  the  supposition  that  it  is  globular. 

If  the  specific  gravity  of  the  body  and  its  real  weight  were  unknown, 
the  solution  of  the  present  problem  would  include  that  of  the  two  pre- 
ceding ones ;  but  in  order  to  abbreviate  the  investigation,  we  have 
supposed  the  specific  gravity  and  the  real  weight  of  the  body  to  be 
given ;  the  process  of  the  solution  is  therefore  as  follows. 


OF  SPECIFIC  GRAVITY  AND  THE  WEIGHING  OF  SOLID  BODIES.       235 

Put  W  zz  the  real  weight  of  the  globular  body, 
S  zz  its  specific  gravity, 
w  zz  the  weight  which  the  body  indicates  when  weighed  in 

water, 

s    zz  the  specific  gravity  of  water, 

vf  zz  the  weight  which  the  body  indicates  when  weighed  in  air, 
s'   zz  the  specific  gravity  of  air,  and 
d   zz  the  required  diameter  of  the  solid  body. 

Then,  according  to  the  principles  of  mensuration,  the  solidity  of  a 
globe  is  expressed  by  the  cube  or  third  power  of  its  diameter,  multi- 
plied by  the  constant  decimal  .5236  ;  therefore,  we  have 
dx  dxd*  .5236  zz  .5236d8 ; 

but  it  has  been  stated  in  a  former  part  of  this  work,  that  the  absolute 
weight  of  any  body,  is  expressed  by  its  magnitude  drawn  into  the 
specific  gravity ;  hence  we  have 

Wz=.5236Sd3; 
consequently,  by  division,  we  obtain 


and  from  this,  by  extracting  the  cube  root,  we  get 

.52365*  (187). 

274.  The  equation  in  its  present  form,  expresses  the  diameter  of 
the  body  in  terms  of  its  absolute  weight  and  specific  gravity ;  this  is 
certainly  the  simplest  and  only  mode  of  determining  the  magnitude 
of  any  body  or  quantity  of  matter,  when  the  weight  and  specific 
gravity  are  known  a  priori ;  but  when  this  is  not  the  case,  we  must 
have  recourse  to  other  methods ;  and  a  very  elegant  and  simple  one, 
consists  in  weighing  the  body  in  water  and  in  air,  as  implied  in  the 
problem,  and  then  proceeding  as  follows. 

By  equation  (185),  Problem  XXXIV.,  it  appears,  that  the  real  or 
absolute  weight  of  the  solid,  expressed  in  terms  of  its  relative  weights, 
and  the  specific  gravities  «of  the  fluids  in  which  it  is  weighed,  viz.  water 
and  air,  is 

!j£/-V^ 
s—  s1    ' 

and  by  equation  (186),  Problem  XXXV.,  the  specific  gravity  of  the 
solid  expressed  in  terms  of  the  same  quantities,  is 


236       OF  SPECIFIC  GRAVITY  AND  THE  WEIGHING  OF  SOLID  BODIES. 

_  sw*  —  s'w 

w'  —  w 

But  the  real  or  absolute  weight  of  any  body,  is  expressed  by  its 
magnitude  drawn  into  the  specific  gravity  ;  consequently,  we  have 

s  w>  —  s/  w 


_ 

W   -  W 

let  this  value  of  the  real  weight  be  compared  with  that  above,  and  we 
shall  have 

.5236  d3  (s  w'  —  s'w)  _  sw'  —  s'  w 

w  —  w  s  —  s' 

If  the  expression  (sw'  —  s'w)  be  suppressed  on  both  sides  of  the 
above  equation,  we  shall  obtain 

.5236rf»_      1 
w'  —  w      s  —  s'  ' 
and  again,  by  suppressing  the  denominators,  we  get 

.5236(5  —  t')d*  =  u/  —  w; 
therefore,  dividing  by  .  5236  (s  —  5'),  we  have 


".5236(5  —  57 
and  finally,  by  extracting  the  cube  root,  we  obtain 


.5236(5  —  5')  (188). 

275.  Now,  the  methods  of  reducing  the  equations  (187)  and  (188), 
or  the  practical  rules  derived  from  them,  may  be  expressed  as  follows. 

1.  When  the  absolute  weight  and  specific  gravity  are  given. 

RULE.  Divide  the  absolute  weight  of  the  body,  by  .5236 
times  the  specific  gravity,  and  the  cube  root  of  the  quotient 
will  be  the  diameter  of  the  solid  sought. 

2.  When  the  weights  indicated  by  the  body  in  water  and  in  air 

are  given. 

RULE.  Divide  the  difference  between  the  weights,  as  obtained 
from  weighing  the  body  in  air  and  in  water,  by  .5236  times 
the  difference  between  the  specific  gravities  of  water  and  air ; 
then,  the  cube  root  of  the  quotient  will  be  the  diameter  of  the 
solid  sought. 

276.  EXAMPLE  1.  The  absolute  weight  of  a  globular  body  is  14  Ibs.* 
and  its  specific  gravity  7  ;  what  is  its  diameter  ? 


OF  SPECIFIC  GRAVITY  AND  THE  WEIGHING   OF  SOLID  BODIES.       237 

This  example  corresponds  to  equation  (187),  and  must  therefore 
be  resolved  by  the  first  case  of  the  foregoing  rule,  observing  to  bring 
the  numerator  into  the  same  denomination  with  the  denominator,  that 
is,  reducing  Ibs.  avoirdupois  into  ounces;  or  thus,  14Xl6:zrW,  the 
absolute  weight,  from  which  we  get 


Hence  it  appears,  that  a  globular  body,  whose  specific  gravity  is 
seven  times  greater  than  that  of  water,  will  weigh  14  Ibs.  when  its 
diameter  is  3.9313  inches,  which  corresponds  very  nearly  with  a  globe 
of  cast  iron. 

277.  EXAMPLE  2.    A  globular   body  whose   specific  gravity  and 
absolute  weight  are  unknown,   indicates   12  Ibs.   avoirdupois  when 
weighed  in  water,  and  13.9975  Ibs.  when  weighed  in  air;  what  is  its 
magnitude,  the  specific  gravity  of  water  and  air  being  to  one  another 
as  the  numbers  1  and  .0012  ? 

This  second  example  corresponds  to  the  conditions  represented  in 
equation  (188),  and  must  therefore  be  resolved  by  the  second  case  of 
the  foregoing  rule,  the  numerator  being  brought  into  the  same  deno- 
mination with  the  denominator,  or  the  Ibs.  avoirdupois  being  turned 
into  ounces,  as  (13.9975  —  12)  16,  from  which  we  obtain 

l-3-^-7^=r=^^4^^§9i»eg  nearly  4  inches,  the  same    $**, 
.5236(1—  .0012) 
as  above. 

From  the  principles  established  in  the  foregoing  Proposition  (V),  and 
the  problems  derived  from  it,  we  deduce  the  following  inferences. 

278.  INF.  1.  When  bodies  of  equal  weights,  but  of  different  magni- 
tudes, are  immersed  in  the  same  fluid  :  — 

The  weights  which  they  lose,  are  reciprocally  proportional 
to  their  specific  gravities,   or  directly  proportional  to  their 


279.  INF.  2.  When  a  solid  body  is  weighed  in  air,  or  in  any  other 
fluid  whatever : 

The  difference  between  its  absolute  weight,  and  the  weight 
exhibited  in  the  fluid,  is  the  same  as  the  weight  of  an  equal 
bulk  of  the  fluid. 

280.  INF.  3.    If  two  solid  bodies  of  different  magnitudes,   when 
weighed  in  the  same  fluid  indicate  equal  weights  : — 

The  greater  body  will  preponderate  when  they  are  brought 
into  a  rarer  medium. 


238       OF  SPECIFIC  GRAVITY  AND  THE  WEIGHING  OF  SOLID  BODIES. 

281.  INF.  4.  If  two  solid  bodies  of  different  magnitudes,  indicate 
equal  weights  when  weighed  in  the  same  fluid : — 

The  lesser  body  will  preponderate  when  they  are  placed  in 
a  denser  medium. 

282.  INF.  5.  If  two  or  more  solid  bodies,  when  placed  in  the  same 
fluid,  sustain  equal  diminutions  of  weight : — 

The  magnitudes  of  the  several  bodies  are  equal  among 
themselves. 

This  is  manifest,  for  the  losses  of  weights  are  as  the  weights  of  the 
quantities  of  fluid  displaced  ;  and  these  are  as  the  magnitudes  of  the 
bodies  which  displace  them. 

PROBLEM  XXXVII. 

283.  If  two  bodies  of  equal  weights,  but  different  specific 
gravities,  be  exactly  equipoised  in  air,  and  then  immersed  in  a 
fluid  of  greater  specific  gravity,  the  smaller  body  will  prevail : — •• 

It  is  therefore  required  to  determine,  what  weight  must 
be  added  on  the  part  of  the  greater  body,  to  restore  the 
equilibrium.  •  • 

Put  s    zz:  the  specific  gravity  of  the  fluid,  in  which  the  bodies  are 

immersed,  after  being  equipoised  in  air, 
/    zz:  the  specific  gravity  of  the  greater  body, 
s"  zz:  the  specific  gravity  of  the  smaller  body, 
w   —  the  common  weight  of  each, 
w'  zz:  the  weight  lost  by  the  greater  body,  by  reason  of  the 

immersion, 

w"  zz:  the  weight  lost  by  the  lesser  body,  and 
x    zz:  the  weight  which  must  be  added  to  restore  the  equi- 
librium. 

Then,  because  by  the  preceding  proposition,  when  a  body  is  im- 
mersed in  a  fluid,  the  weight  which  it  loses,  is  to  its  whole  weight,  as 
the  specific  gravity  of  the  fluid  is  to  that  of  the  body ;  it  follows, 

that 

sr  :  s  :  :  w  :  w' ', 

and  this,  by  reducing  the  proportion,  gives   w'  zz:— —•> 

S 

Again,  for  the  weight  lost  by  the  lesser  body,  we  have 

s"  :  s  :  :  w  :  w" : 


OF  SPECIFIC  GRAVITY  AND  THE  WEIGHING  OF  SOLID  BODIES.       239 

which  by  reduction  gives 
ws 

»=-F- 

Now,  it  is  manifest,  that  the  weight  required  to  restore  the  equili- 
brium, must  be  equal  to  the  difference  between  the  results  of  the 
above  analogies  ;  therefore,  we  obtain 

H  __       w s      ws 

~~7      7r ' 

which,  by  a  little  farther  reduction,  becomes 

X  = 77'        '  (189). 

284.  The  practical  rule  which  this  equation  affords,  may  be  ex- 
pressed in  words  at  length  in  the  following  manner. 

RULE.  Multiply  the  difference  between  the  specific  gravities 
of  the  bodies,  by  their  common  weight  in  air,  drawn  into  the 
specific  gravity  of  the  fluid  in  which  they  are  immersed,  and 
divide  the  result  by  the  product  of  the  specific  gravities  of  the 
bodies,  for  the  weight  to  be  added  in  order  to  restore  the 


It  may  be  proper  here  to  observe,  that  the  weight  determined  by  this 
rule  must  not  be  immersed  in  the  fluid,  it  must  only  be  attached  to 
that  side  of  the  balance  on  which  the  greatest  weight  is  lost. 

285.  EXAMPLE.  Suppose  that  84  Ibs.  of  brass,  whose  specific  gravity 
is  8.1  times  greater  than  that  of  water,  is  equipoised  in  air  by  a  piece 
of  copper,  whose  specific  gravity  is  9  times  greater  than  that  of  water ; 
how  much  weight  must  be  applied  to  the  ascending  arm  of  the  balance 
to  restore  the  equilibrium,  the  same  being  destroyed  by  immersing  the 
bodies  in  water,  of  which  the  specific  gravity  is  expressed  by  unity  ? 
Here,  by  attending  to  the  directions  in  the  rule,  we  get 

84X1X(9  — 8.1)  .. 

*=  8.1X9  =l-0371bB. 

Hence  it  appears,  that  if  a  mass  of  brass  and  of  copper,  each  equal 
to  84  Ibs.  when  weighed  in  air,  be  immersed  in  a  vessel  of  water,  the 
copper  will  preponderate,  in  consequence  of  its  greater  specific  gravity; 
and  in  order  that  the  equilibrium  may  be  again  restored,  a  weight  of 
1 .037  Ibs.  must  be  attached  to  the  ascending  arm  of  the  balance,  or 
that  from  which  the  brass  is  suspended . 


240       OF  SPECIFIC  GRAVITY  AND  THE  WEIGHING  OF  SOLID  BODIES* 

PROBLEM  XXXVIII. 

286.  If  two  bodies  of  different,  but  known  specific  gravities, 
equiponderate  in  a  fluid  of  given  density : — 

It  is  required  to  determine  the  ratio  of  the  quantities  of 
matter  which  they  contain. 

Put  5  —  the  specific  gravity  of  the  fluid,  in  which  the  bodies  are 
found  to  equiponderate, 

m  zz  the  magnitude  of  the  greater  body, 

s'  zz  its  specific  gravity, 

m'zz  the  magnitude  of  the  lesser  body, 

s"  zz  its  specific  gravity, 

w  zz  the  weight  of  the  greater  body  in  the  fluid,  and 

w/zz  the  weight  of  the  lesser  body  under  the  same  circum- 
stances. 

Then,  by  Proposition  V.,  when  a  solid  body  is  immersed  in  a  fluid 
of  different  specific  gravity,  the  weight  which  it  loses,  is  to  its  whole 
weight,  as  the  specific  gravity  of  the  fluid,  is  to  the  specific  gravity  of 
the  solid  ;  it  therefore  follows,  that 

/  :  s  :  :  m  s'  :  m  s  zz  the  weight  lost  by  the  greater  body ; 
but  the  weight  of  the  body  in  the  fluid,  is  manifestly  equal  to  the 
difference  between  its  absolute  weight,  and  that  which  it  loses  in 
consequence  of  the  immersion  ;  hence  we  have 

w  zz  m  s' —  m  s  zz  m  (s' —  s)  ; 
and  by  a  similar  mode  of  procedure,  we  obtain 
s"  :  s  : :  m1  s"  :  m' s  zz  the  weight  lost  by  the  lesser  body; 
consequently,  the  weight  which  it  possesses  in  the  fluid,  is 

w'  zz  m1  s" —  m' s  zz  m'  (s"  —  s). 

Now,  according  to  the  conditions  of  the  problem,  these  are  in 
equilibrio  with  one  another;  therefore  by  comparison,  we  have 

m  (s' —  s)  zz  m'(s" —  s), 
and  by  converting  this  equation  into  an  analogy,  it  is 

m  :  m'  :  :  (s"—  s)  :  (s'—  s); 

and  finally,  if  we  multiply  the  first  and  third  terms  by  s',  and  the 
second  and  fourth  by  s",  we  shall  have 

ms'  :  m's11  :  :  s'(s"—  s}  :  s"(s'—s). 

287.  EXAMPLE.  Twenty  ounces  of  brass,  whose  specific  gravity  is 
eight  times  greater  than  that  of  water,  and  a  piece  of  copper  whose 


OF  SPECIFIC  GRAVITY  AND  THE  WEIGHING  OF  SOLID  BODIES.       241 

specific  gravity  is  nine  times  greater,  are  in  equilibrio  with  one  another 
in  a  fluid  whose  specific  gravity  is  unity  ;  required  the  weight  of  the 
copper  ? 

Here  we  have  given  m  s'  =  20  ounces  ;  /  =  8  ;  s"  —  9,  and  s  —  \ ; 
consequently,  by  substitution,  the  above  analogy  becomes 

20  :  m's"  :  :  8(9—1)  :  9(8—1); 
and  by  equating  the  products  of  the  extremes  and  means,  we  get 

64mVzzl260, 
and  dividing  by  64,  we  have 

m1  s"  in          =19-f^-  ounces. 

It  therefore  appears,  that  20  ounces  of  brass  and  19|^  ounces  of 
copper,  are  in  equilibrio  with  each  other,  when  immersed  in  a  fluid 
whose  specific  gravity  is  unity  ;  but  if  put  into  a  fluid  of  greater 
•density,  the  copper  will  prevail. 

PROBLEM  XXXIX, 

288.  Suppose  a  cylinder  and  cone,  of  the  same  altitude,  base, 
and  specific  gravity,  to  balance  each  other  at  the  extremities  of 
a  straight  lever,  when  immersed  in  a  fluid  of  given  density ;  the 
cone  being  suspended  at  the  vertex,  and  the  cylinder  at  the 
extremity  of  the  axis.  Now,  suppose  a  cone  equal  to  the  one 
proposed,  to  be  abstracted  from  the  cylinder,  and  its  place  sup- 
plied by  another  of  the  same  magnitude  and  half  the  specific 
gravity ;  it  is  manifest  that  in  this  state,  the  cone  will  prepon- 
derate : — 

It  is  therefore  required  to  determine,  how  much  must  be 

taken  from  the  cone,  in  order  that  the  equilibrium  may  be 

again  restored. 

Let  AB  be  a  straight  inflexible  lever,  supported  upon  and  easily 
moveable  about  the  fulcrum  F,  and 
let  the  cone  CDE  and  the  cylinder 
GHIK,  (equal  in  altitude,  base,  and 
specific  gravity,)  be  suspended  from 
the  extremities  at  A  and  B. 

Then  it  is  manifest,  that  in  conse- 
quence of  the  equality  of  the  bases  ,y. 
and  altitudes,  the  magnitude  of  the  cylinder  is  equal  to  three  times 
the  magnitude  of  the  cone ;  and  since  the  specific  gravities  of  the 

VOL.  i.  B 


242       OF  SPECIFIC  GRAVITY  AND  THE  WEIGHING  OF  SOLED  BODIES. 

bodies  are  the  same,  it  follows  also,  that  the  weight  of  the  cylinder  is 
equal  to  three  times  the  weight  of  the  cone ;  consequently,  by  the 
principles  of  the  lever,  the  length  of  the  arm  AF,  is  three  times  the 
length  of  the  arm  BF;  for  it  is  a  well  known  property  in  the  doctrine 
of  mechanics,  that  when  two  bodies  of  different  weights  are  in  equi- 
librio  on  the  opposite  arms  of  a  straight  lever  : — 

The  lengths  of  the  arms  are  to  each  other,  reciprocally  as 
the  weights  of  the  suspended  bodies. 

Now,  suppose  the  cone  LKI,  which  is  obviously  equal  in  magnitude 
to  CDE,  to  be  abstracted  from  the  cylinder,  and  to  have  its  place  sup- 
plied by  another  cone  of  half  the  specific  gravity  as  the  former ;  then 
it  is  evident,  that  if  the  cone  CDE  is  suffered  to  retain  its  magnitude, 
it  will  preponderate  and  cause  the  cylinder  to  ascend  ;  it  is  therefore 
necessary,  in  order  that  the  equilibrium  shall'not  be  disturbed,  to 
diminish  the  magnitude,  and  consequently  the  weight  of  the  equili- 
brating cone;  and  for  the  purpose  of  assigning  the  quantity  of 
diminution, 

Put  m  m  the  magnitude  of  the  conical  body  CDE, 

m'~  the  magnitude  of  the  cylindrical  body  GHIK, 

m"-=.  the  magnitude  of  the  remaining  portion  cab, 

w  1=1  the  weight  which  the  cone  loses  in  the  fluid, 

w'  •=.  the  weight  lost  by  the  cylinder, 

w"—  the  weight  lost  by  the  remaining  cone  cab, 

s  =n  the  specific  gravity  of  the  fluid,  and 

s'  =i  the  specific  gravity  of  the  cone  and  cylinder. 

Then,  since  the  weight  which  a  body  loses  by  being  immersed  in  a 
fluid,  is  to  its  whole  weight,  as  the  specific  gravity  of  the  fluid  is  to 
the  specific  gravity  of  the  body,  we  have 

s'  :  s  :  :  m  s'  :  w ; 

therefore,  by  equating  the  products  of  the  extremes  and  means,  it  is 
wmmsi=. the  weight  lost  by  the  cone ;  but  according  to  the 
principles  of  mensuration,  the  magnitude  of  a  cylinder  is  equal  to 
three  times  the  magnitude  of  a  cone  of  the  same  base  and  altitude ; 
consequently,  we  have 

m'  nr  3m, 

and  for  the  weight  lost  by  the  cylinder,  we  get 

s'  :  s  :  :  3ms1  :  w' ; 

Horn  which,  by  equating  the  product  of  the  extremes  and  means,  we 
obtain 

w/zr  3ms~m's, 


OF  SPECIFIC  GRAVITY  AND  THE  WEIGHING  OF  SOLID  BODIES.       243 

and  in  like  manner,  the  weight  lost  by  the  cone  cab,  is  found  to  be 

w"—m"s. 

But  the  weights  which  the  several  bodies  possess  in  the  fluid,  are 
manifestly  equal  to  the  difference  between  the  absolute  weights  and 
the  weights  lost  ;  and  the  absolute  weights  are  equal  to  the  magni- 
tudes drawn  into  the  specific  gravities  ;  therefore,  we  have 

ms'  —  ms  zz  m  (s'  —  5)  zz  the  weight  of  the  cone  in  the  fluid, 

3ms'  —  3ms—3m(s'  —  s)zzthe  weight  of  the  cylinder, 

m"s'  —  w"  szzm"  (sr  —  s)  zz  the  weight  of  the  remaining  cone. 

Now,  if  from  the  weight  which  the  cylinder  possesses  in  the  fluid, 
we  subtract  the  corresponding  weight  of  the  cone,  and  to  the  remainder 
add  the  weight  of  another  cone  of  equal  magnitude  and  half  the 
specific  gravity  ;  then,  the  reduced  weight  of  the  cylinder  in  the  fluid 
becomes 

2ws'  4-  J*w*'  —  3mszzwi(2Js'  —  3s). 

But  according  to  the  conditions  of  the  problem,  this  weight  is  to 
be  in  equilibrio  with  the  weight  of  the  remaining  cone;  therefore,  by 
the  property  of  the  lever,  we  have 

ro(2js'  —  3s)  :  m"(s'  —  s)  :  :  3  :  1  ; 

and  from  this,  by  equating  the  products  of  the  extremes  and  means, 
we  get 

3m"  (s'  —  s)=m  (2  Is'  —  3s), 

in  which  equation  ra"  is  unknown  ;  in  order  ^therefore  to  determine  its 
value,  divide  both  sides  of  the  equation  by  3(s'  —  5),  and  it  becomes 


6(s'  —  s)  '  (190), 

But  this  that  we  have  determined,  is  the  magnitude  of  the  part 
which  remains,  whereas  the  problem  requires  the  magnitude  of  the 
part  to  be  cut  off;  now,  the  magnitude  of  the  whole  cone  is  m  ;  con- 
sequently, by  subtraction,  we  have 

«__  m(5s'  —  65)  ms' 

-   6(s--s)   :-  6(7=0'  09')- 

289.  The  practical  rule  for  reducing  this  equation  is  very  simple, 
it  may  be  expressed  in  the  following  manner. 

RULE.  Multiply  the  magnitude  of  the  cone  by  its  specific 
gravity,  and  divide  the  product  by  six  times  the  difference 
between  the  specific  gravity  of  the  cone  and  cylinder,  and  that 
of  the  fluid,  and  the  quotient  will  give  the  magnitude  of  the 
part  to  be  cut  off,  in  order  to  restore  the  equilibrium^ 

R  2 


244      OF  SPECIFIC  GRAVITY  AND  THE  WEIGHING  OF  SOLID  BODIES, 

290.  EXAMPLE.  Suppose  that  a  cone  and  cylinder  of  copper,  whose 
specific  gravity  is  nine  times  greater  than  that  of  water,  are  immersed 
in  a  fluid,  whose  specific  gravity  is  1.85,  and  placed  under  the  con- 
ditions specified  in  the  problem ;  how  much  must  be  cut  from  the 
lower  part  of  the  cone  to  restore  the  equilibrium,  the  diameter  of  the 
bases  and  the  altitude  of  the  cone  and  cylinder  being  respectively 
2  and  5  inches  ? 

Here,  by  operating  according  to  the  rule,  we  shall  have 

5.236*  X  9      47.124 
=  6(9=L85)=:1^ 

the  part  to  be  cut  off  from  the  cone,  in  order  that  the  remainder  may 
equipoise  the  cylinder;  or  we  may  calculate  the  magnitude  of  the 
equipoising  cone  by  equation  (190),  in  the  following  manner. 

5.236(5X9  —  6X1.85) 

m"— ^— — ~ i—4.135  cubic  inches; 

o(y  —  l.oo) 

which,  by  subtraction,  gives  1.098  cubic  inches  for  the  part  to  be 
cut  off. 

Put  d  •=.  the  diameter  of  the  base  of  the  cone  and  cylinder,  and 
h  ~  the  common  height  or  altitude. 

Then,  the  equations  (190)  and  (191)  will  become  transformed,  in 
terms  of  the  dimensions,  into  those  that  follow,  viz. 
,,__.2618<fA(5s'— 6s) 

~6(s'-5)  (192). 

This  equation  expresses  the  magnitude  of  the  cone  which  restores 
the  equilibrium,  and  the  following  one  expresses  the  magnitude  of  the 
frustum  which  has  to  be  deducted,  in  order  that  the  equilibrium  may 
obtain  ;  that  is, 

.2618^5' 

6(sf  —  s)'  (193). 

The  above  is  a  better  mode  of  expressing  the  quantities,  than  that 
exhibited  in  equations  (190)  and  (191) ;  since  it  is  not  probable,  that 
the  magnitudes  or  solid  contents  of  the  bodies  will  be  proposed,  with- 
out having  previously  stated  their  linear  dimensions. 

It  would  be  superfluous  to  propose  an  example,  for  the  purpose 
of  illustrating  the  reduction  of  the  equations  in  their  modified  form  ; 
for  since  the  expression  .261 8e?2  A,  is  equivalent  to  the  magnitude  of 

*  The  number  5.236  is  that  which  expresses  the  magnitude  of  the  cone,  for 
22X.  7854X5X$=5.236. 


OF  SPECIFIC  GRAVITY  AND  THE  WEIGHING  OF  SOLID  BODIES.       245 

the  cone,  the  rule  in  words  would  be  the  same  in  both  cases,  and 
there  fore,  4t  need  not  be  repeated. 

PROBLEM  XL. 

291.  If  a  solid  body  be  weighed  in  vacuo  and  in  a  fluid,  and 
the  different  weights  correctly  noted  : — 

It  is  required  from  thence,  to  compare  the  specific  gravities 
of  the  solid,  and  the  fluid  in  which  it  is  immersed. 

The  solution  of  this  problem  is  extremely  easy,  for  the  difference 
between  the  weight  of  the  body  in  vacuo  and  in  the  fluid,  gives  the 
weight  lost ;  therefore, 

Put  w  zr  the  weight  indicated  by  the  body  when  weighed  in  vacuo, 
w'izr  the  weight  when  weighed  in  the  fluid, 
s   —  the  specific  gravity  of  the  fluid  in  which  the  body  is 

weighed,  and 
s'  zz  the  specific  gravity  of  the  body. 

Then  we  have  w  —  w'  ==  the  weight  which  the  body  loses  by  being 
weighed  in  the  fluid ;  therefore,  by  the  fifth  proposition,  we  obtain 
w  —  w'  :  w  :  :  s  :  s' ;  that  is 
w  —  w'        s 

~^r  -T'  (194). 

consequently,  since  the  one  ratio  is  given,  the  latter  can  be  found. 

292.  EXAMPLE.  Suppose  a  piece  of  metal  to  indicate  40  ounces 
when  weighed  in  vacuo,  and  35  ounces  when  weighed  in  water ;  what 
is  the  specific  gravity  of  the  metal  ? 

Here,  by  substituting  the  given  numbers  in  equation  (194),  we  get 
4Q_ 35__1  _s_^ 
40        "  8  "~  s  ; 

hence,  the  specific  gravity  of  the  solid,  is  eight  times  greater  than  the 
specific  gravity  of  water. 

PROBLEM  XLI. 

293.  If  two  solid  bodies  be  weighed  in  vacuo  and  in  a  fluid, 
and  the  different  weights  correctly  noted  : — 

It  is  required  from  thence,  to  compare  the  specific  gravities 
of  the  bodies. 


246       OF  SPECIFIC  GRAVITY  AND  THE  WEIGHING  OF  SOLID  BODIES. 

The  intelligent  reader  will  readily  perceive,  that  the  present 
problem  is  only  an  extension  of  that  which  immediately  pj^cedes  it, 
and  is  proposed  with  the  design  of  detecting  the  law,  by  which  the 
specific  gravities  of  different  bodies  are  compared ;  for  which  purpose, 

Put  W  =  the  weight  of  the  heavier  body  when  weighed  in  vacuo, 
Wizr  its  weight  when  weighed  in  the  fluid, 
w  —  the  weight  of  the  lighter  body  when  weighed  in  vacuo, 
wf  zn  its  weight  when  weighed  in  the  fluid, 
s    =  the  specific  gravity  of  the  fluid  in  which  the  bodies  are 

weighed, 

s'  zz:  the  specific  gravity  of  the  heavier  body,  and 
s"  zz:  the  specific  gravity  of  the  lighter. 

Then,  the  weights  which  the  bodies  lose  by  being  weighed  in  the 
fluid,  are  respectively  W  —  W,  and  w  —  w',  and  because  the  weight 
lost,  is  to  the  whole  weight,  as  the  specific  gravity  of  the  fluid  is  to 
the  specific  gravity  of  the  solid ;  it  follows  in  the  case  of  the  heavier 
body,  that 

W  — W  :  W  ::s  :  s'f 

and  in  the  case  of  the  lighter  body,  it  is 

w  —  w  :  w  :  :  s  :  s"  ; 

therefore,  by  equating  the  products  of  the  extreme  and  mean  terms  in 
each  of  these  analogies,  we  have 

s'  ( W— W)  =  s  W,  and  *"  (w  —  w')  zz:  s  w ; 

consequently,  by  division,  we  obtain 

sW  sw 

-,  and  s  zz: 


•   W— W  "  w  —  w' 

hence,  by  analogy,  we  have 

'          sW  "          sw 

W— W  :  w  —  w" 

and  finally,  by  suppressing  s,  it  becomes 

W  w 

f  *  •  •»'/.  _______ 

W— W  '  w  —  w'' 

Hence  it  appears,  that  the  specific  gravities  of  the  two  bodies,  are 
to  one  another,  as  their  absolute  weights  divided  by  the  weights  which 
they  lose  in  the  fluid;  and  it  is  manifest,  that  the  same  law  will 
extend  to  any  number  of  bodies  whatever ;  therefore,  the  method  of 
comparison  has  been  determined. 


OF  SPECIFIC  GRAVITY  AND  THE  WEIGHING  OF  SOLID  BODIES.       247 

294.  EXAMPLE.  A  solid  body  whose  absolute  weight  is  23  ounces, 
when  weighed  in  a  certain  fluid,  loses  3  ounces  of  its  weight;  and 
another  body  of  800  ounces,  when  weighed  in  the  same  fluid,  loses 
1  02  ounces  ;  what  is  the  ratio  of  their  respective  gravities  ? 

Here,  since  the  loss  of  the  one  is  3  ounces,  and  that  of  the  other 
102  ounces,  it  follows,  that  the  specific  gravities  of  the  bodies,  are  to 
one  another  as  the  numbers  391  and  400  ;  for  we  have 


. 

And  in  like  manner,  if  three  or  more  bodies  be  weighed  in  the 
same  fluid,  their  specific  gravities  may  be  compared  with  one  another, 
and  also  with  that  of  the  fluid  in  which  they  are  weighed. 

PROBLEM  XLII. 

295.  If  a  solid  body  of  known  specific  gravity,  be  weighed  in 
several  different  fluids,  and  the  weights  correctly  indicated  :  — 

It  is  required  from  thence,  to  determine  the  ratio  of  their 
respective  gravities. 

This  problem,  it  will  readily  be  perceived,  is  exactly  the  reverse  of 
the  preceding  one,  and  therefore,  the  method  of  its  solution  may  easily 
be  discovered  ;  it  is,  however,  of  equal  utility  in  philosophical  inqui- 
ries, for  which  reason  we  have  proposed  it  in  this  place. 
Put  Wzz  the  weight  of  the  solid  when  weighed  in  vacuo, 
s   zz  the  specific  gravity  of  the  solid, 
w  zz  the  weight  which  it  indicates  in  a  fluid  whose  specific 

gravity  is  s',  and 

w'  zz  the  weight  which  it  indicates  in  a  fluid  whose  specific 
gravity  is  s\ 

Then  the  weights  which  the  body  loses,  by  being  weighed  in  the 
two  fluids,  are  respectively  W  —  w  and  W  —  w'  ;  but  the  weight  lost, 
is  to  the  whole  weight,  as  the  specific  gravity  of  the  fluid  is  to  the 
specific  gravity  of  the  solid  ;  hence,  for  the  first  fluid,  we  have 

W  —  w:  W  ::s':s; 

from  which,  by  equating  the  products  of  the  extremes  and  means, 
we  get 

s'Wzzs(W  —  w); 

therefore,  by  division,  we  obtain 


_*(W— 


,_ 


(195). 


248     or  SPECIFIC  GRAVITY  AND  THE  WEIGHING  OF  SOLID  BODIES. 

Again,  for  the  second  fluid,  we  have 

W—  w'  :  W  ::«"  :  s, 

and  from  this,  by  equating  the  products  of  the  extremes  and  means, 
we  get 

f*W=r*(W  —  «**), 
and  this  by  division  becomes 

W-tiQ 

~W~  (196). 

hence,  by  analogy,  we  obtain 


w  v 

and  finally,  by  suppressing  the  common  quantities  in  the  second  and 
fourth  terms,  we  get 

,':  W  —  w  ::  5"  :  W  —  w/; 

that  is,  the  specific  gravities  of  different  fluids,  are  as  the  weights, 
which  the  body  loses. 

296.  EXAMPLE.  A  mass  of  brick  whose  absolute  weight  is  64 
ounces,  and  its  specific  gravity  equal  to  twice  the  specific  gravity  of 
water;  when  weighed  in  one  fluid  indicates  37  ounces,  and  when 
weighed  in  another,  it  indicates  only  30  ounces  ;  it  is  required  from 
thence,  to  determine  the  ratio  of  the  respective  gravities  of  the  fluids, 
and  also  the  specific  gravity  of  each  ? 

Here  it  is  manifest,  that  the  weight  lost  by  the  solid  when  weighed 
in  one  fluid,  is 

W  —  w  —  64  —  37  z=  27  ounces, 

and  on  being  weighed  in  another  fluid,  it  loses 

W  —  w'  =  64  —  30  —  34  ounces. 

Now,  we  have  seen  above,  that  the  specific  gravities  of  the  fluid 
in  which  the  solid  is  weighed,  are  to  one  another,  respectively  as  the 
weights  which  the  solid  loses  in  them  ;  consequently,  we  have 

s'  :  s"  :  :  27  :  34. 

This  is  the  ratio  of  the  specific  gravities  ;  but  it  appears  from  equa- 
tions (195)  and  (196),  that  when  the  specific  gravity  of  the  solid  is 
known,  the  specific  gravity  of  the  fluid  in  which  it  is  weighed  can 
easily  be  ascertained. 

If,  therefore,  we  employ  the  specific  gravity  of  the  body  as  given  in 
the  question,  the  specific  gravity  of  the  first  fluid,  by  equation  (195), 
becomes 


OF  SPECIFIC  GRAVITY  AND  THE  WEIGHING  OF  SOLID  BODIES.       249 


. 

and  for  the  specific  gravity  of  the  second  fluid,  it  is 
,.=*(M-3°)  =  1.062  nearly. 

Hence,  the  absolute  specific  gravities  of  the  fluids,  are  respectively 
equal  to  0.844  and  1.062,  that  of  water  being  unity  ;  and  if  we  refer 
to  a  table  of  the  specific  gravities  of  known  fluids,  we  will  find  these 
numbers  to  correspond  with  alcohol  and  acetic  acid,  the  two  fluids  in 
which  the  brick  is  weighed. 

It  is  manifest,  that  what  has  been  done  above  in  respect  of  two 
fluids,  may  be  extended  to  any  number  of  fluids  whatever,  the  law, 
and  the  method  of  determining  the  specific  gravity,  being  the  same 
in  all. 

PROBLEM  XLIII. 

297.  If  a  body  be  weighed  in  air,  and  again  in  a  vessel  filled 
with  water,  the  weight  of  the  vessel  and  water  being  known  :  — 

It  is  required  from  thence,  to  determine  the  specific  gravity 
of  the  body. 

The  principle  upon  which  the  solution  of  this  problem  depends  is 
not  so  evident;  it  may,  nevertheless,  be  briefly  expounded  in  the 
following  manner. 

The  body  is  first  weighed  in  air  ;  then  being  put  into  a  vessel  filled 
with  water,  the  weight  of  which  is  known,  it  will  expel  a  quantity  of 
the  fluid  equal  to  its  own  bulk,  and  because  the  specific  gravity  of  the 
body  is  supposed  to  be  greater  than  that  of  water,  it  is  obvious,  that 
the  vessel  and  its  contents  will  now  be  heavier  than  it  was  before  the 
body  was  put  into  it,  by  a  quantity,  which  is  equal  to  the  difference 
between  the  weight  of  the  body,  and  that  of  an  equal  bulk  of  the 
water  ;  but  the  body  loses  as  much  of  its  own  weight  in  the  fluid,  as 
is  equal  to  that  of  the  water  displaced  ;  hence,  the  determination  of 
its  specific  gravity  becomes  easy,  for  which  purpose, 

Put  w   zz  the  weight  of  the  body  when  weighed  in  air, 

w'  zz  .the  weight  of  the  vessel  and  the  water  before  the  body 

is  put  into  it, 

w"  zz  the  weight  of  the  vessel,  the  water,  and  the  solid,  and 
5    zz  the  specific  gravity  of  the  solid. 


250      OF  SPECIFIC  GRAVITY  AND  THE  WEIGHING  OF  SOLID  BODIES. 

Then  the  weight  gained  by  the  vessel,  by  reason  of  the  immersion 
of  the  solid  body,  is  w"  —  wf,  and  this  expresses  the  weight  of  the 
body  in  the  fluid ;  consequently,  the  weight  which  the  body  loses,  is 
w—(w"—w'). 

But  by  the  5th  Proposition  preceding,  the  weight  lost,  is  to  the 
whole  weight,  as  the  specific  gravity  of  the  fluid  is  to  the  specific 
gravity  of  the  body ;  therefore,  because  the  specific  gravity  of  water 
is  expressed  by  unity,  we  have 

w  —  (w"  —  w')  :  w  :  :  I  :  s ;  that  is 

w 

-w_.(w»_-f£,')-  (197). 

298.  The  practical  rule  for  reducing  the  above  equation,  may  be 
expressed  in  words  at  length  in  the  following  manner. 

RULE.  Divide  the  weight  of  the  body  in  air,  by  the  difference 
between  that  weight,  and  what  is  gained  by  the  vessel  in  con- 
sequence of  the  immersion,  and  the  quotient  will  express  the 
specific  gravity  of  the  solid. 

299.  EXAMPLE.    A  solid  body  when  weighed  in  air,  indicates  a 
weight  of  16  ounces;  and  when  put  into  a  vessel  filled  with  water, 
the  vessel,  the  solid  and  the  water  together,  indicate  a  weight  of  36 
ounces ;  whereas  the  vessel  when  filled  with  water  alone  weighs  only 
32  ounces ;  required  the  specific  gravity  of  the  body,  that  of  water 
being  expressed  by  unity  ? 

Here,  by  following  the  directions  of  the  rule,  we  have 
_  16  _ 

""16  —  (36  —  32)  - 

being  a  measure  of  specific  gravity,  which  corresponds  very  nearly 
with  American  ebony,  a  very  suitable  material  for  hydrostatical  expe- 
riments. 

300.  We  have  hitherto  been  considering  the  nature  of  bodies  that 
are  specifically  heavier  than  the  fluids  in  which  they  are  weighed,  and 
consequently,  such  as  would  sink  to  the  bottom,  if  they  were  left  to 
the  free  action  of  their  own  gravity ;  we  have  therefore,  in  the  next 
place,  to  consider  such  bodies  as  are  specifically  lighter  than  the  fluids 
on  which  they  are  placed,  and  consequently,  such  as  would  float  on 
the  surface,  if  left  to  the  free  exercise  of  their  own  buoyancy. 

This  is  a  very  abstruse,  but  interesting  and  important  department  of 
Hydro-Dynamical  science ;  for  on  it  depends  the  principles  by  which 
we  determine  the  conditions  of  equilibrium,  and  the  stability  of  floating 
bodies. 


OF  SPECIFIC  GRAVITY  AND  THE  WEIGHING  OF  SOLID  BODIES.       251 

PROBLEM  XLIV. 

301.  If  a  solid  body  is  weighed  in  a  fluid  of  greater  specific 
gravity  than  itself,  and  of  which  the  specific  gravity  is  given : — 

It  is  required  from  thence,  to  determine  the  specific  gravity 
of  the  solid. 

The  usual  method  of  resolving  this  problem,  is,  to  attach  the  body 
whose  specific  gravity  is  required,  to  another  body  specifically  heavier 
than  the  fluid,  and  of  a  sufficient  magnitude  to  cause  the  compound 
mass  to  sink ;  then,  by  observing  the  weights  indicated  by  the  sub- 
sidiary body,  and  by  the  compound  mass,  when  they  are  separately 
placed  in  the  fluid,  the  specific  gravity  of  the  lighter  body  will  become 
known. 

Put  rv  zz  the  weight  of  the  lighter  body  when  weighed  in  vacuo, 
TV  zz  the  difference  between  the  weight  of  the  compound  mass, 
and  that  of  the  heavier  body,  when  weighed  separately 
in  the  fluid, 

s  zz:  the  specific  gravity  of  the  fluid,  and 
s'  zz  the  specific  gravity  of  the  solid  required. 

Now,  it  is  manifest,  that  the  effort  of  buoyancy,  or  the  force  of 
ascent  of  the  lighter  body,  is  equal  to  the  difference  between  the 
weight  of  the  compound  mass  in  the  fluid,  and  that  of  the  heavier 
body  in  the  fluid ;  therefore, 

w'  zz:  the  force  of  ascent  of  the  lighter  body. 

But,  the  force  of  ascent  of  the  lighter  body,  or,  as  it  is  more  ele- 
gantly denominated,  the  effort  of  buoyancy,  is  evidently  equivalent  to 
the  excess  of  the  weight  of  a  quantity  of  the  fluid,  equal  in  magnitude 
to  the  lighter  body,  above  the  weight  of  the  lighter  body  when  weighed 
in  vacuo ;  consequently,  the  weight  of  a  quantity  of  the  fluid,  equal 
in  bulk  to  the  lighter  body,  is  expressed  by  rv  -\-  w' ;  hence,  we  have 

from  which,  by  reducing  the  proportion,  we  get 
. sw 

rV-\-n' 

Or  put  W  zz  the  weight  of  the  compound  mass  when  weighed  in  the 
fluid,  and  W  zz:  the  corresponding  weight  of  the  heavier  body  when 
weighed  separately  ;  then  we  have 

/ -\\r         \\rt  . 

rv  —  w  —  vv    , 


252       OF  SPECIFIC  GRAVITY  AND  THE  WEIGHING  OF  SOLID  BODIES. 

let  this  value  of  the  force  of  ascent,  be  substituted  instead  of  it,  in  the 
above  value  of  s',  and  we  shall  obtain 


sw 


-w_j_W—  W"  (198). 

302.  The  practical  rule  for  reducing  this  equation,  may  be  expressed 
in  words  at  length  in  the  following  manner. 

RULE.  Multiply  the  absolute  weight  of  the  body,  of  which 
the  specific  gravity  is  required,  by  the  specific  gravity  of  the 
fluid;  then,  divide  the  product  by  the  absolute  weight  of  the 
body,  increased  by  its  force  of  ascent,  and  the  quotient  will  be 
the  specific  gravity  sought. 

303.  EXAMPLE.  A  piece  of  wood  which  weighs  in  vacuo  22  ounces, 
is  attached  to  a  piece  of  metal  of  such  a  magnitude  as  to  weigh  12 
ounces  in  water  ;  now,  supposing  that  when  the  compound  mass  is 
placed  in  water,  it  is  found  to  weigh  20  ounces  ;  what  is  the  specific 
gravity  of  the  wood,  that  of  water  being  expressed  by  unity? 

Here,  by  proceeding  according  to  the  rule,  we  get 


and  this,  by  referring  to  a  table  of  specific  gravity,  is  found  to  cor- 
respond very  nearly  with  the  medium  species  of  citron  wood. 

PROBLEM  XLV. 

304.  Having  given  the  weight  of  a  vessel  full  of  water,  both 
before  and  after  a  body  of  a  given  weight  in  air  is  immersed 
in  it,  together  with  the  specific  gravity  of  the  air  at  the  time  of 
observation  :  — 

It  is  required  to  determine  the  specific  gravity  of  the 
immersed  solid,  the  weight  of  the  air  being  taken  into 
consideration  . 

This  problem  is  perhaps  more  curious  than  useful  ;  but  since  it 
tends  to  excite  the  reader's  attention,  and  to  render  him  familiar  with 
the  resources  of  analysis,  we  have  thought  proper  to  introduce  it  in 
this  place  ;  and  in  order  to  its  resolution, 

Put  w  i=  the  weight  of  the  vessel  when  full  of  water, 

TV  =n  the  weight  of  the  solid  body  when  weighed  in  air, 
w":n  the  weight  of  the  vessel  with  the  solid  in  it,  when  filled 
up  with  water, 


OF  SPECIFIC  GRAVITY   AND  THE  WEIGHING  OF  SOLID  BODIES.       253 

Put  m  iz:  the  magnitude  of  the  solid, 

s  ~  its  specific  gravity,  or  the  quantity  which  the  problem 

demands,  and 
s'  zr  the  specific  gravity  of  air  at  the  time  of  the  experiment. 

Then,  because  the  weight  of  any  body,  is  expressed  by  the  product 
of  its  magnitude  drawn  into  its  specific  gravity ;  it  follows,  that  when 
the  weight  of  the  air  is  disregarded,  the  weight  of  the  solid  is 

rv'  zn  m  s  ; 

but  the  weight  of  a  quantity  of  air  equal  in  magnitude  to  the  body, 
is  ms',  and  it  evidently  loses  as  much  weight  as  that  of  the  fluid  which 
it  displaces ;  consequently,  when  the  weight  of  the  air  is  considered, 
the  weight  of  the  body  when  weighed  in  air,  becomes 

rv'  m  m  (s  —  s'), 

therefore,  by  division,  the  magnitude  of  the  solid  is 
w' 

consequently,  the  weight  in  vacuo,  is 
snf 

s — s  ' 

Then,  because  rv"  denotes  the  weight  of  the  vessel  with  the  solid  in 
it,  when  filled  up  with  water,  and  rv  the  weight  of  the  vessel  when  full 
of  water ;  then  w" — rv  expresses  the  weight  which  the  vessel  has  gained 
by  the  immersion  of  the  solid,  and  this  is  manifestly  equal  to  the  dif- 
ference between  the  weight  of  the  solid,  and  that  of  an  equal  bulk  of 
the  fluid ;  therefore,  the  weight  of  a  quantity  of  water,  equal  in  bulk 
to  the  solid,  is 

sw  s(rv  -\-w'  —  n")-\-s'(w" — rv) 

-. — rv  -4-  rv  — 7 • 

s — /  s  —  / 

Then,  as  the  specific  gravity  of  the  body,  is  to  the  specific  gravity 
of  water,  so  is  the  weight  of  the  body,  to  the  weight  lost ;  that  is, 

sw        s(w-\-  rv' —  rv")  +  s' (w" —  w) 
:7=^:  s  —  s' 

and  this,  by  suppressing  the  denominator  in  the  homologous  terms, 
becomes 

s  :  1  :  :  sw   :  s(rv-\-rv' — rv")-\-s'(w" — rv), 

and  by  equating  the  products  of  the  extreme  and  mean  terms,  we 
obtain 

s(w  4-  rv' —  rv")  -\-  s'  (rv" —  rv)  —  rv' ; 


254       OF  SPECIFIC  GRAVITY  AND  THE  WEIGHING  OF  SOLID  BODIES. 

therefore,  by  transposition,  we  get 

s(rv  +  nf —  rv"}  rz:  n>  —  s'  (TV" —  w), 

and  finally,  by  division,  we  shall  obtain 

__  ?v' — s'(w" — TV) 

~  w  +  rv'-w"  '  (199). 

305.  The  practical  rule  by  which  the  reduction  of  the  above  equa- 
tion is  effected,  may  be  expressed  in  words  at  length  in  the  following, 
manner. 

RULE.  From  the  weight  of  the  vessel  with  the  solid  in  it, 
when  filled  up  with  water -,  subtract  the  weight  of  the  vessel 
when  full  of  water  only  ;  then  multiply  the  remainder  by  the 
specific  gravity  of  the  air  at  the  time  of  observation,  and 
subtract  the  product  from  the  iveight  of  the  solid  in  air  for  a 
first  number. 

To  the  vjeight  of  the  vessel  when  full  of  water,  add  the 
weight  of  the  solid  when  weighed  in  air,  and  from  the  sum, 
subtract  the  weight  of  the  vessel  with  the  solid  in  it,  when 
filled  up  with  water,  and  the  remainder  mill  be  a  second 
number. 

Divide  the  first  number  by  the  second,  and  the  quotient 
will  give  the  specific  gravity  of  the  solid. 

306.  EXAMPLE.  The  weight  of  a  vessel  when  full  of  water  is  68  Ibs. 
avoirdupois,  and  the  weight  of  a  solid  body  when  weighed  in  air  of  a 
medium  temperature,  is  34  Ibs. ;  now,  when  the  solid  is  placed  in  the 
vessel,  its  bulk  of  water  is  expelled,  and  the  vessel  being  then  weighed, 
is  found  to  indicate  86  Ibs. ;  required  the  specific  gravity  of  the  solid 
body? 

When  the  air  is  of  a  medium  temperature,  its  specific  gravity  is  very 
nearly  expressed  by  the  fraction  0.0012,  that  of  water  being  unity; 
therefore,  by  proceeding  according  to  the  rule,  we  have 

_ 34 -.0012(86-  68) _ 

68  +  34—86 

which,  by  referring  to  a  table  of  specific  gravities,  is  found  to  correspond 
very  nearly  with  the  opal  stone,  a  silicious  material  of  very  great  value, 
for  the  senator  Nonius  preferred  banishment  to  parting  with  his 
favourite  opal,  which  was  coveted  by  Antony. 

I 


CHAPTER  XL 

OF  THE  EQUILIBRIUM  OF  FLOATATION. 


BY  the  equilibrium  of  floatation  is  generally  meant  the  position  of  a  floating  body^ 
when  its  centre  of  gravity  is  in  the  same  vertical  line  with  the  centre  of  gravity  of 
the  displaced  fluid.  When  the  lower  surface  of  the  floating  body  is  spherical  or 
cylindrical,  the  centre  will  coincide  with  the  centre  of  the  figure ;  as,  in  all  circum- 
stances, the  height  of  this  point,  as  well  as  the  form  of  the  volume  of  fluid  displaced, 
must  remain  invariable.  In  the  next  proposition,  we  shall  prove,  that  the  place  of 
the  centre  is  determined  by  the  doctrine  of  forces  combined  with  the  elementary 
principles  of  hydrostatics,  by  considering  the  form  and  extent  of  the  surface  of  the 
displaced  portion  of  the  fluid,  compared  with  its  bulk,  and  with  the  situation  of  its 
centre  of  gravity.  Our  inquiry  will  be  found  to  embrace  also  rectangular  figures ; 
solids  in  the  form  of  paraboloids  and  cylinders,  together  with  the  equilibrium  of 
floatation  of  solids  immersed  in  fluids  of  different  specific  gravities ;  the  theory  of 
construction  of  the  aerometer ;  the  hydrostatic  balance,  and  the  method  of  weighing 
solid  bodies  in  fluids.  The  reader  will  therefore  consider  this  syllabus  as  the 
paraphrase  of  a  definition  for  the  term  equilibrium  of  floatation. 

PROPOSITION  VI. 

307.  When  a  body  floats  in  a  state  of  equilibrium  on  the 
surface  of  an  incompressible  and  non-elastic  fluid  : — 

The  centre  of  gravity  of  the  whole  body,  and  that  of  the 
part  immersed ;  or  which  is  the  same  thing,  that  of  the  fluid 
displaced,  must  occur  in  the  same  vertical  line* 

Let  ADBH  be  a  vertical  section  passing  through  G  and  g,  the  centres 
of  gravity  of  the  whole  body  AHBD;  and  of  the  immersed  part  A  DB, 
which  falls  below  EF  the  plane  of  floatation. 


*  The  vertical  line,  which  passes  through  the  centres  of  gravity  of  the  whole 
body,  and  the  part  immersed  beneath  the  plane  of  floatation,  is  generally  denomi- 
nated the  line  of  support. 


256  OF  THE  EQUILIBRIUM  OF  FLOATATION, 

For  since  g  is  the  centre  of  gravity 
of  the  immersed  part  of  the  body,  or  of 
the  fluid  ADB,  it  is  also  the  centre  of  all 
the  forces  or  weights  of  the  particles  of 
fluid  in  ADB,  tending  downwards;  but 
because  the  body  is  at  rest,  the  same 
point  g  is  also  the  centre  of  all  the 

pressures  of  the  fluid  tending  upwards,  by  which  the  weight  of  the 
body  ADB  H  is  sustained  in  a  state  of  equilibrium. 

Now,  it  is  manifest,  that  the  sum  of  all  the  downward  forces,  is 
equal  and  opposite  to  the  sum  of  all  the  upward  forces,  otherwise  the 
body  could  not  be  in  a  state  of  rest ;  but  the  direction  in  which  the 
weight  of  the  body  tends  downwards  from  c,  is  perpendicular  to  the 
horizon;  consequently,  the  line  CD  which  passes  through  G  and  g, 
the  centres  of  gravity  of  the  whole  body  and  of  the  part  immersed, 
must  also  be  perpendicular  to  the  horizon  ;  for  if  it  is  not,  the  body 
must  have  a  rotatory  motion,  but  according  to  the  hypothesis,  the  body 
is  at  rest;  therefore,  the  line  CD  is  perpendicular  to  the  horizon. 

From  Propositions  III.  and  VI.,  it  is  obvious,  that  for  a  floating 
body  to  remain  at  rest,  or  in  a  state  of  equilibrium,  two  conditions 
must  obtain,  and  these  are, 

The  weight  of  the  floating  body,  and  that  of  the  fluid  dis- 
placed, must  be  equal  to  one  another. 

This  is  manifest  from  the  inference  under  the  third  Proposition,  and 
the  second  condition  to  which  we  have  alluded  forms  the  substance  of 
Proposition  VI.,  viz. 

The  centre  of  gravity  of  the  whole  body  and  that  of  the 
part  immersed,  or  of  the  fluid  displaced,  occur  in  the  same 
vertical  line. 

It  is  extremely  obvious,  from  the  nature  of  the  subject,  that  both  the 
above  conditions  must  have  place ;  for  if  the  first  do  not  obtain,  the 
body  will  ascend  or  descend  in  a  direction  which  is  perpendicular  to 
the  horizon  ;  and  if  the  second  fail,  the  body  will  turn  about  its  centre 
of  gravity,  until  the  centre  and  that  of  the  fluid  displaced  occur  in 
the  same  vertical  line  ;  and  if  both  these  conditions  fail  together,  the 
body  will  partake  of  a  progressive  and  a  rotatory  motion  at  one  and 
the  same  time. 

From  the  Proposition  which  we  have  demonstrated  above,  two  or 
three  very  useful  inferences  may  be  deduced,  as  follows. 


OF  THE   EQUILIBRIUM  OF  FLOATATION.  257 

308.  INF.  1.  If  any  homogeneous  plane  figure  be  divided  symme- 
trically by  its  vertical  axis,  and  placed  in  a  fluid  of  greater  specific 
gravity  than  itself : — 

It  will  remain  in  equilibria  with  its  bisecting  axis  vertical. 

309.  INF.  2.  If  any  homogeneous  solid,  generated  by  the  revolu- 
tion of  a  curve  round  its  vertical  axis,  be  placed  in  a  fluid  of  greater 
specific  gravity  than  itself: — 

It  will  remain  in  equilibria  in  that  position,  that  is,  with  its 
axis  vertical. 

310.  INF.  3.   If  in  any  homogeneous  prismatic  body,  whose  axis  is 
horizontal,  the  centre  of  gravity  of  the  section  made  through  its  middle 
parallel  to  its  base,  be  in  the  same  vertical  line  with  the  centre  of 
gravity  of  that  part  of  the  solid  which  falls  below  the  plane  of  float- 
ation : — 

The  body  will  remain  in  equilibria  in  that  position,  if  placed 
in  a  fluid  of  greater  specific  gravity  than  itself. 

This  is  manifest,  for  the  centres  of  gravity  of  the  whole  prism,  and 
of  the  part  immersed,  may  be  conceived  to  lie  in  those  points,  and 
consequently,  the  prismatic  body  is  in  a  state  of  equilibrium. 

PROPOSITION  VII. 

311.  When  a  solid  body  floats  upon  a  fluid  of  greater  specific 
gravity  than  itself,  and  has  attained  a  state  of  equilibrium  : — 

The  magnitude  of  the  body,  is  to  that  of  the  part  immersed 
below  the  plane  of  floatation,  as  the  specific  gravity  of  the 
fluid  is  to  that  of  the  floating  body. 

For  by  the  inference  to  the  third  proposition,  when  the  body  floats 
in  a  state  of  equilibrium": — 

The  weight  of  the  floating  body,  is  equal  to  the  weight  of  a 
quantity  of  the  fluid,  whose  magnitude  is  the  same  as  that 
portion  of  the  solid  which  falls  below  the  plane  of  floatation. 

And  according  to  this  principle,  the  truth  of  the  above  Proposition 
is  demonstrated  ;  for, 

Put  m  =  the  magnitude  of  the  whole  floating  body, 
m'—  the  magnitude  of  the  part  immersed, 
s   :=  the  specific  gravity  of  the  floating  body, 
w  —  its  weight, 

VOL.    I.  S 


258  OF  THE  EQUILIBRIUM  OF  FLOATATION. 

s'  n:  the  specific  gravity  of  the  fluid,  and 
w/iz:  the  weight  of  a  quantity  of  the  fluid,  of  the  same  magni- 
tude as  that  part  of  the  body  which  falls  below  the 
plane  of  floatation ;  then,  according  to  the  above  in- 
ference just  stated,  we  get 

w~w. 

But  because  the  weight  of  any  body  is  expressed  by  the  product  of 
its  magnitude  drawn  into  its  specific  gravity ;  it  follows,  that 

w  —  ms,  and  w'  —  m's', 
consequently,  by  comparison,  we  have 

ms  =  m's'.  (200). 

Therefore,  if  this  equation  be  converted  into  an  analogy,  the  truth 
of  the  Proposition  will  become  manifest ;  for 

m  :  m'  :  :  s'  :  s, 

being  precisely  the  principle  which  the  Proposition  implies. 
From  the  principle  demonstrated  above,  various  curious  and  inte- 
resting questions  may  be  resolved,  and  by  selecting  a  few  which  point 
directly  to  practical  subjects,  the  information  afforded  by  their  reso- 
lution will  sufficiently  repay  the  labour  of  an  attentive  perusal. 

312.  EXAMPLE  I.  A  cubical  block  of  fir,  whose  specific  gravity  is 
0.55,  floats  in  equilibrio  on  the  surface  of  a  fluid  whose  specific  gravity 
is  1.026  ;  how  much  of  the  block  is  above,  and  how  much  below  the 
plane  of  floatation,  the  entire  magnitude  being  equal  to  512  cubic 
inches  ? 

Here,  by  the  Proposition,  we  have 

512  :  m'  :  :  1.026  :  0.55, 

and  from  this,  by  equating  the  products  of  the  extreme  and  mean 
terms,  we  get 

1.026m' zz  281.6, 
and  finally,  dividing  by  1.026,  we  obtain 

m'zz        '    m  274. 464  cubic  inches. 

It  therefore  appears,  that  the  quantity  of  the  solid  immersed  below 
the  plane  of  floatation,  is  274.464  cubic  inches ;  consequently,  the 
part  extant  is  512  —  274.464  =  237.536  cubic  inches,  being  less  than 
half  the  magnitude  of  the  body,  by  18.464  cubic  inches. 

313.  EXAMPLE  2.  Let  the  specific  gravities  and  the  magnitude  of 
the  body  remain  as  in  the  last  example  ;  what  weight  must  be  added 


OF  THE  EQUILIBRIUM  OF  FLOATATION.  259 

to  the  body,  in  order  that  its  upper  surface  may  be  made  to  coincide 
with  that  of  the  fluid  ? 

Put  x  nz  the  weight  which  must  be  added  to  the  solid,  in  order  that 
it  may  sink  to  a  level  with  the  surface  of  the  water ;  then,  we  have 

m:  m  +  x  ::  0.55  :  1.026, 
and  by  equating  the  products  of  the  extremes  and  means,  we  get 

0.55  (m  +  x)  =  1.026m; 
therefore,  by  transposition,  we  obtain 

0.55*  =  0.476m; 
but  according  to  the  question,  mzr:512  cubic  inches,  hence  we  get 

0.55*:=  243.712, 
and  finally,  by  division,  we  have 

243  712 
x  zn — -—— - —  —  443.1 13  cubic  inches;  but  a 

cubic  inch  of  fir  of  the  given  specific  gravity,  weighs  0.0198  Ibs. 
avoirdupois,  very  nearly ;  consequently,  the  weight  to  be  added  for 
the  purpose  of  making  the  solid  sink  to  the  same  level  as  the  surface 
of  the  fluid,  is 

.0198X443.113  — 8.774  Ibs.  nearly. 

314.  But  to  determine  generally,  the  magnitude  which  must  be 
added  to  the  original  solid,  in  order  that  its  surface  maybe  coincident 
with  that  of  the  fluid  :-— Let  zziithe  weight  to  be  added;  then,  by 
the  Proposition,  we  have 

m  -{-  x  :  m  :  :  s  :  s, 

from  which,  by  equating  the  products  of  the  extremes  and  means, 
we  get 

s  (m  -\-  x)  —  s'  m, 

and  by  separating  the  terms,  it  becomes 

^m  -\-  sx^ns'm, 

and  finally,  by  transposition  and  division,  we  obtain 
_m(s'  —  s) 

s  (201). 

Therefore,  the  practical  method  of  reducing  this  equation,  may  be 
expressed  in  words  in  the  following  manner. 

RULE.  From  the  specific  gravity  of  the  fluid,  subtract  that 
of  the  solid ;  then,  multiply  the  remainder  by  the  magnitude 
of  the  solid,  and  divide  the  product  by  its  specific  gravity  for 
the  weight  to  be  added. 

s2 


260  OF  THE  EQUILIBRIUM  OF  FLOATATION. 

315.  EXAMPLE  3.  A  cubic  mass  of  oak,  whose  specific  "gravity  is 
0.872,  is  placed  in  a  cistern  of  water,  and  when  it  has  attained  a  state 
of  equilibrium  with  its  sides  vertical,  it  stands  20  inches  above  the 
surface  of  the  fluid;  what  is  the  magnitude  of  the  solid,  the  specific 
gravity  of  the  water  being  represented  by  unity  ? 

In  order  to  resolve  this  example,  we  shall  first  investigate  a  general 
formula,  which  will  apply  to  every  case  of  a  similar  nature,  when  the 
specific  gravity  of  the  fluid  and  that  of  the  solid  are  given  ;  for  which 
purpose, 

Put  x  nr  the  length  of  one  side  of  the  solid  ;  then  is 

x — a*— the  length  of  that  portion  which  is  below  the  plane  of 
floatation. 

But  by  the  principles  of  mensuration,  the  magnitude  of  the  whole 
body  is 

razz  a?8, 

and  that  of  the  part  immersed,  is 

m'=  (x  —  a)  Xs'zz  a:8  —  ax* ; 

therefore,  by  the  Proposition,  we  obtain 

m  '  m!  \  '.  s'  i  s  ; 
and  this,  by  substitution,  becomes 

a;8  :  x*  —  a  a?2  :  :  s'  :  s ; 

from  which  by  equating  the  products  of  the  extremes  and  means, 
we  get 

sz'zz  s'O8  —  as2); 
and  by  separating  the  terms,  we  obtain 

sx9  zz  5'  x3  —  as'  x9; 
consequently,  by  transposition  and  division,  it  is 

as' 

~7^7)'  (202). 

And  the  practical  rule  supplied  by  this  equation,  may  be  expressed 
in  words  at  length  in  the  following  manner. 

RULE.  Multiply  the  specific  gravity  of  the  fluid  by  the 
height  of  the  body  above  its  surface,  and  divide  the  product 
by  the  difference  between  the  specific  gravity  of  the  fluid 
and  that  of  the  solid,  and  the  quotient  will  give  the  side 
of  the  cube  required. 

*  The  quantity  a,  is  here  put  to  denote  the  height  of  the  body  abovelhe  fluid. 


OF  THE  EQUILIBRIUM  OF  FLOATATION.  261 

Taking,  therefore,  the  data  as  proposed  in  the  foregoing  example, 
and  we  shall  obtain 

20X1 
#~  , Q  S7,  =r  156.29  inches;  consequently, 

the  magnitude,  or  cubical  contents  of  the  body,  is 

156.29X  156.29X  156.29  =  3815627.7  inches. 
In  addition  to  the  foregoing  examples,  which  might  very  appro- 
priately have  been  ranked  under  the  head  of  problems,  the  seventh 
proposition  affords  the  following  inferences. 

316.  INF.  1.  If  two  bodies  floating  on  the  same  fluid,  be  in  a  state 
of  equilibrium  : — 

The  specific  gravities  of  those  bodies,  will  be  to  one  another 
directly  as  the  parts  below  the  plane  of  floatation ,  and  in- 
versely  as  the  whole  magnitudes  of  the  bodies. 

317.  lNF.^2.  If  the  same  body  float  upon  two  different  fluids,  and 
be  in  a  state  of  equilibrium  on  each  : — 

The  specific  gravities  of  the  fluids,  will  be  to  one  another, 
inversely  as  the  parts  of  the  body  below  the  plane  of  floatation. 

318.  INF.  3.  If  different  bodies  float  in  equilibrio  on  the  surfaces 
of  different  fluids,  and  if  the  parts  below  the  planes  of  floatation  be 
equal  among  themselves : — 

The  specific  gravities  of  the  fluids,  will  be  to  one  another 
directly  as  the  weights  of  the  bodies,  or  directly  as  the  magni- 
tudes of  the  bodies  drawn  into  their  specific  gravities. 

319.  INF.  4.  If  any  number  of  bodies  of  the  same  weight,  but  of 
different  specific  gravities,  float  in  equilibrio  on   the  surface  of  the 
same  fluid : — 

The  magnitudes  of  the  parts  below  the  plane  of  floatation, 
are  equal  to  one  another. 

320.  INF.  5.  If  a  body  float  in  equilibrio  on  the  surface  of  a  given 
fluid,  and  if  the  part  below  the  plane  of  floatation  be  increased  or 
diminished  by  a  given  quantity  : 

The  absolute  weight  of  the  body,  (in  order  that  the  equili- 
brium may  still  obtain,)  must  be  increased  or  decreased  by  a 
weight,  which  is  equal  to  the  weight  of  the  portion  of  the  fluid 
that  is  more  or  less  displaced,  in  consequence  of  increasing  or 
diminishing  the  immersed  part  of  the  body,  or  that  which  falls 
below  the  plane  of  floatation. 


262  OF  THE  EQUILIBRIUM  OF  FLOATATION. 

321.  The  principle  announced  in  the  last  inference,  may  be  demon- 
strated in  the  following  manner. 

Put  ra  —  the  magnitude  of  the  body  at  first,  when  in  a  state  of 
equilibrium, 

ra':=r  the  part  originally  below  the  plane  of  floatation, 

m"=:  the  part  by  which  it  is  increased  or  diminished, 

5    zn  the  specific  gravity  of  the  body, 

s'  —  the  specific  gravity  of  the  fluid,  and 

w  ~  the  weight  by  which  the  body  is  increased  or  diminished, 
in  consequence  of  the  increase  or  decrease  of  the  im- 
mersed part. 

Then,  because  the  quantity  of  fluid  displaced,  is  equal  to  the  mag- 
nitude of  the  body  which  displaces  it,  it  follows,  that  the  weight  of 
the  displaced  fluid  is  expressed  by  (m'-f-  w")s',  and  the  weight  of  the 
whole  solid  with  which  it  is  in  equilibrio,  is  (ms  +  w} ;  consequently, 
we  have 

(rn1  -|-  mH)s' i=.ms-\-w. 
Now  it  is  manifest,  that  in  the  case  of  the  first  equilibrium, 

m V  rr  m  s ; 

it  therefore  follows,  that 
tm"s'  =  w. 

That  is,  the  weight  by  which  that  of  the  body  must  be  increased  or 
diminished,  to  restore  the  equilibrium  : — 

Is  equal  to  the  weight  of  that  quantity  of  the  fluid  which 
is  more  or  less  displaced,  in  consequence  of  the  increase  or 
decrease  of  the  part  below  the  plane  of  floatation. 


PROBLEM  XLVI. 

322.  If  a  solid  body  in  the  form  of  a  paraboloid,  be  in  a  state 
of  quiescence  on  the  surface  of  a  fluid,  whose  specific  gravity 
bears  any  relation  to  that  of  the  body : — 

It  is  required  to  determine  how  much  of  the  solid  will  sink 
beneath  the  plane  of  floatation. 

Let  GACBH  be  a  vertical  section  passing  along  the  axis  of  the  solid, 
and  cutting  the  plane  of  floatation  in  the  line  AB  ;  CD  being  the  axis 


OF  THE  EQUILIBRIUM  OF  FLOATATION.  263 

of  the  solid,  GH  the  diameter  of  its  base, 
and  ABHG  the  portion  which  falls  below 
EF  the  surface  of  the  fluid. 

Put  a  :=:  CD,  the  axis  of  the  paraboloid, 
d  •=.  G  H,  the  diameter  of  its  base, 
m  —  the  magnitude  of  the  entire 

solid  GACBH, 

w'mthe  magnitude  of  the  immersed  part, 
s   —the  specific  gravity  of  the  body, 
s'  =z  the  specific  gravity  of  the  fluid,  and 
x  nrci,  the  axis  of  that  portion  of  the  body,  which  in  a  state 
of  equilibrium,  remains  above  the  plane  of  floatation. 

Consequently,  by  the  seventh  proposition,  we  have 

m  :  m'  :  :  s'  :  s. 

But  by  the  principles  of  mensuration,  the  solidity  of  a  paraboloid  is 
equal  to  one  half  the  solidity  of  its  circumscribing  cylinder  ;  there- 
fore, we  get 

m  —  .3927ad2, 

and  similarly,  we  obtain 

solidity  ACB  =  1.5708^  a;2, 

where  p  is  the  parameter  of  the  generating  parabola. 
Now,  the  writers  on  conic  sections  have  demonstrated,  that  accord- 
ing to  the  property  of  the  generating  curve, 

4ap  z=  d*  ; 

let  this  value  of  d?  be  substituted  instead  of  it  in  the  preceding  value 
of  m,  and  we  shall  obtain 


consequently,  by  subtraction,  we  get 


therefore,  by  substituting  these  values  of  m  and  m'  in  the  above 
analogy,  it  is 

a2  :  a2  —  re2  :  :  /  :  s  ; 

and  from  this,  by  equating  the  products  of  the  extreme  and  mean 
terms,  we  obtain 

s'a?  —  s'#2n=$a2, 
and  by  transposition  we  have 

s'x1  z=a2(s'  —  s)  ; 
therefore,  by  division  and  evolution,  we  obtain 


264 


OF  THE  EQUILIBRIUM  OF  FLOATATION. 


IHCl  A/  f  _ 

V  ' 


and  finally,  by  subtraction,  we  have 


(203). 

323.  The  practical  rule  for  effecting  the  reduction  of  the  above 
equation,  may  be   expressed   in  words   at   length  in   the  following 
manner. 

RULE.  Divide  the  difference  between  the  specific  gravity 
of  the  fluid  and  that  of  the  solid,  by  the  specific  gravity  of 
the  fluid;  then,  from  unity  subtract  the  square  root  of  the 
quotient,  and  multiply  the  remainder  by  the  axis  of  the 
parabola,  and  the  result  will  give  the  height  of  the  frustum 
that  falls  below  the  plane  of  floatation. 

324.  EXAMPLE.   The  axis  of  a  paraboloid  which  floats  in  equilibrio 
on  the  surface  of  a  fluid,  is  29  inches  ;  what  part  of  the  axis  is  im- 
mersed below  the  plane  of  floatation,  supposing  the  body  *to  be  of 
oak,  whose  specific  gravity  is  0.76,  that  of  water  being  unity  ? 

Here,  by  proceeding  as  directed  in  the  rule,  we  get 


v 


1  —  0.76 


)  zz  14.79  inches  very  nearly. 


If  the  vertex  of  the  figure  be  downwards,  as  in  the  annexed  dia- 
gram, then  the  part  of  the  axis  which 
falls  below  the  plane  of  floatation  will 
be  greater  than  it  is  in  the  preceding 
case ;  for  it  is  manifest,  that  since  the 
same  magnitude  or  part  of  the  body 
must  be  immersed  in  both  cases,  it  will 
require  a  greater  portion  of  the  axis, 
towards  the  vertex  of  the  figure,  to 
constitute  that  magnitude,  than  it  would  require  towards  the  base. 

Therefore,  by  retaining  the  foregoing  notation,  we  have,  by  the 
principles  of  mensuration, 

GACBII  — mzz  1.5708pa%  and  ACBZZIW'ZZ  \.57Q8px?; 
consequently,  by  the  seventh  proposition,  we  obtain 

1.5708pa2  :  1.5708;?*2  :  :  s'  :  s ; 

therefore,  by  suppressing  the  common  factors  1.5708p,  and  equating 
the  products  of  the  extreme  and  mean  terms,  we  get 

s'  x"  m  s  a2, 


OF  THE  EQUILIBRIUM  OF  FLOATATION.  265 

and  this,  by  division,  becomes 

*=%> 

and  finally,  by  extracting  the  square  root,  we  get 


(204). 

325.  The  practical  rule  deduced  from  this  equation  is  very  simple  ; 
it  may  be  expressed  in  words  at  length  in  the  following  manner. 

RULE.  Divide  the  specific  gravity  of  the  solid,  by  that  of 
the  fluid  on  which  it  floats  ;  then,  multiply  the  square  root 
of  the  quotient  by  the  axis  of  the  body,  and  the  product  will 
give  the  height  of  the  part  below  the  plane  of  floatation. 

Therefore,  by  retaining  the  data  of  the  foregoing  example,  we  shall 
obtain  as  under, 

c  i  —  x  =  29  V0.76  =  25.251  inches; 

being  a  difference  of  10.46  inches,  in  the  depths  of  immersion,  for  the 
two  cases  of  the  question. 

PROBLEM  XLVII. 

326.  When  a  body  floats  in  equilibrio  on  the  surface  of  a 
homogeneous  fluid,  it  is  a  necessary  condition,  that  the  centre 
of  gravity  of  the  body,  and  that  of  the  fluid  displaced,  shall  be 
in  the  same  vertical  line.     Supposing,  therefore,  that  the  equili- 
brium is  disturbed  by  the  addition  or  subtraction  of  a  certain 
weight  :  — 

It  is  required  to  determine,  how  far  the  body  will  be 
depressed  or  elevated  in  consequence  of  the  extraneous  weight? 

The  above  problem  will  obviously  admit  of  two  cases,  for  a  weight 
may  be  added  to  a  body,  and  it  may  be  subtracted  from  it  ;  in  the 
one  case  the  body  will  descend,  and  in  the  other  it  will  ascend  ;  the 
following  general  solution,  however,  wiH 
answer  both  cases. 

Let  AFBC  represent  a  vertical  section 
of  the  solid  body,  floating  in  a  state  of 
equilibrium  on  a  fluid  of  which  the  hori- 
zontal surface  is  K  L  ;  and  suppose,  that 
when  the  body  is  acted  on  by  its  own 
weight  only,  the  straight  line  D  E  is  coin- 


266  OF  THE  EQUILIBRIUM  OF  FLOATATION. 

cident  with  the  surface  of  the  fluid  ;  but  in  consequence  of  the  addi- 
tional weight  abed,  the  body  descends  through  the  space  GH,  where 
it  again  attains  a  state  of  quiescence,  and  the  plane  of  floatation 
mounts  to  AB. 

Now,  it  is  manifest,  that  when  the  body  is  acted  on  by  means  of  its 
own  weight  only,  (in  which  case,  DE  is  coincident  with  the  surface  of 
the  fluid,)  the  weight  of  the  whole  body  is  equivalent  to  that  of  a 
quantity  of  the  fluid,  whose  magnitude  is  DCE;  but  when  the  weight 
abed  is  applied,  the  compound  weight  is  equivalent  to  that  of  a 
quantity  of  the  fluid,  whose  magnitude  is  ACB;  consequently,  the 
subsidiary  weight  abed,  and  the  weight  of  a  quantity  of  the  fluid, 
whose  magnitude  is  ABED,  are  equal  to  one  another. 

Draw  the  -straight  line  ef  parallel  and  indefinitely  near  to  DE; 
then  is  DE/e,  the  small  elementary  increase  of  the  immersed  portion 
of  the  body,  corresponding  to  any  indefinitely  small  increase  of  the 
weight  abed. 

Put  a  •=.  the  area  of  the  horizontal  section  passing  through  AB, 
determinable   from  the  position  and  the  figure  of  the 
body,  before  the  weight  abed  is  applied, 
n  :z:  the  weight  abed, 

rv  nr  the  fluxion  or  small  elementary  variation  of  w  ; 
x  —  G  H,  the  distance  through  which  the  body  passes  in  con- 

sequence of  the  weight  n  being  applied. 
x  zz  the  fluxion  or  elementary  variation  of  x,  corresponding 

to  w,  and 
s  =:  the  specific  gravity  of  the  fluid. 

Then,  because  the  line  ef  is  supposed  to  be  indefinitely  near  to  D  E, 
it  follows,  that  the  portion  of  the  body  whose  section  is  D  E/e,  may  be 
considered  as  uniform  in  area  between  its  bases,  and  consequently,  its 
magnitude  is  expressed  by  ax  ;  but  DE/C,  is  equal  to  the  quantity  of 
fluid  displaced  by  reason  of  the  elementary  weight  TV,  and  it  is  a  well 
attested  principle  in  hydrostatics,  that  the  weight  of  the  quantity  of 
fluid  displaced,  and  that  of  the  body  which  displaces  it,  are  equal  to 
one  another  ;  therefore  we  have 


and  the  aggregate  of  the  small  elementary  weights,  or  the  whole  weight 
added,  is 

?y  =  /as*.  (205). 

This  is  the  general   form  of  the  equation  of  equilibrium  ;  but  it 
admits  of  various  modifications,  according  to  the  conditions  of  the 


OF  THE  EQUILIBRIUM  OF  FLOATATION.  267 

question  and  the  nature  of  the  body.  For  instance,  if  the  body  be  a 
solid  of  revolution,  and  r  the  radius  of  the  section  coincident  with  the 
plane  of  floatation  ;  then,  the  above  equation  becomes 

w^nrsfax,  (206). 

where  the  symbol  TT  denotes  the  number  3. 1416,  or  four  times  the  area 
of  a  circle  whose  diameter  is  expressed  by  unity. 

327.  The  solution  of  the  problem,  however,  may  be  effected  inde- 
pendently of  the  fluxional  analysis,  especially  in  all  cases  where  the 
floating  body  is  symmetrical  with  respect  to  its  axis ;  for  if  it  be  in 
the  form  of  a  right  cylinder  with  its  axis  vertical,  as  in  the  annexed 
diagram;  then,  the  solution  becomes  an  object  of  the  greatest 
simplicity ;  for  since  the  area  of  the  horizontal  section  is  constant, 
the  space  through  which  the  body  moves  will  be  the  same,  whether 
the  weight  be  added  to  it  or  subtracted  from  it. 

Let  A  B  c  D  be  a  vertical  section  of  a 
cylinder,  floating  in  equilibrio  on  a  fluid 
whose  surface  is  GH,  the  axis  mn  being 
perpendicular  to  the  horizon,  and  sup- 
pose the  weight  n  to  be  placed  on  the 
upper  end  of  the  cylinder  ;  it  is  obvious 
that  the  equilibrium  will  then  be  de- 
stroyed, and  the  body  will  continue  to 
descend,  until  it  has  displaced  a  quan- 
tity of  the  fluid,  whose  weight  is  equal 

to  that  of  the  compound  mass,  consisting  of  the  cylinder,  together  with 
the  applied  body  whose  weight  is  TV;  or  it  will  continue  to  descend,  until 
the  weight  of  the  fluid  displaced  by  the  space  IKFE  is  equal  to  n  the 
weight  of  the  applied  body ;  in  which  case,  the  equilibrium  will  again 
obtain,  and  the  plane  of  floatation,  which  originally  cut  the  cylinder 
in  E  F,  will  now  be  transferred  to  i  K. 

Again,  on  the  other  hand,  if  a,weight  rv  be  subtracted  from  the 
cylinder,  supposed  to  be  in  a  state  of  equilibrium  with  the  plane  of 
floatation  passing  through  EF,  the  body  will  then  ascend,  until  the 
weight  of  the  fluid  which  rushes  into  its  place  becomes  equal  to  the 
weight  subtracted,  in  which  case  the  solid  will  again  be  in  a  state  of 
quiescence  with  the  plane  of  floatation  passing  through  a  b. 

Put  r  ~  id  or  nd,  the  radius  of  the  horizontal  section, 
m  —  the  magnitude  of  the  space  IKFE  or  EF#a, 
x  —  de  or  ec,  the  space  through  which  the  body  is  depressed 
or  elevated  in  consequence  of  the  extraneous  weight, 


268  OF  THE  EQUILIBRIUM  OF  FLOATATION. 

w  zr  the  weight  which   is  added  to  or  subtracted  from  the 

cylinder,  and 

5   z=  the  specific  gravity  of  the  fluid. 

Then,  by  the  principles  of  mensuration,  the  solidity  of  the  cylinder, 
of  which  the  section  is  IKFE  or  EF&O,  becomes 


and  the  weight  of  an  equal  magnitude  of  the  fluid,  is 

m  s  =  3.l4l6r*sx', 

but  this,  by  the   nature  of  equilibrium,  is  equal  to  the  disturbing 
weight  ;  hence  we  have 

w=3.Ul6r*sx, 
and  from  this,  by  division,  we  obtain 


~3.1416rV  (207). 

328.  The  practical  rule  for  reducing  this  equation  is  very  simple ; 
it  may  be  expressed  in  words  at  length  in  the  following  manner. 

RULE.  Divide  the  given  disturbing  weight,  whether  it  be 
added  to  or  subtracted  from  the  cylinder,  by  the  area  of  the 
horizontal  section,  drawn  into  the  speci/ic  gravity  of  the  fluid, 
and  the  quotient  mill  express  the  quantity  of  descent  or  ascent 
accordingly. 

329.  EXAMPLE.  A  cylinder  of  wood,  whose  diameter  is  24  inches, 
floats  in  equilibrio  with  its  axis  vertical,  on  the  surface  of  a  fluid 
whose  specific  gravity  is  expressed  by  unity ;    now,  supposing  the 
equilibrium  to  be  destroyed  by  the  addition  or  subtraction  of  another 
body,  of  which  the  weight  is  56  Ibs. ;  through  what  space  will  the 
body  move  before  the  equilibrium  be  restored  ? 

Here,  by  proceeding  as  directed  in  the  rule,  we  have 

_  56 

a'~~3.1416xl22X.03617*~~3'42 

Again,  if  the  body  should  be  in  the 
form  of  a  paraboloid,  floating  in  equi- 
librio on  the  surface  of  a  fluid  with  its  ~~ 
vertex  downwards,  as  represented  in  the  xj 
annexed  diagram,  where  ACB  is  a  ver- 
tical section  passing  along  the  axis  CD, 
and  GH   the   surface  of  the   fluid  on 

*  The  decimal  fraction  0.03617  expresses  the  weight  in  Ibs.  of  a  cubic  inch  of  water. 


OF  THE  EQUILIBRIUM  OF  FLOATATION.  269 

which  the  body  floats,  with  the  plane  of  floatation  originally  passing 
through  EF,  but  which,  on  the  addition  or  subtraction  of  the  weight  rv, 
ascends  to  i  K  or  descends  to  a  b. 

Put  p  zr  the  parameter  of  the  generating  curve, 

S  ~  ec,  the  distance  of  the  vertex  below  the  surface  of  the 

fluid  at  first, 
m  ±r  the  magnitude  or  solidity  of  the  paraboloidal  frustum,  of 

which  IKFE  is  a  section, 

#/:=r  the  magnitude  of  the  frustum  whose  section  is  EF^a  ; 
x  rn  de,  the  descent  of  the  body  in  consequence  of  the  addi- 

tion of  the  weight  rv, 

x'  rr  ec,  the  corresponding  ascent  in  the  case  of  subtraction, 
and  s  rzi  the  specific  gravity  of  the  fluid. 

Then  we  have  crf  =  5  -j-  x,  and  c  c  rr  8  —  otf,  and  according  to  the 
writers  on  mensuration,  we  have 


and  in  a  similar  manner,  we  obtain 

m'  =  1.5708p(28*f  —  *'2)  ; 

and  since  the  weight  of  any  body  is  expressed  by  its  magnitude  drawn 
into  its  specific  gravity,  it  follows,  that  the  weight  of  a  quantity  of 
fluid  equal  respectively  to  m  and  m1  ,  are 

THSIZ:  1.5708;?s(23;r  +  x2),  andm's—  1.5708ps(fcc'—  x'3). 
Now,  these  according  to  the  conditions  of  the  problem,  are  respec- 
tively equal  to  the  disturbing  weight  ;  hence  we  have  in  the  case  of 
addition, 

m  =  1  .57Q8ps(Z$x  +  *2),  (208). 

and  in  the  case  of  subtraction,  it  is 

tv=  1.5708^5(2^'—  *'8).  (209). 

330.  The  equations  which  we  have  just  obtained,  are  precisely  the 
same  as  would  arise,  by  taking  the  fluent  of  the  expression  in  equation 
(206)  ;  it  therefore  appears,  that  although  the  fluxional  notation  is  the 
most  convenient  for  expressing  the  general  result,  yet  in  point  of  sim- 
plicity as  regards  symmetrical  bodies,  there  is  little  advantage  to  be 
derived  from  its  adoption. 

Suppose  that  in  the  first  instance,  the  equilibrium  is  destroyed  by 
the  addition  of  the  weight  rv,  and  let  it  be  required  to  determine  how 
far  the  body  will  descend  in  consequence  of  the  addition. 

Equation  (208)  involves  this  condition  ;  consequently,  if  both  sides 
be  divided  by  the  expression  1  .5708^5,  we  shall  obtain 


270  OF  THE  EQUILIBRIUM  OF  FLOATATION. 


which  being  reduced,  gives 

x  —  1/8"H —  8-  /oim 

Y  1.5708j9s  (210). 

331.  And   the  practical  rule  for  reducing  this  equation,  may  be 
expressed  in  words  at  length,  as  follows. 

RULE.  Divide  the  weight  which  disturbs  the  equilibrium, 
by  1.5708  times  the  parameter  of  the  generating  parabola, 
drawn  into  the  specific  gravity  of  the  fluid,  and  to  the  quotient 
add  the  square  of  the  distance  between  the  vertex  of  the  figure 
and  the  plane  of  floatation  in  the  first  position  of  equilibrium ; 
then,  from  the  square  root  of  the  sum,  subtract  the  said  dis- 
tance, and  the  remainder  will  express  the  quantity  of  descent. 

332.  EXAMPLE.  A  solid  body  in  the  form  of  a  paraboloid,  floats  on 
a  vessel  of  water  in  a  state  of  equilibrium  with  its  vertex  downwards, 
when  12  inches  of  the  axis  are  immersed  below  the  plane  of  floata- 
tion ;  how  much  farther  will  the  body  sink,  supposing  a  weight  of 
28  Ibs.  to  be  laid  on  its  base,  the  parameter  of  the  generating  parabola 
being  16  inches? 

Here,  by  pursuing  the  directions  of  the  rule,  we  get, 


90 

122H — —  12  =z  1.22  inches. 

1.5708X16X. 03617 

333.  If  the  weight  w  should  be  subtracted  from  the  paraboloid 
instead  of  being  added  to  it,  the  quantity  of  ascent  will  then  be  deter- 
mined by  equation  (209),  where  we  have 


divide  both  sides  of  this  equation  by  the  quantity  1.5708p5,  and  we 
shall  obtain 


1.5708psJ 
which,  by  transposing  the  terms,  becomes 


By  completing  the  square,  we  get 


Or  THE  EQUILIBRIUM  OF  FLOATATION.  271 

and  finally,  by  extracting  the  square  root  and  transposing,  it  is 

'  =  a      V    a'-  (211). 


334.  And  the  practical  rule  by  which  this  equation  is  reduced,  may 
be  expressed  in  words  at  length,  in  the  following  manner. 

RULE.  Divide  the  weight  which  is  proposed  to  be  subtracted 
from  the  paraboloid,  by  1.5708  times  the  parameter  of  the 
generating  parabola  drawn  into  the  specific  gravity  of  the 
fluid,  and  subtract  the  quotient  from  the  square  of  the  dis- 
tance of  the  vertex  below  the  plane  of  floatation  in  theflrst 
position  of  equilibrium  ;  then,  from  the  said  distance  ,  subtract 
the  square  root  of  the  remainder  for  the  quantity  of  ascent 
required. 

335.  If  we  take  the  data  of  the  foregoing  example,  and  proceed 
according  to  the  directions  of  the  rule,  we  shall  obtain 

x1  =  12  —  4  /  144  __  _  ??  _  —1.35  inches, 
y  1.5708X16X.03617 

exceeding  the  descent  in  the  former  example,  by  0.13  of  an  inch. 

Numerous  other  examples  akin  to  the  above,  respecting  bodies  of 
various  forms,  and  placed  in  different  positions,  might  here  be  pro- 
posed ;  but  since  they  are  all  resolvable  by  the  general  formula, 
equation  (206),  we  have  thought  proper  to  omit  them. 

PROBLEM  XLVIII. 

336.  If  a  body  which  is  symmetrical  with  respect  to  its  verti- 
cal axis,  floats  upon  a  fluid  in  a  state  of  equilibrium  :  — 

It  is  required  to  determine  what  weight  must  be  placed 
upon  the  body,  so  that  it  shall  sink  to  a  level  with  the  surface 
of  the  fluid,  the  specific  gravities  of  the  solid  and  the  fluid, 
together  with  the  magnitude  of  the  solid,  being  given. 

In  the  second  example  to  the  seventh  proposition,  we  have  advanced 
principles  of  nearly  a  similar  import  to  those  required  for  the  solution 
of  the  present  problem,  yet  nevertheless,  we  think  a  separate  solution 
in  the  present  instance  is  necessary,  since  it  can  be  somewhat  differ- 
ently represented  ;  for  which  purpose, 

Put  m  ~  the  entire  magnitude  of  the  floating  body, 

w  —  its  weight  before  the  external  body  or  weight  is  super- 
added, 


272 


OF  THE  EQUILIBRIUM  OF  FLOATATION. 


rv'  rr  the  added  weight, 
n>"  —  the  weight  of  the  fluid  displaced, 
s    =z  the  specific  gravity  of  the  fluid,  and 
s'  =  the  specific  gravity  of  the  floating  solid. 

Then,  because  the  absolute  weight  of  any  body,  is  expressed  by  its 
magnitude  drawn  into  its  specific  gravity ;  it  follows,  that  the  weight 
of  the  floating  solid,  is 

w  —  ms', 

and  in  like  manner,  the  weight  of  the  displaced  fluid,  is 
n"  —  m  s ; 

now,  it  is  manifest,  from  the  nature  of  the  problem,  that  the  weight 
of  the  displaced  fluid  is  equal  to  the  weight  of  the  floating  body, 
together  with  the  superadded  weight;  consequently,  we  have 

«/ -J-  rv  ~w' -f-  w/™wis  ; 
from  which,  by  transposition,  we  obtain 

w'=ffi(*  —  «').  ..  (212). 

337.  The  practical  rule  for  the  reduction  of  this  equation  is  very 
simple  :  it  may  be  expressed  as  follows. 

RULE.  Multiply  the  difference  between  the  specific  gravities 
of  the  fluid  and  the  floating  solid,  by  the  whole  magnitude  of 
the  floating  bodyt  and  the  product  will  express  the  value  of 
the  added  iveight. 

338.  EXAMPLE.  A  mass  of  oak,  whose  specific  gravity  is  .872,  that 
of  water  being  unity,  floats  in  equilibrio  on  the  surface  of  a  fluid 
whose  specific  gravity  is  1.038  ;  what  weight  applied  externally  to  the 
floating  body,  will  depress  it  to  the  level  of  the  fluid  surface,  sup- 
posing the  magnitude  of  the  body  to  be  equal  to  8  cubic  feet  ? 

Here,  by  operating  as  the  rule  directs,  we  shall  have 
w'—  8(1.038  —  .872)  —  1.328  cubic  feet  of  water ; 

but  it  is  a  well  known  fact,  that  one  cubic  foot  of  water  weighs  62 £ 
Ibs.  avoirdupois,  very  nearly ;  consequently,  we  have 

wf=1.328X62J  =  831bs. 

PROPOSITION  VIII. 

339.  If  a  solid  body,  which  is  specifically  heavier  than  one  of 
two  fluids  which  do  not  mix,  and  specifically  lighter  than  the 
other,  be  immersed  in  the  fluids : — 


/  UNIVERSITY 


V  Of  .y±S 

OF  TIIR  EQUILIBRIUM  OF  FLOATATION.  273 

It  will  float  in  equilibria  between  them,  when  the  weight 
of  the  fiuids  respectively  displaced,  are  together  equal  to  the 
weight  of  the  solid  body  which  causes  the  displacement ;  the 
specific  gravities  of  the  fluid  being  supposed  known. 

Let  A  BCD  be  a  vertical  section  passing  through  the  centre  of  gravity 
of  the  floating  body,  and  suppose  that 
IK  is  the  common  surface  of  the  two 
fluids,  in  which  the  solid  is  quiescent, 
GH  being  the  surface  of  the  lighter  fluid. 

Now,  it  is  manifest,  that  since  the 
body  ABCD  is  specifically  heavier  than 
one  of  the  fluids,  and  specifically  lighter 
than  the  other,  it  cannot  be  entirely  at 

rest  in  either,  but  must  rest  between  them  in  such  a  position,  that  the 
sum  of  the  weights  of  the  fluids  displaced  shall  be  equal  to  the  whole 
weight  of  the  solid. 

Let  EFD  be  perpendicular  to  i  K,  the  common  surface  of  the  fluids 
in  which  the  body  floats ;  then  it  is  evident,  that  the  pressure  down- 
ward on  any  point  of  the  base  D,  is  equal  to  the  weight  of  the  incum- 
bent line  of  solid  particles,  whose  altitude  is  BD  the  thickness  of  the 
body,  together  with  the  weight  of  EB  the  superincumbent  column  of 
trie  lighter  fluid  ;  and  again,  the  pressure  upwards  on  the  same  point 
D,  is  equal  to  the  weight  of  a  column  of  the  heavier  fluid  whose  alti- 
tude FD,  together  with  the  weight  of  a  column  of  the  lighter  fluid, 
whose  altitude  is  EF. 

Put  d  —  EB,  the  depth  of  the  body  below  the  upper  surface  of  the 

lighter  fluid, 
d'  —  EF,  the  whole  depth  of  the  lighter  fluid,  or  the  depth  of 

the  common  surface, 
•3   —  FD,  the  depth  of  the  body  below  the  common  surface,  or 

the  surface1  of  the  heavier  fluid, 
$  =  BD,  the  whole  thickness  of  the  solid  body, 
5  =  the  specific  gravity  of  the  lighter  fluid, 
s'  zz  the  specific  gravity  of  the  heavier  fluid, 
s"  =  the  specific  gravity  of  the  solid  body, 
p  rzr  the  downward  pressure,  and 
p'  ~  the  upward  pressure. 

Then,  because  the  weight  of  any  body,  whether  it  be  fluid  or  solid, 
is  expressed  by  the  product  of  its  magnitude  drawn  into  its  specific 
gravity,  it  follows  that  the  downward  pressure  on  the  point  D,  is 

VOL.  I.  T 


274  OF  THE  EQUILIBRIUM  OF  FLOATATION. 

p  i=  2'  s"  -|-  ds, 

and  in  like  manner,  the  pressure  upwards,  is 
p'  =  Ss'  -\-d's. 

But  when  the  body  floats  in  a  state  of  equilibrium,  the  upward  and 
the  downward  pressures  are  equal  to  one  another  ;  hence  we  have 


from  which,  by  transposing  and  collecting  the  terms,  we  get 

SY'zzas'-Kd'—  d)s. 

Now,  it  is  obvious,  that  what  we  have  demonstrated  above  with 
respect  to  the  point  D,  may  also  be  demonstrated  to  hold  with 
respect  to  every  other  point  of  the  surface  whose  section  is  ADC; 
consequently,  by  taking  the  aggregate  of  the  upward  and  downward 
pressures,  we  obtain 

&c.)«"=(3  +  S  +  3+  &c.)s'  +  {(d'  +  ef+  d'-f&c.) 


Put  w  zz  (o;  -f-  S'  •+•  2'  -j-  &c.)  to  infinity,  equal  to  the  magnitude 

of  the  entire  floating  body, 
m  —  (3  +  I  4-  3  4-  &C0  to  infinity,  equal  to  the  part  immersed 

in  the  heavier  fluid,  and 
m"=  {(d'  4-  d'  4-  d'  4-  &c.)  —  (d  -f  d  4-  d  +  &c.)}  to  infinity, 

equal  to  the  part  immersed  in  the  lighter  fluid  ;  conse- 

quently, by  substitution,  we  get 

ms"  =  m's'  +  m"s.  (213). 

This  equation  involves  the  principle  announced  in  the  Proposition, 
and  its  application  to  practical  cases  will  be  exemplified  in  the  reso- 
lution of  the  following  Problems. 

PROBLEM  XLIX. 

340.  Suppose  that  a  solid  body  in  the  form  of  a  regular  cube, 
is  observed  to  float  in  equilibrio  between  two  unmixable  fluids 
of  different  specific  gravities  :  — 

It  is  required  to  determine,  how  much  of  each  fluid  is  dis- 
placed by  the  body,  the  specific  gravities  of  the  body  and  the 
fluid  being  given. 

Let  A  BCD  be  a  cubical  body,  floating  in  equilibrio  between  the 
two  unmixable  fluids,  whose  upper  horizontal  surfaces  are  respec- 


OF  THE  EQUILIBRIUM  OF   FLOATATION.  275 

tively  IK  and  GH,  GH  being  the  surface 
of  contact  of  the  two  fluids. 


Put  m  =:  A  BCD,  the  magnitude  of  the 

whole  body, 

x  m  ABCFE,  the  magnitude  of  the 
part  immersed  in  the  lighter 
fluid,  and 
m  —  a;  rr  EFC  D,  the  magnitude  of  the  part  immersed  in  the  heavier 

fluid. 

Therefore,  if  the  specific  gravities  of  the  body  and  the  fluids  be 
respectively  denoted  by  s",  s  and  s',  as  in  the  Proposition;  then,  we 
shall  have 

ms"=:xs-\-  (m  —  x)s', 

from  which,  by  transposition,  we  obtain 

x  (s'  —  5)  zz  m(s'  —  s"), 
and  finally,  by  division,  it  becomes 


_ 

'' 


341.  An  equation  of  an  extremely  simple  and  convenient  form,  from 
which  we  deduce  the  following  practical  rule. 

RULE.  Multiply  the  magnitude  of  the  immersed  solid  by 
the  difference  between  its  specific  gravity  and  that  of  the 
heavier  Jluid;  then  divide  the  product  by  the  difference  between 
the  specific  gravities  of  the  fluids,  and  the  quotient  will  give 
the  magnitude  of  the  part  immersed  in  the  lighter  Jluid. 

342.  EXAMPLE.  A  cubical  piece  of  oak  containing  2000  inches, 
and  whose  specific  gravity  is  0,872,  that  of  water  being  unity,  floats 
in  equilibrio  between  two  fluids,  whose  specific  gravities  are  respec- 
tively 1.24  and  0.716  ;  what  portion  of  the  solid  is  immersed  in  each 
of  the  fluids,  supposing  them  to  be  altogether  unmixable  ? 

The  operation  being  performed  according  to  the  rule,  will  stand  as 
below. 

2000(1.24  —  0.872) 

(1.^4-0.716)-  14°4'58  °ublC  mches' 

This  result  expresses  the  solid  contents  of  that  part  of  the  body 
which  is  immersed  in  the  lighter  fluid  ;  consequently,  the  part  which 
is  immersed  in  the  heavier  fluid,  is 

2000  —  1404.58  =  595,42  cubic  inches. 
T  2 


276  OF  THE  EQUILIBRIUM   OF  FLOATATION. 

343.  In  this  example,  a  greater  portion  of  the  body  is  immersed  in 
the  lighter  fluid  than  what  is  immersed  in  the  heavier ;  but  this  cir- 
cumstance manifestly  depends  upon  the  nature  of  the  immersed  body, 
and  the  relation  of  the  specific  gravities ;  for  an  instance  may  readily 
be  adduced,  in  which  exactly  the  reverse  conditions  will  obtain  : — 
Thus,  let  the  magnitude  of  the  body  and  the  specific  gravities  of  the 
fluids  remain  as  above,  and  suppose  the  specific  gravity  of  the  body 
to  be  1.17  instead  of  0.872 ;  what  then  are  the  parts  immersed  in  the 
respective  fluids  ? 

The  numerical  process  is  represented  as  below. 

2000(1.24—1.17) 
*  =  -        (1.24-  0.716)  =  267.175  cub.c.nches. 

Here  then,  we  have  267.175  cubic  inches  for  the  portion  which  is 
immersed  in  the  lighter  fluid,  while  that  immersed  in  the  heavier  is 
2000-267.175~1732.825  cubic  inches;  this  agrees  with  the  case 
represented  in  the  diagram,  for  there  the  body  displaces  a  greater 
quantity  of  the  heavier  than  it  does  of  the  lighter  fluid. 

PROBLEM  L. 

344.  Having  given  the   specific  gravities  of  two  unmixable 
fluids,  and  the  magnitude  of  a  solid  body  which  floats  in  equili- 
brio  between  them : — 

It  is  required  to  determine  the  specific  gravity  of  the  solid, 
so  that  any  proposed  part  of  it  may  be  immersed  in  the 
lighter  fluid. 

Put  m  zr  the  magnitude  of  the  immersed  solid,  the  same  as  in  the 

preceding  Problem, 

s  ~  the  specific  gravity  of  the  lighter  fluid, 
s'  —  the  specific  gravity  of  the  heavier  fluid, 
x  =  the  specific  gravity  of  the  solid  body,  being  the  required 

quantity,  and 
n  n=  the  denominator  of  the  fraction  which  expresses  the  part 

of  the  body  immersed  in  the  lighter  fluid. 

Then,  according  to  the  principle  demonstrated  in  Proposition  VIII., 
we  shall  obtain 

ms   ,   (m    — m)s' 

mx= \-- — , 

n  n 

and  from  this,  by  a  little  farther  reduction,  we  get 
mnxmmns'  — m(sr  —  s)  ; 


OF  THE  EQUILIBRIUM  OF  FLOATATION.  277 

consequently,  by  division,  we  obtain 

_mns'  —  m(s' — s)  * 

mn  (215). 

345.  And  from  this  equation  we  deduce  the  following  rule. 

RULE.  Multiply  together,  the  magnitude  of  the  body,  the 
number  which  expresses  what  part  of  it  is  immersed  in  the 
lighter,  and  the  specific  gravity  of  the  heavier  fluid ;  then, 
from  the  product  subtract  the  difference  between  the  specific 
gravities  of  the  fluid  drawn  into  the  magnitude  of  the  solid 
body,  and  divide  the  remainder  by  the  magnitude  of  the  bodyt 
multiplied  by  the  number  which  expresses  what  part  of  it  is 
immersed  in  the  lighter  fluid ;  then  shall  the  quotient  express 
the  specific  gravity  of  the  body. 

346.  EXAMPLE.  The  specific  gravities  of  two  unrnixable  fluids  are 
respectively  1.24  and  0.716,  that  of  water  being  unity  ;  now,  sup- 
posing that  when  these  fluids  are  poured  into  the  same  vessel,  a  body 
of  2000  cubic  inches  which  is  in  equilibrio  between  them,  has  one 
seventh  part  of  its  magnitude  immersed  in  the  lighter  fluid  ;  what  is 
the  specific  gravity  of  the  body  ? 

Here,  by  proceeding  according  to  the  rule,  we  have 

mns/  =  2000x7Xl.24  — 17360 
—  7M(5'—s):=  2000  (1.24  —  0.716)  zz  —  1048  subtract 

difference  zr  163 12; 
consequently,  by  division,  we  shall  obtain 

mns'  —  m(s'  —  s)         16312 

x  — s =  r—-— — -  zz:  1.17  nearly. 

mn  2000X7  J 

347.  From  what  has  been  done  above,  it  is  easy  to  ascertain  what 
will  be  the  specific  gravity  of  the  body,  when  equal  portions  of  it  are 
immersed  in  the  lighter  and  in  the  heavier  fluids ;  for  in  that  case,  we 
have  n  equal  to  2,  which  being  substituted  in  equation  (215),  gives 

*~s'—  J(*'—  s).  (216). 

And  the  practical  rule  for  reducing  this  equation  may  be  expressed 
in  words  at  length,  in  the  following  manner. 


*  This  equation  is  susceptible  of  a  simpler  form,  for  by  casting  out  the  common 
factor  m,  it  is 


278  OF  THE  EQUILIBRIUM  OF  FLOATATION. 

RULE.  From  the  specific  gravity  of  the  heavier  fluid,  subtract 
half  the  difference  between  the  given  specific  gravities,  and  the 
remainder  will  be  the  specific  gravity  of  the  solid  body. 

348.  EXAMPLE.  Let  the  specific  gravities  of  the  fluids  remain  as 
above ;  what  must  be  the  specific  gravity  of  the  body,  so,  that  when 
it  is  in  a  state  of  equilibrium,  one  half  of  it  may  be  immersed  in  each 
solid  ? 

One  half  the  difference  of  the  given  specific  gravities,  is 

£(1.24  —  0.716)  =0.262; 
consequently,  by  subtraction,  we  have 

a- =1.24 — 0.262  =  0.978,  and  with  this  specific 
gravity,  a  body,  whatever  may  be  its  magnitude,  will  be  equally 
immersed  in  the  two  unmixable  fluids. 

349.  If  the  specific  gravity  of  the  lighter  fluid  vanish,  or  become 
equal  to  nothing;  then  equation  (215)  becomes 

and  by  converting  this  equation  into  an  analogy,  we  get 
m  :  m'  :  :  s1  :  s". 

This  analogy  expresses  the  identical  principle,  which  is  announced 
and  demonstrated  in  Proposition  VII.  preceding ;  it  is  therefore  pre- 
sumed, that  the  examples  already  given  will  be  found  sufficient  to 
illustrate  the  application  of  this  very  elegant  and  important  property. 
Since  the  magnitude  of  the  whole  floating  body  is  equal  to  the  sum 
of  its  constituent  parts,  it  follows,  that  according  to  our  notation, 

m  =  m'  4~  m"  '* 
consequently,  by  substitution,  equation  (215)  becomes 

(m  4-  m") s"  =  m' s'  4-  m"  s, 
or  by  transposing  and  collecting  the  terms,  we  get 

m«  («"-«)  =  »»'(*-*"), 
and  by  converting  this  equation  into  an  analogy,  we  obtain 

m  :  m"  ::(s"  —  s)  :  (s'—s»). 

By  comparing  the  terms  of  the  proportion  as  they  now  stand,  it 
will  readily  appear,  that  if  the  specific  gravity  of  the  lighter  fluid  be 
increased,  the  term  (s"  —  s)  is  diminished,  while  (s'  —  s")  remains  the 
same ;  consequently,  the  first  term  ni  will  be  diminished  with  respect 
to  the  second  term  m" ;  which  implies,  that  the  part  of  the  body  in 
the  lighter  fluid  will  be  increased ;  hence  arises  the  following  very 
curious  property,  that 


OF  THE  EQUILIBRIUM  OF  FLOATATION.  279 

If  any  body  float  upon  the  surface  of  a  fluid  in  vacuo,  and 
air  be  admitted,  the  body  will  ascend  higher  above  the  surface, 
and  consequently,  the  proportion  of  the  immersed  part  to  the 
whole  will  be  diminished. 

PROBLEM  LI. 

350.  Suppose  a  solid  body  to  float  in  equilibrio  on  the  surface 
of  water,  both  in  air  and  in  a  vacuum  :  — 

It  is  required  to  determine  the  ratio  of  the  parts  immersed 
in  the  water  in  both  cases. 

Put  m  zz:  the  magnitude  of  the  whole  floating  body, 

m'  zr  the  magnitude  of  the  part  immersed  below  the  surface 

of  the  water,  when  the  incumbent  fluid  is  air, 
m"—  the  portion  immersed  when  the  body  floats  on  water  in 

vacuo, 

s   zz:  the  specific  gravity  of  air, 
s'  zz:  the  specific  gravity  of  water,  and 
s"  zz:  the  specific  gravity  of  the  floating  body. 
Then  we  have  m  —  m',  for  the  part  above  the  surface  of  the  water, 
when  the  incumbent  fluid  is  air,  and  m  —  m"  for  the  extant  part  when 
the  body  floats  in  vacuo  ;  consequently,  by  equation  (213),  we  have, 
when  the  body  floats  in  air, 

ms"  zz:  m'  s'  -\-(m  —  ra')  s, 
from  which,  by  a  little  reduction,  we  obtain 

m'  (s'  —  s)  zz  m  (s"  —  s), 
and  finally,  by  division,  it  becomes 

,_*»(*"-*) 

-    (s'-s)    '  (217). 

and  again,  when  the  body  floats  in  vacuo,  we  have 


but  in  this  case,  s  vanishes,  hence  we  get 

ms"  =  m"s, 
and  by  division,  it  is 
_ms" 

(218). 

Let  the  equations  (217)  and  (218)  be  compared  with  one  another  in 
the  terms  of  an  analogy,  and  we  shall  have 

'  -  m(s''—s)  .  .      a  .  ms"  . 

'    (S'  —  5)  S' 


280 


OF  THE  EQUILIBRIUM  OF  FLOATATION. 


therefore,  by  equating  the  products  of  the  extreme  and  mean  terms 
and  casting  out  the  common  quantity  m,  we  obtain 
m"(s"—s)  _  m's" 

s'  —  sY~      :~7"' 
by  clearing  the  equation  of  fractions,  we  get 

m"s'(s"  —  s)  =  nf8l!(S  —  s), 
and  finally,  by  division,  we  have 


«'(«"  —  «)  (219). 

351.  Now,  it  is  manifest,  that  in  order  to  determine  from  this 
equation,  what  part  of  the  body  is  immersed  in  the  water  when  it 
floats  in  vacuo,  it  is  necessary  in  the  first  place,  to  ascertain  how 
much  of  it  is  immersed  when  the  floatation  occurs  in  air : — Equation 
(217)  determines  this,  and  the  practical  rule  deduced  from  the  equa- 
tions (217)  and  (219)  may  be  expressed  in  words  at  length  in  the 
following  manner. 

RULE.  From  the  specific  gravity  of  the  floating  body,  sub- 
tract the  specific  gravity  of  air ;  multiply  the  remainder  by 
the  magnitude  of  the  body,  and  divide  the  product  by  the 
difference  between  the  specific  gravities  of  water  and  air,  for 
the  part  which  is  immersed  in  water,  when  the  incumbent  fluid 
is  air. 

Again.  Multiply  the  difference  between  the  specific  gravities 
of  water  and  air  by  the  specific  gravity  of  the  floating  body  ; 
divide  the  product  by  the  difference  between  the  specific  gra- 
vities of  the  solid  body  and  air,  drawn  into  the  specific  gravity 
of  water ;  then,  multiply  the  quotient  by  the  magnitude  of  the 
part  immersed  in  water  when  the  body  floats  in  air,  and  the 
product  will  be  the  magnitude  of  the  part  immersed  in  water, 
when  the  body  floats  in  vacuo. 

352.  EXAMPLE.  A  mass  of  oak  whose  specific  gravity  is  0.925,  con- 
tains 185  cubic  inches;  what  quantity  of  it  exists  below  the  plane  of 
floatation,  supposing  it  to  float  on  water  in  vacuo,  the  specific  gravity 
of  the  air  being  0.0012  at  the  instant  of  observation  ? 

By  operating  according  to  the  directions  given  in  the  first  clause  of 
the  rule,  the  quantity  below  the  plane  of  floatation  when  the  incumbent 
fluid  is  air,  becomes 

185(0.925  —  0.0012) 

m  •=. : -±£  171.108  cubic  inches; 

(1—0.0012) 


OF  THE  EQUILIBRIUM  OF  FLOATATION.  281 

therefore,  according  to  the  second  clause  of  the  rule,  the  part  immersed 
when  the  body  floats  in  vacuo,  becomes 

0.925(1  —  0.0012) 
=  X171-108: 


= 


353.  If  we  refer  to  the  equation  (218)  preceding,  it  will  readily 
appear,  that  the  above  result  may  be  determined  with  much  less  labour 
and  greater  simplicity  ;  for  the  magnitude  of  the  immersed  part,  when 
the  body  floats  in  vacuo,  is  there  expressed  in  terms  of  the  weight  of 
the  body  and  the  specific  gravity  of  water,  and  the  practical  rule  for 
reducing  the  equation,  may  be  expressed  in  words  at  length,  as  follows. 

RULE.  Multiply  the  magnitude  of  the  body  by  its  specific 
gravity  ;  then  divide  the  product  by  the  specific  gravity  of 
water,  and  the  quotient  will  express  the  magnitude  of  the 
immersed  part  when  the  body  floats  in  vacuo. 

Therefore,  by  taking  the  data  as  proposed  in  the  above  example, 
the  magnitude  of  the  immersed  part  becomes 

185X0.925 
m'~  -  -  -  ^  =  171.125  cubic  inches;   being  precisely 

the  same  quantity  as  we  obtained  by  the  foregoing  prolix  operation. 

If  we  compare  the  computed  values  of  m'  and  m"  with  one  another, 
we-  shall  find  that  the  latter  exceeds  the  former  by  a  very  small  quan- 
tity, that  is, 

171.125—  171.  108  =z  0.017, 

which  verifies  the  concluding  inference  under  Problem  L. 

354.  On  the  principles  which  we  have  explained  and  illustrated  in  the 
foregoing  problems,  depends  the  construction  and  application  of  the 
Hydrometer,  an  instrument  which  is  generally  employed  for  detect- 
ing and  measuring  the  properties  and  effects  of  water  and  other  fluids, 
such  as  their  density,  gravity,  force,  and  velocity. 

When  the  hydrometer  is  employed  to  determine  the  specific  gravity 
of  water,  it  is  sometimes  denominated  an  aerometer  or  water-poise  ; 
and  being  an  instrument  of  very  general  utility  in  numerous  philoso- 
phical experiments,  we  think  it  will  not  be  amiss  in  this  place,  to 
discuss  its  nature  and  properties  a  little  in  detail  ;  and  we  may  here 
observe,  that  the  following  problems  and  remarks  are  quite  sufficient 
to  establish  and  exemplify  its  most  important  applications. 

The  hydrometer,  or  aerometer,  in  general  consists  of  a  long  cylin- 
drical stem  of  glass,  or  other  metal,  connected  with  two  hollow  balls, 


282 


OF  THE  EQUILIBRIUM  OF  FLOATATION. 


into  the  lower  of  which  is  introduced  a  small  quantity  of  mercury  or 
leaden  shot,  for  the  purpose  of  preventing  the  instrument  from  over- 
turning, and  causing  it  to  float  steadily  in  a  vertical  position,  or  per- 
pendicularly to  the  surface  of  the  fluid  in  which  it  is  immersed. 

Numerous  schemes  have  been  promulgated  by  different  ingenious 
and  experienced  philosophers  for  the  improvement  of  this  instrument  ; 
but  however  much  the  forms  which  have  been  recommended  may  differ 
among  themselves,  yet  the  general  principle  is  the  same  in  all. 

The  following  is  a  list  of  the  principal  writers  who  have  registered 
their  improvements  in  the  annals  of  science,  viz. 

Desaguliers,    Guyton,  Nicholson,    Speer, 


Adie,          Charles, 

Atkins,       Clark,  Dicas, 

Brewster,   Deparcieux,    Fahrenheit, 


Jone 


Quin, 
Sikes, 


and 
Wilson. 


355.  It  would  be  quite  superfluous  to  detail  the  various  alterations 
and  improvements  suggested  by  these  authors;  suffice  it  to  say,  that  in 
all  there  is  something  different  and  in  all  there  is  something  common  ; 
but  that  which  merits  the  greatest  share  of  our  attention,  by  reason 
of  the  extreme  delicacy  of  its  indications  -and  the  simplicity  of  its 
construction,  is  the  hydrometer  of  Deparcieux,  which  was  presented 
to  the  Academy  of  Sciences  in  the  year  1766. 

This  instrument,  which  was  intended  by  its  inventor  to  measure  the 
specific  gravities  of  different  kinds  of  water,  is 
represented  in  the  annexed  figure,  where  AC  is  a 
glass  phial  about  seven  or  eight  inches  in  length, 
loaded  with  mercury  or  leaden  shot,  to  prevent  it 
from  overturning;  and  in  order  that  no  air  may 
lodge  below  it,  when  it  is  immersed  in  the  fluid,  the 
lower  part  is  rounded  off  into  the  form  of  a  spheric 
segment. 

In  the  cork  of  the  phial  at  A,  is  fixed  a  brass  wire 
of  one  twelfth  of  an  inch  in  diameter,  and  from  thirty  to  thirty  six 
inches  long,  or  of  any  other  length  which  may  be  found  convenient 
for  the  purpose,  but  such,  that  when  the  phial  is  loaded  and  immersed 
in  spring  water  of  a  medium  temperature,  the  entire  phial  and  about 
one  inch  of  the  wire  should  be  below  the  graduated  scale  DH,  which 
is  fixed  upon  the  side  of  the  tin  vessel  DEFG  ;  to  the  other  end,  or 
summit  of  the  wire,  is  attached  a  small  box  B,  intended  for  containing 
the  minute  weights  which  it  may  be  found  necessary  to  apply,  in  order 
to  cause  the  instrument  to  sink  to  a  certain  fixed  point  in  the 


OF  THE  EQUILIBRIUM  OF  FLOATATION.  283 

different  kinds  of  water  whose  specific  gravities  are  required  to  be 
found.* 

The  white  iron  vessel  DEFG,  is  used  for  holding  the  fluid  on  which 
the  experiment  is  to  be  performed ;  it  is  generally  about  three  feet  in 
length,  and  from  three  to  four  inches  in  diameter,  according  to  the 
circumstances  under  which  it  may  happen  to  be  employed.  The  small 
scale  DH,  is  attached  for  the  purpose  of  measuring  the  different  depths 
to  which  the  instrument  sinks  when  differently  loaded,  or  when  it  is 
immersed  in  fluids  of  different  specific  gravities. 

The  indications  of  this  instrument  are  so  extremely  delicate,  that  if 
a  small  quantity  of  alcohol,  or  a  little  common  salt,  be  added  to  the 
fluid,  the  phial  will  ascend  or  descend  through  a  very  sensible  dis- 
tance, which  circumstance  greatly  enhances  the  value  of  the  aerometer; 
for  in  proportion  to  its  sensibility  and  the  delicacy  of  its  indications, 
are  its  importance  and  utility  to  be  appreciated. 

We  come  now  to  consider  the  theory  of  this  instrument,  and  we 
shall  just  remark  in  passing,  that  the  same  principles,  under  very 
slight  and  obvious  modifications,  will  apply  to  any  other  hydrometric 
instrument,  of  a  similar,  or  nearly  similar  nature  and  construction,  to 
that  which  forms  the  subject  of  our  present  discussion. 

PROBLEM  LII. 

356.  Having  given  the  capacity  or  volume  of  the  phial,  toge- 
ther with  the  dimensions  of  the  immersed  wire,  and  the  entire 
weight  of  the  aerometer : — 

It  is  required  to  determine  the  specific  gravity  of  the  fluid, 
in  which  the  instrument  settles  in  a  state  of  equilibrium. 

Now,  because  the  weight  of  any  body  when  floating  in  equilibrio, 
whatever  may  be  its  form  and  the  substance  of  which  it  is  composed, 
is  equal  to  the  weight  of  the  fluid  which  it  displaces ;  it  follows,  that  if 
we  put  c  =:  the  capacity  or  volume  of  the  phial  immersed  in  the  fluid, 
I  zz  the  length  of  the  immersed  wire, 
r  nr  the  radius  of  its  transverse  section, 

TTZZ:  3.1416,  the  number  which  expresses  the  circumference  of 
a  circle  whose  diameter  is  equal  to  unity, 

*  To  the  bottom  of  the  box  B  we  have  affixed  the  arm  a  b,  from  one  extremity  of 
which  is  suspended  the  wire  cd  carrying  the  index  i,  the  whole  being  truly  balanced 
by  the  small  ball  b  attached  to  the  other  extremity  of  the  horizontal  arm  ab.  In  all 
other  respects  the  instrument  is  that  of  Deparcieux. 


284  OF  THE  EQUILIBRIUM  OF  FLOATATION. 

s  m  the  specific  gravity  of  the  fluid  sought,  and 

w=.  the  entire  weight  of  the  aerometer,  always  known. 

Then,  it  is  manifest,  that  the  capacity  or  volume  of  the  phial, 
together  with  the  magnitude  of  the  immersed  wire,  is  equal  to  the 
quantity  of  fluid  displaced  ;  and  the  weight  of  this  quantity  of  fluid 
is  equal  to  the  weight  of  the  aerometer  ;  but  by  the  principles  of  men- 
suration, the  magnitude  of  the  immersed  wire  is  expressed  by  Trr*l; 
consequently,  the  quantity  of  fluid  displaced  is  c-f-7rr2/,  and  the 
magnitude  of  any  body  multiplied  by  its  specific  gravity  is  equal  to 
its  weight  ;  hence  we  have 

t0  =  (c  +  *•»*/)*;  (220). 

therefore,  by  division,  we  obtain 
w 

(221). 


357.  Here  follows  the  practical  rule  for  reducing  the  equation. 

RULE.  Divide  the  entire  given  weight  of  the  aerometer,  by 
the  capacity  or  volume  of  the  phial,  increased  by  the  quantity 
of  wire  immersed,  and  the  quotient  will  give  the  specific  gra- 
vity of  the  fluid. 

358.  EXAMPLE.  The  whole  weight  of  an  aerometer,  when  so  loaded 
as  to  have  the  attached  wire  depressed  15  inches  below  the  surface  of 
the  fluid,  is  23  ounces;  required  the  specific  gravity  of  the  fluid, 
supposing  the  diameter  of  the  wire  to  be  one  twelfth  of  an  inch,  and 
the  capacity  of  the  phial  40  inches  ? 

Here,  by  the  mensuration  of  solids,  the  magnitude  of  the  wire  is 

IGX^TXTT^0-082  of  a  cubic  inch,  very  nearly; 
therefore,  the  whole  quantity  of  fluid  displaced,  is 
40  4-  0.082  zz:  40.082  cubic  inches  ; 
therefore,  by  the  rule,  we  obtain 


The  number  0.5738,  which  we  have  obtained  from  the  above  calcu- 
lation, expresses  the  weight  of  one  cubic  inch  of  the  fluid  in  ounces  ; 
but  since  it  is  customary  to  express  the  specific  gravity  of  bodies  in 
ounces  per  cubic  foot,  it  becomes  necessary,  for  the  sake  of  compari- 
son, to  reduce  the  above  result  to  that  standard  ;  hence  we  have 

s  zn  0.5738  X  1728  z=  991  .5264  ounces  per  cubic  foot  for 
the  specific  gravity  of  the  fluid  on  which  the  experiment  was  tried. 


OF  THE  EQUILIBRIUM  OF  FLOATATION.  285 

PROBLEM  LIII. 

359.  Having  given  the  capacity  or  volume  of  the  phial,  the 
whole  weight  of  the  aerometer,  the  specific  gravity  of  the  fluid, 
and  the  radius  of  the  wire : — 

It  is  from  thence  required  to  determine,  how  much  of  the 
stem  or  wire  is  immersed  below  the  surface  of  the  fluid  when 
the  instrument  rests  in  a  state  of  equilibrium. 

By  recurring  to  the  equation  marked  (220),  and  separating  the 
terms,  we  obtain 

Trr*sl—  w  —  cs; 

from  which,  by  division,  we  get 

w  —  cs 

T7T'  (222). 

360.  The  practical  rule  for  reducing  this  equation,  may  be  expressed 
in  words  at  length,  in  the  following  manner. 

RULE.  From  the  entire  weight  of  the  hydrometer,  subtract 
the  capacity  of  the  phial  drawn  into  the  specific  gravity  of 
the  fluid ;  then,  divide  the  remainder  by  the  area  of  a  trans- 
verse section  of  the  wire,  drawn  into  the  specific  gravity  of 
the  fluid,  and  the  quotient  will  express  how  far  the  wire  is 
immersed  below  the  upper  surface  of  the  fluid,  when  the 
instrument  floats  in  a  state  of  equilibrium. 

361.  EXAMPLE.  The  entire  weight  of  an  aerometer,  when  so  adjusted 
as  to  remain  at  rest  in  a  fluid  whose  specific  gravity  is  0.5738*nr23 
ounces ;  what  length  of  the  stem  or  upright  wire  falls  below  the  sur- 
face of  the  fluid,  supposing  its  diameter  to  be  one  twelfth  of  an  inch, 
and  the  magnitude  of  the  immersed  phial  40  inches  ? 

Here,  by  the  foregoing  rule,  we  have 
23  —  40x0.5738 
'=  3^1416X^7x07673-8  =  15'33  mcheS  ™** 

362.  If  the  entire  weight  of  the  aerometer  be  multiplied  by  1728, 
the  number  of  cubic  inches  in  one  cubic  foot,  the  formulas  (221)  and 
(222)  become  transformed  into 

*  The  number  0.5738,  by  which  the  specific  gravity  is  here  expressed,  is  the 
weight  in  ounces  of  one  cubic  inch,  which  being  reduced  to  the  standard  of  one 
cubic  foot,  gives  srr 0.57 38x1728 =98 1.5624  oz. 


286  OF  THE  EQUILIBRIUM  OF  FLOATATION. 

d,__1728?<;  —  cs 
c 

from  the  first  of  which  the  standard  specific  gravity  is  obtained,  and 
in  the  second,  the  specific  gravity  as  calculated  from  the  first  must  be 
employed. 

By  comparing  the  quantities  in  equation  (222)  with  each  other,  it 
will  readily  be  perceived,  that  a  very  small  variation  in  w  the  weight 
of  the  instrument,  or  in  s  the  specific  gravity  of  the  fluid,  will  produce 
a  very  considerable  variation  in  Z,  the  immersed  portion  of  the  stem 
or  wire  ;  for  it  is  manifest,  that  the  numerator  of  the  fraction  w  —  cs, 
expresses  the  weight  of  the  fluid  displaced  by  the  wire  or  upright 
stem  of  the  instrument,  and  consequently,  since  r  the  radius  of  the 
stem  is  a  very  small  quantity,  it  follows,  that  the  weight  of  the  fluid 
which  it  displaces  must  also  be  very  small. 

PROBLEM  LIV. 

363.  Suppose  that  a  small  variation  takes  place  in  the  density 
of  the  fluid  in  which  the  instrument  is  immersed  :  — 

It  is  required  to  determine  the  corresponding  variation  that 
takes  place  in  the  depth  to  which  it  sinks  before  the  equili- 
brium is  restored. 

Let  the  notation  for  the  first  position  of  equilibrium  remain  as  in 
Problem  LII.,  and  let  I'  denote  the  immersed  length  of  the  stem  or 
wire,  corresponding  to  the  specific  gravity  s'  ;  then,  by  equation  (222), 

we  have 

_  w  —  cs' 

7T?'V      ' 

consequently,  by  subtraction,  the  variation  in  length  becomes 
w  —  cs      w  —  cs' 


and  this,  by  a  little  farther  reduction,  gives 


(223). 

364.  The  practical  rule  for  reducing  this  equation,  may  be  expressed 
in  words  as  follows. 

RULE.  Multiply  the  whole  weight  of  the  aerometer  by  the 
variation  in  the  specific  gravity  ;  then,  divide  the  product  by 
the  area  of  the  transverse  section  of  the  upright  stem  or  wire, 


OF  THE  EQUILIBRIUM  OF  FLOATATION.  287 

drawn  into  the  greater  and  lesser  specific  gravities  of  the  fluid, 
and  the  product  will  express  the  required  variation  in  the 
position  of  the  instrument. 

365.  EXAMPLE.  Suppose  the  specific  gravity  of  the  fluid  to  vary 
from  0.5738  to  0.5926  ounces  per  cubic  inch  during  the  time  of  the 
experiment,  what  is  the  corresponding  variation  in  the  depth  of  the 
instrument,  its  whole  weight  being  23  ounces,  and  the  diameter  of  the 
upright  stem  equal  to  one  twelfth  of  an  inch  ? 

Here,  by  attending  to  the  directions  in  the  rule,  we  obtain 
.       .,          23(0.5926  —  0.5738) 


Hence  it  appears,  that  by  a  difference  of  0.0325  in  the  absolute 
specific  gravity  of  the  fluid,  there  arises  a  difference  of  233  inches  in 
the  position  of  the  instrument  ;  this  seems  a  very  great  difference,  and 
is  in  reality  far  beyond  the  bounds  prescribed  for  the  whole  apparatus 
to  occupy  ;  it  serves,  however,  to  exemplify  the  extreme  delicacy  of 
the  principle,  and  when  the  changes  in  the  specific  gravity  are  very 
minute,  the  corresponding  changes  in  depth  will  nevertheless  be  suffi- 
ciently distinct  to  admit  of  an  accurate  measurement. 

366.  By  diminishing  the  diameter  of  the  upright  stem,  or  increasing 
the  entire  weight  of  the  instrument,  which  is  equivalent  to  an  increase 
in  the  weight  of  the  fluid  displaced,  the  sensibility  of  the  aerometer 
may  be  greatly  increased.  This  is  manifest,  for  by  inference  5, 
equation  (202),  it  will  readily  appear,  that  if  the  specific  gravity 
remains  the  same,  the  quantity  by  which  the  instrument  sinks  in  the 
fluid  on  the  addition  of  a  small  weight  wr,  varies  directly  as  the  mag- 
nitude of  the  weight  added,  and  inversely  as  the  square  of  the  radius 
of  the  upright  stem. 

Let  us  suppose,  that  by  the  addition  of  the  small  weight  w',  the 
length  of  the  part  of  the  stem  Z,  which  is  originally  immersed,  becomes 
equal  to  /'  ;  then,  by  the  principles  of  mensuration,  the  increased 
magnitude  of  the  immersed  stem  is  7rr*(l'  —  I);  but  the  weight  of  a 
body  is  equal  to  its  magnitude  multiplied  by  its  specific  gravity; 
hence  we  have 

*r\l'  —  l)s  =  w'; 

and  this,  by  division,  becomes 

r-r=+-. 

irr^s 

Now  it  is  obvious,  that  by  the  supposition  of  a  constant  specific 
gravity,  the  quantity  ITS  is  also  constant  :  it  therefore  follows,  that 


288  OF  THE  EQUILIBRIUM  OF  FLOATATION. 

w' 

V  —  I  varies  as  —  . 
r* 

In  the  above  investigation,  we  have  supposed  the  specific  gravity  of 
the  fluid  to  remain  constant  ;  but  admitting  it  to  vary,  so  that  s  may 
become  equal  to  s'  ;  then,  in  order  that  the  upright  stem  may  rest  at 
the  same  depth  of  immersion,  w  must  become  equal  to  (w  -f-  w')  ;  if, 
therefore,  we  substitute  s  and  (w  -)-  w'),  for  s  and  w  in  equation  (223), 
we  shall  obtain 

_  w  -\-w'  —  cs' 

I  —  „      ,  , 

TrrV 

an  equation  from  which  we  find  the  value  of  s'  to  be 
w  —  w' 


and  by  a  similar  reduction,  equation  (223)  gives 


w 


consequently,  by  analogy,  and  suppressing  the  common  denominator, 
we  get 

s'  :  s  :  :  w  -}-  w'  :  w. 

From  this  analogy,  the  difference  between  the  specific  gravities  in 
the  two  cases  can  very  easily  be  ascertained,  for  by  the  division  of 
ratios,  we  have 

s'  —  s  :  s  :  :  w  -\-  w'  —  w  :  w, 

which,  by  reduction,  becomes 
__  w'  s 
~  17*  (224), 

367.  This  is  a  very  simple  equation  for  expressing  the  difference 
of  the  specific  gravities  ;  it  may  be  reduced  by  the  following  practical 
rule. 

RULE.  Multiply  the  added  weight  by  the  lesser  specific 
gravity  ;  then,  divide  the  product  by  the  lesser  weighty  and 
the  quotient  will  be  the  difference  between  the  specific  gravities 
sought, 

368.  EXAMPLE.  An  aerometer,  whose  absolute  weight  is  equal  to 
23  ounces,  is  quiescent  in  a  fluid  whose  specific  gravity  is  0.5738 
ounces,  as  referred  to  a  cubic  inch  ;  but  on  being  put  into  a  denser 
fluid,  it  requires  the  addition  of  0.7536  of  an  ounce,  to  cause  the 
instrument  to  sink  to  the  same  depth  ;   what  is  the  specific  gravity 
of  the  denser  fluid? 


OF  THE  EQUILIBRIUM  OF  FLOATATION.  289 

Here  then  we  have  given  w'  =  0.7536  of  an  ounce,  and  s  =  0.5738  ; 
consequently,  by  the  above  rule,  we  have 


consequently,  the  specific  gravity  of  the  heavier  fluid,  is 

s'  =  0.5738  -f  0.0188  =  0.5926  ; 

and  this,  when  reduced  to  the  standard  of  one  cubic  foot,  becomes 
0.5926x1728  =  1024.0128,  which,  on  being  referred  to  a  table  of 
specific  gravities,  will  be  found  to  correspond  with  sea  water  at  a 
medium  temperature. 

In  the  above  operation,  we  have  taken  the  specific  gravity  as  re- 
ferred to  one  cubic  inch  of  the  fluid  only,  but  the  well  informed  reader 
will  readily  perceive,  that  the  same  result  would  obtain  if  the  specific 
gravity  should  be  estimated  by  the  cubic  foot  ;  for  in  that  case,  we 
should  have  w1  —  0.7536  of  an  ounce,  and  5  =  991.5264,  conse- 
quently, by  the  rule,  we  have 


23 

therefore  by  transposition,  the  specific  gravity  of  the  denser  fluid,  is 
s'  =  991.5264  4-  32.4864  =  1024.0128,  being  precisely  the  same 
result  as  that  which  we  obtained  on  the  former  supposition. 

369.  The  diagram  which  we  have  employed  to  illustrate  the  general 
principle  of  the  aerometer,  is  at  the  best  but  a  very  rude  and  imper- 
fect representation,  and  in  its  present  state,  it  is  altogether  unfitted 
for  ascertaining  the  specific  gravities  of  fluids  with  any  degree  of  pre- 
cision ;  it  is  therefore  requisite,  in  cases  where  extreme  accuracy  is 
required,  to  have  recourse  to  some  other  method  of  indicating  the 
precise  measure  of  density,  and  for  this  purpose,  the  hydrometer  or 
aerometer,  is  very  advantageously  replaced  by  the 

HYDROSTATIC  BALANCE, 

an  instrument  which  determines  the  specific  gravities  of  bodies  with 
the  greatest  correctness,  and  which,  on  account  of  its  simplicity  and 
cheapness,  is  rendered  available  for  almost  every  purpose  in  which 
the  specific  gravity  of  bodies  forms  the  subject  of  inquiry. 

The  Hydrostatical  Balance,  so  called,  is  nothing  more  than  a 
common  balance,  furnished  with  some  additional  apparatus  for  ena- 
bling it  to  measure  the  specific  gravities  of  bodies  with  accuracy  and 
expedition,  whether  the  bodies  be  in  a  solid  or  a  fluid  state.  The 
description  of  the  instrument  is  as  follows. 

VOL.  i.  u 


290 


OF  THE  EQUILIBRIUM  OF  FLOATATION. 


Let  AB  be  the  beam  of  a  balance  very  nicely  equipoised  upon  its 
centre  of  motion  at  c,  and  suspended 
from  the  fixed  object  represented  at  F, 
the  centering  being  so  delicately  exe- 
cuted, that  the  equilibrium  of  the  instru- 
ment is  disturbed  by  the  smallest  por- 
tion of  a  grain  being  added  to  or  sub- 
tracted from  either  arm  of  the  beam. 

D  and  E  are  two  scales,  which,  together 
with  their  appendages  are  also  balanced 
with  the  greatest  exactness ;  one  of  them 
as  E  having  a  hook  in  the  middle  of  its 
bottom  surface,  to  which  the  weight  w 
is  suspended  by  means  of  a  horse  hair, 

or  any  other  flexible  substance  of  such  extreme  levity,  as  to  have  no 
sensible  effect  upon  the  equilibrium. 

p  is  an  upright  pillar  placed  directly  under  the  centre  of  motion, 
and  carrying  the  circular  arc  mmm,  which  serves  to  prevent  a  too 
great  vibration  on  either  side,  and  also,  by  means  of  the  index  i,  which 
is  fixed  on  the  beam  immediately  under  the  fulcrum,  it  indicates  the 
exact  position  of  equilibrium  ;  for  it  is  manifest,  that  when  the  beam 
is  horizontal,  the  pointer  must  be  directly  over  the  middle  of  the  arc. 
The  pieces  in  the  scale  D,  denote  the  weight  of  the  body  when 
weighed  in  air ;  but  when  the  body  is  immersed  in  water,  as  repre- 
sented by  the  figure  abed,  the  scale  D  with  its  accompanying  weights, 
must  evidently  preponderate,  and  for  the  purpose  of  restoring  the 
equilibrium,  small  weights  must  be  placed  in  the  opposite  scale  at  E  ; 
and  since  the  weights  thus  added,  indicate  the  weight  of  a  quantity  of 
water  of  equal  bulk  with  the  immersed  body,  it  follows,  that  the  specific 
gravity  of  the  body  can  from  thence  be  determined. 

The  hydrostatical  balance,  like  the  hydrometer  or  aerometer  pre- 
viously explained,  has  undergone  various  alterations  and  improve- 
ments, according  to  the  ideas  of  the  different  individuals  who  have 
had  occasion  to  apply  it  in  their  inquiries  respecting  the  specific 
gravities  of  bodies ;  but  since  the  general  principle  is  the  same  in  all, 
under  whatever  form  the  instrument  may  appear,  it  would  lead  to 
nothing  useful  to  enter  into  a  detailed  description  of  the  various 
improvements  which  it  has  received,  and  the  numerous  changes  that 
have  been  made  upon  it ;  we  shall  therefore  refrain  from  farther  dis- 
cussion on  the  nature  of  its  construction,  and  proceed  to  exemplify  the 
manner  in  which  it  is  applied  to  the  determination  of  specific  gravities. 


OF  THE  EQUILIBRIUM  OF  FLOATATION.  291 

PROBLEM  LV. 

370.  Having   given  the  specific   gravity  of  distilled  water, 
equal  to  1000  ounces  per  cubic  foot : — 

It  is  required  to  determine  the  specific  gravity  of  a  solid 
body  that  is  wholly  immersed  in  it. 

It  is  manifestly  implied  by  the  total  immersion  of  the  body,  that  its 
specific  gravity  exceeds  the  specific  gravity  of  the  fluid  in  which  it  is 
immersed ;  therefore,  attach  the  body  to  the  hook  in  the  bottom  of 
the  scale  E  by  a  very  fine  and  light  thread,  and  balance  it  exactly  by 
weights  put  into  the  other  scale  at  D  ;  then,  immerse  the  body  in  the 
water,  and  find  what  weight  is  required  to  restore  the  equilibrium,  the 
weight  thus  required  will  measure  the  specific  gravity  of  the  body. 
Put  w  z=  the  weight  of  the  body  when  weighed  in  water, 
w'  ~  the  weight  when  weighed  in  atmospheric  air, 
s   zn  the  specific  gravity  of  water,  and 
s'  —  the  specific  gravity  of  the  body  sought. 

Then  is  w — w'  equal  to  the  weight  which  must  be  put  into  the  scale 
E  to  restore  the  equilibrium;  consequently,  by  the  fifth  proposition, 
we  have  .  w'  —  w  :  w'  :  :  s  :  s' ; 

from  which,  by  reduction,  we  get 

,'-_^i- 

~w'  —  w  (225). 

371.  The  following  is   the  practical  rule  in   words  at  length  for 
reducing  the  above  equation. 

RULE.  Multiply  the  weight  of  the  body  when  weighed  in 
air,  by  the  specific  gravity  of  the  fluid,  and  divide  the  product 
by  the  weight  which  it  loses  in  water  for  the  specific  gravity 
of  the  body. 

This  rule  determines  the  specific  gravity  of  the  body  when  it 
exceeds  that  of  the  fluid  in  which  it  is  weighed ;  but  when  the  body 
is  specifically  lighter  than  the  fluid,  the  method  of  finding  its  specific 
gravity  is  shown  in  Problem  XLIV.,  it  is  therefore  unnecessary  to 
repeat  it  here. 

372.  EXAMPLE.  If  a  piece  of  stone  weighs  20  Ibs.  in  air,  but  in  water 
only  13J  Ibs.;  required  its  specific  gravity,  that  of  water  being  1000  ? 

Here,  by  the  rule,  w=:  13J,  w'=  20,  s=z  1000, 

20X1000      20000 
therefore  s  =— T^T—  ~  3076.923  rz  the  specific  gravity 

Z(j —~  Aug  O.O 

of  the  mass  when  it  is  wholly  immersed  in  water. 

u  2 


CHAPTER  XII. 

OF   THE    POSITIONS    OF    EQUILIBRIUM. 


PROBLEM  LVI. 

373.  Suppose  that  a  solid  homogeneous  triangular  prism, 
floats  upon  the  surface  of  a  fluid  of  greater  specific  gravity 
than  itself,  with  only  one  of  its  edges  immersed : — 

It  is  required  to  determine  in  what  position  it  will  rest, 
when  it  has  attained  a  state  of  perfect  equilibrium. 

Let  ABC  be  a  vertical  transverse  section,  at  right  angles  to  the  axis 
of  the  homogeneous  prism,  floating       A 
in  a  state  of  equilibrium  on  the  fluid 
whose  horizontal  surface  is  IK. 

Bisect  AB,  BC  the  sides  of  the  tri- 
angle in  the  points  r  and  n,  and  D  E, 
D  c  in  the  points  H  and  m ;  draw  the 
straight  lines  CF  and  AH,  intersecting 
one  another  in  the  point  G,  and  CH, 
EWI  intersecting  in  g ;  then  is  G  the 
centre  of  gravity  of  the  whole  triangle 
ABC,  and  g  the  centre  of  gravity  of 
the  triangle  DEC,  which  falls  below 
DE  the  plane  of  floatation. 

Join  the  points  G,  g  by  the  straight 
line  G  # ;  then,  according  to  the  prin- 
ciple announced  and  demonstrated  in  the  sixth  proposition,  the  straight 
line  gc,  is  perpendicular  to  DE  the  surface  of  the  fluid. 

Draw  FH,  and  because  CF  and  CH  the  sides  of  the  triangle  CFH, 
are  cut  proportionally  in  the  points  G  and  g,  it  follows  from  the  prin- 
ciples of  geometry,  that  the  straight  lines  Gg  and  F  H  are  parallel  to 
one  another ;  but  we  have  shown  that  gG  is  perpendicular  to  the  hori- 


OF  THE  POSITIONS  OF  EQUILIBRIUM.  293 

zontal  surface  of  the  fluid,  or  the  plane  of  floatation  passing  through 
DE;  consequently,  F  H  is  also  perpendicular  to  DE,  and  FD,  FE  are 
equal  to  one  another. 

Put  a  —  A  B,  the  unimmersed  side  of  the  triangular  section, 
b  nz  BC,  one  of  the  sides  which  penetrate  the  fluid, 
c  zz  AC,  the  other  penetrating  side, 
d  =2  CF,  the  distance  between  the  vertex  of  the  section,  and 

the  middle  of  the  extant  side, 
0  zz  the  angle  ACF,  contained  between  the  side  AC  and  the 

line  CF, 
^'  zz  the  angle  BCF,  contained  between  the  line  CF  and  the 

side  B, 

s  zz  the  specific  gravity  of  the  solid  body, 
s'  zz  the  specific  gravity  of  the  fluid  on  which  it  floats, 
x  zz  CD,  the  immersed  portion  of  the  side  AC,  and 
y  zz  CE,  the  immersed  portion  of  the  side  BC. 

Then,  according  to  the  principles  of  geometry,  since  the  line  CF  is 
drawn  from  the  vertex  of  the  triangle  at  c,  to  the  middle  of  the  base 
or  opposite  side  at  F,  it  follows,  that 

AC2  -f  BC2ZZ2(AF2-f  CF2), 

or  by  taking  the  symbolical  representatives,  we  shall  obtain 


from  which,  by  reduction,  we  get 

dzziV2(^4-c2)  —  a*.  (226). 

Since  all  straight  lines  drawn  parallel  to  the  axis  of  the  prism  are 
equal  among  themselves  ;  it  follows,  that  the  weight  of  the  whole  solid 
ABC,  and  that  of  the  portion  DEC  below  the  plane  of  floatation,  which 
corresponds  to  the  magnitude  of  the  fluid  displaced,  are  very  appro- 
priately represented  by  the  areas  drawn  into  the  respective  specific 
gravities  of  the  solid  and  the  fluid  on  which  it  floats. 

Now,  the  writers  on  the  principles  of  mensuration  have  demon- 
strated, that  the  area  of  any  right  lined  triangle  :  — 

Is  equal  to  the  product  of  any  two  of  its  sides,  drawn  into 
half  the  natural  sine  of  their  contained  angle. 

Therefore,  if  we  put  a'  and  a"  to  represent  the  areas  of  the  triangles 
ABC  and  DEC  respectively,  we  shall  have  for  the  area  of  the  triangle 
ABC, 


294  OF  THE  POSITIONS  OF  EQUILIBRIUM. 

and  for  the  area  of  the  triangle  DEC,  it  is 


But  according  to  the  principle  demonstrated  in  the  third  proposition 
preceding,  the  weight  of  a  floating  body  :  — 

Is  equal  to  the  weight  of  the  quantity  of  fluid  displaced. 

Consequently,  the  weight  of  the  solid  prism  whose  section  is  ABC, 
is  equal  to  the  weight  of  the  fluid  prism,  whose  section  is  D  EC  ;  that  is, 

\bcs  sin.ty  4-  4>')  =  \xys'  sin.ty  -f  0'), 
and  from  this,  by  suppressing  the  common  quantities,  we  get 

bcs  —  xys.  (227). 

By  the  principles  of  Plane  Trigonometry,  it  is 
F  D2  zz  d*  +  x*  —  2c?a-  cos.<£,  and  F  E2  =z  d*  -|-  y*  —  2dy  cos.^'  ; 
but  these  by  construction  are  equal  ;  hence  we  have 


Let  the  value  of  d  as  expressed  in  equation  (226),  be  substituted 
instead  of  it  in  the  above  equation,  and  we  shall  obtain 


a?*—  x  cos.(j>  V  2(c2  -f  62)  —  a2  —  ?/2— y  cos. f  V  2(c2-f  62)  —  a2.   (228). 
Recurring  to  equation  (227),  by  division,  we  have 

bcs    ,  r    ,  .  ,  .  6Vs2 

?/  zn . ,  the  square  of  which  is  w2  zn ; 

xs  x*s 2 

substitute  these  values  of  y  and  y9  in  equation  (228),  and  it  is 

bcscos.Q'     . 

and  multiplying  by  a?2  we  obtain, 
£rcV bcs  cos.0' 


5  •  s 

from  which,  by  transposition,  we  get 


a-4—  cos.     /  2^+c2—  a2  X  x*-\  --  —  -     262      c2  —  a2  X  x= 


(229). 

374.  The  equation  as  we  have  now  exhibited  it,  involves  the  several 
circumstances  that  accompany  the  equilibrium  of  a  floating  body,  and 
its  root  determines  the  position  in  which  the  equilibrium  obtains  ;  the 
general  form  of  the  expression,  is  however  exceedingly  complex,  and 
rising  as  it  does  to  the  fourth  order  or  degree,  the  resolution  is  neces- 
sarily attended  with  considerable  difficulty,  especially  when  the  sides 


OF  THE  POSITIONS  OF  EQUILIBRIUM.  295 

of  the  transverse  section  are  represented  by  large  numbers  ;  in  parti- 
cular cases,  the  ultimate  form  will  admit  of  being  modified,  and  may 
in  consequence,  be  rendered  somewhat  more  simple  ;  but  it  must 
nevertheless  be  understood,  that  whenever  the  position  of  equilibrium 
is  required  by  computation,  we  must  inevitably  perform  a  very  irksome 
and  laborious  process. 

A  geometrical  construction  may  also  be  effected  by  the  intersection 
of  two  hyperbolas  ;  but  since  this  implies  a  knowledge  of  principles 
higher  than  elementary,  we  think  proper  to  pass  it  over,  and  proceed 
to  illustrate  the  application  of  the  above  equation  by  the  resolution  of 
a  numerical  example. 

375.  EXAMPLE.  Suppose  a  triangular  prism  of  Mar  Forest  fir, 
the  sides  of  whose  transverse  section  are  respectively  equal  to  28,  26, 
and  18  inches,  to  float  in  equilibrio  in  a  cistern  or  reservoir  of  water, 
having  only  one  angle  immersed  ;  it  is  required  to  determine  the  posi- 
tion of  equilibrium,  on  the  supposition  that  the  two  longest  sides  of 
the  section  penetrate  the  fluid,  the  specific  gravity  of  the  prism  being 
to  that  of  water  as  686  to  1000  ? 

By  recurring  to  equation  (229),  and  comparing  its  several  consti- 
tuent quantities  with  the  parts  of  the  diagram  to  which  they  respec- 
tively refer,  it  will  readily  appear,  that  x,  cos.0  and  cos.0'  are  the  only 
terms  whose  values  require  to  be  calculated  ;  of  which  cos.0  and  cos.0' 
are  to  be  determined  from  the  nature  of  the  figure,  and  x  from  the 
resolution  of  the  biquadratic  equation  in  which  its  values  are  involved. 

The  length  of  the  straight  line  CF,  which  is  drawn  from  the  vertex 
of  the  section  at  c,  to  the  middle  of  the  opposite  side  at  F,  is  accord- 
ing to  equation  (226),  expressed  by 


consequently,  by  substituting  the  numerical  values  of  the  sides,  we 
obtain  _ 

d=  1^2(28*  +  262)  —  182  =  25.4754784  inches. 
Hence,  in  the  triangles  ACF  and  BCF  respectively,  we  have  given 
the  three  sides  AC,  AF,  FC  and  BC,  BF,  FC  to  find  COS.ACF  and  cos. 
BCF;  for  which  purpose,  we  have  the  following  equations  as  deduced 
from  the  elements  of  Plane  Trigonometry,  viz.  In  the  triangle  ACF, 
it  is 

4c«       d«  —  a2 


and  in  the  triangle  BCF,  it  is 


296  OF  THE  POSITIONS  OF  EQUILIBRIUM. 

Now,  by  substituting  the  numerical  values  of  a,  b  and  c,  as  given 
in  the  question,  and  the  value  of  d  as  deduced  from  calculation,  the 
absolute  values  of  cos.0  and  cos.0'  will  stand  as  below. 

Thus,  for  the  absolute  numerical  value  of  cos.^>,  we  have 
4(784-f649)  —  324 


and  for  the  absolute  numerical  value  of  cos.^',  it  is 
649)  —  324 


Let  the  numerical  values  of  cos.^>  and  cos.^'  as  determined  by  the 
above  computation,  together  with  the  numerical  values  of  a,  b,  c,  s, 
and  s',  as  given  in  the  question,  be  respectively  substituted  in  equa- 
tion (229),  and  we  shall  obtain 

x4  —  48.2S59*3  4-  23894.7*  =:  249408  ; 

but  in  order  to  simplify  the  resolution  of  this  equation,  it  will  suffice 
to  take  the  co-efficients  to  the  nearest  integer,  for  the  error  thence 
arising  will  be  of  very  little  consequence  in  cases  of  practice,  and  the 
modification  will  very  much  abridge  the  labour  of  reduction  ;  the 
equation  thus  altered,  will  stand  as  below. 

x4—4Sxs  -f-  23895*  =  249408. 

Therefore,  if  this  equation  be  reduced  by  the  method  of  approxima- 
tion, or  otherwise,  the  value  of  or  will  come  out  a  very  small  quantity 
less  than  22  inches  ;  but  taking  it  equal  to  22,  the  result  of  the  equa- 
tion is 

224—  48  X223  4-  23895X22  =  248842. 

By  substituting  the  given  values  of  b,  c,  s  and  s',  with  the  com- 
puted value  of  x,  in  equation  (227),  we  shall  have 

22000^  =  499408, 
from  which,  by  division,  we  obtain 
499408 


22000  ' 

Consequently,  from  these  computed  dimensions,  together  with  the 
sides  of  the  section  given  in  the  question,  the  prism  may  be  exhi- 
bited in  the  position  which  it  assumes  when  floating  in  a  state  of 
equilibrium. 

376.  Construct  the  triangle  ABC  to  represent  the  transverse  section  of 
the  floating  prism,  and  such  that  the  sides  AC,  BC,  and  AB  are  respec- 
tively equal  to  28,  26,  and  18  inches;  make  CD  and  CE  respectively 


OP  THE  POSITIONS  OF  EQUILIBRIUM.  297 

equal  to  22  and  22.7  inches,  and  through  the  points  D  and  E,  draw 

the  straight  line  IK,  which  will  coincide 

with  the  plane  of  floatation,  or  the  sur-          A 

face  of  the  fluid  on  which  the  body  floats.  \     Jx^xT; — -3 

Bisect  A  B,  the  extant  side  of  the  sec-   T \^  ^^1 

tion  in  the  point  F,  and  join  F  D  and  F  E  ; 
then,  the  conditions  of  equilibrium  ma- 
nifestly are,  that  the  lines  FD  and  FE 
are  equal  to  one  another,  and  that  the 
area  of  the  immersed  triangle  DCE,  is 
to  the  area  of  the  whole  triangle  ACB, 
as  the  specific  gravity  of  the  solid  is  to 
the  specific  gravity  of  the  fluid. 

That  the  lines  FD  and  FE  are  equal  to  one  another,  appears  from 
an  inspection  and  measurement  of  the  figure  ;  but  the  following  proof 
by  calculation  will  be  more  satisfactory,  inasmuch  as  numbers  can  be 
more  correctly  estimated  than  measured  lines,  which  depend  for  their 
accuracy  upon  the  delicacy  of  the  instruments  and  the  address  of  the 
operator. 

In  the  plane  triangle  DCF,  we  have  given  the  two  sides  DC  and  CF, 
respectively  equal  to  22  and  25.4754784  inches,  and  the  natural  cosine 
of  the  contained  angle  DCF  equal  to  0.94769  ;  consequently,  the  third 
side  D  F  can  easily  be  found  ;  for  by  the  principles  of  Plane  Trigono- 
metry, we  know  that 

DF2=:DC24-  FC2 2DC.FCCOS.DCF; 

therefore,  by  substituting  the  respective  numerical  values,  we  obtain 

DF2  =  484  +  649  —  2X22X25.4754784X0.94769  =  70.72; 

consequently,  by  extracting  the  square  root,  it  is 

DFZZI V70.72  =  8.4  inches. 

Again,  in  the  plane  triangle  ECF,  we  have  given  the  two  sides 
EC  and  CF,  respectively  equal  to  22.7  and  25.4754784  inches,  and 
the  natural  cosine  of  the  contained  angle  ECF  equal  to  0.93906; 
consequently,  by  Plane  Trigonometry,  we  have 

EF2zrEC9-|-CF2 2EC.CFCOS.ECFJ 

and  substituting  the  respective  numerical  values,  we  obtain 
EF2  =: 515.29  -f-  649  —  2 X22.7  X25.4754784 X 0.93906  =:  77.895 ; 
therefore,  by  extracting  the  square  root,  we  shall  have 
E  F  zz:  V  777895  —  8 .82  inches. 


298  OF  THE  POSITIONS  OF  EQUILIBRIUM. 

Hence/the  length  of  the  line  DF  is  8.4  inches,  and  the  length  of  EF 
is  8.82  inches,  giving  a  difference  of  0.42,  or  something  less  than  half 
an  inch ;  being  as  small  a  difference  as  could  be  expected,  from  the 
manner  in  which  the  co-efficients  of  the  equation  that  furnished  the 
value  of  x  were  modified,  and  also  from  the  circumstance  of  x  being 
determined  only  to  the  nearest  integer,  without  considering  the  frac- 
tions with  which  it  might  be  affected. 

377.  Upon  the  whole  then,  the  position  of  equilibrium  is  sufficiently 
manifest,  from  the  condition  of  equality  between  the  straight  lines  D  F 
and  EF  ;  we  shall  therefore  proceed  to  inquire  if  it  be  equally  apparent, 
from  the  proportionality  between  the  triangles  ACB  and  DCE. 

Since  cos. </>  =  0.94769,  and  cos.  f —  0.93906,  it  follows  that 
<j>  =  l8°  37'  and  </>':=:  20°  6';  consequently,  by  addition,  the  whole 
angle  ACB  becomes 

04-f  =  18°  37'4-20°  6'  =  38°  43'; 

therefore,  in  each  of  the  triangles  ACB  and  DCE,  we  have  given  the 
two  sides  AC,  BC  and  DC,  EC  with  the  contained  angle  ACB  common 
to  both,  to  find  the  respective  areas. 

Now,  the  writers  on  mensuration  have  demonstrated,  that  when 
two  sides  of  a  plane  triangle,  together  with  the  contained  angle  are 
given : — 

The  area  of  the  triangle  is  equal  to  the  product  of  the  two 
sides  drawn  into  half  the  natural  sine  of  their  included  angles 

378.  This  is  a  principle  which  we  have  already  stated  in  the  investi- 
gation, and  expressed  analytically  in  deducing  equation  (227) ;  we 
shall  now  employ  it  in  determining  the  areas  of  the  triangles  according 
to  the  magnitudes  of  the  sides  and  the  contained  angle,  as  given  in 
the  example  and  derived  from  computation. 

The  natural  sine  of  38°  43'  is  0.62547,  and  the  sides  AC  and  BC  are 
respectively  28  and  26  inches ;  consequently,  by  the  above  principle, 
we  have 

a'  =  1(28X26X0.62547)  —  227.671  square  inches. 
The  natural  sine  of  the  contained  angle  remaining  as  above,  the 
sides  DC  and  EC  as  derived  from  computation,  are  equal  respectively 
to  22  and  22.7  inches ;  hence,  from  the  same  principle,  we  have 

a"=  1(22X22.7X0.62547)  =  156.1798  square  inches. 
Now,  according  to  the  conditions  of  the  question,  the  specific 
gravity  of  the  fluid  is  1000,  and  that  of  the  floating  body  is  686; 
consequently,  we  obtain 

1000  :  227.671  :  :  686  :  156.1823. 


OF  THE  POSITIONS  OF  EQUILIBRIUM. 


299 


In  this  case  the  errorx  is  extremely  small,  amounting  only  to 
156.1823 — 156.1798  =  0.0025  of  a  square  inch;  hence  we  con- 
clude, that  the  position  of  equilibrium  under  the  given  conditions,  is 
very  nearly  the  same  as  we  have  found  it  to  be  from  the  resolution  of 
the  equations  (227)  and  (229). 

379.  The  preceding  solution,  however,  indicates  only  one  position 
of  equilibrium ;  but  it  is  manifest  from  the  nature  of  the  equation 
(229),  that  there  may  be  more,  for  by  transposition,  we  have 

bcscos.ti' 

/     a    i  T 


-a2  X  x— 


. 
=  0, 


B 


-,/E 


and  it  is  demonstrated  by  the  writers  on  algebra,  that  in  every  equa- 
tion of  an  even  number  of  dimensions,  having  its  last  term  negative, 
there  are  at  least  two  real  roots,  the  one  positive  and  the  other  nega- 
tive ;  consequently,  the  above  equation  has  two  of  its  roots  real  and 
determinable  ;  but  the  equation  being  of  four  dimensions  has  also  four 
roots,  hence,  the  other  two  roots  may  also 
be  real,  and  in  that  case,  there  will  be  three 
values  of  x  positive  and  the  fourth  negative; 
but  for  every  positive  value  of  x  there  may 
be  a  position  of  equilibrium,  that  is,  there 
may  be  three  positions,  in  which  the  body 
may  float  in  equilibrio  with  the  angle  ACB 
downwards ;  but  there  cannot  be  more. 

380.  If  the  sides  b  and  c  are  equal  to  one 
another,  as  represented  in  the  annexed  dia- 
gram, then  cos.<£  and  cos.^'  are  also  equal, 
and  the  general  equation  becomes 


— «*Xas-f. 


?2—  a*  X  x  —  -JT  =  0.     (^30) . 


Now,  it  is  manifest  from  the  relation  of  the  terms  in  this  equation, 
that  it  is  resolveable  into  the  two  quadratic  factors 

b*s  bzs 

a? — -~r,  and  x* — cos.<W462 — cfXx-}-— r,  each  of  which 
«  s 

is  equal  to  nothing ;    consequently,  the  four  roots  of  the  equation 
(230),  are  the  same  as  the  roots  of  the  two  quadratic  equations 


o;2iz:— r>  and  x2 —  cc 
s 

and  the  positions  of  equilibrium  are  indicated  by  the  number  of  real 
positive  roots  which  these  equations  contain. 


300  OF  THE  POSITIONS  OF  EQUILIBRIUM. 

By  extracting  the  square  root  of  both  sides  of  the  equation  or*zz  _  » 
we  shall  obtain 


This  expression  exhibits  two  roots  of  the  original  equation  (230),  one 
positive  and  the  other  negative  ;  but  the  positive  root  only  becomes 
available  in  determining  the  position  of  equilibrium,  the  negative  one 
referring  to  a  case  whjch  does  not  exist. 

It  has  already  been  shown  in  equation  (227),  that  when  a  solid 
body  floats  in  equilibrio  on  a  fluid  of  greater  specific  gravity  than 

itself;  then  we  have 

xys'  —  bcs, 

but  according  to  the  supposition,  b  and  c  are  equal  to  one  another  ; 
hence  we  get 


from  which,  by  division,  we  obtain 

b*s 

y  =  ^' 
or,  by  substituting  the  above  value  of  x,  it  becomes 


.          (232, 

Hence  it  appears,  that  the  values  of  x  and  y  are  each  of  them 
expressed  by  the  same  quantity;  consequently,  the  triangle  DCE  is 
isosceles,  and  AB  the  extant  side  of  the  section,  is  parallel  to  DE  the 
base  of  the  immersed  portion,  both  of  them  being  parallel  to  the  plane 
of  floatation  or  the  horizontal  surface  of  the  fluid. 

381.  The  practical  rule  for  the  reduction  of  the  equation  (231)  or 
(232),  may  be  expressed  in  words  at  length,  in  the  following  manner. 

RULE.  Divide  the  specific  gravity  of  the  solid  body,  by  the 
specific  gravity  of  the  fluid  on  which  it  floats  ;  then,  multiply 
the  square  root  of  the  quotient,  by  the  length  of  one  of  the 
equal  sides  of  the  section,  and  the  product  will  give  the  portion 
of  that  side  which  is  immersed  below  the  plane  of  floatation,  or 
that  which  is  intercepted  between  the  vertex  of  the  section 
and  the  horizontal  surface  of  the  fluid. 

382.  EXAMPLE.  A  prism  of  wood,  the  sides  of  whose  transverse 
section  are  respectively  equal  to  20,  28  and  28  inches,  is  placed 
with  its  vertex  downwards  in  a  cistern  or  reservoir  of  water  whose 


OF  THE  POSITIONS  OF  EQUILIBRIUM.  301 

surface  is  horizontal  ;  it  is  required  to  determine,  what  position  the 
solid  will  assume  when  in  a  state  of  equilibrium,  its  specific  gravity 
being  to  that  of  water  as  686  to  1000  ? 

Here,  by  the  rule,  we  have 


from  which,  by  extracting  the  square  root,  we  get 
^0.686—0.8282; 

and  finally,  by  multiplication,  we  obtain 
x  =  28  X  0.8282  =  23.1896  inches. 

But  according  to  equation  (232),  y  possesses  the  very  same  value  ; 
consequently,  if  23.1896  inches  be  set  off  from  the  vertex  of  the 
section  upwards  on  each  of  its  equal  sides,  the  straight  line  which 
joins  these  points  will  coincide  with  the  plane  of  floatation,  or  the 
horizontal  surface  of  the  fluid  on  which  the  body  floats. 

383.  This  is  the  most  natural  and  obvious  position  of  equilibrium, 
and  such  as  must  always  obtain  when  the  body  is  homogeneous,  and 
symmetrical  with  respect  to  a  vertical  plane  passing  through  the  axis 
and  bisecting  the  base  ;  but  there  may  be  other  situations  in  which 
the  body  may  float  in  a  state  of  quiescence,  and  the  circumstances 
under  which  they  occur  must  be  determined  by  the  resolution  of  the 
following  equation,  viz. 

b*s 


Complete  the  square,  and  we  shall  have 

2—  o8  X  *  -f  (  1 
and  by  extracting  the  square  root,  we  get 


x—    COS.0V—  a  =     :        JCOS2^(463  —  a2)—  -j» 

hence,  by  transposition,  we  shall  obtain,  (233). 

/ ^ 

—  a*^y    J cos2.^(4^—  a2)  —  -, 


The  corresponding  values  of  y  are  (234). 

y  =  \  cos.<£ V462  —  a?±2\/ 


302  OF  THE  POSITIONS  OF  EQUILIBRIUM. 

Expressions  of  this  form,  arising  from  the  reduction  of  an  adfected 
quadratic  equation,  are  in  general  rather  troublesome  and  difficult  to 
render  intelligible  in  words,  and  even  when  intelligibly  expressed,  they 
are  to  say  the  least  of  them,  but  very  dull  and  uninviting  guides,  from 
which  a  tasteful  reader  turns  with  disgust ;  we  are  therefore  unwilling 
to  crowd  our  pages  with  long  and  formal  directions  for  the  purpose  of 
reducing  equations,  when  it  is  probable  after  all,  that  nine  out  of  every 
ten  of  our  readers  will  pass  them  over,  and  proceed  immediately  to 
discover  their  object  by  the  direct  resolution  of  the  original  equation. 

384.  It  is  however  necessary,  in  conformity  to  the  plan  of  our 
work,  to  express  the  most  important  final  equations  in  words  at  length, 
and  since  the  preceding  forms  are  of  considerable  utility  in  the  doctrine 
of  floatation,  it  would  be  a  direct  violation  of  systematic  arrangement, 
to  omit  the  verbal  description,  and  leave  the  subject  open  only  to 
algebraists ;  we  shall  therefore,  in  order  to  render  both  parts  of  the 
operation  intelligible,  endeavour  to  express  the  method  of  reduction 
in  as  brief  and  comprehensive  a  manner  as  the  nature  of  the  subject 
will  admit. 

1 .  To  determine  the  value  of  x. 

RULE.  From  four  times  the  square  of  one  of  the  equal 
sides  of  the  section,  subtract  the  square  of  the  base,  or  side 
opposite  to  the  vertical  angle  ;  multiply  the  square  root  of  the 
remainder  by  one  half  the  natural  cosine  of  half  the  vertical 
angle,  and  call  the  product  ?n. 

From  four  times  the  square  of  one  of  the  equal  sides  of  the 
section,  subtract  the  square  of  the  base,  or  side  opposite  to  the 
vertical  angle,  and  multiply  the  remainder  by  one  fourth  of 
the  square  of  the  natural  cosine  of  half  the  vertical  angle, 
or  that  which  is  immersed  in  the  fluid ;  then,  from  the  product, 
subtract  the  quotient  that  arises,  when  the  specific  gravity  of 
the  solid,  drawn  into  the  square  of  one  of  the  equal  sides 
of  the  section,  is  divided  by  the  specific  gravity  of  the  fluid, 
and  call  the  square  root  of  the  remainder  n. 

Finally,  to  and  from  the  quantity  denoted  by  m,  add  and 
subtract  the  quantity  denoted  by  n ;  then,  the  sum  in  the  one 
case,  and  the  difference  in  the  other,  will  give  the  two  values 
of*. 

2.  To  determine  the  corresponding  values  of  y. 

RULE.  Calculate  the  values  of  m  and  n,  precisely  after  the 
manner  described  above;  then,  from  and  to  the  quantity 


OF  THE  POSITIONS  OF  EQUILIBRIUM.  303 

denoted  by  m,  subtract  and  add  the  quantity  denoted  by  n, 
and  the  difference  in  the  one  case,  and  the  sum  in  the  other, 
will  give  the  values  ofy  corresponding  to  above  values  of  x. 

385.  These  are  the  rules  by  which  the  other  positions  of  equilibrium 
are  to  be  determined  ;  but  it  is  necessary  to  remark,  that  beyond  cer- 
tain limits  no  equilibrium  can  obtain.  In  the  first  place,  in  order  that 
the  body  may  float  with  only  one  of  its  angles  immersed,  it  is  mani- 
festly requisite,  that  the  equal  sides  of  the  section  should  each  be 
greater  than  m  -j-  n  ;  and  in  the  second  place,  in  order  that  x  and  y 
may  be  real  positive  quantities,  the  expression  J  cos2.^>(462  —  a2)  must 


.  tfs         s  ,        cos2.d>(4£2—  a  ) 

exceed  —  —  ,  or  —  must  be  less  than  -  r.  ,,  -  . 
'  2 


reason  of  these  limitations  is  obvious  from  the  nature  of  the 
quadratic  formula  (233)  and  (234),  but  it  will  be  more  satisfactory  to 
show,  that  unless  the  data  of  the  question  are  so  constituted  as  to 
fulfil  these  conditions,  the  rules  will  fail  in  determining  the  positions 
of  equilibrium  ;  or  in  other  words,  there  is  no  other  position  in  which 
the  body  will  float  at  rest,  but  that  which  is  indicated  by  the  equa- 
tions (231)  and  (232). 

386.  EXAMPLE.  The  data  remaining  as  in  the  preceding  example, 
let  it  be  required  to  determine  from  thence,  whether  under  the  pro- 
posed conditions,  the  body  can  float  at  rest  in  any  other  position  than 
that  which  we  have  already  assigned  for  it,  by  the  reduction  of  the 
equations  (231)  and  (232),  in  which  the  extant  side  or  base  of  the 
figure  is  parallel  to  the  horizon. 

By  the  principles  of  Plane  Trigonometry,  we  have 


\  cos.0  n=  TV  v7  28  +  1  0)  (28  —  1  0)  =  !(0.93406)  =  \  cos.20°  55'  29"  ; 

consequently,  by  proceeding  according  to  the  rule,  we'get 
m~\  cos.</>(462—  as)*zz0.46703  ^4  X282—  20*=:  24.429  very  nearly. 

Again,  to  determine  the  value  of  w,  it  is 

i  cos.80(462  —  a2)  •=.  0.467032(4  X282  —  202)  =  596.768, 

and  for  the  value  of  the  term,  involving  the  specific  gravities,  we  have 


consequently,  by  subtraction,  we  get 

596.768  —  537.824  =  58.944. 

It  therefore  appears  from  the  last  result,  that  both  the  values  of  x 
and  y  are  real  positive  quantities  ;  consequently,  one  of  the  limiting 


304  OF  THE  POSITIONS  OF  EQUILIBRIUM. 

conditions  is  answered,  and  we  shall  shortly  see,  whether  or  not  the 
data  are  sufficient  to  satisfy  or  fulfil  the  other  condition. 

By  extracting  the  square  root  of  58.944,  we  get 


w=-V/58.944  =  7.677  nearly; 
therefore,  by  addition  and  subtraction,  the  values  of  x,  are 

a:  =  m  +  72  =  24.429  +7.677  =  32. 106,  and  x  —  m  —  n  —  24.429— 
7.677  zz  16.752  inches. 

Now,  we  have  seen  by  equation  (234),  that  the  corresponding 
values  of  y  are  expressed  in  the  same  terms,  having  the  signs  of  the 
second  member  reversed  ;  hence  we  have 

y  =  16.752,  and  y  =  32.106  inches. 

But  here  we  have  m  -\-  n  zn  32.106  inches,  being  greater  than  b  the 
downward  side  of  the  transverse  section,  which  by  the  question  is  only 
28  inches ;  it  therefore  follows,  that  with  the  proposed  data  and  under 
the  specified  circumstances,  there  is  only  one  position  in  which  the 
body  can  float  in  a  state  of  rest,  and  it  is  that  which  we  have  already 
determined,  where  the  base  of  the  section,  or  the  extant  side  of  the 
body,  is  parallel  to  the  surface  of  the  fluid. 

But  we  may  here  observe,  that  notwithstanding  the  values  of  a:  and 
y,  as  we  have  just  assigned  them,  do  not  satisfy  the  conditions  of  the 
question,  yet  they  are  not  to  be  considered  as  being  useless ;  for  they 
actually  serve,  with  a  slight  modification  of  the  body,  to  furnish  posi- 
tions in  which  it  will  float  at  rest,  although  those  positions  do  not 
agree  with  the  case,  in  which  only  one  angle  of  the  figure  falls  below 
the  plane  of  floatation. 

387.  The  positions  of  equilibrium  corresponding  to  the  preceding 
values  of  a;  and  y,  are 
as  represented  in  the  an- 
nexed diagrams,  where 
E  D  is  the  horizontal  sur- 
face   of  the   fluid,   ABC 
being  the  position  which 
the  body  assumes  when 
x  is  equal  to  32.106  and 
y  equal  to  16.752  inches, 
and  abc  the  correspond- 
ing position  when  y  is  equal  to  32.106  and  x  equal  to  16.752  inches ; 
these  being  the  respective  values  as  obtained  by  the  above  numerical 
process. 


OF  THE  POSITIONS  OF  EQUILIBRIUM.  305 

That  the  positions  here  exhibited  are  those  of  equilibrium,  is  very 
easy  to  demonstrate,  for  produce  the  sides  CA  and  cb  to  meet  the 
surface  of  the  fluid  in  the  points  E  and  D,  and  bisect  AB  and  a  b  in 
the  points  F  and/;  then,  if  the  straight  lines  FE,  FH  and/D,  /i  be 
drawn,  they  will  be  equal  among  themselves. 

This  is  one  of  the  conditions  of  equilibrium,  as  we  have  already 
demonstrated  in  the  construction  of  the  original  diagram,  and  the 
other  condition  is,  that  the  areas  of  the  immersed  figures  ECH  and 
DCI,  are  respectively  to  the  whole  areas  ABC  and  a  be,  as  the  specific 
gravity  of  the  solid,  is  to  the  specific  gravity  of  the  fluid  which  sup- 
ports it. 

Now,  if  the  first  of  these  conditions  obtain,  that  is,  if  the  straight 
line  FE  be  equal  to  FH,  and/D  equal  to/i,  then,  by  the  principles  of 
Plane  Trigonometry,  we  shall  have 

EC2-j-CF2  —  2EC.CFCOS.ECFZTHC*-}-  c^  —  2HC.CF  COS.FCH  ; 

but  the  angles  ECF  and  FCH  are  equal  to  one  another,  and  each  of 
them  equal  to  0  ;  consequently,  by  substituting  the  literal  representa- 
tives, we  have 

xa  -f-  d*  —  2rf x  cos.^>  —  y3-  -)-  d*  —  2dy  cos.0, 
or  by  expunging  the  common  term  e?4,  we  get 

x*  —  2dx  cos.0  z=  ?/*  —  ^dy  cos.0, 
and  this,  by  transposing  and  collecting  the  terms,  becomes 

#2  —  ?/2  zz:  2e?  cos.^(o?  —  y)  ; 

therefore,  jf  both  sides  of  this  equation  be  divided  by  the  factor  (x — y), 
we  shall  obtain 

x  -f-  y  zr:  2rf  cos.^>. 

Now,  by  a  previous  calculation  we  found  x  to  be  equal  to  32.106 
inches,  y  16.752  inches,  d  equal  to  \/288 — 10%  and  cos.  0  equal  to 
0.93406 ;  consequently,  by  substitution,  we  have 

32.106  +  16.752  =  2x0.93406X6/19"; 
hence  the  equality  of  the  lines  FE  and  FH  is  manifest. 
What  we  have  shown  above  with  respect  to  the  triangle  ABC,  may 
also  be  shown  to  obtain  in  the  triangle  a  be,  the  one  being  equal  and 
subcontrary  to  the  other ;  this  being  the  case,  it  is  needless  to  repeat 
the  process;  but  we  have  yet  to  prove,  that  the  area  CEH,  is  to  the 
whole  area  ABC,  as  the  specific  gravity  of  the  floating  body,  is  to  that 
of  the  fluid  on  which  it  floats. 

therefore,  by  the  principles  of  Plane  Trigonometry,  we  get 

AC  :  AF  :  :  rad.  :  sin.ACF, 
VOL.  i.  x 


306  OF  THE  POSITIONS  OF  EQUILIBRIUM 

or  numerically,  we  shall  obtain 

28  :  10  :  :  1  :  sin.«/>=  0.35714, 

and  we  have  already  found  that 

cos.^  =:  V (28  -f  10)  (28  —  10)  -f-  28  =  0.93406  ; 

but  according  to  the  arithmetic  of  sines,  it  is 

sin.20  —  2  sin.0  cos.0, 
and  by  substituting  the  above  numerical  values,  we  get 

\  sin.20  =  0.35714 X 0.93406  =  0.33359. 

Then  in  the  triangle  ECH,  there  are  given  the  two  sides  EC  and  HC, 
respectively  equal  to  32.106  inches  and  16.752  inches,  together  with 
half  the  natural  sine  of  the  contained  angle ;  to  find  the  area  of  the 
triangle. 

Now,  by  the  principles  of  mensuration,  the  area  of  any  plane  tri- 
angle is  expressed  by  half  the  product  of  any  two  of  its  sides,  drawn 
into  the  natural  sine  of  the  contained  angle,  hence  we  get 

32. 106X  16.752  X0.33359  =  179.417  square  inches. 
Again,  in  the  isosceles  triangle  ABC,  there  are  given  the  sides  AC 
and  BC,  respectively  equal  to  28  inches,  and  half  the  natural  sine  of 
the  contained  angle  ACB,  equal  to  0.33359 ;  to  find  the  area. 
Here,  by  the  principles  of  mensuration,  we  have 
28X28X0.33359  zz  261.53456  square  inches; 

then,  by  the  property  of  floatation,  it  is 
1000  :  686  : :  261.53456  :  179.413  square  inches. 

388.  Since  this  result  agrees  so  very  nearly  with  that  derived  from 
a  direct  computation  of  the  triangular  area,  we  may  reasonably  con- 
clude, that  the  positions  exhibited  in  the  diagram  are  those  of  equili- 
brium ;  it  is  however  necessary  to  remark,  that  since  the  weight  of  the 
body  remains  unaltered  in  what  position  soever  it  may  be  situated,  it 
does  not  readily  appear  in  what  manner  the  adequate  quantity  of  fluid 
is  displaced,  unless  we  conceive  some  physical  plane,  of  sufficient 
breadth  and  totally  destitute  of  weight,  to  be  fixed  on  that  edge  of  the 
solid  which  becomes  immersed  by  reason  of  the  change  of  position 
that  the  body  is  supposed  to  undergo. 

This  plane,  during  the  oscillation  of  the  prism,  will  dislodge  the 
fluid  which  occupies  the  space  EAW  or  vbn,  and  the  weight  of  this 
quantity  of  fluid  added  to  that  which  is  displaced  by  the  quadrilateral 
figure  cj  mn  or  cbni,  will  be  equal  to  the  whole  weight  of  the  float- 
ing body 


OF  THE  POSITIONS  OF  EQUILIBRIUM.  307 

389.  The  above  modification,  however,  does  not  strictly  accord  with 
the  conditions  of  the  problem  ;  we  must  therefore  inquire,  whether  the 
just  principles  of  equilibrium  do  not  depend  upon  some  other  element, 
such  as  the  specific  gravity.  Now,  we  have  already  stated,  that  in 
order  to  have  the  values  of  x  and  y  real  positive  quantities,  it  is  neces- 
sary that 

s  ,       coss.0(46a  —  -a9) 

—  should  be  less  than  -  prs  -  » 
s  4kb9 

and  for  a  similar  reason 

s  . 

—  must  be  greater  than 

And  if  the  specific  gravity  of  the  fluid  be  denoted  by  unity,  as  is 
the  case  with  water,  then  the  specific  gravity  of  the  floating  body  must 
lie  between  the  limits 

cos9^(4&g  —  aa)  and  cos.ft  V  46*  —  a9 
4&2  b 

The  specific  gravity  of  the  floating  body,  as  we  have  proposed  it  in 
the  question,  is  686,  that  of  water  being  denoted  by  1000;  conse- 
quently, when  the  specific  gravity  of  water  is  expressed  by  unity,  that 
of  the  solid  is  0.686;  let  us  therefore  try  if  this  number  lies  between 
the  above  limits  ;  for  which  purpose,  we  must  substitute  28  for  b,  20 
for  a,  and  0.93406  for  cos.0  ;  then  we  shall  have  as  follows. 

0.934062(4X28?  —  202) 
For  the  greater  limit  we  have  s  =  -  ;.     OQa  -  -  —  0.761 

4X  ~° 

nearly. 

It  is  therefore  manifest,  that  the  specific  gravity  of  the  floating 
body,  as  we  have  employed  it,  is  less  than  the  greater  limit,  and  con- 
sequently properly  chosen  with  regard  to  it,  and  we  have  next  to 
inquire  if  it  exceeds  the  lesser  limit  ;  for  which  purpose,  it  is 


28 

Here  then  it  is  obvious,  that  the  lesser  limit  exceeds  the  given 
specific  gravity  ;  and  from  this  we  infer,  that  without  the  modification 
specified  above,  the  body  will  not  fulfil  the  conditions  of  the  problem 
in  any  other  position  than  that  in  which  its  base  is  parallel  to  the 
surface  of  the  fluid  ;  but  if  the  specific  gravity  of  the  floating  body 
fall  between  the  numbers  0.761  and  0.745,  all  other  things  remaining, 
then  the  prism,  besides  the  situation  of  equilibrium  in  which  its  base 
is  parallel  to  the  surface  of  the  fluid,  may  have  two  others,  in  both  of 

x2 


308 


OF  THE  POSITIONS  OF  EQUILIBRIUM. 


which  the  conditions  of  the  question  will  be  truly  satisfied,  for  only 
one  angle  of  the  figure  will  fall  below  the  plane  of  floatation. 

In  order  therefore  to  exhibit  those  positions,  we  shall  suppose  the 
specific  gravity  of  the  floating  prism  to  be  expressed  by  0.753,  which 
is  the  arithmetical  mean  between  the  limits  above  assigned  ;  then,  by 
operating  according  to  the  rules  under  equations  (233)  and  (234),  we 
shall  obtain  _ 

—  tf  —  0.46703  ^4x28'—  202rz  24.429  as  for- 


merly computed  ; 


and  after  a  similar  manner,  we  have 


= 


—  a9)  —       = 


0.46703S(4X289  —  20*) 


- 

1UUO 


=z2.528; 


consequently,  by  addition  and  subtraction,  we  shall  get 
x  =  m  -f  n  —  24.429  -f  2.528  =  26.957  inches,  and  x  =  m  —  n  = 

24.429  —  2.528  —  21.901  inches; 
and  the  corresponding  values  oft/,  are 
21.901  and  26.957  inches  respectively. 

390.  The  positions  of  equilibrium  corresponding  to  the  above  values 
of  a:  and  y,  are  as  represented 
in  the  annexed  diagrams,  where 
it  may  be  shown  that  the 
straight  lines  F  E,  F  H  and  /D, 
fi  are  equal  to  one  another, 
and  also  that  the  areas  of  the 
immersed  spaces  E  c  H  and  D  ci 
are  respectively  to  the  whole 
areas  ABC  and  abc,  as  the 
specific  gravity  of  the  solid,  is  to  that  of  the  fluid  on  which  it  floats, 
or  as  0.753  to  unity  in  the  case  of  water. 

These  conditions  being  satisfied,  the  body  will  float  in  equilibrio  in 
the  positions  here  exhibited;  and  it  from  hence  appears,  that  the 
problem  admits  of  a  complete  solution,  by  retaining  the  specific  gra- 
vity of  the  solid  within  determinate  limits. 

391 .  When  the  transverse  section  of  the  floating  prism,  is  in  the 
form  of  an  equilateral  triangle ;  then  a  and  b  are  equal  to  one  another, 
and  equation  (230)  becomes 

—       _       b  s  cos  id)  \/ 3  b^  s* 

r  $'  5/9          > 


OF  THE  POSITIONS  OF  EQUILIBRIUM.  309 

and  if  the  value  of  s'  be  expressed  by  unity,  as  in  the  case  of  water, 

then  we  have 


—  6V  =  0.      (235). 
Now,  it  is  manifest  that  this   equation  is  composed  of  the  two 
quadratic  factors  x*  —  b*  srz:0,  and  .r'^-^cos.^V^Xa;  4-  b*s=:&, 
whose  roots  give  the  positions  of  equilibrium. 

Since  the  sides  a  and  b  are  equal  to  one  another,  and  s'  equal  to 
unity  ;  then,  the  limits  between  which  the  value  of  s  must  be  retained, 
are 

Jcos*.0  and  cos.<j>^~3—l  ; 

but  in  the  case  of  the  equilateral  triangle,  ^  zr  30°  ;  consequently, 
cos.^>  —  J  ^/  3,  and  cos2.0  —  j  ;  therefore,  by  substitution,  the  above 
limits  become 

Tgs=  0.5625,  and  $•  —  lrz:0.5, 

the  arithmetical  mean  of  which,  is 

£(0.5625  +  0.5)  =  0.53125. 

Let  this  value  of  s  be  substituted  instead  of  it,  in  each  of  the  con- 
stituent quadratic  factors,  and  the  equations  whose  roots  determine 
the  positions  of  equilibrium,  become  respectively 

**  —  0.531256%  and  x*  —  bcos.(j>^/3^x  =  —  .531256*; 
but  by  the  property  of  the  equilateral  triangle, 

0  =n  30°,  and  consequently  cos.0  —  \<J  3  ; 
hence,  the  above  adfected  quadratic  equation  becomes, 

x%—  1.56ar  =  —  .5312563. 

392.  If  b  the  side  of  the  triangle  be  equal  to  28  inches,  as  we  have 
hitherto  supposed  it  to  be;  then,  the  preceding  equations  become 

a2  =  416.5,  and  a2  —  42a  =  —  416.5. 

Now,  it  is  manifest,  that  the  first  of  these  equations  has  one  positive 
and  one  negative  root,  each  of  them  being  expressed  by  the  same 
numerical  quantity,  viz.  the  square  root  of  416.5  ;  for  by  extracting 
the  square  root  of  both  sides  of  the  equation,  we  have 

tf  =  d=V/416.5=:±:  20.4083  inches. 

But  according  to  equation  (227),  we  have  xys'~bcs,  where  by 
the  present  supposition,  b  and  c  are  equal  to  one  another,  and  s'  is 
equal  to  unity  ;  therefore,  it  is 

x  y  —  b^s  =416.5; 
hence,  by  division,  we  shall  get 


310  OF  THE  POSITIONS  OF  EQUILIBRIUM 

*  ~  T  =  ±^83  =  -20'4083 

Then,  by  taking  the  positive  values  of  x  and  y  respectively,  the 
position  of  equilibrium  indicated  by  the 
above  results,  is  represented  in  the  an- 
nexed diagram,  where  CE  and  CH  are 
respectively  equal  to  20.4083  inches, 
and  consequently,  AB  the  base  of  the 
section  is  parallel  to  L  K  the  surface  of 
the  fluid. 

Bisect  the  base  AB  in  the  point  F,  and 
draw  the  straight  lines  er,  FE  and  FII  ;  then  because  CE  is  equal  to 
CH,  and  the  angle  ECF  equal  to  the  angle  HCF,  it  follows,  that  the 
line  F  E  is  equal  to  the  line  F  H  ;  this  is  one  condition  that  must  be 
satisfied,  when  the  body  floats  in  a  state  of  quiescence ;  and  another 
is,  that  the  area  of  the  immersed  triangle  ECH,  is  to  the  area  of  the 
whole  section  ACB,  as  the  fraction  0.53125  is  to  unity. 

Now,  by  the  property  of  the  equilateral  triangle,  the  area  of  the 
section  ACB,  is  expressed  by  the  product  of  one  fourth  of  the  square 
of  its  side,  drawn  into  the  square  root  of  the  number  3,  and  the  same 
property  holding  with  respect  to  the  area  of  the  triangle  ECH;  it 
follows,  that  in  the  case  of  an  equilibrium, 

J«V3>  :  i#V3~:  :  0.53125  :  1* 
or  by  suppressing  the  common  quantity  J\/3,  we  have 

x2  :  62  :  :  0.53125  :  1  ; 

but  xz~  416.5,  and  £2~  784;  therefore,  by  substitution,  we  obtain 
416.5  :784  :  :  0.53125  :  1. 

It  is  therefore  evident,  that  by  the  above  results,  both  the  condi- 
tions of  equilibrium  are  satisfied,  and  consequently,  the  body  floats  in 
a  state  of  equilibrium  when  placed  as  represented  in  the  diagram ; 
that  is,  with  20.4083  inches  of  its  side  immersed,  and  its  base  parallel 
to  the  plane  of  floatation. 

393.  The  adfected  quadratic  equation  a2  —  42z:zz  — 416.5,  has 
obviously  two  positive  roots,  each  of  them  less  than  b  the  side  of  the 
section  ;  from  which  we  infer,  that  besides  the  position  of  equilibrium 
above  exhibited,  the  body  may  have  other  two,  and  these  will  be 
determined  by  the  resolution  of  the  equation,  as  follows. 

Complete  the  square,  and  we  obtain 
^  _  42a;  +  212  =  —  416  5  +  441  =  24.5, 


OF  THE  POSITIONS  OF   EQUILIBRIUM.  311 

extract  the  square  root  of  both  sides,  and  we  get 

x  —  21  =  dby7  2475  =  =t  4.95  nearly; 
consequently,  by  transposition,  we  obtain 

x  :r±21  -f  4.95  ==  25.95  inches,  and  x  =  2l  —  4.95  —  16.05  inches; 
and  the  corresponding  values  of  y  are 


Now,  the  positions  of  equilibrium  supplied  by  the  above  values  of 
a;  and  y,  are  as  exhi- 
bited in  the  subjoined 
diagrams,  where  LK  is 
the  surface  of  the  water, 
ABC  the  position  of  the 
body  corresponding  to 
x  equal  25.95  inches, 
and  y  equal  16.05 
inches;  abc  being  the 
position  which  the  solid 

assumes,  when  the  values  of  x  and  y  reverse  each  other  ;  that  is,  when 
y  equal  16.05  inches  and  y  equal  25.95  inches. 

Bisect  A  B  and  a  b  in  the  points  F  and  /,  and  draw  the  straight  lines 
FE,  FH  andyi,yD  to  meet  the  surface  of  the  water  in  the  points  E,  H 
and  i,  D,  the  points  in  which  the  plane  of  floatation  intersects  the  im- 
mersed sides  of  the  solid  ;  then  are  the  lines  FE,  FH  andyi,yD  equal 
among  themselves,  and  the  areas  ECH,  ICD,  are  respectively  to  the 
whole  areas  ABC,  a  b  c  as  the  number  0.53125  to  unity. 

PROBLEM  LVII. 

394.  Suppose  that  a  solid  homogeneous  body,  in  the  form  of 
a  triangular  prism,  floats  upon  the  surface  of  a  fluid  of  greater 
specific  gravity  than  itself,  in  such  a  manner,  that  two  of  its 
edges  shall  fall  below  the  plane  of  floatation  :  — 

It  is  required  to  determine  its  position,  when  it  has  attained 
a  state  of  perfect  quiescence. 

Let  ABC  represent  a  section  perpendicular  to  the  axis  of  a  solid 
homogeneous  triangular  prism,  floating  in  a  state  of  quiescence  on  a 
fluid  whose  horizontal  surface  is  IK;  A  DEB  and  DCE  being  respec- 
tively the  immersed  and  extant  portions. 


312 


OF  THE  POSITIONS  OF  EQUILIBRIUM, 


Now,  it  is  manifest,  that  since  the 
whole  section  ABC,  is  divided  by  DE 
the  line  of  floatation,  into  the  two 
parts  A  DEB  and  DCE;  it  follows, 
that  the  centre  of  gravity  of  the 
section  ABC,  and  the  common  centre 
of  gravity  of  the  two  parts  into  which 
it  is  divided  occur  in  the  same  point ; 
consequently,  the  centres  of  gravity 
of  the  triangular  areas  ABC  and  DEC, 
with  that  of  the  quadrilateral  space 
A  DEB,  are  situated  in  the  same 
straight  line. 

But  by  the  principles  of  floatation  we  know^  that  when  the  solid  is 
in  a  state  of  quiescence,  the  centre  of  gravity  of  the  whole  section 
ABC,  and  that  of  the  immersed  portion  A  DEE  occur  in  the  same  verti- 
cal line;  that  is,  the  vertical  line  passing  through  their  centres  of 
gravity  is  perpendicular  to  the  horizontal  surface  of  the  fluid  ;  and  for 
this  reason,  the  vertical  line  passing  through  the  centre  of  gravity  of 
the  whole  section  ABC,  and  that  of  the  extant  portion  DEC,  is  also 
perpendicular  to  the  horizon. 

Bisect  the  sides  AB,  BC  and  D  E,  EC  in  the  points  F,  n  and  H,  m  and 
draw  the  straight  lines  CF,  ATI  and  CH,  Din  intersecting  two  and  two 
in  the  points  G  and  g,  which  points  are  respectively  the  centres  of 
gravity  of  the  triangular  spaces  ABC  and  DEC. 

Draw  the  straight  lines  Gg  and  FH,  and  because  CF  and  CH  the 
sides  of  the  triangle  CFH,  are  cut  proportionally  in  the  points  o  and 
g,  it  follows,  that  Gg  and  FH  are  parallel  to  one  another;  but  it  has 
been  demonstrated,  that  Gg  is  perpendicular  to  TK  the  horizontal 
surface  of  the  fluid ;  therefore,  FH  is  perpendicular  to  DE  the  line  of 
floatation;  and  since  DE  is  bisected  in  H,  it  follows  that  FD  and  JE 
are  equal  to  one  another. 

Put  a  —  AB,  the  immersed  side  of  the  triangular  section  ABC, 

b  ~  AC,  one  of  the   sides  of  the  triangular  section  which 

penetrate  the  fluid; 

c  :n  BC,  the  other  penetrating  side  of  the  figure, 
d  m  CF,  the  distance  between  the  vertex  of  the  section  and 

the  middle  of  the  immersed  side  ; 
0  :rz  ACF,  the  angle  contained  between  the  side  AC  and  the 

bisecting  line  c  F* 


OF  THE  POSITIONS  OF  EQUILIBRIUM.  313 

ty'  —  BCF,  the  angle  contained  between  the  bisecting  line  CF 

and  the  side  B  c, 

s   zn  the  specific  gravity  of  the  floating  solid, 
s'  :zr  the  specific  gravity  of  the  supporting  fluid, 
x  :zz  CD,  the  extant  portion  of  the  side  AC,  and 
y  —  CE,  the  corresponding  portion  of  the  side  BC. 

Then,  since  the  area  of  any  plane  triangle,  is  expressed  by  the 
product  of  any  two  of  its  sides,  drawn  into  half  the  natural  sine  of 
their  included  angle,  it  follows,  that  the  area  of  the  entire  section 
ABC,  is  expressed  as  under,  viz. 


and  for  the  area  of  the  extant  triangle  DEC,  we  have 


where  the  symbols  a  and  a",  denote  the  areas  of  the  whole  section 
and  the  extant  portion  respectively  ;  consequently,  by  subtraction, 
the  area  of  the  immersed  part  ADEB,  is 

(a1  —  a")  =  i3in.(0  +  f)(bc  —  xy). 

But  by  the  principles  of  floatation,  the  area  of  the  whole  section 
ABC,  is  to  the  area  of  the  immersed  portion  ADEB,  as  the  specific 
gravity  of  the  supporting  fluid,  is  to  the  specific  gravity  of  the  floating 
solid  ;  that  is 

!&csin.(4>4-<£')  :  £sin.(</>  -\-  0')  (be  —  xy)  :  :  s'  :  s  ; 
from  which,  by  casting  out  the  common  terms,  we  get 

be  :  (be  —  xy)  :  :  s'  :  s, 
and  equating  the  products  of  the  extremes  and  means,  it  is 

bcs  —  bcs'  —  xys'  ; 
therefore,  by  transposing  and  collecting  the  terms,  we  obtain 

xys'  =  bc(s'  —  s).  (236). 

Since  the  line  CF  is  drawn  from  the  vertex  of  the  triangle  ABC,  to 
the  middle  of  the  opposite  side  or  base  AB,  it  follows  from  the  prin- 
ciples of  geometry,  that 

AC?4-BC8:=2(AF2-f  CF3)* 

and  this,  by  substituting  the  literal  representatives,  becomes 

62  +  c2=:2(K-f  d2); 
therefore,  by  transposition,  we  have 

4d2zz2(62  +  c2)-a2, 
and  finally,  by  dividing  and  extracting  the  square  root,  we  get 

d—  4^2(^4-  <r)~-a2.'  (237), 


314  OF  THE  POSITIONS  OF  EQUILIBRIUM. 

This  is  the  very  same  expression  for  the  value  of  d  as  that  which  we 
obtained  in  equation  (226),  as  it  manifestly  ought  to  be,  since  the 
same  letters  refer  to  the  same  parts  of  the  figure ;  but  we  have  thought 
proper  to  repeat  the  investigation,  in  preference  to  directing  the 
reader's  attention  to  the  former  result ;  for  by  this  means,  our  per- 
formance is  rendered  more  systematic,  and  the  several  steps  of  the 
operation  are  more  readily  traced  and  applied. 

Now,  in  the  plane  triangle  DFC,  there  are  given  the  two  sides  CD 
and  CF,  with  the  contained  angle  DCF  ;  to  find  the  side  FD. 

Therefore,  by  the  principles  of  Plane  Trigonometry,  it  is 

a?  4-  d2  —  2dx  cos.0  zz  F  o2 ; 

and  in  the  triangle  EFC,  there  are  given  the  two  sides  CE  and  CF,  with 
the  contained  angle  ECF  ;  to  find  the  side  FE. 

Consequently,  as  above,  we  have 

2/2  -f  d2  —  2dy  cos.f  zz  F  E2 ; 

but  we  have  demonstrated,  that  according  to  the  principles  of  floata- 
tion, the  lines  FD  and  FE  are  equal  to  one  another;  therefore,  their 
squares  must  also  be  equal ;  hence,  by  comparison,  we  have 

xz  —  %dx  cos ,(j>  zz  2/2  —  Zdy  cos.0' ; 
or  by  substituting  the  value  of  d,  equation  (237),  we  get 


x*— cos.0  V  2(63+c2)— a3  X  zzrz/9— cos.f  V  2(^4-c2)— a2  X  y .     (238). 
If  both  sides  of  equation  (236)  be  divided  by  the  expression  #/,  we 
shall  obtain 

bc(s'  —  s) 

y=—^—- 

consequently,  by  involution,  we  have 

y^  °  j>s~s  - 

Let  these  values  of  y  and  y*  be  substituted  instead  of  them  in  equa- 
tion (238),  and  we  shall  have 

^bc(s'— s)cos.<j>' 
xs 


and  multiplying  all  the  terms  by  x  ,  we  get 

6V(s'  —  s)2       bc(s  — 
--    —      -- 


and  finally,  by  transposition,  we  have 


OF  THE  POSITIONS  OF  EQUILIBRIUM.  315 


)  —  a2 

_62cV-5)2 

"I72""  (239). 

395.  The  above  is  the  general  equation,  whose  roots  give  the  several 
positions  in  which  the  solid  may  float  in  a  state  of  equilibrium  ;  it  is 
similar  to  equation  (229),  having  (5'  —  s)  instead  of  s,  and  (s'  —  sf 
instead  of  s2  ;  the  body  may  therefore  have  three  positions  of  equili- 
brium, but  it  cannot  have  more,  the  very  same  as  in  the  case,  where 
it  floated  with  only  one  of  its  edges  below  the  surface  of  the  fluid. 

The  method  of  applying  the  general  equation  to  the  determination 
Of  the  positions  of  equilibrium,  is  to  calculate  the  value  of  d,  cos.0 
and  cos.^>'  from  the  given  dimensions  of  the  section,  and  to  substitute 
the  several  given  and  computed  numbers  instead  of  their  symbolical 
equivalents  ;  this  will  give  a  numeral  equation  of  the  fourth  degree, 
which  may  be  reduced  either  by  approximation  or  otherwise,  accord- 
ing to  the  fancy  of  the  operator. 

396.  EXAMPLE.  Suppose  a  solid  homogeneous  triangular  prism, 
the  sides  of  whose  transverse  section  are  respectively  equal  to  28,  23 
and  18  inches,  to  float  in  equilibrio  on  a  cistern  of  water  with  two  of 
its  edges  immersed  ;  it  is  required  to  determine  the  positions  of  equi- 
librium, on  the  supposition  that  the  two  longest  sides  of  the  section 
include  the  extant  angle,  the  specific  gravity  of  the  prism  being  to 
that  of  water,  as  565  to  1000  ? 

In  order  to  resolve  this  question,  we  must  first  of  all  determine  the 
length  of  the  line  cr,  which  is  drawn  from  the  extant  angle  at  c  to  the 
middle  of  the  opposite  side  AB  ;  for  which  purpose,  let  the  dimensions 
of  the  section  be  respectively  substituted  according  to  the  combination 
exhibited  in  equation  (237),  and  we  shall  have 

d  =  \  V2(282  +  232)  —  1  82  —  \  v/  2302  =  23.99  inches  nearly. 

Consequently,  in  the  triangles  ACF  and  BCF  respectively,  we  have 
given  the  three  sides  AC,  AF,  FC  and  BC,  BF,  FC  to  find  cos.  ACF  and 
cos.  BCF;  for  which  purpose,  the  elements  of  Plane  Trigonometry 
supply  us  with  the  following  equations,  viz. 

In  the  triangle  ACF,  it  is 


and  in  the  triangle  BCF,  it  is 


316  OF  THE  POSITIONS  OF  EQUILIBRIUM. 

Therefore,  by  substituting  the  respective  values  of  a,  b  and  c,  as 
given  in  the  question,  and  the  value  of  d  as  computed  above,  we  shall 
have  the  following  values  of  cos.0  and  cos.^'. 

Thus,  for  the  absolute  numerical  value  of  cos.0,  it  is 

4(28*  +  23.99*)—  182_A 

COS'*  =  --  8X28X23.99        ~  °'95166' 

and  for  the  corresponding  value  of  cos.  0',  we  have 

4(232-4-23.992  —  182 
COS-»  =        8X23X23.99      =°-92747' 

Having  ascertained  the  numerical  values  of  cos.  0  and  cos.^>',  let  the 
respective  quantities  be  substituted  in  equation  (239),  and  it  becomes 

28X23X435X0.92747 


4  —  0. 


1000 
282X232X4352 


10002 
from  which,  by  computing  the  several  terms,  we  get 

*4  —  45.66*3  -f  12466*  =  78478.36. 

The  root  of  this  equation  will  be  most  easily  discovered  by  approxi- 
mation, and  for  this  purpose,  we  shall  adopt  the  method  of  trial  and 
error,  which  Dr.  Hutton  has  so  successfully  applied  to  the  resolution 
of  every  form  and  order  of  equations,  however  complicated  may  be 
their  arrangement. 

By  a  few  simple  trials,  indeed  it  is  almost  self  evident,  that  the 
value  of  x  will  be  found  between  15  and  16;  consequently,  by  sub- 
stitution we  obtain 

154  —  45.66X158  4-  12466X15  —  78478.36  =  15113.64  too  little. 
164  —  45.66X163  +  12466X16  —  78478.36  =     509.72  too  great. 
Here  it  is  manifest  that  the  errors  are  of  different  affections,  the  one 
being  in  defect  and  the  other  in  excess  ;  hence  we  have 

15113.64  +  509.72  :  16  —  15  ::  509.72  :  0.032; 
consequently,  for  the  first  approximation,  we  get 

x  =  16  —  0.032  =  15.97  very  nearly. 

Supposing  therefore,  that  x  lies  between  15.9  and  16  ;  by  repeating 
the  process,  we  shall  have 

164  —  45.66X   163  -f  12466X  16  —78478.36  =  509.72  too  great. 
15.94  __  45.66  X15.93  4-  12466  X  15.9—  78478.36  zz!05.393too  little. 
Here  again,  the  e/rors  are  of  different  affections,  the  one  being  in 
excess  and  the  other  in  defect;  consequently,  we  have 


OF  THE  POSITIONS  OF  EQUILIBRIUM.  317 

509.72-1-105.393  :  16—15.9  :  :  105.393  :  0.017  nearly; 
therefore,  the  second  approximate  value  of  x,  is 

x  =  15.9  +  0.017  =  15.917  inches. 

By  again  repeating  the  process,  a  nearer  approximation  to  the  true 
value  of  x  would  be  obtained,  but  the  above  is  sufficiently  accurate 
for  our  present  purpose  ;  therefore,  let  this  value  of  x,  together  with 
the  numerical  values  of  b,  c,  s  and  s',  be  substituted  in  equation  (236), 
and  we  shall  obtain 

159172/m  280140, 
and  from  this,  by  division,  we  get 
280140 


397.  And  the  position  of  equilibrium  corresponding  to  the  above 
values  of  x  and  z/,  is  represented  in  the 
annexed  diagram,  where  IK  is  the  hori- 
zontal surface  of  the  fluid,  ABED  the  im- 
mersed part  of  the  section,  and  DCE  the 
extant  part. 

Bisect  AB  in  F,  and  draw  the  straight 
lines  CF,  FD  and  FE;  then,  as  we  have 
previously  demonstrated,  when  the  body 
floats  in  a  state  of  equilibrium,  the  lines 
FD  and  FE  are  equal  to  one  another. 

Now,  in  order  to  determine  if  this  equality  obtains,  we  must  have 
recourse  to  equation  (238),  where  we  have 


c*)  —  «'X  x  =  t/2—  cos. 
then,  let  the  computed  values  of  or,  y,  cos.0,  cos.^',  and  the  given 
values  of  a,  b  and  c,  be  substituted  instead  of  them  in  the  above  equa- 
tion, and  we  shall  obtain 

15.9172  — 0.95166  v/2302  X  15.917  — 17.62—  0.92747 /2302X  17.6, 
and  this,  by  transposition  and  reduction,  gives 

—  726.77  =  56.43  —  783.20. 

398.  Another  condition  of  equilibrium  is,  that  the  area  of  the  im- 
mersed part  ABED,  is  to  the  area  of  the  whole  section  ABC,  as  the 
specific  gravity  of  the  solid  is  to  that  of  the  supporting  fluid.  This  is 
a  more  necessary  condition  than  the  equality  of  the  lines  FD,  FE;  for 
such  an  equality  may  exist  when  no  equilibrium  obtains ;  but  it  may 
be  considered  as  a  universal  fact,  that  whenever  the  two  conditions  are 
satisfied  at  the  same  time,  the  body  floats  in  a  state  of  quiescence. 


318  OF  THE  POSITIONS  OF  EQUILIBRIUM. 

We  have  already  found  that  cos.  4>zz  0.95  166,  and  cos.0'=  0.92747  ; 
consequently,  ^=17°  53',  and  <j>'  =  21°  57'  ;  hence  we  have  (<£  -f-  0') 
zr39°  50',  and  by  the  principles  of  mensuration,  we  get 

a'  =  1(28  X  23)  sin.39°  50'  =  322  X  0.64056  =  206.26032, 

and  the  area  of  the  extant  part,  is 

a"  =  J(15-917  X  17.6)  sin.39°  50'  =  140.0696  X  0.64056  =  89.72298  ; 

therefore,  by  subtraction,  the  area  of  the  immersed  part  becomes 

(a'  —  a")  =  206.26032  —  89.72298  =  1  16.53734  ; 

consequently,  by  the  principle  of  floatation,  it  is 

206.26032  :  116.53734  :  :  1000  :  565  nearly; 

from  which  it  appears,  that  both  the  conditions  of  equilibrium  are 
satisfied,  and  therefore  the  body  as  exhibited  in  the  figure  indicates  a 
state  of  quiescence. 

399.  By  finding  the  other  roots  of  the  equation,  other  situations  of 
equilibrium  may  be  assigned  ;  but  since  the  one  above  given  is  that 
which  would  be  adopted  in  practice,  we  consider  that  it  would  be  a 
waste  of  both  labour  and  time  to  search  after  the  others  ;  we  therefore 
leave  the  reduction  of  the  resulting  cubic  equation  for  exercise  to  the 
reader,  presuming  that  he  will  find  his  trouble  and  attention  amply 
repaid,  by  the  satisfaction  to  be  derived  from  the  confirmation  of  the 
principles  by  an  actual  construction. 

400.  When  the  triangle  ABC  becomes  isosceles;  that  is,  when  the 
sides  b  and  c  are  equal  to  one  another  ;  then  cos.0  and  cos.0'  are 
also  equal,  and  the  general  equation  (239),  becomes  transformed  into 

b\s'  —  s)cos.d>    —  -  .  b\s'—  s)2 

2       2 


— 

or  by  transposing  the  absolute  given  quantity  -  -  ^  -  ,  we  get 

s 


(240). 

Now,  by  carefully  examining  the  nature  of  this  equation,  it  will 
immediately  appear  to  be  composed  of  the  two  quadratic  factors 

b\s'—s)  _  W—  s} 

*2  --    r—  •  =0,  and*2—  cosV4£2—  a2X*4--^—  "         ' 


where  it  is  manifest,  that  each  of  these  expressions  involve  two  roots 
of  the  original  equation,  and  the  number  of  the  real  positive  roots, 
indicates  the  number  of  positions  in  which  the  body  may  float  in  a 


OF  THE  POSITIONS  OF  EQUILIBRIUM.  319 

state  of  quiescence,  while  the  absolute  values  of  the  roots  determine 
the  positions  themselves. 

401.  Let  each  of  the  above  quadratic  factors  be  transformed  into 

an  equation,  by  transposing  the  given  term  --^ — -,  and  we  shall 

obtain 

<_yy—<)  2 b\s'— s) 


and  when  the  value  of  s',  or  the  specific  gravity  of  the  supporting  fluid 
is  expressed  by  unity,  as  is  the  case  with  water  ;  then,  we  have  for 
the  pure  quadratic, 

xz—  £2(1—  s).  (241). 

and  for  the  adfected  quadratic,  it  is 
^2  ___  cos^  ^  4#!  _  a2  x  x  __  „  #(i_s).  (242). 

Let  the  square  root  of  both  sides  of  equation  (241)  be  extracted, 
and  we  shall  obtain 

x  =  b^/T^7;  (243). 

but  from  equation  (236),  we  have 

xy  =  b\l—  s), 

and  this,  by  substituting  the  above  value  of  x,  becomes 
b<jT^~sXy=ib\\  —  s); 
hence,  by  division,  we  get 
_61  —  * 

- 


Here  then  it  is  manifest,  that  the  values  of  x  and  y  are  each 
expressed  by  the  same  quantity  ;  from  which  we  infer,  that  the  solid 
floats  in  a  state  of  equilibrium,  when  the  base  of  the  section  is  parallel 
to  the  surface  of  the  fluid  ;  that  is,  when  the  extant  portion  of  the 
section  is  also  isosceles,  having  its  base  coincident  with  the  plane  of 
floatation. 

402.  The  practical  rule  for  computing  the  equation  (243)  or  (244), 
may  be  expressed  in  words  at  length,  as  follows. 

RULE.  From  unity,  or  the  specific  gravity  of  the  fluid, 
subtract  the  specific  gravity  of  the  floating  solid,  and  multiply 
the  square  root  of  the  difference  by  one  of  the  equal  sides  of 
the  section,  and  the  product  will  express  the  value  of  x  and  y. 

403.  EXAMPLE.  Suppose  the  two  equal  sides  of  the  section  to  be 
respectively  equal  to  28  inches,  the  base  18  inches,  and  the  specific 


320 


OF  THE  POSITIONS  OP  EQUILIBRIUM. 


gravity  of  the  solid  0.565  as  in  the  preceding  example ;  how  much  of 
the  equal  sides  is  immersed  in  the  fluid,  and  how  much  is  extant,  the 
body  being  in  a  state  of  quiescence  ? 

Here,  by  the  rule,  we  have 

l—s=  1  —  0.565  =  0.435, 

the  square  root  of  which  is 

v/ 0.435  zz  0.659; 

consequently,  by  multiplication,  we  have 
*or2/zz28x0.659:z:  18.452  inches. 

404.  And  the  position  of  equilibrium  indicated  by  the  above  value 
of  x  and  y,  is  as  represented  in  the  subjoined 

diagram,  where  IK  is  the  horizontal  surface 
of  the  fluid,  ABED  the  immersed  portion  of 
the  section,  and  DCE  the  extant  portion,  DE 
being  the  water  line  or  plane  of  floatation. 

Since  CD  and  CE  are  each  equal  to  18.452 
inches,  it  follows,  that  AD  and  BE  are  each 
equal  to  28  —  18.452  zz  9.548  inches,  the 
extant  part  of  the  equal  sides  being  nearly 
double  of  the  immersed  part. 

Bisect  A  B  in  the  point  F,  and  draw  the  straight  lines  F  D  and  F  E  ; 
then  shall  FD  and  FE  be  equal  to  one  another;  this  is  manifest,  for 
AF,  BF  and  AD,  BE  are  equal,  and  the  angle  DAF  is  equal  to  the 
angle  EBF  ;  therefore,  FD  is  equal  to  FE. 

By  examining  the  nature  of  the  equation  (243)  or  (244),  it  is 
manifest  that  the  values  of  x  and  y  depend  entirely  on  the  value  of  s, 
or  the  specific  gravity  of  the  floating  body  ;  now,  since  this  may  admit 
of  all  magnitudes  between  zero  and  unity,  which  is  the  specific  gravity 
of  water,  it  follows,  that  x  and  y  may  be  of  all  magnitudes  between 
zero  and  28  inches  ;  but  whatever  may  be  the  magnitude  of  the  extant 
sides,  the  position  in  which  the  body  floats  will  be  the  same ;  viz.  that 
in  which  the  base  of  the  section  is  parallel  to  the  surface  of  the  fluid. 

405.  Admitting  the  specific  gravity  of  the  solid  to  fall  within  the 
limits  of  possibility,  the  formula  equation  (242),  when  reduced,  will 
supply  us  with  other  two  positions  of  equilibrium,  in  which  the  body 
may  float  with  two  of  its  angles  immersed  ;  here  follows  the  reduction 
of  the  equation. 

Complete  the  square,  and  we  obtain 

2— a2)— b\\— s), 


OF  THE  POSITIONS  OF  EQUILIBRIUM*  321 

and  by  extracting  the  square  root,  it  is 


«2)  —  b\l—  s), 
and  finally,  by  transposition,  we  get 


2  —  a)  —  #2(1  —  *);     (245). 
the  corresponding  values  of  y  being 


•  —  a?)—b\l—s}.  (246). 
In  order  to  satisfy  the  conditions  implied  in  the  foregoing  equations, 
it  is  requisite  that  the  value  of  s,  the  specific  gravity  of  the  floating 
solid,  should  fall  between  the  limits  indicated  by  the  following  expres- 
sions, viz. 

— -rjz '  ana — — , j 

now,  by  the  principles  of  Plane  Trigonometry,  we  have 

b  :  g  \/462 —  a*  :  :  rad.  :  cos.0, 

which  being  reduced,  gives 


and  by  involution,  we  obtain 


Let  these  values  of  cos.<£  and  cos2.^  be  substituted  in   the  above 
expressions  for  the  limits  of  5,  and  we  shall  get  for  the  greater  limit, 


and  for  the  lesser  limit,  it  is 

__(862  —  a*)X«8 

1664 

and  the  arithmetical  mean  of  these  two  limits,  is 
_ 

Then,  if  this  value  of  s  be  substituted  instead  of  it  in  the  equations 
(245  and  246),  we  shall  obtain  for  the  values  of  x,  as  follows, 


2    /4/2     2\     j.2/1     (,100 — ft  )a  V   /048) 

VOL.  I.  Y 


322 


OF  THE  POSITIONS  OF  EQUILIBRIUM. 


and  similarly,  the  corresponding  values  of  y  are 


The  practical  rule  for  reducing  the  above  equations  must  be  omitted 
in  this  case,  the  forms  being  too  complex  to  admit  of  a  clear  and  com- 
prehensive description  ;  it  is  however  presumed,  that  the  attentive 
reader  will  be  enabled  to  understand  the  method  of  solution,  by  care- 
fully tracing  the  several  steps  of  the  process  as  exhibited  in  the  fol- 
lowing arrangement. 

406.  EXAMPLE.  If  the  dimensions  of  the  section  be  the  same  as  in 
the  preceding  example  ;  then,  the  value  of  s  as  computed  from  equa- 
tion (247),  becomes 


__(16X282—  18a)Xl8*__  3959280 


—  0.2013  nearly. 


32X284  19668992 

Therefore,  let  the  mean  value  of  s  as  thus  determined,  together 
with  the  numerical  values  of  a2,  b*,  cos.0  and  cos2.^,  be  substituted  in 
equation  (248),  and  we  shall  have,  in  the  case  of  the  positive  sign, 


*i=0.47347.v/4X28*— 188-f  ^0.22417(4  X282— 18*)— 0.7989X28* 
—  27.152  inches, 

and  in  the  case  of  the  negative  sign,  it  is 


a:  =:  0.47347 -v/4X28*—182—V0.22417(4X282—182)— 0.7989x28* 

HZ  23.06  inches, 

and  the  corresponding  values  of  y,  are 
t/— 23.06,  and  y  — 27.152  inches. 

407.  Now,  the  positions  of  equilibrium  corresponding  to  the  above 
values  of  x  and  ?/,  are  as 
denoted  in  the  subjoined 
diagram  ;  where  IK  is  the 
horizontal  surface  of  the 
fluid,  ABC  the  position  cor- 
responding to  x  =  27. 152 
and  y  zz:  23.06  inches,  and 
abc  the  position  indicated 
by  x  z=  23.06  "and  y  = 
27.152  inches. 

Bisect  AB  and  ab  in  the  points  F  andy,  and  draw  the  straight 
lines  FD,  FE  and  fd,  fe  intersecting  the  plane  of  floatation  in  the 
points  D,  E,  and  d,  e ;  then  shall  the  lines  so  drawn  be  equal  among 


OF  THE  POSITIONS  OF  EQUILIBRIUM.  323 

themselves.    This  is  very  easily  verified,  for  by  the  principles  of  Plane 
Trigonometry,  we  know,  that  when  two  sides  of  a  plane  triangle  are 
given  together  with  the  contained  angle  ;  then,  the  square  of  the  side 
opposite  to  the  given  angle,  is  expressed  as  follows,  viz. 
In  the  triangle  DCF,  it  is 

D  F2  IZ:  D  C2  -|-  C  F2  -  2DC.CFCOS.DCF, 

and  in  the  triangle  ECF,  it  is 

EF2HrEC2  -f-  CF2  -  2EC.CFCOS.ECF. 

But  by  the  construction,  the  angles  DCF  and  ECF  are  equal  to  one 
another,  and  consequently,  cos.  DCF  =  cos.  ECF;  therefore,  by  sub- 
stituting the  analytical  values  of  the  several  quantities,  the  above 
expressions  become 

D  F*  =z  x*  -\-d?  —  2dx  cos.0,  and  E  F2  zz  yz  -f-  dz  —  2dy  cos.0  ; 
but  when  the  body  floats  in  a  state  of  equilibrium,  these  are  equal, 
hence  we  have 

x*  —  2dx  cos.0  zz  y*  —  %dy  cos.<£, 
and  from  this,  by  transposition,  we  obtain 

2t?  cos.(p(x  —  r/)  zz  a;2  —  y*  ; 
therefore,  by  division,  we  have 


Now,  we  have  seen  by  the  preceding  solution,  that  a; zz 27. 152,  and 
y  zz23.06  inches  ;  consequently,  by  substitution  we  get 

2d  cos.0  zz  27. 152  -f  23.06  zz  50.212  ; 
but  by  the  property  of  the  right  angled  triangle,  it  is 


and  we  have  already  seen,  that 


Let  the  numerical  values  of  a2,  b  and  62  be  substituted  in  each  of 
these  expressions,  and  we  shall  have 

d=  i  V/4X282—  182=z>/703~26.514, 
and  similarly,  for  cos.^>,  we  get 


cos.0  =  — -  V  4  X  28*  —  1 82  r=  0.94693  ; 
oo 

consequently,  by  substitution,  we  obtain 
zz  26.514X0.94693X2  ~ 50.212. 
Y  2 


324  OP  THE  POSITIONS  OF  EQUILIBRIUM. 

408.  The  expression  for  the  area  of  the  immersed  figure  ABED,  is 
^sin.2^>(62  —  xy},  and  the  expression  for  the  area  of  the  whole  section 
ABC,  is  \tf  sin.  2^  ;  and  by  the  principles  of  floatation,  these  are  to  one 
another,  as  the  specific  gravity  of  the  floating  solid,  is  to  that  of  the 
fluid  on  which  it  floats  ;  hence  we  have 


—  xy)  :  i&2sin.20  :  :  0.2013  :  1, 
and  by  suppressing  the  common  term  ^sin.2^,  we  get 
{tf  —  xy}  :  62::  0.2013  :  1, 

and  from  this,  by  putting  the  product  of  the  extreme  terms,  equal  to 
the  product  of  the  means,  we  obtain 


and  finally,  by  substituting  the  numerical  values,  we  have 
27.  152X23.06  =  0.7987  X282  very  nearly, 

which  satisfies  the  other  condition  of  equilibrium  ;  hence  we  infer,  that 
the  subcontrary  positions  represented  above,  are  those  which  the  body 
assumes  when  floating  in  a  state  of  quiescence  with  two  of  its  angles 
below  the  plane  of  floatation. 

409.  When  a,  b  and  c  are  equal  to  one  another  ;  that  is,  when  the 
triangular  section  is  equilateral;  then,  the  general  equation  (239), 
becomes 


b\s'—  *)* 
and  from  this  equation,  by  transposing  the  given  term  --  pj  --  ,   we 

get 

8'- 


s 

Now,  it  is  manifest,  that  the  equation  in  its  present  form,  is  com- 
posed of  the  two  quadratic  factors 


— 

and  these  factors,  by  transposing  the  given  term  -  —,  --  in  each, 
become  transformed  into  the  following  quadratic  equations,  viz. 


OF  THE  POSITIONS  OF  EQUILIBRIUM. 


325 


and  supposing  the  specific  gravity  of  the  fluid,  or  the  value  of  s'  to  be 
expressed  by  unity  ;  then,  these  equations  become 

'Resolving  these  equations  by  the  rules  which  the  writers  on  algebra 
have  laid  down  for  that  purpose,  we  shall  have  for  the  pure  quadratic, 

and  ?/zz:6\/(l  —  s}; 
and  again,  for  the  adfected  form,  it  is 


(251). 


and  y—  b {0.866 cos.^v/ 0.75  cos8.^  — (1  —  r^  f  ^ 
In  the  equations,  (251),  it  is  obvious  that  the  values  of  a;  and  y  are 
assignable,  whatever  may  be  the  value  of  s,  provided  that  it  is  less  than 
unity ;  and  since  x  and  y  are  each  expressed  by  the  same  quantity,  it 
follows  that  they  are  equal  to  one  another,  and  consequently  the  body 
will  float  in  equilibrio,  when  the  immersed  side  or  base  of  the  section 
is  parallel  to  the  surface  of  the  fluid. 

410.  EXAMPLE.  If  the  floating  prism  be  of  fir  from  the  forest  of 
Mar,  of  which  the  specific  gravity  is  0.686,  that  of  water  being  unity; 
then  we  have 


and  if  the  value  of  b,  or  the  side  of  the  equilateral  triangle  be  28  inches, 
we  get  xi=y  —  0.56X28  =  15.69  inches; 

and  the  position  of  equilibrium  corresponding  to  this  common  value 
of  a;  and  y,  is  exhibited  in  the 
annexed  diagram,  where  IK  is 
the  horizontal  surface  of  the  fluid, 
DCE  being  the  extant  portion  of 
the  floating  body,  and  ABED  the 
part  immersed  below  the  plane  of 
floatation  ;  c  D  and  c  E  being  re- 
spectively equal  to  15.69  inches. 
Bisect  AB  in  F,  and  draw  the 
straight  lines  FD  and  FE  to  inter- 
sect the  surface  of  the  fluid  in  the  points  D  and  E  ;  then,  because  the 
triangle  ABC  is  equilateral,  and  CD  equal  to  CE  by  the  construction, 
it  follows,  that  FD  and  FE  are  equal  to  one  another;  this  satisfies  one 
of  the  conditions  of  equilibrium,  and  we  have  now  to  inquire  if  the  area 
of  the  immersed  portion  ABED,  is  to  the  area  of  the  whole  section  ABC, 
as  the  fraction  0.686  is  to  unity. 


\|/ i==~=- 


326  OF  THE  POSITIONS  OF  EQUILIBRIUM. 

Now,  by  the  principles  of  mensuration,  we  know  that  the  area  of  a 
plane  triangle,  of  which  the  three  sides  are  equal,  is  expressed  by  one 
fourth  of  the  square  of  the  side,  drawn  into  the  square  root  of  the 
number  3  ;  consequently,  the  area  of  the  whole  section  ABC,  is 


and  the  area  of  the  extant  part  DEC,  is 


therefore,  the  area  of  the  immersed  part  ABED,  is 

(a'  —  a")  =  JftV  3"—  i*V~3  =  0-433  (fi»  —  x2)  ; 

hence,  by  the  principles  of  floatation,  we  get 

0.433  (&•  —  *')  :  0.43362  :  :  0.686  :  1, 
and  by  equating  the  products  of  the  extremes  and  means,  it  is 

x*  =  b*(\  —  0.686)  —  0.31462. 

But  b  is  28  and  x  15.69  inches  ;  therefore,  if  these  values  of  b  and 
x  be  substituted  instead  of  them  in  the  preceding  equation,  we  shall 

have 

15.69*  nr0.314x282=r  246.176. 

In  this  case  also,  one  of  the  conditions  of  equilibrium  is  satisfied  ; 
hence  we  conclude,  that  the  position  which  we  have  represented  above 
is  the  true  one,  since  both  the  conditions  upon  which  the  equilibrium 
depends,  have  been  fulfilled  by  the  results  as  obtained  from  the  reduc- 
tion of  the  formula. 

The  value  of  x  and  y,  as  exhibited  in  equations  (252),  will  indi- 
cate two  other  positions  of  equilibrium,  subcontrary  to  each  other  ; 
but  in  order  that  those  positions  may  be  coiisistent  with  the  conditions 
of  the  problem,  it  becomes  necessary  to  assign  the  limits  of  s,  or  the 
specific  gravity  of  the  floating  body  ;  for  it  is  manifest,  that  beyond 
certain  limits,  the  conditions  specified  in  the  problem  cannot  obtain. 

411.  Now,  in  the  case  of  the  isosceles  triangle,  it  has  been  shown, 
that  the  greater  limit  of  the  specific  gravity,  is 


and  consequently,  when  the  triangle  is  equilateral, 
s=i~  -ft  =  0.5; 

and  moreover,  it  has  also  been  shown,  that  when  the  triangle  is 
isosceles,  the  lesser  limit  of  the  specific  gravity,  is 


OF  THE  POSITIONS  OF  EQUILIBRIUM. 

which,  when  the  triangle  is  equilateral,  becomes 

7 

s  =  —=0.4375, 
16 

1   and  the  arithmetical  mean  of  these,  from  equation  (247),  is 


327 


Let  therefore  this  value  of  s  be  substituted  instead  of  it  in  the 
expressions,  class  (252),  and  we  shall  obtain 

x  =  25.95,  and  x  =  16.05  inches, 

the  corresponding  values  of  y  being 

y  =z  16.05,  and  y  =.  25.95  inches. 

412.  The  positions  of  equilibrium,  as  indicated  by  these  values  of 
x  and?/,  are  as  represented  in  the  annexed  diagrams,  where  IK  is 
the  horizontal  surface  of  the  fluid,  ABED,  abed  the  immersed,  and 
DEC,  dec  the  extant  portions  of  the  section  corresponding  to  the 
positions  ABC  and  abc,  in  which  CD  and  ce  are  each  equal  to  25.95 
inches,  and  CE,  cd  equal  to  16.05  inches,  being  the  respective  values 
of  x  and  y,  as  determined  from  equation  (252). 


Bisect  A  B  and  a  b  in  the  points  F  and  f,  and  draw  the  straight  lines 
FD,  FE  and  fd,  fe  intersecting  the  horizontal  surface  of  the  fluid  in 
the  points  D,  E  and  d,  e ;  then,  when  the  body  floats  in  a  state  of  equi- 
librium, the  lines  FD,  FE,/C?  and/e  are  equal  among  themselves. 

This  is  very  easily  proved,  for  since  the  triangle  ABC  is  equilateral, 
the  angle  ACB  is  equal  to  sixty  degrees,  and  consequently  its  half,  or 
the  angles  ACF  and  BCF  are  each  of  them  equal  to  thirty  degrees; 
therefore,  by  the  principles  of  Plane  Trigonometry,  we  have 

DF8=CD2-4-CF? 2CD.CFCOS.30°, 

and  similarly,  by  the  same  principles,  we  get 

F  E2  HZ  C  E4  -4-  C  F2  —  2C  E.C  F  COS. 30°  ; 


328  OF  THE  POSITIONS  OF  EQUILIBRIUM. 

but  according  to  the  conditions  of  equilibrium,  these  are  equal,  hence 
we  have 

CD2  —  2C  D.C  F  COS.300  =  C  E2  —  2CE.CF  COS.300  ; 

therefore,  by  substituting  the  analytical  expressions,  and  transposing, 
we  get 

x3  —  z/2  =  2d  cos.30°(a?— y\ 

and  dividing  both  sides  by  (x  —  y),  we  shall  have 
2dcos.30°  =  a:-f  y. 

By  Plane  Trigonometry  cos. 30°  nr  sin. 60°,  and  by  the  property  of 
the  equilateral  triangle,  we  have  e?  =  b  sin. 60° ;  consequently,  by  sub- 
stitution, we  get 

2&sin2.60°~  x  +  y; 

or  numerically,  we  obtain 

2X28x1  =  25.954-16.05  —  42. 

413.  Hence  it  appears,  that  in  so  far  as  the  equilibrium  of  floatation 
depends  upon  the  equality  of  the  lines  FD  and  FE,  the  condition  is 
completely  satisfied,  and  the  same  may  be  said  respecting  the  lines 

fd  and/e  ;  but  it  is  manifest,  that  another  condition  must  be  fulfilled 
before  the  body  attains  a  state  of  perfect  quiescence,  and  that  is,  that 
the  area  of  the  immersed  part  ABED,  is  to  the  area  of  the  whole  section 
ABC,  as  the  specific  gravity  of  the  solid  body,  is  to  that  of  the  fluid  on 
which  it  floats,  or  as  0.46875  to  unity  :  now,  this  condition  is  evidently 
satisfied,  when 

x  y  =  P(l—  0.46875), 

therefore,  numerically  we  obtain 

25.95X16.05  — 282X0.53125i=41.65. 

Here  then,  both  the  conditions  of  equilibrium  are  satisfied,  and  from 
this  we  infer,  that  the  positions  exhibited  in  the  diagram  are  the  true 
ones,  the  downward  pressure  of  the  body  in  that  state,  being  perfectly 
equipoised  by  the  upward  pressure  of  the  fluid. 

414.  What  we  have  hitherto  done  respecting  the  positions  of  equi- 
librium, has  reference  only  to  a  solid  homogeneous  triangular  prism, 
floating  on  the  surface  of  a  fluid  with  its  axis  of  motion  *  horizontal ; 


*  When  a  solid  homogeneous  body,  in  a  state  of  equilibrium  on  the  surface  of  a 
fluid  is  disturbed  by  the  application  of  an  external  force,  it  will  endeavour  to  restore 
itself  by  turning  round  a  horizontal  line  passing  through  its  centre  of  gravity,  and 
th^s  line  on  which  the  body  revolves,  is  called  the  axis  of  motion. 


OF  THE  POSITIONS  OF  EQUILIBRIUM. 


329 


and 


but  there  are  various  other  forms,  which  are  not  less  frequent  in  the 
practice  of  naval  architecture,  nor  less  important  as  subjects  of  theo- 
retical inquiry  :  some  of  these  we  now  proceed  to  investigate. 

PROBLEM  LVIII. 

415.  Suppose  that  a  solid  homogeneous  body  in  the  form  of 
a  rectangular  prism,  floats  upon  the  surface  of  a  fluid  of  greater 
specific  gravity  than  itself,  in  such  a  manner,  that  only  one  of 
its  edges  falls  below  the  plane  of  floatation  :  — 

It  is  required  to  determine  what  position  the  body  assumes, 
when  it  has  attained  a  state  of  perfect  quiescence. 

Let  ABCD  be  a  vertical  section,  at  right  angles  to  the  horizontal 
axis  passing  through  the  centre  of 
gravity  of  the  rectangular  prism, 
and  let  I  K  be  the  surface  of  the 
fluid,  on  which  the  body  floats 
in  a  state  of  equilibrium, 
being  the    extant    portion 
mvn  the  part  which  falls  below 
the  plane  of  floatation. 

Bisect  mn  in  F  and  vn  in  H, 
and  draw  the  straight  lines  DF 
and  mil,  intersecting  each  other 
in  g  the  centre  of  gravity  of  the 
immersed  triangle  mvn.  Join  the 

points  A,  c  and  B,  D  by  the  diagonals  AC  and  BD,  intersecting  in  o 
the  centre  of  gravity  of  the  rectangular  section  ABCD,  and  draw  og. 

Then,  because  the  body  floats  upon  the  surface  of  the  fluid  in  a 
state  of  equilibrium  according  to  the  conditions  of  the  problem  ;  it 
follows  from  the  laws  of  floatation,  that  the  straight  line  Gy  is  perpen- 
dicular to  IK.  Through  F  the  point  of  bisection  of  mn  the  base  of 
the  immersed  triangle,  and  parallel  to  go,  draw  FP  meeting  the 
diagonal  BD  in  the  point  p,  and  join  PTW,  PW;  therefore,  because  the 
straight  line  ga  is  perpendicular  to  mn  the  plane  of  floatation,  it  is 
evident  that  FP  is  also  perpendicular  to  mn,  and  consequently,  pm 
and  PW  are  equal  to  one  another. 

This  is  a  condition  of  equilibrium  which  holds  universally,  and 
another  is,  that  the  area  of  the  immersed  triangle  m  D  n,  is  to  the  area 
of  the  whole  section  ABCD,  as  the  specific  gravity  of  the  solid,  is  to 


330  OF  THE  POSITIONS  OF  EQUILIBRIUM. 

the  specific  gravity  of  the  fluid  on  which  it  floats  ;  when  both  these 
conditions  obtain,  the  body  will  float  permanently  in  a  state  of  equi- 
librium. 

Put  a  zz:  A  D  or  BC,  one  of  the  sides  of  the  section  that  contain  the 

immersed  angle, 

b  zz:  DC  or  AB,  the  other  containing  side; 
s  zz  the  specific  gravity  of  the  floating  solid, 
s'  zz  the  specific  gravity  of  the  supporting  fluid,  or  that  on 

which  the  body  floats, 
x  zz  Dm,  the  part  of  the  AD  which  is  immersed  under  mn  the 

plane  of  floatation, 

y  zz  DW,  the  corresponding  portion  of  the  side  DC  ; 
a'  zz  the  area  of  the  whole  rectangular  section  A  BCD,  and 
o"zz  the  area  of  the  immersed  portion  mvn. 

Then,  since  the  section  of  the  solid  is  considered  to  be  uniform, 
with  respect  to  the  axis  of  motion,  throughout  the  whole  of  its  length, 
we  have 

a"s'  =  a's.  (253). 

But  by  the  principles  of  mensuration,  the  area  of  the  whole  rectan- 
gular section  A  BCD,  is  expressed  by  the  product  of  its  two  sides; 
that  is, 

a'  zz  a  b, 

and  the  area  of  the  immersed  triangle  mow,  is 


Let  these  values  of  a'  and  a"  be  substituted  instead  of  them  in  the 
equation  (253),  and  we  shall  have 

abs=\xys'.  (254). 

By  the  property  of  the  right  angled  triangle,  it  is 

B  D2  zz  A  D*  4-  A  B?> 
or  by  putting  d  to  denote  the  diagonal  BD,  we  get 


from  which,  by  extracting  the  square  root,  we  obtain 


and  by  the  principles  of  Plane  Trigonometry,  it  is 

\/a8  +  62  :  a  :  :  rad.  :  COS.ADB  ; 

and  similarly,  by  Trigonometry,  we  have 

1^/0?  -j-  &2  :  b  :  :  rad.  :  cos.  BDC  ; 


OF  THE  POSITIONS  OF  EQUILIBRIUM.  331 

therefore,  by  working  out  the  above  analogies,  and  putting  radius 
equal  to  unity,  we  shall  have 


COS.ADBIZZ  —  —  ,  and  cos.  BD  cur  —  . 


Since  gG  and  FP  are  parallel,  and  g?  equal  to  one  third  of  DF  ;  it 
follows,  that  GP  is  equal  to  one  third  of  DP;  or  which  is  the  same 
thing,  DP  is  equal  to  three  fourths  of  BD  ;  that  is 
DPz=|Va2-f  b\ 

When  two  sides  of  a  plane  triangle  are  given,  together  with  the 
angle  of  their  inclination,  as  is  the  case  in  the  triangles  WIDP  and 
WDP;  then,  the  writers  on  Trigonometry  have  demonstrated,  that 

7WP2ZZDW2-f-  DP2  -  2D97Z.DPCOS.ADB,  and  WP2  —  DW2-|-  DP2  - 
2DW.DPCOS.BDC  ; 

and  these,  by  the  principles  of  floatation,  are  equal,  hence  we  get 

Dm2  -  SDTW.DPCOS.ADBUZDW2  -  2DW.D  P  COS.B  DC. 

Let  the  analytical  expressions  of  the  several  quantities  Dm,  on,  DP, 
cos.  ADB  and  COS.B  DC,  be  substituted  in  the  above  equation,  and  we 
shall  obtain 

*-!£:=,•-»»  (255). 

If  both  sides  of  the  equation  (254),  be  divided  by  the  expression 
Jars',  we  shall  obtain  as  follows,  viz. 


the  square  of  which,  is 


•  y  ~  : 


Now,  if  these  values  of?/  and  z/2,  be  respectively  substituted  instead 
of  them  in  equation  (255),  we  shall  obtain 

3ab*s 


2  __ 

":  : 

and  finally,  by  reduction  and  transposition,  we  get 

(256). 

And  if  we  consider  the  value  of  s',  or  the  specific  gravity  of  the 
fluid,  to  be  expressed  by  unity,  as  is  the  case  with  water  ;  then  the 
above  general  equation  becomes 


332  OF  THE  POSITIONS  OF  EQUILIBRIUM. 

416.  When  the  specific  gravity  of  the  solid  body  is  so  related  to 
that  of  the  fluid,  as  to  fulfil  the  conditions  of  the  problem,  the  roots 
of  the  above  equation  will  determine  the  positions  of  equilibrium ; 
but  since  there  cannot  be  more  than  three  real  positive  values  of  x  in 
the  equation,  it  follows,  that  there  cannot  be  more  than  three  positions 
in  which  the  prism  will  float  in  a  state  of  rest,  with  only  one  of  its 
edges  below  the  surface  of  the  fluid. 

417.  If  a  and  b  are  equal  to  one  another  ;  that  is,  if  the  transverse 
section  of  the  floating  body  be  a  square  at  right  angles  to  the  axis  of 
motion ;  then,  equation  (257)  becomes 

O  j 

x4' -- 

and  from  this,  by  transposition,  we  obtain 

a;4— — X*3-f  3b*sx  —  46V  =  0.  (258). 

Now,  it  is  obvious,  that  this  equation  is  composed  of  the  two  fol- 
lowing quadratic  factors, 

O  I 

a?2— 262s,  and  x9 —  X  a; -f  2&2*; 

which  being  converted  into  equations,  gives 

x^  —  Ws,  (259). 

and  similarly,  from  the  adfected  factor,  we  obtain 

x9—  ^Xx=— 262s.  (260). 

Since  these  two  quadratic  equations  are  deduced  from  the  factors 
which  constitute  the  particular  biquadratic  (258),  it  follows,  that  the 
real  positive  roots  which  they  contain,  must  indicate  the  positions  of 
equilibrium  according  to  their  number. 

If  we  extract  the  square  root  of  both  sides  of  the  equation  (259), 
we  shall  obtain 

x  =  b^;  (261). 

but  by  equation  (254),  we  have 

lxy  =  bzs\ 
consequently,  by  division,  we  get 

OA2  « 

(262). 


OF  THE  POSITIONS  OF  EQUILIBRIUM. 


333 


Here  then  it  is  manifest,  that  the  values  of  x  and  y  are  each  expressed 
by  the  same  quantity ;  hence  we  infer,  that  the  body  floats  with  one 
diagonal  of  its  vertical  section  perpendicular  to  the  surface  of  the 
fluid,  and  the  other  parallel  to  it. 

418.  The  practical  rule  afforded  by  the  equations  (261  and  262),  may 
be  expressed  in  words  at  length  as  follows. 

RULE.  Multiply  the  square  root  of  twice  the  specific  gravity 
of  the  solid,  by  the  side  of  the  square  section,  and  the  product 
will  give  the  length  of  the  immersed  part ,  when  the  body  is  in 
a  state  of  rest. 

419.  EXAMPLE.  Suppose  a  square  parallelopipedon,  whose  side  is 
equal  to  18  inches,  to  be  placed  upon  a  fluid  with  one  of  its  angles 
immersed,  and  one  of  its  diagonals  vertical ;  how  much  of  the  body 
will  fall  below  the  plane  of  floatation,  supposing  its  specific  gravity  to 
be  0.326,  that  of  the  supporting  fluid  being  equal  to  unity? 

Here,  by  operating  according  to  the  rule,  we  get 

ar=  18V 2 X0.326 zz  14.526  inches, 
and  for  the  corresponding  value  of  y,  we  have 

,  =  ^  =  ^6^, 

Consequently,  the  position  of  equilibrium  thus  indicated,  is  as 
represented  in  the  annexed 
diagram;  where  IK  is  the 
surface  of  the  fluid,  AC  the 
horizontal  and  B  D  the  ver- 
tical diagonal ;  Dm  and  i>n 
being  respectively  equal  to 
14.526  inches,  as  deter- 
mined by  the  foregoing 
arithmetical  process. 

420.  Take  DP  equal  to 
three   fourths   of  BD,    and 
draw  pm  and  pn  meeting 

the  surface  of  the  fluid  in  the  points  m  and  n ;  then  are  pm  and  pn 
equal  to  one  another ;  this  is  one  of  the  conditions  necessary  to  a  state 
of  equilibrium,  when  neither  of  the  diagonals  is  vertical ;  but  in  the 
present  instance,  the  condition  of  equality  will  obtain  wherever  the 
point  P  may  be  taken,  and  consequently,  the  equilibrium  is  not  in- 
fluenced by  the  position  of  that  point. 


334  OF  THE  POSITIONS  OF  EQUILIBRIUM. 

421.  The  only  condition,  therefore,  which  establishes  the  equilibrium 
in  this  case,  is,  that  the  area  of  the  immersed  triangle  mi>n,  is  to  the 
area  of  the  whole  section  A  BCD,  as  the  specific  gravity  of  the  solid  is 
to  that  of  the  supporting  fluid. 

422.  If  the  specific  gravity  of  the  solid  be  equal  to  one  half  that  of 
the  fluid  on  which  it  floats;  then,  AC  will  coincide  with  IK,  and  in 
this  state  the  specific  gravity  attains  its  maximum  value;  for  if  it 
exceeds  this  limit,   more  than  one   angle  of  the  solid  will  become 
immersed,  and  this  is  contrary  to  the  conditions  of  the  problem. 

423.  When   the  specific  gravity  of  the  floating  solid  is  properly 
limited,  the  equation  (260),  has  two  real  positive  roots;   hence  we 
infer,  that  there  are  two  other  positions  in  which  the  body  may  float 
in  a  state  of  equilibrium,  and  these  will  be  determined  by  the  resolu- 
tion of  the  equation. 

Therefore,  complete  the  square,  and  we  get 


and  by  extracting  the  square  root,  it  is 

3b  b      

x  —  •—  —  =4=  -V  (9  —  32s) ; 

consequently,  by  transposition,  we  have 
b  C  


4  -  (263). 

and  the  corresponding  values  of  y,  are 


(264). 

424.  Now,  by  attentively  examining  these  equations,  it  will  appear, 
that  in  order  to  have  the  values  of  x  and  y  real  quantities,  the  value 
of  sy  or  the  specific  gravity  of  the  solid  body,  must  be  such,  that  thirty 
two  times  that  quantity  shall  not  exceed  the  number  9  ;  and  moreover, 
in  order  that  the  greatest  value  of  x  and  y  may  be  less  than  b  the  side 
of  the  square  section,  it  is  necessary  that  thirty  two  times  the  specific 
gravity  of  the  solid  shall  not  be  less  than  the  number  8. 

425.  When  the  value  of  s  is  taken  such,  that  32s  zz  9;  then  we 
have  \/9  —  325  ~  0  ;  in  which  case  the  values  of  x  and  y  are  each  of 
them  equal  to  three  fourths  of  b  ;  but  when  the  value  of  s  is  such,  that 
325  —  8  ;  then  we  have  <\/9  —  32s  =  =t  1  ,  and  consequently,  the  two 
values  of  x  are  b  and  \b  respectively,  the  corresponding  values  of  y 
being  \b  and  b;  and  the  positions  of  equilibrium  corresponding  to 


OF  THE  POSITIONS  OF  EQUILIBRIUM.  335 

the  above  values  of  x  and  y,  are  as  represented   in   the  annexed 


diagrams,  where  i  K  is  the  horizontal  surface  of  the  fluid  ;  No.  1  the 
position  corresponding  to  x  and  y,  when  they  are  respectively  equal 
to  three  fourths  of  b  ;  No.  2  the  position  indicated  by  x  zn  b  and 
y=\b,  and  No.  3  that  which  corresponds  to  the  reverse  values  of 
x  and  y,  viz.  when  x  is  equal  to  \b,  and  y  equal  to  b  ;  in  both  of 
which  cases,  one  angle  of  the  figure  is  under  the  plane  of  floatation, 
and  another  coincident  with  it;  but  this  is  scarcely  consistent  with 
the  conditions  of  the  problem,  which  distinctly  intimates,  that  only 
one  edge  or  angle  of  the  floating  body  shall  be  immersed  in  the  fluid, 
and  this  implies,  that  all  the  other  edges  or  angles  shall  be  wholly 
extant,  or  in  other  words,  that  the  greatest  values  of  x  and  y,  shall  be 
less  than  the  side  of  the  square  section. 

In  order,  therefore,  that  this  condition  may  obtain,  the  specific 
gravity  of  the  body  must  be  less  than  T9T,  which  gives  the  position  in 
No.  1  ;  and  greater  than  3-8T,  which  gives  the  positions  in  Nos.  2  and  3  ; 
consequently,  by  taking  the  arithmetical  mean  between  these  limits, 
we  shall  have  s—  0.265625,  and  the  equations  (263  and  264)  become 


the  corresponding  values  of  y,  being 


But  the  square  root  of  9  —  8.5  is  0.7071  very  nearly  ;  therefore,  if 
b  be  equal  to  18  inches,  as  in  the  preceding  example,  we  shall  have 

18X3.7071 


the  corresponding  values  of  y  being 
y  =  10.318,  and  y  =  16.682  inches. 

Now,  it  is  manifest,  that  none  of  the  positions  represented  above, 
resemble  that  which  is  indicated  by  the  values  of  x  and  y  just  deter- 
mined ;  but  the  true  positions  which  these  values  furnish,  are  such  as 
correspond  to  a  state  of  equilibrium,  and  they  are  exhibited  in  the 


336 


OF  THE  POSITIONS  OF  EQUILIBRIUM, 


subjoined  figures,  whereas  in  all  the  previous  cases,  IK  is  the  horizontal 
surface  of  the  fluid  ;  men  and  mo  A  Bra  being  the  areas  of  the  immersed 
and  extant  portions  of  the  body,  corresponding  to  #—  16.682  inches, 


and  2/z=:10.318  inches  ;  the  subcontrary  figures  odp  and  oabcp  being 
the  respective  areas  when  #=:  10.318  inches,  and  y  zn  16.682  inches. 

Bisect  mn  in  F,  and  through  the  point  F  draw  FP  at  right  angles  to 
mn,  meeting  the  diagonal  AC  in  the  point  p,  and  join  pm  and  PW; 
then  it  is  manifest,  that  the  straight  lines  pm  and  PW  are  equal  to  one 
another,  as  ought  to  be  the  case  when  the  solid  floats  in  a  state  of 
equilibrium;  and  moreover,  the  area  of  the  immersed  portions  men, 
and  odp,  are  to  the  area  of  the  entire  sections  A  BCD  and  abed,  as  the 
specific  gravity  of  the  floating  solid,  is  to  that  of  the  supporting  fluid. 

426.  If  the  conditions  of  the  problem  should  be  reversed,  that  is, 
if  three  angles  of  the  figure  be  immersed  beneath  the  plane  of  floata- 
tion, and  one  extant  above  it;  then,  by  a  similar  mode  of  investiga- 
tion, it  may  be  shown,  that 

\xy  s'  —  «&(«'  —  s) 
and  furthermore,  that 


..  __ 

~     ' 


2    ~  2 

Now,  these  being  similar  equations  to  those  which  correspond  to  the 
case  of  one  angle  being  immersed  beneath  the  surface  of  the  fluid  ;  it 
follows,  that  all  the  other  steps  of  the  investigation  would  also  be 
similar,  and  consequently  they  need  not  be  repeated. 

PROBLEM  LIX. 

427.  Suppose  that  a  solid  homogeneous  prismatic  figure, 
whose  transverse  section  is  rectangular,  is  found  to  float  in  a 
state  of  equilibrium  on  the  surface  of  a  fluid  with  its  two 
edges  immersed  :  — 


OF  THE  POSITIONS  OF  EQUILIBRIUM. 


337 


It  is  required  to  determine   the  positions  assumed  by   the 
solid,  when  it  is  in  a  state  of  quiescence. 

The  solution  of  this  problem  is  attended  with  greater  difficulty  than 
either  of  the  preceding  ones  respecting  the  positions  of  equilibrium  ; 
the  superior  difficulty  in  this  case,  arises  from  the  situation  of  the 
lines,  whose  equality  constitutes  the  second  condition  of  equilibrium  ; 
in  the  foregoing  cases,  this  equality  was  determined  by  the  resolution 
of  the  simple  problem  in  Plane  Trigonometry,  where  two  sides  and 
the  contained  angle  are  given,  and  it  is  required  to  find  the  third  side, 
or  that  which  subtends  the  given  angle  ;  in  the  present  instance,  how- 
ever, this  mode  of  comparison  does  not  take  place,  and  the  equality 
of  the  lines  alluded  to,  or  rather  the  condition  of  equilibrium  depend- 
ing on  such  an  equality,  can  only  be  established  by  a  series  of  com- 
plicated analogies,  arising  from  the  similarity  of  triangles  determined 
by  the  construction. 

Let  A  BCD  represent  a  transverse  section  perpendicular  to  the  axis 
of  a  homogeneous  rectangular 
prism,  which  floats  in  equilibrio 
on  the  surface  of  a  fluid  of  greater 
specific  gravity  than  itself,  and  in 
such  a  manner,  that  two  of  its 
angles  are  wholly  immersed  be- 
neath the  plane  of  floatation  re- 
presented by  HE;  IK  being  the 
horizontal  surface  of  the  fluid, 
HECD  the  immersed  portion  of 
the  section,  and  ABEH  the  extant 
portion. 

Let  G  and  g  be  the  centres  of  gravity  of  the  whole  section  ABCD, 
and  the  immersed  part  HECD;  join  G#,  then,  if  the  position  which 
the  body  has  assumed  be  that  of  equilibrium,  the  line  Gg  is  perpen- 
dicular to  HE  the  plane  of  floatation,  and  the  area  of  the  immersed 
part  HECD,  is  to  the  area  of  the  whole  section  ABCD,  as  the  specific 
gravity  of  the  solid  is  to  that  of  the  supporting  fluid. 

Through  the  point  c,  the  most  elevated  of  the  immersed  angles  of 
the  figure,  draw  cb  perpendicular  to  IK,  and  through  the  points  o 
and  g  draw  GC  and  ge  perpendicular  to  cb;  then,  if  the  position 
which  the  body  has  assumed  be  that  of  equilibrium,  the  straight  lines 
GC  and  g e  are  equal  to  one  another.  The  conditions  under  which  the 
body  floats  in  a  state  of  quiescence,  therefore  are, 

VOL.   I.  7. 


338  OF  THE  POSITIONS  OF  EQUILIBRIUM. 

1.  That  the  area  of  the  immersed  part,  and  that  of  the 
whole  section,  are  to  one  another  as  the  specific  gravities  of 
the  solid  and  the  fluid. 

2.  That  the  horizontal  lines,  intercepted  between  the  centres 
of  gravity,  and  the  vertical  line  passing  through  the  most 
elevated  of  the  immersed  angles,  are  equal  to  one  another. 

Through  the  points  o  and  g,  draw  the  straight  lines  oa  and  gv 
perpendicular  to  BC  the  side  of  the  section;  and  through  the  points 
a  and  v,  draw  ab  and  vd  parallel  to  the  horizon,  and  am,vn  perpen- 
diculars to  GC  and  ge\  and  finally,  through  E  and  g,  and  parallel  to 
CD  and  CB,  draw  the  straight  lines  E£  and  sr,  and  the  construction  is 
finished. 

Then  it  is  manifest,  that  by  means  of  the  parallel  and  perpendicular 
lines  employed  in  the  construction,  we  can  form  a  series  of  similar 
triangles,  which  will  lead  us  by  separate  and  independent  analogies, 
to  the  comparison  of  the  lines  GC  and  ge,  on  whose  equality  the  equi- 
librium of  floatation  depends. 

Put  a  =z  AD  or  b  c,  the  longest  side  of  the  transverse  section, 
b  =:  AB  or  DC,  the  shortest  side, 
d  ~  gv,  the  perpendicular  distance  between   the  centre  of 

gravity  of  the    immersed   part,   and  the  side  of  the 

section  B  c ; 

a'  —  the  area  of  the  whole  section  ABC  D, 
a"~  the  area  of  the  immersed  part  HECD  ; 
x  zz  DH,  the  distance  between  the  lowest  immersed  angle,  and 

the  corresponding  extremity  of  the  line  of  floatation, 
y  —  CE,  the  distance  between  the  highest  immersed  angle  and 

the  other  extremity ; 

and  as  heretofore,  let  s  denote  the  specific  gravity  of  the  solid  body, 
and  s'  the  specific  gravity  of  the  fluid  on  which  it  floats ;  then,  by  the 
principles  of  floatation,  we  have 

a"  :a'::s:  s', 

and  from  this,  by  equating  the  products  of  the  extreme  and  mean 
terms,  we  get 

a's  =  a"sf. 

Now,  by  the  principles  of  mensuration,  the  area  of  the  rectangular 
section  ABCD,  is  expressed  by  the  product  of  its  two  containing  side* 
AB  and  BC;  hence  we  have 


OF  THE  POSITIONS  OF  EQUILIBRIUM.  339 

and  moreover,  the  area  of  the  immersed  part  HE  CD,  is  expressed  by 
half  the  sum  of  the  parallel  sides  drawn  into  the  perpendicular  dis- 
tance between  them ;  consequently,  we  obtain 

abs=  \bs\x  +  y), 
and  finally,  by  multiplication  and  division,  it  is 

2«s  —  s'O  +  y)-  (265). 

The  equation  which  we  have  just  investigated,  involves  one  of  the 
conditions  of  equilibrium,  viz.  that  in  which  the  area  of  the  immersed 
part,  and  that  of  the  whole  section,  are  to  each  other,  as  the  specific 
gravity  of  the  solid  is  to  that  of  the  fluid ;  but  in  order  to  discover  the 
equation  which  involves  the  other  condition,  we  must  have  recourse  to 
a  separate  construction,  as  follows. 

Let  IK  be  the  surface  of  the  fluid,  and  HECD  the  immersed  portion 
of  the  section,  as  in  the  general  dia- 
gram preceding.    Bisect  D  H  and  c  E,         ~  \'-  " 
the  parallel  sides  of  the  figure,  in  the 
points  t  and  z,  and  draw  tz;*  then, 
by  the  principles  of  mechanics,  the   l        =^^-^ 
straight  line  tz  passes  through  the 
centre  of  gravity  of  the  figure  HECD, 
and  divides  it  into  two  parts  such,  that 
gz  :  gt  :  :  SDH-J-CE  :  DH  +  2cE. 

Through  the  point  E  draw  E£*  parallel  to  DC,  and  bisect  DC  and 
HE  in  the  points  i  and  h;  draw  ih,  bisecting  E£  the  side  of  the 
triangle  HE*  in  the  point  q,  and  join  th  and  uq,  intersecting  one 
another  in  the  point  o ;  then  is  the  point  o  thus  determined,  the  centre 
of  gravity  of  the  triangular  space  H  E  t. 

Draw  the  diagonal  D  E,  bisecting  q  i  in  p  the  centre  of  gravity  of 
the  rectangular  space  ECD  t,  and  join  po  intersecting  tz  in  g ;  then  is 
g  the  centre  of  gravity  of  the  quadrilateral  space  HECD  which  falls 
below  the  plane  of  floatation.  Through  the  point  g  thus  determined, 
draw  the  straight  line  gv  parallel  to  DC,  the  lowest  immersed  side  of 
the  section,  and  meeting  CE  in  the  point  v;  then  is  gv  the  quantity 
to  be  assigned  by  the  construction. 

From  the  point  z  and  parallel  to  CD,  draw  zf  meeting  DH  perpen- 


*  It  is  a  circumstance  entirely  accidental,  that  the  lines  tz  and  <E  terminate  in 
the  same  point  t,  and  consequently,  has  nothing  to  do  with  the  conditions  of  the 
problem  j  it  only  happens  when  D  H  is  double  of  c  E. 

z  2 


340  OF  THE  POSITIONS  OF  EQUILIBRIUM. 

dicularly  in  f\  then  are  the  triangles  tzf  and  zgv  similar  to  one 
another,  and  tf  is  half  the  difference  of  the  sides  D  H  and  CE  ;  that  is, 

tf-\(x-y}. 

Consequently,  by  the  property  of  the  right  angled  triangle,  that  the 
square  of  the  hypothenuse  is  equal  to  the  sum  of  the  squares  of  the 
base  and  perpendicular,  we  shall  have 


and  by  extracting  the  square  root,  it  is 


But  by  the  property  of  the  centre  of  gravity  alluded  to  in  the  con- 
struction of  the  diagram,  it  follows,  that 


*  +  (x-y?  :  gz  :  :  3(*  +  y)  :  2 
or  by  equating  the  products  of  the  extremes  and  means,  we  get 


and  from  this,  by  division,  we  shall  obtain 
_(<2 


then,  because  of  the  similarity  of  the  triangles  tzf  and  zgv,  it  is 

tz  :  zf:  :  gz  :  gv, 
or  by  substituting  the  analytical  expressions,  it  becomes 


and  finally,  by  working  out  the  analogy,  we  obtain 

d-W*  +  y\ 

~  3(x  +  y) 

Referring  now  to  the  original  diagram,  or  that  on  which  the  prin- 
cipal part  of  the  investigation  depends,  it  will  readily  appear,  that 
since  sr  passes  through  g,  the  centre  of  gravity  of  the  quadrilateral 
figure  HECD,  it  follows,  that  sg  is  equal  to  gr;  but  by  the  construc- 
tion cv  is  equal  to  sg,  and  consequently  equal  to  |sr;  now  sr  is 
manifestly  equal  to  sw  and  wr  taken  conjointly  ;  therefore  we  have 
cvnr  \(sw  -\-  wr). 

Since  by  the  construction,  the  lines  DH  and  sr  are  parallel  to  one 
another,  the  triangles  HE*  and  EW  are  similar;  therefore,  by  the 
property  of  similar  triangles,  we  have 

E£  :  tii  :  :  EM;  :  wr. 


OF  THE  POSITIONS  OF  EQUILIBRIUM.  341 

or  by  substituting  the  analytical  expressions,  it  becomes 


from  which,  by  working  out  the  proportion,  we  get 

tt.r-(*—  y)(2*  +  y). 

**±9] 
consequently,  by  adding  and  dividing  by  2,  we  obtain 


6(0:  +  y) 

Again,  it  is  obvious  by  the  construction,  that  the  triangles  HE*  and 
vcd  are  similar  to  one  another  ;  hence  we  have 

HE  :  ut  :  :  cv  :  vd; 
but  by  the  property  of  the  right  angled  triangle,  it  is 

HE  —  ^/b*+(x  —  yf; 
consequently,  by  substitution,  we  obtain 


hence,  by  reducing  the  proportion,  we  get 


V 

It  is  furthermore  manifest,   that  the  triangles  HE*  and  vgn  are 
similar  to  one  another  ;  consequently,  we  have 

HE  :  E  *  :  :  gv  :  gn\ 
or  by  substituting  the  respective  values,  we  get 


from  which,  by  reduction,  we  obtain 
) 


~ 

but  ge  =  yn  4.  en  ;  therefore,  by  addition,  we  have 


(*  +  y) 

(266). 

428.  This  is  one  side  of  the  equation  which  involves  the  second 
condition  of  equilibrium,  and  in  order  to  determine  the  other  side,  we 
must  have  recourse  to  the  triangles  acb  and  a  cm,  which  together 


342  OF  THE  POSITIONS  OF  EQUILIBRIUM. 

with  the  triangle  HE*,  are  similar  among  themselves;  consequently, 
we  have  first,  from  the  triangles  list  and  ac£,  as  follows. 

H  E  :  ii  t  :  :  a  c  :  a  b  ; 
which  by  substitution  becomes 


y)»:(*  —  y)::  Ja:a6 
therefore,  by  reduction,  we  obtain 

a(x  —  y) 
-    v         yj      - 


Again,  from  the  triangles  HE£  and  aowt,  we  get 

HE  :  E£  :  :  ao  :  om; 
consequently,  by  substitution,  it  is 


from  which,  by  reduction,  we  obtain 
but  GC  — Gw-f-^c;  therefore,  by  addition,  we  have 


GC. 


429.  Here  then,  we  have  discovered  the  other  side  of  the  equation 
which  involves  the  second  condition  of  equilibrium,  and  consequently, 
we  are  now  prepared  to  determine  the  positions  which  the  body  assumes 
when  floating  in  a  state  of  rest ;  for  which  purpose,  let  the  equations 
(266  and  267)  be  compared  with  each  other,  and  we  shall  have 


Here  it  is  manifest  that  the  equation  involves  two  unknown  quan- 
tities ;  in  order  therefore  to  render  it  capable  of  solution,  one  of  those 
quantities  must  be  eliminated,  and  this  can  very  easily  be  done  by 
means  of  the  equation  (265),  where  we  have 

therefore,  by  transposition  and  division,  we  get 
2as 

y—  7 — x> 

or  by  supposing  s'  equal  to  unity,  as  is  the  case  with  water,  we  have 
t/:n2as  —  x. 


OF  THE  POSITIONS  OF  EQUILIBRIUM.  343 

Let  this  value  of  y  be  substituted  instead  of  it,  wherever  it  occurs  in 
the  above  equation,  and  we  shall  obtain 


which  being  reduced  and  thrown  into  a  simpler  form,  becomes 
2x8  —  6asx2-f  (12aV  —  6a2s  -f  62)a;:=:  8aV-|-  a£2s  —  6aV.  (268). 

Now,  according  to  the  nature  of  the  generation  of  equations,  it  is 
manifest  that  the  above  expression  is  composed  of  one  simple  and  one 
quadratic  factor;  but  as  —  #zzO,  is  obviously  one  of  the  members 
from  which  the  equation  is  derived,  for  in  that  case,  the  whole  vanishes, 
or  which  is  the  same  thing,  when  all  the  terms  of  the  equation  are 
arranged  on  one  side  with  their  proper  signs,  the  sum  total  is  equal 
to  nothing. 

Granting  therefore,  that  as  —  a:~0,  is  one  of  the  constituent 
factors,  then  we  shall  have 

x~  as, 
and  by  referring  to  equation  (265),  we  shall  obtain 


therefore,  by  transposition,  it  is 


Consequently,  the  position  of  equilibrium  assumed  by  the  solid  in 
this  instance,  is  when  x  and  y  are  equal  to  one  another  ;  that  is,  when 
the  side  of  the  body  is  parallel  to  the  horizon,  the  depth  to  which  it 
sinks  being  determined  by  the  measure  of  its  specific  gravity. 

Let  all  the  terms  of  the  equation  (268)  be  transposed  to  one  side, 
and  let  their  aggregate  be  divided  by  (as  —  #),  and  there  will  arise 

2x2  —  4asx+$a2s*  —  6a2s-j-62  —  0, 

and  from  this,  by  transposition  and  division,  we  obtain 

x2  —  2a  sx  =  a*s  (3  —  4*)  —  j#. 

From  this  equation  it  may  be  inferred,  that  if  the  roots  or  values  of 
x  be  both  real  and  positive  quantities,  and  each  of  them  less  than  a 
the  upward  side  of  the  section  ;  then  the  body  may  have  two  other 
positions  of  equilibrium,  which  will  be  determined  by  reducing  the 
equation. 

Complete  the  square,  and  we  obtain 

V  —  3aV(l  —  *)  —  J#  ; 


344 


OF  THE  POSITIONS  OF  EQUILIBRIUM. 


therefore,  by  evolution,  it  becomes 


x  —  asi= 


3azs(l  —  s)  —  i 
and  by  transposition,  we  have 


x  =  as+^/3a?s(l—  s)  —  %b* ; 
and  the  corresponding  values  of  yf  are 


(269). 


3a8s(l—  s)  —  \tf.  (270). 

It  would  be  superfluous  in  this  place,  to  give  a  numerical  example 
to  illustrate  the  reduction  of  equations  (269  and  270) ;  we  shall 
therefore  drop  the  discussion  of  the  oblong  rectangular  section,  and 
proceed  to  inquire,  what  are  the  circumstances  which  combine  to 
establish  the  equilibrium  in  a  square. 

430.  Therefore,  when  a  and  b  are  equal  to  one  another,  that  is, 
when  the  transverse  section  is  a  square ;   then  the  general  equation 
(268),  becomes 

2x* — 6£sa;2-f  68(12s8 —  6s -f  1 )  x  =  ^(Ss8 — 6s*-fs);    (271). 

but  one  of  the  constituent  factors  of  this  equation  is, 

bs  —  x  —  0  ; 
consequently,  by  transposition  and  division,  the  other  factor  becomes 

2a8—  4bsx  4-  &X8s8—  65  -f  1)  =  0 ; 
and  from  this,  by  transposing  and  dividing  by  2,  we  shall  get 

a?8—  2bsx  —  b\3s  —  4s8—  J).  (272). 

Now,  it  is  manifest,  that  when  the  section  is  a  square,  as  we  have 

assumed  it  to  be  in  the  present  instance,  the  factor  bs  —  arzzO,  gives 

x  =  bs, 

and  from  equation  (265),  we  obtain 
y ' —  2bs  —  x  ~%bs  —  b$~  bs. 

Hence  it  appears,  that  the  body  will  float  in  a  state  of  quiescence, 
when  any  of  its  sides  is  horizontal,  and  in  this  case,  the  problem  is 
reduced  to  the  determination  of  the  depth  to  which  the  body  sinks, 
and  this  is  entirely  dependent  on  the  A 

measure  of  its  specific  gravity. 

431.  If  the  specific   gravity  of  the 
floating  solid,  be  to  that  of  the  fluid  j= 
on  which  it  floats  in  the  ratio  of  I  to  2 ;   j 
then  the  body  will  sink  to  one  half  its  f 
depth,  as  represented  in  the  annexed 
diagram,  where   IK   is  the  horizontal 


OF  THE  POSITIONS  OF  EQUILIBRIUM.  345 

surface  of  the  fluid;  EF  the  water  line,  or  line  of  floatation;  EFCD 
the  immersed  portion  of  the  section,  and  ABFE  the  part  that  is  extant, 
and  these  in  the  present  case,  are  equal  to  one  another,  since  the 
specific  gravity  of  the  fluid  is  double  the  specific  gravity  of  the  float- 
ing body. 

It  is  also  obvious  from  the  figure  in  this  case,  that  if  the  body  were 
to  revolve  about  its  axis  of  motion,  till  one  of  the  diagonals  assumed 
a  vertical  position,  it  would  then  float  in  equilibrio  with  one  half  the 
section  immersed,  the  horizontal  diagonal  in  that  case  coinciding  with 
the  surface  of  the  fluid. 

If  we  resolve  the  quadratic  equation  (272),  we  shall  obtain  twa 
other  positions,  in  which  the  body  will  float  in  equilibrio,  provided 
that  the  specific  gravity  be  retained  within  proper  limits  ;  for  it  is  on 
this  limitation  solely,  that  the  equilibrium  of  floatation  depends. 

Complete  the  square,  and  we  shall  have 

a*—  %bsx  +  6V=  b\3s  —  3s2—  j), 
extract  the  square  root,  and  we  obtain 


x  —  bs—=^b^3s(l—  s)  —    , 
therefore,  by  transposition,  we  have 


x  =  b(s+)/3s(l  —  s)  —  J.  (273). 

But  by  equation  (265),  we  have  2bs  =  x  -\-y ;  consequently,  by 
transposition,  we  obtain 

y  —  2bs  —  x; 
therefore,  by  substitution,  we  get 


y  —  b(s  =pV  35(1—  *)  — J.  (274). 

Now,  with  regard  to  the  limits  of  the  specific  gravity,  it  is  easy  to 
perceive,  that  if  the  quantity  ^3s(\ — s)  —  £  be  greater  than  s9 
the  least  values  of  x  and  y  will  be  negative ;  and  if  the  expression 
*  -|--V/3s(l  — s)  —  J  be  greater  than  unity,  the  greatest  values  of  x 
and  y  will  be  greater  than  b ;  consequently,  neither  of  them  satisfies 
the  conditions  of  the  problem.  But  it  is  further  manifest,  that  in 
order  to  have  the  values  of  x  and  y  real  quantities,  the  expression 
3s(l  — s)  must  exceed  the  fraction  ^  ;  now  the  least  value  of  s  that 
will  fulfil  this  condition,  is  s^z  j,  in  which  case  we  have 

35(1— S)ZZ3X|XJ  =  TQ6, 

from  which  subtracting  |  or  T8F,  we  get 


346 


OF  THE  POSITIONS  OF  EQUILIBRIUM. 

the  square  root  of  which  is  J,  hence  it  is 


432.  The  position  of  equilibrium  indicated  by  these  values  of  a;  and 
y,  is  represented   in  the  annexed 

diagram,  where  IK  is  the  hori- 
zontal surface  of  the  fluid;  AE 
the  line  of  floatation  ;  A  E  c  D  the 
immersed,  and  ABE  the  extant  por- 
tion of  the  section. 

Here  it  is  obvious,  that  since  the 
plane  of  floatation  passes  through 
the  angle  A,  and  bisects  the  oppo- 
site side  in  the  point  E;  the  immersed  part  AECD,  is  equal  to  three 
fourths  of  the  entire  section  ABCD,  as  it  ought  to  be,  in  consequence 
of  the  specific  gravity  of  the  body,  being  assumed  equal  to  three 
fourths  of  the  specific  gravity  of  the  fluid. 

It  may  also  be  readily  shown,  that  the  centre  of  gravity  of  the 
whole  section,  and  that  of  the  immersed  part  occur  in  the  same 
vertical  line  ;  but  this  is  not  necessary  in  the  present  instance,  as  we 
are  only  endeavouring  to  discover  the  limits  of  the  specific  gravity. 

433.  The  position   of  equilibrium   corresponding   to  the  value  of 
x—\b,  and  y  zn  b,  is  similar  and  subcontrary  to  the  position  represented 
in  the  preceding  diagram,  and  this  being  the  case,  it  is  unnecessary  to 
exhibit  it  ;  we  shall  therefore  proceed  to  determine  the  greatest  limit 
of  the  specific  gravity  that  will  fulfil  the  conditions  of  the  problem  ; 
for  which  purpose,  we  have 

&(1  -.)  =  !, 
from  which,  by  separating  the  terms,  we  get 

3S  —  3s2—  J; 
therefore,  by  transposition  and  division,  it  becomes 


Complete  the  square,  and  we  obtain 

s2—  s  +  imi  —  1  =  ^, 
hence,  by  extracting  the  square  root,  we  get 


(275). 


and  finally,  by  transposition,  we  have 


434.  From  what  has  been  done  above,  it  is  manifest,  that  the  least 
limit  of  the  specific  gravity  is  f ,  and  the  greatest  is  £(3  -f-  ^  3) ;  the 


OF  THE  POSITIONS  OF  EQUILIBRIUM. 


347 


former  giving  the  position  represented  in  the  preceding  diagram,  and 

the  latter  that  which  is  exhibited  in 

the  marginal  figure  ;  where  the  body 

floats  with   one  of  its   flat  surfaces 

horizontal,  IK  being  the  surface  of 

the  fluid  ;  E  F  the  water  line,  or  line 

of  floatation;  EFCD  being  the  im- 

mersed  part    of    the    section,    and 

A  B  F  E  the  part  which  is  extant,  the 

immersed  part  being  to  the  whole 

section,  as  0.788675  to  1  ;  that  is 

ED:  AD::  i(3  4-^/3)  •!• 

Since  £(3  —  y/'S  is  also  a  root  of  the  equation  (275),  it  follows,  that 
the  body  will  float  in  equilibrio  with 
one  of  its  flat  surfaces  horizontal,  as 
in  the  annexed  figure,  when  the  spe- 
cific gravity  is  equal  to  the  above 
quantity  ;  for  in  that  case  the  radical 
expression  ^  3s(l  —  s)  —  J  in  equa- 
tions (273  and  274)  vanishes,  and 
x  and  y  become  each  equal  to 


,  and  the  immersed  part  of  the  section  is  to  the  whole,  as 
0.211  to  1  ;  that  is 

ED  :  AD  :  :  i(3  —  V*3)  :  1. 

435.  Having  established  the  limits  between  which  the  solid  floats 
in  equilibrio  with  a  flat  surface  upwards,  but  inclined  to  the  horizon 
in  various  angles  depending  on  the  specific  gravity  ;  we  must  now 
return  to  the  equations  (273  and  274),  in  which  the  conditions  are 
indicated,  that  have  enabled  us  to  assign  the  above  limits  to  the 
relative  weight  of  the  floating  body. 

Taking  the  arithmetical  mean  between  the  limits  above  determined, 
we  shall  have 

s  =  |(0.75  +  0.788675)  =  0.7693375  ; 

consequently,  if  the  side  of  the  square  section  be  equal  to  20  inches, 
the  values  of  x  and  y  will  be  determined  by  the  following  operation. 

436.  Let  the  mean  calculated  value  of  s  the  specific  gravity  of  the 
floating  body,  and  the  given  value  of  b  the  side  of  its  square  section, 
be  respectively  substituted  in  equation  (273),  and  we  shall  have,  for 
the  greatest  value  of  a-, 

x  =20(0.7693375+  V  3x0.7693375  x  0.2306625-0.5)  =18.96inches, 


348 


OF  THE  POSITIONS  OF  EQUILIBRIUM. 


and  for  the  least  value  of  x,  it  is 
a:zz20(0.7693375—  v/  3  X  0.7693375  X  0.2306625— 0.5)zzl  1 .8  inches, 

the  corresponding  values  of  y  as  found  from  equation  (274),  are 
y  — 11.8  inches,  and  y  — 18.96  inches. 

Now,  the  positions  of  equilibrium  corresponding  to  the  above  values 
of  x  and  ?/,  are 
as    represented 
in  the  annexed 
diagrams,  where  1 
IK  is  the  hori- 
zontal    surface 
of  the  fluid ;  E  F 
and  ef  the  lines 
of     floatation 

ABCD  the  position  corresponding  to  am  18.96,  and  y  rz  11.8  inches ; 
the  position  abed,  being  that  arising  from  the  reversed  values  of  x 
and?/;  that  is,  a;— 11.8  and  y—  18.96  inches;  EFCD  being  the 
immersed  part  of  the  section  in  the  one  case,  and  efcd  in  the  other. 

437.  If  the  specific  gravity  be  taken  equal  to  the  complement  of 
the  above  mean,  we  shall  obtain  two  other  positions  of  equilibrium  in 
which  the  body  will  float,  corresponding  precisely  to  the  above  figures 
inverted,  af- 
ter the  man-  A^  ^^o 
ner  exhibited 
in  the  mar- 
ginal dia- 
gram ;  where 
ABCD  is  the 
position  cor- 
responding 

to  £  —  20  —  18.96zz  1.04  inches,  and  abed  the  position  correspond- 
ing to  20  — 11.8:^:8.2  inches;  the  immersed  portions  EFCD  and 
efcd  in  the  one  case,  being  equal  to  ABFE  and  abfe,  the  extant 
portions  in  the  other. 

It  is  moreover  manifest,  that  the  centres  of  gravity  of  the  immersed 
and  extant  portions  of  the  section  are  situated  in  the  same  vertical ; 
for  they  are  connected  by  a  straight  line  which  passes  through  the 
centre  of  gravity  of  the  entire  figure  ABCD;  but  in  the  case  of  an 
equilibrium,  the  centres  of  gravity  of  the  whole,  and  the  immersed 
part,  are  situated  in  the  same  vertical  line ;  therefore  also,  the  centres 


K 


OF  THE  POSITIONS  OF  EQUILIBRIUM.  349 

of  gravity  of  the  immersed   and   extant  parts  occur   in   the  same 
vertical. 

438,  We  have  now  to  inquire  if  the  second  condition  of  equilibrium 
be  satisfied ;  that  is,  if  the  area  of  the  whole  section  and  that  of  the 
immersed  part,  are  to  one  another  as  the  specific  gravity  of  the  fluid, 
is  to  that  of  the  solid. 

Now,  the  area  of  the  whole  section,  is  20x20  =  400,  and  that  of 
the  immersed  portion,  is  (1 8.96  -f  11.8)10  =  307.6,  or  92.4;  and  the 
mean  specific  gravity  is,  0.7693375  or  0.2306625 ;  therefore  we  have 

400  :  307.6  :  :  1000  :  769,  and  400  :  92.4  :  :  1000  :  231  ; 
consequently,  the  positions  of  equilibrium  are  as  exhibited  above. 

PROBLEM  LX. 

439.  A  solid  homogeneous  body,  having  the  section  which 
cuts  the  axis  of  motion  perpendicularly,  in  the  form  of  a  common 
or  Apollonian  parabola,  is  supposed  to   float  upon  a  fluid  of 
greater  specific  gravity  than  itself: — 

It  is  required  to  determine  the  position  it  assumes  when  in 
a  state  of  equilibrium,  supposing  its  base  or  extreme  ordinate 
to  be  entirely  above  the  surface  of  the  fluid. 

In  the  resolution  of  this  problem,  we  shall  have  occasion  to  advert 
to  several  properties  of  the  common  parabola,  a  curve  which,  by 
reason  of  its  easy  construction,  and  the  simplicity  of  its  equation,  has 
been  very  extensively  introduced  into  mechanical  science;  and  from 
the  frequency  of  its  occurrence,  it  is  presumed,  that  its  chief  properties 
are  familiar  to  and  clearly  understood  by  the  greatest  part  of  our 
readers  ;  so  that  in  tracing  the  positions  of  equilibrium,  it  will  not  be 
requisite  to  demonstrate  any  of  the  properties  to  which  we  refer,  and 
which,  by  the  nature  of  the  investigation,  we  are  constrained  to 
employ.  A 

Let  AD B  be  a  common  para- 
bola, representing  a  transverse 
section  of  a  solid  uniform  body> 
floating  at  rest  upon  a  fluid 
whose  surface  is  IK,  and  let  T___ J[j 
DC  be  the  axis,  AB  the  base  or 
extreme  ordinate,  (which,  by  "1 
the  conditions  of  the  problem, 


350  OF  THE  POSITIONS  OF  EQUILIBRIUM. 

is  entirely  above  the  surface  of  the  fluid),  and  FII  the  line  of 
floatation. 

Bisect  FH  in  w,  and  through  n  draw  nm  parallel  to  DC  the  axis 
of  the  parabola,  and  meeting  the  curve  in  the  point  m  ;  then  is  mn 
when  produced  to  r  a  diameter  of  the  curve,  whose  vertex  is  in  the 
point  m. 

Through  the  point  m,  draw  mt  parallel  to  AB  the  base  of  the 
parabola,  and  meeting  the  axis  DC  in  the  point  K  ;  produce  CD  to  E, 
making  BE  equal  to  DK,  and  join  EW,  then  by  the  property  of  the 
parabola,  Em  is  a  tangent  to  the  curve  in  the  point  m,  and  it  is 
parallel  to  FH  the  line  of  floatation,  or  the  double  ordinate  to  the 
diameter  mr. 

Let  P  be  the  place  of  the  focus;  join  ?WP,  and  through  H  the 
extreme  point  of  the  line  of  floatation,  draw  uv  parallel  to  AB  the 
base  of  the  figure,  and  meeting  the  diameter  mr  perpendicularly  in 
the  point  v;  then  are  the  triangles  KETW  and  vnii  similar  to  one 
another. 

Take  DG  equal  to  three  fifths  of  DC,  and  mg  equal  to  three  fifths 
of  mn;  then  are  G  and  g  respectively  the  centres  of  gravity  of  the 
whole  parabola  ADB  and  of  the  part  FDH;  join  og,  then  by  the 
principles  of  floatation,  the  straight  lines  Gg  and  FH  are  perpendicular 
to  one  another,  and  consequently,  the  triangles  KEW  and  wng  are 
similar. 

Put  a  m  DC,  the  axis  of  the  parabola  or  section  of  the  floating 

body, 
26zz  AB,  the  base  or  double  ordinate  corresponding  to  the 

axis  DC, 

a'  zz  the  area  of  the  whole  parabola  ADB, 
a"z=  the  area  of  the  immersed  part  FDH, 
x  ~  mn,  an  abscissa  of  the  diameter  mr, 
y  zz:  H  n,  the  corresponding  ordinate, 

z  zz  Km,  the  ordinate  passing  through  m  the  point  of  contact, 
p  zz  the  parameter  or  latus  rectum  to  the  axis, 
s   zz  the  specific  gravity  of  the  floating  solid,  and 
s'  zz  the  specific  gravity  of  the  fluid  on  which  it  floats. 

Now,  supposing  that  all  the  sections  which  are  perpendicular  to  the 
axis  of  motion,  are  equal  to  one  another;  then,  according  to  the 
principles  of  floatation,  we  have 

a'szza'Y.  (276). 


OF  THE  POSITIONS  OF  EQUILIBRIUM.  351 

But  the  writers  on  mensuration  have  demonstrated,  that  the  area  of 

the  common  parabola,   is  equal  to  two  thirds  of  its  circumscribing 

rectangle,  or  equal  to  four  thirds  of  the  rectangle  described  upon  the 

axis  and  the  ordinate  ;  according  to  this  principle  therefore,  we  have 

a'zzi^DcXAC,  and  a"~%vnXmn. 

By  the  equation  to  the  curve,  it  is 


therefore,  by  division,  we  obtain 

& 
DKrr  —  ; 

P 
but  according  to  the  construction,  we  have 


P 

and  by  the  property  of  the  right  angled  triangle,  it  is 


K  E2-f  K  m>—  E  m3  ;  that  is,  E  w2=  — 
therefore,  by  extracting  the  square  root,  we  shall  have 


P 

and  from  the  similar  triangles  KETW.  and  vrm,  we  get 

Ts.m  :  Km  :  :  nn  :  VH; 

and  this,  by  substituting  the  analytical  equivalents,  becomes 
z 


:  z  :  :  y  :  v  H  ; 
P  ' 
consequently,  by  working  out  the  analogy,  we  have 

py 

VH±T 


Hence  then,  by  substituting  the  respective  literal  representatives, 
for  the  quantities  DC,  AC  and  VH,  mn,  the  preceding  values  of  d  and 
a",  become 

a'±z  4«  b,  and  a"= 


Therefore,  let  these  values  of  a'  and  a"  be  substituted  instead  of 
them  in  the  equation  (276),  and  we  shall  obtain 


= 


If  we  suppose  the  axis  of  the  parabola  to  be  vertical,  and  its  base 
or  double  ordinate  horizontal  ;  then  the  points  m  and  D  coincide  with 


352  OF  THE  POSITIONS  OF  EQUILIBRIUM. 

one  another,  and  KTWZHZ  vanishes;  consequently,  in  that  case,  equa- 
tion (277),  becomes 

a  b  s  —  x  y  s'  ; 
but  by  the  property  of  the  parabola,  we  have 

y—  ^/px; 
and  similarly,  we  obtain 

b  zz  ^p  a  ; 
therefore,  by  substitution,  we  get 

a  s  \/pa  zz  x  s  \/p  x  ; 
by  squaring  both  sides,  it  is 

s'V  —  5s  a8,  (278). 

and  finally,  by  division  and  evolution,  we  have 


440.  The  practical  rule  supplied  by  this  equation,  may  be  expressed 
in  words  at  length  in  the  following  manner. 

RULE.  Divide  the  square  of  the  specific  gravity  of  the 
floating  solid,  by  the  square  of  the  specific  gravity  of  the  fluid 
on  which  it  floats,  then  multiply  the  cube  root  of  the  quotient 
by  the  axis  of  the  parabola,  and  the  product  will  give  the 
portion  of  the  axis,  which  falls  below  the  plane  of  floatation, 
or  the  surface  of  the  fluid. 

441.  EXAMPLE.  A  solid  body  whose  transverse  section  is  in  the 
form  of  a  parabola,  floats  in  equilibrio  on  the  surface  of  a  fluid  with 
its  vertex  downwards,  and  its  base  or  double  ordinate  horizontal  ;  it  is 
required  to  determine  how  deep  the  body  sinks,  supposing  the  vertical 
axis  to  be  equal  to  40  inches,  the  specific  gravity  of  the  body  and  that 
of  its  supporting  fluid,  being  to  one  another,  as  686  to  1000. 

Here,  by  operating  as  directed  in  the  rule,  we  shall  have 

s2=6862z=:  470596; 

s'2=10002=:  1000000; 

from  which,  by  division,  we  obtain 


the  cube  root  of  which,  is 
0.470596  =  0.7778  ; 


OF  THE  POSITIONS  OF  EQUILIBRIUM. 


353 


consequently,  by  multiplication,  we  finally  obtain 
a;  =: 40X0.7778  =  31. 112  inches. 

442.  Therefore,  the  position  of  equilibrium  corresponding  to  the 
above  value  of  a?,  is  as  represented 

in  the  annexed  diagram,  where 
AB  is  the  base  or  double  ordinate 
of  the  parabolic  section,  DC  its 
axis;  FH  the  water  line,  or  double 
ordinate  of  the  immersed  portion 
FDII,  DE  the  corresponding  ab- 
scissa, and  IK  the  horizontal  sur- 
face  of  the  fluid. 

That  the  condition  is  satisfied,  in  which  the  centres  of  gravity  of 
the  whole  and  the  immersed  part  are  situated  in  the  same  vertical,  is 
manifest  from  the  circumstances  of  the  case ;  and  that  the  other  con- 
dition is  satisfied,  in  which  the  areas  of  the  whole  and  the  immersed 
part,  are  to  each  other,  as  the  specific  gravities  of  the  fluid  and  the 
solid,  will  appear  from  the  following  calculation. 

Since  the  parabolas  ADB  and  FDII,  are  similar  to  one  another, 
having  the  same  parameter  and  being  situated  about  the  same  axis ; 
it  follows,  that 

a^ a  :  x \/  x  :  :  a   :  a"  ; 

but  by  the  question,  a  is  equal  to  40  inches,  and  by  the  foregoing 
computation,  we  have  found  that  &  =  3 1.1 12  inches;  therefore,  we  get 

SOv/To":  31.112t/3nT2  :  :  1000  :  686, 

which  satisfies  the  other  condition  of  equilibrium,  from  which  we  infer, 
that  if  the  specific  gravity  of  the  solid  be  taken  within  proper  limits, 
the  preceding  diagram  exhibits  a  position  of  floating. 

443.  The  equation  (278)  was  obtained  on  the  supposition,  that  the 
axis  of  the  parabola  is  vertical  and  the  points  D  and  m  coincident,  in 
which  case  the  quantity  z  vanishes  entirely  from  the  figure ;  but  the 
same  result  will  obtain  whether  we  consider  the  points  D  and  m  to  be 
coincident  or  not,  as  will  appear  from  what  follows. 

By  the  property  of  the  parabola,  that  the  distance  of  any  point  of 
the  curve  from  the  focus,  is  equal  to  the  perpendicular  distance 
between  that  point  and  the  directrix,  it  follows,  that  mp  (see  Jig. 
art.  439)  is  equal  to  the  sum  of  DK  and  DP  taken  jointly ;  that  is, 


VOL.  I. 


354  OF  THE  POSITIONS  OF  EQUILIBRIUM. 

Z2 

now,  we  have  already  seen  that  D  K  is  expressed  by   —  ,   and  by  the 
nature  of  the  curve,  we  have  DP  =r  \p  ;  therefore,  by  addition,  it  is 


But  since  by  the  property  of  the  parabola,  the  parameter  of  any 
diameter,  is  equal  to  four  times  the  distance  between  the  vertex  of 
that  diameter  and  the  focus,  we  have 


P 
and  by  the  equation  to  the  curve,  it  is 


consequently,  by  extracting  the  square  root,  we  obtain 


P 

Let  this  value  of  y  be  substituted  instead  of  it  in  equation  (277), 
and  we  shall  obtain 


VP 
but  by  the  equation  to  the  curve,  we  have 


therefore,  by  substitution,  we  shall  get 


VP 
and  multiplying  both  sides  by  ^/p,  we  obtain 


from  which,  by  squaring  both  sides,  we  get 


which  is  the  identical  expression,  obtained  on  the  supposition  of  a 
coincidence  between  the  points  D  and  m  ;  consequently,  the  value  of 
x  must  be  the  same  in  both  cases,  and  the  position  of  floating  depend- 
ing upon  the  specific  gravity  must  also  be  the  same. 

Now,  by  the  construction  we  have  seen,  that  the  triangles  KEW  and 
wng  are  similar  to  one  another;  hence  we  get 
Em  :  EK  :  :  gn  :  wn, 


OF  THE  POSITIONS  OF  EQUILIBRIUM.  355 

or  by  taking  the  analytical  equivalents,  it  becomes 


-  —         —, 

and  from  this,  by  working  out  the  proportion,  we  get 
4xz 


hence,  by  subtraction,  we  have         ,.,'<  j 
ws  —  ns  —  w;w; 

but  because  Emus   is  a  parallelogram,   it  follows,   that  ws—  E?n; 
therefore,  it  is 


Again,  the  triangles  wgn  and  WGS  are  similar  to  one  another;  but 
we  have  shown  above,  that  KEW  is  similar  to  wng;  therefore,  KEW 
is  similar  to  WGS,  and  we  have 

EK  :  EWZ  :  :  ws  :  SG  ; 
taking  therefore  the  analytical  values,  we  obtain 

2£:  ~ 

P      P 


consequently,  by  reduction,  we  have 


2p  5 

But  by  the  nature  of  the  figure,  SG  is  manifestly  equal  to  EG  —  ES  ; 

z2 
now   ESZZ  mn^nx,    and    EG  ^ZDG  -J-  DE;  that  is,    EG  — fa-j 

consequently,  we  have 

s  G  —  f-  a  -| #  ; 

let  these  two  values  of  SG  be  compared  with  each  other,  and  we  get 

~~Tp  "If—If"1'"*'  (279). 

and  finally,  by  reduction,  we  obtain 

~~  10 
Now,  the  value  of  a;,  as  we  have  determined  it  from  equation  (278),  is 


2A2 


356  OF  THE  POSITIONS  OF  EQUILIBRIUM. 

therefore,  by  substitution,  we  have 


(280). 

Here  then,  we  have  obtained  a  pure  quadratic  equation,  which  gives 
two  subcontrary  positions  of  equilibrium,  provided  that  the  specific 
gravity  be  taken  within  proper  limits. 

Extract  the  square  root  of  both  sides  of  equation  (280),  and  we 
shall  obtain 


and  if  the  specific  gravity  of  the  fluid  be  expressed  by  unity,  as  is  the 
case  when  the  fluid  is  water,  then  we  shall  have 


(281). 

But  in  order  to  have  the  value  of  z  a  real  positive  quantity,  it  is 
necessary  that  6a  should  be  greater  than  5p  -f-  6a^s2  ;  in  order  there- 
fore, to  find  the  greatest  value  of  *  that  will  satisfy  this  condition,  we 
must  put  these  two  quantities  equal  to  one  another,  arid  in  that  case 
we  shall  obtain 


transpose,  and  we  obtain 

6a$72=6a—  5p; 
divide  by  6a,  and  it  becomes 
5p 


therefore,  by  involution,  we  get 

'=<'-£>'•..      '"'..,.. 

and  finally,  by  evolution,  it  is 

,1,  •=(>-!•)*• 

Here  then  it  is  manifest,  that  in  order  that  the  positions  determined 
by  the  equation  may  be  those  of  equilibrium,  it  is  necessary  that  the 

specific  gravity  of  the  floating  body  shall  be  less  than  (  1  —  —  ^ 

>         6a/ 

444.  EXAMPLE.  A  solid  body  whose  transverse  section  is  in  the 
form  of  a  parabola,  is  placed  in  a  cistern  of  water  with  its  vertex 


OF  THE  POSITIONS  OF  EQUILIBRIUM.  357 

downwards,  in  such  a  manner,  that  its  base  or  extreme  ordinate  is 
entirely  above  the  surface  ;  it  is  required  to  determine  the  position  of 
the  body  when  in  a  state  of  equilibrium,  the  parameter  of  the  parabolic 
section  being  16  inches,  the  axis  40  inches,  and  the  specific  gravity  of 
the  floating  solid,  to  that  of  the  supporting  fluid  as  1  to  2  ? 

In  this  example  there  are  given,  p  —  16  inches,  am  40  inches,  and 
s  —  0.5,  the  specific  gravity  of  water  being  unity  ;  therefore,  by  sub- 
stitution, we  get  from  equation  (281) 


*  —^       1.6(6X40  —  5X16  —  6x40/0125)  —  3.75  inches. 

And  the  positions  of  equilibrium  corresponding  to  this  value  of  g, 
are  as  represented  in  the  subjoined  diagrams,  and  the  following  is  the 
method  of  construction. 


With  the  parameter  or  latus  rectum  equal  to  16  inches,  and  the 
subcontrary  axes  DC  and  dc  each  equal  to  40  inches,  describe  the 
parabolas  ADB  and  adb ;  from  c  the  middle  of  the  base  and  towards 
the  depressed  part  of  the  figure,  set  off  cr  equal  to  3.75  inches,  the 
computed  value  of  z ;  through  the  point  r,  draw  rm  parallel  to  the 
axis  CD,  and  meeting  the  curve  in  the  point  m ;  draw  the  tangent  WE, 
and  on  the  diameter  mr,  set  off  mn  equal  to  25.19  inches,  the  value 
of  x  as  obtained  by  the  reduction  of  equation  (278)  ;  then  through  the 
point  n,  and  parallel  to  the  tangent  ?WE,  draw  the  straight  line  IK, 
which  will  coincide  with  the  surface  of  the  fluid,  and  cut  the  parabolas 
ADB  and  adb  in  F,  H  and/",  h  the  extremities  of  the  lines  of  floata- 
tion, corresponding  to  the  positions  of  equilibrium  which  we  have 
exhibited  in  the  diagrams. 

We  must  now  endeavour  to  prove,  that  the  positions  in  which  we 
have  represented  the  body  are  those  of  equilibrium;  and  for  this 
purpose,  we  must  inquire  if  the  equation  (279)  is  satisfied  by  the  sub- 
stitution of  the  computed  values  of  x  and  z ;  for  when  that  is  the 
case,  the  line  which  joins  the  centres  of  gravity  of  the  whole  section 
and  the  immersed  part  of  it,  is  perpendicular  to  the  surface  of  the  fluid. 


358  OF  THE  POSITIONS  OF  EQUILIBRIUM. 

Now,  the  values  of  x  and  z  as  we  have  determined  them  by  calcu- 
lation, are  respectively  equal  to  25.19  and  3.75  inches  ;  therefore,  by 
substitution,  equation  (279)  becomes 

162-j-  4X3.75*      2X25.19  __  3X40       3.752 
2X16  ~5~~          ~~5~~       ~16" 

from  which,  by  transposition,  we  have 
162-f  4X3.75*      2X25.19       3x40       3.752 


2X16 


445.  Here  then,  it  is  manifest,  that  one  of  the  conditions  of  equi- 
librium is  satisfied,  viz.  that  in  which  the  line  which  passes  through  the 
centres  of  gravity  of  the  whole  section  and  the  immersed  part  of  it,  is 
perpendicular  to  the  surface  of  the  fluid;  we  have  therefore  in  the 
next  place,  to  inquire  if  the  areas  of  the  whole  section  and  the  im- 
mersed part,  are  to  one  another,  as  the  specific  gravity  of  the  fluid  is 
to  that  of  the  solid.  Now,  we  have  seen,  equation  (277),  that 


—— 

vy+422 

but  by  the  nature  of  the  parabola,  b  zn  \/  ap  ;  hence  it  is 


and  we  have  elsewhere  seen,  that  the  value  of  yy  is 


_ 


consequently,  by  substitution,  we  get 


therefore,  by  expunging  the  common  term  \/p,  and  converting  to  an 
analogy,  we  get  a^a  :  x^/x  :  :  s'  :  s, 

and  this,  by  substituting  the  given  value  of  a,  and  the  computed  value 
of  x,  is 

80/10~:  25.19V2539  :  :  1  :  0.5. 

From  this  it  appears,  that  the  second  condition  of  equilibrium  is 
also  satisfied  ;  we  may  therefore  conclude,  that  the  positions  in  which 
we  have  represented  the  body  are  the  true  ones  ;  but  we  may  further 
observe,  that  by  altering  the  specific  gravity  of  the  body,  other  posi- 

tions may  be  exhibited,  provided  that  the  expression  —  shall  never 
exceed  fl  —  -  —  j*  in  the  common  or  Apollonian  parabola. 


CHAPTER  XIII. 

OF   THE    STABILITY    OF    FLOATING   BODIES    AND    OF    SHIPS. 


446.  IN  the  preceding  pages,  we  have  investigated  and  exemplified 
the  method  of  determining  the  positions  of  equilibrium  in  a  few  of  the 
most  important  cases,  where  the  forms  of  the  floating  bodies  are  such 
as  to  render  them  of  very  frequent  occurrence  in  practical  construc- 
tions ;  we  shall  therefore,  in  the  next  place,  proceed  to  investigate  and 
exemplify  the  conditions  of  Stability,  or  that  power,  by  which,  when 
the  equilibrium  of  a  floating  body  has  been  disturbed,  it  endeavours, 
in  consequence  of  its  own  weight  and  the  upward  pressure  of  the 
fluid,  either  to  regain  its  primitive  settlement,  or  to  recede  farther 
from  it,  by  revolving  on  an  axis  passing  through  its  centre  of  gravity 
parallel  to  the  horizon,  until  it  arrives  at  some  other  position   of 
equilibrium,  in  which  the  principles  of  quiescent  floatation  are  again 
displayed. 

1.  DEFINITIONS  AND  PROPOSITIONS  OF  STABILITY  IN  FLOATING  BODIES. 

447.  It  is  familiar  to  every  person's  experience,  that  when  bodies  of 
certain  forms  and  dimensions,  placed  under  particular  circumstances 
on  the  surface  of  a  fluid,  have  their  equilibrium  deranged  by  the 
action  of  some  external  force,  they  return  to  their  original  position 
after  a  few  movements  or  oscillations  backwards  and  forwards,  in  a 
direction  determined  by  that  of  the  disturbing  impulse. 

It  is  equally  obvious  with  regard  to  other  bodies,  that  however 
small  may  be  the  quantity  of  their  deviation  from  the  original  state  of 
quiescence,  they  have  no  tendency  whatever  to  return  to  it,  but  con- 
tinue to  recede  farther  and  farther  from  it,  by  revolving  about  a 
horizontal  axis,  until  the  deviating  effort  obtains  a  maximum  depend- 
ing upon  the  angle  of  deflexion ;  after  which  the  deflecting  energy 


360 


OF  THE  STABILITY  OF  FLOATING  BODIES  AND  OF  SHIPS. 


continues  to  decrease  until  it  vanishes,  in  which  case  the  body  settles 
in  another  situation,  which  also  satisfies  the  conditions  of  equilibrium. 

Again,  a  solid  body  may  be  so  constituted  with  respect  to  shape 
and  dimensions,  that  in  every  position  which  can  be  given  to  it  on 
the  surface  of  a  fluid,  it  will  rest  in  a  state  of  equilibrium  ;  for  in  all 
situations  and  under  every  condition,  the  centre  of  gravity  of  the 
whole  body  and  that  of  the  immersed  part,  will  occur  in  the  same 
vertical  line  ;  this  being  the  case,  it  is  manifest  that  in  such  a  body, 
the  equilibrium  cannot  be  disturbed,  because  the  external  force,  how- 
ever it  may  be  applied,  can  only  operate  to  turn  the  solid  round  an 
axis,  passing  through  the  centre  of  gravity  in  a  direction  parallel  to 
the  horizon. 

Homogeneous  spheres  are  bodies  of  this  sort,  so  also  are  homo- 
geneous cylinders  floating  with  the  axis  horizontal;  these  have  no 
tendency  to  solicit  one  situation  in  preference  to  another,  and  con- 
sequently, in  whatsoever  position  they  are  placed,  with  reference  to 
the  axis  of  revolution,  they  are  still  in  a  state  to  satisfy  the  conditions 
of  equilibrium,  for  the  centre  of  gravity  of  the  whole  body  and  that 
of  the  immersed  part,  are  always  situated  in  the  same  vertical  line. 

In  the  first  case  then,  where  the  body  has  a  tendency  to  restore 
itself  to  the  original  position,  the  equilibrium  is  said  to  be  stable ;  in 
the  second  case,  where  the  body  deviates  farther  and  farther  from  the 
original  state,  the  equilibrium  is  unstable ;  and  lastly,  in  the  case  where 
the  body  has  no  tendency  to  remain  in,  or  to  solicit  one  position  in 
preference  to  another,  the  equilibrium  is  said  to  be  insensible. 

448.  The  conditions  of  equilibrium  as  here  stated,  are  in  themselves 
sufficiently  simple  and  explicit,  but  in  order  that  none  of  our  readers 
may  enter  upon  this  important  and  difficult  subject,  with  incorrect 
notions  respecting  the  different  species  of  equilibrium,  and  the  various 
conditions  or  circumstances  of  floating  under  which  a  body  may  be 
placed,  we  have  thought  it  expedient  to  subjoin  the  following  expo- 


sitions   and    illustra- 
tions. 

Let  the  dotted  line 
inj^.  1  represent  a 
transverse  section  of 
any  uniform  prismatic 
body,  placed  verti- 
cally on  the  surface 
of  a  fluid,  and  sup- 
pose the  specific 


Ffe.i. 


OF  THE  STABILITY  OF  FLOATING  BODIES  AND  OF  SHIPS. 


361 


gravity  of  the  body  to  be  such,  when  compared  with  that  of 
the  fluid  on  which  it  floats,  as  to  sink  it  to  the  depth  mn.  The 
body  floats  in  equilibrio  in  the  upright  position  ;  suppose  therefore 
that  by  the  application  of  some  extraneous  agent,  it  is  deflected  into 
the  position  abed,  where  it  is  conceived  to  revolve  about  a  horizontal 
axis,  passing  through  G  its  centre  of  gravity,  at  right  angles  to  the 
plane  abed.  If  therefore,  the  body  when  thus  inclined,  requires  the 
force /'to  retain  it  in  that  state,  or  to  prevent  it  from  returning  to  the 
upright  position  ;  then  the  equilibrium  in  which  the  body  is  originally 
placed,  is  what  we  understand  by  the  equilibrium  of  stability. 

Again,  let  the  dotted  Fig.  2. 

line  in  fig.  2  represent 
a  vertical  section  of 
any  uniform  homogene- 
ous prismatic  body ,  float- 
ing upright  and  quies- 
cent on  the  surface  of  a 
fluid,  and  let  the  specific 
gravity  of  the  solid  be 
such  as  to  sink  it  in  the 
fluid  to  the  depth  mn\ 
suppose  now,  that  by  the  action  of  some  external  force,  the  body  is 
deflected  from  the  vertical  position  into  that  represented,  by  a  b  cd ;  it 
is  obvious,  that  the  revolution  is  made  about  the  horizontal  axis 
passing  through  G  the  centre  of  gravity,  at  right  angles  to  the  plane 
abed. 

Hence,  if  the  body  when  thus  inclined,  requires  the  application  of 
the  force/  to  retain  it  in  that  state,  or  to  prevent  it  from  inclining 
further ;  then  the  equilibrium  in  which  the  body  is  originally  placed, 
is  what  we  comprehend  by  the  equilibrium  of  instability . 

Finally,  let  abed,  fig.  3,  be  the  vertical  -*»£•  3- 

section,  and  let  the  specific  gravity  of  the 
body  be  such,  as  to  sink  it  to  the  depth  md; 
the  solid  floats  in  equilibrio  in  the  upright 
position,  and  in  such  a  manner,  that  an 
evanescent  force  will  either  retain  it  in  that 
state,  or  deflect  it  from  it ;  this  is  the  insen- 
sible equilibrium,  or  the  equilibrium  of  indif- 
ference, and  the  solid  is  said  to  overset. 

449.  Of  these  three  species  of  equilibrium,  bodies  floating  on  the 
surface  of  a  fluid  are  manifestly  susceptible;  but  they  admit  of  a 


362  OF  THE  STABILITY  OF  FLOATING  BODIES  AND  OF  SHIPS. 

more  perspicuous  and  comprehensive  definition,  which  may  be  scien- 
tifically read  in  the  following  manner. 

1 .  The  Equilibrium  of  Stability,  is  that  property  in  floating  bodies, 
by  which  on  being  slightly  inclined  to  either  side,  they  endeavour  to 
redress  themselves  and  to  recover  their  original  position. 

2.  The  Equilibrium  of  Instability,   is   that  property  in    floating 
bodies,  by  which  on  being  slightly  inclined  from  the  upright  position, 
they  tumble  over  in  the  fluid  and  assume  a  new  situation,  in  which 
the  conditions  of  floating  again  occur. 

3.  The  Equilibrium  of  Indifference,  is  that  property  in  floating 
bodies,  by  which  they  are  enabled  to  retain  whatever  position  they 
are  placed  in,  without  exhibiting  the  smallest  tendency,  either  to 
regain  the  original  position,  or  to  deviate  farther  from  it. 

In  addition  to  the  different  species  of  equilibrium  described  above, 
there  are  several  other  terms  of  very  frequent  occurrence  in  the 
doctrine  of  floatation,  which  it  will  be  proper  to  explain  before  we 
proceed  to  develop  the  laws  that  regulate  the  conditions  of  stability. 
The  most  common  and  the  most  important  of  the  terms  here  alluded 
to,  are  the  following. 

450.  DEFINITION  1.  The  Centre  of  Effort,  is  the  same  with  the  centre 
of  gravity  of  the  entire  floating  body  ;  it  is  that  point  through  which 
the  horizontal  axis  passes,  and  about  which  the  body  is  supposed  to 
revolve. 

DEFINITION  2.  The  Centre  of  Floatation,  or  the  Centre  of  Buoy- 
ancy, is  the  same  with  the  centre  of  gravity  of  the  immersed  part  of 
the  floating  body,  or  it  is  the  same  as  the  centre  of  gravity  of  the  fluid 
displaced  in  consequence  of  the  floatation. 

DEFINITION  3.  The  Line  of  Pressure,  is  the  vertical  line  passing 
through  the  centre  of  effort,  in  the  direction  of  which,  the  body  is 
impelled  downwards  by  means  of  its  own  weight. 

DEFINITION  4.  The  Line  of  Support,  is  the  vertical  line  passing 
through  the  centre  of  buoyancy  ;  it  is  either  parallel  to,  or  coincident 
with  the  line  of  pressure,  and  is  that  in  whose  direction  the  body  is 
propelled  upwards  by  the  pressure  of  the  fluid. 

DEFINITION  5.  The  Axis  of  Motion,  as  we  have  already  observed 
in  treating  of  the  positions  of  equilibrium,  is  the  horizontal  line  passing 
through  the  centre  of  effort,  and  about  which  the  body  revolves  on 
being  deflected  from  its  original  position. 

DEFINITION  6.  The  Transverse  Section  of  the  Solid,  is  that  indi- 
cated by  a  vertical  plane  at  right  angles  to  the  axis  of  motion,  and 
separating  the  body  into  any  two  parts :  all  the  transverse  sections 


OF  THE  STABILITY  OF  FLOATING  BODIES  AND  OF  SHIPS.  363 

are  parallel  to  one  another,  and  the  principal  transverse  section  is  that 
which  passes  through  the  centre  of  effort. 

DEFINITION  7.  The  Axis  of  the  Section,  is  the  straight  line  which 
passes  through  its  centre  of  gravity,  dividing  it  into  two  parts,  which 
in  the  case  of  a  regular  body  are  equal  and  similar  to  one  another. 
When  this  axis  is  vertical,  it  either  coincides  with,  or  is  parallel  to 
the  line  of  pressure. 

DEFINITION  8.  The  Line  of  Floatation,  or  The  Water  Line,  is  the 
horizontal  line  in  which  the  surface  of  the  fluid  meets  a  vertical  trans- 
verse section  of  the  floating  body. 

DEFINITION  9.  The  Plane  of  Floatation,  is  the  horizontal  plane 
coincident  with  the  surface  of  the  fluid,  and  which  passes  through 
the  water  line,  dividing  the  body  into  the  immersed  and  extant 
portions. 

DEFINITION  10.  The  Equilibrating  Lever,  is  a  straight  line  equal 
to  the  horizontal  distance  between  the  verticals  passing  through  the 
centre  of  effort  and  the  centre  of  buoyancy  ;  or  it  is  the  horizontal 
distance  between  the  line  of  pressure  and  the  line  of  support. 

DEFINITION  11.  The  Stability  of  Floating,  or  the  Measure  of 
Stability,  is  that  force  by  which  a  body  floating  on  the  surface  of  a 
fluid,  endeavours  to  restore  itself,  when  it  has  been  slightly  inclined 
from  a  position  of  equilibrium  by  the  action  of  some  external  agent ; 
or  it  is  a  force  precisely  equal  to  the  fluid's  pressure,  or  to  the  entire 
weight  of  the  floating  body  acting  on  the  equilibrating  lever.  (See 
Proposition  (XI.)  following). 

DEFINITION  ]2.  The  Metacentre,  is  that  point  in  which  the  axis 
of  the  section  and  the  line  of  support  intersect  each  other  ;  it  limits 
the  elevation  of  the  centre  of  effort. 

Upon  these  definitions,  therefore,  in  combination  with  the  following 
simple  and  obvious  propositions,  depends  the  whole  doctrine  of  the 
stability  of  floating  bodies. 


PROPOSITION  IX. 

451.  It  has  already  been  admitted  as  a  principle  in  the  theory 
of  hydrostatics,  that  every  body,  whatever  may  be  its  form  and 
dimensions,  if  it  floats  upon  the  surface  of  a  fluid  of  greater 
specific  gravity  than  itself,  displaces  a  quantity  of  the  fluid  on 
which  it  floats  equal  to  its  own  weight,  and  consequently : — 


364  OF  THE  STABILITY  OF  FLOATING  BODIES  AND  OF  SHIPS. 

The  specific  gravity  of  the  supporting  fluid,  is  to  that  of 
the  floating  body,  as  the  whole  magnitude  of  the  solid  is  to 
that  of  the  part  immersed.  (See  Proposition  VII.) 

PROPOSITION  X. 

452.  If  a  solid  body,  of  whatever  form  or  dimensions,  floats 
upon  the  surface  of  a  fluid  of  greater  specific  gravity  than  itself: — 

It  is  impelled  downwards  by  its  own  weight  acting  in  the 
direction  of  a  vertical  line  passing  through  the  centre  of 
effort ;  and  it  is  propelled  upwards  by  the  pressure  of  the 
fluid  which  supports  it  acting  in  the  direction  of  a  vertical 
line  passing  through  the  centre  of  buoyancy.  (See  Proposi- 
tion VI.) 

Therefore,  if  these  two  lines  are  not  coincident,  the  floating  body 
thus  impelled  must  revolve  upon  an  axis  of  motion,  until  it  attains  a 
position  in  which  the  centre  of  effort  and  the  centre  of  buoyancy  are 
in  the  same  vertical  line. 

PROPOSITION  XL 

453.  If  a  solid  body  of  any  particular  form  and  dimensions, 
floating  on  the  surface  of  a  fluid  of  greater  specific  gravity  than 
itself,  be  deflected  from  the  upright  position  through  a  given 
angle : — 

The  stability  of  the  body  is  proportional  to  the  length  of 
the  equilibrating  lever,  or  to  the  horizontal  distance  between 
the  vertical  lines  passing  through  the  centre  of  effort  and  the 
centre  of  buoyancy.  (See  Problem  LXI.  following.) 

When  the  horizontal  distance  here  alluded  to  is  equal  to  nothing ; 
that  is,  when  the  centre  of  effort  and  the  centre  of  buoyancy  are 
situated  in  the  same  vertical  line;  then  the  stability,  or  the  force 
which  urges  the  body  round  its  axis  of  motion  vanishes,  and  the  equi- 
librium is  that  of  indifference  ;  for  in  this  case,  the  metacentre  coin- 
cides with  the  centre  of  effort. 

If  the  floating  body  be  any  how  inclined  from  the  upright  position, 
and  if,  in  consequence  of  the  inclination,  the  line  of  support  falls  on 
the  same  side  of  the  centre  of  effort  as  the  depressed  parts  of  the 
solid,  then  the  length  of  the  equilibrating  lever  is  accounted  positive, 


OF  THE  STABILITY  OF  FLOATING  BODIES  AND  OF  SHIPS.          365 

and  the  pressure  of  the  fluid  operates  to  restore  the  equilibrium ;  in 
this  case,  therefore,  the  equilibrium  is  that  of  stability. 

But  when  the  line  of  support  falls  on  the  same  side  of  the  centre  of 
effort  as  the  parts  of  the  solid  which  are  elevated  in  consequence  of 
the  inclination  ;  then  the  length  of  the  equilibrating  lever  is  accounted 
negative,  and  the  equilibrium  is  that  of  instability. 

Hence  it  appears,  that  the  stability  of  a  floating  body  is  positive, 
nothing  or  negative,  according  as  the  metacentre  is  above,  coincident 
with,  or  below  the  centre  of  effort:  these  consequences,  however, 
will  be  more  readily  and  more  legitimately  deduced  from  the  general 
formula  which  indicates  the  conditions  of  stability,  and  this  formula 
we  shall  shortly  proceed  to  investigate. 


PROPOSITION  XII. 

454.  The  common  centre  of  gravity  of  any  system  of  bodies 
being  given  in  position,  if  any  one  of  these  bodies  be  moved  from 
one  part  of  the  system  to  another,  it  is  manifest,  from  the  principles 
of  mechanics,  that : — 

The  motion  of  the  common  centre  of  gravity,  estimated  in 
any  given  direction,  is  to  the  motion  of  the  body  moved, 
estimated  in  the  same  direction,  as  the  weight  of  the  said 
body,  is  to  the  weight  of  the  entire  system. 

Therefore,  by  means  of  these  propositions  and  the  definitions  that 
precede  them,  the  whole  doctrine  of  the  stability  of  floating  bodies, 
with  the  train  of  consequences  which  immediately  flow  from  it,  may 
be  easily  and  expeditiously  deduced  ;  but  in  proceeding  to  develope 
the  laws  on  which  the  stability  of  floating  depends,  it  will  be  con- 
venient for  the  sake  of  simplicity,  to  consider  the  body  as  some 
regular  homogeneous  solid,  of  uniform  shape  and  dimensions  through- 
out the  whole  of  its  length ;  for  in  that  case,  all  the  vertical  transverse 
sections  will  be  figures  precisely  equal  and  similar  to  each  other ;  and 
if  the  body  be  divided  by  a  vertical  plane  passing  along  the  axis  of 
motion,  the  two  parts  into  which  it  is  separated  will  be  symmetri- 
cally placed  with  respect  to  the  dividing  plane. 

This  being  premised,  the  principles  upon  which  the  stability  of 
floatation  depends,  will  be  determined  by  the  resolution  of  the  follow- 
ing problem,  in  which  all  the  transverse  sections  are  trapezoids. 


366 


OF  THE  STABILITY  OF  FLOATING  BODIES  AND  OF  SHIPS. 


2.    PRINCIPLES  OF  THE  STABILITY  OF  FLOATING  BODIES. 


PROBLEM  LXI. 

455.  A  solid  homogeneous  body  of  uniform  shape  and  dimen- 
sions throughout  the  whole  of  its  length,  is  placed  upon  a  fluid 
of  greater  specific  gravity  than  itself,  in  such  a  manner,  that  the 
centre  of  effort  and  the  centre  of  buoyancy  are  in  the  same 
vertical  line. 

It  is  required  to  determine  the  stability,  when  by  the 
application  of  some  external  force,  the  body  is  deflected  from 
the  upright  position,  or  from  a  position  of  equilibrium  through 
u  given  angle. 

Let  the  solid  to  which  our  investigation  refers  be  such,  that  the 
vertical  transverse  sections  perpendicular  to  the  axis  of  motion,  are 
equal  and  similar  trapezoids,  as  indicated  by  ABCD  and  abed  in  the 
annexed  diagrams.  The  solid  floats  upon  the  surface  of  the  fluid  IK, 


and  ABCD  is  its  position  when  in  a  state  of  equilibrium  ;  ABFE  being 
the  extant  portion  of  the  vertical  section,  and  EFCD  the  part  immersed 
beneath  the  fluid's  surface.  The  point  G  is  the  centre  of  effort,  or  the 
centre  of  gravity  of  the  whole  section,  the  plane  of  which  is  supposed 
to  pass  through  the  centre  of  gravity  of  the  body,  and  g  is  the  centre 
of  buoyancy,  or  the  centre  of  gravity  of  the  part  immersed  below  the 
surface  of  the  fluid  ;  then  since  the  body  floats  in  a  state  of  equili- 
brium, it  follows  from  Proposition  X.,  that  PQ  the  axis  of  the  section, 
which  passes  through  the  points  G  and  g,  is  perpendicular  to  EF  the 
line  of  floatation. 


OF  THE  STABILITY  OF  FLOATING  BODIES  AND  OF  SHIPS.  367 

We  are  now  to  suppose,  that  by  the  application  of  some  external 
force,  the  solid  revolves  about  its  axis  of  motion  until  it  comes  unto 
the  position  represented  by  abed,  in  which  state  the  equilibrium  does 
not  obtain. 

Here  it  is  manifest  that  PQ,  the  axis  of  the  section  which  was 
vertical  in  the  first  instance,  is  transferred,  in  consequence  of  the 
inclination,  into  the  position  pq ;  and  in  like  manner,  the  line  EF,  which 
before  was  horizontal,  is  transferred  into  the  oblique  position  ef,  and  hi 
is  now  the  line  of  floatation,  or  as  it  is  otherwise  called,  the  water  line. 

Since  the  absolute  weight  of  the  body  remains  unaltered,  whatever 
may  be  the  position  of  floating,  the  area  of  that  portion  of  the  section 
which  is  immersed  below  the  surface  of  the  fluid,  must  also  be  inva- 
riable;  it  therefore  follows,  that  the  areas  hied  and  EFCD  are  equal 
to  one  another ;  but  the  space  efcd  is  equal  to  EFC  D,  hence  the  spaces 
hied  and  efcd  are  each  of  them  equal  to  EFCD;  they  are  therefore 
equal  to  one  another,  and  consequently,  the  extant  triangle  hke  is 
equal  to  the  immersed  triangle  fh  i. 

On  pq  the  axis  of  the  section,  set  off  GH  equal  to  Gg,  the  distance 
between  the  centre  of  effort  and  centre  of  buoyancy  in  the  original 
position  of  equilibrium  ;  then  it  is  manifest,  that  in  consequence  of  the 
inclination,  the  point  g,  which  is  the  centre  of  gravity  of  the  space 
EFCD,  will  be  transferred  to  the  point  n,  which  is  the  centre  of  gravity 
of  the  equal  space  efcd;  and  the  pressure  of  the  fluid  would  act  upon 
the  body  in  the  direction  of  a  vertical  line  passing  through  71,  if  efcd 
were  the  portion  of  the  section  immersed  under  the  fluid's  surface ; 
but  this  is  not  the  case,  for  in  consequence  of  the  inclination,  the 
triangle  fki,  which  was  before  above  the  fluid's  surface,  is  now 
depressed  under  it,  and  in  like  manner  the  triangle  hke,  which  was 
previously  under  the  surface,  is  now  elevated  above  it. 

It  is  therefore  obvious  from  Proposition  XII,  that  by  transferring  the 
triangle  hke  into  the  position  fki,  the  point  n,  which  is  the  centre 
of  gravity  of  the  space  efcd,  must  partake  of  a  corresponding  motion 
and  in  the  same  direction ;  that  is,  the  point  n  must  move  towards 
those  parts  of  the  body  that  have  become  more  immersed  in  conse- 
quence of  the  inclination,  until  it  settles  in  g  the  centre  of  gravity  of 
the  immersed  volume  hied. 

Through  #  the  centre  of  gravity  of  the  immersed  part  hied,  draw 
ym  perpendicular  to  hi  the  line  of  floatation,  and  meeting  pq  the 
axis  of  the  section  in  the  point  m ;  then  is  m  the  metacentre,  and  the 
pressure  of  the  fluid  will  act  in  the  direction  of  the  vertical  line  g'm, 
with  a  force  precisely  equal  to  the  body's  weight ;  and  according  to 


368          OF  THE  STABILITY  OF  FLOATING  BODIES  AND  OF  SHIPS. 

the  principles  of  mechanics,  it  will  act  with  the  same  energy  at  what- 
soever point  of  the  line  gm  it  may  be  applied. 

Through  the  point  n  and  parallel  to  hi  the  line  of  floatation,  draw 
nz  cutting  the  vertical  line  gm  in  the  point  z,  and  through  G  the 
centre  of  gravity  of  the  whole  space  abed,  draw  or  perpendicular 
and  GS  parallel  to  nz,  and  let  k  be  the  point  in  which  the  lines  ef 
and  hi  intersect  one  another;  then,  as  we  have  stated  above,  the 
pressure  of  the  fluid  will  have  the  same  effect  to  turn  the  body  round 
its  axis,  whether  it  be  applied  at  the  point  g  or  the  point  s ;  we  shall 
therefore  suppose  it  to  be  applied  at  the  point  s,  in  which  case  GS 
will  represent  the  point  of  the  lever,  at  whose  extremity  the  pressure 
of  the  fluid  acts  to  restore  the  body  to  its  original  state  of  equilibrium, 
or  to  urge  it  farther  from  it. 

Since  the  effect  of  the  fluid's  pressure,  acting  in  the  direction  of 
the  vertical  line  which  passes  through  g  the  centre  of  buoyancy,  has 
no  dependence  on  the  absolute  position  of  that  point,  but  on  the 
horizontal  distance  between  the  vertical  lines  rG  and  gm  ;  it  follows, 
that  in  the  actual  determination  of  the  positions  which  bodies  assume 
on  the  surface  of  a  fluid,  and  their  stability  of  floating,  the  situation 
of  the  centre  of  buoyancy  in  the  inclined  position  is  not  required,  for 
the  horizontal  distance  between  the  vertical  lines  which  pass  through 
that  centre  and  the  centre  of  effort,  is  sufficient  for  obtaining  every 
particular  in  the  doctrine  of  floatation. 

Bisect  the  sides  of  the  triangles  h  ke  and/Az  in  the  points  u,  v  and 
w,  a:,  and  draw  the  straight  lines  ku,  ev  and  kw,  ix  intersecting  two 
and  two  in  the  points  /  and  o ;  then  are  /  and  o  the  points  thus  deter- 
mined, respectively  the  centres  of  gravity  of  the  triangles  hke  and 

fki. 

Through  the  points  I  and  o  draw  the  straight  lines  ly  and  of, 
respectively  perpendicular  to  hi  the  line  of  floatation,  corresponding 
to  the  inclined  position  of  the  body  ;  then  is  yt  the  horizontal  distance 
through  which  the  centre  of  gravity  of  the  triangle  hke  has  moved  in 
consequence  of  the  inclination  ;  therefore,  by  the  principle  announced 
in  Proposition  XII.,  we  obtain 

area  efcd  :  area  hke  :  :  yt  :  nz. 

It  is  easy  to  comprehend  in  what  manner  the  proposition  cited  above 
applies  to  the  case  in  question ;  for  we  may  assume  the  area  efcd  as 
a  system  of  bodies,  of  which  the  common  centre  of  gravity  is  n.  One 
of  the  bodies  composing  this  system,  viz.  the  triangular  area  hke, 
conceived  to  be  concentrated  in  the  point  /,  is  transferred,  in  conse- 
quence of  the  inclination  from  the  point  I  to  the  point  o,  in  which  the 


OF  THE  STABILITY  OF  FLOATING  BODIES  AND  OF  SHIPS.  369 

equal  volume  fki  is  similarly  concentrated  ;  this  will  have  the  effect 
of  moving  the  common  centre  of  gravity  of  the  system  from  n  to  z, 
in  a  direction  parallel  to  yt,  and  the  distance  nz,  through  which  the 
common  centre  is  moved,  is  what  the  proposition  determines. 

Let  the  position  of  the  point  z  be  supposed  known  ;  then,  if  a  vertical 
line  be  drawn  through  that  point  perpendicular  to  the  line  of  floatation, 
the  centre  of  gravity  of  the  immersed  space  hied,  will  occur  in  some 
point  of  that  line,  as  for  example  at  g ;  but  we  have  already  observed, 
that  it  is  not  necessary  to  determine  the  absolute  position  of  the  point 
in  question,  the  horizontal  distance  GS  or  rz  between  the  verticals 
Gr  and  mz,  being  all  that  is  required. 

Put  a  zz:  hied  or  efcd,  the  area  of  the  immersed  space, 

d  z=  hke  or  fki,  the  area  of  the  triangle  which  has  been 
assumed  as  constituting  an  individual  body  of  the 
system ; 

d  zzr  yt,  the  horizontal  distance  through  which  the  centre  of 
gravity  of  the  triangle  hke  has  moved,  in  shifting  to  the 
position  o  in  the  triangle  fki, 

S  zz:  Gg  or  Gn,  the  distance  between  the  centre  of  effort  and 
the  centre  of  buoyancy,  when  the  axis  of  the  section  is 
vertical ; 

b  zz:  A  B  or  a  b,  the  length  of  the  greater  parallel  side  of  the 
trapezoidal  section, 

/3  zz:  DC  or  dc,  the  length  of  the  lesser  parallel  side ; 

D  zz:  PQ  or  pq,  the  perpendicular  distance  between  the  paral- 
lels AB  and  DC,  or  ab  and  dc, 

c  zz:  EF  or  ef,  the  water  line  or  line  of  floatation  in  the 
upright  position, 

I  zz:  the  axis  of  motion,  or  the  whole  length  of  the  floating 
body,  passing  through  o  the  centre  of  effort ; 

s   zz:  the  specific  gravity  of  the  floating  body, 

s'  zz:  the  specific  gravity  of  the  supporting  fluid,  which  in  the 
case  of  water,  is  expressed  by  unity  ; 

5  zz:  the  stability  of  the  body,  or  the  momentum  of  the  redress- 
ing force ; 

<£  zzr/Az,  or  nGr,  the  angle  of  deflexion,  and 

x  zz:  GS  or  rz,  the  length  of  the  equilibrating  lever. 

Then,  by  substituting  the  literal  representatives  of  the  several  quan- 
tities in  the  foregoing  analogy,  we  shall  obtain 

a  :  a'  :  :  d:  nz,  f*\*  ™> 

VOL.  i.  2  B 

UN1VERS! 


370          OF  THE  STABILITY  OF  FLOATING  BODIES  AND  OF  SHIPS. 

from  which,  by  equating  the  products  of  the  extreme  and  mean  terms, 
we  get 

aXnz~  a'd; 

therefore,  by  division,  we  have 

a'd 

nz  — . 

a 

But  by  the  principles  of  Plane  Trigonometry,  it  is 

rad.  :  3  :  :  sin.0  :  nr, 
which  being  reduced,  gives 

wrzzd  sin.^, 

and  according  to  the  construction  of  the  figure,  it  is  manifest  that  rz 
or  G  s,  is  equal  to  the  difference  between  nz  and  nr,  the  two  quantities 
whose  values  have  just  been  determined ;  consequently,  by  subtrac- 
tion, we  have 

dd      .  . 
X~~a asm*0'  (282). 

456.  The  equation  which  we  have  just  investigated  has  reference 
only  to  a  particular  case  of  the  general  problem,  viz.  that  in  which  the 
vertical  transverse  sections,  throughout  the  whole  length  of  the  body, 
are  equal  and  similar  figures ;  this  condition,  although  it  is  a  restric- 
tion upon  the  general  applicability  of  our  result,  yet  it  allows  an  im- 
mense latitude,  for  the  figures  of  bodies  whose  parallel  transverse 
sections  are  equal  and  similar  areas  are  very  numerous ;  and  if  we 
substitute  the  magnitude  of  the  whole  immersed  volume,  and  that 
of  the  volume  which  becomes  immersed  in  consequence  of  the  inclina- 
tion, instead   of  the    areas   of  the    respective   sections,    the   above 
equation   becomes    general,  because   its   form    and   the  manner  of 
combining  the  terms  admit  of  no  change. 

The  expression  consists  of  five  members  on  one  side,  one  of  which, 
that  is,  the  angle  of  deflexion,  must  always  be  a  given  datum,  or  it 
must  be  directly  assignable  from  the  circumstances  of  the  case,  and 
the  others  must  all  be  determined  by  means  of  the  given  dimensions, 
and  other  particulars  dependent  upon  the  figure  of  the  section ;  but 
the  method  of  applying  the  formula,  and  the  whole  operation  neces- 
sary for  its  reduction,  will  be  sufficiently  exemplified  by  the  resolution 
of  the  following  example. 

457.  EXAMPLE.     A  solid  homogeneous  body,  of  which  the  trans- 
verse parallel   sections  at  right  angles  to  the   axis  of  motion,  are 
equal  and  similar  trapezoids,  is  placed  upon  the  surface  of  a  fluid  in 
such  a  manner,  that  its  broadest  side  is  upwards  and  parallel  to  the 


OF  THE  STABILITY  OF  FLOATING  BODIES  AND  OF  SHIPS. 


371 


horizon  ;  the  body  floats  in  equilibrio  in  this  position  ;  but  suppose 
that  some  external  force  is  so  applied  to  it,  as  to  deflect  it  from  the 
upright  and  quiescent  state  through  an  angle  of  15  degrees;  it  is  re- 
quired to  determine  the  stability  or  the  momentum  of  restoration,  the 
parallel  sides  of  the  section  being  respectively  40  and  30  inches,  the 
perpendicular  distance  between  them  20  inches,  and  the  whole  length 
of  the  body  14  feet,  its  specific  gravity  when  compared  with  that  of 
the  fluid  being  as  270  to  1000,  or  as  0.27  to  1  ? 

458.  For  the  purpose  of  rendering  the  several  steps  of  the  operation 
perfectly  clear  and  comprehensible,  we  shall  refer  to  the  annexed 
diagram,  which  re- 
presents a  trans- 
verse section  of  the 
body  in  the  inclined 
position  ;  ef  being 
the  line  in  which 
it  is  intersected  by 
the  water's  surface 
when  it  is  upright, 
and  hi  the  corres- 
ponding line  when 
it  is  deflected 
through  the  angle  fki.  PQ  is  the  perpendicular  distance  between 
ab  and  cd  the  parallel  sides  of  the  section,  QH  the  depth  to  which 
the  body  sinks  in  the  fluid  as  induced  by  the  specific  gravity  ;  o  and 
I  are  the  centres  of  gravity  of  the  triangles  fki  and  hke;  mt  the 
projected  distance  between  them  on  the  line  hi,  and  Gg  the  distance 
between  the  centre  of  effort  and  the  centre  of  buoyancy,  or  the  dis- 
tance between  the  centre  of  gravity  of  the  whole  body  and  that  of  the 
immersed  part,  when  the  body  is  upright. 

Now,  the  several  parts  which  have  to  be  calculated  in  order  to 
resolve  the  question,  are  the  areas  efcd,  fki,  and  the  distances  mt 
and  og  ;  for  which  purpose,  we  have  given  a  b,  dc,  PQ,  the  angle  fk  i 
and  the  specific  gravity  of  the  solid. 

Therefore,  according  to  the  principles  of  mensuration,  the  area  of 
the  whole  section  is  expressed  by 

20 
|(6  -f-  /3)  X  D  =(40  -f  30)  X  —  =  700  square  inches, 

and  by  the  nature  of  floatation,  we  have 


2fi2 


372  OF  THE  STABILITY  OF  FLOATING  BODIES  AND  OF  SHIPS. 

or  by  putting  s'  equal  to  unity,  and  substituting  the  respective  num- 
bers, we  obtain 

area  efcd  =  a  =  700  X  0.27  —  189  square  inches.         (283). 

459.   Consequently,   by  having  the  area  of  the  trapezoid  efcd, 

and  one  of  its  parallel  sides  dc  given,  the  other  parallel  side  ef  and 

the  perpendicular  depth  Q  H  can  easily  be  found  ;  for  by  the  nature 

of  the  figure  and  the  property  of  the  right  angled  triangle,  we  have 

D     C     /  -       7  20 

«»  =  b=piV  * 


900+  189  —  3o|—  6  inches. 


therefore,  by  the  property  of  the  trapezoid,  we  have 

3(30  +  c)=r  189, 

or  by  separating  the  terms  and  transposing,  we  get 

3c  =  189  —  90=99, 

and  by  division,  it  is 

99 
ef=  c=--=33  inches. 

u 

460.  We  must  next  endeavour  to  discover  the  point  k,  in  which 
the  primary  and  secondary  water  lines  intersect  each  other,  and  for 
this  purpose, 

\)utfk  nr  y,  then  by  subtraction,  we  have  e  k  =  33  —  y  ; 
but  by  the  rules  of  mensuration,  it  is 


and  by  restoring  the  above  values  of  fk  and  e  k,  it  becomes 

yXki  =  (33  —  y)Xkh.  (284). 

Through  the  point  c  and  parallel  to  PQ,  draw  en  meeting  ef  per- 
pendicularly in  n\  then  it  is  manifest,  from  the  principles  of  Plane 
Trigonometry,  that 

*"•*•=£="=«• 

which  corresponds  to  the  natural  tangent  of  75°  57'  49". 
But  by  the  principles  of  Geometry,  the  exterior  angle  cfn  is  equal 
to  both  the  interior  and  opposite  angles  fki  and  fik  ;  consequently, 
by  subtraction,  we  have 

angle  fik  =  75°  57'  49"  —  15°  =  60°  57'  49", 


OF  THE  STABILITY  OF  FLOATING  BODIES  AND  OF  SHIPS.          373 

and  by  Plane  Trigonometry  we  get 
sin.60°57'49"  :  sin.  75°  57'  49"  :  :  y  :  ki  =  1.10967, 
and  by  proceeding  in  a  similar  manner  with  the  triangle  hke,  we 
shall  have 

sin.90°57'49"  :  sin.75°  57'  49"  :  :  33  —  y  :  M  zz  0.9703(33  --  y)  ; 
therefore,  by  substituting  these  values  of  ki  and  kh  in  equation  (284), 
we  get 

1.1096/nz  0.9703(33  —  y)*, 
from  which,  by  reciprocating  the  terms,  we  obtain 


and  extracting  the  square  root,  it  is 


—  =  V  0.874434  —  0.9351  ; 
66  —  y 

therefore,  finally  by  reduction,  we  have 

fk  zz  y  zz:  15.94  inches  nearly. 

461.  Having  thus  determined  the  value  of  fk,  the  value  of  ki  can 
very  easily  be  found;  for  we  have  seen  above,  that  ki  =  l.lQ96y  ; 
consequently,  by  substitution,  we  have 

^—1.1096X15.94  =  17.687  inches; 

therefore,  by  the  principles  of  Mensuration,  we  get 

area/A  zzzo'zrjx  15.94  X  17.687  X  0.25882=36.47  squ.  inches.  (285). 

Let  the  values  of  a  and  a'  as  determined  in  equations  (283)  and  (285), 
be  respectively  substituted  in  equation  (282),  and  we  shall  obtain 


(286). 

but  in  this  equation  the  values  of  d  and  3  are  still  unknown  ;  in 
order  therefore  to  assign  their  values,  we  must  have  recourse  to  other 
principles. 

462.  Now,  since  the  line  kw  which  passes  through  o,  the  centre  of 
gravity  of  the  triangle  fki,  bisects  the  side  fi  in  w,  we  know  from 
the  principles  of  Geometry,  that 


from  which,  by  extracting  the  square  root,  we  get 

2  k  w  = 


and  dividing  by  2,  it  becomes 

kw  = 


374  OF  THE  STABILITY  OF  FLOATING  BODIES  AND  OF  SHIPS. 

But  we  have  already  found  that/A  zz  15. 94  inches,  and  k  zzz  17.687 
inches;  consequently,  their  squares  are  15. 942rz  254.0836,  and 
17.6872zz312.83  respectively;  therefore,  we  have 

kw  —  \  <J  1 133.8272  — /z2, 

and  the  value  offi2  is  found  by  the  following  logarithmic  operation, 
angle  fik  =60°  57'  49"  -  -  log.  cosec.  0.058334 
angle  fki  —  15     00--  log.  sin.  -  9.412996 
side-fk  —15.94  inches  -  log. 1.202488 

Sum  of  the  logs.  =  0.67381 8 

/t*  =  22.2657  nat.  number  -  -  twice  the  sum zz  1.347636; 
therefore,  by  substitution  and  reduction,  we  obtain 


k  w  =  W  1  133.8272  —  22.2657  zz  16.68  inches  nearly. 

But  by  the  property  of  the  centre  of  gravity  as  referred  to  the  plane 
triangle,  we  know  that  k  o  zz  -f  &  w  ;  hence  we  have 
Aozzf-of  16.68  ml  1.12  inches, 

and  by  a  well  known  theorem  in  the  doctrine  of  Plane  Trigono- 
metry, we  have 


from  which,  by  substituting  the  numerical  values,  we  get 
312.83  +  277.89  —  5.5664 


2X16.68X17.687 
consequently,  by  multiplication,  we  get 
kt  =  11.  12X0.99231  =  11.0345  inches. 

463.  Returning  to  the  triangle  hke,  we  find  that  £e  =  33  — 
15.94  zz  17.06  inches,  and  kh  —  0.9703x17.06  zz  16.55  inches; 
therefore,  by  Plane  Trigonometry, 

sin.75  57'  49"  :  sin.  15°  :  :  16.55  :  he, 
which  being  actually  reduced,  gives 

Aezz4.415  inches. 

Therefore,  by  pursuing  a  train  of  reasoning,  similar  to  that  by 
which  we  discovered  the  value  of  k  t,  we  shall  obtain 

—  he9 


from  which,  by  substituting  the  numerical  values,  we  get 

291.0436  +  821.7075—19.4923 
m=  -  -  —       -  zz  11.01  inches. 


OF  THE  STABILITY  OF  FLOATING  BODIES  AND  OF  SHIPS.          375 

Let  this  value  of  km  be  added  to  that  of  kt  already  determined, 
and  the  aggregate  will  give  the  value  of  mt;  therefore,  we  have 

mt  =  d=  11.0345  +  11.01  =22.0445  inches.         (287). 

464.  With  respect  to  the  value  of  the  remaining  quantity  S,  we 
have  only  to  observe,  that  from  the  nature  of  the  trapezoid,  the  posi- 
tions of  the  points  G  and  g  can  easily  be  ascertained,  and  the  distance 
between  them,  is  therefore  expressed  by  the  following  equation,  viz. 


g_D  . 


Now,  according  to  the  conditions  of  the  question,  we  have  6  =  40 
inches,  ft  nr  30  inches,  and  D  zrz  20  inches  ;  and  moreover,  by  com- 
putation, we  have  found  that  ^=33  inches,  and  QHzrG  inches; 
consequently,  by  substitution,  the  above  equation  gives 

*      on        C  20(40  +  60)       6(66  +  30)7       -  .0  -     , 
d  =i  20  —  )     ;      ~  -  L  j  —  i  -  IL  -  L  J.  z=  7.43  inches  very  nearly. 
I  3(40  +  30)  T  3(33  +  30)  }  (288). 

Let  therefore  the  values  of  d  and  S,  as  obtained  in  the  equations 
(287  and  288),  be  substituted  in  equation  (286),  and  we  shall  obtain 

' 


which  being  reduced,  gives 
x  =  4.25  —  1  .923  =.  2.327  inches. 

This  is  the  length  of  the  equilibrating  lever,  but  the  whole  weight 
of  the  body  in  cubic  inches  of  water,  is 

(40  +  30)  10  X  12  X  14x  0.27  =  41752  cubic  inches, 

which  being  reduced  to  Ibs.  gives 
41752X62.5-^1728=:  1510.125  Ibs.  ; 

hence  the  momentum  of  stability  is 
S=  1510.125X2.327  z=  3514.054  Ibs. 

Such  is  the  method  of  calculating  the  measure  of  stability,  when 
the  transverse  sections  are  all  equal  and  similar  figures  ;  but  when 
this  happens  not  to  be  so,  as  in  the  case  of  ships  and  other  vessels 
designed  for  the  purposes  of  navigation,  the  difficulty  of  calculation. 
is  greatly  increased,  for  the  several  terms  of  which  the  equation  is 
constituted,  must  have  their  values  separately  determined  by  intricate 
forms  of  approximation,  the  nature  of  which  can  only  be  known  from 
the  circumstances  which  regulate  the  particular  constructions,  to  which 
the  investigations  are  referred. 


376  OF  THE  STABILITY  OF  FLOATING  BODIES  AND  OF  SHIPS. 


3.       PRINCIPLES    OF    THE    STABILITY    OF    SHIPS. 

465.  We  have  seen  from  the  formula  (282),  that  the  measure  of 
stability,  when  the  body  is  inclined  through  any  angle  from  the  per- 
pendicular, is 

a'd 


in  which  equation,  the  symbol  x  expresses  the  horizontal  distance 
between  the  two  vertical  lines,  one  of  which  passes  through  the  centre 
of  effort,  and  the  other  through  the  centre  of  buoyancy. 

The  same  principle  has  now  to  be  applied  in  estimating  the  stability 
of  ships,  and  this  object  will  be  attained,  if  either  by  calculation  or 
geometrical  construction,  we  find  the  value  of  rr,  which  in  the  inclined 
position  of  the  diagram  to  Problem  LXI.  is  represented  by  GS  or  rz  ; 
then,  if  we  suppose  the  whole  weight  of  the  ship  or  floating  mass  to 
be  denoted  by  w,  it  is  manifest,  that  the  momentum  of  stability  will 
be  expressed  by  the  weight  of  the  vessel  drawn  into  the  horizontal 
distance  between  the  vertical  lines  above  described  ;  that  is, 

771  —  WXj 

where  772  denotes  the  momentum  of  stability,  or  the  effort  by  which 
the  vessel  endeavours  to  regain  the  upright  position,  from  which  it  is 
deflected  by  the  action  of  the  wind,  or  some  other  equivalent  force 
similarly  applied. 

If,  therefore,  we  put  v  to  denote  the  whole  volume  of  fluid  displaced 
by  the  immersed  part  of  the  vessel,  and  v  for  the  volume  which  is 
depressed  below  the  plane  of  floatation,  in  consequence  of  the  vessel 
heeling  from  the  upright  position  through  an  angle  equal  to0;  then, 
the  general  form  of  the  equation  for  the  momentum  of  stability  becomes 

dv  > 

-  aSm'^W;-  (282-). 

466.  Now,  in  applying  this  expression  to  any  particular  case  in 
practice,  it  is  understood,  that  the  position  of  the  centre  of  gravity  of 
the  entire  ship,  and  also  the  position  of  the  centre  of  gravity  of  the 
immersed  volume  when  the  ship  is  upright  and  quiescent,  are  both 
known,  and  consequently,  the  distance  between  those  centres,  which 
is  represented  by  the  line  Gg  or  GW,  is  a  given  or  assignable  quan- 
tity ;  and  moreover,  the  total  displacement  occasioned  by  the  floating 
body,  is  supposed  to  have  been  determined  by  previous  admeasure- 
ments, and  hence,  the  weight  of  a  quantity  of  water,  which  is  equal 


OF  THE  STABILITY  OF  FLOATING  BODIES  AND  OF  SHIPS.          377 

in  magnitude  to  the  displacement,  will  likewise  be  equal  to  the  whole 
weight  of  the  vessel. 

The  quantity  0  is  necessarily  given  from  the  circumstances  of  the 
case,  and  may  be  of  any  magnitude  whatever,  and  therefore,  the 
only  quantities  required  to  be  ascertained,  for  the  purpose  of  dis- 
covering the  momentum  of  the  ship's  stability,  are  v  and  d  in  the 
numerator  of  the  fractional  term,  the  one  denoting  the  magnitude  of 
the  volume  which  becomes  immersed  in  consequence  of  the  inclina- 
tion, and  the  other,  the  distance  through  which  the  centre  of  gravity 
of  that  volume  is  moved  in  a  horizontal  direction,  during  the  deflexion 
of  the  ship  from  the  upright  and  quiescent  position.  In  order,  there- 
fore, to  facilitate  the  determination  of  those  quantities,  the  following 
observations  are  necessary. 

467.  If  a  straight  line  be  conceived  to  pass  through  the  centre  of 
gravity  of  the  ship,  in  the  direction  of  its  length  and  parallel  to  the 
horizon,  traversing  from  the  head  to  the  stern  of  the  vessel ;  then,  such 
a  line  is  called  the  longer  axis  of  the  vessel ;  it  is  the  same  with  the 
axis  of  motion  described  in  the  fifth  definition  preceding,  and  is  so 
called,  for  the  purpose  of  distinguishing  it  from  another  line  also  hori- 
zontal, which  passes  through  the  centre  of  gravity  at  right  angles  to 
the  former,  and  is  called  the  shorter  or  transverse  axis  of  the  vessel ; 
it  is  on  this  axis  that  the  vessel  turns  in  the  process  of  pitching,  a 
motion  which  is  easily  understood  by  considering  an  alternate  eleva- 
tion and  depression  of  the  head  and  stern. 

468.  A  vertical  plane  drawn  through  the  longer  axis,  when  the 
vessel  floats  in  an  upright  and  quiescent  position,  divides  it  into  two 
parts  which  are  perfectly  similar  and  equal  to  one  another,  and  in  this 
respect  at  least,  the  figures  of  vessels  may  be  considered  regular, 
although  that  their  forms  are  not  otherwise  restrained  to  any  uniform 
or  particular  proportions. 

From  the  similarity  and  equality  of  these  two  divisions,  it  necessa- 
rily follows,  that  when  a  vessel  floats  in  a  state  of  upright  quiescence, 
the  similar  parts  on  the  opposite  sides  of  the  plane  of  division  will 
be  equally  elevated  above  the  water's  surface.  A  ship  thus  floating 
in  a  position  of  equilibrium,  may  be  conceived  to  be  divided  into  two 
parts  by  the  horizontal  plane  which  is  coincident  with  the  water's 
surface,  and  the  section  formed  by  this  plane  passing  through  the 
body  of  the  vessel,  is  called  the  principal  section  of  the  water ;  it 
corresponds  with  the  plane  of  floatation  in  the  particular  case  where 
the  vessel  is  upright  and  quiescent,  as  will  readily  be  perceived  by  a 
reference  to  the  ninth  definition  preceding. 


378 


OF  THE  STABILITY  OF  FLOATING  BODIES  AND  OF  SHIPS. 


or 


469.  Let  A  B  c  K  D  represent  a  tranverse  section  of  the  hulk  of  a  ship, 
perpendicular  to  the  longer  axis,  and  passing  through  G  its  centre  of 
gravity,  and  suppose  the  vessel  as  it  floats  upon  the  surface  of  the 
water  to  be  upright  and  quiescent;  then  LK  the  axis  of  the  section, 
according  to  the  seventh  definition,  is  perpendicular  to  the  horizon, 
and  in  this  state 
the  principal  sec- 
tion of  the  water 
passes  through  the 
line  D  c  throughout 
the  whole  length 
of  the  vessel ; 
which  may 
bly  be  better  un- 
derstood, if  the 
principal  water 
section  be  viewed 
endways,  with  the  eye  at  a  great  distance,  it  will  appear  as  if  it  were 
projected  into  the  straight  line  DC. 

While  the  vessel  retains  its  upright  position  and  remains  in  a  state 
of  rest,  the  transverse  or  shorter  axis,  is  that  which  is  represented  by 
the  dotted  line  a  b,  and  the  place  of  the  centre  of  gravity  of  the 
immersed  portion  DKC,  is  somewhere  in  the  line  passing  through  g  in 
a  direction  parallel  to  the  horizon ;  for  g  is  the  place  of  the  centre  of 
gravity  of  the  section  DKC,  which  falls  below  the  principal  section  of 
the  water  passing  through  DC. 

When  the  ship  is  caused  to  heel  or  to  revolve  about  the  longer  axis 
passing  through  G,  until  it  moves  through  an  angle  equal  to  FPC; 
then  it  is  manifest,  that  the  principal  section  of  the  water,  or  the  plane 
in  the  ship  which  passes  through  the  line  DC,  will  be  transferred  into 
the  position  EF  ;  but  the  section  of  the  water  will  intersect  the  sides 
of  the  vessel,  in  the  direction  of  a  plane  passing  through  DC,  which  is 
inclined  to  the  former  plane  passing  through  EF  in  an  angle  equal  to 
the  angle  FPC.  The  plane  which  passes  through  the  line  DC  in  a 
direction  parallel  to  the  plane  of  the  horizon,  may  therefore  be  termed 
the  secondary  section  of  the  water,  merely  to  distinguish  it  from  that 
which  formerly  passed  through  EF,  and  which  we  denominated  the 
principal  section. 

The  principal  and  secondary  sections  of  the  water  must  therefore 
intersect  one  another  in  the  line  denoted  by  the  point  P,  or  rather  in 
the  line  which  being  viewed  endways,  is  projected  into  the  point  P, 


OF  THE  STABILITY  OF  FLOATING  BODIES  AND  OF  SHIPS.  379 

and  stands  at  right  angles  to  the  plane  A  KB.  Consequently,  since 
the  vessel  is  supposed  to  be  inclined  around  the  longer  axis,  it 
follows,  that  the  intersection  of  the  planes  which  we  have  supposed  to 
be  projected  into  the  point  P,  will  be  parallel  to  the  axis  round  which 
the  vessel  is  supposed  to  revolve  in  passing  from  one  position  to 
another. 

But  by  the  laws  of  hydrostatics,  since  the  whole  weight  of  the 
vessel  is  considered  to  be  precisely  the  same,  however  much  it  may 
be  deflected  from  the  upright  and  quiescent  position  ;  it  follows  from 
hence,  that  the  volume  which  becomes  immersed  below  the  water's 
surface  in  consequence  of  the  inclination,  is  equal  in  magnitude  to 
that  which  is  elevated  above  it  by  the  same  cause,  and  consequently, 
the  position  of  the  line  which  is  represented  by  the  point  P,  will  depend 
entirely  upon  the  form  of  the  sides  DE  and  CF. 

Now,  in  a  ship,  of  which  the  breadth  is  continually  altering  from 
the  head  to  the  stern,  and  in  no  regular  proportion  expressible  by 
geometrical  laws,  it  is  manifest,  that  the  place  of  the  point  P,  repre- 
senting the  line  in  which  the  water's  surface  intersects  the  vessel  in 
the  upright  and  inclined  positions,  must  be  practically  determined  by 
some  method  of  approximation,  dependent  upon  the  ordinates  in  the 
vertical  and  horizontal  sections  into  which  the  ship  is  supposed  to  be 
divided. 

By  similar  modes  of  approximation,  the  other  quantities  necessary 
for  the  solution  may  also  be  ascertained  ;  but  in  ships  of  war  and  of 
burden,  constructed  after  the  forms  which  they  generally  assume  at 
sea,  the  calculations  necessary  for  the  purpose  are  unavoidably  prolix 
and  troublesome  ;  and  after  all,  they  must  depend  for  their  accuracy 
entirely  upon  the  skill  and  address  of  the  persons  by  whom  the  requi- 
site ordinates  are  measured  and  registered,  according  to  the  different 
parts  of  the  vessel  to  which  they  particularly  belong ;  for  if  a  very 
nice  and  accurate  arrangement  be  not  preserved  with  regard  to  the 
magnitudes  and  places  of  the  several  ordinates,  it  is  easy  to  be  per- 
ceived, that  the  results  may  come  out  very  wide  of  the  truth,  and  must 
therefore  necessarily  vitiate  the  whole  process. 

In  our  diagram,  the  lines  DC  and  EF,  through  which  the  principal 
and  secondary  sections  of  the  water  pass,  are  supposed  to  bisect  each 
other,  and  consequently,  the  point  P  must  occur  at  the  middle  of  them 
both ;  in  which  case  its  position  is  known  ;  but  the  careful  and  atten- 
tive reader  will  easily  perceive,  that  this  can  very  seldom  happen, 
unless  the  extreme  sides  of  the  zone  which  limits  the  angle  of  the  ship's 
inclination,  are  equally  inclined  to,  and  similarly  situated  in  respect 
of  the  extremities  of  the  intersecting  lines  DC  and  EF. 


380          OF  THE  STABILITY  OF  FLOATING  BODIES  AND  OF  SHIPS. 

When  the  curves  D  E  and  c  F  are  dissimilar  between  themselves,  and 
dissimilarly  situated  in  respect  of  the  intersecting  lines  DC  and  EF  ; 
then  it  is  manifest,  that  the  point  p  cannot  fall  in  the  middle  of  either, 
but  must  occur  to  the  right  or  to  the  left,  according  as  it  is  influenced 
by  the  nature  of  the  curves,  which  define  the  exterior  contour  or  bound- 
ary of  the  vessel. 

Suppose  therefore,  that  the  intersection  takes  place  at  the  point  p, 
a  little  to  the  right  of  the  place  where  the  two  water  lines  are  sup- 
posed to  bisect  one  another ;  and  through  the  pointy,  let  the  straight 
line  <?/be  drawn  parallel  to  EF,  and  meeting  the  sides  of  the  vessel  in 
the  points  e  and/;  then  is  e/the  line  through  which  the  secondary 
section  of  the  water  passes,  on  the  supposition  that  it  intersects  the 
principal  section  in  the  straight  line  passing  through  p,  parallel  to  the 
longer  axis  of  the  vessel. 

470.  Now,  in  order  to  determine  the  position  of  the  point  p,  it  will 
be  expedient  to  conceive  the  volumes  which  become  immersed  under, 
and  elevated  above  the  fluid's  surface,  in  consequence  of  the  inclination, 
and  of  which  fpc  and  epv  are  transverse  sections,  to  be  divided  into 
segments  by  vertical  and  parallel  planes  cutting  the  longer  axis  of  the 
vessel  at  right  angles,  and  at  the  distance  of  a  few  feet  from  each 
other,  the  distances  being  regulated  by  the  dimensions  of  the  vessel, 
and  the  nature  of  the  curves  by  which  it  is  bounded ;  they  may  in 
general,  however,  be  from  3  to  5  feet  in  large  vessels,  and  from  2  to 
3  feet  in  smaller  ones ;  but  in  all  cases,  they  must  be  chosen  accord- 
ing to  circumstances. 


i 7P 


Each  of  these  segments  will  be  of  a  wedge-like  form,  contained 
between  two  vertical  and  parallel  planes  fpc,  f'p'c';  two  inclined 
planes  cpp'c',  fpp'f,  making  with  each  other  an  angle  fpc  or 
f'p'c'  equal  to  the  angle  of  the  vessel's  inclination,  and  the  portion  of 
the  ship's  side  which  is  represented  by  fee1 'fr. 

The  horizontal  distance  between  the  planes  fpc  and  f'p'c',  is  the 
line  pp',  which  being  produced  both  ways  to  the  head  and  stern  of 
the  vessel,  forms  the  line  in  which  the  two  sections  of  the  water  cut 


OF  THE  STABILITY  OF  FLOATING  BODIES  AND  OF  SHIPS.  381 

each  other,  and  is  therefore  coincident  with  the  water's  surface,  and 
parallel  to  the  longer  axis  of  the  ship. 

471.  Since  the  dimensions  of  the  vessel  are  supposed  to  be  known, 
the  lines  D  c  and  ef  will  be  known ;  and  from  these  data,  the  lines  pf 
and  pc  are  to  be  assumed  by  estimation  ;  but  the  angle  fpc  through 
which  the  ship  is  deflected  from  the  upright  position,  is  given  by  the 
nature  of  the  particular  conditions  from  which  the  inclination  or  de- 
flexion arises,  and  consequently,  by  the  rules  of  Trigonometry,  the 
area  of  the  triangle  fpc  becomes  equal  to  J/p  X  jpcsin.  0. 

If  therefore,  the  area  of  the  small  circular  space  fnc  be  determined 
by  any  of  the  methods  of  approximation,  and  added  to  the  area  of  the 
triangle  fpc,  the  sum  will  be  the  area  of  the  mixed  space  fpcn,  and 
by  proceeding  in  a  similar  manner,  the  area  of  f'p'c'm  will  become 
known ;  then,  if  a  mean  of  these  two  areas  be  multiplied  by  the 
perpendicular  distance  pp' ,  the  product  will  be  a  near  approximation 
to  the  solidity  of  the  wedge  contained  between  the  planes  fpp'f 
and  cppc*. 

And  exactly  in  the  same  manner,  the  solid  contents  of  the  opposite 
segment  which  is  elevated  by  the  inclination  is  to  be  obtained,  and  if 
the  aggregate  of  all  the  elevated  segments  be  equal  to  the  aggregate 
of  all  the  depressed  ones ;  that  is,  if  the  entire  volume  which  becomes 
immersed  by  the  inclination,  is  equal  to  that  which  becomes  elevated 
by  the  same  cause,  the  point  p  has  been  properly  determined ;  but  if 
they  are  not  equal,  the  operation  must  be  repeated  until  they  exactly 
agree,  and  when  this  agreement  has  been  obtained,  the  value  of  v  in 
equation  (282°)  becomes  known. 

472.  Now,  in  order  to  determine  the  momentum  of  stability  eli- 
cited by  the  ship  under  the  proposed  inclination,  it  is  requisite  that  the 
product  dv  in  the  numerator  of  the  fraction  should  be  completely  de- 
termined;  and  for  this  purpose,  the  area  of  the  space  fpcnf,  and 
the  position  of  its  centre  of  gravity  have  to  be  found  by  approxima- 
tion, and  also,  the  area  of  the  space  f'p'c.'mfy  with  the  position  of 
its  centre  of  gravity.     Let  the  points  o  and  t  respectively,  denote  the 
positions  of  those  centres,  and  let  the  lines  or  and  ts  be  drawn  at 
right  angles  to  pc  andjo'c';  then  are  pr  and  p' s  the  respective  dis- 
tances of  the  points  o  and  t  from  the  horizontal  line  pp'. 

Take  the  arithmetical  mean  of  the  two  distances  pr  and  p's9  for 
the  distance  between  the  horizontal  line  pp',  and  the  centre  of  gravity 
of  the  solid  wedge  or  segment  fpcf'p'c'.  Find  similar  distances  for 
all  the  segments  between  the  head  and  stern  of  the  vessel,  for  those 
which  are  elevated  by  the  inclination,  as  well  as  those  which  are 


382  OF  THE  STABILITY  OF  FLOATING  BODIES  AND  OF  SHIPS. 

depressed  by  it ;  then,  if  the  solidity  of  each  segment  is  multiplied 
into  the  distance  of  its  centre  of  gravity  from  the  horizontal  line 
passing  along  pp',  and  produced  both  ways  to  the  head  and  stern  of 
the  vessel ;  the  aggregate  of  the  products  thus  arising,  will  constitute 
the  value  of  the  numerator  dv  of  the  fractional  term  in  equation 
(282a),  where  the  momentum  of  the  vessel's  stability  is 


<dv  7 

,=:< o  sm.d>>?tf. 

L  \  ry 


Consequently,  since  the  several  quantities  w,  v,  S  and  0,  are  either 
given  d  priori,  or  determinable  from  the  circumstances  of  the  case,  it 
follows,  that  the  momentum  of  stability  for  any  angle  of  inclination, 
and  for  any  form  of  body,  can  be  found  by  the  above  formula ;  but 
the  labour  and  intricacy  of  the  calculation,  increases  with  the  irregu- 
larity of  the  body  to  which  such  calculations  are  referred,  and  in 
particular  cases,  the  labour  required  to  accomplish  the  purpose  is 
immensely  great. 


PROBLEM  LXII. 

473.  The  vertical  transverse  sections  of  a  ship,  taken  at  the 
distance  of  five  feet  from  each  other  along  the  principal  longi- 
tudinal axis,  are  thirty-four  in  number,  and  are  bounded  by 
curves  approaching  to  a  parabola  of  a  very  high  order ;  cor- 
responding to  these  are  twelve  horizontal  sections  between  the 
keel  and  the  plane  of  floatation,  taken  at  intervals  of  two  feet 
on  the  vertical  axis,  the  first  section  occurring  at  the  distance 
of  nine  inches  from  the  upper  surface  of  the  keel : — 

It  is  required  to  determine  the  measure  of  stability,  when 
by  the  action  of  the  wind,  or  some  other  equivalent  external 
force,  the  vessel  is  deflected  from  the  upright  position  through 
an  angle  of  thirty  degrees;  the  ordinates  corresponding  to 
the  several  sections,  being  as  registered  in  the  following 
table. 


OF  THE  STABILITY  OF  FLOATING  BODIES  AND  OF  SHIPS. 


383 


TABLE  SHOWING  THE  ORDINATES  CORRESPONDING  TO  THE 
SEVERAL  SECTIONS. 


Horizontal  sections,  intervals  on  the  vertical  axis  2  feet. 

J 

& 

& 

4 

1 

I 

1 
b 

o 

1 

h 
ft 

g 

I 

1 
1 

No. 

1 

ft. 

2 

ft. 

3 

ft. 

4 
ft. 

5 

ft. 

6 

ft. 

7 
ft. 

8 
ft. 

9 
ft. 

10 
ft. 

11 
ft. 

12 
ft. 

1 

2 
3 
4 
5 

6 

7 
8 
9 
10 

2.55 
3.09 

3.50 
6.85 
9.78 

1.80 
6.35 
10.45 
13.50 

4.09 
9.00 
13.20 
15.09 

6.20 
11.30 
15.30 
17.55 

1.70 
8.20 
13.50 
16.85 
18.64 

3.30 
10.00 
15.10 
17.84 
19.30 

20.12 
20.65 
20.90 
21.14 

21.28 

21.51 
21.51 
21.51 
21.35 
21.35 

4.90 
11.73 
16.24 
18.55 
19.73 

6.60 
13.25 
17.08 
19.10 
20.08 

8.10 
14.40 
17.70 
19.45 
20.30 

9.60 
15.35 
18.10 
19.70 
20.50 

20.92 
21.21 
21.44 
21.55 
21.60 

21.60 
21.60 
21.60 
21.60 
21.60 

10.78 
16.00 
18.40 
19.85 
20.54 

5.50 
6.75 
8.00 
9.10 
9.80 

12.15 
13.60 
14.55 
15.20 
15.50 

15.60 
16.80 
17.45 
17.88 
18.15 

18.30 
18.30 
1830 
18.30 
18.30 

17.70 
18.65 
19.05 
19.35 
19.53 

19.60 
19.75 
19.75 
19.75 
19.75 

18.91 
19.60 
20.05 
20.25 
20.40 

20.50 
20.60 
20.60 
20.60 
20.60 

19.65 
20.32 
20.62 
20.78 
20.93 

20.93 
21.10 
21.10 
21.10 
21.10 

21.05 
21.00 
20.90 
20.81 
20.67 

20.45 
20.90 
21.20 
21.34 
21.47 

20.65 
21.05 
21.34 
21.45 
21.50 

20.80 
21.15 
21.38 
21.52 
21.60 

20.94 
21.20 
21.38 
21.48 
21.50 

21.56 
21.58 
21.58 
21.56 
21.56 

21.55 
21.53 
21.51 
21.48 
21.32 

11 
12 
13 
14 
15 

10.50 
10.50 
10.50 
10.50 
10.50 

15.90 
15.90 
15.90 
15.90 
15.90 

21.51 
21.51 
21.51 
21.51 
21.51 

21.59 
21.59 
21.59 
21.59 
21.59 

21.60 
21.60 
21.60 
21.60 
21.60 

16 
17 

18 
19 
20 

10.30 
9.80 
9.20 
8.50 
8.00 

15.70 
15.50 
15.35 
15.00 
14.60 

18.20 
18.05 
17.95 
17.75 
17.52 

19.65 
19.55 
19.45 
19.30 
19.15 

20.52 
20.45 
20.35 
20.25 
20.10 

21.32 
21.30 
21.20 
21.10 
21.00 

21.51 
21.50 
21.40 
21.30 
21.20 

21.59 
21.55 
21.52 
21.44 
21.30 

21.60 
21.60 
21.55 
21.50 
21.35 

21.25 
21.05 
20.83 
20.61 
20.40 

21.60 
21.60 
21.52 
21.50 
21.35 

21 

22 
23 
24 
25 

7.20 
6.40 
5.90 
5.10 
4.20 

14.20 
14.62 
12.90 
11.90 
10.60 

17.25 
16.90 
16.40 
15.70 
14.80 

18.90 
18.62 
18.28 
17.75 
17.10 

19.90 
19.70 
19.30 
19.00 
18.46 

•20.50 
20.32 
20.05 
19.75 
19.30 

21.85 
20.68 
20.40 
20.15 
19.85 

21.05 
20.89 
20.65 
20.40 
20.10 

21.18 
21.00 
20.75 
20.55 
20.30 

21.24 
21.08 
20.82 
20.61 
20.43 

21.22 
21.05 
20.82 
20.61 
20.44 

26 
27 
28 
29 
30 

3.35 
2.50 
1.80 
1.40 
1.11 

9.20 
7.20 
5.38 
3-55 
2.40 

13.40 
11.60 
9.35 
6.65 
4.25 

16.15 
14.80 
12.85 
10.10 
7.05 

17.88 
16.60 
15.45 
13.10 
10.10 

6.12 
2.90 
1.32 
0.75 

18.80 
18.05 
17.11 
15.35 
12.90 

19.35 
18.80 
18.10 
16.85 
15.05 

19.73 
19.30 
18.80 
17.90 
16.80 

20.00 
19.59 
19.22 
18.55 
17.82 

20.10 
19.75 
19.44 
19.00 
18.40 

20.12 
19.80 
19.52 
19.15 
18.62 

20.15 
19.85 
19.58 
19.25 
18.77 

31 
32 
33 
34 

0.90 
0.80 
0.62 
0.60 

1.45 
0.98 
0.75 
0.63 

2.30 
1.25 
0.80 
0.65 

3.75 
1.90 
1.00 
0.70 

9.10 
4.55 
1.85 
0.90 

12.00 
7.10 
2.70 
1.05 

14.60 
10.35 
4.31 
1.35 

16.30 
13.40 
7.45 
1.95 

17.40 
15.65 
14.50 
3.40 

17.90 
16.90 
14.90 
7.00 

18.21 
17.52 
1650 
12.95 

From  these  data,  combined  with  others  that  are  either  assumed  or 
determined  by  the  circumstances  of  the  case,  the  stability  of  the 
vessel  or  the  momentum  of  the  redressing  force  is  to  be  found  by 
calculation ;  it  will,  however,  be  an  improvement  on  the  mode  of 


384         OF  THE  STABILITY  OF  FLOATING  BODIES  AND  OF  SHIPS. 

procedure,  if  in  the  first  place,  we  take  a  brief  survey  of  the  principles 
of  construction;  for  this  purpose,  let  act  represent  any  transverse 


section  of  the  vessel,  at  right  angles  to  the  principal  longitudinal  axis 
passing  through  the  centre  of  effort ;  then  is  ef  the  breadth  of  this 
section  at  the  water  line  when  the  .ship  is  loaded  and  the  plane  of  the 
masts  vertical,  and  hi  becomes  the  water  line,  coincident  with  the 
surface  of  the  fluid,  when  the  vessel  is  deflected  from  the  upright 
position  through  the  given  angle  fki. 

It  is  however  manifest  from  the  ordinates  in  the  foregoing  table, 
that  in  this  case,  the  vertical  sections  are  all  different,  both  in  form 
and  in  magnitude,  and  consequently,  the  primary  and  secondary 
water  lines  do  not  intersect  one  another  in  the  point  k  which  bisects 
ef\  let  p  be  the  point  of  intersection,  and  through  the  point/?,  draw 
the  straight  line  mn  parallel  to  hi,  and  making  with  e/the  angle  fpn 
equal  to  the  given  angle  of  inclination. 

Now,  by  considering  the  conditions  of  the  problem,  it  will  readily 
appear,  that  the  position  of  the  point  p  in  any  of  the  sections 
parallel  to  acz,  cannot  be  determined  on  the  same  principles  by 
which  the  place  of  that  point  was  fixed  according  to  the  foregoing 
solutions,  viz.  by  equating  the  areas  of  the  triangular  spaces  mpe  and 
fpn;  for  it  is  evident,  that  the  volume  which  becomes  immersed 
below  the  fluid's  surface  in  consequence  of  the  inclination,  and  that 
which  emerges  above  by  the  same  cause,  will  not  now  be  proportional 
to  those  areas,  in  the  same  manner  as  they  are,  on  the  supposition  of 
all  the  vertical  sections  being  equal  and  similar  figures. 


OF  THE  STABILITY   OF   FLOATING    BODIES  AND  OF  SHIPS.  385 

In  the  present  instance,  the  vertical  sections  being  all  different, 
both  in  form  and  magnitude,  the  water's  surface  intersecting  the 
vessel  in  the  plane  passing  through  the  line  mn  when  the  vessel  is 
inclined,  will  so  divide  the  areas  of  the  several  sections,  that  although 
the  space  fpn  may  not  be  equal  to  mpe  in  any  one  of  them,  yet  the 
immersed  volume  corresponding  to  all  the  spaces  fpn,  estimated 
from  the  head  to  the  stern  of  the  ship,  shall  be  equal  to  the  volume 
corresponding  to  all  the  emerged  spaces  mpe  estimated  in  the  same 
manner. 

Let  ef,  the  breadth  of  the  section  at  the  water  line,  be  bisected  in 
the  point  k  by  the  vertical  line  dc,  and  suppose  a  plane  to  pass 
through  dc  from  head  to  stern  of  the  vessel,  such  a  plane  will  divide 
the  vessel  into  two  parts  that  are  equal  and  symmetrical,  and  it  will 
pass  through  the  point  k  in  all  the  parallel  vertical  sections  made 
throughout  the  whole  length. 

But  it  is  easily  shown,  that  at  whatever  distance  kp  from  the 
middle  point  k,  the  plane  of  floatation  in  the  inclined  position,  inter- 
sects the  primary  line  ef  in  one  of  the  vertical  sections,  it  will 
intersect  the  corresponding  line  in  all  the  other  sections  at  the  same 
distance  from  the  middle  point;  that  is,  the  distance  kp  will  be  the 
same  in  all  the  parallel  sections,  (the  same  lines  and  letters  of 
reference  being  understood  to  belong  to  each  ;)  for  according  to  the 
conditions  of  the  problem,  the  revolution  of  the  vessel  is  supposed  to 
be  made  about  the  principal  longitudinal  axis,  and  consequently,  the 
intersection  of  the  two  planes  passing  through  the  lines  ef  and  mn, 
must  be  parallel  to  the  axis  of  motion,  and  therefore  parallel  to  the 
line  drawn  through  the  point  k  in  all  the  sections,  estimated  from 
head  to  stern  of  the  ship. 

We  have  now  to  determine  the  distance  kp  at  which  the  inter- 
section takes  place ;  and  for  this  purpose  we  must  consider,  that 
according  to  the  given  conditions,  whatever  may  be  the  position  of 
the  point  p  in  all  the  sections,  if  lines  mn  are  drawn  through  those 
points,  making  with  ef,  an  angle  equal  to  the  given  angle  of  inclina- 
tion;  then  it  is  manifest,  that  the  same  plane  will  pass  through  all 
the  lines  mn  that  occur  betwixt  the  head  and  stern  of  the  vessel. 

It  is  therefore  required  to  determine,  at  what  distance  kp  from  the 
middle  points  k,  the  plane  of  floatation  corresponding  to  the  inclined 
position  of  the  vessel  must  pass,  so  as  to  cut  off  a  volume  on  the 
depressed  side  fpn  equal  to  that  which  rises  above  the  water  on  the 
side  mpe. 

In  each  of  the  parallel  vertical  sections,  let  the  common  line  hi 
VOL.  i.  2  c 


386          OF  THE  STABILITY  OF  FLOATING  BODIES  AND  OF  SHIPS. 

be  drawn  through  k,  the  middle  point  of  ef,  and  inclined  to  ef  at  an 
angle  equal  to  that  of  the  vessel's  deflexion  ;  then,  from  what  we  have 
stated  above,  all  the  lines  hi  will  lie  in  the  same  plane;  that  is,  the 
same  plane  will  pass  through  the  line  hi  in  all  the  sections.  If  there- 
fore, the  areas  of  the  spaces  fk  i  and  h  k  e  in  each  of  the  vertical 
sections,  be  determined  by  some  mode  of  mensuration  adapted  to  the 
particular  case,  it  is  easy  from  these  equidistant  areas,  to  ascertain  the 
solidity  of  the  volumes  contained  between  the  planes  passing  through 
the  lines  kf,  ki  and  he,  kh. 

Put  m  —  the  magnitude  or  solid  contents  of  the  volume,  bounded 

by  the  side  of  the  vessel  and  the  planes  passing  through 

A/and  ki, 
m'—  the  magnitude  or  solid  contents  of  the  volume,  bounded 

by  the  side  of  the  vessel  and  the  planes  passing  through 

ke  and  k  A, 
A  —  the  area  or   superficial  contents   of  the  plane  passing 

through  the  line  hki  in  all  the  sections,  estimated  from 

head  to  stern  of  the  vessel,  which  area  is  determined 

by  having  given  all  the  lines  hi; 
(j)  ~fki,  the  angle  through  which  the  vessel  is  inclined  from 

the  upright  and  quiescent  position,  and 
e  nr  m  —  m',   the   difference   of    the   volumes   or   solidities, 

denoted  by  the  symbols  m  and  m'. 

Then,  if  upon  the  line  kf,  which  coincides  with  the  line  of  floata- 
tion when  the  vessel  is  upright  and  quiescent,  there  be  set  off  in  each 
of  the  parallel  sections,  the  line 

kp— f_ 

AXsin.<^' 

and  through  all  the  points  p  thus  found,  let  lines  mpn  be  drawn 
parallel  to  hi,  and  consequently,  cutting  ef  in  the  points  p,  at  an 
angle  equal  to  that  of  the  vessel's  inclination;  then,  if  a  plane  be 
drawn  through  all  the  lines  mpn,  it  will  so  divide  the  vessel,  that  the 
solidity  of  the  volume  contained  between  the  planes  passing  through 
the  lines  fp  and  np,  will  approximate  to  an  equality  with  the  volume 
contained  between  the  planes  passing  through  the  lines  ep  and  mp. 

Therefore,  since  the  surface  of  the  water  coincides  with  the  plane 
passing  through  ef  when  the  vessel  is  upright,  it  will  also  coincide 
with  the  plane  passing  through  all  the  lines  mpn,  when  the  vessel  is 
deflected  through  the  angle  fpn,  whose  magnitude  is  given. 

It  is  very  easy  to  show,  that  by  setting  off  the  distance  kp  in  all  the 


OF  THE  STABILITY  OF   FLOATING   BODIES  AND  OF  SHIPS.          387 

sections,  as  determined  by  the  preceding  equation,  and  thereby  draw- 
ing a  plane  through  all  the  lines  mpn,  the  plane  thus  drawn  is 
coincident  with  the  water's  surface,  and  is  situated  very  nearly  in 
its  true  position.  For  through  the  point  /<:,  draw  the  line  kr  meet- 
ing mn  perpendicularly  in  the  point  r  ;  then  is  krp  a  right  angled 
triangle  in  which  the  angle  kpr  is  given,  and  by  the  principles  of 
mensuration,  it  is  manifest,  that  the  solid  contained  between  the 
planes  passing  through  the  lines  hki  and  mpn  from  head  to  stern  of 
the  vessel,  is  very  nearly  equal  to  the  area  of  the  plane  passing 
through  hkij  drawn  into  kr  the  perpendicular  thickness  of  the  solid. 
Now,  the  solid  of  which  hinm  is  a  section,  is  obviously  equal  to 
the  difference  of  the  solids  of  which  fki  and  hke  are  sections;  hence 
we  have 

m  —  m  —e  —  AXkr  nearly  ; 
consequently,  by  division,  we  obtain 


and  by  the  principles  of  Plane  Trigonometry,  we  get 

kr  :  kp  :  :  sin.  kpr  :  rad., 
or  by  restoring  the  analytical  values,  it  is 

e 
—  :  kp  :  :  sm.^>  :  rad.  ; 

and  from  this,  by  reducing  the  proportion  and  putting  radius  equal  to 
unity,  we  obtain 

*„  —  _  e_ 

-AXsin.0'  (289). 

An  equation  which  is  very  nearly  true  for  small  inclinations,  and 
this  being  the  case,  it  fully  establishes  the  propriety  of  the  above 
construction  ;  if  the  areas  of  the  planes  passing  through  the  lines  hki 
and  mpn  are  equal  to  one  another,  the  construction  as  thus  effected 
would  be  rigorously  correct. 

474.  In  pursuing  the  construction,  it  will  be  necessary,  in  order  to 
avoid  confusion  in  the  lines  and  letters  of  reference,  to  redraw  that  part 
of  the  section  which  includes  the  angle  of  the  vessel's  inclination,  viz. 
the  space  contained  between  the  sides  of  the  vessel  me,  nf  and  the 
dotted  lines  en,  inf.  We  shall  not,  however,  attempt  to  preserve  the 
due  proportion  between  the  several  parts  of  the  figure  ;  this  indeed 
would  be  troublesome  and  altogether  unnecessary,  since  it  is  the  prin- 
ciples of  construction  only  that  we  mean  to  illustrate,  and  not  the 
actual  solution  of  any  particular  example. 

2c2 


388 


OF  THE  STABILITY  OF  FLOATING  BODIES  AND  OF  SHIPS. 


.7 


Let  enfm  be  the  space  in  question,  including  the  angle  of  the 
vessel's  inclina- 
tion ;  draw  the 
lines  me  and  nf» 
cutting  off  the 
curvilinear  areas 
mre  and  nsf; 
bisect  the  sides 
me,  pe  in  the 
points  u  and  TT, 
and  draw  the 
lines pu  and  mir 

intersecting  each  other  in  l\  /is  the  centre  of  gravity  of  the  triangular 
space  mpe.  Suppose  z  to  be  the  centre  of  gravity  of  the  curvilinear 
segment  mre,  and  through  the  points  I  and  z,  draw  lq  and  zy  respec- 
tively perpendicular  to  mn,  the  line  of  floatation  in  the  inclined  posi- 
tion of  the  vessel. 

Again,  bisect  the  sides  w/and/p  in  the  points  w,  $,  and  draw  pw 
and  n(f>  intersecting  each  other  in  the  point  o\  o  is  the  centre  of 
gravity  of  the  triangular  space  npf.  Let  v  be  the  centre  of  gravity 
of  the  curvilinear  area  n  sf,  and  through  the  points  o  and  v,  draw  the 
straight  lines  ot  and  vx  respectively  perpendicular  to  the  water  line 
mpn;  then,  in  the  line  tx  intercepted  by  the  perpendiculars  ot  and 
vx,  take  tc  such,  that  it  shall  be  to  £0:  in  the  same  proportion,  as  the 
curvilinear  space  nsf,  is  to  the  compound  space  pnsf,  and  by  the 
property  of  the  centre  of  gravity,  c  will  be  the  point  in  mn,  where  it 
is  intersected  by  the  perpendicular  through  the  common  centre  of  the 
triangular  and  curvilinear  spaces  npf  and  n  sf. 

Through  the  point  p  in  all  the  sections,  let  a  line  PQ  be  drawn  at 
right  angles  to  mn ;  then,  the  same  plane  will  pass  through  all  these 
lines,  and  cp  will  be  the  perpendicular  distance  of  this  plane,  from 
the  centre  of  gravity  of  the  mixed  space  pnsf.  Therefore,  if  the 
products  arising  from  multiplying  each  area,  into  the  distance  pc  of 
its  centre  of  gravity  from  the  plane  passing  through  PQ,  be  truly  cal- 
culated in  all  the  sections  contained  between  the  head  and  stern  of 
the  vessel ;  then,  by  the  principle  announced  and  demonstrated  in 
Proposition  (A),  Chapter  I,  the  distance  of  the  centre  of  gravity  of 
the  volume,  whose  sections  are  represented  by  all  the  areas  pnsf, 
from  the  vertical  plane  passing  through  PQ  can  easily  be  ascertained. 

Let  pR  be  that  distance,  and  by  a  similar  mode  of  computation, 
suppose  that  pE  is  found  to  be  the  corresponding  distance  of  the 


OF  THE  STABILITY  OF  FLOATING  BODIES  AND  OF  SHIPS.  389 

centre  of  gravity  of  the  volume  whose  sections  are  the  areas  pmre  ; 
then  is  B  R  the  horizontal  distance  between  the  centres  of  gravity  of 
the  volumes  that  are  respectively  immersed  and  emerged,  below  and 
above  the  water's  surface,  in  consequence  of  the  vessel  being  deflected 
from  the  upright  position,  through  an  angle  of  which  the  magnitude 
is  known. 

475.  The  solid  content  of  the  entire  volume  immersed,  or  the  quan- 
tity of  water  displaced  by  the  immersed  part  of  the  vessel,  is  to  be 
obtained  from  the  areas  of  the  several  horizontal  sections ;  for  the 
ordinates  drawn  in  the  several  sections  being  arranged  in  regular  order, 
after  the  manner  which  we  have  adopted  in  the  preceding  table,  the  area 
of  any  section  can  readily  be  assigned,  by  methods  of  approximation 
adapted  to  the  particular  case,  and  from  these  areas  the  solidity  of 
the  immersed  volume  is  to  be  inferred  ;  making  allowance  for  the 
irregularities  of  the  vessel  towards  the  head  and  stern,  if  it  be  at  all 
necessary  to  take  those  parts  into  the  account ;  in  all  practical  cases, 
however,  they  may  safely  be  omitted. 

That  part  of  the  immersed  volume,  comprehended  between  the  keel 
and  the  lowest  horizontal  section,  is  obtained,  by  first  finding  the  areas 
of  the  several  vertical  planes,  between  the  keel  and  the  nearest  ordi- 
nates, and  from  these  areas,  by  means  of  some  appropriate  mode  of 
approximation,  the  magnitude  of  the  part  cut  off  by  the  lowermost 
horizontal  plane  will  be  determined  ;  which  being  added  to  the  solidity 
of  the  part  contained  between  the  extreme  planes,  will  give  the  mag- 
nitude of  the  immersed  volume,  or  the  quantity  of  fluid  displaced. 

476.  Referring  to  the  original  diagram,  it  will  be  observed,  that  from 
the  areas  of  the  several  horizontal  sections,  made  between  the  keel  of 
the  vessel  and  the  plane  of  floatation,  the  distance  kg,  that  is,  the 
distance  between  the  water  line  ef  and  the  centre  of  buoyancy,  or 
the  centre  of  gravity  of  the  immersed  volume,  can  also  be  determined 
by  the  application  of  particular  approximating  rules,  and  the  best 
with  which  we  are  acquainted  for  this  purpose,  are  those  given  by 
Stirling  in  his  "  Methodus  Differ  enlialis"  and  by  Simpson  in  his 
"  Essays ;"  these  rules  may  be  expressed  in  general  terms  as 
follow. 

RULE  1.    Jyx=i(?—\S}Xr, 

where  x  is  the  fluxion  of  the  abscissa,  y  the  perpendicular  ordinate, 
expressing  a  general  term  or  function  of  x ;  r  the  common  distance 
between  the  ordinates ;  S  the  sum  of  the  first  and  last  ordinates,  and 
p  the  sum  of  the  whole  series. 


390          OF  THE  STABILITY  OF  FLOATING  BODIES  AND  OF  SHIPS. 

RULE  2.    y#*  =  (S-f4p-|-2Q)Xir, 

where  x,  y,  r  and  5  denote  as  in  rule  1st;  p  the  sum  of  the  2nd, 
4th,  6th,  8th,  &c.  ordinates,  and  Q  the  sum  of  the  3rd,  5th,  7th,  9th, 
&c.  (the  last  of  the  series  excepted). 


RULES.        /X=(S  +  2p  +  3g)X|r, 

here  again,  x,  y,  r  and  S  denote  as  in  the  preceding  cases  ;  P  the  sum 
of  the  4th,  7th,  10th,  13th,  &c.  ordinates  (the  last  excepted),  and  Q 
the  sum  of  the  2nd,  3rd,  5th,  6th,  8th,  9th,  &c. 

With  respect  to  the  applicability  of  the  above  rules,  it  may  be 
observed,  that  the  first  approximates  to  the  fluent,  whatever  may  be 
the  number  of  the  given  ordinates,  and  the  second  only  requires  that 
the  number  of  ordinates  shall  be  odd.  But  in  order  to  apply  the 
third  rule,  it  is  a  necessary  condition,  that  the  number  of  given 
ordinates  shall  be  some  number  in  the  series  4,  7,  10,  13,  16,  &c.  ; 
that  is,  the  number  of  ordinates  must  be  some  multiple  of  3  increased 
by  unity.  In  every  case,  however,  the  approximate  fluent  can  be 
obtained,  either  from  the  second  or  third  rule  considered  separately, 
or  from  both  taken  conjointly. 

477.  But  to  return  from  this  short  digression,  we  may  remark,  that 
the  position  of  the  point  G,  which  marks  the  centre  of  effort,  or  the 
centre  of  gravity  of  the  whole  vessel,  depends  partly  on  the  equipment 
and  construction,  and  partly  upon  the  distribution  of  the  loading  and 
ballast;  which  circumstances,  therefore,  determine  G#,  the  distance 
between  the  centre  of  effort  and  the  centre  of  buoyancy  when  the 
vessel  is  upright. 

These  several  conditions  having  been  determined,  the  remaining 
part  of  the  construction,  limiting  the  measure  of  the  vessel's  stability, 
may  be  effected  as  follows. 

Through  g  the  centre  of  buoyancy,  or  the  centre  of  gravity  of  the 
immersed  volume,  draw  gt  parallel  to  mn,  and  make  gt  to  BR  (see 
the  subsidiary  figure),  as  the  volume  immersed  in  consequence  of  the 
inclination,  is  to  the  whole  immersed  volume  induced  by  the  weight 
of  the  vessel  ;  through  G  the  centre  of  effort,  draw  GZ  parallel  and  05 
perpendicular  to  gt,  and  through  t  draw  tm  parallel  to  GS,  and  meet- 
ing the  axis  cd  in  M  ;  then  is  M  the  metacentre,  and  GZ  the  measure 
of  the  vessel's  stability  when  inclined  from  the  upright  position  through 
the  angle  fpn  or  gGs. 

The  principles  of  the  preceding  construction  are  general,  and  can 
be  applied  in  all  cases,  whatever  may  be  the  figure  of  the  vessel,  or 


OF  THE  STABILITY  OF  FLOATING  BODIES  AND  OF  SHIPS.  391 

the  nature  of  its  bounding  curves  ;  but  the  arithmetical  operations,  as 
applied  to  any  particular  case,  are  unavoidably  tedious,  and  necessa- 
rily extend  to  considerable  length ;  they  are,  however,  very  far  from 
being  difficult,  as  the  ensuing  process  will  fully  testify. 

478.  By  referring  to  the  table  of  ordinates,  it  will  appear,  that  the 
greatest,  or  principal  transverse  section,  intersects  the  longer  axis,  at 
about  the  distance  of  60  feet,  or  12  intervals  from  the  section  nearest 
to  the  head  of  the  ship ;  we  shall  therefore  delineate  that  section,  and 
in  order  that  nothing  may  be  wanting  to  the  proper  understanding  of 
the  subject,  we  shall  also  delineate  the  plane  of  floatation,  which  cor- 
responds to  the  twelfth  horizontal  section  in  the  preceding  table  of 
ordinates. 

The  ordinate  in  the  table  opposite  the  twelfth  vertical  and  under 
the  twelfth  horizontal  section,  is  21.58  feet,  and  the  whole  vertical 
distance  between  the  keel  and  the  plane  of  floatation,  is  22.75  feet; 
therefore,  draw 
the     horizontal 
line   a  a   which 
make   equal  to 
43.16  feet,  and 
bisect  a  a  per- 
pendicularly by 
AK     equal     to 
22.75  feet. 

Divide  the 
vertical  axis  A  K 

into  twelve  parts,  eleven  of  which  are  2  feet  each,  and  the  first  or 
lowermost  only  three  fourths  of  a  foot,  or  nine  inches  ;  then,  through 
the  several  points  of  division  1,  2,  3,  4,  5,  6,  &c.  and  parallel  to  aa, 
draw  the  several  ordinates,  taken  from  the  twelfth  horizontal  row  in 
the  preceding  table,  which  set  off  both  ways,  and  through  the  extre- 
mities of  the  several  ordinates,  let  the  curve  line  ana  be  drawn,  which 
will  represent  the  boundary  of  the  principal  lateral  section,  so  far  as 
it  is  immersed  below  the  fluid's  surface. 

And  exactly  in  the  same  manner  may  the  whole  of  the  34  vertical 
sections,  into  which  the  longer  axis  is  divided,  be  delineated ;  but  the 
above  being  sufficient  for  illustration,  we  shall  next  proceed  to  describe 
the  twelfth  horizontal  section  which  is  coincident  with  the  water's 
surface,  and  of  which  the  greatest  ordinate  is  21.58  feet,  correspond- 
ing to  a  A  in  the  above  vertical  section. 

479.  Since  the  body  of  the  vessel  is  divided  at  intervals  of  5  feet  into 
34  vertical  sections,  it  follows,  that  between  the  first  section  adjacent 


392 


OF  THE  STABILITY  OF  FLOATING  BODIES  AND  OF  SHIPS. 


to  the  head,  and  the  34th  section  adjacent  to  the  stern,  there  must  be 
33  intervals,  or  33x5  =  165  feet;  therefore,  draw  the  horizontal  line 
mn  to  represent  the  longer 
axis  of  the  plane  of  floata- 
tion, and  make  be  equal 
to  165  feet,  which  divide 
into  33  equal  intervals  of 
5  feet  each;  then  at  right 
angles  to  mn  and  through 
the  several  points  of  divi- 
sion 1,2,3,4,  5,  &c.  to  34, 
draw  the  ordinates  dd,  ee, 
//,  a  a,  gg,  kh,  &c.  to  iit 
and  from  a  scale  of  equal 
parts,  of  the  same  dimen- 
sions as  that  from  which 
be  is  taken,  set  off  both 
ways,  (beginning  at  the 
1st  division  adjacent  to  the 
head  of  the  vessel),  the 
numbers  contained  in  the 
twelfth  column  of  the  pre- 
ceding table;  then, through 
the  extremities  of  the  seve- 
ral ordinates,  let  the  curve 
man  a  be  drawn,  and  it 
will  represent  the  plane  of 
floatation  when  the  vessel 
is  upright,  according  to  the 
foregoing  tabulated  mea- 
surements. It  is  manifest, 
that  by  a  similar  mode  of 
procedure,  the  eleven  remaining  horizontal  sections,  situated  between 
the  keel  and  the  plane  of  floatation,  might  also  be  delineated;  it  is, 
however,  unnecessary  to  pursue  the  subject  of  construction  farther, 
since  what  has  already  been  done,  is  quite  sufficient  to  show  the 
reader,  the  method  and  nature  of  the  delineation  when  pursued 
throughout  the  entire  vessel.* 


*  It  may  be  proper  to  remark,  that  the  scales  from  which  the  vertical  and 
horizontal  sections  have  been  constructed,  are  to  one  another  as  2  to  1 ;  the  one  for 
the  vertical  section  being  l-20th  of  an  inch  to  a  foot,  and  the  other  l-10th. 


OF  THE  STABILITY  OF  FLOATING  BODIES  AND  OF  SHIPS.  393 

480.  It  now  only  remains  to  calculate  the  measure  of  stability,  when 
the  vessel  is  deflected  from  the  upright  position  through  an  angle  of  30 
degrees,  and  for  this  purpose  we  must  again  refer  to  the  original 
diagram,  where,  on  the  supposition  that  it  is  correctly  constructed, 
the  lines  ki  and  kh  are  to  be  carefully  measured  in  each  of  the  sec- 
tions, on  the  same  scale  with  the  original  dimensions;  then,  if  the 
lines  fi  and  he  be  drawn,  the  areas  of  the  triangular  spaces  fki  and 
like,  can  easily  be  determined  from  the  two  sides  and  the  included 
angle ;  and  if  a  series  of  perpendicular  ordinates  be  measured  on  the 
lines  fn  and  he,  the  areas  of  the  curvilinear  spaces/#i  and  hye  may 
from  thence  be  found,  which  being  added  to  the  triangles  fki  and 
hke,  the  sums  will  be  the  whole  areas  of  the  compound  spaces  fki  x 
and  hkey. 

Pursuing  a  similar  process  throughout  the  34  vertical  sections,  we 
shall  at  last  arrive  at  the  magnitudes  of  the  volumes  which  are  con- 
tained between  the  planes  passing  through  kf,  ki  and  kh,  he,  a 
knowledge  of  these  volumes  being  necessary  to  determine  the  posi- 
tion of  the  point  p. 

It  is  presumed  that  it  will  be  sufficient  for  the  exemplification  of  the 
rules,  to  exhibit  the  calculation  of  one  of  the  spaces  fk  ix  ;  to  perform 
the  operation  for  the  whole  series,  would  be  a  very  tedious  and  at  the 
same  time  a  superfluous  proceeding;  and  for  this  reason,  that  the 
constructions  and  calculations  founded  on  them,  for  inferring  the 
results  in  any  one  of  the  sections,  are  similar  to  those  required  for 
obtaining  the  corresponding  results  in  any  other  section ;  and  this 
being  the  case,  the  representation  of  one  process  will  suffice  for  all 
the  rest. 

481.  But  to  proceed,  the  line  ki  being  taken  in  the  compasses,  and 
applied  to  an  accurate  scale  of  the  proper  dimensions,  it  is  found  to 
indicate  22.6  feet,  and  according  to  the  table  of  ordinates,  the  line  fk 
is  21.58   feet;    and  moreover,  according  to  the   conditions  of  the 
problem,  the  angle  of  inclination,  or  that  contained  between  the  lines 
kf  and  ki,  is  equal  to  30  degrees,  of  which  the  natural  sine  is  j,  radius 
being  unity;  consequently,  if  a  denote  the  area  of  the  triangle  fki, 
we  have  by  the  principles  of  mensuration, 

a=|(22.58x21.6)=:  121.927  square  feet. 

Now,  according  to  the  principles  of  Plane  Trigonometry,  the  line/i 
is  expressed  by  the  equation 


;  — V/22.6H-21.582— 2x22.6X21.58xcos.30°zz:  11.55  feet. 


394          OF  THE  STABILITY  OF  FLOATING  BODIES  AND  OF  SHIPS. 


consequently,  if  this  line  be  divided  into  six  equal  parts  of  1.925  feet 
each,  and  perpendicular  ordinates  be  erected  thereon,  they  will  be 
found  to  measure  as  follows,  viz. 


No.  of  ordinates 
Feet     -     -     - 


0.15 


3 
0.30 

4 
0.43 

5 
0.38 

6 
0.23 

Therefore,  if  a'  denote  the  area  of  the  curvilinear  space  fxi ;  then 
by  the  second  of  the  preceding  approximating  rules,  we  have 

•S—  0  4-  0  —  0,  the  sum  of  the  extreme  ordinates, 

4p  =  4(.15  4-  .43  4-  .23)  =  3.24,  the  second  term  of  the  series, 

2Q  =  2(.30  4-  .38)  —  1.36,  the  third  and  last  term  ; 

hence,  by  addition,  we  get 
S  _|_  4?  4-  2Q  =  0  4-  3.24  +  1 .36  =  4.6  ; 

but  one  third  of  the  common  interval  is  0.642  of  a  foot  nearly ;  con- 
quently,  by  multiplication,  the  area  of  the  curvilinear  space  fxi, 
becomes 

a1  =  .642  X  4.6  =  2.9532  square  feet, 

which  being  added  to  the  area  of  the  triangle  above  determined,  the 
area  of  the  compound  space  fk  i  x  becomes 

a  4-  a'  =  121.927  4-  2.953=  124.88  square  feet. 

Again,  if  we  put  b  to  denote  the  area  of  the  triangle  hke,  and  b' 
the  area  of  the  curvilinear  space  hye;  then,  by  proceeding  in  a 
manner  similar  to  the  above,  the  area  of  the  mixed  space  hkey 
becomes 

b  4-  b'  =  1 33.68  square  feet. 

Now,  if  in  this  way,  the  values  of  a  4-  a'  and  b  4-  b'  be  calculated 
for  each  of  the  34  vertical  sections,  the  several  results  will  be  as  exhi- 
bited in  the  following  table. 


OF  THE  STABILITY  OF  FLOATING  BODIES  AND  OF  SHIPS. 


395 


Table  of  Areas  for  the  thirty -four  Vertical  Sections. 


No.  of 

Values  of 

Values  of 

No.  of 

Values  of 

Values  of 

sections. 

a  -\-  a' 

&  +  &' 

sections. 

a  +  a' 

6  +  6' 

feet. 

feet. 

feet. 

feet. 

1 

42.86 

23.61 

18 

124.62 

132.53 

2 

81.53 

58.92 

19 

123.91 

131.05 

3 

100.80 

86.80 

20 

123.21 

129.57 

4 

114.16 

105.27 

21 

121.06 

127.48 

5 

121.56 

115.70 

22 

118.91 

125.40 

6 

121.75 

120.90 

23 

117.50 

122.66 

7 

123.47 

125.36 

24 

116.10 

119.93 

8 

125.20 

129.82 

25 

114.01 

116.88 

9 

124.87 

131.04 

26 

111.91 

113.83 

10 

124.54 

132.27 

27 

108.96 

109.81 

11 

124.69 

132.97 

28 

106.01 

105.80 

12* 

124.88 

133.68 

29 

101.82 

98.92 

13 

124.87 

133.68 

30 

97.24 

91.71 

14 

124.87 

133.68 

31 

92.41 

79.95 

15 

124.82 

133.42 

32 

86.31 

66.06 

16 

124.78 

133.17 

33 

81.60 

48.20 

17 

124.20 

132.85 

34 

68.35 

17.92 

Sums 

1953.85 

1963.14 

Sums 

1813.93 

1737.70 

Therefore,  the  sum  of  all  the  (a  -f-  a')"zz  3767.78  ;  and  the  sum  of 
all  the  (b  -f  i'yzz  3700.84,  and  by  the  conditions  of  the  problem,  the 
vertical  sections  intersect  the  principal  longitudinal  axis  at  intervals 
of  5  feet ;  therefore,  by  applying  the  third  of  the  preceding  approxi- 
mative rules,  the  solid  contents  of  the  volume  contained  between  the 
planes  passing  through  fk  and  ki,  will  be  found  as  follows. 

S=  42.86  +  68.35  zz  1 1 1.21,  the  sum  of  the  extreme  ordinates, 

2pzz(611.82  -f  555.25) X 2=  2334. 14,  the  2nd  term  of  the  series, 
or  twice  the  sum  of  the  4th,  7th,  10th,  13th,  &c.  ordinates, 

3Qzz(1299.17  -\-  1 190. 33)  X  3  zz  7468. 5,  the  third  term  of  the  series, 
or  the  sum  of  the  2nd,  3rd,  5th,  6th,  8th,  9th,  &c.  ordinates ; 

consequently,  by  addition,  we  have 
S  -f  2r  -f  SQ  zz  1 1 1.21  -f  2334. 14  -f  7468.5  zz  9913.85 ; 


*  This  is  the  vertical  section  for  which  we  have  exhibited  the  process  of  com- 
putation. 


396          OF  THE  STABILITY  OF  FLOATING  BODIES  AND  OF  SHIPS. 

and  finally,  by  multiplying  by  three  eighths  of  the  common  interval, 
the  magnitude  of  the  volume  becomes 

m  =  (S4-2p-f3Q)X5Xf  z=  9913.85  x  V  =  18588.47  cubic  feet 

very  nearly. 

Proceeding  exactly  in  the  same  manner  with  the  areas  (b  -\-  £>'),  the 
solidity  of  the  space  comprehended  between  the  planes  passing  through 
the  lines  kh  and  ke,  and  the  intercepted  side  of  the  vessel,  becomes 

m'=.  (5-f  •  2p  4-  3Q)X  y  =  18433.47  cubic  feet; 

therefore,  by  subtraction,  we  obtain 
m  —  m'=  e  =  18588.47  —  1  8433.47  =  155. 

482.  In  the  next  place,  we  have  to  determine  the  area  of  the  plane 
passing  through  all  the  lines  hki  in  the  several  vertical  sections  ;  this 
is  effected  by  measuring  all  the  ordinates  in  that  plane,  taken  at  the 
common  interval  of  5  feet  along  the  axis  passing  through  k  from  head 
to  stern  of  the  vessel. 

When  this  operation  is  performed  in  a  dexterous  manner,  the  area 
of  the  plane  will  be  found  to  be  7106  square  feet  very  nearly  ;  that  is, 

A  =  7106  square  feet; 
consequently,  by  equation  (412),  we  have 


Hence  it  appears,  that  the  distance  of  the  pointy  from  the  middle 
point  k,  is  too  small  to  cause  any  material  error  in  the  result,  we  shall 
therefore  suppose  that  the  plane  of  floatation  corresponding  to  both 
positions  of  the  vessel,  intersect  each  other  in  the  axis  passing  through 
k  from  head  to  stern  of  the  vessel.  Taking,  therefore,  the  mean 
between  the  two  foregoing  solidities,  we  shall  have 

$(ro  4-  m')  —  |(18588.47  -f  18433.47)  =  18510.97  cubic  feet. 

This,  therefore,  is  the  solidity  of  the  volume  which  becomes  immersed 
in  consequence  of  the  inclination  ;  and  by  pursuing  a  similar  mode  of 
procedure  with  respect  to  the  areas  of  the  twelve  horizontal  sections, 
the  solidity  of  the  whole  volume  immersed,  will  be  found  to  be  119384 
cubic  feet  very  nearly  ;  and  moreover,  by  referring  to  the  subsidiary 
figure  employed  in  the  construction,  and  introducing  the  principles  by 
which  the  distance  BR  is  ascertained,  we  shall  have 
BRzr27.32  feet; 

consequently,  by  Proposition  XII.,  Chapter  XIII.,  the  distance  gt  in 
the  original  figure  is  thus  found, 


OF  THE  STABILITY  OF  FLOATING  BODIES  AND  OF  SHIPS.          397 

119384  :  27.32  :  :  18510.97  :  4.25  feet, 

483.  But  in  order  to  infer  the  stability  of  the  vessel  from  the  value 
of  gt  just  discovered,  it  is  necessary  to  have  given  the  distance  eg,  or 
the  distance  between  the  centre  of  effort  and  the  centre  of  buoyancy  ; 
now  it  is  obvious,  that  the  position  of  this  latter  point  is  regulated 
entirely  by  the  form  and  dimensions  of  the  immersed  portion  of  the 
vessel,  and  consequently,  it  may  be  considered  as  absolutely  fixed 
with  respect  to  the  plane  of  floatation  ;  but  since  the  position  of  the 
centre  of  effort  is  regulated  partly  by  the  construction  and  equipment, 
and  partly  by  the  distribution  of  the  loading  and  ballast,  it  can  only 
be  assumed  on  the  ground  of  supposition,  unless  in  cases  where  the 
position  of  that  point  has  been  actually  ascertained  by  accurate 
mensuration. 

In  several  instances,  the  distance  Gg  has  been  measured,  and  found 
to  be  equal  to  about  one  eighth  of  the  greatest  breadth  at  the  plane 
of  floatation  ;  therefore,  by  assuming  this  to  be  the  case  generally,  we 
have 

eg  ==  J-  of  43.16  =  5.396  feet, 

therefore,  by  Plane  Trigonometry,  it  is 

rad.  :  5.396  :  :  sin. 30°  :  gs, 
from  which,  by  reducing  the  proportion,  we  obtain 

#5  =  2.698  feet, 
which  being  subtracted  from  g  t,  the  remainder  is 

S£=:GZ  =  4.25  — 2.698=:1.552  feet,  the  measure  of 
the  vessel's  stability,  or  the  length  of  the  equilibrating  lever.  But  the 
whole  weight  of  the  vessel,  as  found  from  the  solidity  of  the  immersed 
part,  is 

119384 

w  nr —  341 1  tons  very  nearly  ;   35  cubic  feet 

35 

of  sea  water  being  equal  to  one  ton  weight ;  consequently,  the 
momentum  of  the  redressing  force,  or  the  power  which  the  pressure 
of  the  water  exerts  to  restore  the  vessel  to  the  upright  position,  is 
equal  to  3411  tons  acting  on  a  lever  whose  length  is  1.552  feet;  or 
it  is  equivalent  to  a  force  or  pressure  of  245  tons  acting  at  the  dis- 
tance of  half  the  greatest  breadth  of  the  vessel  from  the  axis  ;  for  by 
the  principles  of  the  lever,  it  is 

21.58  :  3411  :  :  1.552  :  245  tons  nearly. 

The  preceding  is  the  method  of  determining  the  stability  of  a  ship, 
on  the  supposition  that  the  data  are  all  assignable ;  the  process  con- 


398  OF  THE  STABILITY  OF  STEAM  SHIPS. 

sidered  in  its  full  extent  is  unavoidably  tedious  and  prolix,  we  have 
merely  pointed  out  the  method  of  conducting  the  calculations  ;  but 
when  it  is  necessary  to  determine  the  stability  of  a  ship  in  actual 
practice,  every  individual  process  must  be  separately  performed,  and 
the  result  obtained  as  above,  will  approximate  very  nearly  to  the 
truth. 

484.  Those  who  have  ever  witnessed  the  spectacle  of  a  ship  tossed  in 
a  tempest,  or  have  read  any  of  the  brilliant  accounts  which  maritime 
tales  afford,  will  appreciate  the  subject  we  have  just  investigated. 
They  may  have  seen,  moreover,  the  vast  bulwark  slide  from  her  cradle 
into  the  calm  water,  on  which  she  first  swung  round  and  heeled  till 
she  regained  her  stability  of  equilibrium  ;  giving  the  imagination  a 
contrast  of  the  stormy  element  on  which  she  was  soon  to  ride  in  awful 
grandeur.     But  seamen  will  best  appreciate  our  labours,  especially 
those  who  in  the  days  of  battle  and  the  nights  of  danger  have  had  to 
manage  the  noblest  work  of  art  and  skill ;   and  who  in  their  country's 
cause  have  encountered  all  weathers  and  every  clime,  traversing  the 
wide  expanse  of  ocean's  bosom,  visiting  all  the  ends  of  the  earth,  and 
identifying  themselves  as  part  of  the  stupendous  ship  which  figuratively 
has  to  do  and  to  suffer  for  her  country,  and  which  in  peace  or  in  war,  in 
sunshine  or  in  storm,   carries  with  her  the  benediction    of  mankind 
pronounced  as  on  a  living  being,  when  she  was  first  launched  in  pre- 
sence often  thousand  enthusiastic  spectators,  one  and  all  sympathizing 
in  the  national  solemnity. 

4.    PRINCIPLES  OF  THE  STABILITY  OF  STEAM  SHIPS. 

485.  When  a  ship  is  set  afloat  upon  the  surface  of  the  waters,  and 
impelled  by  some  power  acting  in  the  direction  of  its  length,  as  is  the 
case  with  steam  vessels,  now  so  extensively  employed,  the  subject  of 
stability  becomes  of  very  great   importance.     This  remark  does  not 
strictly  apply  to  vesels  navigating  still  waters,  or  rivers  where  the 
tides  produce  but  small  effects  ;  but  it  is  well  known  that  the  natural 
motion  of  the  sea,  even  in  its  calmest  state,  causes  a  considerable 
lateral  motion  in  a  vessel  placed  upon  its  surface,  and  in  consequence 
of  this  motion,  the  paddles  are  made  to  dip  unequally  in  the  water, 
by  which  means  some  part  of  the  impelling  power  is  lost. 

It  is  with  the  view  of  avoiding  this  waste  of  power,  that  the  subject 
of  stability  acquires  such  vast  importance  when  referred  to  steam 
vessels  ;  and  it  is  easy  to  perceive,  that  the  best  method  of  attaining 
this  object,  is  to  adapt  the  form  and  capacity  of  the  vessel  to  the 


OF  THE  STABILITY  OF  STEAM  SHIPS.  399 

several  circumstances  by  which  the  floatation  is  regulated,  and  on 
which  the  mode  of  action  depends. 

The  late  Thomas  Tredgold  has  considered  this  subject  in  his  work 
on  the  STEAM  ENGINE,  and  his  views  in  this  case,  as  in  all  others 
where  the  powers  of  his  comprehensive  and  refined  mind  have  been 
called  into  action,  are  concise,  elegant,  and  original ;  and  we  cannot 
close  this  chapter  to  greater  advantage  than  by  adopting  his  theory, 
which  however  we  shall  modify  to  suit  the  plan  and  arrangement  of 
the  present  work. 

486.  In  order  to  simplify  the  investigation  of  stability,  Tredgold 
considers  the  vessel  to  be   a  solid  homogeneous  body  of  the  same 
density  as  water,  with  vertical  or  circular  surfaces  at  the  water  lines 
when  the  vessel  is  in  a  state  of  quiescence.     Now,  it  is  obvious,  that 
with  regard  to  a  ship  which  is  designed  to  carry  burdens  at  sea,  the 
first  of  these  conditions  cannot  obtain ;  this  however  is  of  no  conse- 
quence as  respects  the  result  of  the  inquiry,  for  in  reality  it  refers  to 
a  mass  of  water  equal  in  bulk  to  the  immersed  portion  of  the  floating 
body.    As  another  means  of  simplification,  he  supposes  the  transverse 
sections  of  the  ship  at  right  angles  to  the  axis  of  motion  to  be  in  the 
form  of  a  parabola,  of  which  the  equation  is  px=iyn,  and  for  the 
purpose  of  contrasting  the  extremes  of  form,  he  branches  the  subject 
into  the  two  following  varieties,  viz. 

1 .  When  the  ordinates  are  parallel  to  the  depth,  and 

2.  When  the  ordinates  are  parallel  to  the  breadth. 

For  each  of  these  cases  a  general  equation  is  deduced,  involving 
the  sine  of  the  angle  of  inclination,  the  breadth  and  depth  of  the 
vessel,  and  the  index  or  exponent  by  which  the  order  of  the  parabola 
is  expressed. 

487.  Having    already  investigated   an  expression  by  which  the 
stability  of  a  floating  body  is  indicated,  we  do  not  consider  it  neces- 
sary to  trace  the  steps  of  inquiry  in  the  present  instance,  for  the 
intelligent  reader  will  at  once  perceive,  that  although  the  form  of  the 
equation  is  somewhat  different,  by  reason  of  its  involving  different 
parts  and  different  data,  yet  the  principles  upon  which  the  investiga- 
tion proceeds,  are,  and  necessarily  must  be,  the  same,  or  nearly  the 
same  as  before. 

488.  When  the  ordinates  of  the  parabola  are  parallel  to  the  depth, 
the  general  equation  by  which  the  stability  is  indicated,  becomes 

(290). 


400  OF  THE  STABILITY  OF  STEAM  SHIPS. 

where  £>  —  DC  is  the  breadth  of  the  vessel  at  the  water  line  when 
upright  and  quiescent,  C?=LK  the  corresponding  depth,  0  —  FPC 
the  angle  of  inclination  from  the  upright  position,  n  the  exponent 
denoting  the  order  of  the  parabolic  section,  and  S  the  stability. 

489.  If  we  examine  the  structure  of  the  above  equation,  it  will 

readily   appear,  that  while   b*  is  greater  than   -  '  —  ,  the  stability  is 

n  —  |—  A> 

positive,  and  the  vessel  endeavours  to  regain  the  upright  position  ;  if 
these  two  quantities  are  equal  to  one  another  there  is  no  stability  ; 
and  if  the  latter  exceeds  the  former,  the  stability  is  negative,  and  the 
vessel  oversets.  Hence  it  appears,  that  between  the  breadth  and 
depth  of  the  vessel,  a  certain  relation  must  obtain  to  render  it  fit  and 
sufficiently  stable  for  the  purposes  of  navigation;  and  it  is  further 
manifest,  that  the  stability  increases  directly  as  the  exponent  of  the 
ordinate,  so  does  the  area  of  the  transverse  section  ;  but  in  order  to 
give  the  proper  degree  of  stability,  the  breadth  must  increase  more 
rapidly  than  the  depth. 

By  giving  different  values  to  the  symbol  n  in  the  preceding 
general  equation,  we  shall  obtain  expressions  to  indicate  the  stability 
for  sections  of  different  forms  ;  thus  for  instance,  if  n  zz  1  the  section 
is  a  triangle,  and  the  expression  for  the  stability  becomes 


(291). 

490.  This  equation  is  very  simple,  and  can  easily  be  illustrated  by  an 
example  ;  the  practical  rule  for  its  reduction  may  be  expressed  in  the 
following  terms. 

RULE.  From  the  square  of  the  breadth  of  the  water  line 
when  the  vessel  is  upright,  subtract  twice  the  square  of  the 
corresponding  depth  ;  multiply  the  remainder  by  the  breadth 
drawn  into  the  natural  sine  of  the  angle  of  inclination,  and 
one  twelfth  of  the  product  will  express  the  stability. 

491.  EXAMPLE.   A  floating  body  in  the  form  of  a  triangular  prism, 
has  its  breadth  at  the  water  line  equal  to  28  feet,  the  corresponding 
depth  under  the  water  equal  to  19|  feet,  and  its  density  equal  to  the 
density  of  water;  now,  suppose  the  body  to  be  in  a  state  of  equili- 
brium when  the  axis  is  vertical  ;  what  will  be  its  stability,  or  what  is 
the  relative  value  of  the  force  by  which  it  would  endeavour  to  regain 
the  upright  position,  on  the  supposition  that  it  has  been  deflected 
from  it  through  an  angle  of  15  degrees  ? 

This  is  obviously  a  case  that  is  not  likely  to  occur  in  the  practice 


OF  THE  STABILITY  OF  STEAM  SHIPS. 


401 


of  steam  navigation,  because  the  form  is  altogether  unsuitable  for 
vessels  of  that  description,  and  our  only  object  forgiving  it  here  is  to 
show  the  method  of  reducing  the  equation  ;  this  being  the  more 
necessary  for  the  sake  of  system,  as  it  forms  a  particular  case  of  the 
general  problem,  and  is  deducible  from  it  by  merely  assuming  a 
particular  value  for  the  exponent  of  the  parabolic  ordinate. 

By   the    rule,    we   have    (b*  —  2e?2)  =r  784—  -  380.25X2  =  23.5  ; 
therefore,  by  multiplication  and  division,  we  obtain 

b  sin.  rf> 

—^(^—2rfa)zz28x0.25882X23.5-rl2z=5—  14.19  very  nearly. 

492.  Returning  to  the  general 
equation,  if  we  suppose  wzr2, 
then  the  section  is  in  the  form  of 
the  common  or  Apollonian  para- 
bola, as  represented  in  the  an- 
nexed diagram,  wherein  AB  is  the 
base  or  double  ordinate  of  the 
parabolic  section,  DC  its  axis,  FII 
the  water-line,  or  double  ordinate 
of  the  immersed  portion  FDH,  DE  the  corresponding  abscissa,  and  IK 
the  horizontal  surface  of  the  fluid.  Then,  with  ram  2,  the  expression 
for  the  stability  becomes 


(292). 

The  form  of  the  vessel  of  which  the  stability  is  expressed  by  the 
above  equation,  is  much  better  adapted  for  the  purposes  of  steam 
navigation,  than  the  triangular  form  already  discussed;  but  it  is  obvious 
from  the  relation  of  the  parenthetical  terms,  that  it  requires  a  much 
greater  breadth  at  the  water  line  under  the  same  depth  and  inclina- 
tion, to  give  an  equal  degree  of  stability  ;  and  the  breadth  necessary 
for  this  purpose  maybe  determined  by  reversing  the  expression,  which 
will  then  assume  the  form  of  a  cubic  equation,  wanting  the  second 
term,  and  whose  reduction  will  give  the  necessary  breadth. 

493.  Now,  by  the  preceding  calculation  we  have  found  the  stability 
to  be  14.19  very  nearly,  while  the  depth  is  19|  feet,  and  the  inclina- 
tion from  the  upright  position,  15  degrees,  of  which  the  natural  sine  is 
0.25882;  consequently,  by  substitution  we  obtain 

0.0215763—  24.66=14.19, 

and  if  this  equation  be  reduced,  we  shall  find  the  value  of  b  or  the 
breadth  of  the  vessel  at  the  water  line,  to  be  a  very  small  quantity  in 

VOL.  i.  2  D 


402  OF  THE  STABILITY  OF  STEAM  SHIPS. 

excess  of  34  feet  ;  but  taking  it  at  34,  the  value  of  the  stability  for 
a  vessel  in  the  form  of  a  common  parabola  becomes 

s_  34X0.25882  (1156_  1140t75)=  1M84; 

hence  it  appears,  that  the  breadth  at  the  water  line,  in  the  case  of  the 
parabola,  requires  an  increase  of  more  than  6  feet,  to  give  the  same 
stability  as  the  triangle  under  the  same  depth  and  deflexion. 

494.  If  the  equation  for  the  stability  in  the  case  of  the  parabola, 
be  compared  with  that  for  the  triangle,  it  will  be  seen  that  3d2  occurs 
in  the  one  case,  instead  of  2d2  in  the  other  ;  consequently,  the  practical 
rule  as  given  for  the  triangle,  will  also  apply  to  the  parabola,  if  the 
phrase  "  thrice  the  square  of  the  corresponding  depth"  be  substituted 
for  "  twice  the  square,"  as  it  is  now  expressed  ;  the  repetition  of  the 
rule  is  therefore  unnecessary. 

495.  Again,  if  we  put  wnr3,  then  the  transverse  section  of  the 
vessel  is  in  the  form  of  a  cubic  parabola,  and  the  general  equation 
for  the  value  of  the  stability  becomes 


496.  This  form  is  greatly  superior  to  the  preceding  one  for  a  steam 
vessel,  as  it  gives  the  surfaces  in  contact  with  the  water  a  less  degree 
of  curvature  ;  but  it  requires  a  greater  increase  of  the  breadth  at  the 
water  line  in  proportion  to  the  depth  to  obtain  the  same  degree  of 
stability,  which  is  manifest  from  the  increase  of  the  negative  co- 
efficient, the  form  of  the  equation  being  in  every  other  respect  the 
same  as  before. 

The  practical  rule  for  this  form,  may  be  expressed  in  words  at 
length  as  follows. 

RULE.  From  the  square  of  the  breadth  at  the  water  line 
when  the  vessel  is  upright,  subtract  3.6  times  the  square 
of  the  corresponding  depth  ;  multiply  the  remainder  by  the 
breadth  drawn  into  the  natural  sine  of  the  angle  of  inclina- 
tionr  and  one  twelfth  of  the  product  will  express  the  stability. 

497.  EXAMPLE.  Let  the  breadth  of  the  water  line  be  38  feet,  and 
let  the  depth  and  the  deflexion,  as  well  as  the  density  of  the  vessel, 
be  the  same  as  before  ;  what  then  will  be  the  value  of  the  stability  ? 

Here  by  the  rule  we  have 

(62—  3.6d9)i=382—  3.6  X19.52=:  75.1  ; 

consequently,  by  multiplication  and  division,  we  have 


OF  THE  STABILITY   OF  STEAM   SHIPS. 


403 


12     v-       ;=  38x0.02157x75.1  n:  6 1.6  nearly. 

498.  In  order  to  pursue  the  inquiry  a  step  further,  let  us  suppose 
that  7zz=5;    then,   by   substituting  this  value  of  n  in  the  general 
equation  for  the  value  of  the  stability,  we  shall  get 

(294). 

an  equation  which  differs  in  nothing  from  those  that  precede  it,  but  in 
the  value  of  the  constant  co-efficient  of  the  negative  term  within  the 
parenthesis,  a  quantity  which  indicates  the  increase  of  breadth  at  the 
water  line,  necessary  to  give  the  vessel  the  same  degree  of  stability, 
under  the  same  depth  and  deflexion,  which  it  possesses  when  bounded 
by  curves  of  the  lower  orders. 

499.  If  the  curves  which  we  have  just  considered  were  delineated 
from  a  fixed  scale,  according  to  the  relation  that  subsists  between  the 
ordinates  and  the  corresponding  abscissas,  it  would  be  seen,  that  the 
breadths  towards  the  vertex  become  greater  and  greater  as  the  exponent 
of  the  ordinate  increases;  the  figure  therefore  approaches  continually 
to  the  form  of  a  rectangular  parallelogram,  and  essentially  coincides 


fc' 


with  it,  when  the  value  of  n 
becomes  infinite,  as  in  the 
parabola  A  KB,  wherein  DC  is 
the  breadth,  and  LK  the  depth 
of  the  vessel ;  E  F  the  water 
line,  and  k'k  the  line  of  sup- 
port in  the  inclined  position  ; 
y  the  ordinate  parallel  to  the 
depth,  and  x  the  abscissa; 
DPE  the  immersed  triangle,  I 
and  F  PC  the  extant  triangle.  This  extreme  case  has  a  manifest  rela- 
tion to  the  subject  of  stability ;  for  whatever  may  be  the  effect  of 
giving  to  the  sides  of  vessels  the  forms  of  the  higher  orders  of 
parabolas,  it  is  evident,  that  as  the  exponent  of  the  ordinate  is 
increased,  the  stability  will  approach  to  that  which  would  obtain  if 
the  sides  were  made  parallel  to  the  plane  of  the  masts. 

Now,  it  may  easily  be  shown,  that  when  the  sides  of  the  vessel  are 
made  to  coincide  with  the  form  of  a  conic  parabola,  (fig.  art.  492,) 
the  stability  is  the  same  as  when  the  sides  are  parallel  planes ;  hence 
it  is  inferred,  that  if  the  sides  of  a  vessel  be  formed  to  coincide  with 
a  parabola  of  the  lowest  order,  and  another  to  coincide  with  one  of 
the  highest,  all  other  circumstances  being  the  same,  the  stabilities 
will  be  equal  in  these  two  cases. 

2  D  2 


404  OF  THE  STABILITY  OF  STEAM  SHIPS. 

500.  But  we  must  now  proceed  to  consider  the  second  variety,  in 
which  the  ordinates  are  parallel  to  the  breadth  of  the  vessel  at  the 
water  line  when  the   vessel  is  placed  in  an   upright  and  quiescent 
position  ;  and  in  this  case,  the  general  equation  expressing  the  value 
of  the  stability,  is 

^sin^x  .12nd2      \ 

"ITA6  ~n*+3rc  +  2/'  (295). 

where  the  several  letters  which  enter  the  equation  indicate  precisely 
the  same  quantities,  and  refer  to  the  same  parts  of  the  vessel  as  before  ; 
and  by  giving  particular  values  to  the  quantity  n,  we  shall  obtain 
another  series  of  equations,  indicating  the  stability  according  to  the 
order  of  the  parabolic  curve  by  which  the  vessel  is  bounded. 

501.  If  we  put  w:nl,  then  the  transverse  section  of  the  vessel 
becomes  a  triangle,  and  the  equation  expressing  the  value  of  the 
stability  in  that  case,  is 


(296). 

which  is  manifestly  the  same  expression  as  that  which  we  obtained  for 
the  triangle  in  the  first  variety,  where  the  ordinates  were  supposed  to 
be  parallel  to  the  depth;  hence,  the  value  of  the  stability  when 
estimated  in  numbers  will  also  be  the  same. 

502.  Again,  if  we  suppose  the  bounding  curve  of  a  cross  section  to 
be  the  same  as  the  common  parabola,  then  11=12,  and  this  being  sub- 
stituted in  the  general  equation,  the  expression  for  the  stability  in  this 
case,  is 


(297). 

the  very  same  as  for  the  triangle ;  hence  it  appears,  that  when  the 
ordinates  are  parallel  to  the  breadth,  the  stability  for  a  triangular 
section  is  the  same  as  it  is  for  a  section  in  the  form  of  the  common  or 
Apollonian  parabola. 

503.  But  when  the  boundary  of  the  section  is  in  the  form  of  a 
cubic  parabola,  then  n  =:  3,  which  being  substituted  in  the  general 
equation,  the  expression  for  the  value  of  the  stability  in  this  case,  is 

s= »§*  (*.!.«•>.    .'.'•:  r.  (298) 

If  this  equation  be  compared  with  the  corresponding  one  for  the 
cubic  parabola,  in  the  case  when  the  ordinates  are  parallel  to  the 
depth,  it  will  be  seen  that  the  present  form  is  superior  in  point  of 
stability,  since  it  requires  a  less  breadth  in  proportion  to  the  depth  to 
offer  an  equal  resistance.  This  inference  is  drawn  from  a  comparison 


OF  THE  STABILITY  OF  STEAM  SHIPS.  405 

of  the  constants  belonging  to  the  negative  term,  for  in  the  one  case 
it  is  double  of  what  it  is  in  the  other,  and  consequently,  in  the  latter 
case,  a  less  breadth  is  necessary  to  give  a  positive  result. 

504.  Lastly,  if  n  zr  5,  then  the  parabola  which  bounds  the  trans- 
verse section  A  KB  of  the  vessel,  is  defined  by  the  equation  px=iys, 
as  in  the  annexed  figure,  in 
which  the  several  letters  indi- 
cate  the  parts  already  men- 
tioned, viz.  DC  the  breadth, 
LK  the  depth  of  the  vessel, 
E  F  the  water  line,  k'  k  the  line 
of  support,  y  the  ordinate 
parallel  to  the  breadth,  x  the 
abscissa  ;  then,  the  expression 
for  the  stability  in  this  case,  becomes 


.,/  (299, 

from  which  it  appears,  that  the  higher  the  order  of  the  parabola,  the 
less  increase  of  breadth  is  necessary  with  the  same  depth  to  obtain 
an  equal  degree  of  resistance  ;  but  in  the  case  when  the  ordinates  are 
parallel  to  the  depth,  as  in  the  first  variety,  the  contrary  takes  place, 
a  greater  increase  of  breadth  being  necessary  for  the  same  purpose. 
Hence  we  conclude,  that  the  higher  the  order  of  the  parabola,  the 
greater  is  the  degree  of  stability  ;  but  the  form  in  which  the  ordinates 
are  parallel  to  the  breadth,  is  preferable  to  that  in  which  they  are 
parallel  to  the  depth;  and,  as  Mr.  Tredgold  justly  remarks,  "  this 
species  of  figure  may  be  easily  traced  through  all  the  varieties  of 
form,  and  it  has  obviously  a  decided  advantage  in  point  of  stability, 
and  it  is  so  easy  to  compute  its  capacity  and  describe  it  by  ordinates, 
that  it  is  much  to  be  preferred  to  the  elliptical  figures  which  foreign 
writers  have  chosen  for  calculation." 

505.  In  order  that  the  stability  may  be  the  same  at  every  section 
throughout  the  whole  length  of  the  vessel,  this  being  a  necessary 
condition  in  the  most  advantageous  cases,  the  breadth  should  be 
every  where  in  the  same  ratio  to  the  depth  ;  for  when  this  is  the  case, 
the  vessel  will  suffer  no  lateral  strain  from  a  change  of  position. 
The  preceding  determinations  relate  to  the  vessel's  stability  when  the 
inclination  is  made  about  the  longer  axis  ;  but  the  position  of  the 
shorter  axis,  round  which  the  ship  revolves  in  pitching,  in  all  cases  of 
practical  inquiry,  must  also  be  considered  ;  but  since  the  investigation 
sf  the  several  conditions  would  be  similar  to  that  which  refers  to  the 
longer  axis,  we  deem  it  unnecessary  to  extend  the  inquiry  any  further. 


CHAPTER  XIV. 

OF    THE    CENTRE    OF    PRESSURE. 


506.  THE  subject  of  the  present  chapter  might  have  been  placed  in 
juxtaposition  with  the  doctrine  of  pressure  on  plane  surfaces ;  but  we 
chose  to  reserve  it  for  the  conclusion  of  fluid  equilibrium,  and  in  as 
brief  a  manner  as  possible  we  shall  now  view  the  centre  of  pressure, 
by  illustrating  a  few  select  examples  dependent  upon  its  principles. 

The  Centre  of  Pressure  of  a  plane  surface  immersed  in  a  fluid,  or 
sustaining  a  fluid  pressing  against  it,  is  that  point,  to  which,  if  a  force 
be  applied  equal  and  contrary  to  the  whole  pressure  exerted  by  the 
fluid,  the  plane  will  remain  at  rest,  having  no  tendency  to  incline  to 
either  side. 

It  is  manifest  from  this  definition,  that  if  a  plane  surface  immersed 
in  a  fluid,  or  otherwise  exposed  to  its  influence,  be  parallel  to  the 
horizon  ;  then,  the  centre  of  pressure  and  the  centre  of  gravity  occur 
in  the  same  point,  and  the  same  is  true  with  respect  to  every  plane  on 
which  the  pressure  is  uniform;  but  when  the  plane  on  which  the 
pressure  is  exerted,  is  any  how  inclined  to  the  horizon,  or  to  the 
surface  of  the  fluid  whose  pressure  it  sustains ;  then,  in  order  to 
determine  the  centre  of  pressure,  we  must  have  recourse  to  the 
resolution  of  the  following  problem. 

PROBLEM  LXIII. 

507.  Having  given  the  dimensions  and  position  of  a  plane  sur- 
face immersed  in  a  fluid,  or  otherwise  exposed  to  its  influence : — 

It  is  required  to  determine  the  position  of  the  centre  of 
pressure,  or  that  point,  to  which,  if  a  force  be  applied  equal 
and  opposite  to  the  pressure  of  the  fluid,  the  plane  shall 
remain  in  a  state  of  quiescence,  having  no  tendency  to  incline 
to  either  side. 


01'  THE-  CENTRE  OF  PRESSURE. 


407 


Let  ABC  be  a  cistern  filled  with  an  incompressible  and  non-elastic 
fluid,  and  let  abed  be  a  rectangular 
plane  immersed  in  it  at  a  given 
angle  of  inclination  to  its  surface ; 
produce  the  sides  da  and  cb  di- 
rectly forward  to  meet  the  surface 
of  the  fluid  in  the  points  e  and  /; 
join  ef,  and  through  the  points  e 
and  /,  draw  es  and/r  respectively 
perpendicular  to  the  plane  produced, 
and  coinciding  with  the  surface  of 
the  fluid  in  ef;  draw  also  ds  and  cr, 

meeting  es  andyY  at  right  angles  in  the  points  s  and  r\  then  is  des 
or  c/r,  the  angle  of  the  plane's  inclination,  and  ds,  cr  are  the  per- 
pendicular depths  of  the  points  d  and  c. 

Let  P  be  the  position  of  the  centre  of  pressure,  and  through  p  draw 
pm  and  PW,  respectively  perpendicular  to  cb  and  cd  the  sides  of  the 
rectangular  plane  ;  then  are  cb  and  cd  the  axes  of  rectangular  co-ordi- 
nates originating  at  c,  and  pm,  PW  are  the  corresponding  co-ordinates, 
passing  through  p  the  centre  of  pressure,  supposed  to  be  situated  in 
that  point. 

Now,  it  is  manifest  from  the  nature  of  fluid  pressure  demonstrated 
in  the  first  chapter,  that  the  force  of  the  fluid  against  d : — 

Is  equal  to  the  weight  of  a  column  of  the  fluid,  whose  base 
is  the  point  d,  and  altitude  the  perpendicular  depth  of  that 
point  below  the  upper  surface  of  the  fluid. ^ 

Consequently,  the  force  against  the  point  d,  varies  as  dx  ds  ;  but 
by  the  principles  of  Plane  Trigonometry,  we  have 
rad.  :  ed  :  :  sin. des  :  ds; 
hence  by  reduction,  we  get 

ds=:ed$m.des; 
therefore  the  pressure  on  the  point  d  varies  as 

d  XedXsin.des, 

and  the  effort  or  momentum  of  this  force,  to  turn  the  plane  about  the 
ordinate  pm,  manifestly  varies  as 

d  Xee?Xsin.c?esXPw, 

where  p  n  is  the  length  of  the  lever  on  Avhich  the  force  acts. 
But  by  subtraction,  p  n  -=.ed — /m,  for  ed—fc;  therefore  by  sub- 
stitution, the  force  to  turn  the  plane  about  the  ordinate  pm,  varies  as 


408  OF  THE  CENTRE  OF  PRESSURE. 

dXed*  Xsm.de  s  —  e?Xec?X/wXsi 
therefore,  the  accumulated  effect  of  all  the  forces,  to  turn  the  plane 
round  rw,  must  be  proportional  to  the  sum  of 

{dXed*}Xs\n.des  —  /mXsum  of  {dXed}Xsin-des, 
and  this  by  the  definition  is  equal  to  nothing;  hence  we  get 
yVwXsum  of  {c?Xec?}sin.rfeszzsum  of  {dXed*}  sin.e?es  ; 
therefore,  by  division,  we  obtain 
_  sum  o 
~  su 

But  the  sum  of  {dXed},  is  obviously  the  same  as  the  body,  or 
sum  of  all  the  constituent  particles,  multiplied  into  the  distance  of 
the  common  centre  of  gravity  ;  and  therefore,  by  the  principles  of 
mechanics,  fm  is  also  the  distance  of  the  centre  of  percussion,  if  ef, 
the  common  intersection  of  the  plane  with  the  fluid,  be  considered  as 
the  axis  of  suspension,  the  plane  being  supposed  to  vibrate  flat-ways. 
By  reasoning  in  the  same  manner  as  above,  it  will  readily  appear, 
that  the  effect  or  momentum  of  the  pressure  on  c?,  to  turn  the  plane 
about  the  ordinate  PW,  varies  as 

dXe  dXdnX$in.de  s; 

but  by  subtraction,  it  is  dn  —  cd  —  en  ;  therefore  by  substitution,  the 
force  on  d  to  turn  the  plane  round  PW,  varies  as 


and  consequently,  the  effect  of  all  the  forces  to  turn  the  plane  around 
pn,  must  be  proportional  to  the 

sum  of  {dxdeycd}  siu.des  —  crcXsum  of  {dxed}  s'm.des  ; 
but  by  the  definition,  the  sum  of  these  forces  is  equal  to  nothing;  for 
the  plane  has  no  tendency  to  incline  to  either  side,  being  sustained  in 
a  state  of  quiescence  by  means  of  the  fluid,  and  the  equivalent  oppos- 
ing force  applied  at  the  centre  of  pressure  ;  hence  we  get 
cwXsum  of  [dXed}~  sum  of  {dXdeXcd}  ; 
therefore,  by  division,  we  shall  have 
__sum  of  {dXdeXcd} 

sumof{dXde}     '  (301). 

which  expression  also  indicates  the  distance  of  the  centre  of  percus- 
sion ;  from  which  it  is  manifest,  that  the  centres  of  pressure  and 
percussion  coincide,  when  the  line  of  common  section  between  the 
plane  or  the  plane  produced,  and  the  surface  of  the  fluid  is  made  the 
axis  of  suspension.  This  being  the  case,  it  is  evident,  that  the  formulse 


OF  THE  CENTRE  OF  PRESSURE. 


409 


which  are  employed  to  determine  the  centre  of  percussion,  may  also, 
and  with  equal  propriety,  be  employed  to  determine  the  centre  of 
pressure. 

Now,  the  writers  on  the  general  principles  of  mechanical  science 
have  demonstrated,  that  if  . 

xmed,  the  side  of  the  plane  extending  downwards,  and 
?/:zicc?,  the  horizontal  side  parallel  tofe; 

t\\Gnfm  and  pm,  the  respective  distances  of  the  point  P,  fromfe  and 
fc  the  sides  of  the  plane,  are  generally  represented  by  the  following 
fluxional  equations,  viz. 


/• 


y* 


fm  —  •-  -  ,  andpwzn 

Jxyx  ZJyxx 


(302). 


From  these  two  equations,  therefore,  the  centre  of  pressure  cor- 
responding to  any  particular  case,  can  easily  be  found,  as  will  become 
manifest,  by  carefully  tracing  the  several  steps  in  the  resolution  of  the 
following  problems. 

PROBLEM  LXIV. 

508.  A  physical  line  of  a  given  length,  is  vertically  immersed 
in  a  fluid  :  — 

It  is  required  to  ascertain  at  what  distance   below   the 
surface  of  the  fluid  the  centre  of  pressure  occurs. 

Let  be  be  a  physical  line,  perpendicularly 
immersed  in  a  fluid  of  which  the  surface  is 
AB,  and  produce  cb  to  f,  so  that  the  point 
f  may  be  considered  as  the  centre  of  suspen- 
sion, and  let  m  be  the  centre  of  pressure,  or 
the  centre  of  percussion  ;  then 

by  equation  (302),  we  shall  obtain 

* 


-^ 
f' 


'9 

in  which  equation  y  is  constant  ;  therefore, 

Put  d  —fc,  the  distance  of  the  lower  extremity  below  AB, 
8  —  /£,  the  distance  of  the  higher  extremity,  and 
I  ~bc,  the  whole  length  of  the  line. 


410  OF  THE  CENTRE  OF  PRESSURE. 

Therefore,  by  addition  we  have  d=  I  -f-  £,  and  by  taking  the  fluent 
of  the  above  equation,  we  get 


fm._ 

J™  - 


3(a*-  ar 

and  when  a;  me?,  we  shall  obtain 


(303). 

The  equation  as  it  now  stands,  is  general  in  reference  to  a  line 
of  which  the  extremities  are  both  situated  below  the  surface  ;  but 
when  the  upper  extremity  is  coincident  with  it,  then  8  vanishes,  in 
which  case  c?—  Z,  and  our  equation  becomes 

fm  =  \L*  (304>- 

509.  This  last  form  of  the  expression  is  too  simple  to  require  any 
illustration  ;  but  the  form  which  it  assumes  in  equation  (303),  may 
be  expressed  in  words  at  length  in  the  following  manner. 

RULE.  Divide  the  difference  of  the  cubes  of  the  depths  of 
the  extremities  of  the  given  line  below  the  surface  of  the 
fluid,  by  the  difference  of  their  squares,  and  two  thirds  of  the 
quotient  will  give  the  distance  of  the  centre  of  pressure  below 
the  surface  ;  from  which,  subtract  the  depth  of  the  upper 
extremity,  and  the  remainder  will  show  the  point  in  the  line 
where  the  centre  of  pressure  is  situated. 

510.  EXAMPLE.  Required  the  position  of  the  centre  of  pressure  in 
a   line  of  4  feet  in  length,  when  immersed  vertically  in  a  fluid,  the 


*  Now  what  is  here  true  of  a  physical  line  is  true  also  of  a  plane,  which,  if  it 
reach  the  surface  of  the  fluid  whose  pressure  it  sustains,  will  have  its  centre  of 
pressure  at  a  distance  equal  to  two  thirds  of  its  breadth  or  depth  from  the  upper 
extremity ;  and  this  holds  true  also,  whatever  may  be  its  inclination,  its  centre  of 
pressure  will  he  distant  from  the  upper  edge  by  two  thirds  of  its  surface  or  breadth. 
A  single  force,  therefore,  applied  at  that  distance,  and  exactly  in  the  middle  of  the 
length  of  the  plane,  would  hold  it  at  rest.  And  the  same  would  manifestly  be  the 
case,  if  the  rod,  in  place  of  being  applied  longitudinally  at  a  single  point,  were 
placed  across  the  plane  over  the  point  which  indicates  the  position  of  the  centre  of 
pressure.  All  that  is  required  in  order  to  procure  the  equilibrium  is,  that  a 
sufficient  balancing  force  be  applied  to  that  centre ;  thus,  a  sluice  or  floodgate  may 
be  held  in  its  place  by  the  pressure  of  a  single  force,  applied  at  one  third  of  its 
length  from  its  base,  and  at  two  thirds  of  its  length  below  the  surface  of  the  fluid. 
And  this  suggests  the  practical  importance  of  placing  the  beams  and  hinges  of 
flood  and  lock-gates  at  equal  distances  above  and  below  the  centre  of  pressure, 
which  is  at  two  thirds  the  depth  of  the  gate.  See  Problem  LXVI.  p.  416. 


OF  THE  CENTRE  OF  PRESSURE.  411 

upper  extremity  being  2  feet  below  the  surface,  and  the  lower  extremity 
6  feet  ? 

Here  we  have  d3 —  £3:z:  6s —  23—  208, 

and  (F—  c~  —  62—  2'zz:  32 ; 
consequently,  by  division,  we  obtain 

208 

-55=  6.5  feet, 

and  by  taking  two  thirds  of  this,  we  get 

fm  i=  -|  of  6.5  rz  44  ft. 
and  finally,  by  subtracting  the  depth  of  the  upper  end,  we  obtain 

511.  If  the  upper  extremity  of  the  line  had  been  in  contact  with 
the  surface  of  the  fluid,  then  would 

imzzfof  4z=2tft. 

This  is  manifest  from  equation  (304),  and  if  a  rectangle  be  described 
upon  the  vertical  line,  the  distance  of  its  centre  of  pressure  below  the 
surface  of  the  fluid  will  be  expressed  by  the  equation  (303  or  304), 
according  as  the  upper  extremity  is  situated  below,  or  in  contact  with 
the  surface,  and  this  distance  will  obviously  be  measured  in  the  line 
by  which  the  rectangle  is  bisected. 

512.  If  the  upper  side  of  the  rectangle  coincides  with  the  surface 
of  the  fluid,  as  in  the  annexed  diagram,  where 

adfe  is  the  rectangular  parallelogram,  having  T_  _ 
the  upper  side  ad  in  contact  with  IK  ;  then, 
according  to  equation  (304),  bm  the  distance 
of  the  centre  of  pressure,  is  equal  to  two 
thirds  of  be,  the  whole  length  of  the  paral- 
lelogram, and  consequently,  by  subtraction, 
we  have  mc—^bc;  therefore,  the  tendency 
of  the  plane  to  turn  about  the  base  ef,  is 

equal  to  the  pressure  which  it  sustains,  drawn  into  the  length  of  the 
lever  mc  —  \l,  where  I  denotes  the  whole  length  of  the  plane. 

Put  b  nr  ad  or  ef,  the  horizontal  breadth  of  the  plane, 
p  =:  the  entire  pressure  which  it  sustains,  and 
s  iz:  the  specific  gravity  of  the  fluid  in  which  it  is  immersed. 

Then,  by  equation  (8),  Problem  III.  Chapter  II.  the  whole  pressure 
sustained  by  the  immersed  plane,  is  expressed  by 


412 


OF  THE  CENTRE  OF  PRESSURE. 


and  this  pressure  being  applied  at  m,  operates  on  the  lever  me  to  turn 
the  plane  about  ef,  with  a  force  which  is  equal  to 


Through  the  point  m,  draw  mn  parallel  to  ef,  the  base  of  the 
immersed  plane  ;  then,  the  tendency  to  turn  round  the  vertical  side 
a  e,  is  equal  to  the  whole  pressure  upon  the  plane,  drawn  into  the 
length  of  the  lever  mn  ;  but  mn=i  \b,  and  we  have  seen  above,  that 
the  pressure  is  expressed  by  %bl*s;  consequently,  the  tendency  to 
turn  round  ae,  is 


let  these  two  momenta  be  compared,  and  we  shall  have 


513.  This  is  obvious,  for  by  casting  out  the  common  factors,  and 
assimilating  the  fractions,  the  ratio  becomes 

21  :  3b  ; 

and  when  I  and  b  are  equal  to  one  another,  or  when  the  immersed 
plane  is  a  square  ;  then  the  ratio  is  simply  as  2:3;  that  is,  the 
tendency  of  the  plane  to  turn  round  the  lower  horizontal  side,  is  to  its 
tendency  to  turn  round  a  vertical  side,  as  1  :  1  J,  or  as  2  :  3. 

PROBLEM  LXV. 

514.  A  semi-parabolic  plane  is  immersed  vertically  in  a  fluid,  in 
such  a  manner,  that  the  extreme  ordinate  is  just  in  contact  with 
the  surface  :  — 

It  is  required  to  determine  the  position  of  the  centre  of 
pressure,  both  with  respect  to  the  axis  and  the  extreme 
ordinate  ,  which  is  coincident  with  the  surface  of  the  fluid. 

Let  IK  be  the  surface  of  the  fluid,  and  acd  the  semi-parabola 
vertically  immersed  in  it,  in  such 
a  manner,  that  ad  the  extreme 
ordinate  coincides  with  i  K,  while 
the  axis  ac  is  perpendicular  to  it. 

Let  m  be  the  point  at  which  the 
centre  of  pressure  is  supposed  to 
be  situated,  and  through  m  draw 
mb  and  mn,  respectively  perpen- 
dicular toad  and  ac,  and  suppose 
the  axes  of  co-ordinates  to  originate  at  a  ;  then,  if  we 


OF  THE  CENTRE  OF  PRESSURE. 


413 


Put  I  —  ac,  the  axis  of  the  semi-parabola, 

b  zr  ad,  the  extreme  ordinate,  which  is  in  contact  with  IK, 
x  =z  any  abscissa  estimated  from  the  vertex  at  c,  and 
y  zz  the  corresponding  ordinate, 

then  is  I  —  x  the  distance  between  the  ordinate  and  the  origin  of 
the  axes,  corresponding  to  x  in  the  general  investigation.  Problem 
LXIII.  ;  but  by  the  property  of  the  parabola,  we  have 

l:bl::x:y*; 
and  from  this,  by  reduction,  we  get 


Therefore,  if  l  —  x  and  by   |,  be  respectively  substituted  for 
and  y  in  the  equations  of  condition  numbered  (302),  we  shall  have 


and  for  the  corresponding  co-ordinate,  it  is 


/ 

mn~ 


But  by  the  writers  on  the  fluxional  analysis,  the  complete  fluents 
of  these  expressions  are  respectively  as  follows,  viz. 


—          -  , 

bm=:  -  ^  -  ST-  -  ,and?wn— 
351  — 


the  correction  in  both  cases  being  equal  to  nothing;  but  when  arrr/, 
we  get 

imzz|/,  and  mn  —  Tsjb,  (305). 

and  from  these  values  of  the  co-ordinates,  is  the  centre  of  pressure  to 
be  found. 

515.  EXAMPLE.  A  plane  in  the  form  of  a  semi-parabola,  is  immersed 
perpendicularly  in  a  fluid,  in  such  a  manner,  that  the  extreme  ordinate 
coincides  with  the  surface;  whereabouts  is  the  centre  of  pressure 
situated,  the  axis  being  9  and  the  ordinate  6  inches  ? 


414  OF  THE  CENTRE  OF  PRESSURE. 

Here  then  we  have  given  I  —  9  inches,  and  b  zz:  6  inches  ;  conse- 
quently, by  the  equations  numbered  (305),  we  have 

bm —  $  of  9  zn 5j-  inches,  and  mn^n  T\  of  6  zz  l£  inches. 

Therefore,  with  the  abscissa  ac:=9  inches,  and  the  ordinate 
ad —6  inches,  construct  the  semi-parabola  adc, 
by  means  of  points  or  otherwise,  as  directed  by  the 
writers  on  conic  sections ;  then,  on  the  axis  ac 
and  the  ordinate  ad,  set  off  an  and  a  b,  respectively 
equal  to  5-f  and  If  inches,  as  obtained  by  the  pre- 
ceding calculation ;  and  through  the  points  n  and 
b  as  thus  determined,  draw  nm  and  bm,  respec- 
tively parallel  to  ad  and  ac,  intersecting  each  other 
in  m;  then  is  m  the  place  where  the  centre  of  pressure  occurs,  as  was 
required  by  the  question. 

516.  It  would  be  easy  to  multiply  cases  and  examples,  respecting 
the  parabola  and  other  curves  of  a  kindred  nature,  considering  them 
either  entire  or  in  part,  and  situated  in  different  positions,  as  referred 
to  the  surface  of  the  fluid ;  but  since  the  resolution  in  every  instance, 
depends  upon  the  integration  of  the  general  fluxional  equations  num- 
bered (302),  when  accommodated  to  the  particular  figure,  we  think 
it  quite  unnecessary  to  dwell  longer  on  this  part  of  the  inquiry ;  we 
therefore  proceed  to  resolve  a  problem  or  two   that  depend   upon 
similar  principles,  and  consequently,  are  well  adapted  for  illustrating 
the  manner  in  which  the  inquiry  is  to  be  extended. 

PROBLEM  LXVI. 

517.  A  vessel  in  the  form  of  a  parallelopipedon  with  the  sides 
vertical,  has  one  side  loose  revolving  on  a  hinge  at  the  bottom, 
and  is  kept  in  its  position  by  a  certain  power  applied  at  a 
given  point: — 

It  is  required  to  determine  how  high  the  vessel  must  be 
filled  withjtuid,  before  the  revolving  side  is  forced  open. 

Let  ABC  represent  the  vessel  in  question,  and  let  avdc  be  the 
loose  side  moveable  about  the  hinges  at  e  andy*;  bisect  c  d  in  c,  and 
draw  cb  perpendicular  to  cd,  and  let  n  be  the  point  at  which  the 
given  power  is  applied  ;  then,  because  the  side  axdc  is  just  sustained 
by  means  of  the  power  acting  at  n,  it  follows,  that  the  whole  force 
of  the  fluid  acting  at  the  centre  of  pressure  must  produce  the  equipoise. 


OF  THE  CENTRE  OF  PRESSURE. 


415 


Suppose  m  to  be  the  centre  of  pres- 
sure, and  make  mb  equal  to  twice 
me;  then  by  equation  (304),  the 
point  b  must  coincide  with  the  sur- 
face of  the  fluid. 

Through  the  point  b  and  parallel 
to  «B  or  cc?,  draw  the  straight  line 
rs,  which  marks  the  height  to  which 
the  vessel  must  be  filled  with  fluid, 
before  the  side  is  forced  open. 


C 


Put  b  =  au,  the  breadth  of  the  loose  side  of  the  vessel, 

S  zz  en,  the  distance  from  the  bottom  at  which  the  force  is 

applied, 

yzz:  the  magnitude  of  the  force  applied  at  the  point  n, 
s  zr  the  specific  gravity  of  the  fluid  contained  in  the  vessel, 
p  HZ  the  pressure  of  the  fluid  against  its  side,  and 
z  =  cb,  the  height  to  which  the  fluid  rises. 

Then,  by  the  principle   indicated  in  equation  (8),  Problem  III. 
Chapter  II.  we  have 


and  this  takes  place  at  the  centre  of  pressure,  which,  according  to 
equation  (304),  is  situated  at  two  thirds  of  the  depth  below  the 
surface,  and  consequently,  its  effect  to  turn  the  side  andc  on  the 
hinges  e  and  /*,  is,  according  to  the  principle  of  the  lever,  expressed  by 


Now,  the  effect  of  the  force  applied  at  w,  to  prevent  the  side  from 
being  thrust  open  by  the  pressure  of  the  fluid,  is  expressed  by  the 
magnitude  of  the  given  force,  drawn  into  en  the  length  of  the  lever 
on  which  it  acts,  and  is  precisely  equal  to  the  effect  of  the  fluid 
acting  at  the  centre  of  pressure  ;  hence  we  get 

/8  =#*»,; 
and  by  division  this  becomes 


Z- 

Z    -  -7—  * 

bs 
from  which,  by  extracting  the  cube  root,  we  obtain 


bs 


416  OF  THE  CENTRE  OF   PRESSURE. 

This  is  the  general  form  of  the  equation,  corresponding  to  a  fluid  oi 
any  density  whatever  denoted  by  s;  but  when  the  fluid  is  water,  of 
which  the  specific  gravity  is  unity,  the  above  equation  becomes 


(306). 

The  method  of  reducing  this  equation,  may  be  very  simply  expressed 
in  words  as  follows. 

RULE.  Divide  six  times  the  momentum  of  the  given  force,* 
by  the  horizontal  breadth  of  the  side  to  which  it  is  applied, 
and  the  cube  root  of  the  quotient  will  be  the  height  to  which 
the  vessel  must  be  filled. 

EXAMPLE.  The  horizontal  breadth  of  one  side  of  an  oblong  pris- 
matic vessel,  is  30  inches ;  now,  supposing  this  side  to  be  loose  and 
moveable  about  a  Hinge  at  the  bottom  ;  how  high  must  the  vessel  be 
filled  with  water,  in  order  that  the  pressure  of  the  water,  and  a  force 
of  400  Ibs.  applied  at  the  distance  of  12  inches  from  the  bottom,  may 
exactly  balance  each  other  ? 

Here  we  have  given 

b  =  30  inches  ;  3  zr  12  inches  ;  and  /zz  400  Ibs  ; 
consequently,  we. obtain 


z=f/  6x4QQXl2 
30 


—9.865  inches. 


But  the  centre  of  pressure,  at  which  the  weight  of  the  fluid  is  sup- 
posed to  be  applied  in  opposition  to  the  given  force,  is  situated  at 
one  third  of  the  above  distance  from  the  bottom  of  the  vessel ;  hence 
we  have 

cm  zzi  of  9.865  —  3.288  inches. 

Therefore,  the  pressure  of  the  fluid  acting  on  a  lever  of  3.288  inches, 
must  be  equal  to  a  force  of  400  Ibs.  acting  on  a  lever  of  12  inches  ; 
that  is 

400 X 12  =  30 X  9.865s-:- 6. 


*  The  momentum  of  the  given  force,  is  equivalent  to  its  magnitude,  drawn  into 
the  distance  above  the  bottom  of  the  point  at  which  it  is  applied. 


OF  THE  CENTRE  OF  PRESSURE. 


417 


PROBLEM  LXVII. 

518.  A  vessel  in  the  form  of  a  tetrahedron  is  entirely  filled  with 
water,  and  has  one  of  its  planes  bisected  by  a  line  drawn  from 
the  vertex  to  the  middle  of  the  opposite  side ;  now  supposing 
one  half  of  the  bisected  plane  to  be  loose,  and  moveable  about  a 
hinge  at  its  lower  extremity : — 

It  is  required  to  determine  the  magnitude  of  the  force,  the 
point  of  application,  and  the  direction  in  which  it  acts  with 
respect  to  the  horizon,  when  the  moveable  half  of  the  contain- 
ing plane  is  just  retained  in  a  state  of  quiescence. 

Let  ACB  be  one  side  of  the  vessel,  divided  by  the  line  CD  into  two 
parts,  which  are  equal  and  similar  to 
one  another;  and  let  the  part  BCD 
be  moveable  about  the  hinges  at  e 
and/. 

Suppose  the  centre  of  pressure  of 
the  loose  part  B  c  D  to  be  at  the  point 
m,  and  through  m  draw  the  straight 
lines  mb  and  mn,  respectively  paral- 
lel to  CD  and  AB;  in  CD  take  any 
other  point  E,  and  through  E  draw 
EF  perpendicular  to  CD,  or  parallel 
to  DB,  making  the  triangles  CDB  and  CEF  similar  to  one  another. 

Put  I  =z  AB,  BC  or  AC,  the  side  of  the  containing  plane,  or  the 

edge  of  the  tetrahedron, 
d  r=  CD,  the  length  of  its  perpendicular, 
5  =z  the  perpendicular  depth  of  its  centre  of  gravity  below  the 

vertex  at  c, 

p  =  the  pressure  of  the  water  on  the  loose  triangle  CDB, 
ar^:  CE,  any  variable  distance, 
y  zz  EF,  the  corresponding  co-ordinate,  and 
(f>  z=  the  angle  which  the  direction  of  the  retaining  force  makes 

with  the  horizon. 

Then  it  is  manifest  from  the  nature  of  the  .problem,  that  in  the  case 
of  an  equilibrium,  the  magnitude  of  the  retaining  force  must  be  equal 
to  the  whole  pressure  of  the  water  upon  the  moveable  triangle  CDB  ; 

VOL.  i.  2  E 


418  OF  THE  CENTRE  OF  PRESSURE. 

but  by  Problem  XVII.  Chapter  V.  equation  (56),  the  whole  pressure 
upon  the  side  ACB  is  expressed  by  £/8\/2  '»  consequently,  the  pressure 
upon  the  triangle  CDB,  becomes 


This  is  manifest,  for  by  Proposition  I.  Chapter  I.  the  pressure  upon 
the  triangle  ACB,  is  equal  to  its  area  drawn  into  the  perpendicular 
depth  of  the  centre  of  gravity,  the  specific  gravity  or  density  of  the 
fluid  being  expressed  by  unity  ;  but  by  Problem  XVII.  Chapter  V. 
the  perpendicular  depth  of  the  centre  of  gravity  of  the  side  of  a 
tetrahedron  below  the  vertex,  is 


and  according  to  the  writers  on  mensuration,  the  area  of  an  equilateral 
triangle,  is  expressed  by  one  fourth  of  the  square  of  the  side,  drawn 
into  the  square  root  of  3  ;  therefore,  we  have 


where  a  denotes  the  area  of  the  triangle  ACB;    consequently,  by 
multiplication  we  obtain 


from  which,  by  division,  we  get 

p-=L  -rW's.  (307). 

519.  This  equation  satisfies  the  first  demand  of  the  problem,  and  the 
second  manifestly  requires  the  determination  of  the  centre  of  pressure  ; 
for  by  the  definition,  the  point  of  application  of  a  force,  equal  in 
intensity  to  the  pressure  of  the  water,  must  occur  at  the  centre  of 
pressure  of  the  plane  CDB,  on  which  that  pressure  is  exerted. 

Now,  by  the  principles  of  Plane  Trigonometry,  the  length  of  the 
perpendicular  c  D  is  thus  determined, 

rad.  :  I  :  :  sin.  60°  :  CD, 
from  which,  by  reduction,  we  get 


but  sin.60°=r  %\/3,  and  consequently,  by  substitution,  we  get 


Therefore,  since  the  triangles  CDB  and  CEF  are  similar,  by  the 
property  of  similar  triangles,  we  have 

JJ/3  :  ¥  '  •  x  :  y, 
and  by  reduction,  we  get 


OF  THE  CENTRE  OF  PRESSURE.  419 


Let  this  value  of  y  be  substituted  in  the  first  of  the  equations  of 
condition  (302),  and  we  shall  have,  for  the  value  of  the  distance  en, 
as  follows,  viz. 


>       ,. 

Jxyx     fx*x 

the  correction  being  equal  to  nothing  ;  and  when  a:  zr  c  D  or  d,  we  have 

en  =  1^*3.  (308). 

520.  Again,  for  the  horizontal  co-ordinate  nm,  by  substituting  the 
above  value  of  y  in  the  second  of  the  equations  of  condition,  we  obtain 


here  again  the  correction  is  nothing,  and  in  the  limit  when  a?  =  CD  or 
dj  we  have 


T3GL  (309)- 

8\/3 

521.  It  is  shown  by  the  writers  on  mensuration,  that  the  planes  com- 
posing a  tetrahedron,  are  inclined  to  each  other  in  an  angle  whose 
sine  is  equal  to  §V2>  and  by  the  principles  of  mechanics,  the  direction 
of  the  force  applied  at  m,  must  be  perpendicular  to  the  plane  ;  itjs 
therefore  inclined  to  the  horizon  at  an  angle  whose  cosine  is  $^/2  ; 
but  by  the  principles  of  Trigonometry,  we  have 

sin.^>  —  -v/  1  —  cos2.0, 
or  by  substituting  as  above,  we  get 

sin.^  i=  v/l  —  f  =  $  =  .33333, 
corresponding  to  the  natural  sine  of  19°  28'  15". 

Hence  it  appears,  that  at  whatever  point  in  the  plane  the  retaining 
force  may  be  applied,  its  direction  will  be  inclined  to  the  horizon,  at 
an  angle  of  19°  28'  15"  ;  the  third  demand  of  the  problem  is  therefore 
satisfied,  and  we  have  seen  that  equation  (307)  fulfils  the  first,  while 
the  second  requires  the  application  of  equations  (308)  and  (309),  and 
the  method  of  reduction  will  become  manifest  from  the  resolution  of 
the  following  example. 

2E2 


I 


420  OF  THE  CENTRE  OF  PRESSURE. 

522.  EXAMPLE.  A  vessel, in  the  form  of  a  tetrahedron  has  the  length 
of  its  edges  equal  to  15  inches;  now,  supposing  it  to  be  filled  with 
water,  and  placed  upon  its  bottom  with  the  axis  vertical ;  conceive 
one  of  its  sides  or  containing  planes  to  be  bisected  by  a  line  drawn 
from  the  vertex  to  the  middle  of  one  of  the  bottom  edges,  and  let  one 
half  of  this  plane  be  considered  as  loose,  and  moveable  about  a  hinge 
at  the  bottom  ;  what  must  be  the  magnitude  of  a  force  that  will  just 
retain  it  in  its  place,  and  at  what  point  must  it  be  applied  ? 

By  equation  (307),  the  magnitude  of  the  required  force  is  precisely 
equal  to  the  pressure  of  the  fluid  upon  the  moveable  plane ;  therefore, 
by  substituting  the  datum  of  the  above  example,  we  have 

p  z=  TVX  15s  X  -/2  —  397.74  cubic  inches  of  water ; 

which  being  reduced  to  Ibs.  avoirdupois,  becomes 

p  =  397.74 X62.5  -4-1728  —  14.38  Ibs. 

523.  Again,  for  the  point  at  which  this  force  must  be  applied,  in  order 
to  counteract  the  pressure  of  the  water,  we  have  by  equation  (308), 

cw~|7^/3"— £  of  15  X  -v/3  —  9|  inches  nearly. 
And  in  like  manner,  for  the  corresponding  co-ordinate  nm,  we  have 
by  equation  (309), 

nm  —  ^l  —  -?^  of  15  —  2.8125  inches, 
from  which  the  position  of  the  point  m  can  easily  be  ascertained. 

524.  If  the  vessel  should  be  placed  with  the  bottom  upwards  and 
parallel  to  the  horizon,  then  we  shall  have 

j9 1=7.19  Ibs. ;  DW  — 6.495  inches,  and  nm~  1.875  inches. 

A  great  variety  of  useful  and  interesting  problems  similar  to  the 
preceding,  might  be  proposed  in  this  place  ;  but  as  we  have  already 
overleaped  the  limits  of  this  subject  in  the  present  volume,  we  must 
defer  their  consideration  till  another  opportunity. 


CHAPTER  XV. 

OF    CAPILLARY    ATTRACTION    AND    THE    COHESION    OF    FLUIDS. 

525.  THE  subject  of  Capillary  Attraction,  and  the  Cohesion  of 
Fluids,  considered  merely  as  a  branch  of  philosophical  inquiry,  is 
exceedingly  seductive  and  interesting ;  but  when  viewed  in  the  light 
of  a  demonstrative  and  practical  department  of  physical  science,  its 
application  is  necessarily  very  circumscribed,  and  its  character  is 
unimportant  as  an  analytical  theory. 

It  has,  however,  been  very  extensively  studied,  both  in  this  and  in 
foreign  countries,  and  numerous  philosophers  of  the  greatest  eminence, 
possessed  of  the  loftiest  conceptions  and  the  most  profound  mathema- 
tical attainments,  have  deemed  it  a  topic  worthy  of  their  most  atten- 
tive consideration,  and  have  ascribed  to  its  influence,  a  numerous 
class  of  phenomena,  in  reference  to  the  operations  of  nature  on  the 
various  objects  of  our  sublunary  world.  To  this  it  is  owing,  that  the 
rains  which  fall  on  the  higher  elevations,  do  not  immediately  descend 
and  run  to  the  sea  with  an  increasing  velocity,  but  are  retained  by 
the  soil,  and  being  slowly  filtered  down  through  it,  are  cleansed  from 
their  impurities,  and  delivered  in  springs  and  fountains  at  the  foot  of 
the  hills,  so  as  to  afford  a  constant  and  nearly  uniform  supply  of 
moisture  to  the  lower  levels. 

By  capillary  attraction,  does  the  oil  or  melted  tallow  rise  slowly 
through  the  wick  of  a  lamp  or  candle,  where  it  is  converted  into 
vapour  by  the  heat  of  the  surrounding  flame,  and  rushing  out  in  every 
direction,  is  ignited  when  it  comes  in  contact  with  the  circumambient 
air.  By  capillary  attraction,  the  juices  of  the  earth  are  absorbed  by 
plants,  and  carried  through  their  numerous  ramifications  to  the 
remotest  leaf,  where  they  are  again  partly  discharged  by  evaporation, 
after  a  similar  manner  to  that  in  which  the  oil  is  dissipated  from  the 
wick  of  a  lamp,  or  the  melted  tallow  from  the  wick  of  a  candle. 


422     OF  CAPILLARY  ATTRACTION   AND  THE  COHESION   OF  FLUIDS. 

It  is  also  by  the  principles  of  capillary  attraction,  that  the  lymph 
and  other  fluids  are  taken  up,  and  transferred  through  the  ramifying 
vessels  to  every  part  of  the  animal  frame ;  other  causes  dependent  on 
the  organical  structure  both  of  plants  and  animals,  may  assist  in 
producing  this  effect ;  but  it  is  abundantly  proved  by  observations, 
that  by  far  the  greatest  part  of  it  is  produced  by  capillary  attraction 
alone.  It  is  solely  owing  to  it,  that  a  piece  of  dry  wood  absorbs  a 
considerable  quantity  of  moisture,  and  in  consequence  of  this  absorp- 
tion it  swells  with  a  force  almost  irresistible,  thereby  splitting  rocks 
and  other  bodies  of  inconceivable  hardness  and  tenacity. 

Consequently,  since  the  principles  of  capillary  attraction  are  found 
to  exercise  such  extensive  influence  in  the  operations  of  nature,  philo- 
sophers are  justified  in  attempting  to  acquire  a  more  precise  and 
comprehensive  knowledge  of  the  manner  in  which  it  acts,  and  of  the 
laws  by  which  that  action  is  regulated  during  the  period  of  its  opera- 
tion on  natural  bodies. 

526.  DEFINITION.   Capillary  Attraction  is  that  principle  in  nature, 
by  which  water  and  other  liquids  are  made  to  ascend  in  slender  tubes, 
to  heights  considerably  above  the  level  of  the  fluid  in  the  containing 
vessel ;  it  is  so  called,  because  its  influence  is  only  sensible  in  tubes 
whose  bore  is  extremely  small,  in  general  very  little  exceeding  the 
diameter  of  a  hair,  but  never  greater  than  one  tenth  of  an  inch.     The 
tube  thus  limited,  and  in  which  the  fluid  is  found  to  ascend,  is  called 
a  capillary  tube,  from  the  Latin  word  capillus,  a  hair.   The  principles 
of  capillary  attraction,  and  the  theory  which   it  unfolds,   together 
with  its  application  to  tubes  of  various  forms  and  diameters,  we  shall 
very  briefly  consider  in  the  present  chapter ;  the  chief  and  most  im- 
portant properties  peculiar  to  this  subject  are  detailed  in  the  following 
experiments. 

527.  EXPERIMENT  1.   There  is  an  attraction  of  cohesion  between  the 
constituent  particles  of  glass  and  water. 

This  is  manifest,  for  if  a  very  smooth  plate  of  glass  be  brought  into 
contact  with  water,  and  then  gently  removed  from  it,  it  will  be  found 
that  a  small  portion  of  the  fluid  adheres  to  the  glass,  and  remains 
suspended  from  the  lower  surface  when  placed  in  a  horizontal  posi- 
tion ;  hence  the  existence  of  an  attraction  is  inferred,  and  its  intensity 
must  be  such,  as  to  balance  and  sustain  the  gravitating  power  of  the 
water. 

And  again,  if  a  smooth  plate  of  glass  be  suspended  horizontally 
from  one  arm  of  a  lever,  and  kept  in  equilibrio  by  a  weight  applied 
at  the  other  arm  ;  then,  if  the  glass  be  brought  into  contact  with  the 


OF  CAPILLARY  ATTRACTION  AND  THE  COHESION  OF  FLUIDS.      423 

surface  of  the  water,  it  will  be  found  that  an  additional  weight  must 
be  applied  to  the  opposite  arm  to  effect  a  separation  ;  and  the  magni- 
tude of  this  additional  weight,  is  a  precise  measure  of  the  force  of 
cohesion. 

If  the  water  and  the  glass  be  placed  in  a  vacuum,  and  then  brought 
into  contact,  the  same  effect  will  be  found  to  obtain,  and  conse- 
quently, the.  cohesion  is  not  produced  by  the  pressure  of  the  atmos- 
phere ;  hence  an  attraction  must  exist  between  the  particles  of  the 
water  and  the  glass. 

528.  EXPERIMENT  2.    The  constituent  particles  of  a  mass  of  fluid 
are  mutually  attracted ;   that  is,  they  have  an  attraction  towards 
each  other. 

According  to  the  preceding  principle,  when  a  smooth  plate  of  glass 
is  brought  into  contact  with  the  water  and  gently  withdrawn  from  it, 
a  thin  stratum  of  fluid  adheres  to  its  lower  surface ;  now,  if  this 
stratum  of  fluid  be  carefully  weighed,  it  will  be  found  that  its  weight 
is  much  less  than  that  which  is  required  to  detach  it ;  consequently, 
an  attraction  necessarily  exists,  which  would  keep  the  stratum  united 
to  the  fluid  in  the  vessel  independently  of  its  weight,  and  hence  it  is 
inferred,  that  the  particles  are  mutually  attracted  ;  that  is,  they  have 
an  attraction  towards  each  other. 

529.  EXPERIMENT  3.   The  constituent  particles  of  a  mass  of  mer- 
cury have  an  intense  mutual  attraction  ;  that  is,  they  are  strongly 
attracted  towards  each  other. 

This  becomes  manifest  from  the  circumstance  of  the  smallest  quan- 
tity constantly  assuming  a  globular  form,  and  from  the  resistance 
which  it  opposes  to  the  separation  of  its  parts. 

Another  circumstance  which  proves  the  attractive  principle  in  the 
particles  of  mercury,  is,  that  if  a  quantity  of  it  be  separated  into  a 
great  number  of  parts,  they  will  all  of  them  be  spherical ;  and  if  any 
two  of  them  be  brought  into  contact,  they  will  instantly  unite,  and 
constitute  a  single  drop  of  the  same  form  which  they  separately 
assumed. 

530.  EXPERIMENT  4.  The  attractive  power  which  is  evolved  between 
the  particles  of  glass  and  water,  is  sensible  only  at  insensible  dis- 
tances ;  that  is,  'the  attraction  between  the  particles  is  imperceptible, 
unless  the  distance  between  them  be  very  small. 

This  is  inferred  from  the  following  circumstance,  viz.  whatever  may 
be  the  thickness  of  the  plate  of  glass  which  is  brought  in  contact  with 
the  water,  the  force  required  to  detach  it  is  always  the  same.  This 
indicates  that  any  new  laminte  of  matter  that  may  be  added  to  the 


424      OF  CAPILLARY  ATTRACTION  AND  THE  COHESION  OF  FLUIDS. 

plate,  have  no  influence  whatever  upon  the  fluid  with  which  it  is 
brought  in  contact ;  whence  it  follows,  that  the  indefinitely  thin  lamina 
of  fluid  which  attaches  to  the  surface  of  the  plate,  interposes  between 
it  and  the  rest  of  the  fluid  in  the  vessel,  a  sufficient  distance  to  prevent 
any  sensible  effect  from  their  mutual  attraction  ;  and  furthermore,  it 
appears  that  the  force  which  is  requisite  to  detach  all  the  equal  laminae 
of  the  fluid  is  the  same,  being  that  which  is  required  to  separate  an 
individual  film  of  the  fluid  from  the  rest. 

Again,  it  is  manifest  from  observation,  that  water  of  the  same  tem- 
perature, rises  to  the  same  height  in  capillary  tubes  of  the  same  bore, 
whatever  may  be  the  thickness  of  the  glass  of  which  they  are  con- 
stituted; from  this  we  infer,  that  the  laminse  of  the  glass  tube, 
however  small  their  distance  from  the  interior  surface,  have  no 
influence  in  promoting  the  ascent  of  the  fluid. 

If  the  inner  surface  of  a  capillary  tube  be  covered  with  a  very  thin 
coating  of  tallow,  or  some  other  unctuous  substance,  the  water  will 
not  ascend,  for  in  that  case  the  capillary  attraction  is  destroyed  ; 
hence  we  conclude,  that  the  action  of  gravity  and  capillary  action  are 
different  in  their  nature,  for  if  they  were  similar,  the  capillary  force, 
like  the  force  of  gravity,  would  act  through  media  of  all  kinds,  and 
consequently,  would  cause  the  fluid  to  rise  in  the  tube,  notwithstand- 
ing its  inner  surface  being  coated  with  grease. 

531 .  From  the  preceding  experiments,  and  others  of  a  kindred  charac- 
ter, it  is  inferred,  that  the  force  of  attraction  in  a  capillary  tube,  when  it 
exceeds  the  mutual  attraction  of  the  fluid  particles,  extends  its  influence 
no  farther  than  to  the  fluid  immediately  in  contact  with  it,  which  it 
raises;  and  the  water  thus  raised,  by  forming  an  interior  tube,  in  virtue 
of  its  own  attraction,  raises  that  which  is  immediately  in  contact  with 
itself,  and  this  again,  by  extending  its  influence  to  the  lower  particles, 
continues  the  operation  to  the  axis  of  the  tube. 

The  direction  of  the  first  elements  of  the  fluid,  depends  entirely 
upon  the  respective  natures  of  the  fluid,  and  the  solid  with  which  it 
comes  in  contact ;  if  these  are  the  same  in  all  cases,  the  direction  is 
invariable,  whatever  may  be  the  form  of  the  attracting  surface,  whether 
it  be  fashioned  into  a  tube,  or  retains  the  simple  form  of  a  plane  ;  but 
the  direction  of  the  other  elements,  or  those  which  are  situated  out  of 
the  sphere  of  sensible  activity  of  the  attracting  surface,  depends  solely 
on  the  mutual  effect  of  the  fluid  particles,  and  the  form  which  the 
surface  of  the  fluid  assumes,  is  also  regulated  by  the  same  cause. 

532.  From  numerous  experiments  and  careful  micrometrical  observa- 
tions^ has  been  ascertained,  that  when  water  moves  freely  in  a  capillary 


OF  CAPILLARY  ATTRACTION  AND  THE  COHESION  OP  FLUIDS.       425 

tube,  the  surface  forms  itself  into  a  hemisphere,  with  its  vertex  down- 
wards and  its  base  horizontal,  in  which  position  it  nearly  touches  the 
interior  surface  of  the  tube ;  but  when  the  fluid  rises  between  two 
planes,  the  surface  assumes  a  circular  form,  having  for  tangents  the 
planes  by  which  it  is  attracted. 

These  experiments  and  particulars  being  premised,  we  shall  now 
proceed  to  develope  the  theory  of  calculation ;  and  in  order  that  it 
may  be  invested  with  all  the  interest  of  which  it  is  susceptible,  we 
deem  it  advisable  to  adopt  the  method  of  M.  le  Comte  La  Place, 
one  of  the  most  profound  and  sagacious  philosophers  that  have  existed 
in  this,  or  in  any  preceding  age. 


PROBLEM  LXVIII. 

533.  A  cylindrical  tube  of  glass,  whose  diameter  is  exceedingly 
small,  has  its  lower  extremity  immersed  in  a  vessel  of  water, 
and  its  axis  vertical : — 

It  is  required  to  determine  with  what  force  the  water  rises 
in  the  tube,  by  means  of  the  attractive  influence  of  its  surface. 

Let  abed  represent  a  vertical  section  of  a  vessel,  filled  with  water 
to  the  height  BC,  and  let  AB  be  the 
corresponding  section  of  a  small  cylin- 
drical tube  immersed  in  it  at  the  lower 
extremity,  and  having  its  axis  perpen- 
dicular to  the  surface. 

The  fluid  rises  in  the  tube  above  its 
natural  le^vel,  a  thin  film  being  first 
raised  by  the  attraction  of  the  inner 
surface  of  the  glass ;  this  first  film  of 
fluid  raises  a  second,  and  the  second  a 
third,  and  so  on,  until  the  weight  of  the 

elevated  fluid  exactly  balances  all  the  forces  by  which  it  is  actuated, 
viz.  the  attractive  influence  of  the  glass,  and  the  mutual  adherence  of 
its  own  particles. 

Let  us  now  suppose  that  the  inner  surface  of  the  tube  is  produced  to 
E,  then  carried  horizontally  to  D  and  vertically  to  c,  and  let  the  sides 
of  this  extended  tube  be  conceived  to  be  so  extremely  thin,  as  to  have 
no  action  whatever  on  the  contained  fluid,  and  not  to  prevent  the 
reciprocal  attraction  which  obtains  between  the  real  tube  AB  and  the 
particles  of  the  fluid  ;  that  is,  let  the  portion  BEDC  of  the  tube  be  so 


426       OF  CAPILLARY  ATTRACTION  AND  THE  COHESION   OF  FLUIDS. 

circumstanced,  as  neither  to  attract  nor  repel  the  fluid  particles,  and 
consequently,  the  circumstances  of  the  problem  will  not  be  at  all 
affected  by  supposing  the  tube  to  assume  the  form  represented  in  the 
diagram. 

Now,  since  the  fluid  in  the  tube  A  E  is  in  equilibrio  with  that  in  the 
tube  CD,  it  is  manifest,  that  the  excess  of  pressure  in  AE,  arising  from 
the  superior  height  of  the  column,  is  destroyed  by  the  vertical  attrac- 
tion of  the  tube,  together  with  the  mutual  attraction  of  the  fluid 
particles  in  the  tube  AB  ;  in  order  therefore  to  analyze  these  different 
attractions,  we  shall  first  consider  those  that  take  place  under  the 
tube  AB,  in  which  the  fluid  rises  above  its  natural  level. 

534.  In  the  first  place  then,  it  is  evident,  that  the  fluid  in  the 
imaginary  tube  BE,  is  attracted, 

1.  By  the  reciprocal  action  of  its  own  particles, 

2.  By  the  exterior  fluid  surrounding  the  tube  BE, 

3.  By  the  vertical  attraction  of  the  fluid  in  AB,  and 

4.  By  the  attraction  of  the  glass  in  the  tube  AB. 

Now,  the  first  and  second  of  these  attractions,  are  obviously 
destroyed  by  the  equal  and  similar  attractions  experienced  by  the 
fluid  in  the  opposite  branch  DC;  consequently,  their  effects  may  be 
entirely  disregarded.  But  the  vertical  attraction  of  the  fluid  in  the 
tube  AB,  is  also  destroyed  by  an  equal  and  opposite  attraction 
exerted  by  the  fluid  in  BE,  so  that  these  balanced  effects  may  likewise 
be  neglected,  and  there  remains  the  attraction  of  the  glass  in  AB, 
which  operates  to  destroy  the  excess  of  pressure  exerted  by  the 
elevated  column  BF. 

535.  Again,  the  fluid  in  the  lower  portion  of  the  cylindrical  tube 
AB,  is  attracted, 

1.  By  the  reciprocal  action  of  its  own  particles, 

2.  By  the  fluid  in  the  imaginary  tube  BE, 

3.  By  the  attraction  of  the  glass  in  the  tube  by  which  it  is 

contained. 

But  since  the  reciprocal  attractions  of  the  particles  of  a  body,  do 
not  communicate  to  it  any  motion  if  it  is  solid,  we  may,  without 
altering  the  circumstances  of  the  problem,  imagine  the  fluid  in  AB  to 
be  frozen  ;  then,  since  the  fluid  in  the  lower  part  of  AB,  and  that  in  the 
imaginary  tube  BE,  are  acted  on  by  equal  and  opposite  attractions, these 
attractions  destroy  each  other,  and  consequently,  their  effects  may  be 
neglected ;  hence,  the  only  effective  force  which  remains  to  actuate 
the  fluid  in  AB,  is  the  attraction  of  the  glass  containing  it.  Let  this 


OF  CAPILLARY  ATTRACTION  AND  THE  COHESION  OF  FLUIDS.       427 

force  be  denoted  by  /,  which  obtains  equally  in  both  the  cases  above 
stated ;  therefore,  if  F  denote  the  intensity  of  the  vertical  attractive 
force,  we  shall  have 

*  =  2f. 

536.  But  there  is  a  negative  force  acting-  in  the  opposite  direction,  by 
which  this  value  of  F  is  influenced,  and  which  arises  from  the  attrac- 
tion exerted  by  the  fluid  surrounding  the  imaginary  tube,  on  the  lower 
particles  in  the  column  BE,  and  the  result  of  this  attraction  is  a 
vertical  force  acting  downwards,  in  opposition  to  the  force  2/";  let 
this  antagonist  force  be  denoted  by/',  and  we  shall  obtain 

FZZ2/-/'. 

Put  m  zz  the  magnitude,  or  solid  contents  of  the  column  B  F, 
I  zz  the  density  or  specific  gravity  of  the  fluid,  and 
g  zz  the  power  of  gravity. 

Then  by  multiplying  these  quantities  together,  the  weight  of  the 
elevated  column  is  expressed  by  m%g ;  but  in  the  case  of  an  equili- 
brium between  this  weight  and  the  attractive  forces  by  which  it  is 
elevated,  it  is  manifest  that  they  are  equal ;  hence  we  have 

mlg—lf—f.  (310). 

If  the  force  2/  be  less  than  — /',  the  value  of  m  or  the  magnitude 
of  the  attracted  column  will  be  negative,  and  the  fluid  will  sink  in  the 
tube ;  but  whenever  the  force  2/  exceeds  — /',  the  value  of  m  will  be 
positive,  and  the  fluid  will  rise  above  its  natural  level. 

537.  Since  the  attractive  forces,  both  of  the  glass  and  the  fluid,  are 
insensible  at  sensible  distances,  the  surface  of  the  tube  A  B  will  have 
a  sensible  effect  on  the  column  of  fluid  immediately  in  contact  with 
it;  this  being  the  case,  we  may  neglect  the  consideration  of  curvature, 
and  conceive  the  inner  surface  to  be  developed  upon  a  plane  ;  the 
force  f  will  therefore  be  proportional  to  the  width  of  this  plane,  or 
which  is  the  same  thing,  to  the  inner  circumference  of  the  tube. 

Put  d  zz  the  inner  diameter  of  the  tube, 

TT  zz  the  ratio  of  the  circumference  to  the  diameter, 

0  zz  a  constant  quantity,  representing  the  intensity  of  the 

attraction  of  the  tube  upon  the  fluid,  and 
9'  zz  another  constant,  representing  the  intensity  of  attraction 

which  the  fluid  exerts  upon  itself. 

Then,  by  the  principles  of  mensuration,  we  have  tin  equal  to  the 
inner  circumference  of  the  tube,  and  also  to  the  exterior  circum- 
ference of  a  column  of  fluid  of  the  same  diameter ;  therefore,  it  is 


428       OF  CAPILLARY  ATTRACTION   AND  THE  COHESION  OF  FLUIDS. 


f=i  dirty, 
which  being  substituted  for  /and/'  in  equation  (310),  gives 

m$g  =  dir('2<l>  —  f).  (311). 

This  is  the  general  equation  that  expresses  the  force  by  which  the 
water  is  raised  in  a  cylindrical  tube,  and  its  application  to  particular 
cases  will  be  exemplified  by  the  resolution  of  the  following  problems. 


PROBLEM  LXIX. 

538.  In  a  cylindrical  capillary  tube  of  a  given  diameter,  the 
top  of  the  elevated  column  is  terminated  by  a  hemisphere  : — 

It  is  therefore  required  to  determine  the  height  to  which 
the1  fluid  ascends  above  its  natural  level. 

Let  abed  be  a  section  passing  along  the  axis  of  a  very  small 
cylindrical  tube,  of  whioh  the  diameter  is 
ab;  let  the  tube  be  vertically  immersed  in 
the  fluid  whose  surface  is  IK,  and  suppose 
that  in  consequence  of  the  immersion,  the 
fluid  rises  in  the  tube  to  e  on  a  level  with 
the  surface  IK,  and  from  thence  it  is  at- 
tracted by  the  glass  in  the  tube,  together 
with  the  mutual  action  of  its  own  particles, 
until  it  arrives  at  ab,  where  it  forms  the 
spherical  meniscus  abfg,  and  in  which 

position,  the  weight  of  the  elevated  column  is  in  equilibrio  with  the 
attractive  forces. 

Now,  the  problem  demands  the  height  to  which  the  fluid  rises  in 
the  tube  in  consequence  of  the  attraction,  and  on  the  supposition 
that  its  diameter  is  very  small. 

Put  r  =  am,  the  radius  of  the  interior  surface  of  the  tube, 

h  =n  en,  the  height  of  the  uniform  column,  or  the  distance 

between  the  surface  of  the  fluid  and  the  lowest  point 

of  the  spherical  meniscus, 
A'n:  ev,  the  mean  altitude,  or  the  height  at  which  the  fluid 

would  stand,  if  the  meniscus   were  to  fall  down  and 

form  a  level  surface, 

TT  —  the  ratio  of  the  circumference  to  the  diameter,  and 
m  i=  the  magnitude  of  the  whole  elevated  column. 


OF  CAPILLARY  ATTRACTION  AND  THE  COHESION  OF  FLUIDS.      429 

Then,  by  the  principles  of  mensuration,  it  is  manifest  that  the 
inner  circumference  of  the  tube  is  2r7r,  and  the  solidity  of  the 
uniform  column  whose  height  is  en,  becomes  r*/nr;  now,  the  solidity 
of  the  meniscus  ganbf,  is  obviously  equal  to  the  difference  between 
the  cylinder  abfg  and  its  inscribed  hemisphere  an  b. 

But  by  the  rules  for  the  mensuration  of  solids,  we  know  that  tl>e 
solidity  of  the  cylinder  abfg  is  r87r,  and  that  of  the  inscribed  hemi- 
sphere is  fr37r  ;  consequently,  the  solidity  of  the  meniscus  is 

rs7r  —  fr37r  =  £r87r, 
which  being  added  to  the  solidity  of  the  uniform  column,  gives 

wzrr2/i7r  -f-  T^TJ 
from  which,  by  collecting  the  terms,  we  get 


Now  this  is  equivalent  to  the  solidity  of  a  cylinder,  whose  radius  is 
r  and  altitude  evrz  h'  ;  consequently,  we  have 
m  =  r*K  (h  +  £r)  zz  rV  h'  ; 
whence  it  appears,  that 

h'  =  h  +  ir.  (312). 

539.  Instead  of  din  the  equation  (3  11),  let  its  equal  2r  be  substituted, 
and  instead  of  m  in  the  same  equation,  let  its  equivalent  h'r^ir  be 
introduced,  and  we  shall  obtain 

h'  r9  TT  $g  =  2r  TT  (20  —  0'), 
and  from  this,  by  casting  out  the  common  factors,  we  get 

A'  r  30  =  2(20—  A 
and  dividing  by  5#,  it  becomes 


Now,  since  the  symbols  <j>,  0',  3  and  g  are  constant  for  the  same 
fluid  and  material,  it  follows  that  the  whole  expression  is  constant  ; 
hence,  the  height  to  which  the  fluid  rises,  varies  inversely  as  the 
radius  of  the  tube. 

540.  Instead  of  h'  in  the  equation  (313),  let  its  equivalent  (h  -f-  £r) 
in  equation  (312)  be  substituted,  and  we  shall  obtain 

"     ' 


2(2<6—  0') 
Hence  it  is  manifest,   that  the   constant  quantity  -'—-  —  —  ,  is 

equal  to  the  mean  altitude  of  the  fluid  multiplied  by  the  radius  of  the 
tube  ;  and  it  has  been  shown  in  equation  (312),  that  the  mean  altitude 


430     OF  CAPILLARY  ATTRACTION  AND  THE  COHESION  OF  FLUIDS. 

is  equal  to  the  observed  altitude  of  the  lowest  point  of  the  meniscus, 
increased  by  one  third  of  the  radius  of  the  tube,  or  which  is  the  same 
thing,  by  one  sixth  of  the  diameter  ;  the  value  of  the  constant  quantity, 
can  therefore  only  be  determined  by  experiment,  and  accordingly  we 
find,  that  various  accurate  observations  have  been  made  for  the  purpose 
of  assigning  the  value  of  this  element;  the  mean  of  which,  according 
to  M.  Weitbrecht,  gives 


hence,  finally,  we  obtain 

#r=.0214.  (315). 

541  .  The  equation  (315),  it  may  be  remarked,  is  general  for  cylindrical 
tubes,  if  the  elevated  column  of  fluid  is  terminated  by  a  hemispherical 
meniscus,  and  the  practical  rule  which  it  supplies,  is  simply  as  follows. 

RULE.   Divide  the  constant  fraction  .0214  by  the  radius  of 
the  capillary  tube,  and  the  quotient  will  express  the  mean 
altitude  to  which  the  Jluid  rises  above  its  natural  level. 
If  it  be  required  to  determine  the  highest  point  to  which  the  fluid 
particles  ascend,  it  will  be  discovered,  by  adding  to  the  mean  altitude 
two  thirds  of  the  radius  of  the  tube,  or  one  sixth  of  the  diameter. 

542.  EXAMPLE.  The  diameter  of  a  cylindrical  tube  of  glass,  is  .06  of 
an  English  inch  ;  now,  supposing  it  to  be  placed  in  a  vertical  position, 
with  its  lower  extremity  immersed  in  a  vessel  of  water  ;  what  is  the 
mean  altitude  to  which  the  fluid  will  ascend,  and  what  is  the  altitude 
of  the  highest  particles  ? 

Since,  according  to  the  question,  the  diameter  of  the  tube  is  .06 
of  an  inch,  the  radius  is  .03  or  half  the  diameter  ;  consequently,  by 
the  rule,  the  mean  altitude  to  which  the  water  rises,  is 
h'  =..  0214  -~  .  03  —  0.713  of  an  inch, 
and  therefore,  the  point  of  highest  ascent,  is 
0.713  4-  .02  =z  0.733  of  an  inch. 

543.  If  the  mean  altitude  of  the  fluid  is  given,  the  radius  of  the  tube 
can  easily  be  found  from  equation  (315),  for  it  only  requires  the  con- 
stant number  .0214  to  be  divided  by  the  given  altitude  ;  but  when  the 
observed  altitude,  or  the  distance  between  the  surface  of  the  fluid  in 
the  vessel,  and  the  lowest  point  of  the  meniscus  is  given,  the  radius 
can  only  be  determined  by  the  resolution  of  an  adfected  quadratic 
equation  ;  for  by  equation  (314),  we  have 

i**4-  Ar-zz.0214, 


OF  CAPILLARY  ATTRACTION  AND  THE  COHESION  OF  FLUIDS.      431 

which  being  multiplied  by  3,  becomes 

7-2-f3£r=.0642.  (316). 

544.  Suppose  now,  that  the  observed  altitude  is  0.703  of  an  inch; 
then,  .by  substituting  0.703  instead  of  h  in  equation  (315),  we  obtain 


consequently,  by  completing  the  square,  we  get 

r»+  2.11r  -4-  1.0552—  1.177225, 
from  which,  by  extracting  the  square  root,  we  obtain 

r  +  1.  055  =  1.085; 

hence,  by  subtraction,  we  have 

r—  1.085  —  1.055  =  .  03  of  an  inch. 

Now,  if  one  third  of  the  radius  just  found,  be  added  to  the  observed 
altitude,  the  sum  thence  arising  will  express  the  mean  altitude  ;  and 
if  the  whole  radius  be  added  to  the  observed  altitude,  the  sum  will 
express  the  greatest  height  to  which  the  fluid  rises  in  the  tube. 


PROBLEM  LXX. 

545.  Two  parallel  planes  of  glass  or  other  materials,  are  placed 
in  a  vertical  position,  with  the  lower  sides  immersed  in  a  fluid : — 

It  is  required  to  determine  how  high  the  fluid  rises  between 
them,  their  distance  asunder  being  very  small  in  comparison 
to  their  surfaces. 

Let  ad  and  be  represent  the  ends  or  sections  of  two  plates  of  glass, 
placed  in  a  position  of  vertical  parallelism, 
and  having  their  lower  edges  d  and  c  im- 
mersed in  a  fluid  of  which  the  surface  is  IK. 
Suppose  now  that  a  b  or  cd,  the  distance 
between  the  plates,  is  very  small  in  com- 
parison to  their  extent  of  surface ;  then  it  is 
obvious,  that  the  fluid  will  rise  between  them 
as  high  as  e  in  consequence  of  the  immersion, 
and  from  thence  it  moves  upwards  by  the 
attractive  influence  of  the  glass,  and  the 

mutual  action  of  its  own  particles,  until  it  arrives  at  rs,  where  it  forms 
a  semi-cylindrical  meniscus  rsfg,  whose  diameter  is  equal  to  the  dis- 
tance between  the  planes,  and  its  length  the  same  as  their  horizontal 
breadth. 


432     OF  CAPILLARY  ATTRACTION  AND  THE  COHESION   OF  FLUIDS. 

In  this  position,  the  whole  weight  of  the  elevated  fluid,  and  the 
united  efforts  of  the  attractive  forces,  are  in  equilibrio  among  them- 
selves, and  the  problem  requires  the  height  to  which  the  fluid  rises, 
when  the  powers  of  gravitation  and  attraction  become  equal  to  one 
another  ;  for  this  purpose, 

Put  b  zr  the  horizontal  breadth  of  the  planes  by  whose  attraction 

the  fluid  is  elevated, 

d  •=.  ab,  the  perpendicular  distance  between  the  planes, 
li  zz  e  n,  the  distance  between  the  lowest  point  of  the  meniscus 

and  the  surface  of  the  fluid, 

h'  zz  ev,  the  mean  altitude  of  the  fluid,  or  the  height  at  which 
it  would  stand,  if  the  meniscus  were  to  fall  down  and 
form  a  level  surface, 
TT  zz  the  ratio  of  the  circumference  of  a  circle  to  its  diameter, 

and 

m  zz  the  magnitude  of  the  volume  of  fluid  raised. 
Then,  if  the  constants  <£,  0',  S  and  g  denote  as  before,  the  magnitude 
of  the  elevated  volume  will  be  found  as  follows. 

546.  By  the  principles  of  mensuration,  the  solidity  of  the  fluid 
parallelopipedon,  whose  breadth  is  b,  thickness  d,  and  height  h,  is 
expressed  by  bdh;  and  the  solidity  of  the  fluid  meniscus  whose 
section  is  grnsf,  is  equal  to  the  difference  between  a  semi-cylinder 
and  its  circumscribing  parallelopipedon,  the  length  being  equal  to 
bj  and  the  diameter  equal  to  d,  the  distance  between  the  attracting 
planes. 

Now,  the  solidity  of  the  circumscribing  parallelopipedon  is  |6d8, 
and  the  solidity  of  the  semi-cylinder  is  J6eP?r;  consequently,  the 
solidity  of  the  meniscus,  is 


to  which  if  we  add  the  solidity  of  the  uniform  solid,  the  whole  magni- 
uid becomes 
m  —  bdh  -\-  \bd\1  —  TT). 


tude  of  the  elevated  fluid  becomes 


But  the  periphery  of  the  fluid  which  is  elevated  between  the  planes, 
is  manifestly  equal  to  2(£  +  d)  ;  consequently,  by  substituting  this 
value  of  the  periphery  for  dir  in  equation  (311);  and  for  m,  let  its 
value  as  determined  above  be  substituted,  and  we  shall  obtain 
{bdh+lbd\1-Tr}}Zg=i1(b+d)(<l<t>  —  f), 
and  dividing  both  sides  by  b$g,  it  becomes 


\ 

vr 


OF  CAPILLARY  ATTRACTION  AND  THE  COHESION  OF  FLUIDS.      433 


but  since  d  is  conceived  to  be  very  small  in  comparison  with  b,  the 

— 


horizontal  breadth  of  the  plates,  the  fraction  —  may  be  considered  as 


evanescent,  and  then  we  get 


The  solidity  of  the  fluid  parallelopipedon  corresponding  to  the 
mean  altitude,  is  expressed  by  bdh'i  but  this  is  equal  to  the  whole 
quantity  of  fluid  raised  ;  therefore  we  have 

bdh'=bd{h  +  Jd(l—  J*0}, 
from  which,  by  casting  out  the  common  terms,  we  get 

A'=  A  +Jrf(l  —  £TT);  (318). 

Now,  (1  —  |TT)  is  a  constant  quantity  ;  hence  it  appears,  that  the 
mean  altitude  varies  inversely  as  the  distance  between  the  planes. 

547.  Let  the  symbol  for  the  mean  altitude,  be  substituted  in 
equation  (317),  instead  of  its  analytical  value  as  expressed  in  equation 
(318),  and  we  shall  obtain 


where  the  value  of  the  constant  quantity  is  the  same  as  before  ;  hence 
we  have 

rfA'=.0214.  (319). 

The  practical  rule  which  this  equation  supplies,  may  be  expressed 
in  words,  in  the  following  manner. 

RULE.  Divide  the  constant  number  0.  021  4  by  the  perpen- 
dicular distance  between  the  planes,  and  the  product  will 
give  the  mean  altitude  to  which  the  fluid  rises. 

548.  EXAMPLE.  The  parallel  distance  between  two  very  smooth  plates 
of  glass,  is  0.06  of  an  inch  ;  now,  supposing  the  lower  edges  of  the 
plates  to  be  immersed  in  a  vessel  of  water  ;  what  is  the  mean  altitude 
to  which  the  fluid  ascends  ? 

Here,  by  operating  as  the  rule  directs,  we  have 
h'  =  0.0214  -f.  0.06  —  0.356  of  an  inch. 

In  this  example,  the  distance  between  the  planes  is  the  same  as  the 
diameter  of  the  tube  in  the  preceding  case,  but  the  mean  altitude  of 
the  fluid  is  only  one  half  of  its  former  quantity  ;  hence  it  appears, 
that  if  the  tube  and  the  planes  are  of  the  same  nature  and  substance, 
and  the  radius  of  the  one  the  same  as  the  distance  between  the 

VOL.  i.  2  F 


434      OF  CAPILLARY  ATTRACTION  AND  THE  COHESION  OF  FLUID9. 

other,  the  fluid  will  rise  lo  the  same  height  in  them  both,  if  they  are 
placed  under  the  same  or  similar  circumstances. 

549.  Having  given  the  mean  altitude  to  which  the  fluid  rises,  the 
distance  between  the  plates  can  easily  be  ascertained  ;  for  we  have 
only  to  divide  the  constant  number  0.0214  by  the  given  altitude, 
and  the  quotient  will  give  the  distance  sought  ;  but  if  the  observed 
altitude,  or  the  distance  between  the  lowest  point  of  the  meniscus 
and  the  surface  of  the  fluid  in  the  vessel  be  given,  the  operation  is 
more  difficult,  since  it  requires  the  reduction  of  an  adfected  quadratic 
equation. 

By  recurring  to  equation  (317),  it  appears,  that 


but  we  have   shown,    equations  (315  and   319),   that  the  constant 


quantity  —L~  —  —  ,  has,  from  the  comparison  of  experiments,  been 

*9 
assumed  zz  0.0214  ;  hence  it  is 

d{  h  +  \d  (1  —  |TT)  =  0.0214  ; 

now,  the  value  of  the  parenthetical  quantity  (1  —  JTT)  is  also  known, 
being  equal  to  1  —  .  7854  —  .  2146  ;  consequently,  by  substitution, 
we  have 

0.1073d2+hd=  0.0214, 

and  dividing  both  sides  by  0.1073,  it  becomes 

d2-f-  9.  32hd=  0.1994. 

Let  us  therefore  suppose,  that  the  observed  altitude  of  the  fluid,  or 
the  value  of  h  is  equal  to  0.2913  parts  of  an  inch,  and  on  this 
supposition,  the  above  equation  will  become 

rf*+  2.71473^:=  0.1994, 

and  this  equation  being  reduced  according  to  the  rules  for  quadratics, 
we  finally  obtain 

d  =  1.429  —  1.357  zz  0.072  of  an  inch. 

550.  The  preceding  theory  has  reference  to  the  phenomena  of  capil- 
lary attraction,  as  they  are  displayed  in  cylindrical  tubes  and  parallel 
plates  of  glass  ;  it  would  however,  be  no  difficult  matter  to  extend 
the  inquiry  to  figures  of  other  forms,  and  placed  under  various  cir- 
cumstances ;  but  being  aware  that  an  extended  inquiry  would  elicit 
no  new  principle,  we  have  thought  proper  to  omit  it  ;  the  property 
disclosed  in  the  following  problem,  is  however,  of  too  curious  and 
interesting  a  character  to  be  passed  over  without  notice,  we  shall 
therefore  endeavour  to  draw  up  the  solution  in  the  most  concise  and 
intelligible  manner  which  the  nature  of  the  subject  will  permit. 


OF  CAPILLARY  ATTRACTION  AND  THE  COHESION  OF  FLUIDS.      435 


PROBLEM  LXXI. 

551.  If  two  smooth  plates  of  glass  be  inclined  to  each  other  at 
a  very  small  angle,  having  their  lower  sides  brought  in  contact 
with  a  fluid,  to  the  surface  of  which  the  coincident  edges  are 
vertical : — 

It  is  required  to  determine  the  nature  of  the  curve  which 
the  fluid  forms  upon  the  plates,  by  rising  up  in  virtue  of  the 
attraction. 

Let  ABEF  and  CDEF  be  the  smooth  plates  of  glass,  having  their 
edges  coinciding  in  the  line  EF,  and 
their  planes  inclined  to  each  other 
in  the  angle  BED;  and  suppose  the 
edges  BE  and  DE  to  be  coincident 
with  the  fluid,  while  EF  the  line  in 
which  the  plates  are  brought  toge- 
ther, is  perpendicular  to  its  surface, 
which  is  represented  by  the  plane 
BED;  then  shall  FTWOB  and  FW^D, 
be  curves  described  by  the  particles 
of  the  fluid  upon  the  surface  of  the 
plates. 

Take  any  two  points  t  and  r  in 

the  line  DE,  and  in  the  plane  CDEF,  draw  tn  and  rp  perpendicular 
to  DE,  and  meeting  the  curve  FW^D  in  the  points  n  and  p ;  the  lines 
tn  and  rp  are  therefore  parallel  to  EF  the  line  of  coincidence,  and 
perpendicular  to  BED  the  surface  of  the  fluid. 

Again,  from  the  same  points  t  and  r,  and  in  the  plane  BED  co- 
incident with  the  fluid's  surface,  draw  the  straight  lines  ts  and  rq 
perpendicular  to  DE,  and  consequently,  parallel  to  each  other;  then, 
from  the  points  s  and  q,  in  which  the  lines  ts  and  rq  meet  BE  the 
lower  edge  of  the  plane  ABEF,  draw  sm  and  qv  respectively  parallel 
to  tn  and  rp,  and  meeting  the  curve  FWOB  in  the  points  m  and  o; 
these  lines  are  consequently  parallel  to  EF  and  perpendicular  to  the 
plane  BED. 

552.  Since  by  the  supposition,  the  angle  BED  which  measures  the 
inclination  of  the  planes,  is  very  small,  the  fluid  in  each  section  may 
be  conceived  to  be  elevated  by  the  attraction  of  parallel  planes,  and 
consequently,  by  an   inference   under   the  preceding  problem,  the 
altitude  of  the  fluid  at  any  two  points,  will  vary  inversely  as  the 
distances  between  the  planes  at  those  points  ;  therefore,  we  have 

2r2 


436      OF  CAPILLARY  ATTRACTION  AND  THE  COHESION  OF  FLUIDS. 

in  :  rp  :  :  rq  :  ts ; 
but  by  the  property  of  similar  triangles,  it  is 

Er  :  E£  :  :  rq  :  ts; 
consequently,  by  the  equality  of  ratios,  we  obtain 

Er  :  E£  :  :  tn  :  rp, 
and  by  equating  the  products  of  the  extreme  and  mean  terms,  it  is 

vrXrp  =  EtXtn.  (320). 

Now,  according  to  the  principles  of  conic  sections,  we  have  it,  that 
in  the  common  or  Apollonian  hyperbola,  if  the  abscissae  be  estimated 
from  the  centre  along  the  asymptote,  the  corresponding  ordinates  are 
to  one  another  inversely  as  the  abscissae ;  hence  it  is  manifest,  that 
the  curve  which  the  surface  of  the  elevated  fluid  traces  on  the  plates, 
is  the  curve  of  a  hyperbola,  whose  properties  are  indicated  by  equation 
(320). 

553.  Such  then  is  the  theory  of  capillary  attraction,  in  so  far  as  it 
is  necessary  to  pursue  it;  but  we  shall  just  remark  in  passing,  that 
other  fluids,  such  as  alcohol,  spirit  of  turpentine,  oil  of  tartar,  spirit 
of  nitre,  oil  of  olives,  and  the  like,  are  elevated  in  the  same  manner 
as  water,  but  to  a  less  degree ;  thereby  showing  that  the  affinity  of 
glass  to  water,  is  greater  than  its  affinity  to  any  other  liquid. 

Again,  on  the  other  hand,  some  fluids  are  depressed  by  the  action 
of  the  capillary  force,  such  as  mercury,  melted  lead,  and  indeed  all 
the  metals  in  a  state  of  fusion,  are  more  or  less  depressed,  according 
to  their  density  or  specific  gravity  ;  but  an  inquiry  into  the  quantity 
of  depression  in  this  place,  would  lead  to  nothing  new  or  interesting, 
and  as  a  subject  of  practical  utility,  it  is  altogether  unimportant ;  we 
therefore  pass  it"  over,  and  hasten  to  lay  before  our  readers  a  detail 
of  the  experiments  performed  by  the  celebrated  M.  Monge,  on  the 
approximation  and  recession  of  bodies  floating  near  each  other  on  the 
surface  of  a  fluid. 

The  following  are  a  few  of  the  principal  experiments  that  have  been 
made  on  this  subject. 

554.  EXPERIMENT  1.  If  two  light  bodies,  capable  of  being  wetted 
with  water,  are  placed  one  inch  asunder  on  its  surface,  in  a  state  of 
perfect  quiescence,  they  will  float  at  rest,  and  experience  no  motion 
but  what  is  derived  from  the  agitation  of  the  air ;  but  if  they  are 
placed  apart  only  a  few  lines,  they  will  approach  each  other  with  an 
accelerated  velocity. 

Also  if  the  vessel  be  of  glass,  or  such  as  is  capable  of  being  wetted  with 
water,  and  if  the  floating  body  is  placed  within  a  few  lines  of  the  edge 
of  the  vessel,  it  will  approach  to  the  edge  with  an  accelerated  velocity. 


OF  CAPILLARY  ATTRACTION  AND  THE  COHESION  OF  FLUIDS.     437 


555.  EXPERIMENT  2.  If  the  two  floating  bodies  are  not  capable  of 
being  wetted  with  the  fluid,  such  as  two  balls  of  iron  in  a  vessel  of 
mercury,  and  if  they  are  placed  at  the  distance  of  a  few  lines,  they  will 
move  towards  each  other  with  an  accelerated  velocity;  and  if  the  vessel 
is  made  of  glass,  in  which  the  surface  of  the  mercury  is  always  convex, 
the  bodies  will  move  towards  the  side  when  they  are  placed  within  a 
few  lines  of  it. 

556.  EXPERIMENT  3.  If  one  of  the  bodies  is  susceptible  of  being 
wetted  with  water,  and  the  other  not,  such  as  two  globules  of  cork, 
one  of  which  has  been  carbonized  with  the  flame  of  a  taper ;  then,  if 
we  attempt,  by  means  of  a  wire  or  any  other  small  stylus,  to  make  the 
bodies  approach,  they  will  fly  or  recede  from  each  other  as  if  they 
were  mutually  repelled;    and  if  the  vessel   is  of  glass,  having  the 
carbonized  ball  of  cork  placed  in  it,  it  will  be  found  impossible  to 
bring  the  cork  in  contact  with  the  sides  of  the  vessel. 

In  these  experiments  it  is  manifest,  that  the  approximation  and 
recession  of  the  floating  bodies,  are  not  produced  by  any  attraction 
or  repulsion  between  them ;  for  if  the  bodies,  instead  of  floating  on 
the  fluid,  are  suspended  by  slender  threads,  it  will  be  observed  that 
they  have  not  the  slightest  tendency  either  to  approach  or  recede, 
when  they  are  brought  extremely  near  to  each  other. 

From  an  attentive  consideration  of  the  phenomena  exhibited  in 
these  experiments,  we  may  deduce  the  following  laws. 

557.  (1.)  If  two  bodies,  capable 
of  being  wetted  by  a  fluid,  are 
placed    upon     its    surface    and 
brought  near  to  each  other,  they 
will   approach  as    if  they   were 
mutually  attracted. 

For  if  two  plates  of  glass  AB, 
CD  are  brought  so  near  each 
other,  that  the  point  H,  where 
the  two  curves  of  elevated  fluid 
meet,  is  on  a  level  with  the  rest 
of  the  mass,  they  will  remain  in 
a  state  of  perfect  equilibrium. 

If,  however,  they  are  brought 
nearer  together,  the  water  will 
rise  between  them  to  the  point 
G  ;  the  water  thus  raised,  attracts 
the  sides  of  the  glass  plates,  and 


D 


438      OF  CAPILLARY  ATTRACTION  AND  THE  COHESION  OF  FLUIDS. 

causes  them  to  approximate  in  a  horizontal  direction,  the  mass  of  fluid 
having  always  the  same  effect  as  a  heavy  chain  attached  to  the 
plates. 

The  same  thing  is  true  of  two  floating  bodies,  when  they  come 
within  such  a  distance  that  the  fluid  is  elevated  between  them ;  for 
it  is  obvious  that  the  bodies  A  and 
B,   being   placed    at   a   capillary 
distance  asunder,  have  the  fluid 
elevated  between  them,  and  are 
therefore  brought  together  by  the 
attractive   influence  of  the  fluid 
upon  the  sides  of  the  globules. 

558.  (2.)   If  two  bodies  are  not  susceptible  of  being  wetted  by  the 
fluid,  they  will  still  approach  each  other  when  brought  nearly  into 
contact,  as  if  they  were  mutually  attracted. 

For  if  the  two  floating  bodies  A  and  B,  are  not  capable  of  being 
wetted  by  the  liquid,  it  will  be 

depressed  between  them  as  at  H,  A.       B 

below  its  natural  level,  when  they 
are  placed  at  a  capillary  distance ; 
hence  it  appears,  that  the  two 
bodies  are  more  pressed  inwards 

by  the  fluid  which  surrounds  them,  than  they  are  pressed  outwards 
by  the  fluid  between  them,  and  in  virtue  of  the  difference  between 
these  pressures,  they  mutually  approach  each  other. 

559.  (3.)  If  one  of  the  two  bodies  is  susceptible  of  being  wetted  by 
the  fluid,  and  the  other  not,  they  will  recede  from  each  other  as  if 
they  were  mutually  repelled. 

For  if  one  of  the  bodies  as  A,  is  capable  of  being  wetted,  while  the 
other  as  B  is  not,  the  fluid  will  rise 
round  A  and  be  depressed  round  B; 
hence,   the  depression   round  B   will 
not  be   uniform,   and   therefore,  the 
body  B,  being  placed  as  it  were  upon 
an  inclined  plane,  its  equilibrium  is  destroyed,   and  it  will  move 
towards  that  side  where  the  pressure  is  least. 

These  laws,  deduced  from  experiment  by  M.  Monge,  have  been 
completely  verified  by  the  theory  of  capillary  attraction  as  developed 
by  La  Place ;  from  his  theory  it  follows,  that  whatever  be  the  nature 
of  the  substances  of  which  the  floating  bodies  are  made,  the  tendency 
of  each  of  them  to  a  coincidence,  is  equal  to  the  weight  of  a  prism  of 


HYDROSTATIC  PRESSURE  EXEMPLIFIED  IN  SPRINGS,  &C.          439 

the  fluid,  whose  height  is  the  elevation  of  the  fluid  between  the  bodies, 
measured  to  the  extreme  points  of  contact  of  the  interior  fluid,  and 
minus  the  elevation  of  the  fluid  on  the  exterior  sides.  The  elevation, 
however,  must  be  reckoned  negative  when  it  changes  into  a  depression, 
as  is  the  case  with  mercury  and  other  metals  in  a  state  of  fusion,  as 
has  been  observed  elsewhere. 

HYDROSTATIC     PRESSURE    EXEMPLIFIED     IN     SPRINGS    AND 
ARTESIAN    WELLS. 

560.  The  atmosphere  is  the  uninterrupted  source  of  communication 
between  the  sea  and  the  earth ;  it  is  the  capillary  conductor  of  water 
from  the  ocean  to  the  land.     Water  ascends  in  the  form  of  vapour, 
and  descends  as  dew  or  rain  upon  the  earth,  which  however  it  pene- 
trates but  a  small  depth,  except  by  fissures  and  permeable  strata,  which 
conduct  it  to  subterranean  reservoirs,  whence  it  again  issues  as  in  the 
discharge  of  springs;  or  when  the  earth  is  bored  through,  it  rises  as 
in  wells.     Some  wells  are  fed  by  land  springs — springs  of  shallow 
depth ;  others  are  fed  from  the  percolation  of  water  through  strata 
that  act  as  conduits,  conveying  a  current  of  the  fluid  through  their 
permeable  texture  from  one  high  land  to  another.     Hence  it  is,  that 
in  valleys  and  champaign  districts,  very  deep  wells  are  dug,  in  order 
to  arrive  at  those  great  feeders,  where  the  hydrostatic  pressure  sends 
the  water  up  with  amazing  force.     In  some  cases  we  can  trace  the 
source  of  springs ;   and  with  the  help  of  FATHER  KIRCHER'S  Mundus 
Subterraneus,  a  man  of  a  fanciful  wit  might  present  the  public  with  a 
very  learned  treatise  on  Natural  Hydraulics  and  Artesian  Wells.* 

561.  As  regards  rock  springs,  we  know  of  none  that  surpass  the 
sources  of  the  Scamander,  in  Asia  Minor,  an  account  of  which  will  be 
found  in  some  notes  accompanying  Poems  of  the  Rev.  Mr.  Carlyle,  who 
saw  the  stream  of  the  Menderi  issuing  from  a  cave  surrounded  with 
trees,  and  tumbling  down  the  crags  in  a  foaming  cascade ;  for  there 
the  cavern  that  "  broods  the  flood  divine,"  discharges  its  sacred  stores 
by  two  large  openings  in  the  rock,  which  leads  into  the  cavern.    Upon 
entering  the  recess,  two  other  openings,  nearly  answering  to  the  out- 
ward ones,  like  arches  in  a  cloister,  present  themselves  to  the  sight; 
and  through  one  of  them,  in  a  basin  below,  the  traveller  perceives  the 

*  From  Artois  (the  ancient  Artesium  of  Gaul),  where  perpetually  flowing  artifi- 
cial fountains  are  obtained,  by  boring  a  small  hole  through  strata  destitute  of  water, 
into  lower  strata  loaded  with  subterraneous  sheets  of  this  important  fluid,  which 
ascends  by  pipes  let  down  to  conduct  it  to  the  surface. 


440  HYDROSTATIC  PRESSURE  EXEMPLIFIED 

just  emerged  Scamander.  The  channel  that  conducts  the  stream  into 
the  basin  is  a  cleft  in  the  rock  towards  the  right,  only  about  four  feet 
wide  and  nearly  twenty  in  height;  it  winds  inwards  in  a  curve,  and 
is  soon  lost  in  darkness ;  at  its  bottom  glides  the  current,  which  for  a 
few  moments  seems  to  repose  in  the  basin  beside,  and  then,  by  another 
subterraneous  channel,  rushes  to  the  mouth,  from  whence  it  issues  to 
the  day :  it  here  bursts  from  the  precipice,  and  forms  a  noble  water- 
fall between  forty  and  fifty  feet  high,  broken,  and  furnished  with 
every  accompaniment  that  the  admirer  of  picturesque  beauty  could 
require :  its  sides  are  fringed  with  pine  and  brushwood ;  below,  it  is 
almost  hidden  from  the  view  by  immense  fragments  of  rock  that  have 
fallen  from  the  precipice ;  and  above  it  hang  crags  of  from  two  to 
three  hundred  feet  in  height,  that  jut  over  the  bases  in  large  angular 
prominences.  Such  is  the  spring  which  flows  through  the  sweet 
vale  of  Menderi  in  many  a  winding  turn.  Far  above  this  is  the 
summit  of  Ida — the  snowy  head  of  Khasdag,  the  seat  of  the  immortals 
— from  whence  the  bard  of  yore  could  view  Mysia,  the  Propontis,  the 
Hellespont,  the  JEgean  sea,  Lydia,  Bythynia,  and  Macedonia. 

562.  If  sea  water,  which  is  nauseous  to  taste,  and  of  perceptible 
smell,  be  the  constituent  condition  of  the  fluid  we  call  water,  then  rain 
water,  which  is  without  smell  and  taste,  is  salt  water  distilled  by  the 
atmosphere;    and  this  is   the   common  quality   of  rain,  river,  and 
spring  water,  except  where  accidental  varieties  of  this  last  occur, 
distinguished  by  the  physical  qualities  of  taste,  odour,  colour,  and 
temperature. 

563.  Two  constructions  in  the  physical  constitution  of  the  earth 
contribute  to  originate  springs,  which  from  the  same  circumstances 
never  cease  to  flow :    one  is  the  adaptation  of  the  atmosphere  to 
transport  water  from  the  sea  to  high  lands ;    and  the  other  is,  the 
porous  beds  of  sand,  and  stone,  and  clay,  which  exert  a  capillary 
influence  in  conveying  the  fluids  they  may  be  charged  with  from  one 
elevation   to   another.     Those  beds  or   strata   of  sand   and   stone, 
resemble  sponge,  paper,  or  pipes,  as  conductors  of  fluids  that  are 
heavy  and  incompressible,  as  water;   clay  strata,  which  are  impe- 
netrable by  water,  form  the  great  reservoirs  or  basins  in  which  the 
treasure  of  the  skies  lies  hid.     Dislocations  in  the  general  mass, 
resulting  from  fractures,  intersect  the  strata  and  facilitate  the  dis- 
charge from  the  reservoirs  formed  by  the  clay  stratum. 

564.  The  water-bearing  strata  are  at  various  depths,  from  50  to  500 
feet  below  the  surface,  and  a  sheet  of  impure,  or  mineral  water,  may 
be  perforated  till  the  operation  conducts  to  a  stratum  containing  pure 


IN  SPRINGS  AND  ARTESIAN  WEILS.  441 

water;  for  the  pipe  let  down  into  the  lower  stratum  will  not  allow 
the  impure  water  from  above  to  mix  with  the  pure  ascending  from 
below.  Water  from  two  different  strata  may  thus  be  brought  to  the 
surface  by  one  borehole  of  a  sufficient  size  to  contain  a  double  pipe, 
viz.  a  smaller  pipe  included  within  a  larger  one,  with  an  interval 
between  them  for  the  passage  of  the  water.  The  smaller  pipe  may 
thus  discharge  the  water  of  the  lower,  and  the  larger  pipe  that  of  the 
upper  stratum ;  for  in  either  case  the  fluid  is  but  endeavouring  to 
regain  the  level  at  its  feeding  source  on  the  surface  of  the  earth. 
Fountains  of  this  sort — Artesian  wells — are  very  well  known  on  the 
eastern  coast  of  Lincolnshire  by  the  name  of  blow  wells.  This  district 
is  low,  covered  by  clay  between  the  wolds  of  chalk  near  Louth  and 
the  sea  shore ;  and  by  boring  through  the  clay  to  the  subjacent  chalk, 
a  spring  is  found  that  yields  a  perpetual  jet,  rising  several  feet  above 
the  surface.  But  wells  of  this  kind  are  common  in  many  parts  of  the 
world  ;  in  the  neighbourhood  of  London ;  in  Artois,  Perpignan,  Tours, 
Roussillon,  and  Alsace,  in  France ;  in  some  parts  of  Germany ;  in  the 
duchy  of  Modena ;  in  Holland,  China,  and  North  America. 

565.  But  whence  come  those  vast  issues  of  fresh  water  that  sometimes 
rise  up  in  the  sea,  as  in  the  Mediterranean  near  Genoa,  and  in  the 
Persian  Gulf,  where  the  ascending  volume  is  so  vast  as  to  allow 
mariners  in  the  one  case,  and  divers  in  the  other,  to  water  ships  ? 
Springs  such  as  these  are  the  issues  of  subterranean  rivers,  all  of 
which  consist  of  meteoric  water,  or  that  which  the  atmosphere  had 
transferred  to  itself  from  the  ocean,  distilled  and  discharged  upon  the 
undulating  surface  of  the  earth. 

566.  The  annual  fall  of  rain  between  the  tropics  is  about  ten  feet 
in  depth ;  and  estimating  this  in  other  countries  as  nearly  propor- 
tional to  the  cosine  of  the  latitude,  the  quantity  of  moisture  ex- 
haled in  a  year,  over  the  surface  of  the  globe  of  our  earth,  would 
form  a  sheet  of  water  five  feet  deep ;  therefore  the  number  of  cubic 
feet  of  water  turned  into  vapour,  and  dispersed  through  the  mass  of 
the  atmosphere  every  minute,  would  be  5x10,424,000,000,  or  fifty- 
two  thousand  one  hundred  and  twenty  millions.     But  this  enormous 
mass  Leslie  further  multiplies  by  18,000,  the  mean  height*  of  the 
atmosphere  in  feet,  and  again  by  62  J,  the  weight  in  pounds  avoir- 

*  In  taking  18,000  feet  as  the  mean  height  of  the  atmosphere,  we  have  followed 
Leslie ;  but  the  mean  height  is  27,800  feet  in  round  numbers,  for  air  is  to  water  as 
1§  to  1000;  therefore  we  have  1§  :  1000  : :  34  :  27,818  feet  for  the  height  of  the 
cloud  sustaining  atmosphere  j  that  is  to  say,  there  are  no  clouds  carried  higher  than 
five  miles. 


442  HYDROSTATIC  PRESSURE  EXEMPLIFIED  IN  SPRINGS,  &C. 

dupoise  of  a  cubic,  foot  of  water ;  and  the  final  measure  of  effect  he 
therefore  takes  leave  to  express  by  58,635,000,000  million  Ibs.  and 
equal  to  the  labour  of  80,000,000  millions  of  men.  Now  the  whole 
population  of  the  globe  being  reckoned  at  800  millions,  of  which 
only  half,  or  less  than  that,  is  incapable  of  labour,  it  follows,  that 
the  power  exerted  by  Nature,  in  the  mere  formation  of  clouds,  to 
produce  rain  and  make  rivers  and  springs,  exceeds  by  two  hundred 
thousand  times  the  whole  accumulated  toil  of  mortals,  who,  if  all 
employed  in  carrying  the  water  of  the  ocean  to  the  mountain  tops, 
for  streams,  and  watering  the  fields,  meadows,  and  woods,  could  not 
rival  Nature  in  her  simple  process  of  evaporation,  absorption,  and 
distribution. 

567.  Such  is  the  enormous  power  exerted  in  the  great  laboratory 
of  Nature  above  the  earth.     Let  us  now  contemplate  her  exertions 
beneath  its  crust,  in  the  grand  hydraulic  apparatus  of  permeable 
strata — the  casual  introduction  of  faults  and  dislocations  in  imper- 
vious  strata,  causing  natural  vents  of  water — the   interposition  of 
syphons,  cavities,  thermal  springs,  mineral  waters — all  resulting  from 
the  sea  co-operating  with  the  atmosphere  to  irrigate,  to  fertilize,  to 
bless  the  habitable  earth. 

568.  The  surface  of  our  own  island  contains  67,243  square  miles, 
which  are  watered  annually  by  a  pool  of  water  about  36  inches  deep, 
of  which,  if  one-sixth  flow  to  the  sea,  there  is  still  2|  feet  depth  left 
to  fertilize  the  land,  to  feed  the  permeable  strata,  and  afford  to  each 
individual  the  most  abundant  supply  of  this  inestimable  blessing. 

569.  If  a  vertical  section  of  Hertfordshire,  Middlesex,  Kent,  and 
Surrey,  be  taken,  we  shall  have  a  pretty  fair  type  of  the  sources  of 
Artesian  or  any  other  wells.     Below  the  London  clay  we  have  plastic 
clay,  then  chalk,  then  fire-stone,  then   gault  clay,  and  below  that 
woborn  sand.    It  is  sufficient  to  bore  through  the  tenderest  plastic  clay 
into  the  chalk,  to  obtain  the  finest  fresh  water  in  the  world.     Kent  and 
Surrey   abound  in  chalk,   which  dips  deeply  below  the  plastic  clay 
stratum,  and  makes  its  appearance  at  St.  Alban's  and  Dunstable. 
The  woborn  sand  met  with  at  Sevenoaks  sinks  below  the  fire-stone 
and  gault  clay,  and  re-appears  at  Leighton  Buzzard.     Any  one  may 
for  himself  sketch  a  perpendicular  section  of  these  districts,  and  a 
few  perpendiculars  let  fall  through  the  London  clay,  to  penetrate 
into  the  chalk,   by  passing  entirely  through  the  plastic  clay,  will 
exhibit  the  exact  position  of  the  borer  in  searching  for  water ;  or  the 
reader  will  find  it  done  to  his  hand  in  Dr.  Buckland's  Geology. 


CHAPTER  XVI. 

MISCELLANEOUS     HYDROSTATIC     QUESTIONS,     WITH     THEIR 
SOLUTIONS. 

570.  QUESTION  1.  How  deep  will  a  cube  of  oak  sink  in  fresh  water, 
each  side  of  the  cube  being  15  inches,  and  its  specific  gravity  0.925, 
that  of  the  water  in  which  it  is  immersed  being  expressed  by  unity  ? 

The  solution  of  this  question  is  extremely  simple,  for  by  art.  311, 
page  257,  it  is  announced  as  an  established  hydrostatical  principle, 
that  the  magnitude  of  the  whole  body  is  to  the  magnitude  of  the 
immersed  part,  as  the  specific  gravity  of  the  fluid  is  to  the  specific 
gravity  of  the  solid.  But  since  the  base  of  the  whole  solid  and  that 
of  the  immersed  portion  are  the  same,  it  follows  from  the  principles  of 
mensuration,  that  the  magnitudes  are  as  the  altitudes,  and  conse- 
quently, the  altitudes  are  as  the  specific  gravities ;  hence  we  have 

1  :  0.925  : :  15  :  13.875  inches,  the  depth  required. 

571.  QUESTION  2.  If  a  cube  of  wood  floating  in  fresh  water,  have 
three  inches  of  it  dry,  or  standing  above  the  surface  of  the  fluid,  and 
3J-I-3  inches  dry  when  in  sea  water;  it  is  required  to  determine  the 
magnitude  of  the  cube,  and  what  sort  of  wood  it  is  made  of? 

This  question  may  be  resolved  on  the  same  principles  as  the  last ; 
for  if  we  put  a;  ~  the  side  of  the  cube  in  inches,  and  s  the  specific 
gravity  of  the  wood  ;  then,  by  art.  311,  page  257,  we  have 

1000  :  s  :  :  x  :  TT™»  the  part  immersed  in  fresh  water, 

and  1026  :  s  :  :  x  :    SX  0  the  part  immersed  in  sea  water; 

but  the  part  immersed  and  the  part  extant,  together  make  up  the 
whole  altitude  or  side  of  the  cube ;  hence  we  have 


444  MISCELLANEOUS  HYDROSTATIC  QUESTIONS,  WITH  THEIR  SOLUTIONS. 

-f-  3  :rz  x,  in  the  case  of  fresh  water, 


1000 

O    -V* 

and  -         +  3fH  =  *  in  the  case  of  sea 


therefore,  if  one  of  these  equations  be  subtracted  from  the  other,  we 
shall  have 

533520 

26xzz      '        ,  or  arm 40  inches,  the  side  of  the  cube  required ; 
51  o 

hence,  the  altitude  of  the  immersed  part,  as  referred  to  fresh  water, 
is  40  —  3  =  37  inches ;  and  the  altitude  as  referred  to  sea  water,  is 
36 yVy  inches ;  and  from  either  of  these,  the  specific  gravity  of  the  wood 
is  found  by  the  proposition  referred  to  above ;  for  we  have 

40  :  37  :  :  1000  :  siz:925;  indicating  the  specific  gravity 
of  oak,  when  that  of  fresh  water  is  expressed  by  1000. 

572.  QUESTION  3.  If  a  cube  of  wood  floating  in  sea  water  be  |  below 
the  plane  of  floatation,  and  it  sinks  VV  of  an  inch  deeper  in  fresh 
water ;  what  is  its  magnitude,  and  what  is  its  specific  gravity  ? 

This  question  at  first  sight  would  appear  to  be  the  same  as  the  last ; 
it  may  indeed  be  resolved  by  the  same  principles  ;  but  since  the  im- 
mersed parts  are  given  in  this  instance,  instead  of  the  extant  parts,  as 
was  the  case  in  the  preceding  question,  this  circumstance  suggests  a 
simpler  and  a  better  mode  of  solution ;  for  by  the  inference  in  art.  317, 
page  261,  it  appears  that  the  parts  immersed  below  the  surface  of  the 
different  fluids,  are  to  each  other  inversely  as  the  specific  gravities  of 
the  fluids ;  hence,  if  x  denote  the  side  of  the  cube  in  inches,  then  by 

the  question,  —  is  the  altitude  of  the  part  immersed  below  the  sur- 

3ar        3        3Qx  -4-  12 
face  of  sea  water,  and  — —  4-  TZ  ~ IT; is  the  altitude  of  the 

4     '    10  40 

part  immersed  below  the  surface  of  fresh  water ;  consequently,  by  the 
inference  above  cited,  we  obtain 

1000  :  1026  :  :  —  :  30*  ."*" 12  ; 
4  40 

and  from  this,  by  equating  the  products  of  the  extreme  and  mean 
terms,  we  get 

78x1=  1200,  or  arzr  15TST  inches,  the  side  of  the  cube  required. 

Having  thus  determined  the  side  of  the  cube,  the  specific  gravity 
of  the  material  will  be  found  as  in  the  last  question,  for  we  have 

15TT  :  IX 15TT  : :  1026  :  769,  the  specific  gravity  sought. 


VHlVERSiTY 


MISCELLANEOUS  HYDROSTATIC  QUESTIONS,  WITH  THEIR  SOLUTIONS.  445 

Mr.  Dalby  makes  the  side  of  the  cube  equal  to  13|  inches,  and  the 
specific  gravity  772  ;  but  this  only  shows  that  he  has  employed  a 
higher  number  for  the  specific  gravity  of  sea  water;  1030  brings  out 
his  results. 

573.  QUESTION  4.  How  deep  will  a  globe  of  oak  sink  in  fresh 
water,  the  diameter  being  12  inches  and  the  specific  gravity  925,  that 
of  water  being  1000? 

By  the  rules  for  the  mensuration  of  solids,  the  solidity  of  the  globe 
is  expressed  by  the  cube  of  its  diameter  multiplied  by  the  decimal 
.5236  ;  consequently,  we  have  1728  X  .5236  =  904.7808  cubic  inches 
for  the  solidity  of  the  globe;  therefore,  according  to  art.  311,  page 
257,  we  get 

1000  :  925  :  :  904.7808  :  836.923  cubic  inches,  the  solidity  of  the 
immersed  segment.  Now,  according  to  the  principles  of  mensuration, 
as  applied  to  the  segment  of  a  sphere,  if  x  be  put  to  denote  the  height 
of  the  segment,  then  its  solidity  is  expressed  by  .5236  (36ar  —  2zs), 
and  this  must  be  equal  to  the  solidity  of  the  segment  found  by  the 
above  analogy  ;  hence  we  get 

1  So;2—  x3=  799.2. 

In  order  to  reduce  this  equation,  let  the  signs  of  all  the  terms  be 
changed,  and  put  a;zr  z  -f-  6  ;  then,  by  substitution,  we  have 

xa=  zs  -f  18z2+  108z  +  216, 
and—  18**=:*—  18s2—  216z  —  648; 
hence,  by  summation,  we  obtain 

38—108zr=  —  367.2, 

and  from  this  equation,  the  value  of  z  is  found  to  be  3.9867  very 
nearly  ;  but  by  the  supposition,  x  —  z  -f-  6,  and  consequently,  it  is 

x  •=.  3.9867  4-  6  =  9.9867  inches  very  nearly,  for 
the  height  of  the  segment,  or  the  depth  to  which  a  globe  of  oak 
descends  in  fresh  water,  the  diameter  being  12  inches,  and  the  specific 
gravity  925.  This  result  agrees  with  that  obtained  by  Dr.  Hutton,  in 
the  second  volume  of  his  Course  of  Mathematics. 

574.  QUESTION  5.  If  a  sphere  of  wood  9  inches  in  diameter,  sinks 
by  means  of  its  own  gravity,  to  the  depth  of  6  inches  in  fresh  water  ; 
what  is  its  weight,  and  also  its  specific  gravity  ? 

By  the  corollary  to  the  third  proposition,  art.  233,  page  212  —  214, 
it  is  manifest,  that  the  weight  of  the  body  is  the  same  as  the  weight 
of  the  fluid  displaced  by  its  immersion  ;  that  is,  the  weight  of  the 
entire  sphere,  is  equal  to  the  weight  of  as  much  fluid  as  is  represented 
by  the  solidity  of  the  immersed  segment  ;  but  by  the  principles  of 


446  MISCELLANEOUS  HYDROSTATIC  QUESTIONS,  WITH  THEIR  SOLUTIONS. 

mensuration,  the  solidity  of  the  segment  is  (9x3  —  12  X  2)  x  36  x 
.5236  zn  282.744  cubic  inches  ;  consequently,  the  whole  weight  of  the 
body,  is  282.744X0.03617  —  10.226  Ibs.,  the  decimal  fraction  0.3617 
being  the  number  of  Ibs.  in  a  cubic  inch  of  fresh  water.  (See  note  to 
art.  329,  page  268.) 

Having  thus  determined  the  weight  of  the  globe,  the  specific  gravity 
of  the  material  may  be  found  in  various  ways  ;  but  we  shall  here 
determine  it  by  the  principle  of  Proposition  VII.  art.  311,  page  257; 
from  which  we  have  the  following  process,  viz. 

98  :  (27  —  12)X36  :  :  1000  :  740|f,  the  specific  gravity  sought. 

575.  QUESTION  6.  An  irregular  piece  of  lead  ore,  weighs  in  air  12 
ounces,  but  in  water  only  7  ;  and  another  piece  of  the  same  material, 
•weighs  in  air  14|  ounces,  but  in  water  only  9  :  it  is  required  to 
compare  their  densities  or  specific  gravities  ? 

This  question  may  be  very  simply  resolved,  by  the  principle  stated 
in  Proposition  V.  art.  264,  page  229  ;  which  is  the  same  as  the 
principle  employed  by  Dr.  Hutton  for  the  same  purpose;  from  it 
we  have  • 

12  —  7  :  12  :  :  1000  :  2400,  the  specific  gravity  of  the 
lightest  fragment  ;  and  again,  we  have 

14.5  —  9  :  14.5  :  :  1000  :  2636.36,  the  specific  gravity 
of  the  heavier  piece. 

The  specific  gravities  are  therefore  to  one  another,  as  the  numbers 
2400  and  2636.36.  Dr.  Hutton  makes  the  ratio  as  145  to  132.  (See 
question  52,  page  298,  vol.  ii.  10th  ed.  Course,  1831  ;)  his  formulae 
will  be  found  in  arts.  250  or  251. 

The  above  solution  however,  is  not  correct,  for  the  weight  of  the 
body  in  air  is  not  its  real  weight,  as  it  would  be  exhibited  in  vacuo  ; 
the  correct  specific  gravity  will  therefore  be  obtained  by  equation 
(186),  art.  270,  page  234;  and  the  operation  is  as  follows,  the 
specific  gravity  of  air  being  1|-,  that  of  water  being  1000. 


of  the  lighter  fragment  ;   and  for  the  specific  gravity  of  the  heavier, 
we  have 

14.5X1000-9X1*  .d 

14.5  _  9  34-J6- 

By  the  results  of  our  solution,  it  appears  that  the  heavier  fragment 
is  also  the  densest;  by  Dr.  Hutton's  solution,  exactly  the  reverse  is 
the  case. 


MISCELLANEOUS  HYDROSTATIC  QUESTIONS,  WITH  THEIR  SOLUTIONS.  447 

576.  QUESTION  7.  An  irregular  fragment  of  glass,  weighs  in  air  171 
grains,  but  in  water  it  weighs  only  120  grains;  what  is  its  real 
weight  ;  that  is,  what  would  it  weigh  in  vacuo  ? 

The  answer  to  this  question  is  obtained  by  equation  (185),  art.  267, 
page  232  ;  and  the  operation  as  there  indicated  is  simply  as  follows. 
171X1000  —  120XH       lry 


the  real  weight  of  the  glass  in  vacuo. 

577.  QUESTION  8.  A  fragment  of  magnet  weighs  102  grains  in  air, 
and  in  water  it  weighs  only  79  grains  ;  what  is  its  real  weight,  or  what 
does  it  weigh  in  vacuo  ? 

The  solution  of  this  question  is  effected  exactly  in  the  same  manner 
as  the  preceding,  the  conditions  from  which  the  data  are  obtained 
being  precisely  the  same;  that  is,  the  body  is  weighed  in  air  and 
in  water  ;  consequently,  the  operation  is  as  under. 

102X1000  —  79X1|- 
'• 


the  real  weight  of  the  magnet  in  vacuo. 

From  the  real  weights  of  these  materials,  as  determined  in  the  above 
examples,  the  absolute  specific  gravities  can  be  found  by  the  principle 
of  Proposition  V.  page  229 ;  for  the  weights  lost,  are  to  the  whole 
weights,  as  the  specific  gravity  of  water,  is  to  the  specific  gravities  of 
the  substances  in  question ;  hence  we  have 

— 120  :  171-gVVs-  :  :  1000  :  3350^,    the    specific  gravity 
of  the  glass. 

—  79  :  102¥y¥V  :  :  100°  :  443°Hf»  the  specific  gravity  of 
the  magnet. 

Dr.  Hutton  makes  the  specific  gravities  of  the  glass  and  the 
magnet,  respectively  equal  to  3933  and  5202,  and  says  that  the 
ratio  is  very  nearly  as  10  to  13;  our  own  numbers  give  the  same 
ratio. 

578.  QUESTION  9.  Taking  the  specific  gravity  of  glass  equal  to  3350, 
suppose  that  a  globe  is  found  to  weigh  10  Ibs.  avoirdupoise ;  what  is 
its  diameter  ? 

The  cubic  inch  of  glass  of  the  given  specific  gravity  weighs 
0.1211695  of  alb.;  therefore,  according  to  the  equation  187,  page 
235,  we  have 

/   Z-T 

in  5.402  inches  nearly. 


.5236X0.1211695 


448  MISCELLANEOUS  HYDROSTATIC  QUESTIONS,  WITH  THEIIl  SOLUTIONS. 

579.  QUESTION  10.  Supposing  the  same  piece  of  glass  to  weigh  9.996 
Ibs.  in  air,  but  in  water  only  7.015  Ibs. ;  what  is  its  diameter? 

The  specific  gravity  of  water,  when  reduced  to  pounds  per  cubic 
inch,  is  0.03617,  and  that  of  air  is  0.000045  ;  therefore,  by  equation 
(188),  page  236,  we  have 


9.996  —  7.015  .—  5.402  inches,  the  same  as  before. 


.5236  (0.03617  —  0.000045) 

580.  QUESTION  1 1 .  The  lock  of  a  canal  is  40  feet  wide,  and  the  lock 
gates  being  rectangular  planes,  stand  16  feet  above  the  sill,  with  their 
upper  edges  on  a  level  with  the  surface  of  the  water ;  now,  supposing 
that  the  gates  are  found  to  meet  each  other  in  an  angle  of  141°  35'; 
what  is  the  amount  of  pressure  which  they  sustain,  a  cubic  foot  of 
water  weighing  62 \  Ibs.  avoirdu poise  ? 

Here  the  lock  gates  meet  each  other  in  an  angle  of  141°  35' ;  which, 
according  to  Barlow,  is  the  situation  in  which,  with  a  given  section 
of  timber,  they  obtain  the  greatest  strength.  But  by  the  principles 
of  Plane  Trigonometry,  the  length  of  each  gate  is 

20Xsec.l9°  25'=21.206  feet; 

and  by  the  question,  the  depth  is  16  feet ;  therefore,  the  whole  surface 
exposed  to  the  pressure  of  the  water,  is  2 1. 206  X  32  =  678. 592  square 
feet.  Now  the  centre  of  gravity  of  each  gate  is  8  feet  below  the 
surface  of  the  water,  the  specific  gravity  of  which  is  unity ;  conse- 
quently, by  equation  (8),  page  19,  the  entire  pressure  upon  the  gates,  is 

p=:  8X678.592  —  5428.736  cubic  feet  of  water; 

or  when  reduced  to  Ibs.  it  is 
5428.736X62J  z=  339296  Ibs.,  or  151  tons  9  cwt.  0  qrs.  48  Ibs. 

581.  QUESTION  12.   If  the  diameter  of  a  cylindrical  vessel  be  20 
inches;  required  its  depth,  so  that  when  filled  with  a  fluid,  the  pres- 
sure on  the  bottom  and  sides  may  be  equal  to  each  other  ? 

This  question  is  resolved  by  the  equations  (59  and  60),  page  100, 
where  it  is  manifest,  from  the  construction  of  the  equations,  that 
Bzz:  \d,  and  consequently,  by  substitution,  we  obtain 

.7854D2dzz  3.1416Ddx  Jd,  and  this  expression  is  equivalent  to 

.7854D2d—  1.5708D<f, 

and  by  casting  out  the  common  factors,  we  have  2</~D,  or  by 
division,  c?m  10  inches ;  hence  when  the  depth  of  the  vessel  is  equal 
to  half  the  diameter  of  the  base,  the  concave  surface  and  the  bottom 
of  the  vessel  sustain  equal  pressures. 

NOTES. 


NOTES. 


NOTE  A.— CHAPTER  I. 

ARTICLE  3.  Every  particle  of  a  non-elastic  fluid  presses  equally  in  every 
direction. 

The  truth  of  the  principle  enunciated  in  this  proposition,  is  abundantly  illus- 
trated by  the  experiments  introduced  at  the  end  of  the  sixth  chapter,  and  conse- 
quently, it  needs  no  further  confirmation  here ;  but  from  it  we  may  infer,  that 

The  lateral  pressure  of  a  fluid  is  equal  to  its  perpendicular  pressure, 

Art.  4.  Every  particle  of  fluid  in  a  state  of  quiescence,  is  pressed  equally  in  all 
directions. 

This  is  obvious ;  for  if  possible,  let  any  particle  receive  a  greater  pressure  in  one 
direction  than  in  another ;  then,  since  by  art.  2,  the  particles  of  a  fluid  yield  to  the 
smallest  force  or  pressure,  and  are  easily  moveable  amongst  themselves,  it  follows, 
that  motion  will  take  place  in  that  direction  in  which  the  pressure  is  greatest ;  but 
by  the  proposition,  the  fluid  is  in  a  state  of  quiescence ;  that  is,  there  is  no  motion 
taking  place  among  its  particles ;  they  are  therefore  equally  pressed  in  all  direc- 
tions. 

Art.  5.  When  a  fluid  is  in  a  state  of  rest,  the  pressure  exerted  against  the 
surface  of  the  vessel  which  contains  it,  is  perpendicular  to  that  surface. 

This  also  is  manifest ;  for  if  the  pressure  be  not  perpendicular  to  the  containing 
surface,  the  re-action  of  that  surface  cannot  destroy  it ;  let  the  pressure  therefore 
be  resolved  into  two,  the  one  perpendicular  and  the  other  parallel  to  the  surface  ; 
then  it  is  manifest,  that  the  former  will  be  destroyed  by  the  re-action,  and  the 
latter  continuing  to  act  on  the  particles  of  the  fluid,  will  be  transmitted  in  every 
direction,  and  consequently,  motion  will  take  place ;  but  this  is  contrary  to  the 
supposition,  for  the  fluid  is  stated  to  be  at  rest ;  therefore,  the  pressure  must  be 
perpendicular  to  the  surface. 

Art.  6.  When  a  mass  of  fluid  is  in  a  state  of  rest,  its 
surface  is  horizontal,  or  perpendicular  to  the  direction  of 
gravity. 

For  let  A  B  D  c  represent  a  vessel  of  fluid,  such  as  water, 
and  conceive  the  right  line  A  B  to  be  parallel  to  the  hori- 
zon. Suppose  the  surface  of  the  fluid  to  be  in  the  position 
PB,  any  how  inclined  to  the  horizontal  line  AB;  then, 
since  by  art.  2,  the  particles  of  the  fluid  are  easily 
moveable  among  themselves,  it  follows,  that  the  higher 
VOL.  i.  2  G 


450 


NOTES. 


P  Q 


F 


r> 


particles  at  E,  will,  in  consequence  of  their  gravity,  continually  descend  towards 
the  lower  parts  at  F. 

Again,  the  greater  pressure  which  obtains  among  the  particles  under  E,  and  the 
lesser  under  F,  will  obviously  cause  the  particles  at  E  to  descend,  and  those  at  F  to 
ascend ;  and  thus  the  higher  parts  of  the  fluid  at  E,  descending  and  spreading 
themselves  over  the  lower  parts  at  F,  which  at  the  same  time  are  ascending ;  it  is 
obvious,  that  the  surface  will  at  last  be  reduced  to  the  horizontal  position  A  B  ;  and 
having  attained  that  position,  it  must  continually  remain  in  it,  for  then  there  is  no 
part  higher  than  another,  and  consequently,  there  is  no  tendency  to  descend  in 
one  part  more  than  in  another,  and  therefore  the  fluid  must  rest  in  a  horizontal 
position. 

Art.  7.  If  two  fluids  that  do  not  mix,  are  poured  into  the  same  vessel,  and  suf- 
fered to  subside,  their  common  surface  is  parallel  to  the  horizon. 

Let  A  B  D  c  be  the  vessel  containing  the  two  fluids  which  do  not  mix,  and  let  E  F 
denote  the  common  surface,  or  that  in  which 
the  fluids  come  in  contact.  A  a     L  ft 

The  upper  surface  A  B  of  the  lighter  fluid  is 
horizontal  by  art.  6;  therefore,  let  P  and  Q 
be  two  contiguous  particles  of  the  heavier  •] 
fluid,  equally  distant  from  a  horizontal  plane, 
and  consequently,  equally  distant  from  A  B  ;  if 
they  are  not  also  equally  distant  from  E  F  the 
common  surface,  the  vertical  pressures  upon 
them  will  be  unequal,  for  this  pressure  is  made 

up  of  the  weights  of  two  columns,  containing  different  quantities  of  fluid  matter, 
viz.  PC,  qd  of  the  heavier  fluid,  and  ca,  db  of  the  lighter;  consequently,  the 
pressures  in  opposite  directions  will  be  unequal,  and  motion  must  take  place,  which 
is  contrary  to  the  supposition. 

The  particles  P  and  Q  are  therefore  equally  distant  from  E  F  the  common  surface 
of  the  fluids;  and  the  same  being  true  for  every  other  two  contiguous  particles  in 
the  same  horizontal  plane,  it  follows,  that  E  F  must  also  be  horizontal. 

Art.  8.  The  particles  of  fluid  situated  at  the  same  perpendicular  depth  below  the 
surface,  are  equally  pressed. 

This  is  almost  self-evident,  but  nevertheless  it  may  be  thus  demonstrated ;  for 
let  the  plane  passing  through  E  F,  be  parallel  to  the  surface 
AB;  then,  since  the  height  of  the  fluid  is  the  same  at  all  the 
points  of  E  F,  it  is  manifest  that  the  weights  of  the  fluid 
columns  standing  upon  any  equal  parts  of  it,  must  also  be 
equal,  and  consequently,  the  pressure  on  all  the  points  of  the 
plane  passing  through  EF  is  the  same,  since  they  are  all 
situated  at  equal  depths  below  the  surface  AB. 

Art.  9.  When  a  fluid  is  in  a  state  of  rest,  the  pressure  upon 
any  of  its  constituent  elements,  wheresoever  situated,  varies 
as  the  perpendicular  depth  of  the  particle  or  element  pressed. 
The  demonstration  of  this  principle  is  evident  from  that  to  article  8;   for  the 
pressure  depends  upon  the  weight  of  the  superincumbent  column,  and  the  weight 
of  this  column  manifestly  varies  directly  as  its  height ;  hence,  the  pressure  upon 
any  particle  varies  as  its  perpendicular  depth  below  the  surface  of  the  fluid. 


NOTES.  451 


NOTE  B.— PROPOSITION  I.— CHAPTER  I. 

In  this  proposition,  and  the  several  laws  and  consequences  deduced  from  it,  the 
effect  of  the  atmospheric  pressure  is  entirely  disregarded.  It  may  however  be 
proper  to  remark,  that  in  numerous  delicate  hydrostatical  inquiries,  the  pressure 
thus  excited  must  be  taken  into  the  account:  it  is  equal  to  the  pressure  of  a 
column  of  water  34  feet  in  perpendicular  height, 

NOTE  C.— CHAPTER  VI. 

Experiment  7.  Page  160. — Since  these  experiments  were  selected  and  inserted 
in  this  work,  a  living  eel  has  been  killed  in  the  cylinder  of  the  hydrostatic  press, 
in  which  also  an  egg  has  been  broken.  But  the  eel  would  have  been  killed  by 
suffocation,  if  no  pressure  had  been  applied  to  the  fluid,  and  the  fracture  of 
the  egg  was  due  to  the  air  it  contained  between  the  white  and  the  shell,  or 
to  the  different  densities  of  the  shell,  the  white,  the  yolk,  and  the  water. 
Thus  we  can  easily  conceive,  as  Mr.  Tredgold  remarks,  that  the  trial  of  an 
experiment  may  be  the  means  of  condemning  a  very  useful  principle,  merely 
through  inattention  to  the  proportions  and  the  mode  of  action.  We  may  still 
affirm,  that  fishes  will  endure  a  very  high  degree  of  fluid  pressure,  provided  they 
be  allowed  to  breathe ;  indeed  it  is  recorded,  that  a  whale  in  the  arctic  seas,  being 
struck  by  a  harpoon,  descended  perpendicularly  by  the  line  about  900  fathoms, 
before  it  returned  to  the  surface  to  respire ;  it  was  then  under  a  pressure  of  nearly 
164  atmospheres,  or  2,460  Ibs.  upon  a  square  inch  of  its  surface;  now  if  the  living 
animal  could  sustain  this  natural  pressure  without  inconvenience,  we  are  at  liberty 
to  conclude  that  it  could  sustain  an  equal  degree  of  artificial  pressure.  It  is  manifest, 
that  fishes  which  do  not  come  to  the  surface,  breathe  the  air  with  which  the  water 
is  impregnated,  at  whatever  depth  they  may  be  found.  Moreover,  if  an  eel  were 
killed  by  pressure,  we  suppose  it  would  be  crushed,  or  burst  asunder.  In  short,  we 
require  evidence  of  the  death  by  pressure,  to  remove  our  belief  in  death  by  suffo- 
cation. Air,  which  is  invisible,  by  squeezing  the  heat  out  of  it  by  strong  pressure, 
may  be  compressed  into  water ;  but  the  contraction  which  water  suffers  at  every 
increase  of  pressure,  exceeds  not  the  twenty  thousandth  part  of  what  air  would 
undergo  in  like  circumstances ;  and  fishes  are  at  their  ease  in  a  depth  of  water, 
where  the  pressure  around  will  instantly  break  or  burst  inwards  almost  the  strongest 
empty  vessel  that  can  be  let  down. 

We  are  perfectly  aware  of  the  experiments  of  Mr.  Canton,  in  1760,  which 
established  incontestably  the  compression  of  water.  Indeed  the  theory  of  com- 
pression extends  to  all  bodies :  Dr.  Young  says  that  steel  would  be  compressed 
into  one-fourth,  and  stone  into  one-eighth  of  its  bulk  at  the  earth's  centre ;  but  a 
density  so  extreme  is  not  borne  out  by  astronomical  observation.  And  the  late  Sir 
John  Leslie,  who  suggests  the  idea  that  the  ocean  may  rest  upon  a  subaqueous 
bed  of  compressed  air,*  says  that  water  at  the  depth  of  93  miles  would  be  com- 
pressed into  half  its  bulk  at  the  surface  of  the  earth ;  and  at  the  depth  of  362.5 
miles  it  would  acquire  the  ordinary  density  of  quicksilver.t  Practical  men,  m 
reply  to  all  this  physical  science,  may  justly  reply,  "  We  are  seldom  called  upon 


*  Article  "  Meteorology,"  in  the  Supplement  to  the  Encyclopaedia  Britannica. 
t  See  Leslie's  Elements  of  Natural  Philosophy,  vol.  i. 

2G  2 


452  NOTES. 

to  execute  undertakings  much  below  the  level  of  low  water,  and  those  investigations 
suit  us  best,  which  are  confined  to  the  depth  of  a  few  fathoms,  where  we  know  that 
water  is,  to  all  intents  and  purposes  in  our  business,  wholly  unaltered  by  com- 
pression." 

NOTE  D.— CHAPTER  X. 

The  principle  of  fluid  support,  and  the  doctrine  of  specific  gravity,  which  we  are 
now  considering,  explain  many  curious  facts  that  daily  pass  unobserved.  Thus,  a 
stone  which  two  men  on  land  can  h  ardly  lift,  may  be  borne  along  by  one  man  in  water ; 
and  in  diving,  a  dog  will  bring  to  the  surface  a  human  body,  which  the  strongest  of 
his  species  could  not  lift  on  land :  hence  the  ease  also  with  which  a  bucket  is  lifted 
from  the  bottom  of  a  well  to  the  surface  of  the  water.  And  as  the  human  body  in 
an  ordinary  healthy  state,  with  the  chest  full  of  air,  is  lighter  than  its  equal  bulk 
of  water,  a  man  naturally  floats  with  about  half  the  head  extant ;  "  having," 
as  Dr.  Arnott  says,  "  then  no  more  tendency  to  sink  than  a  log  of  fir."  When  a 
swimmer  floats  on  his  back,  with  merely  his  face  above  water,  in  which  position 
he  can  breathe  freely,  he  exhibits  the  true  position  of  floatation,  in  which  the  human 
body  is  lighter  than  water,  for  its  specific  gravity  is  one  ninth  less  than  that  of  water, 
being  about  0.891.  In  some  cases  however,  the  bodies  of  men  are  heavier:  thus,  a 
person  who  weighs  135  Ibs.  would  be  12  Ibs.  heavier  than  two  cubic  feet  of  river 
water,  and  would  require  a  float  of  cork  equal  to  4  Ibs.  to  keep  him  from  sinking ; 
for  123  +  4#— 135  -\-x,  where  x  represents  the  weight  of  cork;  consequently, 
123  +  3a;=135;  therefore,  3x= 135— 123=12;  whence  x =4=. 

When  a  solid  specifically  heavier  than  a  fluid,  is  immersed  to  a  depth  which  is  to 
its  thickness,  as  the  specific  gravity  of  the  solid  to  that  of  the  fluid,  and  the  pressure 
of  the  fluid  from  above  is  removed,  the  body  will  be  sustained  in  the  fluid ;  for  the 
pressure  from  above  being  removed,  the  body  is  in  the  same  state  with  respect  to 
the  contrary  pressure,  as  if  the  same  weight  filled  the  whole  space  to  the  surface  of 
the  fluid;  which  means,  as  if  its  specific  gravity  and  that  of  the  fluid  icere  equal. 

The  principle  here  enunciated  helps  the  philosophers  in  their  explanation  of  the 
common  experiment  of  making  lead  to  swim,  in  consequence  of  being  fitted  to  the 
bottom  of  a  glass  tube. 

In  the  case  cited  above,  of  solid  bodies  being  lighter  in  water  than  in  air— that 
is  to  say,  being  more  easily  moved  in  the  water  than  on  dry  land — the  meaning  of 
the  proposition  is,  that  all  bodies,  when  immersed  in  a  fluid,  lose  the  weight  of  an 
equal  volume  of  that  fluid.  Thus,  in  raising  a  bucket  of  water  from  the  bottom  of 
a  well,  so  long  as  the  bucket  is  under  the  water,  we  do  not  perceive  it  to  have  any 
additional  weight  beyond  the  wood  it  is  made  of;  but  the  moment  we  raise  the 
bucket  to  the  surface,  and  suspend  it  in  air,  then  we  feel  the  additional  weight  of 
the  water,  which  if  equal  to  6£  gallons,  or  to  one  cubic  foot,  will  add  nearly  62£ 
pounds,  or  1000  ounces  avoirdupois  weight  to  the  bucket.  Now  all  this  weight 
existed  in  the  bucket  when  under  the  surface  of  the  water,  being  supported  by  an 
equal  bulk,  or  62J  pounds.  The  weights  thus  gained  or  lost  by  immersing  the 
same  body  in  different  fluids,  are  as  the  specific  gravities  of  the  fluids ;  hence 
we  affirm  that  all  bodies  of  equal  weight,  but  of  different  volume,  lose  in  the 
same  fluid,  weights  which  are  reciprocally  as  the  specific  gravities  of  the  bodies, 
or  directly  as  their  volumes.  In  the  salt  sea  it  will  be  one  thirty-fifth  lighter  than 
i  n  fresh  water. 


NOTES.  453 


NOTE  E.— CHAPTER  XII. 

It  will  not,  however,  be  out  of  place  to  remark,  that  the  weight  of  the  whole 
solid,  and  that  of  the  portion  immersed  below  the  plane  of  floatation— which 
corresponds  to  the  magnitude  of  the  fluid  displaced — are  very  appropriately 
represented  by  the  areas  drawn  into  the  respective  specific  gravities  of  the  solid 
and  the  fluid  on  which  it  floats.  But  the  most  cursory  observation  shows,  that  a 
solid  may  be  immersed  in  a  fluid  in  numberless  different  ways,  so  that  the  part 
immersed,  shall  be  to  the  whole  magnitude  in  the  given  proportion  of  the  specific 
gravities,  and  yet  the  solid  shall  not  rest  permanently  in  any  of  these  positions.  The 
reason  is  obvious  :  the  floating  body  is  forced  down  by  its  own  weight,  and  borne 
up  by  the  pressure  of  the  fluid ;  it  descends  in  the  direction  of  a  vertical  line 
passing  through  its  centre  of  gravity ;  it  is  pushed  up  in  the  direction  of  a  vertical 
line  passing  through  the  centre  of  gravity  of  the  part  immersed,  or  the  displaced 
fluid.  Unless  therefore,  these  two  lines  are  coincident,  or  that  the  two  centres  of 
gravity  shall  be  in  the  same  vertical  line,  it  is  evident  that  the  solid  thus  impelled, 
must  revolve  on  an  axis  until  it  finds  a  position  in  which  the  equilibrium  of  floating 
will  be  permanent. 

To  ascertain  therefore,  the  positions  in  which  the  solid  floats  permanently,  we  must 
have  given  the  specific  gravity  of  the  floating  body,  in  order  to  fix  the  proportion  of 
the  part  immersed  to  the  whole;  and  then,  by  geometrical  or  analytical  methods, 
determine  in  what  positions  the  solid  can  be  placed  on  the  surface  of  the  fluid,  so 
that  the  centre  of  gravity  of  the  floating  body,  and  that  of  the  part  immersed  may 
be  situated  in  the  same  vertical  line,  while  a  given  proportion  of  the  whole  volume 
is  immersed  beneath  the  surface  of  the  fluid. 

The  incumbent  weight  may  be  considered  as  collected  in  the  centre  of  gravity  of 
the  floating  body,  and  the  sustaining  efforts  as  united  in  the  centre  of  buoyancy, 
which,  as  we  have  already  said,  is  the  same  as  the  centre  of  gravity  of  the  water 
displaced,  or  of  the  immersed  portion  of  the  uniform  solid.  To  these  two  points 
therefore,  the  antagonist  forces  are  directed,  and  the  line  which  joins  them,  called 
the  line  of  support,  will  have  constantly  a  vertical  position  in  the  case  of  equilibrium. 

The  centre  of  gravity  of  the  whole  mass,  about  which  it  turns  in  the  water,  must 
evidently  continue  invariable ;  *  but  the  centre  of  buoyancy  will  change  its  relative 
place,  according  to  the  situation  of  the  immersed  portion  of  the  solid.  If  these 
two  centres  should  coincide,  the  body  will  float  indifferently  in  any  position  of 
stability.  It  will  therefore  float,  as  often  as  a  vertical  line,  drawn  from  the  centre 
of  buoyancy,  shall  pass  through  the  centre  of  gravity.  But  this  will  obtain  when- 
ever the  line  of  support  becomes  perpendicular  to  the  horizon.  The  equilibrium 
may,  however,  be  either  permanent  or  instable.  It  is  permanent,  if  on  pulling  the 
body  a  little  aside  it  has  a  tendency  to  redress  itself,  or  to  recover  its  original 
position ;  it  is  instable,  when  the  body,  on  being  slightly  inclined,  tumbles  over  in 
the  liquid  and  assumes  a  new  situation.  These  opposite  conditions  will  occur  in  a 
body  of  irregular  form,  when  the  centre  of  gravity  occupies  the  highest  or  the  lowest 
possible  position,  (when  the  centre  of  gravity  is  the  lowest  possible,  the  situation  is 
that  of  maximum  stability)  for  though  the  volume  of  immersion  remains  the  same, 
the  solid  will  evidently  be  less  or  more  depressed  in  the  fluid  medium,  according 


*  This  is  not  strictly  true,  but  it  causes  no  difference  in  the  theory  that  it  is  otherwise. 


454  NOTES. 

to  the  width  of  its  section  or  water  lines.  We  have  a  curious  proof  of  this  in  the 
construction  of  the  French  ship  of  the  line,  of  74  guns,  called  Le  Scipion,  fitted  for 
sea  at  Rochfort,  in  1779;  but  she  wanted  stability,  which,  after  various  fruitless 
attempts,  was  achieved  by  applying  a  bandage  or  sheathing  of  light  wood  to  the 
exterior  sides  of  the  vessel.  This  cushion,  bandage,  or  sheathing,  was  from  one 
foot  to  four  inches  in  thickness,  extending  throughout  the  whole  length  of  the 
water  line,  and  ten  feet  beneath  that  line.  We  are  left  to  infer  Le  Scipion  then 
floated  with  permanent  stability. 

If  the  centre  of  buoyancy  stand  higher  than  the  centre  of  gravity,  the  floating 
body  will,  in  every  declination  maintain  its  stability,  and  regain  its  perpendicular 
position ;  for  though  made  to  lean  towards  either  side,  the  vertical  pressure  exerted 
against  that  variable  point  will  soon  bring  it  back  again  into  the  line  of  support. 
But  the  elevation  of  the  centre  of  buoyancy  above  that  of  gravity,  is  by  no  means 
an  essential  requisite  to  the  stability  of  floatation;  on  the  contrary,  it  falls  in  most 
cases  considerably  below  the  centre  of  gravity  about  which  the  body  rolls.  The 
buoyant  efforts  may  be  considered  as  acting  upon  any  point  in  the  vertical  line, 
and  consequently,  as  united  in  the  point  where  the  line  crosses  the  axis  of  the 
floating  body.  If  the  point  of  concourse,  thus  assigned,  should  stand  above  the 
centre  of  gravity,  the  body  will  float  firmly,  and  will  right  itself  after  any  small 
derangement.  If  it  coincide  with  the  centre  of  gravity  of  the  homogeneous  body, 
this  will  continue  indifferent  with  regard  to  position;  but  if  the  vertical  line 
should  meet  the  axis  below  the  centre  of  gravity,  the  body  will  be  pushed  forwards, 
its  declination  always  increasing  till  it  finally  oversets. 

Thus,  a  sphere  of  uniform  consistence  floated  in  water,  will  sink  till  the  weight 
of  the  fluid  displaced  by  the  immersed  portion  shall  be  equal  to  its  own  load.  The 
centre  of  gravity  of  this  body  is  the  centre  of  the  sphere  itself;  but  the  centre  of 
buoyaney  must  be  the  centre  of  gravity  of  the  volume  of  immersion,  which  there- 
fore lies  below  the  centre  of  gravity  of  the  body,  in  an  axis  perpendicular  to  the 
water  line,  or  line  of  floatation.  The  ball  is  hence  pressed  down  by  its  own 
weight  collected  in  its  centre  of  gravity,  and  pushed  up  in  the  opposite  direction  by 
an  equal  force  combined  in  the  centre  of  buoyancy ;  both  of  the  forces,  however, 
concurring  in  the  centre  of  gravity  of  the  immersed  sphere.  Wherefore,  being 
always  held  in  equilibrium  by  those  antagonist  forces,  it  will  remain  still  in  any 
position  which  it  may  happen  to  occupy.  But  this  indifference  to  floating  will 
obtain  only  when  the  sphere  is  perfectly  homogeneous,  and  its  centre  of  gravity 
coincides  with  the  centre  of  magnitude,  for  otherwise,  the  former  descending  as 
low  as  possible,  would  always  assume  a  determinate  position. 

A  cylinder  will,  according  to  its  density,  and  the  proportion  of  its  diameter  and 
altitude,  exhibit  the  three  features  of  a  floating  body,  in  indifference,  instability,  or 
permanence  of  equilibrium.  For  example,  a  cylinder,  the  specific  gravity  of  which 
is  to  that  of  the  fluid  in  which  it  floats,  as  3  to  4,  its  axis  being  to  the  diameter  of 
the  base  as  2  to  1,  if  placed  on  the  fluid  with  its  axis  vertical^  will  sink  to  a  depth 
equal  to  a  diameter  and  a  half  of  the  base ;  and  as  long  as  the  axis  is  sustained  in 
a  vertical  position  by  external  force,,  the  centre  of  gravity  of  the  solid  and  the 
centre  of  the  immersed  part  will  be  situated  in  the  same  vertical  line ;  but  the  solid 
will  not  float  permanently  in  that  position,  for  as  soon  as  the  external  force  is 
removed,  it  will  overset  and  float  with  its  axis  horizontal.  But  a  cylinder  whose 
axis  is  one  half,  instead  of  twice  the  diameter  of  the  base,  being  placed  in  a  fluid 
with  its  axis  vertical,  will  sink  to  the  depth  of  three  fourths  of  a  diameter^  and  will 


NOTES.  455 

float'  permanently  in  that  position.  Incline  it  as  you  may,  on  being  left  to  itself  it 
will  ultimately  settle  permanently,  with  its  axis  perpendicular  to  the  horizon.  The 
differences  of  the  phenomena  in  this  case,  arise  from  the  change  which  takes  place 
in  the  position  of  the  line  of  support;  and  what  is  true  of  the  cylinder  is  true  also 
of  other  figures ;  for  when  a  solid  changes  its  position,  by  revolving  on  an  axis  on 
the  surface  of  a  fluid,  any  position  of  equilibrium  is  always  succeeded  by  a  position 
of  equilibrium  which  is  of  a  contrary  description. 

A  segment  of  a  sphere  floating  in  water,  will  have  its  centre  of  gravity  below 
the  centre  of  the  sphere,  when  the  segment  floats  with  its  vertex  downwards, 
arid  in  an  axis  at  right  angles  to  its  base ;  but  the  centre  of  buoyancy,  or  the 
centre  of  gravity  of  the  immersed  segment,  must,  in  every  situation  of  the 
floating  mass,  occur  in  a  perpendicular  bisecting  the  water  line,  and  conse- 
quently passing  through  the  centre  of  the  sphere.  In  the  case  of  equilibrium  this 
perpendicular  must  have  a  vertical  position,  or  the  involved  base  of  the  segment 
must  form  a  horizontal  plane.  If  this  body  be  now  drawn  aside,  into  a  position 
which  shall  incline  ,jts  base  in  any  angle  with  the  water  line,  it  will  be  pressed 
down  by  its  own  weight,  collected  in  the  centre  of  gravity  of  this  body,  and  pushed 
upwards  by  an  equal  buoyant  power  exerted  at  the  centre  of  buoyancy.  This  force 
may  now  be  conceived  to  act  upon  any  point  in  the  line  connecting  the  centres  of 
gravity  and  buoyancy,  and  therefore  at  the  concourse  of  the  axis  in  the  case  of  equi- 
librium, and  of  the  vertical  line  when  the  body  is  drawn  aside.  The  buoyancy  trans- 
mitted to  this  point  pushes  the  axis  of  inclination  obliquely,  the  greater  part  of  it 
bearing  the  point  of  concourse  in  the  direction  of  the  axis  of  permanent  floatation, 
while  another  small  part  of  this  force,  pressing  perpendicular  to  the  axis  of  inclina- 
tion, makes  the  body  turn  about  its  centre  of  gravity,  from  the  higher  or  lower 
point  of  inclination  of  its  upper  surface,  till  it  ultimately  coincides  with  the  water 
line.  Every  derangement  is  thus  corrected  by  a  restoring  energy  which  maintains 
a  permanent  equilibrium. 

An  oblate  homogeneous  spheroid  will  sink  in  a  manner  similar  to  the  segment  of 
the  sphere,  and  carry  the  centre  of  buoyancy  in  a  like  position.  The  declination  of 
its  axis,  by  drawing  the  body  aside  from  the  position  of  permanent  equilibrium,  is 
restored  to  its  vertical  position  by  the  effort  of  buoyancy  exerted  at  a  point  above 
the  centre  of  gravity  of  the  spheroid,  which  tends  to  redress  the  floating  body  and 
secure  its  stable  equilibrium. 

On  the  other  hand,  a  prolate  spheroid  will  have  its  centre  of  buoyancy  and  plane 
of  floatation,  each  the  same  height  as  in  a  sphere  described  on  the  longer  axis  of 
the  spheroid.  But  the  shifting  of  its  centre  of  buoyancy  will  be  diminished  in 
proportion  to  the  narrowness  of  the  spheroid.  The  vertical  will  meet  the  principal 
axis  below  the  centre  of  gravity  of  the  solid,  and  will  push  it  aside  more  and  more 
till  the  spheroid  falls,  and  extends  its  longer  diameter  in  a  horizontal  position.  It 
may  then  roll  indifferently  upon  that  line,  as  the  sphere  turns  about  its  diameter. 

A  solid  of  any  form,  not  abruptly  irregular,  set  to  float  in  water,  will  be  divided 
into  correspondent  equal  portions  by  its  principal  axis,  which  will  cross  the  plane 
of  floatation  at  right  angles.  If  the  body  be  inclined,  its  centre  of  buoyancy  will 
shift  its  place  as  the  inclination  varies,  until  the  antagonist  forces  meet  in  a  point 
in  the  axis,  where  the  effort  of  the  body  to  redress  itself  remains  unaltered,  like  the 
centre  of  gravity  itself.  That  characteristic  point  standing  always  above  the  centre 
of  gravity  of  the  mass,  and  limiting  its  greatest  elevation  in  the  case  of  permanent 
stability,  has  been  called  the  metacentre. 

If  the  floating  body  be  a  homogeneous  parallelepiped  placed  vertically  in  the 


456  NOTES. 

fluid,  it  will  evidently  sink  till  the  immersed  part  shall  be  to  its  whole  height,  as  its 
density  is  to  that  of  the  fluid.  The  centre  of  buoyancy  will  be  below  the  centre  of 
gravity,  but  both  will  be  in  the  axis  of  the  solid ;  the  former  midway  between  the 
base  of  the  parallelepiped  and  the  water  line ;  the  latter  halfway  between  the  base 
and  summit  of  the  body.  If  the  solid  be  inclined  to  one  side,  its  water  line  will 
shift  its  position  on  the  body;  the  centre  of  buoyancy  will  make  a  corresponding 
change,  describing  a  small  arc  of  a  circle,  till  it  be  raised  in  relation  to  the  altitude 
of  the  centre  of  gravity  of  the  extant  triangle,  as  the  area  of  the  adjacent  rectangular 
figure  is  to  that  of  the  triangle,  while  the  moveable  centre  of  buoyancy  is  carried 
laterally  in  the  same  ratio.  And  when  the  metacentre  coincides  with  the  centre 
of  gravity,  the  solid  floats  passively  and  indifferent  to  its  position.  If  the  paral- 
lelopiped  become  a  cube,  then  its  breadth  and  length  being  equal,  the  two  densities 
of  indifferent  floatation  are  expressed  in  the  numbers  ^  and  |§.  Between  these 
limits  there  can  be  no  stability,  but  above  and  below  them  the  floating  body 
acquires  permanence. 

Both  experiment  and  calculation  prove,  that  a  parallelepiped  of  half  the  density 
of  water,  and  having  9  inches  for  its  altitude,  and  11  inches  for  the  side  of  its 
square  base,  will  float  indifferently ;  but  it  will  gain  stability  if  its  density  be  either 
increased  or  diminished.  With  a  density  two  thirds  that  of  water,  the  metacentre 
will  stand  ^  parts  of  an  inch  above  the  centre  of  gravity ;  and  $  parts  of  an  inch 
above  it  if  the  density  be  reduced  to  one  third.  With  such  proportions,  a  paral- 
lelopiped  might  therefore  in  every  case  continue  erect ;  and  copper  or  sheet-iron 
tanks,  with  such  proportions,  would  float  safely  as  pontoons  for  flying  bridges. 

But  this  is  not  all :  we  can  prove,  that  if  the  parallelepiped  be  set  upon  water, 
with  one  of  its  solid  angles  uppermost,  the  stability  will  be  limited  within  the 
densities  of  395  and  ||.  jn  a  Word,  let  the  specific  gravity  be  greater  than  ^  or  less 
than  |§,  the  solid  would  permanently  float  in  that  position  :  but  were  the  specific 
gravity  either  less  than  the  former,  or  greater  than  the  latter,  the  body  would 
overset.  Were  the  parallelepiped  thus  set  on  water,  with  one  of  its  diagonals 
immersed  and  the  other  vertical,  its  equal  side  being  18  inches,  then  it  would  sink 
about  14^  inches  on  the  side ;  9%  inches  of  the  diagonal  would  be  immersed,  and 
nearly  16  extant;  supposing  the  specific  gravity  of  the  solid  to  be  0.326,  that  of  the 
fluid  being  equal  to  unity. 

In  short,  the  determination  of  the  positions  of  equilibrium  of  a  solid  body,  floating 
on  a  fluid  of  a  given  density  greater  than  itself,  is  reducible  to  a  problem  of  pure 
geometry,  which  may  be  better  expressed  as  follows  : — 

To  cut  any  proposed  "body  by  a  plane,  so  that  the  volume  of  one  of  the 
segments  may  be  to  that  of  the  whole  body  in  a  given  ratio ;  and  such  that 
the  centre  of  gravity  of  the  whole  body,  and  that  of  one  of  its  segments,  may 
be  both  found  in  a  line  perpendicular  to  the  cutting  line. 

In  order  to  the  complete  solution  of  this  problem,  it  is  necessary  in  each  parti- 
cular case,  to  express  the  two  conditions  of  equilibrium  by  means  of  equations,  the 
solutions  of  which  will  make  known  all  the  directions  that  can  be  given  to  the  cut- 
ting plane,  and  whence  necessarily  result  all  the  positions  of  equilibrium  of  the  body. 

This  is  precisely  the  plan  we  have  pursued,  and  all  our  investigations  proceed  to 
ascertain  these  two  conditions  of  equilibrium;  and  from  the  resulting  or  final 
equations,  to  draw  up  a  geometrical  construction  of  the  positions  so  determined ; 
for  calculation  is  here  an  instrument  of  necessity,  and  not  a  vain  exhibition  of 
analytical  formulae,  difficult  to  follow  and  still  more  difficult  to  apply. 


NOTES.  457 


NOTE  F.— CHAPTER  XIII. 

The  investigations  pursued  in  this  and  the  previous  chapter,  explain  the  cause 
of  the  oversetting  of  the  large  icebergs  which  sometimes  float  within  the  limits  of 
the  temperate  zone.  These  enormous  blocks  of  frozen  fresh  water  assume  various 
forms  i  some  are  columnar,  others  approach  the  parallelepiped  in  their  outline, 
others  again  resemble  mis-shapen  cylinders;  but  all  evidently  different  in  form 
below  the  plane  of  floatation  to  what  they  exhibit  in  their  extant  volume.  The 
action  of  the  atmosphere  as  the  summer  advances,  slowly  thaws  the  upper  surface ; 
the  under  side  likewise  melts  at  first,  but  becomes  soon  protected  by  a  pool  of  fresh 
water  of  the  same  temperature,  consisting  of  the  dissolved  portion  of  the  ice  which 
is  upheld  by  the  superior  density  of  the  surrounding  medium.  The  principal  waste 
of  the  icy  mass  taking  place  along  its  immersed  sides,  the  current  of  melted  water 
continually  rises  upwards,  and  leaves  a  new  surface  to  the  attack  of  a  warmer 
current.  Whenever  therefore,  the  breadth  of  the  vast  column  becomes  so  reduced 
that  it  approaches  to  three  fourths  of  its  altitude,  the  icy  parallelepiped  will 
overset,  and  present  a  new  position  of  equilibrium.  Thus,  if  the  whole  height 
of  the  mass  were  1000  feet,  890  feet  would  be  submerged  in  the  ocean,  and  110  feet 
would  be  extant,  towering  amidst  the  waves.  In  this  case,  the  elevation  of  the 
centre  of  gravity  beyond  that  of  buoyancy  would  be  35  feet,  which  is  the  limit  of 
the  metacentre  after  the  base  of  the  column  has  been  reduced  to  a  breadth  of 
766  feet. 

An  iceberg  of  a  cylindrical  form  1000  feet  high,  would  sink  889  feet  in  the 
ocean ;  but  when  the  diameter  of  its  base  was  reduced  to  nearly  the  same  dimen- 
sions, say  885,  it  would  overset  and  take  a  new  position.  The  instability  of  the 
cylinder  takes  place  earlier  than  that  of  the  parallelepiped,  or  when  the  width  below 
becomes  eight  ninths,  instead  of  three  fourths  of  the  whole  height.  Since  then  the 
extant  portion  wastes  more  slowly  than  the  immersed  portion,  the  greater  the 
extension  of  the  summit,  the  more  it  will  hasten  the  change  of  position  by  over- 
whelming the  icy  mass. 

And  if  the  block  be  wasted  and  rounded  below  into  the  shape  of  a  parabolic 
conoid,  it  will  suffer  a  total  inversion  the  moment  its  base  is  reduced  to  Its  depth 
in  the  ratio  of  about  11  to  20,  and  its  lowest  point  will  become  the  summit  of  the 
extant  mass.  This  form  of  a  body  of  ice  would  therefore  suffer  a  greater  previous 
waste ;  but  its  balance  is  in  the  end  more  effectually  destroyed.  In  every  case, 
stability  becomes  precarious  after  the  breadth  of  the  block  is  inferior  to  its  depth. 

NOTE  G.— CHAPTER  XIV. 

Upon  the  pressure,  cohesion,  and  capillary  attraction  of  fluids  that  are  heavy, 
depends  their  transmission  through  fissures  of  the  earth  and  between  its  strata, 
which  are  pervious  to  the  percolation  of  water.  We  can  penetrate  but  a  small 
distance,  say  500  fathoms,  in  digging  for  coal ;  a  less  depth  suffices  for  some  ores, 
and  water  is  found  at  all  depths,  from  a  few  feet  to  three  hundred,  as  in  the  neigh- 
bourhood of  London.  In  the  great  coal  area  of  Britain,  extending  lengthwise  260 
miles,  and  in  breadth  about  150  miles,  in  a  diagonal  line  from  Hull  to  Bristol,  in 
England,  and  from  the  river  Tay  to  the  Clyde,  in  Scotland,  we  find  a  great  variety 


458  NOTES. 

of  rocks  or  strata,  piled  up  at  a  small  angle  with  the  horizon,  though  in  some 
instances,  like  the  primitive,  nearly  vertical.  These  strata  consist  of  sand-stone, 
clay-slate,  bituminous  slate,  indurated  argillaceous  earth,  or  fireclay,  argillaceous 
ironstone,  and  greenstone  or  blue  whinstone.  And  to  possess  the  valuable  treasures 
concealed  among  these  rocks,  we  employ  a  vast  capital  in  money,  and  tax  all  the 
ability  of  the  human  mind  in  the  science  of  engineering. 

To  bring  the  subject  matter  of  capillary  attraction,  as  regards  Artesian  wells, 
springs,  mountainous  marsh  lands,  or  bogs,  fairly  before  the  reader  in  a  very  brief 
manner,  we  shall  avail  ourselves  of  a  vertical  section  of  the  strata  in  Derbyshire, 
selecting  our  materials  from  the  valuable  work  of  Mr.  Whitehurst, "  On  the  original 
State  and  Formation  of  the  Earth." 

If  the  reader  conceive  the  alluvial  covering  to  be  removed,  the  strata  will  at 
once  appear  on  the  upper  surface,  as  in  the  external  contour  of  the  country  between 
.Grange  Mill  and  Darley  Moor,  in  Derbyshire.  Let  now  the  numbers  1,  2,  3,  4, 
&c.  represent  the  strata  in  their  vertical  position,  bassetting  towards  8,  with  the 
river  Derwent  running  over  a  fissure  filled  with  rubble  in  the  centre. 


Then,  the  upper  stratum,  or  No.  1,  at  Darley  Moor,  is  Millstone  Grit,  a  rough 
sandstone,  120  yards  deep,  composed  of  granulated  quartz  and  quartz  pebbles, 
without  any  trace  of  the  animal  or  vegetable  kingdoms. 

The  next  stratum,  called  No.  2,  which  is  found  on  both  sides  the  Derwent,  is  a 
bed  of  Shale,  or  Shiver,  120  yards  deep,  being  a  black  laminated  clay,  much  indu- 
rated, without  either  animal  or  vegetable  impressions.  It  contains  ironstone  in 
nodules,  and  the  springs  issuing  from  it  are  chalybeate,  as  that  at  Buxton  Bridge, 
or  that  at  Quarndon,  and  another  near  Matlock  Bridge,  towards  Chatsworth. 

Next  in-  succession  we  have  No.  3,  Limestone,  50  yards  thick,  productive  of  lead 
ore,  the  ore  of  zinc,  calamine,  pyrites,  spar,  fluor,  cauk,  and  chert.  This  stratum 
is  full  of  marine  debris,  as  anomince  bivalves,  not  known  to  exist  in  the  British  seas ; 
also  coralloids,  entrochi  or  screw  stones;  and  amphibious  animals  of  the  Saurian, 
or  lizard  and  crocodile  tribe,  some  of  which,  in  a  fossil  state,  are  of  enormous  size. 

Following  this  we  have  No.  4,  a  bed  of  Toadstone,  16  yards  thick,  but  in  some 
instances  varying  in  depth  from  6  feet  to  600  feet.  It  is  a  blackish  substance, 
resembling  lava,  very  hard,  with  bladder  holes,  like  the  scoria  of  metals  or  Iceland 
lava.  This  stratum  is  known  by  different  names  in  different  parts  of  Derbyshire. 
At  Matlock  and  Winster  it  is  loadstone  and  blackstone ;  at  Moneyash  and  Tidswell 
it  is  called  channel}  at  Castleton,  cat-dirt;  and  at  Ashover,  black-clay.  "  This 
toadstone,  channel,  cat-dirt,  and  black-clay,  is  actually  lava,  and  flowed  originally 
from  a  volcano,  whose  funnel  or  shaft  did  not  approach  the  open  air,  but  which 
disgorged  its  contents  between  the  adjacent  strata  in  all  directions,"  at  a  period 
when  the  limestone  strata  and  the  incumbent  beds  of  millstone-grit,  shale,  argilla- 


NOTES.  459 

ceous  stone,  clay,  and  coal,  had  an  uniform  arrangement  concentric  to  the  centre 
of  the  earth. 

Beneath  all  these  we  have  No.  5,  a  Limestone  formation,  50  yards  thick,  and 
similar  to  No.  3;  that  is  to  say,  laminated,  containing  minerals  and  figured  stones. 
It  is  productive  of  marble  ;  it  abounds  with  entrochi  and  marine  exuviae :  it  was 
thence  at  one  time  the  bed  of  a  primaeval  ocean. 

No.  6  is  Toadstone,  40  yards  deep,  and  similar  to  No.  4,  but  yet  more  solid, 
showing  that  the  fluid  metal  was  much  more  intensely  heated  and  combined  than 
No.  4. 

No.  7,  Limestone,  very  white,  60  yards  deep ;  laminated  like  No.  3  and  5,  and 
like  them  it  contains  minerals  and  figured  stones,  and  was  either  a  continuation  of 
Nos.  3  and  5,  the  entire  mass  having  been  split  at  different  depths  by  the  expansive 
power  of  the  boiling  lara. 

No.  8,  is  Toadstone,  22  yards  deep,  similar  to  No.  6,  but  yet  more  solid. 

No.  9,  Limestone,  resembling  Nos.  3,  5,  and  7. 

To  this  enumeration  of  the  Derbyshire  strata  we  must  now  add  six  other  strata  f 
too  minute  to  be  expressed  in  the  same  scale,  but  which  are  in  fact  the  capillary 
strata,  which  we  may  liken  to  the  glass  plates  referred  to  hi  Problem  LXXI. 
Miners  call  these  minute  parallel  strata,  clays,  OT  way-boards :  in  general  they  are  not 
more  than  four,  five,  or  six  feet  thick,  and  in  some  instances  not  more  than  one  foot. 
They  are  the  channels  for  water,  and  all  the  springs  flowing  from  them  are  warmy 
like  those  at  Buxton  and  Matlock  Bath.  The  first  stratum  of  clay  separates  Nos.  3 
and  4 ;  the  second,  Nos.  4  and  5;  the  third,  Nos.  5  and  6;  the  fourth,  Nos.  6  and 
7 ;  the  fifth,  Nos.  7  and  8 ;  the  sixth,  Nos.  8  and  9 :  and  what  is  very  remarkable, 
by  these  clays  the  thickness  of  the  other  strata  may  be  ascertained,  which  would 
otherwise  be  difficult,  as  the  limestone  beds  consist  of  various  lamina. 

There  are  several  circumstances  illustrative  of  this  capillary  attraction,  which 
receive  illustration  from  the  diagram  before  us ;  to  these  we  shall  now  address- 
ourselves ;  and,  first,  it  is  observable  that  all  the  parallel  strata  basset  or  shoot 
towards  the  surface,  occasioning  thereby  a  diversity  of  soil ;  and  as  the  beds  or 
layers  of  rock,  £c.  contain  fossil  remains,  we  may  expect  to  meet  with  shells,  corals,, 
bones,  plants,  trees,  &c.  on  or  near  the  surface.  All  these  rocks  ranged  in  beds  or 
layers,  whether  perfectly  horizontal  or  shooting  up  at  any  angle,  are  called  strati- 
fied;  while  abrupt  masses  of  granite,*  having  none  of  this  masonic  appearance,  are 
said  to  be  unstratified.  It  is  obvious,  from  what  has  been  observed  above,  that  the 
stratified  parts  of  the  globe  are  those  in  which  we  must  look  for  capillary  veins 
and  sheets  of  water. 

In  the  diagram  before  us  all  the  strata  are  distinctly  marked  with  their  various 
dislocations  and  fissures.  The  river  Derwent  is  supposed  to  flow  over  a  vast  fissure, 
R  ;  the  letters  A,  A,  A  indicate  lesser  fissures  ;  G,  G,  G  do  the  same,  and  all  these 
fissures  are  hi  the  limestone  strata.  Hence  it  appears  that  the  toadstone  or  lava 


*  Granite  consists  of  distinct  aggregations  of  quartz,  felspar,  mica,  and  hornblende,  each  in  a 
crystalline  form.  Felspar  is  of  a  whitish,  sometimes  of  a  reddish  colour,  quite  opaque,  aad  occasion- 
ally crystallized  in  a  rhomboidal  form ;  quartz  is  less  abundant,  somewhat  transparent,  and  of  a 
glassy  appearance ;  mica  is  dispersed  throughout  in  small  glistening  plates,  the  colour  is  dark  and 
the  appearance  metallic ;  hornblende  imparts  a  deep  green  colour  to  rocks  called  greenstone  and 
basalt. 


460  NOTES. 

strata  are  attended  with  many  peculiar  circumstances,  very  different  from  their 
associates,  3,  5,  7,  9.  These  peculiarities  are  : 

1.  Toadstone  is  similar  to  Iceland  Lava  both  in  its  appearance  and  chemical 
qualities.  2.  It  is  extremely  variable  in  thickness.  3.  It  is  not  universal.  4.  It 
has  no  fissures  corresponding  to  those  in  limestone.  5.  It  frequently  fills  up  the 
fissures  in  the  stratum  underneath  it,  as  at  H,  and  the  bottom  of  the  shaft  s,  which 
enters  a  fissure  of  toadstone  that  in  a  liquid  state  has  flowed  into  the  limestone 
stratum,  numbered  9.  Throughout  the  limestone  strata  of  Derbyshire  the  fissures 
we  have  marked  correspond ;  and  in  these  fissures,  and  between  their  lamina,  the 
minerals  are  found.  The  mines  in  the  fissures  are  called  rdke-wor ks ;  the  mines  in 
the  laminae  are  called  pipe-works.  Thus  in  the  stratum  No.  3,  we  find  Yatestoop 
mine ;  the  Portaway  and  Placket  mines  are  in  No.  5. ;  in  No.  7.  we  have  Mosey- 
meer,  and  in  No.  9  Gorseydale  mines ;  Hangworm  mine  is  on  Bonsai  Moor.  The 
stratum  No.  5  bassets  and  forms  the  surface  of  the  earth  at  Foolow  and  Bonsai 
Moor.  No.  8  bassets  and  becomes  the  base  of  the  land  called  Grange  Mill.  No. 
3  again  bassets  and  becomes  the  districts  Trogues  Pasture  to  the  right,  and 
Wensley  to  the  left  of  the  great  shaft  sunk  at  o  and  trending  below  ground  to  the 
fissure  G  in  No.  5.  Here  we  have  a  beautiful  illustration  of  the  genius  of  geological 
engineering.  A  spring  occurs  at  I  in  the  fissure  G,  No.  3,  too  powerful  to  be 
overcome,  or  too  expensive  to  be  kept  under ;  accordingly  a  shaft  is  sunk  at  o 
higher  up  the  acclivity.  The  miners  pioneer  to  a,  descend  to  the  fissure  G  by 
driving  a  gallery  or  gate,  as  they  term  this  tunnel,  and  this  is  a  common  practice, 
and  never  fails  in  producing  dry  work  in  the  stratum  No.  5,  for  the  close  texture 
of  the  toadstone  will  not  allow  the  water  in  the  seam  between  3  and  4  to  percolate 
its  impervious  mass,  although  the  pool  may  accumulate  from  10  to  15  fathoms  in 
No.  3.  If  the  water  in  3  rise  not  to  the  horizontal  level  L  L,  it  can  never  incom- 
mode the  shaft  o  a.  The  grand  geological  fact  elicited  here  is,  as  regards  capillary 
attraction,  that  toadstone  turns  water,  is  free  from  fissures,  nay,  sometimes  fills 
up  fissures,  as  at  s  and  H,  which  the  miners  call  troughing.  In  the  Slack  and 
Salterivay  mines  on  Bonsai  Moor,  some  forty  years  ago,  these  cros.-ralte  fissures 
were  noticed  by  Mr.  Whitehurst.  Their  occurrence  in  other  mines  need  not 
astonish  geological  engineers. 

In  other  districts  in  Britain,  we  find  that  the  coal  formations  sometimes  repeat, 
in  precisely  the  same  order,  arid  in  nearly  the  same  thickness,  the  following  earths 
and  minerals :  sandstone,  bituminous  shale,  slate  clay,  clay  iron,  stone,  coal ;  or 
the  coal  is  covered  with  slate,  trap,  or  limestone,  or  rests  upon  these  rocks.  The 
strata  generally  follows  every  irregularity  of  the  fundamental  rock  on  which  they 
rest  j  but  in  some  instances  their  directions  appear  independent,  both  of  the  surface 
of  the  rock,  and  of  the  cavity  or  hollow  in  which  they  are  contained,  and  in  general 
take  a  waved  outline,  seldom  rising  greatly  above  the  level  of  the  sea. 

We  have  now,  however,  merely  represented  the  general  arrangement  of  the 
strata ;  not  all  the  particular  circumstances  accompanying  them,  with  respect  to 
their  several  fractures,  dislocations,  &c. ;  but  it  will  enable  us  to  reason  upon  the 
chemical  effects  of  water  upon  limestone  and  gypsum  rocks,  where  we  meet  with 
caverns,  caves,  and  extensive  fissures,  that  reach  sometimes  to  the  surface,  some- 
times dip  to  a  greater  or  less  distance,  and  afford  channels  for  great  springs  and 
subterranean  rivers.  These  caves  in  the  gypsum  and  chalk  formations  vary  in 
magnitude  from  a  few  yards  to  many  fathoms  in  extent,  forming  upon  the  surface 
of  the  ground,  when  their  superincumbent  roofs  give  way,  those  funnel-shaped 


NOTES.  461 

h6llows  of  such  frequent  occurrence  in  gypsum  districts.  The  limestone  strata, 
besides  being  "  loaded  with  the  exuviae  of  innumerable  generations  of  organic 
beings,"  says  Dr.  Buckland,  "  afford  strong  proofs  of  the  lapse  of  long  periods  of 
time,  wherein  the  animals  from  which  they  have  been  derived,  lived,  and  multiplied 
and  died,  at  the  bottom  of  seas  which  once  occupied  the  site  of  our  present  con- 
tinents and  islands."  *  With  how  much  reason  then  may  we  not  suppose  those 
formations  to  have  held  large  beds  of  rock  salt,  which  the  percolation  of  water,  in 
the  lapse  of  ages,  removed,  and  left  the  chambers  empty,  or  the  receptacles  of 
meteoric  water.  The  percolation  of  water  through  felspar  rocks,  must  of  necessity 
wash  away  the  alkaline  ingredient,  which  combining  with  iron  will  form  hydrate, 
or  by  its  decomposition  oxidate  the  metallic  substance.  Hence  result  chalybeate, 
acidulous,  sulphureous,  and  saline  springs,  all  the  result  of  capillary  attraction  in 
the  strata  of  the  earth,  and  the  disintegration  by  water  of  the  various  ingredients 
which  the  universal  solvent  holds  in  a  state  of  fluidity. 

Supposing  these  cavities,  to  which  we  have  just  referred,  to  have  been  freed  from 
their  original  salt  deposits,  by  water  percolating  the  fissures  leading  to  and  from 
the  masses  of  salt,  we  trace  the  operation  of  salt  springs.  For  in  all  cases  in  which 
water  holds  any  mineral  in  solution,  it  acts  by  combination,  but  where  it  simply 
destroys  the  mineral  aggregation,  the  mineral  falls  into  small  pieces  with  an  audible 
noise,  as  is  observed  in  bole ;  or  it  falls  without  noise  into  small  pieces,  which  are 
soon  diffused  through  the  fluid,  without  either  dissolving  in  it  or  becoming  plastic, 
as  in  Fuller's  earth,  and  some  minerals,  as  unctuous  clay ;  it  renders  plastic  other 
minerals,  absorbs  water  in  greater  or  less  quantity,  by  which  their  transparency, 
and  also  their  colour,  are  changed. 

The  toadstone,  which  intersects  mineral  veins,  totally  cuts  off  all  communication 
between  the  upper  and  lower  fissures,  and  by  the  closeness  of  its  texture  permits 
not  the  water  in  the  clay  strata,  or  way-boards,  to  filtrate.  Hence  toadstone  is 
said  to  be  capable  of  turning  water,  as  we  have  shown  in  the  shaft  and  gallery, 
o  a  G  G.  Sandstone  strata,  of  an  open  porous  texture,  becomes  a  great  feeder 
of  water.  Several  of  the  sandstones  are,  however,  impervious  to  water,  and 
almost  all  the  beds  of  light-coloured  argillaceous  schistus,  or  fine  clays,  are 
particularly  so,  being  very  close  in  their  texture.  But  the  percolation  of  water 
at  the  beds  or  partings  of  two  strata  is  an  occurrence  so  general,  that  our 
wonder  ceases  when  examining  parts  of  the  country  where  the  strata  basset  or 
shoot  to  the  surface  in  an  acute  angle,  to  find  the  alluvial  covering  in  places 
swampy,  marshy,  and  overrun  with  puddles,  springs,  and  all  that  species  of  soil, 
which,  being  damp  and  cold,  subjects  its  inhabitants  to  rheumatism,  agues,  and  a 
train  of  diseases,  unknown  in  regions  that  are  not  incumbent  on  the  extremities  of 
way-boards  and  capillary  strata.  The  source  or  feeder  of  these  subterranean 
capillaries  receiving  a  constant  supply,  keeps  up  the  train  of  human  ills  from  one 
generation  to  another,  while  local  interests  or  associations  bind  the  natives  to  their 
hereditary  doom. 

Capillary  attraction  and  cohesion,  besides  expounding  the  phenomena  of  fluid 
ascent  in  strata  of  earth,  direct  us  in  penetrating  those  troublesome  quicksands  and 
beds  of  mud,  which  in  the  winnings  of  collieries  are  met  with  in  mining,  and  where 


*  Dr.  Buckland's  Bridgewater  Treatise,  pp.  112-116,  1st  ed.  vol.  i. 


462  NOTES. 

cast-iron  tubbing  is  employed  to  support  the  sand  or  mud  bed,  and  carry  the  water 
down  to  the  bottom  of  the  pit. 

Water  stands  higher  in  narrow  than  in  wide  glass  tubes,  but  quicksilver  mounts 
higher  if  the  inside  of  the  tube  be  lined  with  bees-wax  or  tallow.  We  can  easily 
conceive  that  the  lateral  action  may  yet  cause  the  perpendicular  ascent  j  for  it  is 
a  fundamental  property  in  fluids,  that  any  force  impressed  in  one  direction  may  be 
propagated  equally  in  every  other  direction.  Hence  the  affinity  of  the  fluid  to  the 
internal  surface  producing  the  vertical  ascent.  A  drop  of  water  let  fall  on  a  clean 
plate  of  glass  spreads  over  the  whole  surface,  in  as  far  as  there  is  liquid  to  cover 
the  glass,  the  remoter  particles  extending  the  film,  yet  adhering  with  the  closest 
union.  The  adhesiveness  of  fluids  is  still  more  clearly  shown  in  their  projection 
through  the  pores  of  minerals,  plants,  animals,  gravel,  earth,  and  sand.  Water 
rises  through  successive  strata  of  gravel,  coarse  sand,  fine  sand,  loam,  and  even 
clay :  and  hence,  on  the  sea-coast,  those  quicksands,  which  have  engulfed  armies 
and  ships,  the  pressure  and  elevation  of  the  ocean  at  flood  tide  sending  its 
advanced  column  up  in  the  sand  to  a  level  with  its  surface  far  out  at  sea.  Gravel 
divided  into  spaces  of  the  hundredth  part  of  an  inch,  will  allow  water  to  ascend 
above  four  inches ;  it  would  mount  up  through  a  bed  of  sixteen  inches  of  this 
material,  supposing  sea  gravel  to  be  the  500th  part  of  an  inch.  Fine  sand,  in 
which  the  interstices  are  the  2,500th  part  of  an  inch,  allow  the  humidity  to 
ascend  seven  feet  through  a  new  stratum;  and  if  the  pores  of  the  loam  were 
only  the  10,000th  part  of  an  inch,  it  would  gain  the  further  height  of  25f  feet 
through  the  soft  mass ;  thence  originate  v astce  syrtes.  The  clay  would  retain  the 
moisture  at  a  greater  altitude ;  but  the  extreme  subdivisions  of  the  clay,  which 
enable  it  to  carry  water  to  almost  any  elevation,  yet  make  it  the  most  efficient 
material  in  puddling  or  choking  up  the  interstices  of  masonry. 

The  ascent  of  water  in  a  glass  tube  is  due  chiefly,  we  think,  to  the  excess  of  the 
attractive  power  of  the  glass  above  the  cohesive  power  of  the  fluid  mass  over  itself. 
Were  the  attractive  and  cohesive  forces  equal,  the  fluid  would  remain  balanced  at 
a  common  level.  Mercury  hence  sinks,  by  reason  of  the  strong  cohesive  power  of 
its  own  particles.  Hence  we  account  for  mercury  closing  over  a  ball  of  crude 
platinum,  which  nevertheless,  being  gently  laid  on  the  mercury,  will  float,  although 
its  specific  gravity  is  above  that  of  mercury. 

It  is  however  the  province  of  chemistry,  rather  than  of  mechanics,  to  measure 
the  cohesive  power  possessed  by  different  fluids,  or  by  the  same  fluid  under  different 
degrees  of  temperature. 

The  suspension  of  water  in  any  stratum  through  which  it  can  percolate,  must 
depend  entirely  upon  the  smallness  of  the  upper  orifice,  or  superficial  extent  of 
the  deflection  with  which  the  stratum  slopes  off  horizontally  above  ground,  and 
upon  the  relative  elevation  of  the  extremities  of  the  impervious  stratum.  Thus, 
suppose  a  and  b  to  be  two  extremities  of  a  stratum  pervious  to  water ;  the  central 
column  of  water  at  c  is  pressed  with  the  whole  weight  of  the  space  be,  and  this 
pressure  upon  c  a  pushes  the  fluid  out  at  a 
by  the  excess  of  force  in  be  above  that  in 
«a;  and  therefore,  while  the  ground  or 
land  at  b  is  generally  dry,  that  at  a  is  per- 
haps boggy;  at  all  events  it  will  exhibit 
springs  at  its  surface,  be  cold,  damp,  and 
its  inhabitants  subject  to  rheumatism  or 


NOTES.  463 

agues.  A  column  of  water  of  this  description  may  occupy  a  space  of  many  miles 
extent  between  b  and  a ;  and  c  may  be  many  hundred  feet  deep  below  the  hori- 
zontal level  of  a.  In  digging  for  water  at  d,  we  should  find  it  at  e. 

The  cohesion  of  the  particles  of  water,  and  its  extreme  facility  to  obey  any 
impression,  fit  it  admirably  for  percolating  through  fissures  of  the  earth,  when  in 
the  tenderest  filaments  it  is  detached  from  the  general  fluid  mass,  and  penetrates 
only  by  the  laws  of  capillary  attraction  from  one  point  to  another  in  an  extensive 
stratum  of  clay,  precisely  as  if  it  flowed  through  a  pipe  in  passing  from  one  hill  to 
another.  Hence  the  certainty  with  which  we  meet  with  water  in  boring  to  a 
proper  depth  in  the  earth,  and  hence  also  the  origin  of  Artesian  wells,  which  finely 
expound  the  varied  phenomena  of  a  retreating  and  subsiding  column  towards  the 
body  of  the  fluid,  as  if  an  equal  and  opposite  pressure  from  the  sides  of  a  capillary 
tube  had  come  into  action.  We  may  hence  infer,  that  in  strata  pervious  to  water, 
the  capillary  ascension,  however  much  it  may  be  accelerated  or  retarded  by  the 
parallel  sides  of  the  stratum  and  the  material  of  which  it  is  composed,  is  governed 
by  these  three  principles  which  we  have  fully  discussed,  pressure  from  above, 
cohesion  subsisting  among  the  particles  of  the  liquid,  and  attraction  of  the  parallel 
sides  of  the  stratum.  Were  this  attraction  equal  to  the  antagonist  cohesion,  the 
fluid  would  remain  at  rest,  balanced  at  a  common  level,  till  overcome  by  the  weight 
of  the  contents  in  the  longer  branch  of  the  fluid  column  forcing  the  contents  of  the 
shorter  column  out  at  the  discharging  orifice.  All  the  springs  which  are  below  the 
London  clay,  at  the  depth  of  150,  200,  250,  or  300  feet,  are  fed  by  sources  con- 
siderably elevated  above  the  Hampstead  level.  With  what  ease  then  might  the 
metropolis  be  provided  in  every  street  with  spring  water  from  an  Artesian  Well  ! 

Any  of  our  readers  who  may  be  desirous  of  acquiring  a  practical  and  thorough 
knowledge  of  geology,  must  chiefly  prosecute  his  studies  by  laborious  researches  in 
the  great  field  of  nature,  and  must  there  explore  for  himself  the  various  phenomena 
presented  to  his  view.  His  first  step  must  be  to  understand  by  reading  the  leading 
facts  and  principles  of  the  science ;  he  must  learn  to  recognise  at  once  the  principal 
simple  minerals,  entering  into  the  composition  of  rocks,  and  also  the  various 
metallic  ores  and  other  minerals  which  usually  occur  in  veins.  He  must  likewise 
be  acquainted,  the  more  minutely  the  better,  with  at  least  the  more  common  forms 
of  fossil  organization,  and  with  the  general  mode  of  their  distribution  in  the  rocky 
masses  constituting  the  crust  of  the  globe.  Some  preliminary  knowledge  of 
chemistry,  although  not  perhaps  essential,  will  form  a  very  desirable  addition  to 
the  qualifications  already  named.  Thus  provided  with  the  knowledge  requisite 
to  decipher  the  instructive  pages  on  which  nature  has  recorded,  in  her  own 
language,  the  history  and  revolutions  of  our  planet,  the  student  may  now  com- 
mence the  most  valuable,  but  far  the  most  laborious  part  of  his  career.  He  must 
visit  the  deep  recesses  of  our  mines,  which,  although  too  much  neglected,  afford 
the  finest  examples  of  many  of  the  most  important  facts  on  which  the  science  of 
geology  is  built.  He  must  observe  the  strata  as  laid  open  in  our  quarries,  and  as 
displayed  in  the  deep  cuttings  of  our  roads,  railways,  and  canals.  Every  excavation 
will  indeed  present  something  worthy  of  notice  to  Ms  view;  but  not  contented 
with  observing  merely  these  spots,  where  the  labour  of  man  has  penetrated  into 
the  interior  of  the  earth,  he  must  wander  around  the  base  of  the  lofty  cliffs  which 
overhang  the  ocean,  and  observe  the  grand  and  instructive  sections  which  nature 
herself  presents,  and  of  which  our  own  islands  afford  such  numerous  and  admirable 
examples.  He  must  pursue  the  course  of  rivers  into  the  interior,  and  observe  the 


464  NOTES. 

strata  laid  open  by  the  excavations  of  their  currents;  but  his  most  instructive 
studies  will  be  found,  when  he  has  arrived  far  inland  at  the  mountains,  where  they 
take  their  rise.  Here  he  will  find  that  nature  has  revealed  the  structure  of  the 
globe  on  the  grandest  scale ;  here  the  marks  of  ancient  revolutions  will  be  found 
imprinted  in  characters  not  to  be  mistaken,  and  the  truth  both  of  facts  and 
theories,  before  known  only  by  description,  will  at  once  be  impressed  on  his  mind. 
By  researches  of  this  kind,  extended  over  considerable  tracts  of  countries,  so  as  to 
embrace  all  the  great  series  of  geological  formations,  and  by  careful  study  and 
comparison  of  all  the  phenomena  presented  to  his  view,  both  as  regards  the 
mineral  structure  of  the  globe,  the  forms  of  organized  bodies  peculiar  to  each 
species  of  rocks,  and  the  physical  changes  now  taking  place  on  the  earth's  surface, 
the  student  wifl  at  length  become  a  practical  geologist,  and  be  enabled  by  his  own 
observations  to  improve  and  advance  the  science  he  has  been  studying.  But  the 
course  which  has  here  been  pointed  out,  although  essential  to  a  practical  and 
thorough  knowledge  of  the  subject,  can  only  be  pursued  by  few;  and  a  general  idea 
of  its  most  important  facts,  and  the  practical  consequences  arising  from  them,  is 
of  comparatively  easy  attainment.  The  great  principles  of  geology  have  been  most 
ably  brought  together  in  various  publications ;  and  where  only  a  general  knowledge 
is  required,  geological  maps  and  sections  may  be  made  in  some  measure  to  supply 
the  place  of  travelling  and  observation.  A  few  words  then  on  these  important 
documents,  which  are  the  medium  of  expressing  some  of  the  most  important 
practical  results  of  the  labours  of  geologists  in  the  field,  may  not  be  misplaced.  A 
map  which  combines  with  the  geographical  and  physical  features  of  a  country, 
a  view  of  its  internal  structure,  supposes  all  wood  and  vegetation  to  be  absent, 
and  that  every  species  of  superficial  soil  and  covering  removed,  so  that  the 
actual  rocks  and  strata  which  compose  the  solid  crust  of  the  globe  beneath  shall  be 
perfectly  exposed  and  laid  open  to  our  view.  The  space  occupied  at  the  surface  by 
these  rocks  and  strata  is  then  distinctly  shown  by  different  tints  of  colour,  in  the 
same  manner  as  territorial  divisions  are  indicated  on  ordinary  geographical  maps. 
But  although  we  thus  obtain  a  perfect  view  of  the  surface  distribution  of  the  solid 
materials  of  the  globe,  it  is  evidently  essential  to  know  in  what  manner  they  are 
arranged  below,  and  what  relations  they  bear  to  each  other  in  the  internal  parts  of 
the  elobe.  This  object  is  accomplished  by  means  of  geological  sections,  the  nature 
of  which  will  acquire  but  little  explanation.  A  geological  section  supposes,  that  on 
any  given  line  the  internal  structure  of  the  earth  is  laid  open  in  the  direction  of  a 
vertical  plane,  as  in  our  section  between  Barley  Moor  and  Grange  Mill  in  Derby- 
shire. It  therefore  merely  represents,  although  generally  on  a  much  more 
extended  plane,  the  same  thing  which  we  see  in  many  artificial  excavations,  and 
which  nature  herself  exhibits  to  our  view  in  cliffs  and  precipices.  Geological 
sections  are  indeed  merely  a  combination  of  sections  of  this  kind,  in  which  they 
bear  the  same  relation  as  the  map  of  a  large  country  would  do,  to  the  smaller 
plans  and  sketches  from  which  it  was  compiled,  especially  connected  with  the 
Mechanics  of  Fluids.  Such  a  map  is  that  of  Messrs.  J.  and  C.  WALKER. 

It  appears  that  the  present  annual  value  of  the  mineral  produce  of  Great 
Britain,  may  be  estimated  at  somewhere  about  20,000,000?.  sterling;  independent 
of  any  subsequent  process  of  manufacture,  and  not  including  the  cost  of  carriage 
on  coal. — Burr's  "  STUDY  OF  GEOLOGY,"  London,  1836. 


A    TABLE 


THE  SPECIFIC  GRAVITIES  OF  DIFFERENT  BODIES. 


In  consulting  this  Table  of  Specific  Gravities,  it  must  be  borne  in  mind  that 
water  is  taken  as  the  unit  of  measure  for  solids  and  liquids ;  and  atmospheric  air 
as  the  unit  of  measure  for  the  different  gases.  Water  at  the  common  temperature 
is  1,000,  and  mercury  13.568 ;  whence  we  conclude  that  mercury  is  13£  times 
heavier  than  water.  We  mean  that  a  cubic  foot  of  water  weighs  1000  ounces ; 
therefore  a  foot  of  mercury  weighs  1 3,568  ounces,  and  a  cubic  foot  of  bar  iron 
7788  ounces;  a  cubic  foot  of  vermilion  4230,  of  Portland  stone  2496,  of  indigo 
0,769,  and  of  cork  0,240  ounces. 


METALS. 

Antimony,  crude     '  ^  ..   .-     .,.  4064 

glass  of     .       ,-.:-.  4946 

molten      .        .         .  6702 

Arsenic,  glass  of,  natural    .        .  3594 

molten     vi>?;i;i,   •        •  5763 

native  orpiment    .         .  5452 

Bismuth,  molten         ...  9823 

native          .         .         .  9020 

ore  of,  in  plumes          .  437 1 

Brass,  cast,  not  hammered          .  8396 

ditto,  wire-drawn    .          .  8544 

cast,  common            .         .  7824 

Cobalt,  molten            .         .         .  7812 

blue,  glass  of      ,.,^  ,'    .  2441 

Copper,  not  hammered       .         .  7788 

the  same  wire-drawn    .  8878 

ore  of  soft  copper,  or 

natural  verdigris      &ggjy.  •  3572 
Gold,  pure,  of  24  carats,  melted, 

but  not  hammered       .         .  19258 

the  same  hammered  .        .  19362 

Parisian  Standard,  22  car. 

not  hammered                     .  17486 
VOL.  I. 


Gold,  hammered        .    ,,/••••'•;./•!•  17589 

guineaofGeo.il.     .        .  17150 

guinea  of  Geo.  III.    .        .17629 

Spanish  gold  coin      .         .  17655 

Holland  ducats          .         .  19352 

trinket  standard,  20  carats 

not  hammered         .        .  15709 

the  same  hammered  .        .  15775 

Iron,  cast       ;[,';•,!;:••,•         •     7207 

cast  at  Carron  .       „    ' ,  .    7248 

ditto  at  Rotherham    .        .7157 

bar,  either  hardened  or  not    7788 

Steel,  neither  tempered  nor  hard- 
ened       ....     7833 

hardened,butnot  tempered     7840 

tempered  and  hardened     .     7818 

. ditto,  not  hardened   .        .    7816 

Iron,  ore  prismatic     .         .         .     7355 

ditto  specular    .        .        .    5218 

ditto,  lenticular .        .        .5012 

Lead,  molten     ....  11352 

ore  of,  cubic      .        .        .    7587 

ditto  horned      .         .         .     6072 

ore  of  black  lead        v        .    6745 

2  H 


466        A  TABLE  OF  THE  SPECIFIC  GRAVITIES  OF  D1FFEUENT  BODIES. 


Lead,  ore  of  white  lead 

ditto  ditto  vitreous    . 

ditto  red  lead    . 

ditto  saturnite  . 

Manganese  striated   .      •» ,. 
Molybdena       •  •*'>  v  „.. 
Mercury,  solid  or  congealed 

fluent         .     .!• 

natural  calyx  of . 

precipitate,  per  se 

precipitate,  red  . 

brown  cinabar    . 

red  cinabar      -  •  , 

Nickel,  molten  .     •    .  -••-•*   ,•> 

ore  of,  called  Kupfer- 

nickel  of  Saxe 

Kupfer-nickel  of  Bohemia  6007 

Platina,  crude,  in  grains    . 
purified,  not  ham- 
mered 

ditto  hammered  . 

••  ditto  wire-drawn . 

ditto  rolled  . 

Silver,  virgin,   12  deniers,  fine, 

not  hammered 

ditto,  hammered    . 

Paris  standard 

shilling  of  Geo.  II. . 

shilling  of  Geo.  III. 

French  coin    . 

Tin,  pure  Cornish,  melted,  and 
not  hardened  . 

the  same  hardened     . 

of  Malacca,  not  hardened  . 

the  same  hardened    . 

ore  of,  red 

ore  of,  black       . 

-  ore  of,  white       .    • "  *.  '• ;: ' :  r i ' 
Tungsten        ;    .  •     ..  V    .  ij  •"",- 
Uranium  .... 
Wolfram  .... 
Zinc,  molten      .... 


PRECIOUS  STONES. 

Beryl,  or  aqua-marine  oriental  . 

ditto,  occidental 

Chrysolite  of  the  jewellers 


4059 

Chrysolite  of  Brazil    . 

2692 

6558 

Crystal,  pure  rock  of  Madagascar 

2653 

6027 

of  Brazil         .         ...'<*/ 

2653 

5925 

European 

2655 

4756 

rose-coloured         .        , 

2670 

4738 

yellow   .... 

2654 

15632 

violet,  or  amethyst     ,-,  * 

2654 

13568 

white  amethyst 

2651 

9230 

Carthaginian     -    .  .  •  v 

2657 

10871 

hlirlr 

Qflf>4 

1UO  1  I 

8399 

Diamond,  white  oriental    . 

£\JiJi± 

3521 

10218 

rose-coloured  oriental 

3531 

6902 

orange  ditto 

3550 

7807 

,  ,                  1-4.4. 

3524 

blue  ditto  .  •  *  v  **»*' 

3525 

6648 

Brazilian   . 

3444 

L  6607 

-^,.11  nrr. 

3519 

yeiiow        ... 

15602 

Emerald  of  Peru 

2775 

Garnet  of  Bohemia    .       >•        «r 

4189 

19500 

of  Syria 

4000 

20337 

dodecaedral  . 

4063 

21042 

volcanic,  24  faces  . 

2468 

22069 

Girasol       

4000 

Hyacinth,  common     .         .     "•'." 

3687 

10474 

Jargon  of  Ceylon 

4416 

10511 

Quartz,  crystallized    . 

2655 

10175 

in  the  mass    .  ^"v^W 

2647 

10000 

•  brown,  crystallized 

2647 

10534 

^Fftflfflfe 

2640 

10408 

milky    .... 

2652 

fat,  or  greasy 

2646 

7291 

Ruby,  oriental    .... 

4283 

7299 

fivtivhAH 

3760 

7296 

Ballas       .... 

3646 

7307 

Brazilian  .... 

3531 

6935 

Sapphire,  oriental 

3994 

6901 

sJi4-4-jt  v'liifn 

3991 

6008 

of  Puys 

4077 

6066 

Brazilian    . 

3131 

6440 

Spar,  white  sparkling 

2595 

7119 

red  ditto    .... 

2438 

7191 

green  ditto 

2704 

blue  sparkling  . 

2693 

green  and  white  ditto 

3105 

transparent  ditto 

2564 

adamantine 

3873 

3549 

Topaz,  oriental  .... 

4011 

2723 

pistachio  ditto  . 

4061 

2782 

Brazilian         l;-    . 

3536 

A  TABLE  OF  THE  SPECIFIC  GRAVITIES  OF  DIFFERENT  BODIES.       467 


Topaz  of  Saxe    .        .    x    . 
white  ditto 

3564 
3554 
4230 

2590 
2638 
2625 
2607 
2667 
2632 
2616 
2664 
2606 
2665 
2587 
2615 
2630 
2612 
2623 
2623 
2591 
2613 
2594 
2582 
2612 
2565 
2950 
2966 
2983 
2359 
2681 
2661 
2691 
2710 
2711 
2735 
2696 
2816 
2750 
2628 
2114 
2684 
2664 
2654 
2609 
2612 

Pebble,  stained  .... 
Prasium     ..... 

Sardonyx,  pure  .        .         . 

speckled     . 
veined        .        . 

blackish     . 
Schorl,  black  prism,  hexaedral   . 
octaedral 
tourmalin  of  Ceylon 

2587 
2581 
2603 
2606 
2622 
2595 
2595 
2628 
3364 
3226 
3054 
2923 
3156 
3286 
2416 
2111 
2143 
2484 

&c. 

2730 
2762 
2699 
2744 
2691 
2693 
2699 
2713 
2638 
2876 
1078 
926 
909 
2313 
2578 
3073 
2864 
1104 
2000 
2790 
2727 
2784 
2168 
2306 
2274 
2311 
2312 

SILICIOUS  STONES. 

Agate,  oriental  .  •>•(:&  .  •    ..  ..._.„ 

cloudy    .   •,    ;..    ifutt  •-.*>}, 
speckled 
veined    .        .   %>»;<.;•',:- 

Chalcedony,  common     »:.-.*  ,'>t;*'-> 

Stone,  paving     .        .        .  •>«  tfai 
cutlers'    .  :f?',;HfiJifJ-jr~—  *—  • 
grind       .        .        .   .  ~»~ 
_  -mm 

veined       .         .     H-JK' 
reddish     . 

rj            hliifch 

onyx      <^d»f<<K3vf  •»;?$ 
Cornelian,  pale  .     .•  •  *  ;j  .y  Tt'i  n  •  <  :  •  :  •  •  • 
speckled    .    jt;»*i*nt<>» 
-  veined       ,       ^  •.,.>;• 

stalactite  .   ft&dftt/l 

VARIOUS  STONES,  EARTHS, 

Alabaster,  oriental  white    .  id  •-*- 
do.  semi-transparent  . 

Flint,  white        .       ?  -..  •->...  '  awittr.  - 
black        .        ..'.-ray  Ira*;' 
veined       .  Ltf  ttvf'J  lu  jt  >A  < 
Egyptian  .  i  i^i'l  .huMlfo 
Jade,  white        .  *>O  Mfiiit^iiltfb 
green         .        .    U-*iteV. 
olive          .         .    -tifM  -,t$ 
Jasper,  clear  green     .        *  j  .*& 
brownish  green       .  ••<  .-us 

brown    .        .        .  V>  '*•> 

stained  brown    .  -V  v* 
veined        .   )  "V-^'w  •$•- 
—  of  Piedmont       .  -3  -.— 
of  Malta     .  o&9o8-tf- 

•  of  Valencia  *I*tfoIv~r- 
•  of  Malaga  .        .  •>!  -.*-- 
Amber,  yellow  transparent  '*.-.*• 
Ambergris          .         .         ,/T^  -.— 
Amianthus,  long         .  .  sstfiv  *  -.— 

Asbestos,  ripe     .... 
starry. 
Basaltes  from  Giants'  Causeway 
Bitumen  of  Judea 
Brick         ..       .,  'astral.. 
Chalk,  Spanish  .        .        .-•'.'••««» 

violet     .... 
cloudy    .... 
•  -  veined    .         .        ..'•-.*& 
onyx       .         .         .  ^hJ« 
red  and  yellow 

Opal 

British     .... 
Gypsum,  opaque         .         .   -     . 
—  semi-transparent 

Pearl,  virgin  oriental 
Pebble,  onyx      .... 

English           ..       '.:«*a!* 

rhomboidal 

468       A  TABLE  OF  THE  SPECIFIC  GRAVITIES  OF  DIFFERENT  BODIES. 


Gypsum,  cuneiform  crystallized 
Glass,  green        .         . 

2306 
2642 
2892 
2733 
3189 
3329 
2654 
2876 
2894 
3054 
4360 
5000 
2500 
2270 
3179 
3156 
3182 
2742 
2724 
2717 
2838 
2726 
2695 
2650 
2700 
2710 
2705 
2678 
2755 
2708 
2858 
2728 
2718 
2668 
2714 
2649 
2348 
1329 
1714 
2146 
2341 
2385 
2765 
2676 
2793 
2754 
2728 
4954 
3900 

Pyrites,  ferruginous,  round 
ditto,  of  St.  Domingo    . 
Serpentine,  opaque,  green  Italian 
ditto,  veined  black  and 
olive 

4101 
3440 
2430 

2594 
2627 
2586 
3000 
2669 
2672 
2854 
2186 
2766 
2324 
2478 
915 
2722 
2415 
2945 
2771 
2520 
2510 
2049 
2496 
2470 
1981 
2460 
2122 
2357 
2274 
2378 
2034 
2201 
2033 
1991 
2792 
2089 
2246 
2655 
2900 
2704 

1841 
2125 
1271 
1580 

bottle        .... 
.  Leith  crystal    .         .      --*- 

-  fluid 

Granite,  red  Egyptian        .  >  ~v 
Hone,  white  razor      .-  'A  I1.  •«  i     -  -r 
Lapis  nephriticus 
.—  Ti7ii1i 

ditto,  fibrous 
ditto,  from  Dauphiny 
Slate,  common  .... 

Judaicus  .... 
Manati     .         ."rr,»Vf-t--  ;.->  i- 
Limestone       •;*••-    *  •    ~W&fj:;'t 
white  fluor  .    VsuJu* 

Marble,  green,  Campanian  Ife 
red        .        .    >  .  .:'•'. 
white  Carrara       .        i* 

flesh  polished  .-   Iv^-.- 
Stalactite,  transparent 
opaque      .        .    -  •-;  — 
Stone,  pumice    .        fcOBtoy  -»rS 
prismatic  basaltes    .    -  —  .— 

~   tOUCh          .            .  i"--54     -«-;— 

Siberian  blue  .  £$»**'•  -«nH 
oriental  ditto   .     -sin  *  ,s-s*>- 

-    Bristol 

-  Portland               ff/Ji**  -  -»—» 

Castilian       .  i-ralfcv-  —  .-- 
Valencian     .  infctfa  —  *~ 
white  Grenadan    .     -r-dfr 
Siennien       .    i5I  io  —  *- 
Roman  violet    &  '!•.••  —  »  - 
African         .         .     —  *- 

rotten       .         .         .    '•''.,  i  a  _ 
hard  paving 
—  rockofChatillon      . 
clicard,  from  Brachet 
ditto,  from  Ouchain      v.r" 

Norwegian    .     !£!?<>  -nr- 

St.  Maur  .         .         .  9?&* 

St    Cloud            :         "5  VEV">     5* 

green  Egyptian     . 
Switzerland  . 
French  .... 
Obsidian  stone    .         ..       »       >• 
Peat,  hard         .         .   rvm^'  — 
Phosphorus        .      ...»   •     . 
Porcelaine,  Sevres      .    :*!s-i  'i-  »  ew 

Sulphur,  native  .... 
molten 
Talc,  of  Muscovy         .       :'.    rfi*- 
black  crayon 
ditto  German     .       **«lbi7:  *- 
yellow        .         •  •  -'-.  • 
black         .... 
white 

LIQUORS,  OILS,  &c. 

Acid,  sulphuric  .... 
ditto,  highly  concentrated 
nitric         .... 
ditto,  highly  concentrated  . 

Porphyry,  red    .... 
green 
.-  red,  from  Dauphiny  . 
.-  red,  from  Cordoue 
-  -  green,  from  ditto 
Pyrites,  coppery         .         - 
ferruginous  cubic  . 

A  TABLE  OF  THE  SPECIFIC  GRAVITIES  OF  DIFFERENT   BODIES.       469 


Ac-id,  muriatic    . 

.     1104 

Oil,  of  walnuts  .... 

0923 

red,  acetous       .      jih  •>;.'* 

.     1025 

of  whale    .... 

0923 

white  acetous     . 

.     1014 

of  hernpseed 

0926 

distilled  ditto     . 

.     1010 

of  poppies           . 

0924 

fluoric 

.     1500 

•  rapeseed 

0919 

acetic         .  . 

.    1063 

Spirit  of  wine.     See  Alcohol. 

0837 

phosphoric       .  .   '-.  »> 

.     1558 

Turpentine,  liquid       .         .    •  —  .- 

0991 

formic        .         .      i  .     ; 

.    0994 

Urine,  human     .• 

1011 

Alcohol,  commercial  . 

.    0837 

nooq 

Water,  rain        c 

Atr.-i.ivi^A 

1000 

T  Ai"lA 

mixed  with  water 

uo^y 

•  sea  (average)  .        .        ; 

lUUU 

1026 

15-16ths  alcohol 

.    0853 

of  Dead  sea     . 

1240 

14-1  6ths  ditto 

.    0867 

Wine,  Burgundy         .         .   fi:u;';j 

0992 

13-16ths  ditto 

.    0882 

Bourdeaux        .         .         . 

0994 

12-16ths  ditto 

.    0895 

Madeira   .... 

1038 

ll-16ths  ditto 

.    0908 

Port          .         .        .  -      \t 

0997 

10-lGths  ditto 

.    0920 

.T*A«ftAW** 

1033 

9-16ths  ditto 

.    0932 

8-16ths  ditto 

.     0943 

7-16ths  ditto 

.    0952 

RESINS,  GUMS,  AND  ANIMAL 

6-16ths  ditto 

.    0960 

SUBSTANCES,  &c. 

5-16ths  ditto 

.    0967 

4-16ths  ditto 

.    0973 

Aloes,  socotrine  .        .        ^-Hji'-I 

1380 

3-l6ths  ditto 

.    0979 

hepatic    .        .        . 

1359 

2-16ths  ditto 

0985 

Asafcetida  .        i. 

1328 

l-16th  ditto 

.    0992 

Bees-wax,  yellow 

0965 

Ammoniac,  liquid       .      <  o  >; 

.    0897 

white 

0969 

Beer,  pale           .         . 

.     1023 

Bone  of  an  ox    .         .        .        '. 

1656 

_  —  -     brown       •        •        • 

.     1034 

Butter        

0942 

Cider          .,,,..-. 

.    1018 

Calculus  humanus 

1700 

Ether,  sulphuric 

.    0739 

flQftQ 

ditto     .... 

l)<|44>.n 

1240 

T  Af\A 

muriatic  .       ,,«  ;r'5  «„  Li 

uyuy 
.    0730 

Camphor  ..... 

14d4 

0989 

acetic 

.     0866 

Copal,  opaque    .        .  .      .'.    »*i 

1149 

Milk,  woman's   .         . 

.     1020 

Madagascar     . 

1060 

cow's 

.     1032 

Chinese   .         .         .  •.;.... 

1063 

ass's         '  .r..  ,'    V  ,  (     •  ,, 

.     1036 

Crassamentum,  human  blood 

1126 

ewe's  .... 

.     1041 

Dragon's  blood  .... 

1205 

goat's        ..    ".".  '   .  ".  ' 

.     1035 

Elemi         

1018 

mare's       .         .  .       .!& 

.     1034 

Fat,  beef  

0923 

cow's  clarified  .        fc:< 

.     1019 

hog's  wj--. 

0937 

Oil,  essential,  of  turpentine 

.    0870 

mutton       .         .         .    .     .. 

0924 

psscTiti3/l  of*  IsvcndGr 

0894 

0934 

,  ,  (lit  to  of  cloves 

1036 

1212 

j    -  -  -  (Jit/to  of  cin.n£Linoii 

1044 

1222 

of  olives 

.     0915 

Gum,  ammoniac         .        .    ,  .  ^ 

1207 

of  sweet  almonds 

.    0917 

Arabic      .... 

1452 

of  filberts  .    '    ~.    V7^ 

.    0916 

Euphorbia 

1124 

linseed 

.     0940 

seraphic  .  - 

1201 

470       A  TABLE  OF  THE  SPECIFIC  GRAVITIES  OF   DIFFERENT  BODIES. 


Gum,  tragacanth 

.     1316 

Cedar,  Indian    .        .  .    ':&.'* 

.     1315 

bdellium  . 

.     1372 

American       i^^fAt^ 

.     0561 

Scammony  of  Smyrna 

.     1274 

Citron        .         .       ;.---•  - 

.     0726 

ditto  of  Aleppo      w0  ,  ,  j, 

.     1235 

Cocoa-wood 

.     1040 

Gunpowder,  shaken    .      fi&& 

.    0932 

Cherry-tree        .        .  .      .** 

.  .     0715 

in  i  lnn~p  TIPIH 

.     0836 

Cork        .  .  -      .  -      ««      ,  Srt 

.    0240 

nstlS'l 

.     1745 

Cypress,  Spanish    -    .  *.  -.1  ->**«r 

.     0644 

Honey  ,(,_,.,  r.  ,  nstiu 

.     1450 

Ebony,  American       .      i  H« 

.     1331 

Indigo        .        .        .        .  in 

.    0769 

Indian    .     ^:|i|;n.-«rs»r"«« 

.     1209 

Ivory    .      .,.      .        ...  !i::»> 

.     1826 

Elder-tree  .        .    .   •••     '.  ;  - 

.    0695 

Juice  of  liquorice    '  *.:"••  v;i.)  i 

.     1723 

Elm,  trunk  of    .  •  iJ  «w  hv.*» 

.    0671 

—  —  of  Acacia  .       -...•-  't  »i'f 

.     1515 

Filbert-tree  .t-«U  «Uti;  l-«.I. 

.     0600 

Labdanum      ,;+.  .     ..'^fcao*;1 

.     1186 

Fir,  male   .         .  >-.;'.M-t-». 

.     0550 

Lard       ,  .        .        .',  s  ;  h<>. 

0948 

female 

0498 

Mastic       .        .        .      i-iKj 

1074 

Hazel                 .            -'  :. 

0600 

Myrrh        .      .  .        ., 

.     1360 

Jasmine,  Spanish        .  '  -1  '.. 

.     0770 

Opium       .        .         .       -fftfii 

.     1336 

Juniper-tree       .         .    •#  '  . 

.    0556 

Scammony.     See  Gum. 

Lemon-tree        .  •  »«(i<JI^!  . 

.     0703 

Serum  of  human  blood 

.     1030 

Lignum-vitae      .         .    -^  . 

.     1333 

Spermaceti        .        .     )  <i  > 

.    0943 

Linden-tree        .        .  •  -  V  . 

.    0604 

Storax        .... 

.     1110 

Logwood.     See  Campechy. 

.     0913 

Tallow       .        .        .        * 

0942 

Mastick-tree      .        .   -^  . 

.    0849 

Terra  Japonica  .      ....  a«  »(??••. 

.     1398 

Mahogany      iMti&  «&£  i  -f-  . 

.     1063 

Tragacanth.     See  Gum.     . 

.     1316 

Maple        .  '-'.U  -')•:*«''  . 

.    0750 

Wax.     See  Bees-wax. 

.    0965 

Medlar      .  *«*$    if  .  T--v  . 

0944 

shoemakers'       .     >ria  '. 

.    0897 

Mulberry,  Spanish      .    -I  . 

.    0897 

Oak,  heart  of,  60  years  old 

.     1170 

Olive-tree       ...      .   . 

.    0927 

WOODS. 

Orange-tree        .  •      .  •      *; 

.    0705 

Pear-tree  ..<.*•      .,  .   .'.-. 

.    0661 

Alder       ....,;     ...   ^.,-    ^ 

.    0800 

Pomegranate-tree      .    -•»*';;•:• 

.     1354 

Apple-tree       ..,.,..     <-v^ 

.    0793 

Poplar       .  -:A    .  •      .        .a' 

.    0383 

Ash,  the  trunk  .        »     .  .  . 

.    0845 

white  Spanish        '.  -  H 

.     0529 

Bay-tree    .     ,„  ,  -..•*,  '<'*;•** 

.    0822 

Plum-tree      .    .-      .  v.  •## 

.    0785 

Beech        .        . 

.    0852 

Quince-tree        .        .  -   s'iiu 

.    0705 

Box,  French      .         .         . 

.     0912 

0482 

Dutch        .     ;f4:.-i-- 

••Htfj.  1328 

Vine           .... 

.    1327 

Brazilian  red      .     ^  V; 

**•#•  1031 

Walnut      .        »  ,      .  •      .  * 

.    0671 

Campechy-wood 

.    0913 

waiow     .      .      ...     ;•¥ 

.    0585 

Cedar,  wild        .  .      ..,,"     .  • 

:  •*.;•  0596 

Yew,  Dutch 

.     0788 

Palestine    ,  .  ..  .      .  . 

-  -,y;  0613 

Spanish 

.    0807 

471 


WEIGHT  AND  SPECIFIC  GRAVITY  OF  DIFFERENT 
GASES. 


Fahrenheit's  Thermom.  55°        Barometer  30  inches. 

Spec.  Gray.  Wt.  Cub.  Foot. 

Atmospheric  air  .        1.2          .      525.0  grs. 

Hydrogen  gas  .  0.1  43.75 

Oxygen  gas       .  ;        1.435       .      627.812 

Azotic  gas         .  .        1.182      .      517.125 

Nitrous  gas        .         v<fj      1.4544     .      636.333 

Ammoniac  gas.        ""*.-..       .7311     .  319.832 

Sulphureous  acid  gas    .        2.7611     .  1207.978 

In  this  table  the  weights  and  specific  gravities  of  the  principal  gases  are  given, 
as  they  correspond  to  a  state  of  the  barometer  and  thermometer  which  may  be 
chosen  for  a  medium.  The  specific  gravity  of  any  one  gas  to  that  of  another  will 
not  exactly  conform  to  the  same  ratio  under  different  degrees  of  heat  and  other 
pressures  of  the  atmosphere. 

And  if  common  air,  the  standard,  be  taken  at  unity  (1);  chlorine  oxy muriatic 
acid  will  be  2.500 ;  and  hydrogen  0.069 ;  whence  we  conclude  that  chlorine  is  2£ 
times  heavier  than  hydrogen,  and  this  last  is  14  times  lighter  than  common  air. 
For,  to  arrive  at  the  absolute  weight  of  the  gases,  we  have  only  to  assume  100  cubic 
inches  of  atmospheric  air  to  weigh  30.5  grains,  and  as  there  are  1728  cubical  inches 
in  a  cubic  foot,  the  simple  proportion 

100  :  30.5  grains  : :  1728  :  527.04  grains, 
the  weight  of  a  cubic  foot  of  common  air. 

And  for  any  other  gas,  it  is  only  necessary  to  observe  its  specific  gravity  in 
relation  to  that  of  common  ah-;  for  example,  chlorine  has  a  specific  gravity  of  2.5; 
hence  a  cubic  foot  of  chlorine  will  weigh  2£  times  as  much  as  a  cubic  foot  of 
common  air ;  for 

527 .04  x2£  =1317.6  grains, 
the  weight  of  a  cubic  foot  of  chlorine. 

To  determine  the  weight  of  any  gas  lighter  than  common  air,  we  also  compare 
their  specific  gravities :  thus,  the  specific  gravity  of  ammoniacal  gas  is  0.5,  and 
that  of  atmospheric  air  being  =1,  we  have  1  :  0.5  : :  1728  :  864.0,  or  simply 
1728-J-2=864  grains,  for  the  weight  of  a  cubic  foot  of  ammoniacal  gas;  and  so  on 
for  all  the  other  gaseous  bodies,  as  they  are  arranged  in  the  following  table. 


472 


WEIGHT  AND  SPECIFIC  GRAVITY  OF  DIFFERENT  GASES. 


Tf  atmospheric  air  be  taken  at  unity  (1),  then  the  various  gases  will  stand  as 
under : — 


Atmospheric  air         .  . 
Ammoniacal  gas 
Carbonic  acid    . 
Carbonic  oxide  . 
Carburetted  hydrogen 
Chlorine    . 
Chlorecarbonous  acid 
Chloroprussic  acid    ,. 
Cyanogon 
Euchlorine 
Fluoboric  acid  . 
Fluosilicie  acid  . 
Hydriodie  acid . 


1.000 
0.500 
1.527 
0.972 
0.972 
2.500 
3.472 
2.152 
1.805 
2.440 
2.371 
3.632 
4.346 


Hydrogen          ....  0-69 

Muriatic  acid    ....  1.284 

Nitric  oxide       ....  1.041 

Nitrogen 0.972 

Nitrous  acid      ....  2.638 

Nitrous  oxide    ....  1 .527 

Oxygen 1.111 

Phosphuretted  hydrogen  .        .  0.902 

Prussic  acid      ....  0.937 

Subcarburetted  hydrogen  .         .  0.555 

Subphosphuretted  ditto     .        .  0.972 
Sulphuretted  ditto     .        .        .1.180 

Sulphureous  acid       ...        .  2.222 


CONCLUSION. 

THE  reader  will  have  seen  in  this  volume  how  the  road  to  abstract 
science  may  be  smoothed  ;  but  he  may  rest  assured  that  any  popular 
version  of  Hydrostatics  is  quite  illusory,  for  no  portion  of  sound  know- 
ledge was  ever  acquired  without  some  corresponding  exertion  of  mind. 
It  is  one  of  the  improvements  to  be  made  in  our  systems  of  education 
for  the  various  professions,  and  in  books  written  to  retrieve  the  de- 
clining taste  for  science,  that  students  in  Mechanics  should  devote 
themselves  methodically  to  the  profitable  but  toilsome  drudgery  of 
computation ;  and,  in  their  geometrical  constructions,  be  as  clever 
with  their  hands  as  ingenious  with  their  heads.  Science  and  know- 
ledge are  subject,  in  their  extension  and  increase,  to  this  law  of 
progression  :  the  further  we  advance,  instead  of  anticipating  the  ex- 
haustion of  their  treasures,  the  larger  the  field  becomes — the  greater 
power  it  bestows  upon  its  cultivators  to  add  new  measures  to  its 
rapidly-expanding  dominions.  It  is  the  science  of  calculation  which 
has  grasped  the  mighty  masses  of  the  universe,  and  reduced  their 
wanderings  to  fixed  laws  ;  which  prepares  its  fetters  to  chain  the 
flood,  to  bind  the  ethereal  fluid  ;  and  which  must  ultimately  govern 
the  whole  application  of  Hydrostatics  to  the  Arts  of  Life. 


London  :  J.  Rider,  Printer,  14,  Bartholomew  Close. 


0^  CALIFORNIA 


NGV    5   1947 


One  dott«on-Sev 


REC'D  LD 


JUL  Ibl 


e«h  day  overdue. 


RECEIVED 

NOV27'67-1QP 


ewe 


I 

28Ju1'59AJ 


-10Om.l2,'46(A2012sl6)4120 


UNIVERSITY  OF  CALIFORNIA  LIBRARY